You are on page 1of 565

Anurag Mishra Mechanics 1 with www.puucho.

com

www.puucho.com
Anurag Mishra Mechanics 1 with www.puucho.com

·' !II Published by: 't§l


SHRI BALAJI PUBLICATIONS
(EDUCATIONAL PUBLISHERS & DISTRIBUTORS)

6, Gulshan Vihar, Gali No. 1,


Opp. Mahalaxmi Enclave,
Jansath Road, Muzaffarnagar (U.P.)
Phone: 0131-2660440 (0), 2600503 (R)
website : www.shribalajibooks.com
email : sbjpub@gmail.com

!II First edition 2009


!II Fourth edition 2012
!II Fifth edition 2013
!II Reprint 2014

!II © All rights reserved with author

!II Price : { 428.00

!II Typeset by :
Sun Creation
Muzaffarnagar

!II _printed at :
Dayal Offset Printers
Meerut (U.P.)

!II All the rights reserved .. No part of this publication


may be reproduced, stored in a retrieval system
or transmitted, in any form or by any means,
electronic, mechanical, photocopying, recording
or otherwise, without the prior permission of the
author and publisher. Any violation/breach shall
be taken into legal action.

www.puucho.com
Anurag Mishra Mechanics 1 with www.puucho.com

, · Preface
I have been involved in teaching Physics for last 16 years. This book is an opportunity to
present my experiences. During my interaction with UT-JEE aspirants. I realised that most
feared topic is mechanics. Some of the reasons put forward by students behind this thought
were:
@ No spontaneous thoughts.appear after reading a problem. Mind goes blank. Can not
. proceed in a problem.
@ How to ~roceed in a problem? Which law is applicable; that is a given problem will
involve conservation of energy or momentum or both.
@ If some one says solve the problem in non-inertial reference frame, horrible thoughts
appear inmind.
@ Total confusion about CM frame.
@ Proper understanding of constraints.
@ S)lort cut approach in relative motion.
@ No single book available that gives large no. of solved examples with elaboration of
concepts in asolµtion.
This book will help the students in building analytical and quantitative skills,
addressing keyl,Jlisconceptions and developing.confidence in problem solving.
I sincerely wish that this book will fulfill all the aspirations of the readers. Although
utmost
. full care has been taken to make the book free from error but some errors.
- .

ina_dvertently may creep-in. Author and Publisher shall be highly obliged if suggestions.
regarding improvement and errors are pointed out by readers. I am indebted Neeraj Ji for
providing me an opportunity to write a book of this magnitude.
I am indebted to my father Sh. Bhavesh Mishra; my mother Smt. Priyamvada Mishra, my
wife Manjari, my sister Parul, my little kids Vrishank and Ira for giving their valuable time
which I utilized during the writing of this book and people of Morada .
bad, who supported.
·me throughout my career.
In the last, !also pay my sincere thanks to all the esteemed members ofM/s. ShriBalaji
Publications in bringing out this book in"the present form.

Anurag Mishra.

www.puucho.com
Anurag Mishra Mechanics 1 with www.puucho.com
r

Our Other Publications


for

(Engineering Entrance Examination€)

Er. Anurag Mishra Er. Anurag Mishra • Electrostatics

Mechanics • Simple Harmonic Motion


Electricity& , •, Electric Current

!•JEE • Wave Motion


Magnetism j• yapacitors
I
• Solid and Fluids forJEE l•
'
The Magnetic Field
• Gravitation 1• Electromagnetic Induction
Vol. II
/ and A.G. Circuits
'I-----~

)feat&
Er. Anurag Mishra
I' .
Temperature, Heat &
Thermodynamics
1 •

tl)e. equation of State. Optics . Geometrical Optics'

/•JEE Heat Transfer


• Thermodynamics
. foJ EE l" Wave Opti~

.I ,
' . : : ·,-:' \ ~

www.puucho.com
Anurag Mishra Mechanics 1 with www.puucho.com

How to face the challenge ?


Following are some doubts which arise in the mind of almost all the students
but may face them by taking some care.

1. I can not solve numerical because my concepts are not clear. In fact numerical
solvingitselfis an exercise t~ learn concepts.

·z. 1 /can
'
not study because I am in depression, I fell into it because I was not
studying! Depression is escape mechanism of people afraid of facing failures.
Failure is integral partoflearning.

3. I understand everything in class but can not solve on my own.WRITING work is


· vital. It is a multiple activity, initially idea comes in mind then we put into
language to express it, we are focussed in hand eye coordination, eyes create
visual impression on brain which isrecorded there. WRITING WORKS ARE
EMBOSSED ON BRAIN LIKE CARVINGS OF AJANTACAVES.

4. In exams my brain goes blank, but I can crack them at home. Home attempt is
your second attempt! you are contemplating about it while home back. You
do not behave differently in exam you replicate your instincts. Once a fast
bowler was bowling no balls. His coach placed a-stump on crease, in fear of
injury he got it right. CONCEPTUALIZATION, WRITING EQUATION, SOLVING,
THEN PROBLEM GETS TO CONCLUSION!

5. I am an average student. It is a rationalization used by people afraid of hard


work. In their reference frame Newton's first law applies "if I have a
misconception I will continue with it unless pushed by an external agent
even I will surround him in my web of misconception yielding zero
resultant:' AVERAGE IS NOT DUE TO CAPACITY LACUANEBUT DUE TO LACK
OF DETERMINATION TO SHED INERTNESS.

6. A famous cliche "/ do not have luck in my favour' PRINCIPLE OF CAUSALITY:


CAUSE OF AN EVENT OCCURS IN TIME BEFORE OCCURRENCE OF THAT
EVENT i.e., cause occurs first then event occurs. SHINING OF LUCK IS NOT AN
iNSTANTANEOUS EVENT IT IS PRECEDED BY RELENTLESS HARD WORK.
Sow a seed ofaspiration in mind, water it with passion, dedication it will bear
fruit, luckcan give you sweeter fruit.

www.puucho.com
Anurag Mishra Mechanics 1 with www.puucho.com

t. Do not take study as a burden actually )ts a skill like singing and dancing. It
has to be honed by proper devotion and dedication. ·

2. Withou't strong sen_se of achievement you can't excel. Before entering the
cmppeti.tive field strong counselling by parents is must. Majority do not
· know what for they are here. No strategic planning, they behave like a tail
: ende_r batting in frontofSteyn's bouncers.

3. - 'science is not a subje,ct based on well laid dowh procedures or· based on
learning sonie facts, it ipvolves very intuitive and exploratory approach.
Unless their is desire and passion to learn you can not discover new ideas. It
requires p'atience and hard work, whose fruits may be tangible later on.

4. Some students realize very late that they are studying for acquiring skills and
, .honingthem. Their is a feeling that they can ride at the back of instructor and
,
· , achieve ~xcellence. Study comes as tqrtu·rous exercise enforced on them and
'
their is some mechanism that can take this burden-of them. -

5. Science is not about gaining good marks, up toXth by reading key points good
marks are achieved but beyond that only those survive who have genuine
interest in learning and exp to ring. Selfstudy habit is must.

6. · IF YOU WANT TO GAIN LEAD START EARLY. Majority of successful students


try to finish .major portion-elementary part of syllabus before they enter
Coaching Institute. Due to this their maturity level as comparecl to others is
' :i. · more tliey get ample time to adj4st with th~ fast pace. They are less·
traumatized by the scientific matter handed over. For those who enter fresh
must be counselled to not get bullied by ·early starters but work harder
initially within first two months initial edge is neutralized.·

7. Once a sl:l,ldent lags behind due to scime forced or unforced errors his mind
begins to play rationalization remarks like I am an average student, my mind
is not sharp enough, I have low IQ etc: These words are mechanisms _used to
· a\Toid hard work. These words a,re relative terms a person who has .early start
may be intelligentr~lative t9y6u.' . . .
0

lntelligence means _cu~ulative i-~sult of h_ard _work of previous years, that


hard work has eventuallY. led to a developm~nt of instinct tci crack 'things -
easily. _., ·
/

www.puucho.com
Anurag Mishra Mechanics 1 with www.puucho.com

I '

CONTENTS
(j UNIT AND DIMENSIONS
(j I. DESCRIPTION OF MOTION
Subject of kinematics (19), Vector notation (20), Displacement I-
--
vector (20),Parallelogram law of vector addition (21).,
Component ofvector(23), Unitvector(24), Expressing a vector
in unit vector notati_on (25), Position vector (27), Rectangular
resolution of a vector in three dimensions (28), Vector
multiplication (31), The scalar product of two vectors (32),
The vector product of two vectors (35), Rectilinear motion of
a particle (37), Calculus supplementary (41), Rectilinear
motion (44), Instantaneous velocity ( 45), Integration ( 48),
Interpretation of graphs (53), Average velocity & Average
speed (58), Two-dimensional motion with constant acceleration (69), Projection on an inclined plane
(82), Relative motion (87), Application of advanced concept of relative motion (88), Equation of
motion for relative motion (98), Projection of a particle in an accelerated elevator (101), Projection of
a ball in horizontally moving trolley (101 ), Closest distance of approach between two moving bodies
(102), Problems: Level-1 (106), Level-2(113), Level-3 (117),Answers(123), Solutions (124).

Q 2. FORCE ANALYSIS
The concept of force (138), Reference frame (139), Ideal string ~~~~~p=--=~-=::;~
(143), ideal pulley (143), Contact force (143), Concept of
external and internal force (144), Pulley system (145), Tension
in a hanging rope (145), Constrained motion (148), Pulley
constraint (154), Normal constraint (155), Elastic force of
spring (158), Parallel combination (159), Friction (165), The
laws of sliding friction (165), Direction of kinetic Friction
(171), FBD when arm is in_ deceleration (185), Circular
motion (186), Angular velodty vector (187), Concept of ,
pseudo force (192), Non-inertial reference frame (193),
Whirling rope (195), Lift Force on an airplane (196), Non-uniform circular motion on horizontal plane
{196), Problems: Level-1 (210), Level-2 (221),Level-3 (230),Answers (238), Solutions (240). I
I 1
\_ ______________ ---------------- www.puucho.com --------··~
Anurag Mishra Mechanics 1 with www.puucho.com

--·

-CJ 3. WORK AND ENERGY


Work done (265), Unit of work (266), Conservative and non-conservative
force (268), Concept of potential energy (269), Classical work energy theorem
(270), Conservation of mechanical energy (271), Work done by friction
(271), Work done by spring force (271), Work depends on the frame of
reference (272), Work due to internal force (friction) (272); Work energy
theorem in a non-inertial reference frame (273), How to apply
coriservation of energy_ equation (273), Vertical circ_ular motion (283),
Power (293), lntemai energy so!-lrces & work (296), Problems Level-1
(299), Level-2 (306), Level-3 (310),Answers (316), Solutions (318).

CJ 4. IMPULSE AND MOMENTUM


Impulse (328), Conservation of momentum (328), Conservation
momentum for a two particle system (329), Relative velocity and the
conservation of momentum (330), Recoil disintegration, explosions (335),
Impulsive force (336), Centripetal acceleration revisite~ (338), Centre of
mass (340), Position of COM of two particles (340), Centre of gravity (341 ),
Motion of the centre of mass (34,1), Kinetic energy of a system of particles
(342), Most important concept (343), Finding the centre of mass by
integration (353), Collisions (361), Models for elastic & inelastic
collisions (362), Oblique impact (365), The velocity of the centre of
mass for collisions (370), Elastic collisions in the CM reference frame (371), Inelastic collisions in CM
reference frame (372), System of variable mass; Rocket propulsion (380), Problems Level-1 (383),
Level-2 (395), Level-3 (399), Answers (406), Solutions (408). ·

(J 5. RIGID BODY MOTION


What is rigid body(421), General rigid body motion (421 ), Rotation about centre
of mass (422), Kinematics of fixed axis rotation (422), Vector representation of
rotational quantities (425), .Torque (427), Newton's second law for rotation
(429), Rotational kinetic energy and moment of inertia (430), Rotational
kinetic energy of a collection of particles (431), Perpendicular axis theorem
(437), Dynamics of a rigid body(441), Angular momentum (447), The ladder
(450), Work done due to torque (457), Angular ,, momentum of a projectile
(462), Angular momentum of an i[]verted conical pendulum (462), Angular '=--~='-~=-~=~--
impulse0angular momentum theorem (464), Two bodies rotatory system
(466), Kinematics ofrigidbody rotation (475), Total kinetic energy of body (484), Dynamics of rigid body in
plane motion (486), Torque on the rotating skew rod (506), Problem: Level-1 (511); Level-2 (519), Level,3
(525), Answers (531), Solutions (533), Exercisg_,_advanced problems (544), Comprehension based
pro blems (553), Assertion and reason type problems (562). I

,.I
www.puucho.com
________________ .,..,.,,-/
Anurag Mishra Mechanics 1 with www.puucho.com

UNIT AND DIMENSIONS .

Physics is that branch of science in which we observe, (a) Magnitude of Physical Quantity=
measure and describe natural phenomena related to matter Numerical value x Unit
and energy. Like all the science, physics is ultimately based th us for a given physical quantity when the unit will
on observation. To ~ssemble the relevant observations into a change, numerical value will also change, e.g. density of
coherent picture by, constructing a logical framework is water =lg- cc- 1 = 10 3 kg- m-3 and not lkg- m-3 .
called theory. Theory enables the physicist to account for
past observations and to decide how new ones should be (b) Larger the unit smaller will be the magnitude and
made. Nearly all physical observation are quantitative; they vice-versa, e.g., 1 kg= 1000 gm then as 1000 is greater than
require measurement. 1, gm is smaller unit than kg of mass.
Every measurement is a comparison of a Criteria for Standards
quantity with a standard quantity that is, an The choice of the standard is arbitrary. However, several
agreed upon quantity of the same kind. To measure a criteria must be met if a standard is to be as useful as
length for example, you adopt as your standard a convenient possible.
measuring rod, whose length you use as the unit of length. 1. Stability : The standard should not vary with time.
You count the number of times that the rod fits into the If this criterion is satisfied, measurements made at different
length to be measured. This number given the length in time, using the same standard, can be meaningfully
terms of the chosen unit. compared.
Physical Quantities 2. Reproducibility: The standard should be accurately
The quantities by means of which describe the laws of reproducible so that copies, ideally identical with the
physics are called Physical Quantities. A physical quantity is standard itself can be used elsewhere. If this criterion is
complete specified if it has - satisfied, measurements made at different places can be
~---e>(A) Numerical value only ratio e.g., compared.
refractive index, dielectric constant, etc.
or 3. Acceptability : The standard should be universally
accepted so as to eliminate clumsy and possibly inaccurate
Magnitude only Scalar e.g., comparisons among measurement made with separate
mass, charge,current etc.
or standard.
4. Accessibility : As nearly as possible, the standard
Magnitude and direction Vector e.g., should be readily accessible to everyone ·vho needs to use it.
displacement, torque, etc. 5. Precision : It should be possible to measure the
In expressing the magnitude of a physical quantity we standard itself with a precision at least as great as the
choose a unit and then find physical quantity how many times precision with which any comparable measurement can be
that unit is contained in the given physical quantity, i.e. made.

www.puucho.com
Anurag Mishra Mechanics 1 with www.puucho.com

6. Security : The standard should be as safe as D. International system of units [SI units] , ,
possible from and preferably' immune from all possible· In 1971 the International Bureau of weight and
causes of damage. · . . · measures held its meeting and deci,ded a system of units
When a' standard meets these criteria as nearly as. which is known.as the international system of units._.If is
possible, it can. be·taken as the-primary standard. The 1 abbreviated ·as SI units from .the French name Le 'system·
primary standard Ca.I\ 'the!J be,yse_d to produce secondary· International d Units and is the extended MKS system
standards'calibrateo in terms of the primary, and so on. applied to the whole of physics. Now-a-days, most _of the
· A set of base units, toget:li,(!r with the ~es required to engineers and physicists use this system of units. Table 2.
express·all other units fn tenns·of.them, constitutes a system gives the fundamental quantities and their SI units.
of units. The systein,in general use throughout the world is Besid~s ·-the above seven fundamental units, two
called _the Interil.ati(!nal System, of Units. The, short supplementary units are also defined viz, radian (rad) for
form SI (from the French System International) is used in plane angle and steradian (sr) for solid angle. '
all languages. ' The SI Unit of Angle ('l"he Radian)
Systems of Units'. ·. ,. A circular arc is divided into 360 degrees. The degrees
A complete ·set ~f tmits; both.fundamental anc\ derived are. subdivided into 60 minutes of arc, ·each of· which
for all kinds of phys.ical quantities, is called ·a system of contains 60 secon4s. Angle is measured as' the ratio of arc
units. There are· several systems of units· which liave been· length and the radius of the scale (Fig. 1). The measured
employed for describing measurements. A few common angle does not depends on the radiu~ of the scale, sinqi'the
systems are give'n ):,elo.;,. : ' ' .. length of the arc is proportional to its radius, In figure the
A. CGS system . _ , shaded portion of the cir~le is a fracti~n f of the whole cirde,
The systeni is' also <;ailed· Gaussian system' of.units. In it where '
length, mass and time have lieen taken as, the fundamental

ItI
quantities, and corresponding _fundamental units are ·
centimeter{cm), gram (g) and second (s) respectively. The
unit of force in _this system is dyne while that of work- or
energy is erg. ,

.· ~'·,:-~
B. FPS system · I
It uses foot, poupd ;md second for the length, n:iass, and
time measurements respectively.' In this system force is a 1- . ---- ---- ,' ·,
witJ:i

L
derived quantity ·unit poundal. · ·'
c. MKS system ' ' .
- - - - - - -
., ', ., ,
. L )
' '
.j-
j ,.'

In this system the lf?ngth, mass .and time have ]:,een --·_"_··f_lg_:~_..,,___ ,; ' :,, .
taken as the fundamental.quantities, and the corresponding
fundamental miitsaietiie-nietre, kilogtani and' second.,'Fhe f = angl!!(degrees) ~ aidength = arc length
units ofl all ~ther mechanica\ 'quantities like force, work 360° circumference 21t x radius
power, etc., are derived in terms of t:Jiese fundamental uni~.· ·) ' 360° iirclength ' · . . '
so ang1e Cd egrees = ---''- ·
For example, ·the unit of force is" thitt force which will 21t ; radius ·
produce and acceleration of .1 m/ s 2 in a body of mass 1 kg . The number (360°/21t) has no physical significance. We
. ' '
and is called newton. Tlie unit of work or energy is joule, may define a unit of angle by cutting ·a circle into any
while, of power is watt: · number of pieces, we obtain a· particularly conveniel)t unit,
Table:1 Units of Some Physical, the SI unit of angle or radius (abbreviated rad), by choosing
Quantities in Different Systems · 21t pieces.
An angle ofl radian corresponi:ls to an arc length equal ·
to the radius of the circle. ·
. arclength S
Angle(radians) = - - " -
. radius R

Force · ; / Y '' dyn~ _':' >~~o.i4N ;. _ poiipd;,y ,;"\


Workoren~;gy .::> .
e~·,._\ :".J&tUe/~.J. ft:-powidal
1
••

[PO~er t. ~,.~ •1 X .,.~rgts'~' . ·;_..·.w~~-~·~.: ~ ft~P~~~dal/s-"'~


. ';~ . www.puucho.com
Anurag Mishra Mechanics 1 with www.puucho.com

- -
' UNiT AUD DIMcllSIONS
5. Fullstops are not written after the abbreviations and
Concept : How to express an angle of 1° in tenns of
units, e.g. 1 litre = 1000 cc (and not c.c.) emf, amu, etc.
radians.
Table-2
The arc lertgth c~rresponding to 1' is 1/360 of the
circumference. : \ Fundamental \ Name of unit \symboi \Represen, I
Thus S = (2rrr)
360
I quantity
1. Length metre m
.tatlon ,
L
,·-
1° (measured in radians) 2. Mass kilogram kg M
= ~ = ~ = 1.745 x10-2 rad 3. Time second s T
R 108°
The degree is a small fraction of a radian_ For rough 4. Electric current ampere A A

calculation, with rt z 3 , we mqy use 1° z _!_ rad. 5. Temperature kelvin K e (or) K


60
6. Luminous intensity candela cd cd
Fundamental and Derived Units
7. Amount of substance mole mo! Mo!
Normally each physical quantity requires a unit or
standard for its specification, so its appears that there must
be as many units as there are physical quantities. However, it STANDARDS OF LENGTH, MASS AND TIME
is not so, It has been found that if in mechanics we choose The Unit of Length
arbitrary unit of ariy three physical quantities, we can Length is the measure of intervals in space. The SJ unit
express the units of all other physical quantities in of length is the meter (s-ymbols m). The name is derived
mechanics in terms of these. Arbitrarily the physical from the Greek word metron meaning 'measure'. The meter
quantities mass, length and time are chosen for this purpose. is now defined to be the distance the light travels, through
So any unit of mass, length and time in mechanics is called a . 1.
fundamental, absolute or base unit. Other units, which can vacuum, 1n - - - - - s .
299,792,458
be expressed in terms of fundamental units, are called
derived units. For example, light-year or km, kg/m are Three imponant considerations underlie this definition.
derived units as these are derived from units of time, mass First, the speed of light is now defined to be precisely
and length respectively. 299 792 458 meter per second. Should more precise
measurements be made of the speed of light, the effect
Note:
would be to change the length of the meter slightly. Second,
(i) Only four additional fundamental quantities temperature
length and time can now be measured with comparable
current, luminous intensity and amount of substance are
needed to deal all other branches of physics. precision. Third, and most important of all, the
(ii) Apart from fundamental and derived units we also speed of light in vacuum is precisely the same for
sometimes come across practical units. all observers. This is a fundamental in this theory,
(a) These may be of fundamental or derived quantities e.g., and so strong is the confidence placed in it by
light-year is a practical (fundamental) unit of distance while scientists, that the definition of the meter can be
horse-power is a practical (derived) unit of power. soundly based on the constancy and universality
(b)These may or may not belong to a system but can be of the speed of light in vacuum.
expressed in any system of units, e.g. We also have some other practical units which are
1 mile= 1.6 km= 1.6 x 103 m =1.6x10 5 cm. frequentiy used for small and large lengths. They are :
(a) 1 fermi = 1 fm = 10-13 m
1. Even if a unit is named after a person the unit is not (b) 1 X-ray unit = lXU = 10-13 m
written with capital initial letter. Thus we write newton (not
Newton) for unit of force. (c) 1 angstrom = 1A ~ 10-10 m
2. For a unit named after a person the symbol is a capital (d) 1 micron = 1 µm = 10-6m
letter. Symbols of other units are not written in capital (e) 1 astronomical unit = 1 AU= 1.49 x 1011 m
letters. For example, N for Newton (and not n) while m for [Average distance between sun and eanh i.e., radius of
metre (not M). earth's orbit]
3. The symbols or units are not expressed in plural form. (t) 1 light-year = 1 ly = 9.46 x 101s m
Thus we write 50 m or 7 erg and not 50 ms or 7 ergs.
[Distance that light travel in 1 year in vacuum]
4. Not more than one solidus is unused. For example, 1
poise should be written as 1 poise = 1 g/s cm or lg s- 1 cm- 1
and not 1 g/s/cm. www.puucho.com
Anurag Mishra Mechanics 1 with www.puucho.com

14 - _ --- __-- __ . _______ MECHANics.il


' (g) 1 pars~c =~pc = 3.08 x 10 m = 3.26 light-year
6
The numerical value in this definition of the
[The distance at which a star subtends an second was so chosen as to make the new
angle of parallax of 1 sec at an arc of 1 AU] standard compatible with the old one. The new
standard is, however, about 1000 times as precise as the old
The Unit of Mass·
one. Just as fluctuations in a human pulse rate can be
Mass is a basic property of matter. The standard kg is measured by comparing the pulse with the swing of a
the mass of a platinum - iridium cylinder stored in a special pendulum, fluctuations in the earth's rotation rate (which
vault in the International Bureau of standards in severs, determines the length of the day) can be measured in terms
France, th~ accuracy.of this standard is 1 part in 10 8 parts. of the period of the microwaves produced in an atomic
To measure mass of atoms or molecules we use the unit clock.
'atomic mass unit' abbreviated as amu (or now u). At As a result of the redefinition, the day is no longer
present atomic mass unit is defined as (1/12) th the mass of exactly 86400s long. This is awkward for astronomers and
an· atom of carbon-12 isotope. Now as the mass of a others who continue to use the 'mean solar second' , defined
carbon-12 atom is , 1
12 12 as - -- day. To keep the two systems compatible, an extra
-----=
Avogadro's no. 6.02 x 10 23
g 86400
'leap second' is added to the mean solar day every few years
so 1 amu (or u) as needed, by international agreement.
12 Atomic standards have advantages other than
= 2_ X ( , ) = 1.67 X 10-24 g = 1.67 X 10-27 kg precision over arbitrarily constructed standards.
23
12 6.02 X 10
Because all atoms of a given kind are indentical,
The Unit of Time there is no need to construct and maintain a
We measure a time interval by comparing it with a unit standard in a central laboratory. We need not
of the same kind a unit of time. The unit of time must be worry about the possible destruction of the
defined in terms of some physical system that behaves in a standard and we need not transport secondary
repetitive way. We can use the time interval between standards to it for checking.
repetitions, called the period, to define the time unit. Every good physical measurement shares a number of
When it had become possible to measure and common features. Suppose, for example, that you mr·asure
periods of atomic phenomena far more precisely a sailboat's length as 10 meter infact it is a compari: n of
than the periods of larger systems, and to use the boat's length with that of another object, the metet :ick.
such phenomena in establishing standards. A Every physical measurement is a comparison
standard based on a atomic phenomenon is called of two similar physical quantities.
an ato~c standard. The atomic standard of time Second, we accept the meter stick as a valid device for
relies on the fact that an atom emits a specific measuring the boat's length. At the factory, the meter stick
kind of electromagnetic 'light' wave when the was marked by a machine, itself adjusted by comparison
arrangements of its electrons undergo a specific ' with a standard length.
cha,:ig~ ·called an atomic transition. Like all repetitive To be valid, a measuring device must be
waves, electromagnetic waves are periodic. Since 1967, the compared against a widely accepted standard.
second has been defined in terms of a particular atomic Also, the procedure must be stable ·so that we know how
transition, in which an outer electron of a cesium - 133 atom to compare different measurements made at different times.
'flips' its orientation relative to the atom' nucleus. This flip Accuracy describes how much a measurement might
causes the atom to emit a wave that has a very sharply differ from another measurement made with greater care.
defined period. The device used to measure the period is For good measurement requirement is adequate precision.
called an atomic clock. The clock contains electronic The precision of a measurement is the
components that both stimulate and detect the repeated smallest amount of the measured quantity that
can reliably be distinguished.
flipping in the many cesium atom within the clock. The
second is defined as the time required for 9, 192, 631, 770 Greater precision requires a more carefully
manufactured device.
periods of the microwaves that stimulate these transitions.
Like all SI units, the name second has an international The process of comparing a particular
measuring instrument against a standard is called
standard symbol, which is s.
_calibration.

www.puucho.com
Anurag Mishra Mechanics 1 with www.puucho.com

IUNIT AND DIMENSIONS


· Table 3, Some SI Prefixes and . Velocity [i}T,_1 ]
4. Acceleratmn = - - ~ = =-~~ [M 0 L1 r 2
]
Multiplication Factors Time .. [T1 ]

I F~ai:- \ Prefix, '\Symbol \ ~r~c-, \ Prefix \ Symhe1


<t1on. _ . ,, lion ·
I
1
Thus dimensions of unit of acceleration are O in mass,
1 in length and -2 in time. ·
I
5. Force = Mass x Acceleration
10·' deci d 10' deca da ,,
I = [M1 ][L1r 2 J = [M1 L1T-2 ]
I 10-2 .centi C 102 hecto h I Thus dimensions of unit of force are 1 in mass, 1 in
'
I 10-J milli m 103 kilo k I length and -2 in time.
6. Work (energy ) = Force x Distance
I 10"'" micro µ 10• mega M II
= [M1 L1r 2 ] [L1 ] = [M1 L2 r 2 ]
I 10-9 109
'
nano n giga G l Thus dimensions of unit of work or energy are 1 in
I 10012 pico p 10'' tera T I mass, 2 in length and -2 in time.
I 10-JS femto f 10'5 peta p '
I 7. Power = Work= [M'L'T-'] = [M1 L2 r
Time IT']
3]

! 10-18 atto a 1018 exa E '''


Thus dimensions of unit of power are 1 in mass, 2 in
length and -3 in time.
Dimensions of a Physical Quantity
8. Momentum = Mass x Velocity = [M 1L1 r 1 ]
If any derived unit depends upon the r th power of the
fundamental unit, it is said to be of r dimensions in that Thus dimensions of unit of momentum are 1 in mass, ·1
fundamental unit. in length and -1 in time. ,
The unit oflength is represented by [L], the unit of mass 9. Impulse= Force x Time,= [M1 L1 T-2 ]IT1 ]
by [Ml, unit of time by [Tl, the unit of current by [A], the = [M1 L1 r 1 J
unit of temperature by [Kl and that of intensity of Thus dimensions of unit of impulse are 1 in mass, 1 in
illumination by [CJ. In mechanics the various quantities length and -1 in time.
depend only on the units of length, mass and time. As an 1 1 2
example -
10. Pressure, P =-- [M L r ] [M'L-'T-'J
Force =---~= ·
2
The area of a square of side L·= Lx L = L2 . Area [L ]
and volume of a cube of side L =.Lx Lx L = L3 Thus dimensions of unit of pressure are l,in mass, -1
Thus, the area and volume are said to be of 2 and 3 in length and -2 in time.
dimensions in length respectively. The unit of area which is 11. Kinetic energy = .!2·Mass x (Velocity) 2
the product of two lengths is represented as [L x L] or [L2 ].
Similarly, for volume we can write [L3,]. = [M1 ][L1 r 1 2
] = [M1 L2 r 2
]

. Distancetravel.led These are the same as those of work.


The speed or ve1oc1ty = - - - . - - -
- Time 12. Potential energy = Force x Distance
= [L'J = [L'r'J = [M1 L1 r 2 ][L1 ] = [M1 L2 r 2 ]
IT'] These are also the same as those of work.
Since area does not depend upon mass and time the 13. Couple or Torque = Force x Length of arm
dimensions of M and T are zero. The area is thus completely = [M1 L1 r 2 ][L1 ] = [M1 L2 T-2 ]
represented as [M 0 L2 T0 J. Thus dimensions of unit of couple, or torque are 1 in
Dimensional Equation (Formula) of Some Physical mass, 2 in length and -2 in time. ·
Quantities
14. Ang1e = -Arc- - = -[L'J = No d"1mens1ons.
1. Area = Length X Length = L1 X L1 = L2 = [M 0 L2 T0 J Radius [L1 ]
2. Volume = Length x Length x Length
15. Angular velocity = ~gle = IT- 1 ]
= L1 X L1 X L1 = L3 = [M 0 L3 :r 0 J Time
Thus dimensions of unit of angular velocity are -1 in
3. Velocity= Distance= [L'J = [MoL'r'J
time
Time IT'J
Thus dimensions of unit of velocity are O in mass, 1 in
length and -1 in time.
www.puucho.com
Anurag Mishra Mechanics 1 with www.puucho.com

·~-. ,,.~-, -;:, • '"'-r-r".;": ......~••",.L•'


15--· , •"' ,, ,:";:i • ,-
1 ,.,, ._, .:·' ' ,,:: •. ', ,.,, ,.. JJ"'''-···"MECHANICS-1:j
--~-_c---·---~·..,,,__,
, · An · . Change in angular v~locity'"".--'-'-'----
16• gu 1ar acce1eratton = h = Jj__ = [~'1•;.-2l.= [M1L2F'J,
· Time interval , V ,: [ r ) , _-,
. 1
= rr- l = rr-•1 Force '
27. Force constant = - - - - -
. ['I"l] Displacement .
17. Angular impulse = Couple x Time [ 1L1F 2_]
M , '
= =-------"-" [M1r•1
= [M1L2T-2]['I"1] = [M'L-2T-'] [L']
The dimensions pf unit of angular impulse are 1 in 28. Surface tension : It is defined as the force per unit
mass, 2 in length and -1 in· time. length in the surface of a liquid. : ·.,.,
2
_ Force _ [M1L1r J _ ·[M11_,T_21 . . Force [M L T-2]
1 1
18. ,Stress - - - - ~ - 2- ~ - :. Surface tensmn = - - - - -1- - [M1T-2]
· Area . [L ] . ,, Length. [L ]
The dimensions of unit of stress are 1 in mass, -1 in 29. Temperature = [8 ]1
length and -2 in time. Now-a-days it is taken as fundamental quantity in SI
. Change in length units and is expressed in kelvin (8).
19. Stram=--~-~~ [L: l = No dimensions
Original length [L] 30. Heat = Energy = [M1r:'r2]
. 8 or o ther T.
20. sm . ratios
ngonometr1c . = -[L'] It has the same dimensions as those of work i.e.,
[L'J [M1L2 r 2
]

= No dimensions Energy
31. Specific heat = - - - - - - ' ~ - -
Mass x Temperature .
21. · Density = Mass = [M'J = [M1L-3] I 2 2] · · . -
. Volume [L3] = [ ML r . = [L2T-20-'J
[M1 ][81 ] .
The, dimensions of unit of density are 1 in mass and ~3
in length. 32. Latent heat , = Heat
energy = [M'L2T-2J [L•r•J
"fi .
22 • Spee, c grav,ty =
Mass of body ·
. Mass [M1J
, Mass of equal volume of water Pressure X Volume
33. Gas constant , R = - - - - - - - -
. · Moles x Temperature
= [M:] = No dimensions
[Ml [M1L-1T-2frL3]
1
= '[mol1][8]
23. · Frequency; v = [T-1]
Timeperiod = [M1L2T-20-1mol-1 ]
24. Gravitational constant : The force of attraction 34. Boltzmann's constant,
between two masses m1 and m 2 lying a distance r Heat energy· · · [M1L2r 2] ·
k -= [M1L2T-28-1]
apart is give~ by F =· G m; m2 Temperature [81]
r•
These are the same as those of the gas constant ·R
where G is the gravitational constant. Thus
35. Coefficient of thermal conductivity : The total
Fr 2
G=-- quantity of heat Q flowing through an .area A of a slab
m1m2. of thickness d in time t, when the two opposite faces
11 2 2 are at temperatures 8 2 and 8 1 is given by
or [G] = [M L r ][L ] = [M""'L3r2]
[M1 ][M1 ] • Q=KA.(0 2 .:.0,)t ·
25. Young's modulus of elasticity: It is defined as d
the ratio of the stress to the.longitudinal strain. Thus where K is the co-efficient of thermal conductivity.
y =Stress= F/A = F.L Thus,
Strain 1/L A. I
[M1L1T-2][L']
or [Y] = =--___..;=--=-
2 1
[M1 1:1·r2 J
[L ][L ]
26. Planck's constant : According to Planck's law, the
energy E in a wave of frequency v is given by Applications of Dimensional Equations
E=hv To change from one system of units to the
':"her~ h is the Planck's constant. Hence other : If the measure of a quantity in units u1 is n 1 and

www.puucho.com
Anurag Mishra Mechanics 1 with www.puucho.com

that in terms of units u 2 is n 2 , then as the quantity measured


is the same in both ·the systems, we .have n 2 =1.013x10 - - -1- 6[
1000 100
1]1[ ]-'[1]-2•
-
1
n1u1 = n2u2 , 6 3 2
n2 = 1.013 X10 x 10- x 10 = 1.013 x 10 5 Nm-•
Suppose a given quantity q has dimensions fl, b and c in
mass [Ml, length [L] and time [Tl respectively, then the
dimensional equation for this quantity is [M"LbT'].
[E~l~t~1 3 ~
--,
If the fundamental units are M1 , L1 and T1 of the first Find by dimensional ll!ethod the value of.Yin SI units when I
system in which the numerical value is n1 , then the quantity Y=20x10 11 dmecm-2 • _ · _ . ·_. _ _ _ _ _ 1

q = n1 [MfL~T{l Solution. Dimensions of Y = [M1 L-1 T 2]

=n,[::]'[~:r'[;:r
Similarly, for the second system for which the 2

fundamental .units are M2,L 2 and T2 and the numerical Now, n2


value is n 2; we have
' . . '· a b C
q = n,[M2J;2 T2l = 20_x 1011[~]'_ [lcm]-'[lsec]-2
Hence,. n 1 [MfLiT{] = n,[M~L\T~] · 1kg .- lm 1sec
1
~2 =n,[::r[~:r[;:]' = 2ox1011 x --x 100.
1000
= 20 X 10 10 Nm-2
1- -= --
. . . .,-:..::::~~
~~_g~m;,~ 4 ~ '
C . . . ------.
/dsin,i,ilime~ions find the value of'g' in'MKS system. TheJ 'if velocityof. light, acce/eration due. to gia_vity· and normal
\Value_iri;cgs syst~98{). _ " •:,'. ' C : '
atmospheric pressure are taken asf ~~~'fun~amental units,
Scil!,ltic:m: Her~ n1 = 980,. n 2 = ? what will 'be t1ie units of. mass, length 'and time ·?-·Given
L1 = 1cm, i. 2 = lm = 100cm velocity of.light 3 X 10 8 ms-1, fl!'Celeratfon uue.to gravity j
. T1 = 1sec, T2 = 1sec 10 ms-2 and normal p.ressure -·1Q_5 Nm~2 .: _ _J

Dirirensions .of acceleration are L1 T-2. Solution.

"'"."'a": ~--~r::m:r[;J
Velocity= [L1T 1 ] = 3 x 10 8 ms-1 •.. (1)
Acceleration= [L1T 2] = l0ms-2 ... (2)
Pressure= [M1 L-1 T 2] = 10 5 Nm~2 ... (3)
0 2 Dividing, eqn. (1) by (2), we get
'
= 98o[M
M2
1
J [__!__]
100 1
[I:]-
1

T1 = x
3 108
= 3x 107 sec ... (4)
10
= 9.8 Multiplying, eqn. (1) and (4), we get
Hence the value ofg in MKS system is 9.8 m sec-•.
11 = 3 X10 8 X 3 X107 = 9 X10 15 m
b~~~~.t~~;f21)a> From eqn. (3), we get
,----- - ~ ... M'= 105
iConvert a pressure of.76 cm of. me~cury into.Nm - 2 • Density of.:J [L-1r21
imercl!!)'. is 13.q_g,111.'._g:. __·___- - ~ - - - - - - - - - - ' -
Solution. Pressure P = hpg
. ,.
= 10 5 x 9x.1015 x 9x 1014
=8lxl0 34 kg
= -76Xl3.6X980

Now
= 1.013 x 106 dyne/cm 2
n1 = 1.013 x 10 6 n 2 =? ]
(E~F!l'7d
-------------~
~ 5

M1 = lg
1 1 ·= 1cm
T1 = 1sec
M2 = 1kg = 1000 g
L2 = lm = 100cm
T2 = 1sec
F
1
ck the accuracy of. the equati~ii · ..

'·,
1' (ii
n= 21,Vrif ·
!where l is.the length of.the string, m. iis mqss p~r unit length,
·)
Fj
'

Dimensions of pressure = [M1L-1T 2] ~he stretching force and n the frequencyyf.yibration. ____ '

www.puucho.com
Anurag Mishra Mechanics 1 with www.puucho.com

\8 MECHANICS-f 1
Solution. Dimensions of left hand side n = .! = [ r1 ] or t=kff
T
I
2 2 The constant k can be found out experimentally. It
Dimensions of right hand side= __!_[M'L'T- ]
comes out to be equal to 21t.
[11 ] M 1L-1
= [L-1 ][11 T-1 ] = [ r 1 ] t = 27tff
AB the dimensions of left and right hand sides are equal,
the relation is correct. ~~~~~~
~~Btm,l?J~J
1 · -··- ·- ··-·- -~-- --
6 ~-
-·- -.-----------1 ----r--· --··;
r;;:riv; dimensionally the relatio~
I. ...... -·---------·---
S = ut + !_;2 • ·--- --- ·--:

- - - - - _;;/____ ··- ___,. ___,


:check the a~c~racy of the relation
.
s=ut + -2 at 2
where s is)
. Solution. The distance s travelled by the particle
w/
;the distance travelled, by the body th. unifonn acceleration a: depends upon its initial velocity u, acceleration! and time t.
,in time. (and having ,initial velocity u. . . ___ · . · _ .. · __ J · Let
s =· ku" fbt'
Solution.
Substituting the di~ensions of various quantities, we
Dimensions of left hand side = [1 1
]
have
[L1 ] = [L1 r 1]"[L1 T-2 ]b [T1 ]' = [La+b][T-a-2b+,J
Dimensions of right hand side ut = [L1 r 1][T1 ] = [11 ]
Comparing the dimensions of similar quantities, we
_!,at 2 = [L1 r 2][T 2] = [L1] have
2
a+b = l; -a-2b+_c = 0
AB the dimensions of each term on the right hand side
are equal to the dimensions of the term on the left hand Since there are three unknown quantities a, b and c,
side, the relation is correct. two equations are not sufficient to find their valµes. To solve
it the problem can be split into two parts :
l2E~~~·Gl> ---- ·-·--,
(1) When the particle has.no acceleration. In such a case
s = k1uatc
''Deduce the relation for the time period ofa simple pendulum.J
or [11 ] = [L11"1 J"[T1 ]' = [L"TC-a+cl]
Solution. The time period 'of a simple pendulum can
a=l,. -a+c=O
possibly depend upon :
or a=c=l
(i) The length I of the pendulum,
Hence, s = k1ut
(ii) The acceleration due to gravity g,
(2) When the particle has no initial velocity. In such
(iii) The mass of the bob m and
a case s = k 2 Jbt'
(iv) The angular amplitude 0.
or [L ] = [L1 r 2 Jb[T 1 J' = [LbTC-2 b+cJ]
1
Since the circular measure of an angle has no
dimensions, let the time period t be proportional to a th b = l, -2b + C = 0
power of 1, b th power of g and c th power of m, then or c=2b=2
t = kl"gbm' ... (1) Hence,
where k is the dimensionless constant of When the body has both an initial velocity as well as
proportionality. acceleration, the equation of its motion must contain both
Substituting the dimensions of the various quantities, the parts.
we have the dimensional equations of both sides. s = k 1ut + k 2 ft 2
M0 L0 T1 = [L"(L1 r 2 )bM'] = [La+bT-2bM'] The value of k1 comes out to be equal to unity and k 2
According to principle of homogeneity by comparing 1
equal to-.
the dimensions on either side, we have 2
a+ b = 0, c = 0 and -2b = 1 Hence,
1 2
S=Ut+-ft
1 1 2
or b=--anda=+-
2 2
Substituting values in eqn. (1), we have
t = k11/2g-1/2mo
www.puucho.com
Anurag Mishra Mechanics 1 with www.puucho.com

: UNIT AND DIMENSIONS


Substituting the dimensions of all physical quantities.
[ML2 T-2 ]
[h] ; c....__ _c

If a composite physical quantity in tenns of moment of inertia [r'J


I, force F, velocity v, work W and length L is defined as, or [h] ; [ML2 T- 1 J
Q ; (IFv 2 I WL3 ), So SI unit or Planck's constant is kg-m 2/s which can also
find_the dimen_sions _of Q and identify i(. be written as (kg - m 2/ s2 ) x s. But as kg- m 2/s 2 is joule, so
Solution. As, [I]; [ML2J, [FJ=[MLr2J unit of h is joule x sec, i.e., J-s.
- -
[v]; [Lr 1 ]and [W]; [ML2T-2] Concept (i) From examples (A) and (B) it is clear that
[ML2][MLr2J[Lr1 ] 2 'if a physical quantity is given, its dimensions are unique.
[Q]; - - ~ - - - [MT-2 ]
[ML2r 2] [L] 3 '(ii) There are also physical constants and coefficients which
As [MT-2] are dimensions of surface tension, force ;are dimensionless. For example, mechanical equivalent of
constant or surface energy, i.e., energy per unit area, the 'heaJ J.
physical quantity may be any one of these.
CONVERSION OF UNITS
Note:
This is based on the fact that for a given physical
From this problem it is evident that if dimensions are given,
quantity. Numerical value x Unit = Constant
the physical quantity may or may not be unique.
So when the unit changes, numerical value will also
change.
TO FIND DIMENSIONS OF PHYSICAL A. The Newton into Dyne
CONSTANTS OR COEFFICIENTS
The newton is the SI unit of force and has dimensional
Write any formula or equation incorporating the given
formula [MLT-2] ;
physical constant and then substitute the dimensional
formulae of all other quantities to find the dimensions of the so 1 newton= 1 kg m/s 2
required constant or coefficient, but 1kg; 10 3g and lm; 10 2 cm
A. Gravitational Constant G lN; (103g)(l02cm) 10s gem; lOsdyne
so
Method-I Method-II s2 s2

From Newton's law of From the relation between B. Gravitational Constant G from CGS to MKS
gravitation, we have G and 'g' we have System
GM The value of G in CGS system is 6.67 x 10-8 CGS units
F
; Gm1m2
T g;F? while its dimensional formula is [M- 1 L3T-2];
so G;6.67x10-8 cm 3/g-s 2
or G; Fr or G;-
gR2
but 1cm; 10-2 m and lg= 10-3 kg
m1m2 M
2
so G; 6.67 x 10-8 (l0" m)3
Substituting the dimensions of all physical quantities. (10-3 kg) (s 2 )
[MLr2] [L2] [Lr2J[L2]
[GJ ; [MJ[M] = [M] ; 6.67 x 10-11 m 3 /kg - s 2
i.e. G ; 6.67 x 10-11 MKS units (or SI units)
or [G] = [M-1 L3 r 2 J
C. Density from a given System to a New System
So its SI unit is m 3/kg-s 2 or Nm 2 / kg 2
Suppose we have a new system of units in which unit of
B. Planck's constant h length is 5 cm and unit of mass 20 g, i.e.,
Method-I Method-II 5 cm= 1 La (say) and 20 g = 1 Ma (say)
According to Planck : de-Broglie : then density of a substance, which is (say) 8 glee, in this
new system will be,
E ;hv ,. ; _!:_ _g_; [lMa / 20] SO Ma
mv 8 8
cm 3 [lLa / 5] 3 La 3
E
or h;- i.e., in this new system the value of density will be 50
v
units.

www.puucho.com
Anurag Mishra Mechanics 1 with www.puucho.com

[10
. ·~=-c":C=:-c--==-=- :=::-===c:~ ·. •.. ,, MECHANICS-II ·
-TO CHECK THEDINIENSIONAL····---'~'--A-.-E-in_s_t-ei-n=M_a_s_s_-E_n_e-rg~y~R'e~l-a-ti-on-~-,"'---~~
CORRECTNESS OF A GIVEN PHYSICAL If it is known that when mass is convened into energy.
RELATION Let the energy produced depend on the mass (m) and speed
'Principle of homogeneity' states that the of light (c), and the function to be product of power
dimensions of each term on both sides of an equation must functions of m and c, i.e.
be the same. Mass can be added to mass· to give mass and E =Kmxcy
not to length or time.
Where K is a dimensionless constant. If the above
If the dimensions of each term on both sides are same, relation is dimensionally correct.
the equation is dimensionally 'correct, otherwise not, A
dimensionally correct equation m·ay ·or may not be [ML2 r 2 1 = [M]"[Lr 1 ]Y
physically correct. or [ML2 r 2
l = [MxI!rYl
A. Check the Correctness of the Formula Equating the exponents of similar quantities on both
2
F = mv /r 2 sides of the equation
Dimensionally, x=landy=2
[MLT-21= [Ml [Lr 1 l 2/[Ll 2 Thus the required physical ,relation becomes
i.e., = [Mr2J, E =Kmc 2
The value of dimensionless constant is found unity
As in the above equation dimensions of both· sides are through experiments
not same ; this formula is not correct dimensionally, so can
E = mc 2
never be correct physically.
B. Stokes' Law
B. Check the Correctness of the Formula
When a small sphere moves at low speed though a fluid,
2
S=Ut-(½)at . the viscous force F, opposing the motion, is found
experimentally to depend on the radius r, the velocity of the
Dimensionally, sphere v and the viscosity Tl of the fluid. ·
.[Ll = [Lr1 l[Tl-[Lr2l[T 2l If the function is product of power functions of 11; r and
i.e., [Ll = [Ll - [Ll v,
As in the above equation dimensions of each term on F = KTtxryvz ... (1)
both sides a_re same, so . this .equation. is dimensionally where K is dimensionless constant. If the given relation
correct. .. . is dimensionally correct.
However, from equation of motion· we know that [MLT-~] = [ML-1 r 1 ]"[L]Y[Lr1 ]z
s = ut + (1/2)at 2 . or
. '
So the given equation is phys/~ally wr_ong thoU:gh it is Equating the exponents of similar quantities on both
correct dimensionally. sides of the equation.
C. Check the Correctness of the Formula x=I;-x+y+z =1 and -x-z =-2
T = 21t.,fI/mgL I Solving these for x, y and z, we get x = y = z = 1
Dimensionally, So eqn. (1) becomes
[T] [ML2l = [Tl F=Kwv
[Ml[Lr2 l[L] On experimental grounds, K = 6Jt;
As in the above equation the dimensions of both sides so F = mtTtrv
are same, the given formula is dimensionally correct. It may C. Planck's Length =·
or may not be physically correct. However,.from the theory Construct a new physical quantity hi~ng-dimensions of
of physically pendulum we know that T = 21t.jI I mgL. So length in terms of universal constants G, c and h
the given formula is both dimensionally and physically
correct. If the function is the product of power functions of G, c
and h,
AS A MATHEMATICAL TOOL TO ... (1)
DERIVE NEW RELATIONS
where K is a dimensionless constant proportionality.
The principle of homogeneity of dimensional analysis
If the above relation is dimensionally correct,
provides us with a powerful tool to discover new laws
relating different physical quantities. -r-
[LJ.=[M-113 2]"[Lr1 JY[ML2 r 1 Y
Following examples will illustrate the method : i.e., [Ll = [M"x+zL3>c+y+2zT-2x-y-zl
www.puucho.com
Anurag Mishra Mechanics 1 with www.puucho.com

·-~!U_Nf_TAN_D_Df_M_EN_SI_ON_S_ _ _ _ _ _ _ _ _ _ _ _ _ _-'--_ _;__-'----'·-:,.,.~,:~·c,;..c\"----~----·''·i_j]


Equating the exponents of sl~ilar quantities on both if dimensions· are g!ven. ·For example,._if.)he di111ensional
Sides Of the equation, ' . , formula of a physical quantity is'[ML2T-2l, it may be"work or
'i I ,

-x+z =·o, 3x·+ y + 2z·=·l 'and -ix·-y-z = 0


'
energy or torque.
. .
- ,., . , , .
0

Solving ·these for x, y and z, we get (2) Numerical constant [Kl having no dimensions such
1 ·, .:.3 1 as (1/2), 1 or 21t, etc., cannot be expressed by,.the niethods
x=2; Y =2 and Z=- of dimensions. · · .,.-
2
So eqn .. (1) becomes, (3) The method of dimensions cannot used. to derive
QL =KG112c-,12h v2 relations involving produci:,of physical qua~tities.,'it-cannot
be used to' derive relations other )hari product of power
If the constant K is assumed, to be unity functions. For example, · ' . ,, ,
. QL =~Gh/c 3,' s = ut + (1/2) at. 2 , Qr·... y =.asincot '
0

Cannot be derived by USlng- thi~, th~Ory.: , ' , I , •

L.!=~"A-t;ll\RJ~,.@> We can check the dimensional correctness of these


fij ::e~city,. f~;c~ ~d- time are tizl~n to be fundamental relations. , . ,
(4) Dimensioni,J analysis is 'not ~seful for· deriving
tquantities find dimensional formula for (a) Mass, and ·
~(l,)_Efl~r~--_ _ _ _ _ _ _ _ _ _ _ _ _ _ _ _ _ _., formula for a physical quantity that depends·on more than 3
physical quantities as then ,there will be less number of
Solution Let the quantity be Q, then equations than the unknowns. However,. still we can check
· Q = f (v,F,T) correctness of the given equation dimensionally.
Assuming that the function is the product of power For example, T =21t~I I mgL cannot ·b~ derived _by
functions of v, F and T, · theory of dimensions but its dimensional correctness can be
Q =K vxpyyz .... (1) checked.
where K is a dimensionless constant of proportionality. (5) Even if a physical quantity depends on 3 physical
quantities, out of which two have same dimensions, the
The above equation dimensionally becomes.
formula cannot be deriv~d liy theory of dimensions, e.g.'
[Q] = [LT- 1 Y[MLT-2]Y[T]'
· formula for the frequency of a tuning fork f =.(d/L2 )v
i.e. .. .. (2) cannot be derived by· dimensional analysis.
' '
Now
PHYSICAL QUANTITIES FROM ., .
(a) Q = Mass i.e., [Ql = [Ml . HEAT AN!;> TH~,R~ODYNAMICS
So eqn. (2) becomes
[Ml = MyLx+yT-x- 2y+zl 1. Temperature: It is _a 'fundamental quantity with
dimensions [0l and unit kelvin '[Kl.
its dimensional correctness requires. 2. Heat: By definition," it is energy transferred due to
y = 1,x+y = Oand-x-2y +z = 0 energy transferred, so its dimensions' ate [ML2T-2l arid SI
which on solving yields unit joule (J). ~racticai unit of heat .is calorie (cal} and 1
x=-1,y=landz=l calorie = 4.18 joule. · . · .
Substituting it in eqn. (1), we get 3. Coefficient of lin~ar-Expansion a
' ' Af.
Q=Kv-1Ff, It is defined as a= - ' .
(b) Q = Energy i.e. [Q] = [ML2T-2l L,i0
Le., '[al =[B-1 l . '
So eqn. (2) becomes
[ML2T-2l = [MYLx+yrx-2y+zl So its unit is (C 0 i-1 or K~J'
Which· is the light of principle of homogeneity yields 4. Specific heat c
y = l;x+y = 2and-x-2y +z = -2 As Q= mc,iB;
Q'
which on solving yields So c=---
mi\0·. -:
X=y=z=l
. [ML2 F 2 l
So eqn. (1) becomes Le. [cl= c......,~~
' "[M][0l
Q=KvFf
i.e. [cl= [L2 r 20-1 l
Limitations of Theory of Dimensions
(1) If dimensions ate given, many physical quantities So its SI unit -will be . J/kg:~. "".hile· practical unit
have same dimensions. Physical quantity may not be unique cal/g-co.
www.puucho.com
Anurag Mishra Mechanics 1 with www.puucho.com

12
5. Latent Heat L
By definition, Q = mL ,
[p + Va2 ] (V - b) = RT
i.e., [L] = [ML2T-2] / [M] a ab
or PV+--Pb--=RT
i.e., . [L] = [L2r2] V V2
So its SI unit will be J/kg while practical unit caVg. As this equation is dimensionally correct, each term on
6. Coefficient of thermal conductivity K either side will have same dimensions i.e.,
According to law of thermal conductivity, Heat [a/V] = [PV]
transferred per second or [a] = [ML-1r2J [L3] [L3] = [MLsr2]
dQ =KA dB and [P x b] = [PV]
dt dx or [bl= [V] = [L3 ]
or [K]=--~-
[ML2 r 2
/T]
.
2
[L ][0/L]
11 _~
i.e., [Kl= [MLr30-11
l~rt. is es_ timat·e·d· ;h.at p-;;;;;_.in.~te ~~c-h ~~--_i_~i~~rth.: -~_;_e_"iv;;:;j
about 2 c°;lorie of heat energy from the sun. This constaizr ~
Its SI unit is W/m-k while practical unit is caVs-cm-C'. 1
7, Mechanical equivalent of heat J called solar constant S. Fxpress solar constant in SI units._;_!
According to Ist law of thermodynamics work and heat Solution. Given that
are related as S = 2cal/ cm 2 -min
W=JH But as 1 cal= 4.18J, 1 cm= 10-2 m'and 1 min= 60 s
or
w
J=- S= 2x4.18 -1. 4 x 10 3_J_
H
c10-2 mJ2(60s) m 2s
= [ML2 r 2 ] '
2 2
or [J] or S=l.4k-W/m 2
· [ML r ]

i.e., - [J] = [M 0 L0 T 0 ] r .
.d:E,xa•:"u.;.,i;e:~~-----[i21"--,-;---.
t.-·--··--~:\ii.~-:..=.:.~.-J
12. ~
i.e., J has no dimensions. Its practical unit is J/cal r----------·· -- ---------------·
and has value 4.18 J/cal. Ip= P0 e-cot2J
I
8. Boltzmann constant k iflnd_@IJ)~nsions_of a.,. where P~= pressure, t = lime.~_ __J

According to kinetic theory of gases, energy of a gas


Solution. Exponential and trigonometric function are
molecule is given by
dinlensionless.
3
E =-kt [a.t 2 ]= [M0 L0T 0 ]
2
2 2 [a.]T2 = [MOLOTO]
-Le· [kl= [ML r ] i.e. = [ML2r 20-11
[0] [a.J = r2
So its SI unit is J/K and its practical value
1.38 x 10-23 J / K
9. Gas constant R
According to gas equation, for perfect gas,
PV=µRT
f-=Exam,,.,.il,e
- -- --"-""' ___ -
,.

',
;! l j~
_____ ___ -------
a
~

. a-ct
The dimension of b in the equatwn P_= _b_x_ whe-re
P = pr:_es_sur_e,_~ =_g~p_lacement aml L.=.. time _ _ __
2
J
[ML-1r 2][L3]
So [R] =- - - ~ -
[mo!] [0] Solution. [Pl =[ b:]-[ c::]
or [Rl = [ML2r 20-1mol-1J
By principle of Homogeneity,
So its SI unit is J/mol-K while practical unit is
cal/mol-K. It is a universal constant with value 8.31 J/mol-K
or 2 cal-K.
[P] = [b:] = [ c::]
10. van der Waals constants a and b
[i]i=[ML-1T-2]
According to gas equation, for one mole of a real gas

[i]=[Mr2]

www.puucho.com
Anurag Mishra Mechanics 1 with www.puucho.com

(uNIT AND _DIMENSIONS 13


Method-II
V= p•pb
r=- . . ----- - ---,
The position of a particle at time t, is given by the equation, I V = kP"pb:

x(t) = v; (1-e.-ac ), where v 0 is~ constant and ct> 0. 'The\ [1r1] = [ML"1T-2]"[M1-3 t
1 1
_dimensions ofv 0 • & ct are respectively. _. __________ ; a=- b =--
2.' 2
Solution: [v 0 ] = [x][cx] = [M 0 11 r 1 J => . [v] = [P1'.2P-112]
and [ct][t] = M01°T 0 , [ct]= [M0 1°r11
E,E~o:~f~,t~J.ii];;>- kJ;_:>S.QnilRl&J 17 f~
~-;;-ng,; mod~lus oi s;eel is 19 XJ010N/m 2
• Express i~-ij
'
'When a solid sphere moves through a liquiq, the liquid ,clyne/cm 2
• Here dyne is the CGS unit of force.
iopposes the motion with a force E The magnitude of F depends
Solution: The unit of Young's modulus is N/m 2.
10n the coefficient ofviscosityT] of the liquid, the radius r ofthe
lsphere and the speed v of the sphere. Assuming that E is This suggest that it has dimensions of Force
!proportional to different powers of these quantities, guess a (distance) 2
iformulafor F using the method of dimensions. Thus, [Y] = [Fl = [M1F2] = [ML1r2]
Solution: Suppose the formula is F = kri"rbv' [12][12]
2
N/m is in SI units, ,
Then, [M1r2] = [M1-1r 1]"[1]b[½]' So, 1 N/m 2 = (1 kg) (1 m)-1 (ls)-2
= [M"1-•+b+cT-a-c] and 1 dyne/cm 2 = (1 g) (1 cm)-1 (ls)-2
2
Equating the exponents of M, 1 and T from both sides,
a=l
-a+b+c = l
so, 2
: ; ~ =(\:g)(1
1
1
:r\~:r
= lO00x-xl = 10
· -a-c = -2 100
Solving these, a = l, b = l and c = 1 or, 1 N/m 2 = 10 dyne/cm 2
Thus, the formula for Fis F = /crirv. or, 19x1010 N/m 2 =·19xl011 dyne/cm 2, -
b~E?<943ll'l~"I~,__~

/If P is-~he pressure o_if a gas andp. is its d.-


!dimension ofvelocity.
(a) pl/2p-112 (b) pl/2pl/2
.,;i;,, JE 1-,E~x:~mpJg.J~
imensional formula for viscosity of fluids is,
. T\ = [M'i-ly-:l J
Fmd how many poise (CGS _unit of viscosity) is equal to 1
]

tc)_P-11~P.112 Cc!LP-112r>.-1/2 __ lp_oiseuill~(SI unit of.viscosity)]__ · ___________


Solution: Method-I Solution: T\ = [M1 1-1 r 1 1
[P] = [ML-1r2] ... (1) 1 CGS units= g cm-1s-1
[p] ': [M1-3] ... (2) 1 SI units_= kg m-1s-1
Dividing eq. (1) by (2), = 1000 g·(loo ~mi-1 s-1
[Pp-1] = [12r2] . = 10g cm-1 s-1
=> [1 r1] = [P112p-112] Thus, 1 poiseuille = 10 poise
=> [v] = [P112p-1121

www.puucho.com
Anurag Mishra Mechanics 1 with www.puucho.com

s
'j
\ji
1""'
V ~")

-~~----h-:-·~
',;-r::~:t
~ ,,. ,~· '

i. In two systems of tmits,"the relation betw~en velocity,


acceleration ,': ,:, iµid ''. ,force : is ' given . by [Ans. u~ = (p\ )uPJ
2
' 'v·e '· · , , F • , .
v 2 =-1-·-,a:i=·a1E-r,F2 =_!.; where e and 't are 6. Specific heat of hydrogen at constant pressure, ·
't_ .E't
I,.~ . , .•,' 1
C P = 29 joule _kelvin-1 mo1-1 •
constants then.find in this new·,system:
m2 . . . . ', (b.) L2- , (a) Find dimensions of C P.
( a) . . . .
. ', ml ' . '. . ~I
(b) Unit of length is changed 'to 50 cm, unit•of ti.me is
changed to 2 sec, unit of temperature is changed to
' ' '
, [Ans. (a). ·}
'

E -r
'
~ ; (9)
'

't .
;1 ·3 '.
2K. keeping units of mass· and amount of substance
same. Find the value of specific heat of hydrogen in,
. •· . . a .
. nRT - - . · · new system of units: .
2. In the formula P =- - e ·Rn,;, find the dimensions of [Ans. (a) [ML2 T-2 K-1 mol- 1],(b)928]
. ,: V-b . ' . ·
a and b where p ' = pressure, n = no: of moles,
T "" temperature, -V = volume'. and R = universal gas.
constant. . , . · ·
m_oniy~o;;';"Altefu';;tiC'ilifoI@~
[Ans. (a),.;. [ML5 T-2 inol~1 ], (b) = [L3 Il 1. E, m, L, G denote energy, mass, angular momentum &
3. The loss of.pressure when a fluid_ flows through a pipe ·gravitation constant _respectively. The dimensions of
is'!liven by P-;= -kp"l Vbd'µ .where d and l are diameter EL2
""'"s'2 will be that of :
arid"length\,ftlie pipe respectively, p, d and µ are the mG
ma~s, ·density arid coefficient of viscosity of the fluid, V (a) angle (b) l~ngth
is the mean ·velocity of flow through the pipe and k is a (c) mass (d)' time ,
huinerka! ·constant. Find the ~alues .of a, .b and c. 2. The dimensional formula for which of the following
[Ans a·=ib'"h'= pair is not the same ?
• • . ' c:'='-21-
M°gh·, ·~.;. i·
i__ , .
,.,
. . . (a) impulse and momentum ' . ''
nxYT - - · . .
4. P =--. e .nxi;; where n is number of moles, P is (b) torque and work
Vo ·, . . . . ,, . . (c) stress and pressure
pr~ssui-e, Tis te~perature, Vo i; vol~~e, ,M is mass, g (d) momentum and angular momentum
~epresent~ ~ccel~ration iliie to g~avity and h is height. 3. If the speed of light (c), acceleration due to gravity•(g)
····Finddiinehsioii'ofxandvalueofy., _·. · · · ,, and pressure (p) are taken as fundamental units, the
[Ans. [Mi.2 r 2 K-1 mo1-rJ; y = 1l · dimensions of gravitational constant (G) are : ·
5. The uni~· CU) ~fvel~city,:;ic~eleratio~ ruid force in two (a) [c2g3p2] (b) [cog2p-']
systems are related as under : . (c) [c2g2p-2] (d) [cog p-3]

(a) U~ = ~Uv
p ', .
2 '
. (b) u: = ,. (aP)U•.
.
4. Which of the following combinations of three
dimensionally different physical quantities P, Q, R can
never be a meaningful quantity?
Cc) u~ = [_!_];F (a) PQ - R (b) PQIR
. ap. . . . . ,
(c) (P-Q)/R (d) (PR-Q 2 )/QR
'All th~ pnme<hymboii, b~long to one system and 'cct') ·s. In a view unit· system, I unit of time is equal to i'o
· unprim¢'d' pnes CU) b.elong to the other systems ..a and second, I unit of mass is 5 kg and I unit of length is 20
p are· climensiohless ~onstants: How:m9mentum units m. In the new SVStem of units_ ,, nnit nf Pm>ro-v is equal
of the' !WO systems. are 'related,? , . . : . . . to:
. I-'·-;,' . ·.' ; ' ..
' i:_. •., , '' '~ www.puucho.com
Anurag Mishra Mechanics 1 with www.puucho.com

\UNIT AND DIMENSIONS ·


(c) Dimensional formula 9f k)s [MLT2]
Jc_ joule
{a) 20 joule
(c) 4 joule
(b)
20
(d) 16 joule
(d) Dimensional formula of l is [T]
. . a. . a-t 2 13. If P and Q have different non-zero dimensions, which
6. The d1mens1ons of - m the equanon, P = - - where
b bx of the following operations is possible?
P is pressure, x is distance and t is time, are: (a) P +Q (b) PQ
(a) [M2LT-s] (b) [MT-2]
(c)P-Q (d)l-~
3
(c) [LT ]' (d) [ML3T 1] Q
2
7. The time dependence of a physical quantity p is given 14. In the formula X = 3¥Z , X and Z have dimensions of
. 2
by p = p 0 /-a< J where cxis constant and tis time. The capacitance and magnetic induction respectively. What
constant a: are the dimensions of Y in MKS system?
(a) [M-3L-tT3Q4] (b) [M-3L-2T4Q4]
(a) is dimensionless
Cb) has dimensions [r-2] (c) [M-2L-2T4Q4] (d) [M-3L-2T4Q']
2
(c) has dimensions [r 2] 15. A cube has a side of length 1.2 x 10- m. Calculate its
(d) has dimensions of p volume:
8. If area (A), velocity (v) and density (p) are base units, (a) l.7xl0-6m 3 Cb) l.73xl0-6 m 3
(c) l.70xl0-6 m 3 (d) l.732xl0- 6 m 3
then the dimensional formula of force can be
represented as: '• 16. Pressure depends on distance as, P =!:exp(--az}
(b) [Av 2p] ~ k8
(a) [Avp]
(c) "[Avp 2 ] (d) [A 2vp] where ex,~ are constants, zis distance, k is Boltzmann's
constant and 8 is temperature. The dimensions of ~
9. Two forces P and Q ad at a point and have resultant R.
2 2 are:
If Q is replaced by (R - P ) acting in the direction (a) [MoLoTo] (b) [M-lL-IT-1]
Q (c) [M 0 L2T 0 ] (d) [M-1L-.1T 2]
opposite to that of Q, the resultant :
17. A 'wire of length I =6 ± 0.06 cm and radius
(a) remains same Cb) becomes half r = 0.5 ± 0.005 cm and mass m = 0.3 ± 0.003 g.
(c) becomes twice (d) none of these
Maximum percentage error in density is :
10. If instead of mass, length and time as fundamental
(a) 4% ·Cb) 2%
quantities, we choose velocity, acceleration and force
(c) ·lo/o (d) 6.8%
as fundamental quantities express their dimensions by
18. Which of the following sets have different dimensions?
v, a and F respectively, then the dimensions of Young's
(a). Pressure, Young's modulus, stress
. modulus will be expressed .as : Cb) Emf, potential difference, elec)ric potential
(a) [Fa 2 v-4] Cb) ·[F 2 v-1 a] (c) Heat, work done, energy
(c) [Fa 2 v-1 ] . . · (d) [Fav-2 ] (d) Dipole moment, electric flux, electric field
11. Which of the following statements is correct about 19. Which of the pair have same dimensions ? ·
conversion of units, for example 1 m = 100 cm? {a) Force and strain
(a) Conversion of units have identical dimensions on Cb) Force and stress
each side of the equal sign but not the same units. (c) Angular velocity and frequency
Cb) Conversion of units have identical dimensions on (d) Energy·and strain
each side of the, _equal sign but not the ss1me .units. 20. Tlie physical quantities.not having same dimensions are:
(c) If a larger unit is used then numerical value of (a) torque and work
physical quantity is large. (b) momentum and Planc~s constant
(d) Due to ·conversion of units physical quantity to be (c) stress·and Young's modulus
measured will change. (d) speed and (µ 0 e 0 )-1/ 2 ; ·
12. If the speed v of a particle of mass m as function of 21. The dimension of coefficient of viscosity is : .
time t is given by v = ro A sin[( l}] . ~ere A has
(a) [ML-1 r 11
0
(c) [ML T ] 2
Cb) [MLT2l
(d) [MLT1 ]
dimension of length. 22. A particle is moving eastwards with a velocity of 5
(a) The argument of trigonometric function must be a rn/s. In 10 sec, the velocity changes to 5 rn/s
dimensionless quantity.' , · northwards. The average.acceleration in this time is:
Cb) Dimensionai'formula of ro is [LT-1] (a) zero
-----.
www.puucho.com
Anurag Mishra Mechanics 1 with www.puucho.com

\16 MECHAf/1~

(b) }z ms- 2
towards north-west 5. The pairs of physical quantities that have the same
dimensions in (are):
(c) }z ms- 2 (a) Reynolds number and coefficient of friction
towards north-east
(b) Curie and frequency of a light wave
(c) Latent heat and gravita,tional potential
(d) .! ms-2 towards north (d) Planck's constant and torque
. 2
23. Out of the following the only pair that does not have
identical dimensions is:
(a) angular momentum and Planck's constant
m:F~~i~:.~· "~
(b) moment of inertia and moment of a force A student forgot Newton's formula for speed of sound
(c) work and torque but he knows there were speed (v), pressure (p) and
(d) impulse and momentum density (d) in the formula. He then start using
24. Which of the following units denotes the dimensions dimensional analysis method to find the actual
ML2 /Q 2 , where Q denotes the electric charge? relation.
V = kpxdy
(a) weber (Wb) (b) Wb/m 2
(c) henry (H) (d) H/m 2 Where k is a dimensionless constant. On the basis of
above passage answer the following questions:
25. The dimension of magnetic field in M, L, T and C 1. The value of x is :
(Coulomb) is given as :
(a) 1 . Cb) I
(a) [MLr'c'J Cb) [MT 2 C-2 J 2
(c) [Mr'c'J (d) [MT-2 C- 1 J 1
(c) -- (d) 2
2
mM~re.~~!!!2~1ternative:,:~"c;;~~ 2. The value of y is :
(a) 1 (b) .!
1. Which of the following dimensions are correctly 2
matched? (8 = temperature) 1
(c) -- (d) 2
(a) Angular momentum{M1 L2 T-1 ] 2
(b) Torque{M1 L2 r 2 J 3. If the density will increase the speed of sound will:
(c) Stefan's constant{M1 r 3 e-4J (a) increase (b) decrease
(d) Planck's constant{M1 L2 T-2 ] (c) unchanged (d) none of these
2. The gas equation for n moles of a real gas is
(P + ; )cv- b) = nRT where P is the pressure, Vis
=M=~!!,i~g Typ~];obi~~~
2
1. Match the column :
the volume , Tis the absolute temperature, R is the molar
gas constant and a, b are arbitrary constants. Which of
the following have the same dimensions as those of PV?
(a) nRT
(c) Pb
(b) a/V
(d) ab/ V2
I
1
(a) Energy density
(Energy per unit volume)
(p). dyne/ cm2 I
J
3. The dimensions of the quantities in one (or more) of I (b) Force constant of a spring (q) kg-m/s /
the following pairs are the same. Identify the pair(s) :
(a) Torque and work I (c) Pressure (r) erg/cm2
(b) Angular momentum and work
'
(s) pascal
(c) Energy and Young's modulus
· (d) Light-year and wavelength 2. Suppose two students are trying to make a new
4. The dimensions of length are expressed as Gx cY h •, measurement system so that they can use it like a code
where G, c and h are the universal gravitational measurement system and others do not understand it.
constant, speed of light and Planck's constant Instead of taking 1 kg, 1 m and 1 sec, as basic unit
respectively, then: they took unit of mass as a kg, the unit of length as f3 m
(a) X = (1/2),y = (1/2) and unit of times as y second. They called power in
(b) X = (1/2), Z = (1/2) new system as ACME then match the two columns.
(c) y = (-3/2),z = (1/2)
(d) y = (1/2), z = (3/2)

www.puucho.com
Anurag Mishra Mechanics 1 with www.puucho.com

UNIT AND DIMENSIONS 17 1

I \ Column-I \ \ Column-II : :j 4. Match the following:


(a) IN in new system (p) o;-lp-2y2
I \ Column-I ii... \ \ Column-II I
(b) lJ in new system (q) o;-lp-ly2 (a) F=Asin(Bt)+ v (p) [Al =[M 1 L1 r'l,
C ln(Dx) [Bl= [M 0 L0 r'J,.
(c) 1 pascal (SI unit of pressute) in (r} o:-'p y2 For above equation to be [CJ =[M 0 L0 r 1J
new system : dimensionallv correct
(d) o; ACME in watt (s) cx'p2y-J (b) Pressure = P + -1 pv
.2
+ gX (q) [Al =[M 0 L1 r 1
1,
' 2 [BJ = [M L T-1],
0 0
3. Match the physical quantities given in Column I with
[CJ =[M 0 L0 r 1J,
dimensions expressed in terms of mass (M), length
(L), time (T), and charge (Q) given in column II. (c) X=At+ V (r) [A] =[M 1 L1 r 2),
: B ln(Ct) [Bl= [M 0 L0 r'J,
I \ Column-I \ \ Column-II 1 [CJ =[M"1 L0 T 1],
(a) Angular momentum (p) [ML2 r 2l i
: (s) Dimensionally
(b) Torque (q) [ML2r1l ' incorrect
:
(c) Inductance (r) [M-1L-2T'Q2l (Where F = force, P = pressure, p = density, t = time, ,
' v= velocity, a = acceleration, X = displacement)
(d) Latent heat (s) [ML'Q-2l '
'
(el Capacitance (t) [ML3 r 1Q-2l '
(f) Resistivity (u) rer2l

www.puucho.com
Anurag Mishra Mechanics 1 with www.puucho.com

. . . ~ - MECHANICS-I '
'-1!1 -- - ------.-- --.----···-·--·•,--.-----·---·--------···T----~----- ··-- ----------·-·· - - __ J

AN9WER9 ------ -- . . -- - . ---

1. (a) 2. (d) 3. (b) 4. (c) 5. (a) 6. . (b) 7. (b) 8. (b) 9. . (a) 10. (a)
11 (a) 12. (a) 13. (b) 14. (b) 15. (a) 16._ (c) 17.· (a) 18. (d) 19. (c) 20. (b)
-- .
21. (a) 22. (b) 23. (b) 24. (c) 25. (c)

1. (a, b, c) 2. (a, b, c, d) 3. (a, c) 4. (a, b) 5. (a, d) 6. (b, c) 7.- (a, b, c)

=.!!!s~age:~ =:~
1. (1,) 2. (c) 3. (b)

=~l~~:,j'~f~~op1emj~
1. (a)-p, s; (b)-r ; (c)- p, s
2. (a)-q ; (b)- p ; (c)- r ; (d) s
3. (a) -q ; (b) -p ; (c) -s (d) -u; (e) -r; (f) -t
4. (a) -r ; (b) -s ; (c) -q

www.puucho.com
Anurag Mishra Mechanics 1 with www.puucho.com

-·, \\

DESCRIPTION OF MOTION

Mechanics is the branch of physics for studying the y y


motion of bodies, i.e., the change in their position in space
and time. y(t) ••
•••• •• t _ __ I
The position of a body in space can be specified only y(t)
relative to some other body or bodies. Therefore, when we ·~
speak about motion, we mean relative motion, i.e., the
motion of a body relative to another body which is x(t)
o ......: :'. X 0 X
conditionally assumed to be fixed. If we mentally attach a x(t)
coordinate system to the body taken as fixed and called the z<t> ......... ::Y
reference body, this system, together with the chosen (a) (b)
Flg.1.1
method of measuring time;forms the reference system.
Normally, the Cartesian coordinate system, is used.
2. Position of a particle in space is determined relative
to some fixed
SUBJECT OF KINEMATICS point. Position of
Kinematics is the part of theoretical mechanics in which a particle is not
the mechanical motion of particles and rigid bodies is studied absolute; it
without regard to the acting forces. depends on the
position of
1. In order to describe the movement of an object we observer. AJ

must specify its position relative to observer. One of Consider a train Fig. 1.2
the most convenient coordinate system is moving with
Cartesian coordinate system. It consists of velocity 10 m/s. A block kept inside the train is
three mutually perpendicular axes designated as visualized by two observers one on the ground and
x-axis and y-axis and z-axis. Location of any point is the other inside the train. For an observer in the
specified by three coordinates x,y, z as shown in Fig. train the block is at rest and for the ground observer
1.1. the block moves with the train (Fig. 1.2). In the
language of physics a technical term frame of
reference is used to describe a coordinate system
and position of observer. We say that in reference
frame of ground the block is in motion and in
reference frame of train the block is at rest.

www.puucho.com
Anurag Mishra Mechanics 1 with www.puucho.com

,20.
. ---- -- - -- MECHANICS-fl
- '"··---- ---- --------- ---- ------ -----· - - - · - - - - - - - - - - - - _J

Concepts. 1. In order to define the position of a particle magnitude and also follow laws of vector algebra are
in space, it ls tzecessa,y to have a fixed body or a system o/ called vector quantities.
co-ordinate axes attached to it which i.s called a reference C
system. !
2. By a reference sjstem ls meant an absolutely rigid boay,
or a co-ordinate system invariably attached to it with respect to·
wh_ich a given motion i.s considered.
The motion of a given body ls revealed only by
comparison with a reference system.
In some cases a moving reference system which executes
motion with respect to the basic reference system ls considered•
'

(a)
A

(b)
4"' A'
(c)
Fig.1.5
in kinematics.
3. In nature, no fixed bodies exi.st and consequently there The displacement from a point A to a point B is a
can be no fixed reference systems. A fixed reference system i.s vector quantity. Its magnitude is the straight line distance
usually assumed to be a system of co-ordinate axes attached to from A to B; its direction is that of an arrow that points from
the earth. · A to B. Points B and C are equidistant from point A but the
two displacements are different because they have different
3. Sometimes polar y directions. Two displacements (vectors) are equal if they
coordinates are have same length and same direction (Fig. 1.5).
used to specify the
position of a VECTOR NOTATION
A vector quantity is represented by a bold later with an
] ]·
partide. In it spatial-
position is denoted arrow above it or a bar above it.
by length r from
II
>, e ...,
origin and. angle e is ~::::==::=;~x
x=·rcose :
A or A
...,
generally measured The magnitude of vector is represented as IA I and it is
from ,p6sitive x-axis Flg.1.3 ...,
(Fig. i.3). referred as modulus of A.
x = rcos0'.' and y = rsin0 Geometrically a vector is shown _by an arrow dr& wn to
an appropriate scale. The direction of arrow represents
r=~X2+y2 and. tan0 = ~ direction of vector and length of arrow represents
X
magnitude of vector.
4. Trajectory of a particle y Displacement Vector
denotes the actual path
followed· by it. Path Displacement vector represents change in position of a
moving object. A car starts from Kota and travels north-east
length s(t) is defined as
to reach Delhi. Its displacement vector will be represented
the distance travelled
by an arrow joining starting point Kota to terminal point
along a trajectory in
time t. It is measured Delhi.
--- ---- - l
from the_ starting point '--------•x,
of the motion at t = 0. Flg.1.4
Path length is the total
laistance covered; it can only increase with time as a
particle moves, hences is·a]ways a positive quantity
(Fig. 1.4).
5. We always express results of our measurement in
terms of a number, e.g., room temperature is _25°C.
The value 25 is called the magnitude of the quantity.
Some quantities do not have direction associated
with them; such quantities are called scalar
quantities. Motion is a quantity that involves
direction as well as magnitude. We say a car is
. _ _ ___ Fig._ 1;6 _______ ..•
movin~ with velocity 10 km/h eastward. Such
quantities which have direction as well as

www.puucho.com
'
Anurag Mishra Mechanics 1 with www.puucho.com

, -DESCillPTION OF MOTION
:__________ - ·----- 21,

Displacement vector is straight line segment from initial


point Kota to terminal point Delhi. _, _, _, 7 ii'
Significance of Resultant Displacement
A team of hikers begins from A travel to B, C and D the
successiVe destinations. Net displacement, resultant
displacement, total displacement mean same thing.
Resultant displacement vector is straight line segment from
--;
A
2:::· --;
A

--;
.

--;
initial point A to final point D. Draw-A first and then B
--;
~---~A
o'.i··:..~--~
.
r.__, ., ,._1
--; --;
Draw B first and thenA

Fig.1.9

·A" ... <\s' ;_,;,.i"···-····· S2 Parallelogram Law of Vector Addition


-..J ..-
Actual path followed
_, _,
Consider two vectors P and Q, draw these vectors tail to
Fig.1.7
tail such that they represent the two adjacent sides of a
--) --) --) --)
parallelogram, the resultant of these vectors is represented
SResultant =S1 + S2 =+ S3 by the diagonal of the parallelogram passing through that
The above expression is_ symbolic representation of point.
vector summation. It_ just expresses that net displacement .............................. C
can be obtained by vector summation of individual
.,'
displacements, but how summation is to be carried we still
--; .: ''
---~/4
R
have to learn.
--;
Geometric_ ~eprese!1!ation_ of Sul!! of Vectors p

--;
s, Fig.1.10

Diagonal AC completely represents resultant vector


from triangle AEC
AC 2 ~AE 2 +CE 2
--;
s,
=(AB+BE) 2 +CE 2
~ Tail of second
/ '\ vector
=AB 2 +BE 2 + 2(AB)(BF' L.'£ 2
Head of first Substituting BE 2 +CE 2 =BC 2
vector
and BE =BC cusr
Fig.1.8
We get,
Each vector in a sum is to be drawn with its tail at head AC = AB 2 +BC 2 + 2ABBC cos8
of preceding vector. Resultant vector has tail at the tail of R =~P 2 +Q 2 + 2PQcos8 ... (1)
first vector and head at head of final displacement vector.
_, . _, Qsin8
Any of the vectors A and B can be drawn first. and tanct=-~-- ... (2)
P+Qcos8
Note: - ------------------
Alv. ys substitute magnitude of P and Qin above equations
(1 J and (2).

www.puucho.com
Anurag Mishra Mechanics 1 with www.puucho.com

'22 MECHANICS-I
Case I: If (1) Vector addition can be represented in two ways
..., ...,
IPl=IQI

and
...,
IRI= 2Pcos8/2
a=B/2
R
->
v,
+ ~ aLJ~ ->
v,
-> vectors to be added
p
Fig.1.11 are placed tai~ to tip.
Fig.1.15
Significance of Vector Addition
Illustration 1. Consider a
block that is pulled by two boys
Fa 100N
V2
··~·.·········...- :

simultaneously. Each boy exerts


force of 100 N. We can easily ~Fa100N
t ··" (2) ->
v,
a ~
->
v,
guess that block will move at 45° Correct
Fig. 1.12
angle. From rule of vector representatio'n
Fig.1.16
addition we can see that resultant force is
FR =..JlQ0 2 + 1002 Vectors to be added are placed tail to tail such that
= 10Q-J2 N they represent consecutive sides of a parallelogram.
Diagonal of parallelogram represents resultant.
Net force is ·vector sum of all the forces acting on the
object.
Illustration 2. A boat moves with velocity Y8 in still
....v, 1/ v;/"'(~·-··/;
water [Fig. 1.13 (a)]. Velocity measured by observer on
+ = ~ .... v,
ground will be y8 . But if water flows at a certain rate it will Incorrect
make the boat moye f~st':r ~s!<'~e~ ~:11e11d~ng_ on whether representation
.Fig.1_._17 _ _. ---· ··- _ _

t Vector Addition is Commutative


It is not important in which order vector are added
-+ --+ --+ --+
A+B=B+A
....Va
-,
V
->
V

(a) (b) Flg.1.18


Fig.1.13
'. '----~----··-----
..., ___ J
...,
the boat moves along the stream or opposite to the stream. Draw A first then draw B
I------ --
The velocity measured by an observer on the shore is the ->
A
vector sum of the velocity of the boat v 8 and the current
velocity ,I c ·
-> ..., ...,
V=Ve+Vc
Fig, 1.19
i
Illustration 3. Similarly if an ···- --_,--·--·-- I _,
airplane moves in a wind its res.ultant ..., ->
-> f-VwJnd ! Draw B first then draw A
velocity will be a combination of its , Vairplane --+ ' !
own velocity and velocity. of wind. V resultant I Resultant obtained is same in both cases.
--+ --+ , --+ '
V resultant =V airplane~ V wind _F_!g. ~-1_4 __ l

www.puucho.com
Anurag Mishra Mechanics 1 with www.puucho.com

DESCRIPTION OF MOTION - ' . ""ii]


' ....
~-: ----·--· ·,,_
1
p (X, y).'
v,!L'•
Qi v,
'i

-->
A
v, t
t0.
I".. • -
Fig. 1.20
• t ., ,_v, •
A vector sum is independent of the order in which the \lL+·'v;
·c;i
vectors are added, Le., it obeys the associative law (Fig.
1.21).
it.,
? •
.,. J(
~ rU<.
,..'il" +'
. ,.// rca:
I C, ...'! ._ ! .•
1 )( 'o ....
'1
r"
)'~ i--
•. ·
.. ··:' --+
i ~•••• ••• : 'B Fig. 1.22 (b) ·
A
Fig. 1.21 Component expresses effective value of a
vector in_ a particular direction. Consider a ball
--+
(A+B)+C
--+ --+ --+
= A+(B+C)
--+ --+
moving with velocity in v y
. north-east direction [Fig.
Component of Vector 1.22 (b)]. After a certain P (x, y)
Resolution of vector means separating a vector into sum time interval the ball will
--+ be at position P relative
of two or more vectors. We can write A as the sum of two ;;-
--+ --t --+ --+ --+ --+ --+ to the origin [Fig. 1.22 II
>,
vectors Ax andAy:A=Ax+Ay,whereAx andAy are (c)]. It displacement
0
projections of A on x- and y-axis. [Fig. 1.22 (a)] along east (x-axis) takes 7~::====;;r-+x
X = Yx I
place with x-component
y 1 of velocity whereas
I Fig.1.22 (c)

l "' i ' .
C •" ~"" """".
displacement along
north takes place with
-~A, A : y-component of velocity.
.'.<('!..!" 8
:: Figure shows a block being pulled by a force on a
. +--A,--. X horizontal table
A.,_ =Aco~e i" y
_____ Fig.1.22_(a) __ · . _i 4

Ax =Acose; Ay =Asin0
Magnitude of A in terms of components is given by --+
--+ ' F
IAl=~A;+A;
Direction of vector A in terms of its components is given
f/':..___...........
;rr·····--+
Fy X

by . F,
:•' Fig. 1.22 (d)
'
Fx = x-component of force vector
Fy = y-component of force vector
When block is displaced it travels in plane its position
can be described by x and y-coordinates. Cause of motion
www.puucho.com
Anurag Mishra Mechanics 1 with www.puucho.com

r,-·-··
,24 --------- _-- ----------~ ______________________ ~--·------ -- , - --- MECHANICS-I
• along x-axis is x-component of force, similarly cause of
motion along y,axis is y-component of force. N
illustration 4. A boat -- -- - --- - N-W
N-E
is tied to river bank with
ropes as shown in figure.
Force exerted by ropes on 45°
37°
boat is shown in figure. What
.... ---->
IF21• 30N [
w E
ate x and y-components of F1
....
and F2 vectors? Fig. 1.23 (a)
.... S-W S-E
Force F1 can be written as
.... .... .... s
F1 = F1x+ F1y .- (a)
.....
[F1xl= F1 cos45° ....F1 '
:
N

= 40cos45° ': 45" -) :


-----4'~~±.!:Fi,: 30° North
=2WZN of east
....
[F1y[ = F1 sin45° 30°
w E
= 40sin45° 60°
(b)
=2WZN y

.....' ' ....


.... 60° South of
F2, west s Shore line
[F2xl= F2 cos37° X
.... 37°
/
(b) (c)
= 30cos37° F2,
.... Fig. 1.25
=24N F2
.... (c)
[F2y[= F2 sin37° v .L = component perpendicular to shore
Fig.1.23· __ j
= 30sin37° v 11 = component parallel to shore
= 18N v .L = v sin 60° = l0sin 60° = 15-J2 m/s
Note that y-component of F2 is negative. v 11 =vcos60°= 10cos60°= Sm/s
illustration 5. A box of mass m is placed on a UNIT VECTOR
smooth frictionless incline. i.,yeiglit of an object acts in ....
vertical direction. Consider x and y-axis parallel and Consider a vector A, shown in figur':
perpendicular to incline. What are x and y-components of A vector has magnitude and direction. If we divide
weight W of box?
....

!~::·· J.i(,' .
vector A by its magnitude, we are left with a
quantity that has unit magnitude and its
direction is same as that of vector A, this left
.... yFig. 1.26
over quantity is a unit vector represented by
A
:~ ______ c,
Wease WWsin8(r· I1 ....
AA

_ Fi!J. 1_02~
Thus A=-
...,
[A[
Wx = W sin0 = mg sin0 .... ....
WY = -W cos0 = -mg cos0 Also vector A can be expressed in terms of unit vector A
* Weight of object= mass x acceleration due to gravity as
.... .... A

=mg A=[A[A
illustration 6. A man rows a boat with a speed 10 ....
m/s along N-E direction. The shore line is 15° south of east. Suppose a force of magnitude 10 N acts in direction of A
. What are components of the velocity vector along and
.... ....
vector, this force F can be expressed as F = (lOA)N.
perpendicular to shore?
www.puucho.com
.,
Anurag Mishra Mechanics 1 with www.puucho.com

DESCRIPTION OF MOTION 25
Concept: 1. Unit vector is dimensionless vector it can and
_, -
ay = ayj
be multiplied to any quantity without changing it dimensions.
2. Sole purpose of unit vectors is to represent a direction in Thus
_, - -
a=axi+ayj ... (2)
space.
Also Iitxl= acosa.
A unit vector is a dimensionless vector with unit
magnitude. It is a mathematical device to convert a scalar layl= asina Fig. 1.29 (b)
quantity into a vector. They are used to specify a particular Finally we have
direction in space. In Cartesian coordinate system there are --+ '; • ':
three unit vectors l ],
k whose directions are parallel to a= acosa1+ asmaJ ... (3)
coordinate axes x-, y- and z-axes respectively (Fig. 1.27). We can express vector aby any equation (1), (2) or (3).
z _,
z ax = a cos a= x-component of vector a
or
__....... A,k ,•' component of vector along x-axis
A
k
1'-~--,.:" . f _,
~
ay = a sma = y-component o vector a
A
k A
y
or
A
A
j J component of vector along y-axis.
-; y
A
A,i ...../ Concept: Unit vector in direction ofa
A _,
X j
X -
a=- a
Fig. 1.27 lal
acosai+ asina]
_, = a
We can define any vector A through a combination of
unit vectors
-
a= cosai + sinaj -
_, -
A =Axi+Ayj+A,k
-
Similarly (i)
vector A,i has magnitude Ax and direction positive
b = -b cosai + b sin a]
x-axis. Similarly, we can define Ay] and A, k.
.6 = - cosai + sin a]
Expressing a Vector in Unit Vector Notation

' f /4,l
Figure shows a vector in x-y plane.
_i_:_:.;
- Fig.1.28 Fig, 1.30

a
Redraw vector with its tail at origin. lz_.,
, Fig, 1.29 (a)_
(ii)
--+ '; ':
C=-Cxl-CyJ

= -ccosai- csinaJ
C=-cosai-sina;j
y
Concept: A vector can be displaced parallel to itself,,
anywhere in space without changing its value. It is referred as·
translation of vector.
_, _, _,
a= ax+ay ... (1) .
www.puucho.com
Fig.1:31 _
We can write
Anurag Mishra Mechanics 1 with www.puucho.com

26
.... . d = (cos45°)i-(sin45°)j
(iii) d = dx i- dy]
= dcosai-dsina] i-]
=
d=cosai-sinaj
J'2
N

+......... .
~· s
: · __ Fl~,
,
1~~
d
SE,
· '\
Fig. 1.32
Illustration 8. A boat is moving in
illustration 7. Write unit vector in direction of N-E, direction -4i + 3] with ,a speed of 10 : ' y._···:
~.
N-W, S-W andS-E. rn/s. Write velocity vector of boat in unit 3
/.' ....... •. )···.
I -4j ', c x_'l
vector notation. ,

:.
I I
, .' . I
~:]
Direction of motion of boat is along
unit vector Fig. 1,3l !
--·- ---" o--- -.•-·'-'
-> -4i+3j
' , l V=-,,====
.
l
. ''. .J42 +32
s ' ' 4, 3 •
Fig, 1,13 j =--1+-j
5 5
a= (cos45°) i +(sin45° )j Thus velocity vector of. boat is

= Fz
i+]
~ = (lOm/s)(-ii+~J)
b = -(cos45° )i + (sin45° )j = (-8i+6])rn/s
-i+j illustration 9. A hill is inclined at 0 with horizontal.
= Fz Write a unit vector in direction parallel and perpendicular to
hill, in standard x-y-coordinate system.

:~ .,]
Let the required vectors be a and b respectively.
. .. ----·-- .......... _ ..... ''}
flop~J'\' ..· I
.
r· . . :··---~Lx!
'
s.
_Fig: 1:Jj,_
I . !
c = -(cos45° )i- (sin45° )j ) '' -~-t=-c_:~~-~f~-~1n_~j !
- -i-j ! 0. ' " . '!

,*;
- Fz ir ·
1
W. E· ' , ~ · ti='sin81+cos8j
1.

45° 8 . .
'. C i.
·sw Fig."1.38
s
Fig. 1:35
a= - cos0i + sine]
b = sin8i+cosej

www.puucho.com
Anurag Mishra Mechanics 1 with www.puucho.com

DESCRIPTION OF MOTION
-~ ·---·- ·--------·----
27
.

....
Concept: Vector A is indc;·endent of choice of cuordinatc .1:r - l:2 _ 1!1
axes, although components ofvecrordepend on the choice ofco· - final position vector - initial position vector
ordinate axes (Fig. 1.39).
At time t 1 , the particle is at point P1 , and 'its position
vector can be expressed as rl = X1 i + )' 1J. Similarly at time
.... . .
t 2 position vector is r2 = x i + y j and displacement of
2 2
.... . .
particle Li r = (x 2 - x 1 ) i + lv 2 - y 1 )j.
Average Velocity
Speed is a scalar quantity which describes rate of
motion, but velocity is a vector that gives the direction of
motion as well as rate.
A body's average velocity is defined as its displacement
Fig. 1.39 divided by time M. Velocity vector is parallel to the
....
displacement S.

y
Path of particle
t + t>t

Magnitude of a vector is independent of the choice of,'


' :t>y 1~
coordinate axes, hence it is a scalar; whereas vector
1component depends on the arbitrary choice of a coordinate!
y ---.--------
,Ix
Displace~ent
-. --. -.-,: L . . . . .J"'
v,
axis; it cannot be a scalar; we call it simply a vector: -1----------'--X
component. (x + t>x)
X

1. If a vector is zero then all its components are Fig.1.41


individually zero
Velocity components vxor vyequal to the change in
A,i+Ayj +A,k. = Q
corresponding coordinate Lix or Liy divided by Lit
then A X =A y =A Z =0 Lix Liy
V =- V =-
x At' y At
2. If two vectors are equal, then their components along
the rectangular axes are also equal i.e., if Position Vector of a Point with Given Coordinates
A,i + Ayj + Azk. "B,i + Byj + B,k. Consider a point' P' with Cartesian coordinates (x,y, z)
relative to the origin 'O' then the position vector of' P' is
(Ax -Bx )i+ (Ay -By)j + (A. +B. )k = 0 given by
Ax -Bx= 0, Ay -By= 0, Az -B, =0 _, . .
OP = r = xi+ yj + zk
• l (x, y, z)
y p
POSITION VECTOR
The position vector _,
Y Path of a particle r
of a particle is a vector
drawn from origin to the
position of particle (Fig.
1.40). For a particle at
the point P(x, y) position "'--------------x
Flg.1.42
vector is
.... . . _,
r=xi+yj If vector OP makes angle o:, pand y respectively with x, y
0
The displacement and z coordinate axes then components of vector are
Fig. 1.40
vector · Li 1 is the rx = r cos ex
difference in position vector. ry = rcosp
www.puucho.com rz = rcosy
Anurag Mishra Mechanics 1 with www.puucho.com

[38________ ---- -- _ ·.:.,---'------ . - - - - - - ' - - - - ' - - - - - - - - · : - '-·_,_MECH~IC~:U


Angles a, p and y are referred _as direction cosines. thus cos 2 a+ cos 2 P+ cos 2 y = 1
The direction cosines of are, r Where ·cosa, cos!}, cosy are known as direction cosines,
'X y Z these are the cosines of the angles chat the vector makes
cosa =-,cos!}= - and cosy=-, where, with the ·x, y and z-axes respectively.
r r r
How to Obtain a Unit Vector in a Given Direction ?
r =-1-r!"" ~ x 2 + y~ + z 2 position vector of A w.r.t. B defined
--> --> -->
If we are given two points in space, we want to define a
as, rA/B = rA- rB mrit vector along a· line which begins at the point
'A'(x1 ,y 1 ,z 1 ) and passes through the point'B'(i2,y 2,z 2).
Position vector of point A(x,y,z) with w.r.t.
First we find position vector of point ':B' w.r.t. point' A'.
B(x 2 ,y 2 ,z 2 ) is given by,
Position vector of A is given by
, 1A/B = (X1 - X2)i(y1 + Y2)J+ (z1 -z2)lc --> • • ,._
rA =x1i+yd+Z1K'
. Rectangular Resolution of a Vector in Three Similarly, position vector .ofB "is
Dimensions ·
. --> rB = X2i + Y 2J + Z 2lc
Suppose the vector r is to be resolved into three
,--+ --+ --+
inutually perpendicular component vectors along the rB/A = rB-rA
directions of x-axis, y-axis and z-axis.
In accordance with polygon law of addition of vectors, rB/A = (X2 -X1)i+(y2 -y1)J+Cz2-z1)lc
Le., r = rxi+_ryj +rzlc Now unit vector in the direction of tl:iis position vector is
Where rx, ry and rz are the magnitudes of components given by,
!)long x, y and z-axes respectively. ·
By the geometry of Fig. 1.43,
• rBJA.
rB/A = - - =
ri!B/A I
2 -,=======.====~~==~=
(X2·-X1)i+(y2-Y1)J+Cz2-Z1)lc
~(X2 -
2 2
i1)
X1) + (y2 - ,Y1) + Cz2 .-
r = l,2 + ,2 + 1'.2
.Y X y z
Sinlilarly unit vector in the opposite direction of this
·.1 position vector is given by,
-->
--+ rB/A --+
r A/B = ----::;-- = - rB/ A
lrB/A I

r
Note that vector A/B is opposite to vector 1B/A.
Shortest Distance Between Two Points
If the rectangular Cartesian coordinates of two poims' A'
and' B' with position vector rA/0 and rB/O relative to the

X
' . origin 'O' be (x1 ,y 1 ,z1 ) and (x2 ,y 2 ,z 2) respectively, then
---+ --+ --+ --+
Flg.1.43 · C •• '
AB= rB/A = rB;o- rA/o
if a, p and y are the angles which r makes the direction Where
of l Jand 1c- respectively then
and
rx = r cosa, ·ry = r cos!}, rz = r cosy
__, -->
r r Therefore AB= rB/A
So chat cosa = ..l = x
.r ~r2 + r2 + r.-2
X y z = (X2 - X1) i + (y 2 -
Zt) le y 1) j + (z 2 -
. r .
· sinrilarly cosp = Y · Therefore, the shortest distance between the points
J 2 2
' -vrx +ry +rz
2 (x1 ,y1 ,z1 ) and (x 2 ,y 2 ,z 2 ) is
1'. .
IABI = ~r(_X_2---X-1-)2~+-(y_2___Y_1_)2~+-(z_2___Z1_)_2
and- COS"(= z

"Jfr2
X
+ r2
y
+ r.2
Z

www.puucho.com
Anurag Mishra Mechanics 1 with www.puucho.com

i DESCRIPTION OF MOTION 29
or P+Qcos0=0 (·:tan90°=oo) ... (i)
Also 2
R =P +Q 2 +2PQcos0
2
... (ii)
------
! . -- - --- ---- - -- ---
• j
given P +Q =18 ... (iii)
'A bird ,moves with velociry 20 m/ s in a direction making an;
:angle of 60° with the eastern line and 60° with verticalj or Q=(l8-P)
lupward~present the velocity vector in rectan!(Ular form. __ _j From equation(i),
Solution: Let eastern line be taken as x-axis, northern Qcos0 =-P
as y-axis and vertical upward as z-axis. Let the velocity v Substituting Q =(18 - P) and Q cos0 =P in equation
makes angle ex, pand y with x, y and z-axes respectively, then (ii), we get
=
ex= 60°, y 60°. We have cos 2 a+ cos 2 ~ + cos 2 y = 1. p 2 + (18-P) 2 + 2P(-P) =144
or cos 2 60°+ cos 2 ~ + cos 2 60° =1 or P 2 + (324+ P 2 - 36P).:. 2P 2 =144
1 Which gives P=5N
or cos~= .fi. Q= 18-P = 13N
and
-> A A ,.
Method-II: It is clear from· the figure that the
· v = vcosexi+vcos~j +vcosyK -> -> ->

=20[.! i +-2... j +
2 .fi. 2
.!k] resµltant of P and R is equal to Q.
p2.+R2 =Q2
or Q2-P2 =R2
=l0i+1o.fi.J+10k
=122 =144
LEx:a!ltn'""'te
~ --·---:S::I:!::.
-127......._
--..:::i~ or (Q +P)(Q-P) =144 ... (i)
---------- ----- ---·· -- -----
,-- given P+Q =18 ... (ii)
;Two vectors, both equal in magnitude, hav.e their resultant!! 18(Q-P)=144
!equal in magnitude of the either vector. Find the angle or Q-P= 8 ... (iii)
'./!etw_g_en the vectors. ·--·-- _ _ __._! Now from equation (ii) and (iii), we get
Solution: Let 0 is the angle between the vectors P=5N
A 2 =A 2 +A 2 + 2AAcos0 Q=l3N
1
which gives cos0 =- -
2
r e-.·-
f:: ,=2$1!eJI$\~
""'
g"'-4~i>.,_
-
i
:;;;;-;:;r-;:,-_- - - ,____J

or 0 =120° --·- ·---------·-···-----· --------·--·-1


1 IA student starts from his physics classroom considered to bel
= ~.:..,:.~-1:'!:~M~ 3
LExa~""-'e ],> ·
!~ !origin, walks 20 m down the corridor, then stops, turns
r- -.s- ----------------·· ---1 ,around and walks 5.0 m back towards the classroom. He
!The sum of the. magnitudes of two forces acting at a point isl stops 15.0 mfrom the door. Tota1time of motion is 25.0 sec.
i 18N and the magnitude of their resultant is 12N. If thel What is his average speed and average velociry?
:resultant makes an angle of 90° with the force of smallerj
bzm1itucle, what are the mwmit!!..d§..9f.l&.lWO ft,mc.,cc,,,escc?_ __,
Solution: Method-I:
Let P < Q and 0 is the angle between them.
-~- 5~-:~: ::?a:-2~:-~
tan 900 = Qsin0
'---- -----' --- Fl~--1_E_.4____ _,_I _______ j I

P+Qcose Solution: According to definition of average velocity,


----------,
••••••••••••••••••••• I
/!,x Xf - X,
Vav =-=
t.t t1 -t,
Q i't \\···-......Q I . - 15 ·0 - O.O 0.600 m/s
25.0-0.0
total distance travelled

l._~_~.:,., >· I
Average spee d = ---.- - . - - - -
tune mterval
= 20.0+5.0 l.00 m/s
25.0

www.puucho.com
Anurag Mishra Mechanics 1 with www.puucho.com

r-- --- MECHANICS-I I


,30
- ---- - -- - --~--- ----------
-+ -+ --) -) -+
or C=-(A+B)=-A-B
-+ ,. " It. -+ " " "
..., ..., ..., Consider two vectors A =2i -3 j + 5 k and B=-1-2 j + 7k .
Consider three vectors A, Band C as shown in Fig. lE.5. -+ -+-+-+
We have to find a vector C such that A+ B + C = 0. Clearly
:Perform graphically the following vector additions and, -+ -+ -+ " ,. I\
' """?-+ -+-+-+ """7-+ C=-(A+ B)which is -i + 5j-12k.
,subtractions: (a) A+B, (b) A+B+<:; (c) A-B, (d)
!-+ -+ -+
'A+B-C.
- - - 1

l{i --),._,._,...

'Vector A= 3i+5j -2k and vector B


-+ -+
,vector- C such that 2 A + 7 B + 4 C = 0.

Solution: Let
-+ -+
-)

= -3j+ 6k
A.A

Find a,_

• I

and
Flg.1E.5

..., ..., ...,


-+ -+ --+ -+ -+ -+ The x-, y- and z-components of vector 2A+ 7 B+ 4C are
Solution: In case (d) A+B-C=(A+B)+(-C),
..., respectively
we have to reverse the direction of vector C and add it to the x-component = 2Ax + 7Bx + 4C x
-+ -+ -+ -+
resultant of A and B, Le., A + B . y-component = 2Ay + 7By + 4C Y
z-component = 2A. + 7B. + 4C.
~ ..., ..., ...,

to
+
tai
+
t<
l'c,

~'
"~·; ~-·: ......
.......
Vector 2A + 7 B + 4C is zero if each of its components is
zero,
i.e.,
and
2Ax+7Bx+4Cx=0,
2Az + 7B. + 4C, = 0
2Ay+7By+4Cy=0

Thus we have three independent equations to determine


A-B
Cx, Cy and c •..
(a) (b) (c)
On substituting numerical values, we have
2x3+7x(0)+4Cx =0 or Cx =-1.5
2 x 5 + 7 x (-3) + 4C y = 0 or Cy = 275
2 x (-2) + 7 x (6) + 4Cz = 0 or c. = - 9.5
-+ .A A A

1'0/ Thus vector C = - l.5i + 2.75j - 9.5k.


i fCJ
tci, l~~c;im~~~~- 177>-
+
T..;
J
1\vo unit vectors i and are directed along x-axis and y-axis'
respectively. What is the magnitude and direction of the
(d)
vectors i_+ J and i-) ?
Fig.1E.5
d y
Remark:-------------------
In the figure shown, -C =A+ B
............ h
j h
i
: .
X ·······~~··

PR i .
goo
a
~-'-:--,-+--x
h
-j <.,~ ,j
I

Flg.1E.7
(a) (b)
Solution: From parallelogram law of vector addition,
www.puucho.com
Anurag Mishra Mechanics 1 with www.puucho.com

DESCRIPTION,OF MOTION
·---~~----------------
(b) The magnitude of average velocity vector is
1i + j I,;, J1 i 12 + I j 12 + 2I111 j I cos 90'
I 'if av I =Jc2.50) 2 + (2.50) 2
\i\=IJl~l
=3.54 m/s
where \i+J\=-J2wrlt Average speed is measured by the length of the path
travelled
1 .
tan a =- ' -\j\sin0
=-'------ --- (21t X 25.0)
\i \+ \ J\cos 0 Average speed =_,4_ _ __
. 10.0
= 1-sin90' =l
= 3.93 m/s
1 + 1 · cos 90°
VECTOR MULTIPLICATION
or a= 45°
Similarly i-J=i+(-j)
1. The components of a vector are 1
scalar quantities. The multiplication of
Components i
l-) - ~ - "
the unit vectors, by the cartesian ,A=:A,_:i +:A,:j +:~:kl
We have· to reverse the vector j and add it tp

Ii -11 =J1 i \ 2 2
i.
+ I j \ + 21 11111 cos 90°
components leads to a vector sum of
three mutually perpendicular vectors.
1
1 \ __ .
,_ _'29· 1.44 (a)___J
I v~~:rs .'.'.JI
In the figure shown the
=J1 2
+ l 2 + 0 =-J2 writ -> --- - -- --- ---- --,
---- YA
vector A is the vector sum I
of the vectors j .I
tan a= IJ I sin (-90°) = _1
AxtAy],A,k. ... /~ /"~,(~
I 1I+ I j I cos C-90°) The multiplication of a
~--------~------- ' -Azk :
->
:, :-)
, A :• :A".t
, yl
or a= -45°
vector A by a scalar a : : : A :

changes all the cartesian : : : AI : xi


--------·---------------- ----------~
;A girl is jogging along a circular path of radius. 25. 0 m. In 1
components ' by the same
factor:
1)·:::::·:......... l.-·· :
.':!!I.:. 1.,_4~J'!L____ j
-> • •
110.0 second she jogs a. quarter ofa circle starting from point1 a A= a(Axi + Ayj + A,k) : ~- _
P. (a) Compute her displacement and average velocity, and =aA)+aAyj +aA,k
(b) Compute the magnirude of the runner's average velocity
and her <_1verage speed. 2. Vectors in cartesian form may be added or subtracted
-------
y
1
provided they are of same type, representing same kind of
physical quantities such as displacement add to
'
'"""'-- displacements, forces add to forces. For example
F+ 1 =(F)+Fyj +F,k) + (fxi+ fyj + f,k)
= (Fx + fx)i+(Fy + fy)i+(F, + f,)k
-> ->
I
1
ri
Fig.1E.8
P
xJ You can see ):hat the components of vector sum (F+ f)
are the scalar sums of the respective components of the
'--------..... -~- individual vectors. Similarly,
Solution: (a) Her initial and final position vectors
are
F- f =(F)+Fyj +F,k)-(Jxf+ fyj + f,k)
--+ ,...--+ ,...
ri =(25.0) i ; r1 =(25.0) j =(Fx - fx)i+ (Fy - fy)j +(F, - f,)k
-> ->
The displacement is given by Hence the components of the vectors F- f are the
-> -> ->
~r=r1 -ri scalar differences of the respective scalar component of the
individual vectors.
=(25.0)j - (25.0) i 3. In vector addition and subtraction the vectors
The average velocity is involved. must be of the same type, While in vector
_, C-25.0i + 25.oj l multiplication there are two distinct ways; each of these
Vav = -----~- ways has its own set of rules notations and applications in
10.0 physics. ·
=(-2.50! + 2.50j) \11/s
www.puucho.com
Anurag Mishra Mechanics 1 with www.puucho.com

l32 MECHANICS:!'.] .
THE SCALAR PRODUCT OF TWO VECTORS (iv) When the angle 0 between vectors is acute
Multiply a vector by a scalar produces another vector. (0 < 90°' cos0 > 0) the scalar product is positive; if the angle
The scalar product is different from this multiplication. The is obtuse (0 > 90°, cos0 < O) the scalar product is negative.
scalar product is a way to multiply two vectors to yield a The scalar product of two vectors that are perpendicular
,.. -+ -+
scalar result. .(orthogonal) is zero.
..., Scalar
..., product of any two A and B vectors is
written as A · B and defin~d as
..., ...,
A·B=ABcos0
..., ...,
where A and B are magnitudes of the vectors A and B,
and 0 is the angle between them when they are drawn tail to
t.ail.
(i) The angle between
two vectors always is taken to be
90s 8 > O;·so scalar cos 0,< O;,SO scalar"
the smaller angle between the prOdvqti~," Po~itiVe , ,. eroduct is negative
vectors when they are ·drawn from })'\,,, J.,f

L
a common point. In Fig. 1.45 shown
..., ...,
angle between vector A and B is 0.
With this convention 0 is always less
than or equal
_ to 180°. f'lg, 1.45
';"s 90° ~ O; So scalar product' is
Az~~ , - , • I
(ii) In order to geometrically interpret the scalar
..., ..., Flg.1.47 ,. __J
product we draw A and.B drawn with their tails together.We
. . -+ -+ (v) The scalar product of a cartesian unit vector
drop a perpendicular from the dp of B to line containing A. with itself is unity. For example
...,
The quantity B cos0 is called projection of B or component of
..., ..., i- i = (1) (l)cosO = 1
B on i:he line containing A. Imagine light shining
..., ..., also i- j = (1) (1) cos 90° = O._
perpendicular to A then the shadow of vector B on the line So we have
..., ...,
containing A has length equal to the projection of B or i-i= 1 i-J=0. i-k=O
..., ...,
component of B on line of A. j-i= 0 J-1=1 J-k=O
k·i=O k-1=0 k•k=l
'. Light ,
perpendicular,"-
-.,. The li~e
cont~~~1ngA.
-,
_,
to'A , , ' - • , .·. f .... (vi) We can find magnitude
_..., of any vector A by
taking the scalar product of A with itself
· ' af'·-'... /2'. -Th~projec~n Acr:Je ~-;;;,
~
~ ofBalongAis
·<B cos 8 B cos ·, •
~~ -
8
,t ·v, ,"
: .
: The hne ~
A·A = (Axf+Ayj+A:k)·(~xf+A_,j+A,k)
= AxAx (i-i) +AxAy(l · 1) +AxA, (i· k)
(Note that,B cos a : containing A +AyAx cj · i) + AyAy cj · 1) + AyA, cj · k)
can be·< 0 if-8 > 90~}
,_
- -i-A,Ax (k -iJ + A,Ay (k- j) + A,A, (k · k)
Flg.1.46
---~.~-- or A2 = A2 +A2y +A2
X Z
We can also take projection the other way around.
..., ..., A= '\J1A X2 +Ay2 +AZ2
A· B = A (B cos0) = (A cos0)B
-+ -+ -+ -+ (vii) When two vectors are expressed in cartesian form
A·B=B-A the scalar product becomes
(iii) The scalar product is commutative. -+ -) A • ',. A A

A-B = (Axi+Ayj +A,k):(Bxi+Byj +B,k)


The scalar product is also distributive, Le.,
-+-+ ---t -+-+ -+-+ = AxBx + AyBy + A,B,
A, (B1 + B 2 r= A· B1 + A: B2 (viii) Angle between two vectors can be calculated
with the help of dot product:

www.puucho.com o,
Anurag Mishra Mechanics 1 with www.puucho.com

rDESCRIPTION OF r~OTION - ...


---+---+
A·B
cos0=--
AB
k1!,~~~)~l~:19f~
---+---+
Where + AyBy + A,B,
A· B = AxBx
~!:!~h~i;jr_onent of -:i': 2i + 3] along·~;~; ~irectio~:j
and A= ~A; +A; +A;, B = ~BX2 +By2 +B z2
(ix) Work done : ---+ • •
---+---+
Solution: Let b = (i + j)
W = F- s =Fscos0 ---+ ---+
The component of a along b
(x) Angle between the vectors:
---+---+
We have,A- B = AB cos0 acos0b=[-;~b},
---+---+
A-B
cose = - -
AB (2i + 3J) · (i + Jl (i + Jl
A 1 B1 + A 2B2 + A3B3 = -'-~====,,....=-'· ,=;;===;;=
=,=;;===§===§c-'i=;;====§===;;= )1 2 + 12 . )1 2 + 12
~Af +A~ +A~~Bf +B~ +B~ 2x1+3xl (i+Jl
=--~-
(xi) Component or projection of one vector . ./2. -/2
along other vector : 5 • •
i----· --------+··-------·- = -(i+ j)
2
! B
/ L1;:¥~,~:t~~
I--+_ • • ---+ • •
:If A= 3i + 4j and B = 7i +24j, find a vector having the
i ~ " ~
~me mamftude as B and paralle!Jo A - - - - - - ~
Solution: The required vector is= BA
;: B=)7 2 +24 2 =25
'L Fig.1.48
·-------· - ------ ---+ • •
---+ ---+ and A=A= 3i+4j
(a) component of vector A along vector B A ) 32 + 42
A cos0B = AB case B
B = .!.(3i+ 4J)
5

-[A·BJ.
- -- B
B
• 1 • •
BA= 25 x-(3i+ 4j)
5

.... ---+
= 15i+20J
(b) Component of vector B along vector A
r----·- s .----- .. -;;·------·------ .
!Under a force (10i - 3 j + 6 k) newton a. body of mass 5 kg!
/moves from position (6i +5]-3k) m to position!
• • I
j(IO i - 2j + 7 k)m Dedu_c_e the work don!__ ___ 1

IO LiJ_cos e ~ ....A Solution: As displacement


---+ ---+
s = r 2 - r1
---+

C Flg.1.49_ _ __
i.e., s = (lOi- 2j +7k)- (6i + -j- 3k)
= (4i-7J + 10k) m
Bcos0A = ABcose A
A So w= i.1 = c101-3j + 6kl-c41-7J + 101ci
---+
i.e., W = (40+21+ 60)
=[A~iJA = 121joule
www.puucho.com
Anurag Mishra Mechanics 1 with www.puucho.com

g· _es "h-.. -,-


.;l'·~
--·-1.~ - -l··~

.
f'F~am,,i!.ia 12fo-·
r- , ' , .---· " .
~ parti~ie tli~vd.in th~ ·x-~~la~~'µha~i ~he action of &force ~
'T,'~-"'l'.~"1,..--::t

(A point P)ie-s i!l rl1e x-y plan~. Its po~itiq/l ca~:be ip~iifi,,ed cy
i··_ ,,.,· ,!/'\;·,_.','. -._,,~·,::,:·_",·i_· ~. ,, .. litf -·~· y ~tgoi;,dinates. orby 1;·i11,HaJ1y direc1,i~/~~ptok
such th~qlie value of its lin~~r,n;t_~1~iitum p at any time tis I~ (xf + y J),maFirig an an~l-~. 0 with;th,e_jcccms. F/~1:~,teff~r
~ ~- - . ' -~ . .
",_ ·~-" . -~
p';,:",_ '2.cost and.Py = 2 ~int. Wha£is the angle 0 between F
. . '/ . '
,ir · of unit rr/agqitu!fe in the qirectl9n of} vector _r and·'! 'vector
7
l~~i; at:~ given ~~ t? ·· ' . ,
I··
;x,yp)ane.' __ · · __
.•
·
~ '
lie of unit !71Cig7Jitude normal to _the ,vectori;a.nd)yi~_(h'tli~
' "
.. . . • ·.··· . • .

..., ..., ...,


Solution: As p=ipx+JPy Solution: By definition rY
. ..., ..., ,--+ A A

= i (2cost) + j (2sint) ir =.!.= xi+_yj


r r
IPI= ~[(2cost) 2 + (2sint) 2 ,;, 2 Or, i, .= i~os0+ }sine.
' --+
I, \

Now as,.
.-'>
_. F=-·-
dp x = rcos0, y = tsinB
dt ' }low; Let_ le= la+ jp,.' ·
,..., d •. . .. • H,

F·= dt[i(2cost)'+j(2sint)] where a and. p are coefficients Flg.1E.14 ', '

So,
..., .
F = i(-2sillt) +j (2cost)
. to be determined. ·· ·,
Using the definition of scalar product; we have ·
with IFI= ~[(-2sint) 2 + (2cost) 2 = 2
i,.i 0 = (icos0 + j siri0)(ia + JP)
--+ --+ - • Or, a=-Psin0
Now as ·· F'.p = FxPx +FyPy . cos0
Since 10 is a vector of unit magnitude :
.! . : ~~::~[t·;]c:::~-~2[co~t)](:s:::~1= 0 a2+P2=1
0 0
.. Fp- 2x2 Therefore, p = + cos0 and a= -siri0·
• ( , 'I' • • •
' -+ . --+ .
Thus, le = -lsin8 + j cos0
i.e., 0 .= 90° i.e., F and p are orthogonal, We have taken p = + cos0 and not the other solution,
lli .. "~~:,::;;;r:-:-1,--,,.., p = - cos0 because we define a system i,, i0 vectors in the
~€!fjl~J\~J~ 13 . ~ directions of increasing r and 0.

b~~:tijlfl'fl~~ 15 ~
[v1-·. two , ;~~-;e;.~ .vectors A' :;:;~-:~
-t -t· -+ (. .
obey

~olution: Here v = Let 0 ·be the angle 31 + 2j + 3k. [A+B = A:-B, the angle,betw~enthem_is :.
1(g}_l2Q."...:._.,_·_(b}_90° .· . (f).-60° • '(cJ)~0~0- - ~
between the line and the velocity-of the particle. Then
.. 3-2+3 4 , -t -t· -t -t
~
. cos0 = -T22 -./3 ./66. Solution: Given (A+B) =; (~-B)
Taking self product of both sides
. Compon~nt of velocity along this line=lvlcos0 -t-t -t-t -t-t -t-t
.. •. .- ' · .. · 4 4 (A+B) · (A+B) =(A-B)·(A-B)
.-:.,, '., ="22-=-. -t -+ -+ -t '-+ -t -t -t -t -+ -t -t -t ~ -t -t
·.. .-. ·c. ' . '·.·. -./3 . -If' orA-A+A·B+B·A+B·B=A·A-B-A-A-B +B·B
... (1)
Vector component =c./3 d is the unit·vector along the
-t-t-t-)-t-t -t-t•.
2
line, : '.,. · · ' We know that A·B= B-A; A·A =A and B·B= B 2
., ' ' "i-}+k· Substituting _these values in equation no. (1)
~ut : .-0..,;=. -~ . . -t-t -t-t C·
,, /i . A 2 +2A·B+B 2 ':'A 2 -2A·B+B 2 .,
-+
Vect~r comp~nent
. ' ,
=. j3 (f - 'j + ~) . -:·:'•-i or
-t -t
4A·B=0 or
-t
4IAIIBlcos0=0
--~l · . ..., ..., '

" Because A and B·are non-zero vectors hence.-,


www.puucho.com ' ·~;;~'
1:r
Anurag Mishra Mechanics 1 with www.puucho.com

.... ....
(iii) If~ is the angle between vector (2i + 3]) and (I+ j)
IA!;,, 0 and !Bl¢ 0
then by the dot product
cos0 = 0 = cos90°
0 = 90°
ry·
Angle between two. vectors is 90° hence alternative (b)
is correct. ~ A A
(i + i)

,. ..... --·---------- .. - ---"·-·:7


Jand Jare unit. vectors along x,axi;; and y-axi;; respectfve(y._l A A

IWhat is the magnitude and direction of the vector i + j and: iY (i - D


IfI - ]? What are ·_th_e. magnitudes
- of c.omponents of a vectoj
• I
,_ !:!~: _1E.16Jc)_. j
:; = 2f + 3] along the --directio~ of i + j and i :- ]?
........
---- --------. --------~J: a-b = abcos0.
r;·---- . _,_,
Solution: Use the relation; IY (2i + 3])- (i + ]J =I (2i + 3])/1 (i+ ]JI cos~
.... .... ~-~---- 'h
la+hi=.Ja 2 +b 2 +2abcos0 .i I(2i + 3])/ cosp I(i + ]JI =(2i + 3J) · (i + j)
bsin0 2i-i+ 2i-]+3J-i+ 3j. j
and a= tan· 1 I (2i + 3j) Icosp
a+bcos0 li+]I
(i) Angle between i and j 2+Sj.i+3 5 .
is 90° Fig. 1 E.16 (a) --=--- = - umts
../z ../z
Since these.are unit vectors therefore /i/=IJ I= 1 ....
Magnitude of component of a along the direction of
2 2
Ji+ Ji=~/ i./ +1]1 +2i.l. ~os90° (i+ J) is
=.J1 2 +1 2 +0 . / (2i + 3J) Icos(90°+~) = (Zi + 3 J) ·(i- j)
Ii+ Ji= ../z units . ../z
. -1 Iii sin0 1. sin 90° _ 2i-i+ 3J-i-2i-J-3j. j
a=tan •• - ../z
lil+Ulcos0 l+l.cos90°
=-1-=l- acos(90°+1i)'= -~
l+O . v2
tana = 1 =}a= 45° THE VECTOR PRODUCT OF. TWO VECTORS
Ciil .-. Ii- j I= J~Ii-12-+~I
J-12 -+2-1,-.Il~Jl-co-s-90-0 (i) The vector product or cross product of two
vectors yields another vector. The vector product of two
=.Ji 2 + 12 + O =../z units ~ ~ ~ ~

vectors A and B is written symbolically as Ax B. The


, IJI sin(-90°)
tana =-.~~.---- magnitude of the vector product is defined to be
Iii +Ul cos(-900) .... .... ....
-l-sin90° -1 ICl=IAx Bl= ABsin0
tan a.' - - - - =
'l+lcos90° l+O ' ' . where 0 is the smaller of the angles between the two·
....
r = -1 = tan(-45°) vectors, The direction of vector C .is defined to be
.... ....
perpendicular both A and B. Keep the two vectors until the
tails of the two vectors coincide. The two vectors then define

7
a plane as shown in Fig. 1.50. The direction of the vector
A
j product is perpendicular to this plane,
....
(ii) The direction of ve~tor C can be determined by
a vector product right hand rule. Curl fingers of your
~- : = 90° { right hand and imagine swinging them from ·the directilln of
-J~~
y' ,_-J
-J
!1 first vector in the vector product to the direction of the
second vector as shown in Fig. 1.50. The extended thumb of
_ _ Ftg.1E;1~(b) ___ J your right then indicates the appropriate direction of the
vector product. · /
www.puucho.com I ., -
Anurag Mishra Mechanics 1 with www.puucho.com

r-'·
1·. 1i~·,._;:._:·-=-~---~-~--"'--'-,,-~-~-"--'-------'-'.h
r~----.. - ~ --+ --+ A

Ax B = (Axi+Ayj +A,k) x (Bxi+Byj+B,k)


A A A '

'!
' = (AyB, -A.By)l+(A.Bx -AxBz)J
I
)· "l
+(AxBy -AyBx)k
In determinant form we can write it as

Ii -, -, -i
AxB= Ax
j k
Ay !1.
l Bx By B,
: -. ,· :· ____ ____ , Flg.1.~0 ___ _ j
-,
(iii) The angle between any vector A and itself is 0',
hence the magnitude of the vector product with itself is zero.
-, _, ---+· :... _,.,·' --+ .,,., ,,. --+ ::--::7
.....
, IAxAI= Msin0'= 0 Given a =i+2j +3k, b;= -i +.2j tk and c = 3i + j.
Find the argle of t.,_;ultant :with ~:aii,s: Also find a unitv_'iicto~
rhe vector product of any vector with itself is zero.
in the direction of the resultant of these vectors. Also.find a
(iv) The magnitude of the vector product is the· area -, -,
of the parallelogram formed by two vectors, also, the un_it vector_ r w_hicli is nonnal to both a and I>. What is thej-
4 --+ ~
magnitude of the vector product is twice the area of the [nc!ingtio[l_oi_r and c? - . ·· ' - , ·
tri~ngle formed by connecting the tips of the vectors. _, _, -,
(v) If the vector product of the two vectors is zero Solution: Resultant vector= a+ b + c
and neither vector is of zero magnitude then the sine of = 1+2] + 3k:-l+2j +k+ 31+ j
angle between the two vectors must be zero i.e., the two
·vecto;s_ ~re·either parallel or antiparallel. · = 31 + sj +4k . _,
--+
Unit vector"tl. by the property is given by = ~
,' 1 . --+
If AxB=0 _, '

_, IAI
and and - B ~ 0, then
-, -,
Unit vector of resultant 31 + sj + 4k
Ax B = ABsin9 =·o ft= 3l+sj+4k
, implies sin9 = 0 J3i+ sj + 4kl
Thus 9 is either 0' or iS0'. 3i + sj + 4k (3i + sj + 4k)
--+ --+ --+
Although Ax-Band Bx A have.the same magnitude in
--+ • = =
~(3)2 + (5)2 + (4)2 s./2
accordance with right- hand· rule their directions are _, -,
Vector perpendicular _to vector a and bis cross product
opposite . .- ,- · .:
• '. . --+ --+ --+ ,--+. _, _, -, .
r = axb = (i+ 2j + 3K) x (-i+2j +K)
. ,._ . . ,._
i.e~, · AxB=-BxA
. .. - ' -
The order of the terms in a vector product is important. = '-ix i-2j x i-3kx i.+ ix 2j + 2j x 2f
(vii) The vector product obeys distributive law as long +3kx 2j +ix k+ 2jxk+ 3kxk
as the order of the terms is preserved.
--+ --+ --+ --+ --+ ';' --+ , = 0+2k-3] + 2k+ 0-61+-j + 2i+ 0
Ax(B,+ B,) = Ax B,+AXB2 = '-4l-41+4k = --4(i+ j-k)
(viii) ix i = 0, as vect_or product of any vector with --+ --+ --+ --+
itself is zer-o. · r-c=lrllclcos9
jixJI= Clf (l) sin 90'= 1 -4(i + j- k) · (3i + J) = l--4Ci + J-klll (3i + JJI cos9
From right·hand rule the direction of vector product is -4(i- 3i+ j · 3i-k- 3i+ i- j + j ·J-k· j)
perpendicular to both i and J, i.e., parallel to k. Similarly
= ~42[1 2 + 1 2 + (-1) 2] ~(3) 2 + 12 cos9
'ixi=O ix'J=k ixk=-J
=} -4(3 + 0-0+ 0+ 1- 0) = 4,,13.JTh cos9
Jxi=-k JxJ=O ]xk=i ' -4
cos9=--
kxi=j kx]=-i kxk=O ../3o
(ix) The vector product of two vectors expressed in 1
e~cos- (~)
cartesian form ,as '
www.puucho.com
Anurag Mishra Mechanics 1 with www.puucho.com

[~sc111PJl~N o(r.ijT101(__ -_-___________


~>i~mi:ili Gal> X

,- - .
,If~= 2i - 3] + 6k and b = 6i + 3] - 2k, find the angle, Xo -••••••••••••••••••
: --+ --+
,between vector a and b. Also find unit vector perpendicular to
-> ->
poth a _and b.
Solution:
->-> lo
(i) a- b =ab case =a, b, + a2b2 + a3b3 Fig.1.51
where
The distance x can be measured by taking snapshots of
the moving particle at definite moments with a fixed
cemera.
=~(2) 2 + (-3) 2 + (6) 2 and b =~(6) 2 + (3) 2 + (-2) 2
a=7 and b =7 Concept: 1. Note that in this manner we obtain the
coordinate x of the point on the given moments but not the
7.7 case= 2.6+ (-3)(3) + 6(-2)
distance travelled.
or 49cose =-9
2. The distance travelled can be found from the
-1 -9
or e=cos - coordinate x only in the case when the particle moves in one
49 direction.
-> ->
(ii) We know ax b represents a vector which is In the graph of x-t show the coordinate of point cannot
-> -> be greater than x 0 although after time t 0 the distance s(t)
perpendicular to both a and b.
travelled by the point exceeds x 0 while the coordinate x
i j k becomes less than x 0 •
-> ->
Now axb= 2 -3 6 =-12i+ 40] + 24k
-2 Concept: If a car moves constant(y in one direction the
6 3
distance traveled ls equal to the coordinate describing the
-> -> r--------- motion but when the direction of motion changes to the
1 ax bl= ~(-12) 2 + (40) 2 + (24) 2 = 4v'145 oppo.site direction the distance travelled still in creases while
-> -> the coordinate decreases.
. • axb
Unit vector n : : : - --
_, ->
Velocity of a Particle in Rectilinear Motion
lax bl
The velocity of a point is a physical quantity
• -12i+ 40] + 24k -3i+ 10] + 6k determining rate of change of the coordinate with time. The
n=---=~--
4v'145 "'145 magnitude of the average velocity is equal to the ratio of the
distance travelled by the point to the time taken. If particle is
RECTILINEAR MOTION OF A PARTICLE at x 1 at time t 1 and at a point x 2 at time t 2 then its average
When a particle is in a rectilinear motion it moves along velocity is
a straight line, its distance from a fixed point on the line
increases or decreases with time. In such cases we associate
a reference frame with that _straight line and consider fixed
Concepts: 1. Average velocity depends on the time
reference point as origin.
interval for which it ls computed.
In order to completely determine the law of motion of ' average velocity ls the same
2. If in a given motion, the
particle the coordinate x of the point with respect to origin for any time interval the motion has the constant velocity
and as function of time must be known. and ls said to be uniform.
To plot the graph of the dependence of the coordinate x 3. In case of uniform motion which starts from the origin
on time t we choose a certain length scale and put values of there ls no difference between the value of the coordinate and
the coordinate x on the axis of ordinates and time t on axis :that of the path travelled.
of abscissas. 4. If a particle travels unequal distances in equal time
intervals its motion ls seid to be non-uniform. In a
non-uniform motion the average ve_locit.)r ls no longer a
www.puucho.com
Anurag Mishra Mechanics 1 with www.puucho.com

~[3::_:s=··==·=·=:0='>·::::·:~::;1~::L$::>:'.:;====:=:'·::7!7::;·,.1::::1_;::·-~;:::===::::::::::~::;:::;:;{li(\r,~:·=~--'~~r·;t""';-~~~~-·~-~·---"--·-::=~·-:,;~:._~}> I
5
-/ :~~·.' 'ME($~1,CS-:I'
constantqii~nt.ityanddependso~ti~eintervalforwhit:li tt-;;J v(t) = Lim v· = Lim t,x = dx(t)
• d . ;~ . . l . ' M--+ 0 av. M--+ 0 --6.t . dt
co.mpu..te : F,·._P___ r,_a. non-um,_,orm. mo.ti.on av_e.r.age ve oc1ty·c-·annot1·
describe. the:V9riations of motion of.the body. Such '.a IJ!(?tion i.e., the instantaneous velocity is the derivative w.r.t.
can be described adequately by' instantaneous velocity. . .. time of the position function.
~--~-,-- - -~= .... - ... - - ~ - - - - ··---~ +In the graph of xversus t the slope at each point (at
1. Average velocity: Fig. 1.52 (a) shows the
positions of a car at times t 1 and t 2 • each instant of time) is equal to the value of
instantaneous velocity v at that point. Note that slope
Average velocity is defined as the ratio of displacement at t is zero, i.e., instantaneous velocity is .zero.
t,x to time interval .M. 2

(a)
.j
F>O~itlQn_ yersu~_ume,:_g;:apti,.
:.: ...'........ ~:.' _, ... 7i:~>~2

+Fig.1.52 (b) depicts average velocity graphically. Join


initial point P1 and final point P2 by a straight line.
Slope of this li;,e is t,x. Hence, the average velocity is
. t,,~ .
the slope of the straight line connecting points (t 1 , x 1 )
and (t 2 , x 2 ).
2. Instantaneous velocity: If we decrease time +The speed of an object is the magnitude of its
interval M, for very small time interval, the line P1P2 will be velocity.
tangent to the curve at point t 1 • The slope of this line is _,
defined as the instantaneous velocity at time t 1 • d r · ..
Insta;,taneous velocity is velocity at a single instant of.time. Speed= I_V I= I dt I
Mathematically, it is defined as Since sp~ed is the magnitude of a vector, it is .a scalar
I ... X • • -~~ :,c-,.-;~-. quantity that is never negative.

r''·'·

i A_tri
,£1 ·...
·S

,,i~ . . '. •
..

'
.•

:
R2
3. Acceleration·: Acceleration of an object signifies
how rapidly the object's velocity is changing, both in
magnitude and direction, whether the object is speeding up
or slowing down. · ·
lt. ..:.'·.:jJ V2 - V1 ,iv
Average acceleration =--"-~
J''" ·.. t 2 - t1 M
'.
I[,_. ;', .. I• . a
Instantaneous acce1eration = L;~ ,iv =-dv
·~~·~·F~ig~1.5~.:_~~
llll -

t..,-,q M dt

www.puucho.com
Anurag Mishra Mechanics 1 with www.puucho.com

--as;;i
+ In velocity versus time
graph (Fig. 1.55), the slope
of line P1P2 gives average I
I
acceleration and the slope
of tangent at point P gives I ds2
instantaneous acceleration. l ds
+ In accelerated motion v aJ?,,d L,'____F1~g._1_.s_1_ _ __,
a are in the same direction.
Similarly for distance Jds =total distance
·.,!1 Jas= as,+ as 2 + dss ;t-, .. +as.
Fig.1.55 => total path length or distance
From Fig. 1.58. Displacement vector can be expressed as
4. Equations describing motion with constant _, _,
acceleration: vector sum of d x and d y vectors
_, _, _,
· v(t) =v 0 + at ... (1) ds=dx+dy
2
x(t) = x 0 + v~t + (1/2) at ... (2)
v
2
=v~ + 2a(x - x0) ... (3)
x =x 0 + (l/2)(v 0 +v)t ... (4)
v(t)--, velocity at time t
v O --, velocity at time t =0 (initial velocity)
Fig.' 1.1!8
x(t) --, position at time t
x 0 --; position at time t =0
a ~ acceleration
as =Id-; I= -Jcdx) 2 + Cdy) 2
+Thus displacement in time interval tis x(t) - x 0 • J-Jcdx) 2 + (dy) 2 =total distance or total path~length
+For constant acceleration, the velocity varies linearly
with time hence _the average velocity is the mean Graphical Representation of Motion in One Direction
value of-the initial and final velocities If maximum power of xis 1 and maximum power of y is
Vav. = (1/2)(v 0 + v)
1 graph is straight line. ·
This relation is valid only for constant acceleration • y=mx+c-
motion.
Total Displacement and To~I Distance
mx
y-=:mx-c
X
$~, X
Ex, :iz····,;,v.
Consider a particle that moves along path represented
,,

' .
.
. ',
'
V

by arrows. Entire path can be divided in very small [,

displacement segments. . _...___ ___


L----------'--~Flg.:1'.59· i ·;· "'-""'.._.- .. .,,~~-~-
~·l.:.n.• -~ -,1 -~~~

--+- •. If maximum power of xis 2 and 'maximum power of y is

'~~~·1,; :
Jd s =net displacement 1 cir vice-versa graph is parabola.

-,..;
Path of particle. ""'. -
,,..--,._-..'::::,.:_~~- si
~:--i ()/.. r) ...
dS3 - i x ~ x " •'; -~, ,/'' -··_ '·_'''x
I
_, x2 =4ay y2 =4ax y2 =-4ax -
0
x2 = -4ay
i~S1

I .' Flg:1.56
-
-- -----' Flg:1.60
:
If maximum power of x and y is 2 graph may be circle,
'... . - .
·--~------~-'---'
L----~-'-'- - - - ~ - ~ -
----> ellipse, two straight line ~tc.
vector AB represents net displacement s-tcurve
--+ --+ --+ ·--+ --+ --+
sR = Jds=ds1 +ds 2 +ds 3 + ... +dsri If we puts on y-axis and t on x-axis for every value oft
we have a value of s.

www.puucho.com
Anurag Mishra Mechanics 1 with www.puucho.com

(b) Uniform acceleration:


We have a particle moving
with uniform acceleration a
and initial velocity u. Its
displacement s at any time t
can be represented as
1 2
s =ut+-at
1. The average velocity from time t 1 to t 2 will be 2
v av = s2 - s, = slope of ll~e joining p 1 and' P2 Curve is parabola.
' . t2 -t, Velocity at t 1 is tan 8.
For a particle moving along a straight line when We
plot a graph of s versm t, v av. is the slope of the
Wir-=~-.'>!G-~-,~~ii;J 19 ~
r-·· -,-.. - -., ..--- -----··· .......... - ·. ·""'" ..•. -,
straight line that connects two particular points on
the s(t) curve: one is the point that corresponds to
1kcydiststa,;t;1ngfrom a point Atravels 200 ~ due' north'to a!
!point B, at constant speed of 5 ms"'' He. rests at B for, 30i
s2 and t 2 , and the other is the point that
corresponds to s1 and t 1 • Like displacement, v av. has w
:sec?ndf q~d ~he,:, :travelsc300_ :m <ll!lc south a point~ ~~ al
1cons.tant speed p{J O ms 1 • F,1nd~qvera1<e velocity._ . , , ... , i I
both magnitude and direction (it ,is another vector
quantity). Its magnitude is the magnitude of the Solution: ...,..., ... ~---· ____" ..
7
line's.slope. A positive Vav. (and slope) tells us that;
the line slants upward to the. right; a negative v av. · .
(and slope), that the line slants downward to, the B200
•'
right. 100
2. Instan~ani,ous velocity: A
According to definition . P2 C-100., _20_ ••.• _( ... 0·1
.. t,i s ------,·--·--
$ ,,2 "' '
v= 11m-
M--)o .1t
t; r. · 'Fig.1E.11!_,__
: _ __ 1.·

In ·curve, if M ~ 0 the i s, ., ...


~1

From A toB
point p 2 comes very close ,t,"b , :· t-
x=200; v=S; t=40sec
to point p 1 .• '~---''-~;..;_\~j FromB to C
Notg;)------~---~::;::==~:;:;:; Displacement = 300 m, time taken = 30 sec
The instantaneous velocity can be found Net displacement = -100 m
by determining the .slope of the tangent to
Total time = 100 sec
the displacement time graph atthat instant.
Velocity at point p1 or time t1 is v.
ts, . · = - 100
Average ve1oczty l
,
-1
- - = - ms ..
v = tan a
100 ,
v-t curve
By using dependence of v on t we
can plot a v-t graph.
Cases: * Slope of·v-t curve at any point
(a) Uniforin velocity: represents acceleration at that
If velocity is· uniform slope of
t
. ,S .
J'
instant.
curve must remain 21-,,~~.-- tan 8 = acceleration at time t 1
unchanged. Curve with Area underv-t graph andt-axis.
uniform slope is straight line.
If velocity is 1 ms-1 As we know dx = vdt
=> s = vt => s = t and f.vdt = x = Area ~nder v-t graph.
tane = 1
~--.....-n--...~

t
1-
. " 't, l
2
j
~"~-~·-~Fig0 J_.67 _ _:___~
www.puucho.com
Anurag Mishra Mechanics 1 with www.puucho.com

[P.escRrPrioti"o(~.-o_ti_ori_·-----'----~-·-~---~~,_.::·~-------~--__ __···_··--_-_-- -·-·-- .. ·.i1J


Thus, area under curve will represent displacement Alternative:
in that time period. V = 10-2t
No\11_:~.------------------- Area (1) = .! x lOx 5 = 25
(1) Area above /-axis +ve displacement. 2
(2) Area below I-axis is -ve displacement. Area (2) = .! x 3 x 6 = 9
2
Thus, Displacement= 25- 9 = 16 m
1. Total displacement will be sum of areas with Distance = 25 + 8 = 34 m
appropriate signs. Objective:
2. Total distance will be sum of areas without sign. . 1. Using graph, distance can be calculated directly.
Cases: 2. Total displacement will be sum of areas with
(1) For uniform velocity: appropriate signs.
acceleration = 0 3. Total distance will be sum of areas without sign.
slope= 0 !!~_!I
I
i -
1
4. To plot straight line using equation of motion.
[kl..i:=2&9me;'fec-.:....~~
. . . --r::-1----
L r:·----- -. -.. . ---- --------- ·---- . - --- --
Flg.1.68 (a)tj
(2) For uniform straight line curve:
tan 0 = acceleration
rr acceleration l !ms
A particle moves .in a straight line with constant velocity of 5 '.
1
· for 2 seconds. It then moves With a constant accele~ation'
For increasing velocity:
Ivlu 8
I, 1
of-2 ms·2 for 8 seconds. Draw velocity-time graph for 10:
!seconds of motion and find. '
;I_ t-+ii
Fig. 1.68 (b)
t!l) _Ejrzql 11_ijgs:_ity_ ___(/J) Qisp_lgc_e_me_nt__ _(c)_ 1htal_dfstance '
Solution: Area (1) = 5 x 2 = 10
tan 0 = acceleration ,r· -- V- ---

For decreasing velocity:


(slope is -ve) i.e., 0 > 90°
it
lv decreasing
!
I
5
'! a I' 1 :' 2 6 8 10
Note: 0 is always with +ive '
x-axis. l 1:......: ·1 2 4 3
~ __ Fig.1.68(c) _ i

-----------·---- --- - - -- -- ------,


j-11+-----'>I
,_ _ ---- _F_i!!:..~~-2_1__ --------'
·A particle is travelling in a straight line. It has a. initial/
;velocity of 10 ms-1 • When it is subjected to an acceleration of: Area (2) = .!_x 5 x 2.5 = 6.25
2
-2 ms-2 for 8 seconds. Find displacement and distance]
:tn:1Ver~<il1J {l_s~fol!!fs~-- ___ _ _ _ __ ____ : Area (3) = _ _! x (11) x 5.5 = -30.25
2
Solution: s = 10 x s-.! x 2 x s x s Displacement= -14m
--- 2--_-·p-:--7 Distance = 46.25 m
I
V r - - - - · . . - - - -- _,:, _________ - - -- - - --·
I10 L._ -~AlCULUS SUPPLEMENTARY
Differentiation
Derivative of a Constant Function :
~(c) =0
1-6 dx
I
Fig.1E~D____ j The Power Rule :
If n is a positive integer, then
Displacement= 16 m
- d ( X ") ::::;nx n-1
Displacement = s1 = 10 x 5 - .! x 2 x 25 = 25 m dx
2 The Power Rule (General Version) :
Now u = 0; a= 2; t = 3 If n is any real number, then
1
s2 =--x2x9=-9 -d ( X ") ::::;nx n-1
2 www.puucho.com dx
Anurag Mishra Mechanics 1 with www.puucho.com

Solution: (a) since f(x) = X-2 , we use the power ("3,-5)


rule with n =-2 :
) _- -dX
f '( X ( -2) -
_ -Xz -2-1 •.• _ Flg.fE.23 _ .. ··-
dx .
Thus dy/dx= 0 if x = 0 or x 2 - 3 = 0, that is, x = ±-Ji
= -zx-3 = _2 So the given curve has horizontal tangents when x =0, ../3,
x3
and-.J3. The corresponding points are (0,4), (../3,- 5) and
(b) dy = ~',/[x2 = ~ (x2/3) (-../3,-5). (~ee Fig. lE.23)
dx dx dx
The Product Rule :
=-x2 (2/3)-1 ==-x
2 -1/3
3 · 3 If f and g are both differentiable, then
The Constant Multiple Rule ! [f(x)g(x)] = g (x) ! [f(x)] + f(x) ! [g(x)J
If c is a constant andf is a differentiable function, then
d d 1 E•Yd"'? · I e '! 24 t>,c..
dx [cf(x)] = c dx f(x) l'.s:+-~3.lf:iiiJ!.t':.\~~-:.~
d 4 d· 4 3 3
e.g., (a) -(3x ) = 3-(x ) = 3(4x ) = l2x
dx dx
Solution: By the product rule, we have
(bl ~(-x) = ~[(-l)x] = (-l)~(x) = -1(1) = -1 d . d d
dx dx dx f (x) = -(xex) = x-(ex) + ex -(x)
dx dx dx
The Sum Rule :
= xex + ex. l = (x + l)ex
If f and g are both differentiable, then
. d d d
-[f(x) + g(x)] =-· f(x) +-g(x) l&~~~lru\BJ~~l25j~
dx
The Difference Rule :
·dx

If f and g are both differentiable, then


dx
'Differentiate the function f(t) 7 ./t(l-t). ·_ ·. ·_ .•/ .:7
Solution: Using the product rule, we have
![f(x)-g(x)]= !t(x)- !g(x) r:: d d
f (t) ="'' -(1-t) + (1-t)--./t
dt dt
e.g., ~
dx
4x 4 + 10x3 - 6x + 5)
(x 8 -12x 5 -
. . . =-./t(-1) + (1- t) .!r-112
2
= ~(x 8 )-12~(x 5 )-4~(x 4 ) + 10~x 3 - 6~(x) =-.Jt + 1-t = 1-3t
dx dx dx dx dx
+ ! (5)
2.Jt 2.Jt
We can also proceed directly without using the product
rule.
=Bx -12(5x ) - 4(4x 3) + 10(3x 2 ) - 6(1)- 0
7 4
f(t) = .Jt -t.Jt = t!/2 -t.3/2
=Bx 7 - 60x 4 -16x 3 + 30x 2 - 6
f (t) =.!t-1/2 - ~t!/2
~~~·p~~ 2 2

!Find thep~~nts on the curve i. = x 4 - 6x 2 + 4 wher;-thei


'la~nt.line_js horizontaL ·. •.•· ......J --·······-·-·---·-. -----··-----···]
':". .ftg(x) where g(4) =>2 and g' ( 4) =3,find']' (4}'.
i1J f(x)
Solution: Horizontal tangents occur where the
·derivative is zero. We have Solution: Applying the product rule, we get. .
dy =~(x;)-6~(x 2 )+~(4) f (x) = ~[.ftg(x)] = .ft~[g(x)] + g(x)~[.ft]
dx dx dx dx dx dx dx
www.puucho.com
Anurag Mishra Mechanics 1 with www.puucho.com

. = ./xg' (x) +g(x).! x- 112


' 2
.:; ; 1
= -fig' (x) + g(~ :Differentiare-~X = x 2 sinx - - - - :..:::_~ ~ !
2'1X
' ' (4) . 2 Solution: Using the product rule we have
f(4l = "4g'(4J+L = 2-3 + - = 6.s
· .. 2"4 2-2 dy_ = x 2 .!._ (sin x) + sinx.!._ (x 2)
dx dx ' dx·. '
The Quotient Rule: 2
= x cosx+ 2xsinx ',,
If f and g are differentiable, then
. d d
.!._. [f(x)] = g(xla;:[f(x)]- f(xla;:[g(x)]
dx g(x) , ·[g(x)]2 !Differentiate y ='tan x
·2 ~ . ~' ' '
e.g., Let y= X +x~2. '
.·, ·I' .
S oIut10n: u. sinx.
smg tan x = - - and quonent
. 1 we
rue,
x 3 +6 ·· ·, cosx1
:' have
then
3 d '2 . 2 d 3: ~(tanx) = j__(sinx)
(x +6)-(x +x-2)-(x +x-2)-(x +6) dx dx cosx '
'
y= . dx dx
(xs + 6)2 d,(" )··. d()
cosx- s1nx - s1nx- cosx
= _ __,,dx=-·_ _ _ _.,,dx,,___
(x +6)(2x+ll-:-(x 2 +x-2)(3x 2)
3
= 'cos 2 x
(xs + 6)2 cosx. cosx-s'inx(-sinx)
(2x 4 + x 3 + 12x + 6)- (3x4 + 3x 3 6x 2) =
- coS 2 x
=
cos x+sin 2 x
2
1 2
= 2
=--=sec
2
x
cos x · ,. cos · x
d 2
dx (tan x) = sec x "
Derivatives of Trigonometric Functions

'd ,-' d'- ',•,


!Find an . ~qll~dpn 'oA th~ ta'ns-~nt" Une to the ·curve! -(smx) = cosx ~(~ose<:x) = 7 cose~x,cotx
dx i di, . ., ~ , '.r ,-' -~-:,~~· '
IY "'ex /(I+,:X~~a(the·poiht '(1,e/2),, · '
. "
---·--'l
I
d .. .
-;·
,.,d . - ' ' ·. : '
Solution: According to the quotient rule, we have dx (cos~) = -sin x ··-(sec·xJ a:a secxtanx
'dx - ·,.
(l+x 2 )~(ex)-ex .!._(l+x 2)
• '• C
', ·-~ -. ,; '

d ' ·, . ·- ' : d'' ' .· .


dy dx dx - (ta11x) a= sec2 x · ,~{cotx) = -cosec2 x
dx= (l+x2)2 dx ·: ' " ,:dx ' '
, _ (l+x 2 )ex -ex(2x) _ ex(l-x) 2 The Chain Rule :
- - (l+x2)2 - (l+x2)2 If y = j(u) and u = g(x) are both differentiable
so the slope of the tangent line at (1, e/2) is functions, then
dy dy du
dyldx -ox=I
-=--
dx du dx

This means that the tangent line at (1, e/ 2) is horizontal


and its equation is y = e / 2
b~~~.Jii 30 ~ '
Differentiation Formulas
.,'' -
. [;nd F (x) ~{1,'(x) = E¾1) ::·:·~~-;- ?"'T\, :.i]
I
d' ' .'
-(c)=O
dx ' ' dx
d
/"
'
- (x") =·n,x':',, ·..
"
~(e"J=e'.·
dx "'
II Solution: If we let u = x 2 +land y =·.Ju, then
< )• .
I (cf]'= cf.' (j+g)'=f'4:g'-·
.,
(f-g)'e. f"-g' l F (x) = dy du = -2_ (2x)
'\
du dx 2./u
(Jg)'= h'fgf,

·>-
(f)' = gf:'C.:.~'
• -., ·-2- .•
g'. '
.
\ '" '

· . 1
= ~==(2x) = ~==
X

.. ' '
2.Jx 2 +l .Jx2 +1
www.puucho.com
Anurag Mishra Mechanics 1 with www.puucho.com

:44 -~- ~- :.~ -_~_______MECHANICS-lj


Concept: In using the chain rule start from the outside Solution: Firstrewritef:f(x) = (x 2 + x+ 1)-113 • Thus
to the inside. In chain rule we differentiate the outer function·
f [at the innerfunction g (x)] and then we multiply by the, f (x) = _.!(x 2 + x+ 1)-413 ~(x 2 + x+ l)
3 dx
.derivative of the inner function. i
2
1. If y = sin(x ), then the outer function is the sine = _ _!(x 2
+ X + l)-413(2x+ l)
3
function and the inner function is squaring function. So the
chain rule gives. ;-- . .. ·. . . r:::::l -:-
L ,l;;,>fJ::!.!\'.QJ21.~ , 33 I_'"'.>
~
dy = ~ sin
t,._ - ~--'""·-----. -· ,,
(x 2 ) = cos (x 2 ) • 2x
dx dx '---,---' '---,---' ' - - - v - - ' '-v---' ·Find the derivative of the function
outer evalilated derivative evaluated derivative
function at inner of outer at inner at inner 9
function function function function t-2
g(t)= ( -
= 2xcosx 2 i 2t + 1 )
.. ,'
Similarly, _note that sin 2 x = (sin x) 2
Solution: Combining the _power rule, chain rule and
dy d
~--~
2
- = - (sinx) = 2 (sin x) . cosx quotient rule, we get
dx d x ~ - - ~ ...._,_., '------,--'
outer
function
derivative
of outer
function
evaluated
at inner
function
derivative
inner
function g'(t) = 9(;t-}lr ! (;t-}lJ
= 2sinxcosx =
9( t - 2 J (2t + 1)1- 2(t - 2) = 45 (t - 2)
8 8

In general, if y = sin u, where u is a differentiable 2t + 1 (2t + 1) 2 (2t + 1) 10


function of x, then, by the chain rule,
-dy = -dy- du
- = cosu-
du LE~,~~Rl~ 1341;>
dx du dx. dx
Differentiate y = e'inx.
Thus d ( ' )
- smu = cosu- du
dx dx Solution: Here the inner function is g(x) = sinx and
2. Ify = [g(x)]", then we can writey = g(x) = u" where the outer function is the exponential function f(x) = ex. So,
u = g(x). By using the chain rule and then the power rule, we· by the chain rule.
,get dy d . . d .
- =-(esmx) = esmx -(sinx) = esmx cosx
dy dy du n-1 du [ ( )] n-1 , ( ) dx dx dx
dx = du dx = nu dx = n g x g x
Note:·-------------------
3. If 11 is any real number and u = g(x) is differentiable, i We can use the chain rule to differentiate an exponential
then
function with any base a > o
-d (u ") =nu n-1 -du 8 x =(elnay =e(lna)x
dx dx
the chain rule gives
r:.,E-~fl-~Rf~ _: ~1 )> !!.... (a')=!!_ (el lna)x) =e(lna)x !!__(Ina )x
dx dx dx
:Differentiate y =_ (x~ -1) 100 . =e(fna)x_ In a =ax In a

Solution: Taking u = g(x) = x 3 -1 and n = 100, because In a is a constant. So we have the formula
we have !!....(a')=a' Ina
dx
dy = ~ (x3 -1)100 =100(x3 -1)99 ~ (x3 - 1)
dx dx dx
= 1OO(x 3 -1) 99 3x 2 = 300x 2 (x 3 -1) 99
RECTILINEAR MOTION
lJ;:~~~-~gJ~: G'~.L> (MOTION ALONG A LINE)
We will assume that a point representing some object is
1
Find f (x) if f(x) = allowed to move in either direction along a coordinate line.
, ~x 2 +x+l This is called rectilinear motion. The coordinate line might
be an x-axis, a y-axis or an axis that is inclined at some
www.puucho.com
Anurag Mishra Mechanics 1 with www.puucho.com

LD~S~RIPTION OF MOTl~N__ 45

angle. We will denote the coordinate line as the s-axis. We For example, in figure the rabbit is moving in the positive
will assume that units are chosen for measuring distance direction between times t = 0 and t = 4 and is moving in the
negative direction between times t = 4 and t = 7.
and time and that we begin observing the particle at time t
2. There is a distinction between the terms speed and
=O. As the particle moves along the s-axis, its coordinate is velocity. Speed describes how fast an object is moving without
some function of the elapsed time t, says= s(t). We calls (t) regard to direction, whereas velocity describes how fast it is
the position function of the particle, and we call the graph of moving and in what direction. Mathematically, we define the
instantaneous speed of a particle to be the absolute value of its
s versus t the position versus time curve.
instantaneous velocity; that is,
instantaneous ] =[v (t) [= Idsl
[ speed at time t dt
Particle
is on the For example, if nvo particles on the same coordinate line
positive side have velocities v = 5 m/s and v = -5 m/s, respectively, then
of the ori in
Particles is on th the particles are moving in opposite directions but they both
egative side o have a speed of [v [= 5 m/s.
the origin

Flg.1.69 L E;:x~_t:r\p}~ L.~5_i:-->


Let s(t) = t 3 - 6t 2 be the position function of a particle
Fig. 1.69 shows a position versus time curve for a moving along an s-axis, where s is in meters and t is in
particle in rectilinear motion. We can tell from the graph seconds. Find the instantaneous acceleration a (t) and show
that the coordinate of the particle at time t = - s0 and we the graph of acceleration versus time.
can tell from the sign of s when the particle is on the Solution: The instantaneous velocity of the particle is
negative or the positive side of the origin as it moves along v (t) = 3t 2 -12t, so the instantaneous acceleration is
the coordinate line. dv
a(t) = - = 6t -12
dt
INSTANTANEOUS VELOCITY a
The instantaneous velocity of a particle at any time can 40

be interpreted as the slope of the position versus time curve


of the particle at that time. The slope of this curve is also
6 8
given by the derivative of the position function for the
particle.
Acceleration versus time
, Concept: 1. The sign of the velocity tells us which way -40
the particle is moving a positive velocity means thats position Fig. 1E,35
of particle w.r.t. origin is increasing with time, so the particle
·is moving in the positive direction ; a negative velocity means and the acceleration versus time curve is the line shown
that s is increasing with time, so the particle is moving in the in Fig.lE.35. Note that in this example the acceleration has
positive direction; a negative velocity means that s is units of m/ s 2 , since v is in meters per second (m/s) and time
decreasing with time, so the particle is moving in the negative is in seconds (s).
direction (Fig. 1.70).
Concepts: 1. A particle in rectilinear motion is
speeding up when its instantaneous speed is increasing and is
slowing down when its instantaneous speed is decreasing. An
object that is speeding up is said to be "accelerating" and an
s(t) s(t) object that is slowing down is said to be "decelerating", thus,
s(t) increasing s(t) decreasing
v(t) = s' (t) "° 0 v(t) = s' (t) < O one might expect that a particle in rectilinear motion will be
speeding up when its instantaneous acceleration is positive
(a) (b) and slowing down when it is negative.
Fig.1,70
2. This is true for a particle moving in the positive
direction and it is not true for a particle moving in the

www.puucho.com
Anurag Mishra Mechanics 1 with www.puucho.com

f45 ..· ..... ··- ··-· - : MECIIANics:,-1


L--------· ~ ·-····-· -- ----, -·- - - ...
,- . .
- ~---------··~-,--
- -· .. ·-·-. .
·,,_)
---- ··-········--· ·1
;negative direction-a particle with negative velocity is speeding Concepts regarding position versus time: curve;
•up when its acceleration is negative and slowing down When:
The position versus time curve contains all of the j
its acceleration is positive. i
' ' ·significant information about t/le position and 'velocity of a 1
i " This is be.cause a positive acceleration implies an: 'particle in rectilinear motion. I
'increasing velocity and increasing a negative velocity:
idecreases its .absolute value : similarly, a negativei
: !· If s(t) > 0, the particle is on the positive side qfthei
;s-axzs. _ _ _ 1
;acceleration implies a decreasing velocity and decreasing a_l · 2. Ifs (t) < Q the particle is on the negative side of thei
;negative velocity increases its absolute value. ' s-axis · ·· ;
3. Interpreting the sign ·of acceleration A: 3. The slope of the curve at any time is equal to the 1
'particle in rectilinear motion is speeding up when its velocity :instantaneous velocity at that, time. · . /·
;and acceleration have the same sign and slowing down when I 4. Where the curve has positive slope, the velocity is 1
,they have opposite ,signs. · , :positive and the particle is moving in the positive direction. j
, 4. From the velocity versus time curve and the! l 5. Where the curve has negative slope, the velocity is
!acceleration versus time curve for a particle with position I 1 negative and the particle is moving in the negative direction. 11
functions s(t)=t 2 -6t 2• I I 6. Where the slope of the curve is zeta, the velocity is ;zero, I
. '
• Over the time interval O < t < 2 the velocity and' :and the particle is momentarily .stopped. · '
!acceleration are. negative, so the panicle is speeding up. This· : 7. Information about the acceleration of a particle in 1
iis consistent with the speed versus time. curve, since the· speed! :rectilinear motion can also be d~duced from the position!
1
is increasing over this time interval. Over the time interval! 'versus time curve by examining its concavity. Observe that ilie
i2 <t < 4 the 'velocity is negative a:nd the acceleration isl :position versus time curve will be concave up on· intervctl.s
•positive, so the particle is slowing down. This is also consistent: ;where s' (t) > Q and it will be concave down on intervals
;with spied versu,s time. curve, since'.the, speed is decreasing overl 1where s" (t) < U But we know from ( 4) that s' Ct) is the
!this time interval. Finally, on the time interval t > 4 the! 1instanta11eous acceleration, so that on intervals where "the
!velocity and acceleration are positiVe, so the particle isl ,position versus time curve is concave up the particle has a,
:speeding up, which again is constant with the speed versus ,positive acceleration, and on inte_rvals 'where it is. concave'
ltime curve. .
--·--·---·---·-·-----·~-
.
- --
·'
-·----·--·-·-·-·-·---- -~---·-·-· _._,d _ _,
!down the particle has_ a negative. a~celeration. -- , __ · _• ' J
Summarizes Our Observations About the Position versus Time Curve

i Position versus time curve Characteristics of the curve at t =! 0 = t, I

kl
_______ , M,!
Behaviour ofthe particle at time t '
*s(t) >0 * Particle is on the positive side of the origin.
* Curve has positive slope * Particle is moving in the positive direction.
I ' . * Curve is concave down * Velocity is decreasing.
lo . ,
I
-- ,,--- - ti. * Particle is slowing down.

lli!
,

*s(t 0 )>0 * Particle is on the positive side ol the origin.


* Curve has negative slope * Particle is moving in the negative direction.
* Curve is concave down * Velocity is decreasing.
. .. !o __ i * Particle is speeding up

·~·
. --~ - ~ ,'
*s(t 0 ) < 0 * Particle is on the negative side of the origin.
. ' * Curve has negative slope * Particle is moving in the negative direction.
• _· ' I
'
l tI
I * Curve is concave up * Velocity is increasing.
I ' '
* Particle is slowing down.

6;
I '
,_ " ·- ··- . I
- .. , .
*s(t 0 ) > 0 * Particle is on the positive side of the origin.
* Curve has zero slope * Particle is moving stopped.
! I
! lI
* Curve is concave down * Velocity is decreasing.
, ... : -_to: -~

www.puucho.com
Anurag Mishra Mechanics 1 with www.puucho.com

DESCRIPTIONOF tAOT!ON

Conceptual Examples: Suppose that the position described schematically by the curved line in Fig.I. 71 (c). At
function of a particle moving on a coordinate line is given by time t = 0 the particle is at s (OJ = 3 moving right with
s(t) = 2t 3 - 2lt 2 + 60t + 3. Analyze the motion of the velocity v (O) = 60, but slowing down with acceleration
particle for 1 ;;,, 0 . a (OJ = -42. The particle continues moving right until time
t = 2, when it stops at s (2J = 55, reverses direction, and
Solution: The velocity and acceleration at time t
begins to speed up with an acceleration of a (2J = -18. At
are v (t) = s' (t) = 6t 2 - 42t + 60 = 6(t - 2)(t - 5) time t = 7 I 2 the particle begins to slow down, but
a(t) = v' (t) = l2t - 42 = 12(t -7/2) continues moving left until time t = 5, when it stops at
At each instant we can determine the direction of s (SJ = 28, reverses direction again, and begins to speed up
motion from the sign if v (t) and whether the particle is with acceleration a (SJ = 18. The particle then continues
speeding up or slowing down from the signs of v aad a(t J moving right their after with increasing speed.
together Fig.1.71 (a) and (b)]. The motion of the particle is

0 2 5
O++++++O- ________ O+++++++++ s;gn of v(t) = 6(1 - 2) (t - 5)
60
.Pos_itive Negative Positive Direction of motio_n
direction direction direction 40
20

-Analysis of the partiCles direction


(a)
a
7
0 2 2 5 t
0++++++0- ________ O+++++++++ sign ofv(t) = 6(t- 2) (t- 5)
40
Positive Negative Positive
direction direction direction
20
- - - - - - - - - -O++++t++++++++++++ sing of a(l) = 12 (t- 7/2)
sloWlng speeding slowing speeding
dOwn up ·down up Change in speed 1 2 4 5 6 7

-20

Analysis of the particle's


(b)

t=0
t =5 '--
-------~'t=2
:,'
..
r___,,...1=....1,_2~

03 28 55
(c)
Fig.1.71

. ~.:,

...

www.puucho.com
Anurag Mishra Mechanics 1 with www.puucho.com

f4a -- - . -·- MECHANICS-I I


" -·-·-- ·-~·-,,~-- --·-· -'
Concept: The curved line is above Fig. 1. 71 (c) is! lf(S)- f(3) l=l 20- DI= 20m
'descriptive only. The actual part of particle is back and forth· The total distance is 4 + 4 + 20 = 28 m
'on the coordinate line. · ' r--
- ·- "--~ ·--.-' -1
1 INTEGRATION 1
Indefinite Integrals
Fundamental theorem of calculus establish connections
'The position of a particle is given by the equation. between antiderivatives and definite integrals, if f is
s = f(t) = t 3 - 6t 2 + 9t continuous, then (f(t) dt is an antiderivative off and
:Where tis measured in seconds ands in meters.
(a) Find the velocit,y at time t J:f(x)dx can be found by evaluating F (b)-J (a), where Fis
(b) What is the velocit,y after 2s ? after 4s ? an antiderivative off
'(c) When is the particle at rest ? ! J f(x) dx is traditionally used for an antiderivative off
:(d) When is the particle moving forward (that is, in thej and is called an indefinite integral. Thus.
positive direction) ?
'(e) Find the total distance traveled by the particle during the:
[
f
f(x) dx = F(x) means F' (x) = f(x)
For example, we can write
first five seco_nds _ __ __ __ __ _ __ _ _ i
Solution: (a) The velocity function is the derivative of
the position function, that is ,
2
J x dx = x; + C because !( x; + C) = x 2

s = f(t) = t 3 - 6t 2 + 9t So we can treat an indefinite integral as representing an


entire family of functions (one antiderivative for each value
v(t) = ds = 3t 2 -12t+9 of the constant C ) .
dt
A definite integral J:f(x)dx is a number, whereas an
(b) The velocity after 2 s means the instantaneous
velocity when t =2_. that is, indefinite integral Jf(x) dx is a function (or family of
functions).
v(2) = dsl = 3(2) 2 -12(2) + 9 = -3 m/s Any integral formula can be verified by differentiating
dt t=2
the function on the right side and obtaining the integrand.
The velocity after 4s is For instance
2
V (4) = 3(4) -12(4) + 9 = 9 m/S d
2
J sec xdx = tanx+C because dx (tanx+C) = sec 2 x
(c) The particle is at rest when v (t) = 0, that is,
3t 2 -12t + 9 = 3(t 2 -4t + 3) = 3(t -l)(t - 3) = 0
and this is true when t = l or t = 3. Thus, the particle is
! [F(t)] = f(t) and f f(t)dt = F (t)_ + C
at rest after 1 s and after 3 s. are equivalent statements,
(d) The particle moves in the positive direction TABLE
when v (t) > 0, that is, [)eriyative Equivalent
3t 2 - 12t + 9 = 3(t -1) (t - 3) > 0 ·-,F~nnula Integration formula
This inequality is true when both factors are positive ~[x 3 ] =3x 2
(t > 3) or when both factors are negative (t < 1). Thus, the dx
particle moves in the positive direction in the time intervals
t < l and t > 3. It moves backward (in the negative .E.+ixJ=:!r
dx. 2vx I 2"1dx=.fx+C
direction) when 1 <t<3.
(e) Because of what we learned in parts (d), we need to i_ [tan t] =sec 2 t Jsec 2
tdt =tant+C
calculate the distance traveled during the time intervals [O, dt
1], [1, 3] and [3, SJ separately. ~[u312] =i!u112
dx 2
The distance traveled in the first second is
For simplicity; the dx is sometimes absorbed into the
lf(l)- J(O) I=14- DI= 4m integrand, for example,
From t = l to t = 3 the distance traveled is Jldx can be written as Jdx
I f(3)- f(l) l=IO- 41= 4m
J_!__ can be written as J dx
From t = 3 to t = 5 the distance traveled is x2 x2

www.puucho.com
Anurag Mishra Mechanics 1 with www.puucho.com

'DESCRIPTloN oFMoriori -- --- - . ·- - 49'


[______ ·- --- --- - ....... -- - - . -..
4 y
Integration Formulas
Integration is guesswork - given the derivative f of a
J2dx = (area of rewu1gle)
I
function F, one tries to guess what the function F is. 5
x3 = 2(3) = 6 4
Jx 2 dx= +c r=2 (b) The graph of the
3 integrand is the line
3
y=2
x4 2
Jx 3 dx=-+C r=3 y = x+ 2, so the region is a
4 trapezoid whose base extends 1
-+-<~~-+-+-.. x
1 _5 x-5+! 1 2 3 4 5
J-dx= Jx dx=--+C=--+C r=-5 from x = -l to x = 2 [Fig.
x5 -5 + 1 4x 4 lE.38 (b)]. Thus, Fig. 1 E.38 (a)

!+1 y
x2 2 312 2
f Jxdx= f x 112
dx = --+C = -x +C = -(Jx)3 +C
~+l 3 3 y=x+2
2 4
1 3
r=-
2

r· Differentiation
Formula
TABLE
Integration formula
-+--+~-~~-+--•X
-2 -1 1 2 3 4
Fig. 1 E.38 (b)
1
f dx=x+C 2
J(x + 2) dx = (area of tapezoid) = -1 (1 + 4) (3) = -15
-1 2 2
d
I2.- [x'•'] =x'(r,e-1)
dx r+l
-- Jx'dx =-+C
r+l
xr+l
(r;t -1) (c) The graph of y
y = -J1- x 2 is the upper

_3. ! [sin x] ~ cos x Jcosxdx=sinx+C semicircle of radius l, centered


at the origin, so the region is
the right quarter circle
d
4. dx[-cosx]=sinx Jsinxdx=-cosx+C extending from x = 0 to x = l
(Fig. lE.38 (c)], Thus, -+----'---..x
I d 2
5. dx [tan x] =sec x _Jsec 2
xdx;:::: tan x+C
Fig, 1E.38 (c)

16. -! [- cot x] = cosec2x fcosec xdx =- cot x + C


2
JI vlC--x- ,dx (area of quarter-circle)= 1
2
-it (1 2 ) = -
1t

I 0 4 4
' 7. -[secx]
d ==·sec.x tan x Jsecx tan x dx = secx+C
; dx
i' B. -d[-cosec x] = cosec x cot x Jcosec x cot x dx-- -cosec x + C
Evaluate
' dx 2 2
(a) J(x-l)dx (b) Jcx-l)dx
0 0

I . Solution. The y
!Sketch the region whose area is represented by the definite graph of y = x - l is
' .
•integral and evaluate the integral using an appropriate shown in figure and
:fonnula form geometry. we leave it for you to y=x-1
4 2 I verify that the shaded
(a)J2dx (b) J(x+2)dx (c) J,,/1 - x 2 dx triangular regions
_1 -1 0 1
both have area - .
Solution: (a) The graph of ,Le integrand is the 2
horizontal line y = 2, so the region is a rectangle of height 2 Over the interval [0,2]
extending over the interval from 1 to 4 [Fig. lE.38 (a)]. the · net signed is
Thus,
Fig.1E.39

www.puucho.com
Anurag Mishra Mechanics 1 with www.puucho.com '

A 1 -A 2 =
1
- 1
= 0, and over the interval [0,1] the net I JeX.dx = ex +C f cixdx=_':__+c
X

2 2 I Ina
f
signed area is -A 2 = _ _!_Thus,
i '
fsi~xdx'.=,-cosx +C ·f co.s:xdx = sin.x+C
2
f (x-l)dx= 0
2

and foI (x-


.
1
l)dx = --
2
l 2
Jsec xdx=·tanx+C fc:osec 2 xdx = -cotx+C
0

Fundamental Theorem of calculus


\f secx tan xdx = secx + C f cosec x cot xdx .= -cosecx + C l
l J·-.-dx'= tan- x+C · '·.r:---z
I l dx . x+.C ,;I
!ff is continuous on [a, b] and Fis any antiderivative off l · 1 .- " · ·1 -1 .
=S!Il
on [a, b], then I
I
x +1 ., .·' •.
2
-v.1-x·
. . 'I
b .
f J(x)dx =F(b)-F(a) The most general antiderivative on a given interval is
a obtained by adding a constant to a particular antiderivative.
Thus we write
Cexctmfut~>r40·.1:";;.>
'y-
~-=-~i(',.r'~"tF:::::::;.~;:{; __
f_!_dx=-.!+c
~ 2
,- -·- - - -~ ;·. • •• ' --~- > ··--~·. .---- '
x
I(a) Find th~ a,rea undfr the CU[".e y = cos X over th~ int~rval)
- - _-~:. -:,--~--. ' .;---7 .. _
X

;[o, -rr/2] .
and
••
[O, it]•
' •-'" . • •
.• -
.. •
•. "•
. ·
" - · · - - · .••
. •• . ·I
. , l.c:E•xa:1m1r:1fl e ·J 41__ j
I. ~
-~
!b~·..='r--::;:.;;J-i,:r~--£:~;;\
y
1· -- - -- :_3. , . '
' I iEvaluate Cic 3 J - 6x}dx
I_. __ -- -o_· .. , ...
Solution: We have
I 3
J3(x 4 2]
-,1
I 3
- 6x)dx = ~ - 6~
'!
0 4 2 0
·}•

i - - - ,.,
• l
Fig.1E.40
----~ ·- ......... -·· . .,,,,,-~.
, a! (1 4
= 4.3 -3-3 2) - (14·0 4-3-02)
Solution.Ca) Since cos x;:: 0 over the interval [O, Jt/2] , = -81 - 27 - 0 + 0 = -6. 75
the area A under the curve is 4
n/2
A= Jcosxdx=[sinx]~/ 2 =sin2:-sin0=1
o 2
- -- - - ~--
f:2(,zx,' 6x + + dx
(b) The given integral can be interpreted as the signed
area between the graph of y = cos x and the interval [O, it].
i
iFind 3
"-;-
• ). · ~ '
'i
I

The graph in figure suggests that over the interval [O, it] the
{ 0 . X +l •
. "" -- ,_,
'
--·-··-· .J
L--- -~ ..... J. -~-··· --- -
portion of area above the x-axis is the same as the portion of Solution: The Fundamental theorem gives
area below the x-axis, so we conjecture that the signed area 2
2
is zero ; this implies that the value of the integral is zero.
This is confirmed by the computations. o
3
J(2x -6x+---i-)dx= 2x -6x +3tan-
X +l 4 2
4
1
x] O

·rcos xdx = sin x]" = sin 1t - sin O = 0 = .!x 4 -3x 2 + 3tan-1 x]~
2
0 0
Table of indefinite _integrals
= .!c2J 2 - 3(2) 2 + 3tan-1 2-0
f c f(x) d. cff(x) dx ., " jU(x)'+ g(x)] ,ix•, : ' 2
. _.,:: · · ·' \.aJJ(x)~'.,-Jg,(~)dx =-4+3tan-1 2
This is the exact value of the integral.
fkdx=}:x+C
!
'J
1 X "dx_ =_-·-_}-::.+
x .. ·
ri+l, ._, '
. .n:t-1J . ' .. J:-cdx = lnlx!·f:C
···cc·
. ., ·.'.1 .. ' .. CIE-x~mele.;
.;;,-::--· -= ~~~..::::.,.:· "'1-:: .;_
43
-~-:V l~
·n;t-1 -· .. ••:, X,,. .
. ., ,

www.puucho.com
Anurag Mishra Mechanics 1 with www.puucho.com

-. -----·~-
~.'-.'-'-'"';,._----"'---,.._--- ~,~-' __ . _ _ ____ _;.·,_, ~--'·':'"'""'~- ______·_s1~.,I
DESCRIPTION OF MOTION ,,

Solution: First we need to write the integrand in a 4. If the rate of growth of a population is dn/dt, then
simpler form by carrying out the divi~ion 2
' dn
9 2 2r: 9 J-dt =n(t 2 )-n(t 1 )
J2t +t i"t-1dt = Jc2+t112 _r-2)dr ,, dt
1 t 1
is the net change in population during the time period
9
from t I to t 2 • (The population increases when births
r 312 c 1 · 3 312
=2t+----. =2t+-t
~ -1 ] 2
+-] 1
t
9

1
happen and decreases when deaths occur. The net
change takes into account both births and deaths)
2 . I 5. If C (x) is the cost of producing x units of a commodity,

3 3/2 +9 -
= [ 2·9 +2(9) l] ( 3 3/2 +l
2-1 +2·1 1)
then the marginal cost is the derivative C' (x). So
<2
JC' (x)dx = C (x 2 )-C(x1 )
· 1 2 4 ,,
= 18+ 18+- -2---1 = 32-
9 3 9 is the increase in cost when production is increased
froin x 1 and units to .'C 2 units.
Applications
6. If an object moves along a straight line with position
Fundamental theorem of calculus says that if f is functions (t), then its velocity is v (t)- s'(t). So
continuous on [a, b], then ,,
b
JJ(x)dx = F(b)-F(a) Jv(t)dl = s(t 2 )-s(t1 )
a
is the net change of position of displ~cement, of the
where Fis any antiderivative ofJ. This means that F' = f,
particle during the time period from t 1 10 t 2 . It is always
so the equation can be rewritten as. We know that F (x) true. In order to calculate the distance trnveled during the
represents the rate of change ofy = F (x) with respect to x time interval, we have to consider the: intervals when
and F (b)-F (a) is the change iny when x changes from a to v (t) > 0 (the particle moves to the right) and also the
b. . intervals when v (t) < 0 (the particle moves to the left) note
The Net change Theorem that during motion particle may reverse its direction of
The integral of a rate of change is the net change : motion. In both cases the distance is computed by
b . integrating lv(t) I, the speed. Therefore
JF' (x)dx =F(b)-F(a) ,,
a Jv (t) Idt = total distance traveled·
'1 .
Here are few applications.
1. If V (t) is the volume of water in a reservoir at time t, Fig. 1. 72 shows how both displacement and distance
then its derivative V' (t) is the rate at which water traveled can be interpreted in terms of areas under a velocity
curve.
,,
flows into the reservoir at time t . So

,,fV'(t)dt = V(t 2 )-V(t1)

is the change in the amount of water in the reservoir


between time t 1 and time t 2 •
2. If•[C] (t) is the concentration of the product of a
· chemical reaction at time t, then the rate of reaction is
the derivative d[C]/dt. So .. f'ig._!,72
2
' d[C]
,,
f - d t =[C](t 2 )-[C](t1 )
~ ~ . .
displacement= Jv(t) dt = A 1 -A 2 + A 3
<1
·is the change in the concentration ofC from time t 1 to ,,
distance= Jiv(t)jdt =A 1 +A 2 +A 3
t2,
3. If the mass of a rod measured from the left end to a ,,
pointx ism (x), then the linear density is p(x) = m' (x)
So
,,
The acceleration of the object is a (t) = v' (t), so

,, J a(t)dt = v(t 2 )-v(t 1)


Jp(x)dx = m(b)- mra) '1
<1 is the change in velocity.from time t 1 to time t 2

www.puucho.com
Anurag Mishra Mechanics 1 with www.puucho.com

1s2 ·:cc· _.· : ·: · · .· ~ -~·:, -- ·-- · ~- L ~-:


7
'>"~ M~CHA~icBJ
displacement is positive if the final position of the particle is
to the right of its initial position, negative ifit is to the left of
. ._-"".t:,~~'-·= " ' -· " - --·,_ - ,___ .,._ ·-·- -~,-; its initial posi#,on, and zero if it coincides with the _initial
[..4. partfrlemov~ ii}lmi a l_ine so piat its veloi:ity at time tis: position (see Fig. 1.73) . , · . • ,· '·
~(t)~t?cfel~~(nieqsured in mii~rs persecoryd). ' . :
tiJ/cEi'},~JIJ1i'dr&~1a~.einent of S~~ Rarticle during ihe :time;
k,,,-.. penod:!.·=,;,t :;;,.4; '·· . ·, · · · .
: --P.o.sitiv~:isp;ace~e~t - -~-, Ne~ative : : : : : ; "

. ..
-~1
, - ·l
~.:.Einci.nu(iJJ.stance
. .- . ·- . travel§§Audrti this ti1jie pe.rioiL:. :-+--'------'+-+-~ -1-----'---'---~-'-;__,l...'--
Solution:· '(a) By equatio~, the displacement is l•(lo) :.~(t,) . ,s(t,J ; ' ·, . ' }\o! ·. ·1

. · .·. , 4 4 '•·- .., ·--~-~='""-·--···---------


Flg.·1,,Z3 -. .. ' __• -.,.. •.p:. ·•
s·(4)__;_·s(l)',;; Jv (t) dt = f (t 2 -t - 6) dt ,.. -~~
Distance Jraveled in Rectilinear motion
-.-.-·)·:·:·.>- 1_ ·1_
4 In general, the displacement of a particle is not the same
. . :. ,• · =~[_c_£.:__6t] = 9
as the distance trayeled by the particle .. For example, a
.. •.·. , , . . 3 2 1
2 particle t4at travels 200 min the positive-ditection and then
This'171eans.that the particle moved 4.5 m toward the 200 m in 'the negative' direction travels a distance of400 m'
left. .· . :._."......
'.
'
.
but has a displacement or zero, since it returns to its.starting
position. The only ciise in which the displacement and the
(b) Nii'i1'·that ..v(t)=t 2 -t-6=(t-3)(t+2) and so
distance traveled are the )ame oc_curs when the particle
V (t) ,,; 0 op)!i,e''i~terval [!, 3] and V (t) :2 0 on [3; 4]. Thus, moves in the positive direction without reversing the
the distah~e 'traveled is . . .. direction of its motion.
' •.4· •• .. . 3 ' 4
1
. . , . -- ..... , ... :··:--r,---· . . ·--::·~
flv(t).Jdt = fr-v.(t)]dt+ fv(t)dt Conc,ept: Wh~t is . th_r;re/~tionship betwee:1/Jize
1_'; 1 '.·" 1. - . . 3 ;displacement of a pa,rti.cl~ and it~~ distance it ti:'ayf/Sif·'i/1e
3 4 'particle moyes iri tlie negative direction without reyefsitig the, ·
=.f (-t 2 +t + 6)dt +f (t 2 -t - 6)dt :direction ofmotion? · · , 1· - ',' :. ·, . • ·'. ;-
/ Integr:p.ting the velocity fu~ctidti:of a particle ov¢rd,time
: t3 -~2 ... ·J3 1. [t3 -t2 .]4 3.=-=10.l?m
61 !interval yields the displizcement'of a:particle.s .over tha't/tiine
[
· ~-+·-+6t · + -+-+6t
3 ·. :2 . ' \ . 3 . 2. 3 6
·
;interval. Whereas to find the,fofal d~tance traveled:0by't~'e
,particles over. the time ,inierv'?il (th~ ·_distance'., traveled'i~, tlie
~-. ~71"'~,;...~-~
•:c'.~~b,f;e:i 45 ·r,~ 'positive direction plus ihe /listimce' traveled the, 1'1egativ~1 ih
--'--...: --1~'!'::1.1 . ·~ ,direction); we must integrate the absolute valii~ oftheyelqcityl
"----~_i,;.:;,,,,r,,...,__,; ·• ,'---' . ·*1-·~---- -·-- . ···.-· ,,
junction; that is, we must-integrate the speed . · ,, ...
[Find ~,;~V'j;_i.,~itiPl1.i ':fi!ncticm oj>a: particle that moves with . . '
fveuici~ v[t'.fi rtli:ct along a,coqrdinate line, assuming .that , ' '[total distance·trav~l~d]'; '... ,
lthe partiqle'li,g:s cgiirdinate s=4 at timet = Q
i=c...--~""'i-¾~_~:~-::-~:--..------,_,,-,_ -- ---· · • I during ti~e inte1:yi :,;, J;;iv(t) ldt_
Solutjon: the position function is [to,t1J ·
. ·,--· "··- -----~~
.', -~:.:· :S{t) = fv(t)dt = f cosittdt = ¾sin1tt +C · r~l$;~iAfurii:l~J46~
Sin~e. · '• 1= 4 when t "" 0, it follows that
, 4~ s(O) = .!.sinO+C = c·
:~upp~;e th~~ a. p~rticl~ ;;;o~~-~~ I~traight line ~o.t/i~t'
j~
. It
,velocity attime tis v (t)= (t 2 ":' 2't) nVs. · · · ••· \
. 1 (a) Find the displacement of the .particle during·thl ,~ime,
Thus, s(t) = - sin1tt + 4
It
interva!0s;ts3. · · ·, . . . · ·, _· I
Displacement in. Rectilinear Motion
,Cb) ~ind the distance u:avel~d by.~he, particle duri% Ihe tpne1
111terval. Q ,,; t ,,; 3, _ .. 0 •• _ .::. ". ·,,, ... _ ..__ •• . ·• • ·:..;J
Suppose thats (t) and V (t) aje the position ~nd velocity . ) '

functions of a particle moving on a coordinate line. Since Solution: (a) The displacement is
v (t)is the rate of change of s (t) with respect to t, integrating
v (t)°<'ver an interval [t 0 ,t1 ] will produce the change in the
value ,f s(t) as t increases from t 0 to t 1 : that is,
JlvCtl ldt = Ji Ct
0 0
2
- it) Icit=

[.c -r
3
2
]
3

0
.= o
J:>(t)dt ,..·J;;s•(t)d_t =S(t1)-s(to)S Thus, the particle is at the same position at time t = 3 as
at t = 0
The expression's(t 1Y- s (t 0 ) in this formula is called the (b) · The velocity can be written, as
displacement or change in position of the particle over the v(t)=t 2 -2t=t(t-2), ·from· which we • see that ·
time interval [t 0-, t,J. For a particle moving horizontally, the
www.puucho.com
Anurag Mishra Mechanics 1 with www.puucho.com

[DESCRIPTION OF MOTION
. - - ·- !?3 I
- - - -- ·- ---------,-----'-"-----'---'
v(t) s Ofor Ost s-2 _and v(t) ~ Ofor 2 st s 3. Thus, the ' . ---~--J.:7~
distance traveled is p,E~~~H~ , ~
J31v(t)[dt
0
=J2-v(t)dt+
0 _
s:v(t)dt , For each of the _veloc_,·ty .Ve;~;;· d,;;~_~;rv~_
in Fig. 1E.4S,~nil'
= J02-(t 2-2t)dt+f"2 (t 2 -2t)dt .
:the total distance traveled by, the paracle over the time
.interval Os t s 4.
- .. - -.. --- ------------ -- .
· ----~ . I
, u (mis) -,...----, I
=-[t:-t2J: +[t:-t2I =i+i=~m 2

B
Analyzing the Velocity versus Time Curve : C 6
A valuable information can be .obtained from the -=--1''-.---i'--r-+---''c--'f,
0
- time (sec)

velocity versus time -1


2 3 4 5
o:'
J
curve. The integral v dt
''
V '
can be interpreted . -2 ---------------------------- '
geometrically as the net
area between the graph of
Flg.1E.48
- - -· -- ._._____ -
v (t) and the interval Solution: In an three pans of figure' the -total area
[t 0 ,t1 ], and it . can ·between the curve and the interval [O, 4] is 2, so the.particle
interpreted physicany as travels a distance of 2 units during the time period in an
the displacement of the Fig.1.74
three cases, even though the displacement is different in
particle over this interval. each case.
For a particle in rectilinear motion, the net signed area
between the velocity versus time· curve and an interval
[t 0 ,t1 ] on the t-axis represents the displacement of the
l~..i9i.p;~~,J~>
particle over that time interval (Fig. 1.74). 'Find. th; total area b,"tween the curve. y = 1 - ~ 2 and .th4
r~i=-:;. . ,--~ x-axis
over the interval [O, 2/ (!':ig, ·_lE:.~9~ _ . _ · I
~g~p.L~-,i~ , 1
V
I. .
J
, ;
.' I'
!Fig. lE.47 shows three velocity versus time curves for a,
!particle in rectilinear motion along a horizontal line. In each
---''---,f--.'1,-~- t l !
-1 -1 '
case, find the displacement of the particles over the time -2 le'
interval Ost .s 4, and explain what it tells you about the 1 -,3 I
r
motion of the particle.
I . -,,- V -- ---- V
Fig.1E.49 _ ·---=-='- ·____,_:,_,_::•___
1- : 1 Solution: The area A is given by
i
2 1 2
A= Jl1-x 2 ldx= JC1-x 2 )dx+ J-_(l-x 2 )dx"
0 0 1

(c) =[x-X:-J:-[x-X:r =1-(-34 )=2


Finding Distance Traveled from tile Velocity versus
-
Solution: In part (a) of figure the net ·signed area Time Curve . , . .
under the curve .is 2, so the particle is 2 units to the right of For a partide_ in rectilinear ni~tion,. the total area
its starting point at the end of the time period. In pan (b) the between the velocity .versus time. curve and an interval
net signed area under the curve is - 2 units, so the particle is [t 0 ,t1 ] on the t-axis represents the distance traveled by the
2 units to the left of its starting point at the end of the time
p~rticle over that· time interval.
period. In pan (c) the net signed area under the curve is ·o,
so the particle is back at its starting point at the end of the INTERPRETATION OF GRAPHS
time period. (i) Given the s-t Graph, Construct the 11-;t Graph:
We- can also interpret geometrically the total distance The velocity at any instant is determined by mea~uring the
traveled by a _particle in rectilinear motion by calculating net slope of the s-t graph, i.e., ·
area.
www.puucho.com
Anurag Mishra Mechanics 1 with www.puucho.com

-- u--- - - ---
I
~
s
u -
0-dtt=O
So

s·2.
s, 53
u,

,Q t1 . ti.. .. t3
• 1,
. Fig. 1.75 (a)
Fig.1,76 (a)
ds
:.............=v } a
dt
Slope of s-t graph ~ velocity ·

For example, measurement of the slopes v 0 , v 1 , v 2 , v 3 at


the intermediate points (0, OJ, (t 1 ,s 1 ), (t 2 , s 2 ), (t 3 , s 3 ) on the
s-t graph. Fig. 1.75 (a), gives the corresponding points on the
a, a,
a0 = o 83
v-t graph shown in Fig. 1.75 (b). t,
10 t, t, \3
''
Flg.1,76 (b)

Concept: Since differentiation reduces a polynomial of1


:degree ~ to that of degree ,n -1, then if the s-t graph is'
.parabolic (a second-degree curve), the v-t graph will be a
'straight l1ne (a first-degree curve), and the a-t graph will be a
~ons!_<1n_£ a.r a_ horiz_ol!!_al line (<)_ zero-degr.ee ~urve): .. _ _ i
u,

\·-,:p~.'l@:mJ?.J,~;J 50 ~v
. --- ----r:::7""--.

Fig 1.75 (b)


r--~ bi;cl; ~~v~ - ~l~~ ~ straight road s~~h -th~~-i~s1
iposition' i5 described by the graph shown in Fig. lE.50 (a).
(ii) Given the v-t Graph, Construct the a-t !construct the v-t and a-t graphs for Ost $ 30s.
Graph: The acceleration at any instant is determined by
measuring the slope of the v-t graph, i.e.,

· .du
l --~
I 500s (mis)
- . . . . ... . . .
A .
I
~;a I
dt
.. _ ~!_ope pf_v-t graph_": ~ccelera_tion_ !
•=201-100 I
'I '
'
For example, measurement of the slopes a0 , a1 , a 2 , a 3 at I = t2
'I
the intermediate points (O,O),(t 1 ,v 1 ), (t 2 ,v 2 ), (t 3 ,v 3 ) on I 100
S
' I
the v-t graph. Fig. 1.76 (a), yields the corresponding points 'I
on the a-t graph shown in Fig, 1.76 (b).
'
!
10 30 t(s)
Fig. 1E.50 (a)

www.puucho.com
Anurag Mishra Mechanics 1 with www.puucho.com

DESCRIPTION OF MOTION 55
Solution: /',.v=fadt.
change in velocity = area under a-t graph
Concepts: v-t Graph: Since v = ds/dt, the v -t graph
can be determined by differentiating the equations defining First calculate the particle's initial velocity v O and then
u
the s-t graph, Fig. IE.SO (a).
We have
ds
0 5 t < 10 s; V = - = 2t
dt u,
ds Uo
lOs<t,;; 30s; S = 20t - 100, V = - = 20
dt
(a) (b)
Values of vis the slope of the s-t graph at a given instant.
For example, at t = 20 s, the slope of the s-t graph is Fig. 1.77
determined from the straight line from 10s to 30s, i.e.,
add to this small increments of area (1',. v) determined from
u(m/s) the a-t graph. In this manner, successive points,
v 1 = v 0 + I',. v, etc., for the v-t graph are determined, Fig.
u = 2t 1.77 (b). Notice that areas lying above the t-axis correspond
20 r,1--,,-------i to an increase in v ("positive" area), whereas those lying
below the axis indicate a decrease in v ("negative" area).

Concepts: If the a-t graph is linear (a first-degree


"---.,>10~----3-'-0--t (s)
curve), integration will yield a v-t graph that is parabolic (a
second-degree cur-ve), etc.
Fig. 1 E,50 (b)
1',.s (iv) Given the v-t Graph, Construct the s-t
t = 20s; v=- Graph. When the v-t graph is given, Fig. 1.78 (a), it is
1',.t
possible to determine the s-t graph using v = ds/dt
= 500-100 = 20m/s ils = Jvdt
30-10
s
Concept: a-t Graph. Since a= dv/dt, the a-t graph u
can be determined by differentiating the equations defining,
the lines of the v-t graph.
a (m/s2)
s,
.1.s:=~1udt

t,
2>----~ t,
(a) (b)
Fig. 1.78

'-----'-10_ _ _ _ _3_0_ 1 (s) displacement = area under v-t graph


We calculate the particle's initial position s0 and add
Fig. 1 E.50 (c) (algebraically) to this small area increments ils determined
from the v-t graph, Fig. 1.78 (b).
This yields
05t <10s; V = 2t dv
a=-=2
dt
l!;§?<~iit~J_jiJ>
dv 'An experimental car in Fig. lE.51 (a) starts from rest and
lO<t 5 30s; V = 20 a=-=0
dt ·travels along a straight track such that it accelerates at a
The result are plotted in Fig. lE.55 (c). Show that ·constant rate for 10 s and then decelerates at a constant rate.
a = 2 m/ s 2 when t = 5 s by measuring the slop of the v-t ,Draw the v-t and s-t graphs and determine the time t' needed
graph. '_to_ stop the car. How far has the car travelled?
(iii) Given the a-t Graph, Construct the v-t
Graph. If the a-t graph is given, Fig. 1.77 (a), the v-t
graph may be constructed using a= dv/dt. www.puucho.com
Anurag Mishra Mechanics 1 with www.puucho.com

- ---
; - ~ , - -
When t = lOs, s = 5(10) 2 = 500 m. Using this initial
Solution: Concept: v-t ·Grnph: Since dv =,adt,:
condition,
the v-t graph is detennined by integrating 'the straight~line,
',s_egments of t/ie a-t graph, · l0ss;t s;60s; V = -2t+l20;
. .
' s (m)

10
a (m/s2)

~- 3000

s"= 5t2

500
A1 ;=-t?+ 1201-600
t'
t (s)
-2
10 A2 I '""'---1.--------'---t (~) i
10 60 1
Fig. 11:.St (a) Fig. 1E.51 (<;t ______ _J!
= 0 when t = 0, we have
I'
Using the initial condition v
ds = f'10 (-2t+l20)dt,
0 -< t < 10s·' a= 10·' o = f'o
Jv dv - ' v = lOt
lOdt· 500

s- 500 = -t 2 + 120t-[ -{10) 2 +120(10)]


When t = lOs, v = 10(10) = l00m/s. Using this as the
initial condition for the next time period, we obtain velocity s = -t 2 + 120t-600
as function of time. Position at the moment car. stops is obtained.
, , v· (m/s) Whent'= 60s
' V = 101 s = -(60) 2 + 120(60)-600= 3000 m
100 Second method : Ti}e s-t graph is shown in Fig.
V = -21 + 120 lE.51 (c). The triangular area under the v-t-graph would
yield the displacement I!. s = s .'.. 0 from t = 0 to t' = 60 s.
Hence, I

I!. s = .!:. (60) (100) ~ 3000 m


i(__...L_ _ _ _ _ __:,,._ I (s)' 2 . .
10 t' = 60 (v) Given the a-s Graph, Construct the v-s
Fig.1E.51 (b) Graph. For given a-s graph for the particle points on the v-s
graph can be determined by using v dv = ads. Integrating
10s<t ~t 1 ; a==-2; this equation between the limits v = v 0 at s = s0 , and v = v 1
f1:o dv = f:o -2dt, ands= s1 ,we have,

V = -2t + 120
1 2 -v 2 ) = <1 ads
-(v 1 0
J
2 '•
The time taken to stop the car can be obtained by = area und~r a-s graph
substituting. v = 0.
t' = 60 s V

Second method : The area under the a-t graph is


equal to the change in the car's velocity. Thus net area under
curve gives I!. v = 0 = A1 + A2 . Fig. lE.51 (a). Thus ia
0 = 10m/s2 (10s) +(-2m/s 2 )(t'-10s) = 0
V1
t'= 60 s .I
I
Sin'ce ds =v dt, integrating the
··1
1
Concept: s-t Graph: I

equations of the v-t graph yields the corresponding equations,


of the s-t graph. Using the initial condition s = 0 when t ;, 0, i
we have (a)
s,
'(Ii)
J
i

0s;t s;lQs; v = l0t;


_Flg,~_.7y . __ , .... ___ - __ J
, ds. = I, lOt dt, s = 5t 2
IO O

www.puucho.com
Anurag Mishra Mechanics 1 with www.puucho.com
J
._ I I
'
' 'DESOIIP'tiON
• . " OF,MOTION
. a
• ' ,,_ i

Area under the a-s gra;h, fsos, ads, is determined and the 5
u•O
motion-= . u=5.
initial value of v 0 at s0 = 0 is known, then ''
1 2
V1 = (2fs ads+v~)1/ , .__ _·..;F:..,11!. l E,52
so
200
(vi) Given the v-s Graph, Construct the a-s · distance = 2 x Us + 228 = + 228 m
3
Graph- If the v-s graph is known, the acceleration a at any
position s can be determined using ads = v dv, written as Objective
dv) ·
Equation of motion not applied if acceleration is
a=v ( ds variable.

- ; - - - - - - --.: ---=--7
acceleration = velocid times slope v-s graph Concept: If a is gi_·ven asfun_ction of-s or v
1 ·-
~~~ation vdv = ads

ij~xam~4e.;;;1 53 ~
, ., . · _
_;e c~xi use
,

1 If a particle acce_1er_ates with a .= °icv_ 2 ana initial ;~iacity = u


V
. ) 1
then fim:1 velocity aftgsdisp)acement. ·
a =v (du(ds)
,I
! Solution: Jal= dv; kv 2 = dv; kdt = dv
/_ dt dt v2
l •.
1 s
'-t..___-_-S___J---"·-'------i- ~-~___.___--'--- S, On multiplying both sides of eq. by 'v'
I
L r-- ---i
------
, i--- s-i
ca>- - - ,E!!!c..1:~J___ - . -
' .
Thus, at any point (s, y)in Fig. 1.SO(a), the slope dv/ds of
(bl 'I
ds
-·kdt=v-;
dt
dv
v2
f
dv f
kds= -;lnv=ks+Cats=O;

C=lnu; ln(~)=ks
V

the v-s graph is measured Then since v and dv/ ds are known,
the value of a can be cak:ulated, Fig.1.80 (b). v = ueks

~~~riiJi~
.-
[:::,ea:a~eu;J;til:~i~~~velocity. at -t = 0, u = 5; . Findl jGiv:~ ~ '." "'cos.t; at t =0; u =·o; x=.1
IPosttwn at t = 1t
Solution: a= 4J 2t · ~l!!d d~J.JJ.!!£.e:.Ji:om O to 21t,
V
1 )dt
t
f dv = f (4;--2t · · Solution: a= - cost
V t
S 0
V
2
= 4t - t + 5 = -t + 4t + 5 2 Jdv = -f costdt
0 0
- = -(rt - 4t - 5) = -(t - 5)(t + 1) V =-sint
After 5 sec, velocit: will be negative X • "'
t t3
f dx = f-sint dt
f dx = f (4t -t
X
2 2 1 0
+ l) dt =} X = 2t - - + St
0 0 3 0 to 2it, v will change sign. velocity upto it :will be

=-m.
100
3
x 5 = x = 2(5) 2 C5l + 25
3
[In forward direction]
negative and r::11:: :!1:11:::t dtl = 2+ 2 = 4.

3
and t = 12 sec kEx~IB:l;e.cj 55 ~
· • (12) 3 .

~here ~
X12 = X = 2(12)'.:.. - - + 5(12) = -228 m
3
porti<1, """" o/O"K - ;;~ """'"'""' 0 a ;{, -
w.t is in second. If the particle. is initially at the ong{n. and ·
it moves alimg positive x-axis with v 0 =2 m/s,jind the nature

I
[
of motion oftlte particle. . · -: -~
www.puucho.com
Anurag Mishra Mechanics 1 with www.puucho.com

158 ----------------------
. dv Average Velocity and Average Speed
Solution: G1vena=-
dt'
If a body is moving along a straight line and its velocity
We have -dv = 6(t-l) t=O t=t v
--<:>--------o-+-'-o- x,
Vo
as function of time is known.
dt V = f(t)
s
We have total displacement
then average velocity
Jvo"dv = Jort6(t-l)dt = 3t -6t
2 Fig. 1E.55 (a)
total time
t, V dt
Substituting v 0 = 2 mis, we have = Ito
dt
- 6t + 2
V = cJx = 3t 2
J'" dt
, 'o
Again integrating both sides, we have
Concept: Average speed may be greater than or equal to'
s: dx = J~(3t 2 - 6t + 2)dt ,the magnitude of average velocity. If a particle turns during
:motion then speed and magnitude of average velocity' will be,
X = t3 - 3t 2 + 2t = t(t -1) (t - 2). 'different.
Following points are noted from the above ! Put v =0 and find the different instants of time when!
example. velocity' is zero, let they be t 1 ,t 2,t 3 , ... tn. Split the limits of,
Putting x = 0 in x = t (t - l)(t - 2) we have t = 0, 1 integrationfor those instants where velocity becomes zero and;
and 2. That means, the particle crosses the origin twice at ;lie between t 0 and tn and take modulus for every individual'
t = 1 and t = 2. After t = 2, x is positive. Hence its 1
integratioTL Reversal of motion is possible at those moments:
displacement points in positive x-direction and goes on
,on(y when velocity' becomes zero and that is why we have
increasing its magnitu~e.
'splitted the (imit of time.
t=O This gives total distance.

1
1=1 1
t,=1- ,/3 , total distance =J J;~ v dt j-t;I J:,2 v dt J+ ... + JJ::, v dt J
12=1 + ,/3 '------{}-t-=2_.....,_ total distar.ce
average speed = _
total tlmi

Since v = 3t 2
- 6t
Flg.1E.55_(b)
+2, putting v = 0, we have Js:~ 2
vdt J+IJ~ vdt J+ ... +JJ::, vdtl

t 1 = 1- ~ andt 2 = (1+ ~} Hence the particle remains Jto'" dt


f ···--·- • - · - - • •• ,---l

6>·,
I

stationary at t = t 1 and t 2 • . l( LaJ~:~,q~~,,t,~ -I 56


The displacement, velocity j'

and acceleration of the A p;rticl; moves. ~long a stra~ht ll~e a;d its ~eloci~·d;pencb;I
particle in different time t ' on time as v = 6t-3t 2 where•,]_• is in m/sec and 't' is in,
intervals are given in the 1' : 1 2
x/t graph ,second. Find average velocity' and. iVerage speed for first four,
-Xo .t-:-43-:-·• • 1
following table and shown I ' ' •1+- rseconds. __ _ _ _ _ _ _ \ . __ . _ _ _j
in the following graphs. V i i ,/3
Solution: Average velocity'~ Total displacement
(i) O<t<(1- ~) +vo ' Total time
f--'',--,..-,~-..,....-t
v-t graph
J:vdt '5J:tdt-3J:t 2 dt
x> O;v> O;a< 0

00(1- ~)<t<l
·Vo
Vav = J: dt \ 4
'
48- 64 I
+1 ... ,........ ------ =- - - =-4m/sec
x > O; v < O; a < 0
(iii)l<t<(l+ ~) a-t graph
I
For average speed, put v:
0 antetermine roots oft for
which reversal of motion can take ~ace.
x<0;v<0;a>0 Flg.1E.55 (c) 0= 6t-3t 2 I
I
(iv) 1 + ~ < t < 2 x<O;v>0;a>0 => t = 0 and t = 2 sec.

(v) 2 < t < = x> 0·v> 0·a> 0


www.puucho.com
\I
Anurag Mishra Mechanics 1 with www.puucho.com
J.

DESCRIPTION OF MOTION 59!

~X9..!Jn.~l.ej~
Thus, average speed= IJ: v dt l+I J; vdt I
J; dt · IA particl~ moves along a straigh.t line, x. At time t = Q ;;I

= I6J:t dt -3 s:t
2
dt 1+16 s: t dt -3 s: t 2
dt I
a:~!
!position is at x = 0. The velocity, v, of the object changes
!function of time t, as indicated in the Fig. lE.58; t is in
\seconds, V in m/sec and X in meters.
4
(a) What is x at t = 3 sec ?
\ 4[+1- 20[ = 24 = 6m/sec 1
(b) What is the instantaneous acceleration (in m/sec ) at,
2
4 4 t = 2·sec? t-- ________ ....._, _____
7
Average Velocity by Integration f +3 I

If velocity is a function of time- I \


v = f(t) I +2
\ v (m/sec)
J vdt
then -J-dt
is the time average of velocity
+1


If velocity is a function of position-
3:
-1 I
v = f(s) I

Jvds 1
then-J- is the space average of velocity i.e.,
ds .
-2 I
averaged over position.
...;i Fig.1E.58 I I
b~~~!'J¥))'1L~~ (c) What is the average velocity·(i~-m/sec) between t = 0.andj
-------·-·----------------~
'
\ t=3sec? ·
!Velocity vector of a particle is given as rd) tw::~~s! t;e average speed (in ~s~:__~etween t = 1 an~
I --+ ,., ,.,
\ V=4ti+3j
Solution: (a) x at t = 3 sec
!At t = 0 position of the particle is given as :i'0 = 2j:
= Area (0- l)sec+Area(l - 2)sec-Area(2- 3) sec
IFind the position vector of the particle at t = 2 sec and the " 1 1
b,ergge acceleratioru,f the p_article for t = 0 to t = 2 sec. =3xl+-x3xl--x3x1=3m
2 2
--+ . ,.. A

Solution: v=4ti+3j (b) Acceleration is constant fort = 1 sec tot = 3 sec


... Acceleration at t = 2 sec : a =
v, -v-
'
--+ J'->
J~
_ dr=
ro
0
vdt t f -ti

--+ --+, t ,.., t ,..


t; = 1 sec, t1 = 3sec, vi= +3 m/s, v 1 = -3m/s
=> r- r 0 = J0 4t idt+ JO 3dt j -3-3
a=--=-3m / s2
2
= 2t 2 i+ 3tj . . displacement 3m
(c) Average velocity= . = - - = 1 m/s
tune 3sec
Distance
Att = 2sec. (d) Average speed = ='--.- -
time
--+ ,.. ,.. ,.._
r = 2j+ Bi+6j Area with magnitude only
=
time
= Bi+ Bj
!_x3xl+!_x3xl
For average acceleration initial velocity at t = 0 Ar(l-2)+Ar(2- 3) 2 2
-> =3j
A = 3 3
1 m/s
1:,Vi

Final velocity at t = 2 sec.
_, A A

v 1 = Bi+3j
_, _, Given that x = 120-15t-6t 2 +t 3 (t > 01 find the time
when the velocity is zero. Find the displacement at this
--+
a=~--
VJ-Vi
Bi = 41
instan."'t·~-------
t 2 www.puucho.com
Anurag Mishra Mechanics 1 with www.puucho.com

Solution: X = 120-15t - 6t 2 +t 3 Solution:


dx (a) Distance travelled is given by the area under
v=-=-15-12t+3t~ =0
dt velocity-time curve.
Solving, we get · t = 5, - 1 sec. As t > 0, t = 5 sec. Therefore distance = Iarea of A I + Iarea of BI
putting t = 5 in the expression of x, we get + Iarea of Cl+ Iarea of DI
x=20m (All areas are considered +ve irrespective of the nature .
of the quadrant in which they lie).
~!il:m,i-ii,l,e~f7ol~
~ ,_:: CE v & : ~ ~ =~x2x2+2x2+~xlx2+~xlx2
-r- - - - , ,___ ... - --- ~- - . - ~.----· .•,, 2 2 2
(The figure shows the (V,t) graph for the train .accelerating' =Sm
ifrom resi up to a maximum speed of v
1
1
. and then ms- fb) Displacement= area of A+ area of B + area of C
decelerating to a speed of l Oms·- . The acceleration and
+ area of D
deceleration. have , the same magnitude which is equal to
0:5'·.
. m s. •
I
z ., .
./ -~---
. - . • ,
_ · .. ~=-.-"'--
(Proper signs of areas are considered according to the
v (ms·') I nature of the quadrant in which they lie.)
i 1 1 1
V =-x2x2+2x2+-xlx2--xlx2
2 2 2
=6m
·' 10

., ,
o
0 t
t (s)
:
~:~~e,!~J62lb>,
~ · Fig. 1E.60 (a) i .. IA birdfl.iesfor 4s with a velocily v = (t - 2) m/s in a straigh~i
§h6'0h<it the~distance travelled)s_02V!::J,OOtr,;etre. _ ·-~J ~:;l!;!e::~e;e~~;i;:e~f::i~ Ca.lc~late the displace~ent,arutj
v2 -u2
Solution:. v 2 -u 2 = 2as s=--- Solution : The displacement is given by
2a 4 4
For the motion between 0 -·----·---
v,(rps·1)
s=J 0 vdt=J 0 (t-2)dt
.
andB
', •V
Distance (s 1
·v 2 -o 2
) = -.- - = v =1t:-2t["=o
2x0.5
', For the motion between A 10 The velocity of the bird become zero at;
. and B 0 = t - 2 .=> t = 2 s
102 -V2
,Distance (s 2 ) = - - - -
2x(-0.5)
Distance s = JJ: vdt HJ: vdt I
'
= v 2 -100 =Js:ct-2)dt J+Js: (t-2)dt I
Total distance = s1 + s2 = 2V 2 - 1.00
=1t:-2ti: + [t:-2.ti:

The velocily-time graph for a travelling along a


straight line is.shown in the Fig. 1E:61. Find -·-
particle 2 2
=1 :-2x21+1(~~2x4)-( :-2x2)/
. ' v (mis) =2+2=4m . '
2 ···········~--~
Graphical Method The velocity-time and
B speed-time graphs of motion of the bird are as follows:
A: C 6
time (sec)
0
-1
1. 2 3 4 5 o:'
''
'
I -2 ---------------------------- '
1, Fig. 1 E.61
·-------~~--------
(a)·distance travel(edfrom zero to 6sec.
'@Jlispjactimentinft<Laboyetime'interval. _ _ _www.puucho.com
_ __,__,
Anurag Mishra Mechanics 1 with www.puucho.com

I-L_ DES(ilPTION OF MOTKlN --


••• __ __ .. -- ·- --- ----------- 61 !
_,
velocity (mis) (d) [a[=l
V = t-2 _, _,
(e) ar = (a-v)v
2

B
= ([j. (i+t j+k)]J (i+t j+k)
·~t 2 +2 ~t 2 +2
+
0 + - - - - 7 ! " ' - - - - t - - - - time (s)
t (i+tj+ k)
A 2 4 _, ( t )· or ''
ar = ~t2+2 v 2
(t + 2)
-2

Variation of velocity with time


(a)
speed (mis)

2 ----------------------------

A
+ + B
O + - - - ¥ ' - - - - ' - - - - - t i m e (s)
2 4
Variation of speed with time A particle moves in xy plane with a velocity given by,
_, A A

(b) v = (St - 2) i+ 2j. If it passes through the point (14, 4) at'


Flg.1E.62 t = 2 sec, then give equation of the path.
From the graph, Solution: = 8t - 2
vx
displacements =I area Al-I area Bl= 0 Jdx = JC8t-2)dt
and distances =[AreaA[+[AreaB[ x=4t 2 -2t+c
1 1 At t = 2; X = 14 =} 14 = 4 (4) - 2 (2) + C C =2
=-x2x2+-x2x2=4m =}
2 2 x=4t 2 -2t+2;vy=2
i,_€,i~qi:p_p_l)~ ;~-~ ;:.> f dy = f 2dt
y = 2t+c
A particle moves in such a way that its position vector at any[ Att=2;y=4 =; c=O
·time tis; =ti+-1:.t 2 j+tk. Find as a function of time (a)' y = 2t
2 Equation x=(2t) 2 -2t+2
the velocity, (b) the speed, (c) the acceleration, (d) the·
'magnitude of the acceleration, (e) the magnitude of the =; x=y 2 -y+2
-component of acceleration along velocity (called tangential' =;x+y-y 2 =2
acceleration), (f) the magnitude of the component of
acceleration perpendicular to velocity (called normal
acceleration).
--) "' t2,._ " First, an auto starts from rest and accelerates uniformly for 15
Solution: r =ti+-j+tk s acquiring a velocity of 30m/s. Secondly, the auto then moves:
2
_, at this constant velocity of 30 mfs for the next 15 s, after,
_, dr O ~ A
'which thirdly, the auto decelerates uniformly by braking at
(a) V=-=HtJ+k
dt 1.5 m/s2 until it stops.
(a) Sketch the velocity-time graph.
(b) speed l;l=~t 2 +2 ·(b) Sketch the acceleration-time graph for auto.
,(c) Sketch displacem_eT)t-time graph for guto. _
(c)

www.puucho.com
.\ Anurag Mishra Mechanics 1 with www.puucho.com

162 MECHA~·~-· I
Solution: [2v 112 ]~ 0 = -at
(a) r:·;;~~---··
ell
-- ----·-7 ' • '
2[ -v¼ l = -at 2

§. ·! : I
" ~
~ .' I '' ! t=~--
O, ', .15 30 ,50 J a
t (sec)' . ! (d) Velocity at any time t is
Fig.1E.65 (a)
------------......1
i V dv
f v 0 -v1/2 ft
- = - adt
r----- ,- I o
oo:i
IA 2
I
I
[2_vlf2 ]~0 = -at
' C I! 2[v1;2 -v¼2] = -at
' :8
ij o 15 30 50t(sec)I
v=(Fo-a;r
, ~-1.5

I '
'
i _____ Fig._1E.65(b) _ ..S_'.J
V = cJx = (Fo - at)2
dt 2
(c) Displacement between t (0-15) = !. x 30 x 15
\I =225m
2 f~ dx= f~ (Fo-a:rdt
2
r::....
'E
975 · - - ·-· . - •
1
• --
---·7 I
at 3 at Fo
X=Vot+--
I:;:- ' 12 2
Iii 675 . 2.,Fo
E at t=--
"
~-· a
I
rl ~250 15 30
50 t (sec) j
i
X=--0-
2 v'f2
3 a

L _ .... _ Fig. 1E.65 (c) ···- _ _ !


is .giv.~
Displacement between t (15 - 30) = 30 x 15
=450m
IA
I
~
particle . ti-av.els
A
--,
1
· so that . its acceleration
A
'
j
o/1
a = 5 cost i- 3 sint j. If the particle is located at (-3, 2) at
Displacement between t (30 - SO) = !_ x 30 x 20

L1z,x~mti
~ """ -
h.D-
1·e?~ 66.....J~--
--·)'· -~'.(?~~ll
2
=300m
1
time t
IFind
= 0 and is moving with a velocity given by (-
.
( a) the velocity .at time t and· , · ·.
'(!,J_Jhe_p~/JjQJJ~l'_e_c_to_r_pfJlz~:p_aJ:tisk.at..ti11i.e_(t 2'......0)_.___ _
J
3·f + 2 j) .

~ parric!; ~i~i_a v;l~u;, ~ =v O at_ t = 0 is decele~ated ~~ ...,

Solution: a=Scosti-3sintj
A A

I
the rate Ia I= a ,hi,'where a is a positiye constant.
. - - 2.,Fo.
(a) The particle comes to rest at t = -·- - '
Ji;= Jscostdti-J3sintdtj
bJ The particle will
I
'(c) The distance travelled.
I(d) The distance travelled
,• ... ,
.come to rest at infinity.
by the particle is - -
,.
a
0

2 v3/2
by the particle is - - 0
- •

- , .
.
. ., .
• 2v3/2
. j f v, dv, =f' Scostdt
-3

v,=Ssint-3
dx = (Ssint-3)
dt
0

I
l_ Mark
3 a
the correct opJion: _ _ __ · ________ _
-
f-3x cix=J'0 (Ssint-3)dt ',·
x+3=5-Scost-3t =}_X=2-Scost-3t
Solution :(a) a= -a.Jv
dv
-=-av 112,
Similarly, dvy = -J'3sintdt J"' 2 ··~ 0
dt vy-2=3(cost-l)
fo dv =J' -adt vy = 3 cost -1
v0 v1/2 o

www.puucho.com
Anurag Mishra Mechanics 1 with www.puucho.com

i- DESCRIPTION
- - ---- .--
OF MOTION- - -
-- - - -
63

J: dy = J~ (3cost-l)dt projection_ Generally initial direction of motion is


considered to be positive. Since g is almost constant,
equations of motion with constant acceleration can be used.
y-2=3sint-t
(a) Body dropped from a height:
y=2+3sint-t x 0 = 0, v 0 = 0, a=+g,
""7 A A
Equations of motion are V =gt
Thus, v = (Ssint -3)i+(3cost -l)j
""7 A A
h =_!gt2
and s = (2- 5 cost - 3t) i+(2 + 3sint -t)j 2
2
= 2gh
- -
l=.~Pl'.l'\P,~=- 1 68
- r--~
'> v

(i) A particle is moving in three dimensions.Its position vector


is given by
~ A A A :-··,
r = 6i+ (3+ 4t)j-(3+ 2t -t 2 )k '' ''
'' ''
1Distance are in meters, and the time) t, in seconds.
''' '''
( a) What is the velocity vector at t = + 3 ? ' :'
,o'
(b) What is the speed (in m/sec) at t = + 3 ? ¼
(c) What is the acceleration vector and what is its magnitude (c)
(a) (b)
(in m/sec 2) att = +3?
(ii) Now the particle is moving only along the z-axis, and its Fig.1.81

position is given by; (t 2 - 2t - 3) k at what time does the (b) Body projected upwards:
-particle stand still? _ x0 = 0, v0 = u, a =- g
Equations of motion are v = u - gt,
Solution: "i! = 6i+(3+4tJJ-C3+2t-t 2Jk 2 2 2
h =ut-½gt ; v =u -2gh
-+
""7 dr A A
(c) Body projected from a height:
(i) (a) V = - = 4j-(2-2t)k
dt
(1) Upward projection (2) Downward projection
""7 A A

At t = 3, V = 4j+4k (m/s)
x0 = 0, v0 = u, a= -g x0 = 0, v 0 =u, a=+g
___ (l)
(b) speed =l~I= ~42 + 4 2 = if2 m/s v=u-gt ... (1) v=u+gt
v 2 = u 2 - 2gh ... (2) v2 = "2 + 2gh ... (2)
~ dV
= ut - -1 gt 2 h = ut + .! gt 2
A

(c) a= - = 2k (constant) h ... (3) ... (3)


dt 2 2

At t = 3 also -+ m s2 +
A/-
a= 2k , I al= 2m s I 2 In both the cases if we want to calculate time taken to
reach the ground we should solve quadratic eqn, (3) for
""7 time t
r = (t 2 -2t-3)k
A

(ii)
In first case, eqn. (3) will be
""7 dr
-+
A
-h =ut _ _! gt 2
V =- = (2t - 2) k 2
dt
or gt 2 -2ut-2h = 0
-+
2
= -2u-±-~4u
stand still means v = 0 ~ t = 1 sec - Bgh
or t '---=-
Motion Under Gravity: 2g
Experiments show that when air resistance is neglected, In second case, eqn. (3) will be
all bodies near the earth's surface fall with the same l 2
+ h =ut+-gt
constant acceleration, denoted by g. We call this the 2
acceleration due to gravity and for practical purposes its or gt 2 + 2ut - 21! = 0
magnitude is 9 .8 m/s 2.
In describing motion of an object projected near earth's
surface, we use the y-coordinate with origin at point of

www.puucho.com
Anurag Mishra Mechanics 1 with www.puucho.com

j!i4._._ _ _ _ _ _ _ _ __ . -- . . - - --- MECHANICS-I I


---···-·----- ----------------------. -----:'.2..l

or
-2u
t=-------
± ~4u 2 + Sgh ...
s
distance (s)
2h
2g
In both the cases we will neglect the negative root oft.

&152&g.m;p~ 69 ~
h ................... .

[A·b;dy ;rojec;;d vertical~-up;;r-:i;-;;;,; ;he ;;;-oj-;, ;~;e;! t


II
ireaches the. ground in time t 1 • If it is projected vertically: 0 o·"""''---1---L--1,
·{2hlg 2-,J2h/g
' downwards from the same position with the same velocity, it\ -,J2h/g 2-.j2h/g
reaches the ground in time t 2 . If a body is released from rest•
and from the same position, then what will be the time (t),
~required by the bod·po reach the ground? .. _. ____ . .. _ I
Solution:
.r-·· - ..···- -· -- -...... · 1·

Ql<'----'---~---
-.j2h/g

... (i) '


"1 C '
For B,
1 2
-h = -Ut2 --gt2
2
... (ii) :t i A
t: I0
'
!
(v)
speed ...
a

'.lI
:t
Fore;
: : -.J2gh ....... ..
91--------
-h = o-.!:.gt 2 ... (iii) ' ' :
2
Multiplying equation (i) by '
t 2 and (ii) by t 1 and adding I.
__ ~ig. 1_E_,6_9__ . 0
' t 0
togethei; we get
1 Fig, 1E.10· • .!
-h(t 2 +t 1 ) = --gt 1t 2 (t 2 +t 1 )
2
1
-h = --gtit2 ... (iv)
·r --~---·-··---
2
;A body thrown up and return back to its initial position. Draw:,·
Equating equations (iii) and (iv), we get ! ~- 1

1 2 1 ,.;:- !displacement (s)-time (t), distance,(s) -time (t) and velocity!


-2gt = -2gtit2 ~ t = ,tlrl2 '>-'Jo .' -) '
'
!Cv)-time (t), speed (v) time (t) ana acceleration (a), - timej
0

~J;:x~me1•~rm1> [(t) _graph_sfqr: the mptio_n of t_h~ body._



Solution: Let the body is thrown with initial velocity
,
i

[.4. b~<fy-Jai,; fr~~·~~m; he~h; and· -;;;;,-:.u· b~ck to initi;li


u, it takes time t!. to reach the highest position. It goes to a
·position. Draw displacement(;) -time (t), distance (s) -time\ 2 g '
~ . height h = !:.... Neglecting air resistance we have following
;Ct) and velocity (v) -time (t), speed (v), time (t) and; 2g
~ • I
graphs:
!accelera~~n_ (~) :~Tl!e _(~2!5fEPhs jj_~_t~e !'!O!ion_<Jf_ tJ,e /Jody. ,
Di.stance (s)
Solution: Let body falls from height h. It takes time
....
J¥ to strikes the ground, its velocity just before strike is
•s u2f--',----~
g

.J2gh. Neglecting time of collision,' we have following u2


2gl---7"'
graphs:

· 2u I o,'----'u'----2.1.u_,. t.
___g - .. -- g g

www.puucho.com
Anurag Mishra Mechanics 1 with www.puucho.com

rDESCRIPTION OF rionoN
i...,. - -- , _ _ ,,_ - · - - -- -·-·--- -----·-· ~-
-----· _______ 6aj
-, phase corresponds to vertical lines on the velocity versus
V speed (u)
time graph.
u
-------r::7
L~'==~~,mpll~a:'.:1 73 ~
' -- - - -
Of----u""'"-~2u~- ·A boy throws a ball vertically upward with an initial speed o/
g g '15.0 m/s. The ball was released when it was at 2.00 m above,
'ground. The boy catches it at the same point as the point of
-u -------------------- . .
pro1ectzon. '
(a) What is maximum height reached by the ball?
-, (b) lfo'!!! lQIJg is_ t!Je f/all in the air?
a
Solution: (a) The ball will continue
to move upward as long as it has velocity. At
01-----+-----+- the maximum height v 1 = 0. We choose
u 2u
2g g point of projection as origin and upward
--g direction to be positive.
Fig. 1E.71 From equation,v; = u; + 2ayy
V2 =U2 - rym,
y y "&I'

At maximum height, Fig.1E.73


u2
A ping pong ball is dropped from a height H and bounces I - y
Ymax. - 2g
three times before it is caught. Sketch graphs of its position, ·
velocity and acceleration as functions of time. Take upwardi (15) 2
direction 11§ positive._ _ _____ ; = 2x 9.8 11.5m
Solution: Position versus time graph is parabolic.
hmm. =Ymax. +2= 11.5+2= 13.5m
Velocity versus time graph is a straight line.
(b) When the ball returns to starting point, y = 0.
Is le lo le tF 1 2
From equation, y = u y t + -2 ay t

0 = Uyt
1 gt 2
- -
2
2uy
~~-~~~---~-1(s) ' or t=--
g
= 2(15.0) = 3.06 s
9.8
Remark:-------------------
(i) Students can choose either of the directions upward or
downward as positive. But one should apply this· sign
convention to all the vectors throughout the problem.
If upward is positive, a= -9.8 m/s 2
~-+11--+---lll---+--clll--l-l(s) If downward is positive, a=+ 9.8 m/s 2
(ii)
1
~
-4
-81---lll--+--llf-__,_ _-+-_
Draw a coordinate axis on your diagram and assign
origin. In the problem, identify the special condition
regarding the object, e.g., maximum height reached
"' implies v, = D at the topmost point, or if the body returns
-12~~-~~----~- to origin then y =0.
Use letters x, x 0 , v, v 0 or t for unk ·owns in equations.
Fig.1E.72
Look for equations which involve unknown quantities.
At each collision the velocity of ball changes from One of them might provide you a solution. If more than
one unknowns are involved, then try to formulate as
negative to positive. When the ball is in contact with the
many equations as there are unknowns.
floor the velocity changes substantially during a short time
interval which shows that the acceleration is very large. This

www.puucho.com
Anurag Mishra Mechanics 1 with www.puucho.com

~ m~st~ridi!lt iii.
cili elevdt~; o~~ebi".S. ascrew fall from the !A speeder, moves:at a ~qns~antJS ,;,;.sin a scho;(zdti-;A
. tceUirlg; Tlle'ceiliflgis•3 mabove:theJlopi: · · , : , . . /police car startsfrom rest)ust,as:the:sp_eeder passed it.·Tfr~
(a.ii Ift/te,eleva'toi'it,inaving upwai-q-w{th a sp~ed of 2.2 ,nls, !police. car acce.lerqtes at.2 m/s 2 u_ntil it reaches its.maximum!1
t' _;Jiqw l~,ng clqes,'it.t'*for-_th~ scr~iti to, hit the floor? ! . : l~e!ori_ty .o! 20_,m/s. Wh~re ·and, when does the ·speeder _ge;I
(!,) J-1:fqw long is.the so:ew m mr_if the:~lev~torstarts from rest k@g/lt? ' ; ' -
I,~..·:when_the,screwfalls,,_and.moves. upwards with a constant
Solution: When two particles are involved in the same
l ·acceleration ofA~4.0mls 2
? .• · · ··
problem, we use . simple subscripts to distinguish the
·.·solution:,, (a). We consider the elevator. floor to be variables, as shown in Fig.1E.75(a). The motion of the
origin., •The elevator floor moves with constant velocity.
Equation for floor, y f ·= v ft = (2.2m/s)t. The screw falls 0 ~ Vs:= 15 mis [!J ~ -~s
0--
with acceleration due to.gravity. .. - !Pl· .. - IPl: '
,.. ,
, .. h.. (22)' l
,· Y, = _+ . t,-;:-2gt 2 y·:'.~'~P .y· -i- ~, ~~~- [;l-
·--·
.--~ C:(~x t=L\t1' ·' t = ~t1+dt,2:
' . _ _ _ _ _ _ _F..,,,lg.1E.75,(a)
.__
police car has two phases: one at constant acceleration and
one at constant velocity. In such problems, it is convenient to
use M instead oft in the equations. The police may or may
not catch the speeder during the acceleration phase. This
has to be checked. We set the origin at the police lookout,
which means x 0 s = x 0p = 0.
Acceleration Phase: Let us say this takes a time·
interval Llt 1 .
From v=v 0 +at, we have 20=0+(2)M1 , thus
~e., "-X, =yf M 1 =10s.
· ·c2.:d~
- -
=,,..:..
.
c2.2Jt - .!2 gr 2 At this time, the positions are given by
1 2;
,· or · : t= ~=~ 2( 3) =0.78s
x = x +v t +-at
0 0
2 1 -
fg· 9.81 Xs = (15) (10) = 150 m; Xp = - (2) (10) 2 ~ 100 m
2
, Note tlianhe screw's position at tirrie t = 0 was h and The speeder is still ahead.
due .to inertia' of inotion at the instant ofrelease its velocity
Constant Velocity phase: Let us say this_ takes a time
will b~ slririe.a~ that.of elevator.'
interval M 2.
Cb) Eqtiationfor floor is·: : Given: Xos = 150 m; x 0p = 100 m; Vs = 15 m/s;
,.-~ .
~ · ·vp = 20in/s; as= ap = 0
. 1 .2
' ' Equation for ~crew ~ .y, = h --gt ·unknown: x~ = ?; Xp ;;;; ?; L1t 2 =?
2
The cars meet when they have the same position, that is,
When screw meets the floor, . y' "' y f X s = X p • However, we cannot find where until we find when
.J 1
,; 1 3
i • 2 aft = - 2 gt·2
2
Xs =150+15M 2; Xp =100+20M2
On setting Xs = Xp we find r-·-xc--------,-:,
:.· - ~ 2(3)
.
'' . :
t. '.:'
''
vg+
af ';" (9:81 + 4.0) ..
Llt 2 = 10 s. Substituting into
either equation gives
300
'(m)
..,______,,
-,
·= 0.66 S· ,
x = 300 m. The speeder is ,... ,
caught at 300 m after a period
Note rll~t
iri: this situation the elevator starts from rest;
of 20 s. The graphical solution is 1so·· I
.._ ·. so initial velocity of screw is zero. Time of fall of screw was i
depicted in Fig. lE.75 (b).
independent of the speed of. the elevator as long as it moved· 100 1-
with constant, velocity.. When the elevator accelerates, we
can say .. that screw experiences effective acceleration ==:;==:;::->;.,'t(•)I
At1 412
y' = g .j.' 'o./.
If acceleration of elevator af = - /J' the time of ~~-F~ig~.1~_{1;!_,)_-=--''-'-I
I

fall becomes.1nfjnite, i.e., it appears to be weightless. ·


www.puucho.com
·-·--·-
Anurag Mishra Mechanics 1 with www.puucho.com

DESCRIPTION OF MOTION

course, we could have checked this at the outset, but that's


hindsight.
This is a good point to look at the graphical solution.
;rwocars approach each other.on a straight road. Car A moves] Fig. lE.76 (b) shows that parabola (i) stops at 8 s, whe~ea,s · ·
!at 16 m/s and car B moves at 8 m/s. When they are 45 m
llapart, both drivers apply their brakes. Car A slows down at parabola (ii) stops at 2 s. From them on the graphs are .
2 2
.2 m/s , while car B slows down at 4 m/s • Where and when' horizontal lines. The solutions 3 and 5 s are the intersections
l4q_ th~ collide? . of the two parabolas. These would have been physically'.
acceptable if the accelerations had remamed constant in
Solution: Fig. lE.76 (a) is a simple sketch of the
both magnitude and direction.
situation. We choose the origin at A's initial position and
point the positive axis in the direction of its velocity. The graph helps us to find the proper solution. We must
find the intersection of the horizontal line for B and the

-Lx .. ~
parabola for A. Let us find where B stops. At t = 2 s, (ii) gives ,
Xe= 45-8(2)+2(2) 2 = 37 m
~ - --c~I
~
aA<F=
voA voe'
~ aa
-
, xA
B stays at this position until it is hit by A. The condition
= Xe becomes
<•l I 16t-t 2 =37
X (m) '
I Thus, t = 2.8 s; 13.2 s. We reject 13.2 s since there can
50 : ,..,
(i)
' be only one collision. The collision occurs at 2.8 sand 37 m.
__,,
~iE.~qm~

,.

0 3 4 6
(b)
~---~Flg.1E.76
Carefully check the signs. The cars meet when x A = x e,
so we set up general expressions for these quantities using
1 2
x = x 0 +v 0 t +-at :
2 R
2
XA =l6t-t ... (i)
2
Xe =45-8t+2t ... (ii)
When we set xA = Xe,
We find 3t 2 ~ 24t + 45 = 3 (t - S)(t - 3) = O
'-----·-L__ ig_._1E_-·_11_ _ _ _ _ _ _ __
F.. I
, ••• I

Solution: (a) Length of the chord P1P2 = 2R cos 0


We seem to have two possible times for the collision: Acceleration of bead along v,,ire = g cos 0
t=3sandt=5s. •
From equation, v 12 = vi2 + 2ax
Try to find the flaw in the above argument.
Let us look at the velocities to see what has happened? = 0 2 + 2(g cos 0)(2R cos 0)
Att=3s, or vt =zJiii. cos0
VA = VoA + aAt = 16+ (-2)(3) = 10 m/s (b) From equation, , v t = V; + at
Ve =Voe +aet =-8+(4)(3)=+4m/s = o+ at
Does this give you a hint of the difficulty? v A shows
or t = Vt = 2Jgii. cos0 = 2,/iiJi
nothing unusual; A has slowed down. But look at the sign of a g cos0
.
Ve· We seem to have found that when the brakes were
applied the car reversed its velocity. You can easily verify which is indepe11dent·of0; thus time of travel along any
that B stops at 2 s and then stays at rest. This means that (ii) chord is same.
is not valid after 2 s. Sinillarly, (i) is not valid after 8 s. Of
www.puucho.com
Anurag Mishra Mechanics 1 with www.puucho.com

[68 'O_J MECHANICS-I. I

IA car· is speedi!ig°at 25 m/s. in-~-lli;~pe~d-~one. A poli~e car) IA spaceship launched vertically from_ Mars has recicl,~d a
/starts fr~ll}·C~tljust as the sp,eed~fp~ses and ilccele;ates at~ !heig~tof30Oin and a velocity of:80 in/sat time t ·=; 0.f'.tthis
,constant:rate
! .. -. ""
0
of 5 m/s 2 • -
. ' .
,,__


'
,
'
,
C 0
\i!JStant its controls are switched. off. It continues ·to move
(a) When does the police car ca.tch the speeding, car? upward under the influence of Martian gravity,
'(bj' Howfast ts·t1tep'o1ice car trav~lling when it catches up approximately equal to 3.72 m/s 2• At the same instant
!' with the speedet?· -· · . ,anothrr spaceship at height 1500 m is moving downward at
· :(c) How far have the cars· trave(l_ed when the p9lice cari ,25 m/s and slowing down at a rate of 0.80 m/s 2 •

"'I
_
L __ catches the weeder? · ___ . _ · _ _______ _. __ _
Solution:. (a) For speeder, x, = v,t
r,J first
,
1
= 1/re - ~ ""™ """"'j,,,m M,,i fr'
Wh,ntime?
(b) How high above the planet's smface will the first meeting
time (t),.,l
: !
I(cL_takqil_ace? . . · · . .. ' ,. . , ·
What)s the yeloatyyf eac/J;§pacesh1p when they meet?
.--~~---: j
L~-~---.{ \ Solution: (a) We write equations for positions of
"·rn-, - ..1,
:l
.,-.,,-----i !
spaceships 1 and 2.
\..!:~.r--·vO = o· : ______ : 1 y 1 = 300 + 80t - l.86t 2 , y 2 = 1500- 25t + 'o.4t 2
·
---'----"-~...:..:._ _.;•, i
., x.=i::•.
_-. ap=5m/_s~ !

--·-·· - (2) -t.


_ _;._-"----=----'-----~-.

-T l
w, "_ '(1)
,,,_g u,=;=-2S'm/s
' ..- : a::: 0 _8-m/sZ•.---=- ~ u,';: 80 mis
Y 'Yo= 300m
:+ve 300m , a =-gM =-3,72.mls 2
'--L--~-'--....'-__.x
Surface of 111.ars +v~

For police car, x P = .! ap1 2 a= O:B m1s2


.2
At time t both the cars are at same position x, = x P
''
300m -
' 1 2
:-:
'·.:·1'
vt=-at
s 2 p .
Fig, 1E.79
t= 2v, = 2(25) =lOs ---------
aP 5 (b) . When the spaceships meet,

(b) Velocity of police car is given by Yi =Y2


300 + 80t - l.86t 2 = 1500 - 25t + 0.4t 2
VP= apt= (5)(10) = 50m/s
(c) Distance moved by speeder= v,t = (25)(10) We get two solutions, t 1 = 19.0s and t 2 = 27.4s
=250m (c) The two spaceships meet twice in their journey. We
Remark:----------------- require y 1 att 1 •
Distance covered by t~e two cars is same; hence they must have Y1 = 300 + 80 x 19 - 1.86(19) 2
same average velocity. If the police car waits for I =2 second or
= 1.14 km.
t =4 second, it will catch the speeder after c9~ring a larger
distance as shown by dashed line in graph. ---....._ Spaceship 2 first meets spaceship 1 when the latter is
moving up and a second time when it is moving down as
shown in the graph. Students are suggested to interpret the
curves for a= l.Om/s 2 and a= 0.4m/s 2 •

www.puucho.com
Anurag Mishra Mechanics 1 with www.puucho.com

I DESCRIPTION Of MOTION
~exct_!E.~~fsot;> = (v,; i + Vy.j) + (ax i + ay j )t
...v ·;:::: ...vr + ...at·
Ball 1 is released from the top of a smooth inclined plane, an~ 1
1 2
at the same ins tan~ ball 4 is projected from the foot o};the' Simllarly, Xf =X·+V
l XI-t+-aX t
plane with such a velocity that they meet halfway up the 2
incline. Determine:
1 2
Yi =y-+v -t+-a
2 yt
- - - - - - - - -1 l yt

i We may write the above equations for final po_sition


vector.

li
hi
-+
r1
__,
-+
= r; + v, t + -
A
---+

A
1-+
2
a t
2

!: ~ Yd
where rr =xii+
... A A

4.LJ"'-------' vi =v.ni+vyij
Fig. 1E.80
! J
...

a= ax
A

i + ay j
A

--·'
(a) the velocity with which balls are projected and · For a particle moving in a plane we may write the
(b)Jhe velocity of each ball when they mee,,,,t.~------ following equations (note that we have assumed initial
Solution: (a) Accelerations of both the balls are position to be origin, i.e., xi = 0, y, = 0).
a1 = g sin0 and a 2 = - g sin8 down the incline.
Ball 1: .!. =(0)t + .!. g sin8t 2 ... (1)
2 2
Ball 2: .!. =v- t + .!. (-g sin8)t 2 ... (2)
2 ' 2 V,;f =V,; + a/ Vyt =V_y;+ Of.
Adding eqn. (1) and (2), we get vJ = vJ + 2n,(x1 - x,) v.J.=;; + '2il,0/1-Y1l
1
l=v,t or t=-
v,
Substituting it in eqn. (1), we get
2 -- ---------- - - -·-------.------- ---------
i2 =.!.2 g sine (_l__)
[
-+ ,.. ,A '1

v, The velocity of a particle at t = 0 is"u. = 4i+3j m,/sec and aj


-+ ,.. A' . • I
or v, = .Jg1sin0 = .Jii1 _constant acceleration is a= 6i+4j m/sec 2 • Find the velocity:
(b) From equation, vJ = vf + 2ax @!d disp)acement_ofihe JLarticle at t = 2 sec. ___ . _ _ j
... A A

For ball 1: vf = O+ 2g sine.!.2 Solution. Since a= 6i+4j=constant


.........
Therefore using =U+at
v 1 = .Jglsin 8 = .Jii1
V
or -+ ,...,... "'"
For ball 2: v~ = "f + 2(-g sin8) .!. V = 4i+3j +(6i+ 4j) X 2
2 ... A A

= g1sin8-g1sin8 = 0
=> v=16i+lljm/sec
The displacement is
or v 2 =0 -+-+ 1-+2
Two-Dimensional Motion with Constant Acceleration S =Ut+-at
2
1. The position vector for a particle moving in the A A 1 A •A

,y-plane is given by =(4i+3j) X 2 +-(6i+4j) X 4


... A A 2
r =xi+yj
Velocity of the particle is obtained by
... A A

v=vxi+vyj
Because "i is assumed to be constant, its components ax 'The accel_erqtio.n of a moving body at any_ ·me 't' is giv_en_ by: u_

and ay are also constants. Hence we may apply equations of -+ " 2 ,. 2 '-+ ' • ' !
• If ti =·0thenfind the velocity,
a= (4t)i+.(3t. )j · m,/sec
kinematics to the x and y-components, e.g.,
...

Vt =(V,; +axt)i+(v_y;+ayt)j
A A t .
oftheparticleaJ4:sec. _ ________ .. -
.
. ,
I

www.puucho.com
Anurag Mishra Mechanics 1 with www.puucho.com

MECHANICS-I 1
..., A A

Solution: Since a= (4t) i+(3t 2 )j is time dependent


..., ..., ...,
· therefore we cannot apply v = u + at
We can solve by applying calculus

Thus
dv
-=a
...,
..., .iuti=
...,
2xl = 1;
dt

fo
;:: -+
f 4 A

(4t i+3t 2 j)dt


A ' ~2 ~t 21= ~2 X (2) X. (1)2
or dv= O I
=1
=.{:J \dt }1; {3Jt dt }J 2 ½°BF ....................... :""'>
...,
0
A A
0
1;1= ~c1) 2 + c1) 2 '' .:
v = (32)i+(64)j rn/sec =..J2 m
Equations of Motion in Vector Form 2
.
~~t J·
The equation of motion in the case of a uniformly tan(j>= l2..., 1 $ iut
accelerated motion is written in vector form as-
..., ..., ..., -+-+ 1-+2
lutl Fig. 1E.83 (b) .
(i) v.; u+ at (ii) .s=ut+-at co} =45°~
2
-+ -+ ~ -+ -+ -+ Displacement is at an angle 45° with the direction of
(iii) V·V = U·U+ 2a· S initial velocity.
· ,--'(Application only when acceleration vector is constant, motion
' • may be along straight line or along curved path.) '

·..+iam~ii 03 ~ I particle is moving in xy,plcine•. :At certain instan~ .the


compon~nts·of its. velocity and'acceleration ,are:as1o(lows
'fA_ pCl!tide 0J'.n1~s 1 kg has a~ velocity· of2171/sec. A cons~ant Vx = 3 mis, Vy = 4 m/s, ax = 2 ~ 2 and ay =.1 mls 2'. The
if9rce, of 2N acts;on· the. particl~fer l sec in a direction rate.of change of speed at this moment is ,l' r·
1p'erpe~dicul11",;.:'.t~ .1ts initial ve/qcirg.. Find. the velocity:r/nd (a) 4 m/s 2 , , • · (b), 2 m/s 2 · · ,
' fdisplac~mentoj't/t'e particle at th~ end ofl sec. ' ',, ,' ' (e)_../5_rIJls 2 · (d):,..g ws~2 - ~ _ , _ _ _ ,
Solution: Solution: vx = 3m/s, vy = 4m/s

Acceleration of the particle a = m = 2 rn/sec


F 2 ax = 2 rn/s2, ay = 1 rn/s 2 ,""f"B/~;.. ,
-+ ,AA-+ AA

V = 3i+4j, a= 2i+lj ~u
Here acceleration is constant therefore .
..., ..., ..., ....... ,··acos8

v=u+at V· a =10 , Va COS8 =10 < , •• "'


Flg.,1E.84
')<

..., ..., 10 10
Since v and at are perpendicular. acos8=-=-=2
V 5
-+. -+ ' -+
'Hence !vi=· iul 2 +l.atl 2 ,
Rate of change of speed = a case = 2 rn/s.
... ~. . . V" Ee±xgtmtjr->'I~~ 85 ~
where ju!= 2rn/sec;
..., at r:··i;:····--,;··;··: i ' ------~-~

'·.
i

' •. " 'I ••. -;,:

latl=2Xl . The figure shows the velocity and 'the acceleration t of' a
r point-like body at the· initial moment of its motion. _The
... =2rn/sec . 'direction. and. the absolute value of the acceleration rem1iin
lvl= 2..J2 rn/sec constant. Find,cthe time in secona.fwhen the velocity reach its .

8 = tan-1 Iat!
... F:~. 1E.83 (a) '.u I minimum value ?
(Data : a= 6m/s 2 , v 0 = 24.m/s, · q, = 143°)
or ...
iul
1
= tan- (1) = 45°
Hence the velocity of the pa1ticle after one second is 2..J2 ,,
rn/sec at an angle of 45° with its initial velocity. Flg.1E.85.(a)
•'---..C..·- - - ' - - - - ' - - - - - - - ' - - - - - ' ~ ' - · - '
For the displacement equation of motion is
''
www.puucho.com
Anurag Mishra Mechanics 1 with www.puucho.com

'.DESCRIPTION OF MOTION
1--··----
Solution: x-components y-components 2. Projectile motion: An object is flight after
ux = -v 0 cos37° uy = v 0 sin37° being launched or thrown is a projectile. We assume that
the distance travelled by a projectile is much smaller than
ax= a ay =0 the radius of the earth so that the acceleration due _to gravity
Vx =- 4vo +at 0 3v
V =-- is constant; secondly air resistance is negligible.
5 y 5
y

/'i:'--"e!----Vo' a=-gj
A

a
..-·
i Fig. 1E.85 (b)

when vx =0
t = - x - = 3.2 sec
4 24 f-'---.----------,---.-lt-r_.,.,.... __ •
5 6 a ~xi a
lliustration 10: Consider a ball initially moving along
the x-axis as shown in Fig. 1.82 (a). At time t O it gets a
constant acceleration ay in the y-direction. At any time 1 ';'
lfyJ=-Vyli
t > t O the x- and y-displacements are
1
y=.-ayC
2 - -·· ----- - - - .. !~:!·~_(bl _______ _J

2 Fig. 1.82 (b) shows the path of a projectile with velocity


The superposition of these displacements is a curved vectors. Let the launch point be (x,, y ,); y is positive upward
_, and x is positive to the right. Projectile is launched with
path. The total velocity vector v at any time is tangent to the
initial velocity vi at an angle e. Since motion of a projecti)e
curved path of the ball. Velocity vector is an angle e relative
takes place in a plane we will set up equations for x- artd
to the x-axis, given by 8 = tan- 1 (vy/vxl, · which y-components separately.
continuously changes with time.

,,.1,:2 y
~:~¼ /' Vx
"x = 0
vxf :::vxi+ Cl;.:t

=V;COS8; ... (1)


a,=-g
V.>f =Vy;+ G/

;::; vi sin ei-gt ... (4)


.... . IJ -~
1 2 ' ' ,1 •
V2 Xt=X-+V
l X1
,t+-n•
2 "".r" Yt =.)'(+v_;.,'.-~'2af
",, = ., t, '. -~

= x, + v, cos8,t · ... (2) =y,+


- •.._:-- . .
v, sme,t--gt
1:, 2
I
... (5)
Y2 =½ay t~ ---------------·.-- v 2 ..
.t:''
' '
X

v;, =v~ + 2a1 (y 1 - y 1)


....' :
1 2
Uy1 = ay ti(,,/ l 81
V1

:
Y1 = 2aY t1 ------·.: '., v ;
V ,• •1 X ' 3. Problem Solving Strategy:
Q-2..+,,..... ..• : X
Y = 0 ta Vx X1 = Vxt1 X2·= Vxl2 X3 = Vxt3 1. Imagine the situation of problem; draw a picture
which shows the object and its possible trajectory.
Strajght-line Curvilinear
2. Choose a coordinate system, choice of origin is
motion +av motion
vy=O L.:... arbitrary. Generally point of projection is assigned the
origin. If range is to be calculated along horizontal
"x
atto=O level, take y-axis parallel to acceleration due to
gravity. For calculating range along incline take x-axis
parallel to it and y axis normal to it.
0

Flg.1.82 (a) ____ _

www.puucho.com
Anurag Mishra Mechanics 1 with www.puucho.com

MECHANICS-Q

3. Identify the initial position, irntial velocity and


acceleration. If irntial velocity and acceleration are
not along assigned x- and y-axis, then resolve them ,·--------------------·----- ---------
IA bomber ii; moving horizontally at a speed v 1
= 72 m/s at a
into x- and y-components.
height oJ;h = 103 m. An enemy ta_nk is moving horizontally
4: Identify the unlmown quantities and assign them (x,axis) w;t6
constant speed. At the instant .the. bomli is
letter symbols u, v 0, a, etc. released a '.tank is at a distance_ X a~ 125 m from origin;
5. Make two set of equations, one for x-components and Origin is directly below a bomber at the instant of release ofij
one for y-components. bomb. Assuming the tank to be 3 m high, find the velocity v 2 ,
6. Try to identify special condition of the problem that and. the time of fl.ight of bomb.
~----·-·------ -------7
may define the object's position or velocity at the l V1 ! I
point of interest.
7. After writing appropriate kinematic equations in
I - I
I
\
I
I
component form solve them.
8. Always remember that time of flight is same for both i
component~ of motion. '
l L-C'---' "'--~-"
I
\ • •

·ekam~'. a6 - ~ L-~---..,.l.~ i g . 1 E : ~ - - - - - - _ J ___ ,.__I


lt:footballer throws d. ball from a height oJ 2.00 m 'aboye_ the Solution: Initial velocity of the bomb will be same as
Iground with an initial veloc_ity o/20. 6 m/s at an gngle pf 30° that of bomber
jabove the hof_fzont~L (a)How lonq,does the ball taki-wcro~ vxi = v 1 = 72 m/s;
[the goal (ine l2:0mfrom the po~n~of_releas~?{b) Wh~tis the For the bomb:
IJigll's heyghLabove·the grftundASJt cro§ses the.goal lute? -·
Solution: We have to find timet when x = 32.0m. In
the second part we have to findywhenx = 320m or we can
say, find y at the time it crosses the goal -line. We take origin
at the point of release, xi = 0, y i = 0. When the bomb hits the
~325m .. tank,;
f;eom110nents cy=3m ·
2
3=103-(4:9)t
. - 1 2··
Y• =Y; "': Vjif ~ j a_,t 2 _ 100
t -- -, t =4.52s
4.9
=9 + Vo-.,,,,Sill ~J-f gt
;
2
·:
' . For the tank:
_ X - X; _ 325 - 125 _ / i\
- 44 . 25 m S
. -. -" f26:o}J@30°'d.ssJ; .•.
V2----
,· ;, ''. · 1 ' ,_:. ' t 4.52
• ". ·--- >< (9.8)(1.85)2 ,cC'.
(, -" .·2-~ ,; ,:,~,- .. -.. ·,~,i
=1.73m>
@exg~~~
i---~~-- -'-,- ---- - -- - - - - - ' -
1Aboj throws~ ball with v;loqty v0 =10,/2 mis.at an angle
, X,=v~t
t=~;.
------t=?
y=?
of 45° as ·shown in the figure.'After collision with the ball the
vertical component of ball's velocity is unchanged and the
Vx ',
rv~---"---,--,---'x horizontal.i:dmponent is reversed in d(rection. Where do~ the
, 32.0 32.0m
=--
17.32 _
="1.sss' ~.
ball hit the ground?
, r -,-:,
··, · ·

•.. -----··-··-.._.:·.-
·

'. "S'.,..
'- .. ··
·~o-/2m/s -.
· We have assumed y = 0 at 2 m from ground. So height
of ball above ground is 3.73 m.

l .,_.,;.... .. ,
_,, Fig.1E.88
-~-'----'-----·--'-------~
~

www.puucho.com
Anurag Mishra Mechanics 1 with www.puucho.com

.. . -· .• .. ... - --- . .. .. . ... .I


P>ESCRIPTIO~ ~-~~OTI_O_N______ 73:
---------- - • · · - - - - · · - - ,1

Solution. We divide the problem into two parts, i.e., For Policeman:
motion from boy to wall and from wall back to ground.
(i) Motion from boy to wall
X·GOl!IP0111ll)ls .. x=v;t ... (1) 1
y=O--gt 2 ... (2)
.'Y"//'l,,Y"·<··
= (5) (0.782) 2
v0 cos45°_t,= 4 m y=v0 sin8t-½gt
2
I = 3..91 m
-3. = - _! (9.8)t 2
2
t=--'--
Vo COS
4
45°
,.IO..J2 -~ _.!_ X10 X
.Jzsz
(~)1
sJ1 _t=0.782s
4 16 I
=-m Since horizontal distance by policeman is less than 4 m,
10..J2 ·(l/./2) 5 the policeman fails to cross the gap.
2 Ball strikes at a height of 16/5; For Thief:
=-sec
5 tnfrom _ I
boy's hand. · ~;;;;;:;;--;...,c-;-:-;-;-;-;-:--;Y;:'C;:;O:;;m;;:p~o:;;n:;;e;;;n;;;fs;--'_.•·G7
vx = v0 cos45° =.10 m/s
vy =v 0 sin 45° - x =vnt =Vicos0t 1 2
gt
We can find horizontal dis~ y =V_,.;t + ZU/
1
10./2 2 tance- travelled covered for
= ---"7,;-10 X :- =6,n/S
-v2 5 i positive value_ -Oft. . 'f '2 }
-3=v-sm8t--gt
X = (5) COS 4,5° X (1.22)
' 2
(ii) Motion from wall to ground
We choose point of impact as origin and upward positive
=4.:nm ,or 2.gt
l 2 -vism
• 8t-_-3 = 0:

x-cofup8iiehts
<..-,".¼"Fl<'.L·,
··
v1sin 0
t= ~
-265 =6t' _!2 gt' 2 g

where t 'is time of flight.from waii


i
± ]:_
------'
Jcv, sin 8)2 - (-6)(g);
_ l0./2 1 78
-..J2X. to·gr~und l g .
2 6
- · · = 6t' _..!_ x lOt':(
=17,8 m =-0.Ss or t =1.22s
5 2
i Hence the ball lands l7.8 or St'2-
26
6t' - 5 = 0
i from the foot of the wall .
, 6±~36+4(5)(26/5)
ort:
10
,- --- ., .. ··-- - '.

t' = 1.78 sec (neglecting -ve va!u~) !A helicopter is flying at 100 m and flying at 25 m/s at an.
1angle 37° above the horizontal when a package is dropped·
!from it.
~a) Where does the package land? ,
- . .. ··-·---.- -- . . , -----~ - - - - · . --·-·-1
(b) If the helicopter flies at constantvelocity, where is it when,
A policeman is in pursuit of a thief Both are running a_t ;il 'i the package lands?
·m/s. Suddenly they come across a gap between buildings asj I - -- ____, - - .-

~
:shown in figure. The thief leaps at 5 m/s and at 45° while the'
'policeman leaps horizontally. i I
'
i(a) Does the. policeman clear the gap? j .
!(bJ By ().QW _mµch _does.the thiefelegr_.th&gap? ___ . . i
Solution: We choose original launch point with
upward direction positive.
- -·- -- -- '"' '"'" ·- -------
!
i
I
I ·.· . Fig.1E.90
L----'---·-----· ·---·-·----· -··-
Solution: Due to inertia of motion the initial velocity
of the package is the initial velocity of the helicopter. We

www.puucho.com
Anurag Mishra Mechanics 1 with www.puucho.com

MECHANICS-I

choose origin to be directly below the helicopter, i.e., X; = 0,


y 1 = 100m.
y-components A bullet with muzz/r ··•lncity 100 m/s is to be shot at a target
30 m away in th, 'wrizontal line . .How high above the
X =:Vxit -
Y-Y; +v_nt +,ay
I t2
•target must the re .• aimed so that the bullet will hit the
=vi COS 37° X t 11=100m+ (25)(sin 37')t 'target?
:Note that in order to calculate - _!c (9.8l)t 2
2 u ... ··
range we must know t. Which
!we can calculate from equations = 100 + (lS)t -4.9t 2

'of y-coordinate. ~;::::.::'.=-=i31DOrrm;::=~ Traget I


y = 0 at t = 6.30 s
(: =_(2_0_'.~-~:~fu)
= 126 m and t=-3.24s
Fig.1E.92
l l .. We choose positive time
;_,. :' t = 6.30s Solution: Horizontal range of bullet is 30 m.
',I -~3"'.2~4-s--+----..6,-,3!-:0-s••
..t:____ . 2 • 20
u sm = 30
For helicopter: g
y-components· · . .,,, 30 X 10
. i'
or sm = = - - -
(100)2

or sin 20 = 0.03
,; (20) (6.30) =100+(15)(6.30)
For small 8, sin 8 = 8
= 126m = 194.Sm i.e., 20 = 0.03
Therefore 8 = 0.015
Note that the negative time indicates the time when the
package would have been if its motion had started earlier. The rifle must be aimed at an angle 8 = 0.015 above
horizontal.
~f[:~,ilijpJ~i;J917;> Height to be aimed = 30 tan 8 = 30(8)
= 30 X 0.015
'A particle is projected from the origin in such a way that it = 45 cm.
passes through a given point P (a, b} What is the minimum
rN!J.ir:ed,. sp~ed_ to. do _50;> Concepts 1. Relation between maximum possible!
Solution: Equation of trajectory of a particle is
·range and greatest height for any angle 0 !
2 Rmax is attained for 0 = 45' I
y=xtana- 2
gx
2 u'
2u cos a Rmax'=g
If projectile passes through (a, b),
H max is attained for vertical projection i.e., 0 = 90'
ga 2 ga 2
b=atana- atanu---2
(l+tan 2 a) u'
2u 2 cos 2 a 2u H max =2g
-
or ga 2 tan 2 a - 2au 2 tan a+ (ga 2 + 2bu 2 ) = 0
Rmax ;;::; 2Hrmrx-for same ' u 1•
This quadratic equation in tan a must give real roots for
2. If H 1 maximum height for the angle of projection 0 and:
a particle to pass through (a, b).
H 2 maximum height for the angle of projection;
Thus Discriminant ~ 0
~~-~ I
i.e., y I
4a2u 4 - 4ga 2 (ga 2 + 2bu 2 ) ~ 0 :
!
or u4 - 2gbu 2 - g 2a2 ~ 0
or u4 - 2gbu 2 + b 2 g 2 ~ b2 g 2 + a 2 g 2
or (u 2 - bg) 2 ~ (b 2 + a 2 )g 2 x\
!'
~ ~bg + g~a 2
Flg.1.83
or u + b2 -- - -- ------ -- -- -·

www.puucho.com
Anurag Mishra Mechanics 1 with www.puucho.com

IDESCRIPTION OF MOTION
'--------------------- - - - - - - - - - - - - ---- - ---------
- ----- ------- 751
. - - -- ----·-- -- - -
In both the conditions the magnitude of velocity of, u,
A • :ti;;_··-..

l ..
I V
projection is same so horizontal range will also be' .. ,v~'--;_-o:;_-···>
same and let that be 'R' then "2 ..... •....
R = 4~H1H2
and Hi = tan 2 8
hl ·-. .·.'.' . . . .~;--..._!:
H2 .
... \ . ....
3. Relation between horizontal range andl : \ \
maximum height
2
I Fig. 1.86
u sin2B u 2 sin 2 0
R - - - and H=--- '
g 2g i
R = 4Hcot8 ... (1) ;
Iwo particles A & B are projected from the same point in,
y
different directions in such a manner that vertical
components of their initial velocities are same :
(a) Find ratio of time of flight
(b) Find ratio of range.
,Y
-----R---+ X

Fig. 1.84

If the range of projection is n times the maximum


height of the projectile, then angle of projection is
given by
Using equation (1) Fig.1E.93

1
8= tan- (;) • 2u 2u sine
Solution: (a) Time of flight= _Y_ = ---
g g
. 4. Relation between angle of projection e and angle of
elevation <jJ of the highest point of trajectory from the So, time of flight same for the two since
point of projection _ 2v A sine A T 2vB sin Sn
T1 - , 2
H u 2 sin 2 8 2g g g
tan<j,=-=---x-~-
R/2 2g u 2 sin2B
1
tan<j,=-tan0 (2usin8)
2 (b) Range = - - - x u cose
5. (i) If t AD = time interval to travel from A to D g
UA sin8 A = Un sin8n
t BC = time interval to travel from B to C
RA (2uA sinSA)(uA cosSA)I g
2 2 8(h2 - h1l - = ..:....-'-'--'-'-'-'-"-----'-"--"-
(tAD) -(tBc) Rn (2un sin8nun cos8nl/ g
g
= [UA COSSA]
Un COS8n
,h2-h1!/ B~C '\ RA = sin Sn cose A = tan Sn

. r ---" Fig. 1.85


Rn sine A coseB tan0 A

(ii) Consider three projectiles first is projected at e angle:


above horizontal, second below horizontal and tfiird! :Four cannon balls, S, T, U and V are fired from level ground. '
horizontally. 'Cannon ball S is fired at an angle of 60° above the horizontal
If lu 1 sin0 1 l=lu 2 sin0 2 I and follows the path shown. Cannon balls T and U are fired at
,an angle of45° and Vis fired at an angle of30° a~ove'the
Then . t 3 _=J_t 1t 2 ___ _
:horizontaL Wl:!(clt_s:Cinn_on l!all has the largest initial speed?

www.puucho.com
Anurag Mishra Mechanics 1 with www.puucho.com

176 MECHANICS,! j
2

~I
y =(tan0)x-( g )x
2u 2 cos 2 0
The equation obtained is of the form
- _., .
y =ax-bx 2
---~---- Fig. 1E.94 J which represents a parabola, so path of projectile is
parabolic.
Solution: Expressions for range and maximum height Above equation of trajectory is applicable only when
u 2 sin28 u 2 sin 2 0 motion takes place in a particular plane (say xy-plane) and
are given by R = - - - ; H = - - -
g 2g point of projection is origin. Angle 0 must be measured from
-From figure T, V have same range positive side of x-axis and the only acceleration in the flight

sin 2 90°= sin 2 60° u~ must be constant and in the -ve direction of y-axis.
The above equation can also be expressed in terms of
Thus u¥Cl) ~ u}(¾) range 'R' of the projectile
2 y = xtan0 :._ . gxz tan0
or uv = ,Jgur or uv > ur 2 2
2u. cos 0 tan 0
Now we compare U and T, (Range)u < (Rangeh y = x(l---gx~_cosO)tan0
As projection angle is same uu < Ur 2u 2 cos 2 0 sin0
Now we compare Sand T, Hs = Hr
y=x(1-~}an0
u S2 sin 2 60° =_r
u2_ 2
sin_ 45°
_
2g 2g I Conc.-,pts: Application ~f--e-q_ua
___ti_o_n__o_'f_tr_a1-·e_ct_f)__ ry--_ "'I
or u 5 =urx~ or Ur>us
I 1., Co~ider a proJe~tile project~d from _a, step/ofi
height h, after following a parabolic trajectoJY it'1ands, .
Thus, Uy > Uy, Uy > Uu A deptMh' below the horizontal plqne ofprojection; tlt~n-
and ur > Us ~y
Therefore. Uy is m'!l{imum.
. '
Equation of Trajectory
Trajectory refers to path followed by a particle. Equation
of · trajectory is obtained by eliminating time t, from
"!'pression for x-and y-coordinates.
~J -

.........
id!...................
--

'
.. ·

' :·
1. -. , .-. . .
h
A

••••••••••
-

x = (u cos0)t
y = (usin0)t _.!_gt 2
... (i)
... (ii)
? ... . I_ - ....
J -

2 Fig.1.88
After eliminating 't' from equations (i) and (ii), we get The time, of flight can be obtained from expression't~r y
- • X 1 X2 i.e., time in which y is equal to -_h. · ·
y = (usm0)----g~~-
u cosa· 2 u 2 cos 2 0 • _- 1 2·
y =U t+•-a• t
- y 2 ~
_, • ' 1
-h a;_(usin0)t--gt 2 .
' --
i1 __..,.:------..'""'P--(x,y) ½gtf":(usin0)t-h ".' 6,
' ;. ' 2
'

~,
Eroduct of the roots-
y- ' - 2h
-e t1t2 =---
--- ----------···········-····-··
.. g
I
X Since product of the roots is neg~tive, so one of the roots
L__ . Flg.1.87 is negative other must be positive.
'--''--------'---~--------~
www.puucho.com
Anurag Mishra Mechanics 1 with www.puucho.com

iDESCRIPTION OF MOTION -
L __ "" _ _ _ --- - - - - - - - - -
------ -------- - -
'
, Let negative roots be t 1 then it represents time for part co' 0=tan-1 a and
1
cos0= r;--;,
'if motion did exist before t = 0 and positive root t 2 represents vl+a 2
time for part OAR.
-~f--=b ... (iii)
2.If a body is projected from the ground so that on its 2u 2 cos 2 0
'Wey it just clears two vertical walls of equal height on the
,ground then with the help of equation of trajectory we can
,determine range of projectile.
y
U=~i
2
U=~;(l+a )
(b) In case of ground to ground projection
horizontal range is the x-component of displacement when
the y-component of displacement becomes zero.
Putting y = 0 in the equation of trajectory, we get
x,- X
0 = ax-bx 2 => 0 = x(a-bx)
- - - - x,,------ x=0 and x = (a/b) = R (horizontal range)
- - - - - X2------ Maximum height is the maximum value of y-component
of displacement, which is obtained at x = R/2
Fig.1.89
Putting the value of' x' in the equation of trajectory we
Path of the projectile grazes the top of the walls, so get-
coordinates of the top point of the wall must satisfy the a2 a2
equation of trajectory. Ymax=2b- b
4
y = xtan0 g x2
a2
2 2
2u cos 0 or H = 4b (Maximum height)
h =xtan0- g x2 I ·----- --- - - - - - - - - r:::::7-
2u 2 cos 2 0 : ,~~~·~~J<1l~,1 96 I ~
If x1 and x 2 are roots of this quadratic equation.
A gun is mounted on a plateau 960 m away from its edge as
x1 - x 2 = d ( difference of two roots gives desired relation) shown. Height of plateau is 960 m The gun can fire shells
x1_+ x 2 (s_UTI! of the roots) gives horizontal range. with a velocity of 100 m/s at any angle. Of the following
choices, what is the minimum distance (OP)xfrom the edge of
i-gxg,ijlil~j 95 [> plateau where the shell of gun can reach?
I -
A particle is projected from origin in xy-plane and its equation
,of trajectory is given by y = ax - bx 2. The only acceleration
' -----960m
- -

960m
;in the motion is' f' which is constant and in -ve direction of,
'y-axis.
( a) Find the velocity ofprojection and the angle ofprojection.
(b) Point of projection is considered as origin and x-axis:
0
---.--
Fig. 1E.96 (a)
along the horizontal' ground. Find the horizontal range:
and maximum height of projectile. Projectile completes its (a) 480 m (b) 720 m
flight in ho~onta_l p_~ane of projection. _(c) ~60 m _(d) none
Solution: (a) Given equation of trajectory is Solution: P is point on
y = ax - bx 2 ... (i)
plane for shell when it passes
near edge of plateau for greater
Let' u' is the velocity of projection and 0 is the angle of angle of projection. In region AP
projection, then the equation of trajectory is given by shell cannot hit
y = xtan0 f x2 ... (ii) Xmax = AP
2u 2 cos 2 0
Comparing (i) and (ii), we get Equation of trajectory for
tan0 = a projectile
fig, 1 E96 (bl
www.puucho.com
Anurag Mishra Mechanics 1 with www.puucho.com

MECHANICS-I I
2
y=xtan8-gx (l+tan 2 8) £.o6~Qc..l'.J~]?~J~;>
2u 2
For point E,y = 0 and x = 960 m IA~;oj;c:i~e isfire~-~th·~:;:c_i_ty_v__fr_.-om_a_gu
__n_a_d_if_us_t_ed_fi_or2
2 0
=;. tan
2 S 10 X (960)2
x ---''------'c-- ( S) 10 X 960
- tan 960 + ----=- =
0 imaximum range. It passes through two points -P and Q wlio]e1
2 X (100) 2 2 X (100) 2 jheights above . the horizontal _are h each. Show that, the
=;. tan8=3/4,4/3 ,separation of the two points is·!l..Q.~v~ -4gh.
for tan8 = 4/3, AP is least I ------·- g -·----· .
Now, equation of trajectory becomes, Solution: The trajectory of projectile is given by

Y =ix- 2~~0( 2:) y = xtan 8- gx (1 + tan 2 8)


2v~
2

for landing point on plane (P), y = -960 m


Gun is adjusted for maximum range; therefore o. = 45°.
4 x2
=;._ -960= - x - - x2
3 720 .y=x-g,
solving we get acceptable solution x = 1440 m Vo
AP= x-960 = 480m g . 2
Fory=h,wehave h=x--x
2
Vo
tLExam~~~e;:f977~~
- --~.....___1~ 2 2
or -x 2 -v 0x+!l..Q.h=O
!Jwo particles were projected one ·by ~;~ with the same initiail g g
1 velocity from the same point on level ground. They follow thel If x 1 and x 2 are roots of the above equation,
same parabolic trajectory and ar.e found to be in _the same v2
horizontal,lev~ts~parated b:y a distan~e.oflm, 2seconcis affy -X1 + X2 = _Q_
the second particle was projected. Asst{me that the· horizontal g
comporie'nt of their velocities of 0.5 m./s.
and
Which of the following s·tatements w/ll ·be true about their
motion? ·
(a) The horizontal range of the parabolic path is 3 m.
(b) The maximum height for the parabolic path is 45 m.
I
I

'(c) Th~ to. al ti7?1e. of flight in the parabolic path for each·
t.

I1 particle = 4s

- :-~
·
(d). The horizontal nmge offeparabolic I!Qth is 6m ,
Solution: Distance travelled by 2 nd

r 2
-i,·=4s'11)
)-. kExam"t,c{!l,eJ
===------~~~ 99 t'~
;:-::::,.
. -. ·-7
= ~ !A football is kicked as shown in Fi1;. ,E.99. The angles 8 and <j,'
~
i.-1--- 1.
1--*I
!locate the point of maximum height. Show that
.___~_,_,Flg.1E.97
I t~-~-"'-t~ e + tan <P

Particle in 2 sec= 0.5 x 2 = 1 m I A

I'

l
Horizontal range = 1 + 1 + 1 = 3 m
Flight time = 4 + 2 = 6 sec. ,?.1~:::·.·:::::·.J~:::·).C. B l
,,.;
6 = 2u sine, u sine= 30
g
2 2
______ · .• •·____.,______
Fi_g._1_E_.9_9_ _...___..~---·----J
u ~in 8 = 900 = 45 _ Solution: The equation of trajectory is
H
2g 2x10· gx 2
y=xtano. ... (1)
Particle will strike the ground after 2 sec. 2u 2 cos 2 a.

= xtan o. [l - 2u 2 co:o. sino.]

www.puucho.com
Anurag Mishra Mechanics 1 with www.puucho.com

~ - - -
DESCRIPTION OFA\OTIOII

Range of projectile is 2

_.½ g:
2 4
R = 2 u 2 smo:

coso: ... (2) Ynmx. =X " [1 + ~
2
]
g gx U g X

From eqns. (1) and (2), we get 1 2


2 -gx
y =xtano:[1-~]
or
Ymsx. =u2g -~
2

The coordinates of A are (h cote, h) and range The shell can hit an area defined by
R =h cot 8 + h cot<j> . Substituting in eqn. (3), we get 2 I. gx2.
y:5~--2__
h =h cotetano: (1- h cote ) 2g u2
h cote+ h cot<j>
On substituting numerical values, y = 250 m, u = 100
tan e = tano: cot<j>
or mis, g =10 m/s 2 , we get
cote+ cotcp
2
or tan8cot8 + tanecot<j> = tano:cot<j> _x_ :5 250
2000
or 1 + tan8 = tano: or -500,/2 :5 x :5 500,/2
tan cj> tan cp
The fighter jet, can travel 1000,/2 m while it can be hit.
or tan 8 + tan q, = tan o:.
i-·-·· - --- r--1-. So the plane is in danger for a period of l000,/2 =2../2 sec.
k~~Pt!_?.Pl;~J 100·1> 500

_An enemy fighter jet is flying at a constant height of 250 m


with a velocity of 500 m/s. The fighter jet passes over an
anti-aircraft gun that can fire at any time and in any A shot is fired with a velocity ~ at a vertical wall whose·
direction with a speed oflO0 m/s. Determine the time interval distance from the point of projection is x. Prove the greatest'
during which the fighter jet is in danger of being hit by the height above the level of the point of projection at which the
u4-g22 ,
gun bullets. bullet can hit the wall is x
2gu2
Solution: The equation of trajectory of bullets is
1 2 Solution: Let 8 be the angle of projection.Suppose y
-gx
2 is the height at which bullet hit the wall. We have, from
Y =xtan 8 - - -
2
u2- (1+tan 8) ... (1)
equation of trajectory.
y

E wall
0
~
X
Fig.1E.100
X
For a given value of x, maximum y can be determined Fig.1E.101
from
2
1 2 y =xtan8 gx
dy 2gx 2u 2 cos 2 8
--"--- = X - - - (2 tan8) = 0
d(tan8) u2 gx sec 8 2 2
u2 =xtan8-~-~-2
... (i)
or tane=- 2u
gx dy gx2
On substituting the expression for tan 8 in eqn. (1), we - = xsec 2 8- --2sec8(sec8tan8)
get
ae 2" 2
2

www.puucho.com =xsec 2 8-gx sec 2 8tan8


u2
Anurag Mishra Mechanics 1 with www.puucho.com

8
. = xsec 2 0[1~ xg~~ ] usin0± lu 2 sin 2 0-4xfxiz
t = . 'J 2
For y to be maximum, dy = O 2xf
de 2
xgtan0]=0' u sine±4u
t = ___
2
sin 0'-~2gh
.,_____
2
u2 or c_

g
xgtan0].=
2 . .0 t to be real (u 2 sin 2 0-2gh) 2' 0
u.
u2 or u 2 sin 2 0 2' 2gh
or tan0 = - or u 2 (1-cos 2 0) 2' 2gh
gx ... (ii)
'Substituting tbis value in equation (i), we get From equation (i),
. y
max
= xu2
gx
~_!gx2
2 u2
[1+~] x2g2 case = -
v = ucose,

· ... (iii)
u
u2 gx2 u2.. 2 u4 -g2x2 -u4 Now from equation (ii), we.have
=g - 2zi 2 - 2g = 2u 2g
2 2 2
u4-g2x2 ~ (1-::)2c2gh or u -v. 2'2gh
or, Ymax
2gu2
or umin = 4v 2 + 2gh
pri:~<i!~l;~~ Substituting this value in
equation (iii), we get
IAn ~ero~lane~ies horizo~taily at:~~;ight ,,-~t.a· ~pe~d-~-"411 cos0=-v-
a7!ti-ai, craft. Kl!nfires ashell.at t~pldnewhen"it iil!irti.'c:#(ty umm.
_izbove .the.• gun,,{Shqiy that the iriinimum mU2zle ;ve/qcify V
=-=~=
req~ired ~o: Jtfi:. the plc11;,e is::411~ +·2gh at A'! )d~g~ 4v2 +2gh
. -1.(fiifi'J'.' . ·· .-.
tan .-. -.-·. ;.
. .,· .. . . .·'..·· 'I;I tan0 = ~2gh
and
' ' ·~·, :,·,·':,-",, aL 'J '~: '" "oWi:: . .r:1 • V

· Solution:
0
Suppose tbe muzzle velocity of tbe shell is
u and it is fired at an angle 0 witb tbe horizontal. To hit tbe.
f~l1~~
~ci~~l~:~~,:ojectedfro'!'.aR:~~~nth~. level gt~it~J~~d~M
plane, tbe displacement of shell along tbe motion of plane in
time t is equal to_ tbe displacement of tbe plane. Thus we f~height If lirwlien at .ho,;izontgfd~t<ln.ces a.and 2a from ..it<)
haye
P,Oint Of TJJ:01~.'itf®:J'ind the V¢lQClty ofjirojection.Jc;~ "J
... (i)
Solution: Ifv 0 is tbe velocity ofprojecticin arid a tbe
angle of projection, tbe equation of trajectory is
'
. 1 gx2
y =xtana-- 2 ... (1)
2 v 0 cos 2 a.
Witb origin at tbe point of projection,
2
gx - 2v~ sina.cosa. · x + 2v~ cos 2 a· y = O ... (2)
· Since tbe projectile passes ·through two points (a, h) and
(2a, h), tben a and 2a must be roots of equation (2),
. 1 2 2v~ sin a. cosa
and h =usm 0t--gt a+ 2a --''--- ... (3)
2 ·g
2 •
2v~ cos 2 a. h
or gt -usin0t+h=O and ax 2a ... (4)
2 g
Solving above quadratic· equation fort, we have Dividing eqns. (3) by (4), we get
3a tana 3h
or tana=-
www.puucho.com 2a2 =-h-
2a
Anurag Mishra Mechanics 1 with www.puucho.com

. ga2 (12.5 sin 0) 2


From eqn. (4), v~ =-,;- sec 2a i:e.,
2g
Sm

ga2 or sin2 S = 5 X (2 X 10)


=- (1 + tan 2 a;) 12.5x 12.5
h
= ga2
h
(1 + 9h2)
4a2
or sin0=~
5
and cos0=~
5
and v 0 sine= (12.5) x (~) = 10 m/s
2
=~ (4ah .+.9h)
4 .
T1me tak en to reach maximum
. . h't = 2vo
herg ~~-
sin 8.
g
2 2x 10
or v0 = 1 (4a
·-,;-+ 9h ) g = - - = 2 second
2 10
(c) Horizontal distance ofloop from point of projection
ii~¥cimmi,~~> = (12.5 COS 0 + 10) X 1
=17.5 m
IA man is riding_on q.flat car travelling With aconstan~ speed!
of 10 m/s. He. wishes to ,th,:ow a follthro,igh a stationary
hoop 15 m above the height of his hands ·in such.a manner
[,;1=xa-1mf,:;c~3
i ---- ~""k~ 105
"---- -·
J.~-
~

that the ball ;,,ill move horizontqlli as)tpasses. through· th~ IA ball ~ projected· with velocity vO and at an angle ofi
hoop. He throws the ball with ,a speed· of 12.5 IIVs w:r.t:1 ~fojection a. After what time is the ball moving at right a'!gles!
. If. _ . •. .
h. 1mse ' ------~, : .
'
. ., , I ltSLthe initial direction? _________ _____ ,, .. _________ · .• i
1------ ---------------- -- r.·, · ·solution: Method 1: If initial velocity v 0 and

I Sm f
velocity at time t are perpendicular, then the final velocity
will be at an angle a; with the vertical.
,.- - - -- --- - - ---

~~! . . :1 &-
it1:
;
1
1
C

velocity oftf!e.ball?, _ - _.,


'" "'!'
'(a) What must qe th~ verticd'l compQnent of the initia() '
·';::, . _. ·- .; J'
~:-
i

[_ .·., _, Fig; 1E.1-05 (a) ______


goo ··--••.

t ~ ·_-~----
_.' :f"i:£'--..J
i(b) How many,ieconds after IJe teleases t_h~ ball Will it pass
1 Horizontal component . of velocity is unchanged
through the.hoop?' .• · · · · - ,· I
1 throughout the motion.
cc) At what horizoutal dista.nce infrqn~. o(t. he fo'op musthel
I release thl!.11!#1? _ . · , · ___, _. .: _·_ • · . · , Therefore v O cos a = v sin a
or v = v 0 cote,;_
Solution: Two important aspects to be noticed in this
problem are: Vertical component of velocity after time t = - v cosa
(1) Velocity of projection of ball is relative to man in From the equation v y = v O sin a - gt
motion. -vcos a= v 0 sine,; - gt
v sin a+ v cosa
(2) Ball clears the hoop when it is at the topmost point. or t = -0" - - - - -
.., .., .., g
V ball, man = V ball - V man v 0 sina + v 0 cotacosa
.., .., .., ·=-"---~----
V ball = V ball, man· + V man g

(a) Now we apply the above relation to x- as well as ~ v0 [sin a_+ cos a]
2 2

y-component of velocity. If ball is projected with velocityv 0 g sma


and angle e, then
.., = Vo coseca
x-component of v ball = (v O cose + 10) m/s g
.., Method 2: We choose x-axis along the initial velocity.
y-component ofvba!l = (v 0 sin0) m/s
If after time t the velocity is perpendicular to initial
(b) Since vertical component of ball's velocity is direction, v x must be zero after time t,
unaffected by horizontal motion of car, we can use the
formula for time of flight,
www.puucho.com
. .·~..
Anurag Mishra Mechanics 1 with www.puucho.com

' .. . ..,.,_ ~

1a2 ..~.~···~·=-~_. ._:._J_-~'-":f__ ~-· ..~-~ -·.t·· ~ir~i:_.~-':-~---~-----~-


r--..-, _ .,;~:~~).· The equation of trajectory of a projectile is
y = x tan a - - ~ -2-
gx2 ___ (l)

l~,i
L_ Flg.1Ei105 (b) -_ j Hence R sin P = R cos!} tan a -
2u 2 cos a
Point (R cos p, R sin I}) must satisfy equation (1).
- gR 2 cos 2 1}
2u 2 cos 2 a
i.e.;..._.---- 0-v 0 g smat-- gR 2 cos R
or R(tan a - tan P) = ~c--"-
or t =~ 2u 2 cos 2 a
gsina gli cos p
or
Method 3: Slope of trajectory at the point of cos a cos P 2u 2 cos 2 a
projection, 2
R = 2u sin (a - I}) cosa
m1 = tan0 or
--~--_--7~-.;--, .:7 , g cos 2 p
Method 2: We take axes along incline and
,' perpendicular to incline as shown in Fig. 1.9L '1n this
coordinate system, components of velocity and acceleration
' along the incline and normal to incline are
I "
. t~ - - -. . . ''
. ._, --~ ' ux c:a u cos (a - !}), ax = - g sin,P
L,_~Fig.~5(c)·· _, ,
uy = u sin (a - !}), aY = - g cos p
Slope of trajectory after time t, i-_---------- --· u
' dy dy/dt \; ' y
m2 =tana=-=-.- I ,
dx dx/dt \, ' '

Vy
=-=~---
0 v sina-gt ' u, = u sin (a-~) X

Slopes are perpendicular,


vosina-gt)ctana) = - l
( v 0 cosa _

or t=~ When projectile lands at A, its y-coordinate is zero.


gsina 1 2
O=ut+-at
y 2 y
PROJECTION ON AN INCLINED PLANE
or 0 = u sin(a - J})t - .!_~cos J}t 2
A particle is projected ry, ·: . ---;--:-;--- , ,. r 2
from point O on the foot of 2u sin (a - P)
an· inclined plane. The [ '. ~- · ,,u ~ath :fR~i~c\i;e : : i or t=----~
g cos p
::-.< ,; /A'' . :_,'.. ,·
l ,
velocity of projection is u,
This is the expression for time of flight from O to A.
angle of projection a with - - : ,. ':· ( - .5'
"'-
x-axis, angle of incline P ,, R• ·,.. :i ,. ,,.Cl)
For motion along inclined plane (x-axis),
,, .0:::
[see Fig. 1.90]. We wish to ~- : ,- 1 2
X = Uxt + - axt
determine range along .,:;o.,, ,·
'_:,;· ~, Rcos 13----.
B, ,, _x 2
I
incline, . time of flight, 1 -·~- -
' Fl 190 , =UCOS(a-J})t,-.!_gsinl}t 2
vertical height at which Lr_·_·_---~·--·-~---~ 2
projectile strikes. Substituting expression for time of flight, we get
(a) Range Along Inclined Plan·e 2
R = 2u sin (a-P) cos a
Method 1: Point A where the projectile lands has g cos 2 p
coordinates (R cos p, R sin p}.

www.puucho.com
Anurag Mishra Mechanics 1 with www.puucho.com

DESCRIPTION OF MOTION

Method 3: We revert Projection Down the Inclined Plane


y
back to a new coordinate y
system with x-axis in
horizontal and y-axis in
vertical direction [see Fig.
1.92].
x=u coso:t
2u sin(o: - Pl
-x-
Fig. 1.92

= u coso: - - - ~ ~
gcosp
R = ~ = 2u coso: sin(o: - P)
2
and x"'
cos P g cos 2 p Fig. 1.93

(b) Vertical Height at Which Projectile Strikes From figure we have,


Method 1: From the equation y = u sino:t - ~ gt 2, ux =ucos(0+o:), ax =gsino:
2 "Y =usin(0+o:l, ay =-gcoso:
on substituting time of flight t, we get Time of Flight
2
. 2u sin(o: - Pl - -g
y =usmo:----~ 1 (2u sin (o: - PlJ As displacement become zero along y-direction in time
g cos p 2 g cosp 'T'.
2u 2 coso: sinp sin(o: - Pl 1 2
O=uyT+-ayT
2
g cos 2 p
or 0 =usin(0 +o:)T- ~(g cos·o:)T 2
Method 2: y = :nanp_ 2
u cos o: x 2u sin(o: - P)
=--------'---'--'-X tanp
A
or T = 2u sin(0 + o:)
gcosp g coso:
2
2u coso:sinP sin(o: - P) Range Along Inclined Plane (R):
=----~=----'--
g cos 2 P 1 2
R=uxT+-axT
2
(c) Angle of Projection for Maximum Horizontal
2
Range
=ucos (0 +ex l[ 2usin(0+o:l] +-gsma
1 . [2usin(0+o:l]
-----
Range R, is given by g coso: 2 g coso:
2
R = 2u sin (o: - P) coso: After simplifying, we get
g cos 2 p 2
2 R= u [sin (20 + al + sin a] ... (ii)
u [sin(2o: - Pl - sinpi g cos 2 a.
- g cos 2 P For maximum range sin(20 + al =+ 1
For R to be maximum sin (2o: - P) must be maximum. or (20 + o:l = 90° or 0 = 45°-_<:
1l 2
Hence 2o: - p = - 2
2 R = u (l+sino:l = u 2 (1+sino:l
m~
or 0:=-+-
Jt p 2
g cos o:
2
g (1 - sin o:l
4 2 u2
The maximum range,
or Rmax =- - - -
g (1- sino:l
u 2 (1 - sin Pl - -- - '
Rmax. = ----:;--_;___
2 k.:~-?5::~tD.l?}e ~06 _.;-
g cos P
_ u 2 (1 - sin Pl uz 'A heavy particle is projected from a point at the foot of a fixed
- g(l - sin 2 Pl g(l + sin Pl plane, inclined at an angle 45° to the horizontal, in the
vertical plane containing the line of greatest slope through the
point. If$ (> 45° l is the inclination to the horizontal of the
initial direction of projection, for what value of tan $ will the
particle strike the plane:
(i) horizontal
(ii) at right angle?
www.puucho.com
Anurag Mishra Mechanics 1 with www.puucho.com

Solution: Let the particle be projected from O with or 2tan<j,-2=lttan<!>


velocity u and strike the plane at a point P after time t. or tan<j,=3 ·
Let ON = PN = h; then OP = h-Jz.
(i) lf the particle strikes the plane horizontally, ~~~~~
then its vertical component of velocity at P is zero.
Along horizontal direction, [0e a~la(,,dkation ofan enem,y's_P?Sition on a_hil('h\~
· h = (u cos <!>)Ct)
Along vertical direction,
... (1)
\;!Jecdle~in1:!r:~:~%~; ~~e t;;e:i1mum ;efo~1trjo{i'tfe,j
0=usin<!>-gt Solution: 'O' is the point of projection of the shell
or usin<!>=gt ... (2) and 'A' is the position of enemy at a height 'h' above the
·----- level of'O'.

I
. _. ·'. F . . .·
•,Flg.1E,106 (a) .
=---''--'-' -,~+-·•-
. 1 ' 2
and h=usin<!>t--gt ... (3)
2
Using eqns. (1) and (2) in (3), If 'u' is the minimum initial velocity of the projectile to
shell the enemy, then 'QA' must be the maximum range up
(ucos<!>)(t) = (usU-:<1>)(t)-:! (usin<!>)t the inclined plane of angle a .
2 ·u
So OA=---- ... (i)
tan<!>= 2 0 g(l+ Sina)
(ii) If the particle strikes 'th~ plane at right angles
From 6.0AB, OA = h coseca ... (ii)
at P, then the ·component of velocity parallel to the plane is
zero. From eqn. (i) and (ii),
Along perpendi9ular to the plane, u,;, ~gh (coseca + 1)
~~~,r~ ·:: · ·,~/:;'4'·,
(• , . . · v, .g cqs, ..
r5~~~~p~;~:-~-;,J . De:-::--~:=·~~-·-,
,;:;E•""rU!l:Oi!Wf1 ei - 108 t,:,~
~--~---~~~- ~_}~,
!t-$fn:45; · ·-~ 1• ,· "', · · -._..,,-,:,,,.-'/," ' • ' ~ - : ; _ , , •.,,,,-, -~:-~'""".'" 0 ' C ' ' ' ' '., ,_.-,7',--· -:--r ,;, C ,' ,,

i{~~}\~~-~.Q/ .: :.• ' ~\J, ;'(4J-_45_o,} -:~ prOjeft(le /,t t/-irQWn, at ar( G71$/e ;~With an inf/i~ed plci.rtfOJ
w:~~-,_ ,.,,- .U· .' rlric(irtqtion ~- as .shoWrt. id'
f:ig.: ).~:108.. Find ,the r~ldtio~
. I~,-:,· .;> • .h .bet:)l(een~q11i:!Bj(:. . _ ,,,, • ,,:',.' ,',, ';'/ C·
. f(a) p"oJectf,le,stri~es the inclinedpCane,perpendicular1y1, {'
<ti·-· - , .450 ; ~-i:
~'
,_ N· -
ll;,),p:r:._ojectiie,~@<es· the inclinefiuilqn~h'g_rj.;Qntab,,.::i;;;:/~.-
f?/:f:;-. 1;0 i;_·_·.r.:~-~> :··.:, ,.
:" Solution: (a) If projectile strikes perpendicularly.
b:fu,.;.L...::_ Fig: 1E:tOI! (b) , ' . ___ j i -i: , . 7
1\·. :.:'. ">X·8X1Sl
' '
0 = usin (<j,- 45°)t - .!g cos 45°t 2
2
zJzu , .
t = - - sin C<i> - 45°)
g
Along the plane,
u cos_(<!>- 45°) = (g sin 45°)t
= -~_ _JzJzu sin (<I> - 45° )]
,Jz l g
or ucos (<!>- 45°) = 2u sin(<!>- 45°)
or .! = tan (<I> - 450} = _tan_<l>_-_l
.2 l+tan<!>
www.puucho.com
Anurag Mishra Mechanics 1 with www.puucho.com

I _D~~~RIPJION O_F MO_TION ssl


-------- - - --------------------~
2u sin9 as
we also that T = --- Vx=ux+axt
g cosp or 0 = u -g sin60°t
u cos9 2u sine
=> => 2tan9 = cotp => t= u 10-./3 =2s
g sinP gcosp g sin 60° x -/3
(bl If projectile strikes horizontally, then at the 10
2
time of striking the projection will be at the maximum (bl Initial velocity along y-axis is zero. The velocity
height from the ground. Therefore, along y-axis after 2 s;
~:. ____________ ' Vy= Uy+ Uyt
= O-gcos60°x2
1
0 tf1(_______________________ . . . . _____ _ =-10x-x2=-10m/s
2
(cl We have, v; = u; + 2axs
Fig.1E.108 (b)
Since vx = 0
2u sin9
tap=--- and ax = g sin 60°, u = 10-./3 m/s
gcosp
O = (10-./3l 2 -2xg sin60°x (OQl
2u sin(9 + Pl
tap=---~- 2
2xg or OQ = 10 X 3 = l0-./3 m
2usin0 2usin(0+Pl -./3
2xl0x-
=> 2
g cosp 2g
=> 2sin0 = sin(0 + Pl cosp. Distance PO = O+ .!:. g sin 30° x (2) 2
2

l -~-~~':Tl-r. "'7 I~cjJ.-> 1 1


= - x lOx- x 4 = 10 m
2 2
'lwo inclined plane.s OA and OB having inclination 30° and Therefore height h of point P,
60° with the horizontal re.spectively intersect each other at 0,
h = PQ sin 30° = 10 x .!:. = 5 m
as shown in Fig. lE.109. A particle is projected from point P 2
with a velocity u = 10-./3 m/s along a direction perpendicular
(dl Distance PQ = ~P0 2 +OQ 2
to plane OA. If the particle strike.s plane OB perpendicular at
Q. = ~(10l 2 + (10-./3l 2 = 20 m

l :!.~~G9!!11?J,f7: .G10 1;>


u nvo guns situated on top of a hill of height 10 mfire one shot,
each with the same speed 5-./3 m/s at some interval of time.,
One gun fire.s horizontally and the other fires upwards at an'
angle of 60° with the horizontal. The shots collide in air at a,
30°
point P. Find
0
(al the time interval between the firings and
Fig.1E.109 (bl the coordinates of point P. Take the origin of coordinate
Calculate system at the foot of the hill right below the muzzle and
(a) time of flight trajectorie.s in the xy-plane,
,(b) velocity with which the purticle strikes the plane OB u,
(c) height h of the point P from point 0
( d) distanc~ PQ.
---~
_,;..---ll,un 1
P(x,, Yr)
Solution: Consider the .motion of particle along the
axes shown in figure. We have
10 m
ux = u, ax= -gsin60° P(x,y)
Uy= 0, ay = -g cos60° (0,0) x-
(a) As the particle strikes the plane OB·
perpendicularly, Fig.1E.110.(a)

www.puucho.com
Anurag Mishra Mechanics 1 with www.puucho.com

Solution: Let gun 1 and gun 2 be fired at an interval Method 2: We take point of firing as origin and x-
Llt, such that and y-axis as shown in Fig. lE.110 (b). Equation :of
t1 = t 2 + M ... (1) trajectory of '\ projectile is
2
where t 1 and t 2 are the respective times taken by the y = X tan0- gx
two shots to reach point P. 2v 2l cos 2 0
For gun 1: · For gun 1 1 . 0 = 60°.
X' -~ anent y
X, -·.'Xt, =11,t q>S 60° ~l
. . .
X
1
= )(.+,.;....·V·t1 . . ./3 . . l i'
\' 2 · Y=Y·+-v-t 1 -~•t1
l : • I 2 I z9•.

2gx2
=x-./3--
(a) Now we can equate x- and y-coordinates of v?l
shots, For gun 2, 0 = 0°.
i.e.-, ... (2) -gx2
y = 2v 2 ·
l
and
Two shots collide at point P; therefore their coordinates
must be same;
or ... (3) 2 2gx2
i.e., ~gx · = x-./3 - - -
On substituting t 1 from eqn. (2) into eqn. (3), we get 2v2l v?l
-./3 1 2. 2gx2 gx2 3gx2
. v; (2t2) + Z g(-3t2) = 0 or x-./3 = - - - - = --
2 v2l 2v?!' 2v 2
or t 2 (-./3v, -1
.
gt 2 ) =0
2 V·
·Or x=O and
2v2
x--' --=~-
- -,J3g -
l

2(5/3)2
./3(10)
or t2 =0 and t2 =- -'-
-./3 g = 5-./3 m
=lx(7o3)=ls and
-gx2 _ (10) (SV3)2
y=--=--~~
. 2vr 2(5-./3) 2
-

Therefore,t 1 = 2t 2 = 2(1) = 2 s =-Sm


M = t 1 - t·2 = 2 - 1 = 1 s · If originis assigned at ground the coordinates of point P
(b) Tbe coordinates of P at which the . two shots will be (5-./3 m, 5 m).
collide are Now We consider x-component of displacement for both
x=xi+vit 2 the shots.
= 0 + (5-./3)(1) Gun 1: x = 5-./3 m = v,t = (5-./3 m/s)t
= 5-./3 m or t, = 1 s
and
1 2 5,/3
y=y,-2gt2 Gun 2:o X = 5-./3 m= V; COS 60° t2 = - - t2
2
= 10 - .!_ (10)(1) 2
or t 2 =2s
2
Time interval between two shots is Llt =t 2 - t1 =1 s
=Sm

www.puucho.com
Anurag Mishra Mechanics 1 with www.puucho.com

[ DESCRIPTION OF MOTION . a~J


lJ~.j~}J~J~·~ 0~( .->
1

->
->
V panicle, box
->
= :V particle, ground
->
->
- V box, ground
->
A large heavy box is sliding without friction down a smooth V particle, ground = V particle, box + V box, ground
plane of inclination 8. From a point P on the bottom of the- Applying above equation to x-components,
box, a particle is projected inside the box. The initial speed of
the particle w.r.t. box is u 1 and the direction of projection
o =u cos (a + 8) - v cos8
ucos (ex+ 8)
makes an angle a with the bottom as shown in the figure. or v=
cos8
Method 2: The above y
condition can be meet if the box
covers exactly the same distance
as the range of particles,
i.e.,
Fig. 1E.111 (a)
I ( u; : : : J=v( 2gu:~~0cx) 0
(a) Find the distance along the bottom of the box between the
point of projection P and the point Q where the particle 2 Fig.1E.111 (c)
1 . e(Zu sina)
+-gsm
lands. (Assume that the particle does not hit any other:
2 g cose
swface of the box. Neglect air resistance.)
(b) If the horizontal displacement of the particle as seen by u sin0sina
or ucosa= v + - - - -
an observer on the ground is zero. Find the speed of the' cose
box w.r.t. the ground at the instant when the particle was, cosa cos0 - sin a sine)
or v = u( - - - - - - - -
projected. cose
Solution: (a) Motion of the particle will be in u cos(a+0)
reference frame of box. cose
x-component Relative Motion
Fig. 1. 94 shows an observer on ground, a balloon and an
uy =u sin ex
airplane, we denote them by G, B and A respectively. At any
ay=gcose instant position. vector. of airplane for -an observer on
ground, on balloon have been represented.
or a,=gsin0-gsin0= 1 2
y =U/ --g/
2
x=u,! =ucosat Put y =0 for calculating time'
ar flight. I
2u sin ex) '
2 1
Q::;::usincxt _!gcos0t
=U COSCX
( g cos a 2

u2 sin 2a or t
2u sin a
gcosa g case

(b) According to problem the horizontal displacement


of the particle as seen by an observer on the ground is zero. Fig. 1.94
If we analyse the situation in the reference frame of ground,
resultant velocity of particle in x-direction must be zero. .... ....
rA/B
r;y8 position vector of airplane for an
X .--J ~ observer on balloon
object observer
Q
.... ....
rB/G
rB/G = position vector of balloon for an
.--J ~ observer on ground
object observer

Fig.1E.111 (b)
.... ....
rA/G
rt\'G= position vector of airplane for an
.--J ~ observer on ground
object observer
www.puucho.com
Anurag Mishra Mechanics 1 with www.puucho.com

_, _, _, _, _, _,
From figure rA/G = rA/B+ rB/G (al VP/E = VP/G-VE/G
Rate of change of position vector is velocity.
_, _, _,
or VP/G = VP/E+VE/G
Thus,
which implies that absolute velocity of the passenger is
_, _, _, the vector sum of his velocity relative to escalator and
or, VA/B = VA/G V,rG _, _,
.j, .j, .j, velocity of .escalator relative to ·ground. v P/E and v E/G both
Velocity of Airplane Velocity of Velocity of pciint towards right as ~hown in Fig. 1.95 (b)
as observed by A for observer balloon for
observer on balloon on ground observer on
r- : ~T~~,-Ir·~-~,
\, .. }
ground i
!
i
When we say velocity of airplane w.r.t. balloon or .I
velocity of airplane in inference frame of balloon. it means I

,JA/ 8 referred as relative velocity.


Application of Advanced
l
Concepts of Relative Motion ·I

River Condition
Fl~.1,.95
;(b) i'
Consider a swimmer in still water. The swimmer can
generate a velocity due to its own.!'ffort. We call this velocity,
velocity of swimmer in still water.
_, ----,_~,,.-----~------· ~,·- ~-~-·------,--~
Velocity of swimmer relative to water = v s/w '.(a) "Find but the motion of t;Je, ql~d (I~d ~Id IJlan(lS ~e~n by
Next consider a person with a life jacket in a river I . boy: ;' .. · · ,. ·. ·· • · ·· ·•; ... .
flowing with a velocity. Person makes no effort to swim, he (b) Firi4 out,m~tioft oltree, bird, boy as seen.by,oid man,
just drifts due to river flow. Velocity imparted due to river re) Find'qut11uitio[l.Pftre.§, DOYcartd old man as seenbjzoira.
flow is called velocity of water relative to ground, i.e., it Solution: (a)With respectto boy:
denotes the rate at which water flows.
_, Vtree =4m/s (~) .
Velocity of water flow relative to ground = v w/o vbinl =. 3 m/s (I) and O m/s c~)

Next consider a swimmer applying Iris effort in flowing


water. In this case swimmer's net velocity resultant velocity
will be decided by two factors
Ci) his own effort
(liY water flow.
Thus resultant motion is obtained by vector sum of two
velocities imparted to swimmer..
Resultant velocity of swimmer relative to ground =
velocity of swimmer relative to water + velocity of water
flow relative to ground. ·
_, _, _,
VS/G = Vsjw+vw/G (b) With respect to old man:
Esc,ilator Condition vboy=6m/s(~)
Here is an analogous
..
treatment. Just
_, imagine an = 2m/s (~)
Viree

escalator moving horizontally with velocity v E/G. A person vbird =.6 m/s c~
l and 3 m/s (I)
_, . . (c) With respect to bird:
begins the run with velocity v P/E fa the same dii;ection as . vtree = 3m/s (.J,) and 4 m/s (~)
escalator. What do you think about resultant velocity of Void man= 6m/s (~) and 3m/s(.J,)
passenger?
Vboy = 3 m/S (.J,)
We assign a latter to each body P, passenger; E,
escalator; G, ground. www.puucho.com
Anurag Mishra Mechanics 1 with www.puucho.com

, _DE_SCRl~TION OF MOTION 89,

f_-~?f9:'.T\P Ie
~--
113 ",;>
.
. y-.
A helicopter is trying to land on a submarine deck which is
moving south at 17 m/s. A balloon is moving at 12 m/s with
~E =iA/P +4J__,, Vp/E
Vp/E

wind into the west. If to the submarine crew the helicopter is 8


descending vertically at-'5 m/s, what is its speed? (a) relative 1-----v='-------'=illF--,1+._-+•
to the water and (b) relative to the balloon. See fig.

EX
Fig.1E.113

Solution: Velocity of ant relative to paper


(a) Vhel/water= Vsub/water + Vhel/sub Fig. 1.96 (b)
= l7j + (-S)k = (17j-Sk) mis
(b) V hel/ballodn = V hel - V balloon Girl Moving in a Train
=(17j- 5k)-12i Illustration 11. Fig. 1.97 shows top view of a girl
=(-12i+17j-5k)m/s (G) walking in a moving train (T). Two observers one in the
train and the other on the ground (E) determine the position
Ant Moving on an Ruler
vector of the girl._, _, _,
Fig. 1.96 (a) shows an ant scampering along a ruler. The
_, rG/A =rG/B + rB/A ••. (1)
ruler has been displaced w.r.t. Earth by SR/E, the ant
_, The position of the girl walking in the train relative to
undergoes a displacement SAfR w.r.t. the end of the ruler. frame of reference of A is different from her position relative
The net displacement of the ant w.r.t. Earth (i.e., w.r.t. a fixed to frame of reference B (Fig. 1.97). Time derivative of eqn.
point P0 on the ground) is given by the vector sum (1) gives the ralation between various velocities.
_, _, -> -> _, ->
SA/E =SAjR + SR/E ... (1) VG/A =V G/B+ VB/A ... (2)

Taking time derivative of eqn. (1), we get the Ya


corresponding velocity expression
_, _, ->
V A/E =V A/R+ VR/E ... (2) G Train

~t:;z=::==:f---Xa
Reference B frame
fixed to train

~------+XA
QA Reference frame A
Fig. 1.96 (a) fixed to Earth

Eqns. (1) and (2) are valid irrespective of the direction (a)
of two vectors. Fig. 1.96 (b) shows the motion of an ant
walking across a sheet of paper, that is itself being moved at - - - - - - - - v'G/r---tVelocity of girl
relative to train
a speed .f P/E. The ant is carried along with the paper so that 1Grr+~ -JTIE__,, Velocity of train
·1 ~ relative to Earth
it actually moves north-east w.r.t. Earth.
VT/E VG,E---t Velocity of girl
'--------'=--' relative to Earth
(b)
Fig. 1.97
www.puucho.com
Anurag Mishra Mechanics 1 with www.puucho.com

MECHANICS,! ]
Velocity of particle G relative to reference frame A ,· ~ ------·--:
i ~--1I
= velocity of particle G relative to reference frame B
+ velocity of reference frame B relative to reference .
,_ Vsr,,
___,___
~ ~ _- River flow.I
!
frame A
If the girJ, walks across the compartment, her resultant ...,
I
1
Vw/G ._vstG
• Fig.1:98(b) ____ , ,

velocity will be as shown in Fig. 1.97 (b).


..., ..., ..., Iv s;w I=vs =velocity of swimmer relative to water
V G/T = V G/E-VT/E ...,
..., ..., ..., _I v W/G I= v w =velocity of river water flow

or V G/E = V G/r+Vr/E ..., -> ->


Iv s/q l=lv s;w l+lvw/G I
i.e., velocity of girl w.r.t. earth (reference frame of
ground) is vector sum of its velocity relaitve to train and .=vs+vw
velocity of train relative to earth. Case (ii) Swimmer moves opposite to river flow
(upstream)
Airplane-wind Condition
when swimmer moves upstream.
..., Consider an airplane moving in still air, with ve!O(;ity -> -> ...,
V AfW• lvs;G l=lvs;wl-lvw/G I

Plane in which Note:]------=-'-V£.s_-_v_,w"----------


~AJW • airplane moves
A floating object like a wooden log move with the
------- ____ / _;:f, ------- / --
•••••
••••
. -··t:e
\
~H
•• -,

• :
->
VAJG
(..
•••
•••
•••• ••••• !
' •
:
velocity of river flow.

Step 1: Problem Solving strategy:


''' .
'' . ' Assign the initial point as origin of a coordinate system.

!_...... -· ...-_· .. '· -··----------.--. .x::,··· ........, .f


' · Ground plan~
r-:~---·y . ---,--,--·1
I, -+--.........,.~--,- i
->
Vsr,,
swimmer
W~E' . ~--, begins here
#/
·''"O
s '
0
Flg.1.98 (a)
~--- -·--- -- ··---· ...,------·- --- - . --·- - Fig.1.99

Wind· flows with velo~ity v W/G due east direction, Step 2: Draw vector diagram
Resultant velocity of air plane will obtained by equation __
----_---....·-=-===;~-:::-7
-> -> ->
• :Position P6sition
V A/G = V A/W +vw/G where man where man
In all the previous real situations there is an object that
moves on a moving medium
I heads actually reaches
A B

..··
River condition
Object
Boat
Swimmer
Medium
Water II Initial direction
of motion of man
Escalator condition Passenger escalator
Ant-ruler condition ant ruler
Girl-train condition girl train
~---- -------- ---- ., _____ ·-----
Concept ----,
V object/ground
j

= V object/medium + V medium/ground
Resultant velocity • velocity of object _ velocity of medium
j
.
0

---- Flg.1.100
X

relative to medium ·
~---- --- -··---~ ----- - - - - - When swimmer reaches Bits x-component displacement
River Condition Revisited is x whereas d represents of y-component of displacement.
Case (i) Swimmer moves in direction of flow (down Step 3 : Apply component method of vector addition.
stream)
www.puucho.com
Anurag Mishra Mechanics 1 with www.puucho.com

DESCRIPTION
-- - --
OF
- MOTION
·- __ -
,~,_., , -· ---· ·-------··-··---- ------ - - ·-
Concept Motion in x-direction is due to x-component of (vs;a\ = Vs sin0.
resultant velocity, similarly motion in y-direction due to Time taken to cross river
y-component of resultant velocity. d
t=---
..., A A
VsSin0
Vs;w = Vs cos0i+vs sin0j Drift= (vs -vw cos0) x d_
..., A Vs sm0
Vw;a =vwi
..., A A
Concept: Note that drift can be zero if
Vs;a =(vscos0+vw)i+vssin0j Vs= vw case
x-component of resultant velocity In this case swimmer moves along shortest path.
(vs;alx =vscos0+vw But above condition can be satisfied only ifvw > Vs-
Ifvs > vw, drift can be minimized but it cannot be zero.
x-component of displacement For minimum drift
x = (vs cos0+vw)t d
-[vwcosec e+vs cot0] = 0
Similarly y-component of displacement dw
y = Vs sin0t vwcosec0cot0-vscosec 20_= 0
Thus time taken to cross d width of river Vs
or cos0=-
Vw
It= Vs ~n 8

Drift during crossing of river


x= (vs;alx t A boat moves right across a river with velocity 10 km h-1
(VsCOS0+vw)d relative to water. The water has a uniform speed of 5.00
x= km h- 1 relative to the earth. Find the velocity of the boat'
VsSin0
relative to an observer standing on either bank. If the width of
Concept: How to obtain time taken to cross river? river is 3.0 km, find the time it takes the boat to cross it.
y-component of displacement ...,
t=--"---'-----'------''---- Solution: v B/R -, velocity of boat w.r.t. river
y-component of resultant velocity ' ...,
What is drift ? v R/E -, velocity of river w.r.t. earth
...,
Distance the swimmer is carried away along flow while v B/E -, velocity of boat w.r.t. earth
crossing river. --t ---), --t --t ---), --t

Xdrift =[vSJalx x time VB/R =VB/E -VR/E or VB/E = VB;R+vR/E

Position Position
where man where man
heads actually reaches
A B
• ...
·· ...
,, ·- ..
. ->
->
VRIE

Actual direction
.. Ay VR/E
River flow
of motion of man ······ ...
lniUal direction
of motion of man
Man begins
atO
0 X
Fig.1.101 D

What happens if swimmer moves opposite to flow ?


---), ,,._ A ,.,_
Fig.1E.114
Vs;a = -vs cos0i+vw i+vs sin0j
(vS/G lx = (vs -vw cos0) www.puucho.com
Anurag Mishra Mechanics 1 with www.puucho.com

Hence, Angle at which boat starts is given by


VB/E = ~V~jR + V¼E • · VR/E 1
sm8=--=-
vB/R 2
·= ·/i0 2 + 52 = 11.Zkmh-1
.... or 8 = 30°
The direction of v B/E is Method 2: , Consider point O to be. origin of a
coordinate system x,y.
8 = tan-:(VR/E) Vy = VB/E
VB/R
vx =vB/Rsine-vR/E
=tan -1(105) =tan-1(1)2 If x-component of resultant velocity vanishes the boat
will move straight, along y-axis.
.... .... .... .
Hence, VB/R sin8 = VR/E
Note that v B/E is resajtant of v B/R and v R/E • Effective
.... or
.
sm8=--·
VR/E
velocity of boat in y-direction is v B/R • VB/R
Hence time taken to cross the river is Crossing River Along Shortest Possible Path ;_e.,
_!!__ = ~ = 18minute. Moving Perpendicular to Flow
VB/R lQ
In this case, x-component of resultant velocity is zero.
·---- -·,,. B. • -----1
cft=~-,-
J£1X0,mi.:i~5 ··' ·
~r}\'.!,~~ v.·· I

~7&G,.:
, _. : I
Wthe b;;t-i>j'i,receding ,;:m,pl~ ,"ttd\>et_; with s~me 'jp~eij !
(relative to the riv.er and is to be iowards fight across, il(w/laq
iiJir{ctiol!!.s/;lo.!!lsl.Jtb,ead? -':;,.:::.:,_ ' . . '/:_j
:_.·,.,A - x,
Solution: Method 1: The boat must h~ad at certain
~-~lg.1;1~~----·-_j
angle upstream so that vector sum of ·velocity of boat
relative to river and velocity of river relative to earth must [vs;alx = O' .
be directed right across, -v 5 cose·+vw = 0
-) -) -) . V
i.e, VB/R+VR/E = VB/E i.e., cos8=_.!!',
Vs
From Pythagoras' theorem, Direction of swimmer's velocity relative to flow
VB/E = ~rv-:~/-R___V_¼c--E- direction is (180° -8).
= ~10 2 - 5 2 '= 8~66 km/h -> · ~-- /2 2.
lv's;al=vssm8=Vsv1-l-;;';) =,iVs-Vw
I
Time taken to cross the river
d d .d
t = - - = - - - = ,====
Vs/G v 5 sin0 ~v~-v~

t
I. E
~---~-0 .. -'"" y"

C!>nce~t: What happens if vs < v w .swimmer ca~


'

cross along sfiortest.path because v5 ,cos8 < vw•diift,will'.bej


, . ~· -~-·--1
nod
always po1iff,ve .swimmer can . mqv~ ,right across qnly if/
'-"-..'-'!-dre~----·-----~. ·- -·---·~----.'.....---·-··-~I
v 5 >vw, i'. · .• ,

Crossing River in Minimum Time


If swimmer begins at angle 8 with µver bank, time taken
to cross river will be given by
d
t=---
Vs sin8
For tmin, Sine must be maximum
d
tmin = -
Vs
www.puucho.com
Anurag Mishra Mechanics 1 with www.puucho.com

DESCRIPTION OF MOTION 93
Drift in this case
N

Concept: Swimming in a desired direction:


Many times the person is not interested in minimizing the,
time or drift. But he has to reach a particular place. This is'
common in the cases of an airplane or motor boat.
B

Flg.1E.116

______ ....,. . - ----------------·


Solution: If bird is to move along AB, component of
.:.-:::Vmf ." _-_-_-_-_-_-_-_-_-_-_-_-_-_-_-_-_-_ ' velocity of bird and wind perpendicular to AB cancel out .
........... ·0················.

v,
..., (a) 4sina = 2sin37°:; a= sin- 1
3
0
) (i
Fig.1.103
The man desires to have this final velocity along AB in other' =} 37°+sin-1
3
with east. (io)
'words he has to move from A to B. We wish to find the ..., ..., ...,
direction in which he should make an effort so that his actual' (b) vb = vbw+ v w = vw cos37°+4cosa
velocity is along line AB. In th~ method we assume AB to be'
the reference line the resultant of v mr and v, is along line AB. =~+ 2foi = 8 + 2v'91 [·.- cos a= ffi]
Thus the components of vm, and v, in a direction: 10 5 5 10
perpendicular to line AB should cancel each other. : (c) t = lOOx 5 = 250 sec
X 8 + 2ffi 4 + ffi
B . ., .. .
[,g-?::$<:l_-!'.!'.PJ_'::_j 117 !;>
··--,
·:::::::::: Ymr::· .::::·.::::::·
A woman is running through rain at a speed of 5.00 m/s.
:Rain is falling vertically at a speed of 20.0 m/s. (a) What is
the velocity of the rain relative to the woman? (b) How far in
front of her would an umbrella have to extend to keep the rain
Fig.1.104 'off if sh~ hold_s the umbrella 1.50 m above her feet?
..., ..., ...,
or
vm =vmr+vr
...
-
I,

-:J m = [vm, cosai + vmr sina]J +[v, cos8i- v, sin8j]' ' ...
VR/ 20.0 mis
Vw/E

14.0~
and, Vmr sina-v, sin8 = 0 5.00 mis
1.50 m
~ Vmr sinCX = V 1 sin8

l' ...
VW/E
d

(a) (b)

Fig.1E.117
Wind is blowing in the el!St direction with a speed of 2m/s. A,.
bird wishes to travel from tree A to tree B. Tree B is 100 m · Solution: We assign the following letters: W, woman;
away from A in a direction 37° north of east the velocity of · ...,
bird in still air is 4 m/s. R, rain; E, earth. We have to find v R/W.
(a) Find the direction in which bird should fly so that it can: --+ --+ --+ --+ --+
reach from A to B directly. (a) V R/W = V R/E - Vw;E = V R/E + (-vw!E)
(b) Find the actual velocity of the bird during the flight. From vector diagram,
(c) Find the time taken by the bird to reach B. -------
--+ I --+ 2 --+ 2
I VR/w I= \f(VR/E) + (-vw/E)
= ~(20.0) 2 + (5.00) 2
= 20.6 m/s
www.puucho.com
Anurag Mishra Mechanics 1 with www.puucho.com

..., ..., ..., ...,


1-vw/E I= (5.00) = _! Solution: VR/M = VR/G--.VM/G
and tan0 ...,
_, I 20.0 4
I VR/E -4j= VR/a-(2i + 3J)
...,
or 0 = tan-1 .! = 14° VR/G = 2i- j
·4
(b) From Fig. lE.117, when man starts running downv'M;a·= -(2i + 3J)
..., ...,
d = (1.50) (tan 0) '
V R/M = VR/G-V 'M/G .

.
= (1.50) X-
1
= 2i-j+2i +3j= 4i +2j
4
= 0.375 m = 37.5 cm Speed =!.;:~Ml= .J16+4 = ..J20 mis
. ~77.:7~
g~~.im~~~~~ 'dE-x~~""le·,j
i§_ .. -=--~.: 120 ~-'lk•·.
~~=:c.::t.'i!. ~- .

- ~-~----·------- ~. •"-~ ~ - - - - , - - - ----- - ............... --- -·------- ----.··-- I


[A boat is movi~ towards eastwith v~lodty 4in/s w_ith re,1p~c9 IAn aeroplan~ A' is flying horizontally due east at a'.speed oJi
Ito still water _dnd river is flowing towards north w'.th veloc\tyi 1400 km/hr. Passengers in A, .observe another aeroplane Bi
2 fn/s and the wind is blowing: towards north wzth vel9c1tyi \moving [)er:p.eridicular to direction of motion at A. Aeroplane!
0

6m./s. ..The. -.di~.ec···t.iqn.. oftheflagblqwn_·_by


. "er,by the win.dh·o.~t _ Jd·.· ,B is actu:a(lY_:moving in a directicm 30_ north of east,il) thej
on the . boat IS • . >' , • .· ,1 •• same horizontal plane as. slzown m. the Fig. 1R120.,,
(a) northswest .
c,JJaTL'Jl/2.Jwith east~CdJ _,nr>rJL_·__
(b) sbuth,east . · . ·. · ·
. _ _ .\;:c;,
Determine tlje .tlodty of B. _____ ·· ·

·,,I
Solution:
:f~-4:··-·- _ _ _. . -· ~;+,.; II'
- ' ,, .. --3~
0 :...1__ EI
1B
' .-::.J.......... ~7' .. >
I
1,

450
Fig.1E.120 , ,
i., ~~=--=-.,,_~-- - ' -
~I__F_ig_:J_l::J18." Solution: ... (i)
-+ -+ -+ A A

,VB/G = VB/R+VR/G = 4i+ 2j


...,
Vw/G = 6i
- ,,

-+ -+-+ """"'"
. Vw/B = Vw-VB = 6i-4i-2j = 2i-2j
Direction will be north·west.

be=a~{ifie--.11119
• ,;;,-~~-::::.::..-.~~£::!l ~- j~

[r~-a marz ru_ntu_:ng Up.wa~ds on ~he_ ill; the ra~ appea_ rs tof_a !pl! .
Ii.

vertically dol1'.71wards w,th 4 fn/s; T!te ve/oc,ty vector pfi;tlze


man w.r.t. ear:th- is (2!+ 3J) fn/,s.·Jfthe man starts rr:lrzning

.......
down the hill with the same speed, then determirze tile reldt/ye
lspeed of the ralrz_ll!,r.t. 111arz. · ·· ·
'Y

....
I www.puucho.com
Anurag Mishra Mechanics 1 with www.puucho.com

' DESCRIPTION OF MOTION

A river is flowing with a speed of 1 km/hr. A swimmer wants


to go to point 'C starting from 'A'. He swims with a speed of 5
km/hr, at an angle ew.r.t. the river flow.' If AB =BC= 400 m.
30° At what angle with river bank should swimmer swim ? Then
Fig, 1E.121 (a) the value of e is:
400 m
-), " -), --t
Solution: = =

- Jo
VR/M Xj VR-VM B C

;M = 2v'3[cos30i+sin30j] = 3i+v'3J t
400m
..., A ,0 A

=> vR

rx
=-3i+(x-v3)j
! A
Fig. 1 E.123 (a)

'f i-/3
~
Solution:
Concept: Resultant path of swimmer is at 45° with
(b) (c) bank therefore x-and y-components of swimmer's resultant'
Fig.1E.121
velocity must be equal.
5=~3 2 +(x-v'3) 2 Condition for reaching the point C
=> 16= (x-v'3) 2 => 4+v'3 = X -->
..., A A
VM
VR =-3i+4j
tane = 3/4 => 0= 37°

·A pipe which can be swivelled in a vertical plane is mounted


on a cart (see Fig. lE.122). The cart moves uniformly along a
horizontal path with speed v 1 = 2 m/ s . At what angle a to the Fig, 1E.123 (b)
horizon should the pipe be placed so that drops of rain falling Vy
plumb with a velocity v 2 = 6 m/ s move parallel to the walls of tan45°= - , Vy= Vx
Vx
the pipe without touching them? Consider the velocity of the
drops as constant due to the resistance of air. (VR + VM COS0) = VM sin0
l+ Sease= Ssin0
On squaring,
1 + 25cos 2 e + lOcose = 25- 25cos 2 e
socos 2 e + lOcose - 24 = o
On solving, We get e = 53°

Fig. 1E,122
1__.§!',~~P'~ ·_124 L>
The minimum speed with respect to air that a particular jet
Solution: Rain drops will move parallel to the walls aircraft must have in order to keep aloft is 300 km/hr.
of the pipe if their velocity relative to pipe is along the pipe.
..., Suppose that as its pilot prepares to take off, the wind blows
First we find v Rain. Pipe· eastward at a .ground speed that can vary between O and 30
--t --t --t -), km/hr. Ignoring any other fact, a safe procedure to follow,
V Rain, Pipe = V Rain - V Pipe =V consistent with using up as little fuel as possible, is to:
-->
According to condition of problem velocity vector v (a) take off eastward at a ground speed of 320 km/hr
must coincide with axis of pipe. This will occur if (b) take off west.ward at a ground speed of 320 km/hr
tana=-=3
v, (c) take off westward at a ground speed of 300 km/hr
V1
((1.) take off westward at a ground speed of 280 km/hr
www.puucho.com
Anurag Mishra Mechanics 1 with www.puucho.com

----------·=·:__: MEC~~i~
~Soluti~m"~!>nc;pt: Fin~i,,;;;:ae of veloci~:01 for Umin• J(e) = 3sine-4cose should be maximum
f(elmax = 5
l~irc~aft relanve to w~d. Iv'A,lwl> 30~~: .____...,:.. ____ ~J · 16
Umin= -m/s
_, =Xl
Vw ' 16
5

(3 sine - 4cose) = 16
_,
VA =-VAi ' 5 .
3 sine - 4cose = 5
_, A/W = (x+vA)t'
V sin(e-a) = 1
_, e-a= 90"
I(v A/W) I= x + v A ~ 300
=> e = a+ 90° = 53°+90°
=> VA~ 300- X
e = 143° with river flow
x varies between 0 to 30 km/hr means v A ~ 300 km/hr
in westward direction. (.i.~~in1W~.fml> .
ri~~r has ~..·width. d. A fish_ ~nnan _in a b;~t eras;_ th;-_·.rlJ~;,_j•
1
-~ ' ' -- ----··- • - ' ._,,·J
¼
!twice. During the first crossmg, his goal 'ts to mmzmize~~he
IA man wants to !'.each poind3 on the opposite bank ofa :rivet/ fcrossing time: 'During the second. ,;tossi1!-&, his goal ~-Ni
l,zowing "at a.1peed. 4 mis as shown the Fig. lE.125 (a~.\ in
What minimrim:speed relative to water should the man ·have
!.$othcit he can reafhl?oint B'.directly6yswiniming? ln which
I lminimize the distance that the boat is carryed downstream;}rt1
[the first case, the crossing time is-1'0 • -In the second cdse,<thej
!crossing time is 3T0 • What is the speed of the ri~erflo'A;?; 'I;jn~l
ldirection_shouldhe,sWim?
.~ • i. -., I
~30 •...m..:_~---! -
·.·. ::·
-
11 · : ·b1-~'_l!L
I,,L_I/OSSL a wer. · · J ,.,..JC .."J'
.... - .- ·--·- ... - --« ~..~-------'-'-
! .I ,r---~==:"')--ll. I
i : I -~ol!J!io!l: ... --·--- __ ..... ·, , -·,- ______ , .....,:oc,
i''. .
\ , • .1
!
40m
4m/s I I
Concept: Case (iJ : If v 1,.. <.v B, boat can cross ~~r'I·
I .J , along a path perpendicular to flow.,, . . , . ;,;: ...
I . .,
l JC--_ __,__ _ __

lL-,,====""""'-----""'""'""-~,.,.L--_-:_--::::-_-_-.:_::::
I Fig. 1E.125 (a) :
f Case (ii) lfv~ < v R drift can ~otb~ zero appl_y caicul~t~!
lt./Ji.Lc.ase, ___ -··-- --···. _ --··--··· . . ·: · · · '1·, J
_.-~~ Case-I: If vR < vB

-.. :so.both
angle luti. ~---!l. ~
.•,.-~--n.~e.pt:we
are: I(nknown .N.o.have
te th·_·.a_ to
·.t.. s•_.·peed
determine~-,j swimme: _ ati._d.
function of ·_1_
Shortest Path:
VB
d
sine
:ITo .. :. (i)
~.peed and maximize· ·;t .time ta~en 'far x-component qf,j d
Quickest path: -=To .... (ii)
oc~C:.Qllll!onent of::di:splace1JJ.l!.~-~l!.'ll!2,. · ___ :. ,.,.. VB
I
0 --- - -
. 30';;;-·"·s'
also v R - v B cose = O for shortest path ... (iii)
Thus, sine= 1/3fromeqn.(i) and (ii) · ·-- --~-'"'~
40m
or VR = VB cose --~1c--.-.
·~-·-"·...
- :v•,J
= j__ '11- (1/9) = 2./2d
To :ITo Fig. 1 Ec126 . )
I __ ,_,..,.__.,.s,' 4'" ' ~ ,.,,

·case-II: If vB < vR
....Vs!R =ucosei+usinej
- -
x = ( . d_
· )cvR -vB-cos~)
_,
VRiE=Vi
- d
szne
VB
.
. ,,·
x = -.(vR cosece-vB cote)
. VB
~s;E = (ucos8+v)i+usinej
30 40 dx
----=--
For min. x, -=0
u cose + v 4sine de
=> 3usin8=4ucose+16 or vB(-cosec 2e)+vR cosececote = 0
3u sine - 4u cose = 16 cose = v 8 fvR .
16 Time taken in this case is given by
U=-----
3sine-4cose
www.puucho.com
Anurag Mishra Mechanics 1 with www.puucho.com

' DESCRIPTION OF MOTION

VP= [60 2 + 120 2 ]1/ 2 = 134.16km/hr


VJ\ 120
tan8=-=-=2
V1 60
Hence 8=tan-1 2

'lwo motor cars have their wind screens at 81 = 30° and


An airplane i.s observed by two persons travelling ilt 60: 6 2 = 15° respectively. While moving in a hailstonn their,
km/hour in two vehicles moving in opposite directions on a: drivers see the hailstones bounced by the windscreen of their,
straight road. To an observer in one vehicle the plane appears· cars in the vertical direction. What i.s the ratio vifv 2 of the
to cross the road track at right angles while to the observer in: velocities of.the cars? Assume that hailstones fall vertically.
. the other vehicle the angle appears to be 45°. At what angle: Solution: According to observers in cars hailstones
does the plane actually cross the road track and what i.s itsi bounce in vertical direction which implies that the angle of
speed relative to ground? reflection is 81 as shown in Fig. lE.128, which is same as
--;
Vp, angle of incidence in the cars' reference frame. Velocity of
A C .... ....
hailstones relative to first car is v - v 1 as shown in Fig.
--;
v, --;
lE.128.
8 Vp

0
Fig, 1E.127 (a) -:
Solution:
....
Let v p be the velocity of plane relative to
....
-:!_- - -;
v,
the ground, at angle 8 to velocity v 1 of observer in car 1.
Fig, 1E.128
.... ....
In case (i), VJ\ =Vp -V1
From figure,
.... .... V
or Vp=VJ\ + V1 a+ 28 1 = n/2 and tan a=-
--; v,
Vp,
C c• Hence tana = tan(1t/2-28 1 )F cot 28 1
V
or - = cot 28,
v, .
cc>
Vp, Similarly for second car,
--;
v, -
V
= cot 28 2
45'
V2
B B Therefore ratio of velocities of the two cars,
(b) (c) v 1 cot 28 2 =
3
Fig.1E.127 v2 cot 28 1
Vector diagram is shown in Fig, 1E.127(a). Note that
according to observer in car 1 the plane crosses the road at
right angles.
, -
!,, E:.x_qmpJ~
, ..... - ~,.,- - ... -
r ---
, 129
---- ----
--

An annoured car 2 m long and 3 m wide is moving at 13 m/s


Similarly, in case (ii) when a bullet hits it in a direction making an angle
.... ....
Vp =Vp +V2 tan-1 (3/4) with the car as seen from the street. The bullet
2
enters one edge of the car at the comer ·nd passes out at the
We can combine Figs. lE.127 (a) and (b). diagonally opposite comer. Neglecting any interaction
From the velocity diagram, between bullet and the car, find the time for the bullet to cross
tan 45° = AC the car.
AB
v!\ = (v 1 +v 2 )tan45°
= 120 X 1 = 120 km/hr
www.puucho.com
Anurag Mishra Mechanics 1 with www.puucho.com

lss MECHANI@

Solution: Letthespeed -.=·2m-. -+ ,. --+ ...

of the bullet be v. Velocity of '


bullet relative to car along
~
Jf: / _,
f _. are1 = -(g + a)j
Sret
1
=Ure1t+2arelt, we ave
and
2 h
sre1 = -hj in the formula

x-axis = (~ cos 8 :- 13) _and _m j,/ ,lvea,1 "-13 mis


along y-axis = v sm 8. Smee · ·
3
bullet appears from diagonally ·: . ,: :'
. · ./,' / -h] = _ .!_2 (g + a)t 2. 3'
opposite · corner, its
displacements relative to car @ • .'
"----' 1 This yields t-
-
~ g+a·
2h
along x- and y-axis are 2 m .__ Flg.1E.129
and 3 m respectivel}j · Concept: . If the elevator accelerates downJ
2 = (v cos 8 - 13)t '
i.e., ... (1) ci,e1 = "ii,- "ii,
1
= (-g]J-{-ai) = -(g - a)j. Substituting 1
and 3 = vsin 8t
On eliminating v from eqns. )~ and (2), we get
... (2) --+
arel
1....
' ·-~-+
= ;.._(g,;- a)J,, urel = 0
- REh
-
_[!.nd
--+
.srel
'
= 0,
• --+
m Srel
"""?'
= Urel t +
._ ..
· , t = _!_ ( -- - 2
13 tan 8
3

Equation of Moti!)n for Relative Motion:


0 =
13
= 0.15 s -a,.1t 2 , __ we- have t
2 ,· ·
=_
.
,For freely falling ·elevato~
_-·- _ - . '

a= g. Th~n, i = =. Hence the coin will never touch" the


g-a . . ·

.... .... .... surface. _It just ha~., below t/ze roof of the freely falling
Vrel = Urel + are1 t elevator.
--+ --+ 1-+ 2
s,e1 = Ure1 t +-2 a,e1 t
2
Vrel
2
= Urel + 20 ret 5 reI
Sometimes a body moves in a moving reference frame !Find the velocity of the coin relative to ground when_· it ~trikes I
/the /!_ase..]Jf.the elevator. •
or we have to analyses motion of a particle from point of
view of a moving observer. In such situations above --+ 0 --+ A

Solution: Substituting ure1 = 0, are1 = -(g + a)j, we


equations prove useful as illustrated by following examples.
have
----~·~J 130 ~
keoxa!m}:B).s.\,~c ~
v~1 = -2(g + a)j. (-hJ)
This yields = Vre1 = .J2(g + a)h. Since the coin moves
Consider yourself standing in an elevator which is moving down, (relative to the elevator), we have
with an upward acceleration a: A coin is dropped from res
om the roof of the elevator, relat:ive to you. After what time
...
V rel = -_i2(g + a)h j
-
the coinyil[ strike the base of the,elevator? '
As the coin strikes the elevator after a time t =~ 2
h ,
,(g + a)

~.
Solution: Here, we need. to apply the rr.=e;==a1
,, 11,,,
1 -> 2
-> -> •
formula s = ut + - at for the com •
relanve . a, r•1,ff
· · +a
9 the velocity of the elevator at that time is

to the elevator 2 t srawh _, _,


V = at = av{gtt)J
-+ --+_ 1 --+ 2 Flg.1E.130
Sre1 = Ure1 t +- arel t Then substituting ; rel = -.J2(g + a)hj and
2
_, _,
where t = time of fall of the coin, u re1 and are1 are the -; = a~ 2h j, in the equation -; , =; rel+;, we have
(g+a)
initial velocity and the acceleration of the coin· relative to the
elevator, respectively. Since the coin was attached with the
elevator, both coin and the elevator would move with equal
....
; , = (a~ 2
h -.J2(g + a)h
(g +a) .
)j =- ~ 2gh
1+!!.
j
velocity at the time of release of the coin. Hence u rel = 0. ' g
_, -
a,.1 = -(g + a)j_ b~~~~RJ.~.J 132 ~
As the coin moves down with a displacement of
magnitude h,
_, -
= -hj. Now, substituting
_,
= 0,
IA lift is movi~ with uniform downward acceleration of 2 /
sre1 ure1
Im/s2 • A ball is dropped from a height 2 metre from the fl_ oor ofi1'
rlift. I:ind the time after which baU will strike the floor. .

www.puucho.com
Anurag Mishra Mechanics 1 with www.puucho.com

DESCRIPTION OF MOTION ggj


Solution: Initial velocity of ball with respect to lift
....
Ure! =0
k:~gm~'fJrnl>
.... Cann~; A is loca;d cin a pl-am-._a_d_is-ta-nce L from a Wall o.fi
ab= -g = -10
.... height H. On top of this wall is an irlentical cannon (cannon
u, =-2· ~BJ. Ignore air resistance throughout this problem. Also ignore
acceleration of ball with respect to lift the size of the cannons relative to L and H. The two groups of,
--+ --+ --+ 2 gunners aim the cannons directly at each other. They fire atl
.are! = ab- a 1 = -8 m/s ,each other simultaneously, with equal muzzle speed v 0. What/
Displacement of ball with respect to lift till it strikes the ;is the value ofv 0 forwhich the two cannon balls collide just as
floor /they hit the ground?_ _ __
' B

.....··
..···· H

·, L

t·· ·-·----
L____ _ )_..c.._________.
...._Fig.1E.1~ (•..

IA toy train moves clue north at a constant speed 2 nVs along a Solution:
straight track which is parallel to the wall of a room. The wall
2v~
v 0 cos e(- 0 sine)
- +v 0 cos e(2v 0 sine)
-~ - =L
is to the east of the track at a distance 4 m There is a toy dart g . g
gun on the· train with its barrel fixed in a plane perpendicular
to the motion of the train. The gun points at an angle 60° to H
sine
the horizontal. There is a vertical line drawn on the wal~
stretching from fl.oar to ceiling, and the dart gun is fired at the L H
instant when the line is due east of the gun. If the dart leaves, cose=,====
~H2 +L2
/the gun at speed BnVs relative to the gun,find the distance by[
1which the dart misses the vertical line. That is, find how far 1 , v2 gL L
· (north or south of the vertical line is the point at. which the i 0
4sinecose Fig.1E.134 (b)
g(L2+H2)
!~~h~ts-the wal~) 3m ---· (c) lm .. __(d) Sm ---- _j Vo=
4H
Solution: Consider east as x, north as y and
vertically upward as z velocity of dart w.r.t. to train at firing

¼platform is moving upwards with a constant acceleration ofl


, ·r ~y(N'. )~all !2tn1sec 2 • At time t =Q a boy standing on the platform throws
/a ball upwards with a relative speed of Sm/sec. At this instant
,..· .··
l ~ I
x(E) I ;platform was at the height of 4 m from the ground and was
. :moving with a speed of 2tnlsec. Take.g =l0m/sec 2 • Find
I• d •• -··
.•
I ( a) When and where does the ball strikes the platform?
'I '(b) Maximum height attained by the ball from the ground;
--·· __ Fig. 1E.133 •.••.• 1
'(c) Maximull!- distance_ of the ball _fr2m the platfg_rm. __
;d, =8cos60°i+Bsin60°"ic= 4i+4J3k Solution: (a) We solve the problem in reference frame
velocity of dart w.r.t. ground at firing of platform
.... A
---t •-) --+ A Ar;:;A A

ud = ud,+vr = 4i+<tv3 k+ 2j V Ball/platform =8 j


-) A -) A

Time taken to strike wall t = df 4 = l sec aP/E =2j and aB/E =-g j
Displacement along y = 2 x t = 2 m (North) .... .... A

are! =aB/P =-12j


1 2
by Srel = Ure! t + 2 arel t
www.puucho.com
Anurag Mishra Mechanics 1 with www.puucho.com

I 1·00·
LI..=-.;c.__~.::;t:...._.._.._:_... _ _ _ _ · ~ - - ~ - - - ~ - - - - - - - -
MECHA~~

0 = Sxt-_! X 12t 2 For vertical motion


2 .!gt 2 + (200sin0)t _.!gt 2 = 1000
4 2 2
t = - sec
3 sin0t = 5 ... (2)
. 4 10 From (1) and (2)
Tota1 tune = 2 + - = - sec. sine 1
3 3
1+ cos0 - ../3
. 1 . f 1 ~ .. lO
dtsp acement o p auorm m - sec. On solving, 0 = 60°
3
2 (ii)
=4+2x.i+.!x2x(i) -, A A

3 2 3

r-------A -- ·.. --:·1


v 8 = -200cos53° i+200sin53° j
76
=-m 30-8
9
-, -, A

(b) = lOj & aB/E = -lQj


rL' ' ~ !j
V B/E
2 2
by v = u + 2as w.r.t. earth
(0) 2 = (10)2 - 2(10)s1 : ; v3 ·
[_____ Fig.1E.136 (b) -·--
s1 = Sm
Hmax=5+4=9m = -2oox~i+ 2oox.iJ
(c) Also platform frame 5 5
. v2 =u2+2as = -120i+160j
or (0) 2 = (8) 2 + 2(-12)s -, -, -,
VA/B =VA-VB
8
or s=-m
3 = (200 + 120) i - 1'60 j
~g:xam.Rle6~
' ' . -- --· : ' ; ------------·· - ..1
tan0 =I-~~~I=½
,IAn aircraft i§ .flying. horizontally with a constant vefocity AB= 2km
i= 200m/s, at a·height =lkm. ciboye the ground. At the. BP = minimum distance = AB sin(30° -0)
tmoment shown, a. bomb is released from the aircraft and the
BP = 2[sin 30° cos0 - cos30° sin0]
jcan~on-gun ~elow fires a shell with. initi~l speed =·~O~ m/s, at
,some arigle.0. ·For what value_of'0' will the proJectile shell
1destroy
I .
the bomb in mid,air?• If the
.. • . . , •
value of0 is 53°, find
..
the.
= 2[½ ls-~
X X Js]
:minimum distance between the bomb and.the shell as they.fly
:past each other. Take sin 53° = 4/5. ' _ .... __ = 2-../3 km
,. -~ ..Js
----~.... ..
· I 1 km · ··n. _ ~ r·--.
1A balloon
"
is moving vertically upward with constant
-~--,

!_ , __ - - -
I.
:
:

i

.
1
·...
:-.\'\'\'\\.'\)$'\'-"\~\\,,\\\\,,\\\\'\.'\'\~\.,-W.........
.v3km
Flg.1E.136 (a)
-=-=-_.,...,,.~,.,--=--' -------~-
• • ·~,_.,,._,...._...,,.,,.... __
0
••

i
5 .... !
'
t

,
I ~
!acceleration (g /2) in upward direction Particle .'il was
!dropped from the balloon and 2 sec later another particle 'B'
'was dropped from the same ballooni Assume that motion ofi
the balloon.remains unaffected. Find the separation di.st.ailce
between 'A'.and.' B ', 6 sec after dropping the particle 'B ', !'JO[!e
of the particles reaches the ground during the time interval
urrd~Lf.Oll§igqggo1i (g-=.10 m,ls_gc_~) __ . __ .
Solution: (i) Suppose shell destroy the bomb at time
't ' then for horizontal motion Solution:
t(200 + 200 cos0) = ../3 x 1000
t(l + cos0) = s../3 ... (1)
! ·- c~~;~pt: s;-:c~- ~articles are being dropped fro~-~
:moving body i..e., a moving reference frame, we used reference/
. ,frame of balloon itself for both the pa_rtz_·c_les_._ _ _ _ _ _)

www.puucho.com
Anurag Mishra Mechanics 1 with www.puucho.com

I,DESCRIPTIQN Of l\'IOTION '1.01 I


Motions of particles '.A' and 'B' are Acceleration of platform relative to the ground
_, _, A

w.r.t. balloon, so balloon is reference ap~gr = (a) j .


point therefore it is assumed to be at rest.
We denote balloon by b Velocity and acceleration of the particle with respect to
the platform
For A u,01 = 0 -> A A

a,.,= -2
3g u,.1 = (ucos0)i+(usin0-v)j
-> A

1 2 and a,.1 = -(g + a)j


Sret = Urelt +zarelt
At the end of flight y-component of displacement of
s,., =0-21(3g)
2 (8) 2 projectile relative to platform becomes zero.
1 2
(A falls off 8 sec) Y rel = UYrel t + 2 ayre/
Srel =SA/balloon= -48g = -480m
3g Yrel =(usin0-v)t-.!.(g+a)t 2
For B urel = 0; arel =- - 2
2
1 2 0 = (u sin0-v)t - .!.(g + a)t 2
Seel = Ure1t +-are1t
2 .
' 2
2(u sin0-v)
1 3g 2 t
s,el =SB'balloon = 0---(6) g+a
· ' 2 2
SB/balloon= -27 g = -270m or t =0
Separation distance between ' A' and 'B' and T= 2Cusine-v) (time of flight)
= ISA/balloon 1-1 SB/balloon I
g-1:a
= 480-270 = 210m 2ucos0(usin0-v)
Range CR) = (u cos e)T =
Projection of a Particle in an Accelerated g+a
Elevator Projection of a Ball in a Horizontally Moving
A hori2ontal platform is moving vertically upward with Trolley
constant acceleration' a'. When velocity of the platform is A trolley is moving horizontally with a constant
'v ', a particle is projected from the platform with velocity' u' acceleration' a'. When velocity of trolley is' v', a particle is
relative to the ground at an angle 0 with the horizontal. Find projected with velocity' u' at an angle 0 above the horizontal
the hori2ontal range and the time.of flight of particle on the from the position which is
platform. at distance 11 from the •y
I
front wall and 12 from the ;
Concept:, In case of projection from a moving platform
rear wall. This velocity and . 1·

entire motion takes plane on platform, always use reference


angle of projection are hf v .
!frame of platform.
----.---- relative to the ground.
_, ->
a
Velocity of projectile relative to the ground Velocity vector u, v and +'---,.,r-""'--.,,.,.-'----+!
-> A A -> X'
u P/gr = (u cos0) i + (u sin 0) j acceleration vector a are in I
la upward
the same vertical plane. I__ _ I
l y
v j moving
Find the time of flight and Fig.1.106 _ --·---
the horizontal range of particle on the trolley. Also discuss ·
the condition for whicli particle will fall

~~,,~:~J
Acceleration of projectile relative to the ground
(i) in front of point of projection
(ii) at the point of projection
(iii) behind point of projection
_,

uP/gr = (ucos0)i+(usin0)j;
A A

-> A

aP/gr = (-g)j
.' and·
->
aP/gr = -(g)j
A

Velocity of platform relative to the ground at the time of


projection
-> A

and a,1gr = (a) i


www.puucho.com
Anurag Mishra Mechanics 1 with www.puucho.com

.. ~·
102 .. ,..,-:o< MECl-l~NICSi!j
->
u,., = (u cose-v) i+u sinej
-> A A
A A

l-----
Concept: (i) If the di;ection of relative velocity is
:directed towards , th~ position of;A' then the body \Bl meets
and a,el = -(g)j-(a)i ·'A1 , .
We assume that the flight completes on the floor of
trolley. It does not strike the roof or the front wall or the
!through
1
.(ii) theWhenposition
.direction of relative velocity does_ not pass
of' A' then perpendicular' AN' f,wn the
back wall. ': . · ·i _4
1 2
Iposition of'.,!:' on the line of action of relative velocit_y(vB/A)
Y rel = UYrel t + 2 aYrei t
!gives the m_ ·. _i_'nimum possible .,d_ista. nee between 'A_.·, a. nd 'JB'
0= (usine)t-~gt 2 iduring their'_motion · , . ·. ·· ,
2 -···--•.. - _ _ AN= dsino: · · .. ,
2u sine ->
Thus time of flight is T = --- a is the angle which v B/A forms with y-axis.
g
For range
-> A
First we will determine velocity and acceleration of vB/A =-(vBsin0 2 +vAsine 1 )i
particle in reference frame of trolley.
Horizontal range as observed from the trolley +(vB cose 2 -vA cose,)j
1 2 (vB cose 2 -v A case,)
Rre, =(ucose-v)T--aT tan ex = --'-.;...---"'---"--"--
2 - (v B sine 2 -vA sine,)
As observed from the ground
From tan a determine sin a and· cos a
u 2 sin28
Rrel Time required to come closest is given by
g
BN dcoso:
Closest Distance of Approach Between Two t=--=--
-> ->
Moving Bodies lvB/~I lvB/Ai
->
, Two bodies are moving with constant velocities v A and Ll:E:~p~l~~
->
v:i, as shown in Fig. 1.107.
r1wo roads •_z_~.;erse~~ at righ/a_ngle-.s-._C_a_r-.A-.-is-s-itu_a_t_e_-a.a:·-;i
y ~
which is 500 m from the intersection O on one of the roads.I
:' Car B is situated at Q which is 400 m from the intersect/.6n on
A_,; the other road. They start out ,at· the same time and, ti:aVel
:: towarcls the intersection at 20 mis and 15 m/s respectively.

... :
' '
..... i
:.
: .
What is the}ninimuni distance between them? How'long' do
they tdke to reach it? _ _ _ _ _ · ·· ·

E.
500m
.' :-' .
~ 20 mis ~ L _ - - - ! 0
:fd . · vA
..
. .

••••• ')
~
.
:. . . .S t .

..._.... _... --· i


\
~
Vs :

a ,
'a i
vi;~~-,~·
J, 'i)J),JIV P Car A

-'vl\.
2 .
..
. -·········----····>x
400m

B CarB
Flg.1·.f07
-> A A

v A = (v A sinei) i+ (v A cose,)j
Fig.1E.138 (a)
-> A A

and sine2H+ (VB cose2)j


VB= -(VB

Motion of' A' relative to' B', is along a straight line in the
->
direction of relative velocity (v B/A).

www.puucho.com
Anurag Mishra Mechanics 1 with www.puucho.com

DESCRIPTION OF MOTION

Solution: First we find out A 20


the velocity of car B relative to A. e 0
As can be seen from Fig.
lE.138 (a), the magnitude of
15
velocity of B with respective :
vA = 20 m/s, vn = 15 m/s,
OP = 500 m; OQ = 400 m
15 3 4 Fig.1E.138 (b)
tan0=-=-· cos0=-·
20 4' 5' B D
Fig. 1E.139 (l>)
. 0 = 3-
sm
5 Therefore, relative
3 acceleration between them is
OC = ADtan0 = 500x- = 375 m
4 zero i.e., the relative motion
.p 500 m between them will be straight A 10mis 5.Jam/s
BC =OB-OC
=400-375=25m
e o line. Now assuming A to be at
rest, the condition of collision
BD = BC (cos0) 625m c 375m .., .., ..,
4 will b e that V CA = V c-V A = Fig.1E.139 (c)
=25x-=20m
5 relative velocity of C w.r. t. A
should be along 0\.
Shortest distance = 20 m a e
-
..,
PD= PC +CD
= 625+15= 640
Fig.1E.138 (c) _j ..,
VA=

..,
lOi
-
Vn=-5i-5v3j
,n' -
VaA
C

--- 30;60°· 1
10m
r
VBA A

'T
l~ABl=25m/S =-5i-5..J3j-10i
640 .., - r;;'
t =- = 25.6 sec :. VBA = -15i-5v3j
25
' .. .
r.;:;;:;l • :. tan60°= ~
l}=~g,tp:12;1~ 11391.> 10 --'ss~=d=~o
d=l0-./3 m Fig.1E.139 (d)
Two towers AB and CD are situated a distance d apart as:
shown in Fig. 1E.139 (a). AB is 20 m high and CD is 30 m
high from the ground. An object of mass mis thrown from the
l~c:;;;~m21~8> -
top ofAB horizontally with a velocity of 10 m/s towards CD. ·
,On a ftictionless horizontal surface, assumed to be the X·Y
C .plane, a small trolley A is moving along a straight line.
:parallel to the y-axis [see Fig. IE.140 (a)] with a constant:
,velocity of ( ,Jj-1) m/s. Ata particular instant when the line'.
Am
:oA makes an angle of 45° with the x-axis, a ball is thrown
ialong the surface from the oriiµn 0. Its velocily makes an'
Bi Di
·angle <I> with the x-axis and it hits the trolley.
·y
Fig: _11:.139_ (a)

Simultaneously another object of mass 2m is thrown from the _..nA


·top of CD at an angle of60° to the horizontal towards AB with
the same magnitude of initial velocity as that of the_ first' .· •'45°
X
.?bjec~. Tl'.e two ob!ects move in the same vertical plane, collidej 0
m mtd-mr and suck to each other. : Fig.1E.140 (a)
Calculate the distance d between the towers. :Ca) The motion of the ball is observed from the frame of the
2 trolley. Calculate the angle 0 made by the velocily vector
Solution: Acceleration of A and C both is 9.8 m/s
· of the ball with the x-axis in this frame.
downwards.
(b) Find the speed of the ball with respect to the surface, if
• _ <I> =_40/3_. --- __ _

www.puucho.com
Anurag Mishra Mechanics 1 with www.puucho.com

.
- - - ~ · - - - - :::'::::::====::==· ·~';:;::;::==:'.:::::::::,~-_:'.:::.::..:J:::;
Sol""•".' [,) ""A ~ml, fu, ®lley rnd B fud•lL ~· -
Relative velocity ~f B with respect to A(~_;!A) should be
1 I
·
F ·' 8
P . a'
,

· j
. 1)1~(1!A~ld-il,S,I

alongOAfortheballtohitthetrolley.HencevBA will make ,',"' , :: Fig.lE.


141
(•) j , . "' .
450 th
an angle of wi positive x-axis. (a) find ihe?listance,_a_lo_n""g-th-e"bo-tt-o;;;~f the bo)( be·i:ween the
(b) tan0 = vBAy = tan45° .· point of projection P and the point Q.where the particle
v BAx _' la!1ds'.(Assunie that tlie pafticle does not hib:a!lS{·other
or v BAy = v BAx ... (1) · surfate,of the box. Neglec(i:Lir resistance) '" ~
Further
or
vBAy =vBy - v Ay
2 . (b) If
ant·h.observer
J. h• o.rlz···on···t.al
on the disp.la
ground•is ~
..c.em.· .nt_o.if
zero,th.e pa. r.tide··.as·····
find the speed en the
. •·.s.". of by
v BAx = v Bx - O ... C ) bo~- with respect to the ground 'at the instant When the
VBAy = VBy -(-J3-l) 3
... ( ) ,__,,.P.f!!:tigle \'!IJ§..PI9jected. , ,, . . . . • --- •
tan<jl = vBy Solution: (a) Acceleration of particle and box both are
vBx shown in Fig. lE.141 (b).
or vBy = tan<jl · vBx ... (4)
' ·..· '·--··/",,.1
From eqs. (l), (2), (3) and (4), we get
or vBx ("3 -l) and vBy ("3 - l) · tan<j, · ~·.. ·i ·1J !
:
g'.'.s.in.e..
tan<J>-1' tan0-l · .. 8 i
" '
•~'-----·-F_i~g;~:1E:1M (b) · - - ,
<I>= 40 = ic4so)
3 3 Acceleration of particle with respect to box
Speed of ball w.r.t. surface vB = ~v~ +v~ = Acceleration of particle - Acceleration of box
"3-1 = (g sin0i + g cos0j)- (g sin0)i = g cos0j
=---sec<J> Now motion of particle wlth respect to box will be
tan<jl-1
projectile as shown in Fig. lE.141 (c).
Substituting <jl = 60°, we get ,· ·.·. 7
VB= 2m/S
Alternative: Relative to frame of A ex
r-- ._.... -- - ~;r--
e
R.:.
gc~s·e'
lv sin 150. v~,)\· ·
15• Fig, 1E.1ll1 (S)__
The only difference in g will be replaced by g cos 0.
· ·u 2 sin2a
PQ = Range(R) = - - -
. g case
' 45 u 2 sin2a
0
08
PQ
• ~['_··._.·--~_i;"'~-'1=E:140j~:__c__ _·~
gcos0
(b) Horizontal displacement of particle with respect to
Resultant velocity is along OA, so perpendicular ground is zero. This implies that initial velocity with respect
components = ·O . to ground is only vertical, or there is no horizontal
VB X5m/s= ("3-l)COS45° component of the velocity of t;he particle.

.
VB= (-J3-l) ,_!_ = 2m/s
Sm/s ,/2
t ,.·
y/;in (cxu+8) ~1
1

~~A~FJI~ 141 ~ . Gi.:1.·


'!-_<-- --
ucos(a+0)
· /VCOS8
.. ,j
L4 large h~~ box ~ sliding witho~;fricti~n °dow.n J:~;,;;,:J . . 8 ,.J· ,. ""--"'-'-=---,1(,
/Jilane <if inclination 0. From a pq{nt.P on• the bott6,,r,ofth;1 f'----·"-''~'_·---~Fig. 1E;141: (~L ______,
lbox; a particl~ is.projected inside:tJie box. The initigl sp~ed p}j Let up, b(Hl is. component of velocity of particle w,r.t. box
in horizontal direction.
'!th·e· par.,tz.'¢1e :W. .·. .ith respect to th~ bo>1.. is u.'.and the, d.ir,ectt.;o·'·il····.:'o··1
.pro;ectton makes an ·angle a Wlth,.the' bottom as s/town·m 'the up,b(Hl =ucos(a+0)
jFig. 1E.14lfa). _ _" _ , ' , _ ·. · , ., :·,,',: If vb is .speed of block along the incline w.r. t. ground.
www.puucho.com
Anurag Mishra Mechanics 1 with www.puucho.com

I DESCRIPTION OF MOTION 1051


vb H = component of box's velocity in We find arbitrary constant c by employing initial
' horizontal direction conditions v = 0 at t = 0 which yields
VbH = -Vb COS0 mg
c=ln--
Now, -ub,g(Hl
up,b(H) =up,g(H) k
up,g(H) = up,b(HJ +ub,g(HJ On substituting the value of constt. c in eqn. (3), we get
=ucos(o:+0)-vb cos0 1n(mg -v)-lnmg =-!t
But, as up,g(Hl = 0 k k m
~ u cos(o: +0)-vb cos0 = 0
u cos(o: +0)
or 1n{mgfk-v} = _!t
vb= mg/k m
cos0
or. v = mg (1-e-Cl;'m)t) ... (4)
k
,------ - ---,,." ---··----·-------- - ·- ···~ l = mg (1- e-<f,)
!A small sphere of mass mis released from rest in a large vessel k
i_filled with oil where it experiences a resistive force where T = m is called time constant.
;proportional to its speed, i. e, Fd = - kv. k
'(a) Find the law according to which the'ball's speed varies. (b) When the particle reaches terminal speed, the
'(b) After a certain time the sphere reaches a terminal'speed; acceleration of the particle becomes zero. When the
: find it. magnitude of the resistive force equals the sphere's weight,
1
( c) Time constant, is the time it takes the sphere to reach acceleration is zero and from then on the particle continues
\ 632% of its terminal speed; find it if m = 2.00 g and to move at constant speed called terminal speed.
I terminal speed is 5. 00 cm/s. mg= kv, or v, = mg/k
'(d) Determine the time it takes the.sphere to reach 90% of its
(c) k = mg = (2.00)(980) = 392 g/s
j terminal speed. 1 , , , . - - - - ~ · v, 5.00
m 2.00
Time constant, T=-=--
i k 392
.I = 5.10 X 10-3 S
. (d) Speed of particle as
function of time is given by eqn.
I v "')>
;,_0/
Fig.1E.142 (a) (4). . ~0/
Vt •••••••••••••••• - •••

Solution: (a) Force acting on sphere = mg - kv i 0,63v1 ••• ••_:;-' .

where k is a constant. We have assigned downward direction I :


positive and upward negative. or 0.900v, =v,(1-e-C</<l)
~
Acceleration of ball
dv
dt=g-mv
k

Separating variables, we obtain


... (1) or
or
or
1- e-,1, = O. 900
e-</< = 0.100
-t/T=ln(0.100)=-2.30
t = 2.30T
l O
Fig. 1 E.142 (b)
t l
I

dv =-!dt ... (2) = 2.30 X (5.10 X 10-3 s)


mg -v m
k = 11.7 X 10-3 S
On integrating the above expression, we obtain In the graph of v versus t for the ball, the slope of v versus
t graph gives a. At t = 0, v = 0 and a= g. Ast becomes large,
ln(m:-v)=-:t+c ... (3) · v approach es v, and a approach es zero.

www.puucho.com
Anurag Mishra Mechanics 1 with www.puucho.com

110s . ' ,MECHANIE~J

_, _, _,
9. The resultant of a and b makes angle a with a and~
_,
.with b, then (a, b represent magnitudes of respective
i.It is possible to add five. unit vectors to get an unit vectors):
vector, The statement is : (aJ a < ~ (bJ a < ~ if a < b
(aJ True (bJ False (c) a <~if a> b (dJ a<~ if a= b
2. If a vector is rotated by angle 0 then it is necessarily
_, _, _,
10. Let C = A+ B.:
changed. The statement is : _, ,_,
(aJ 1hie (bJ False (aJ ICJ is always greater than IA 1.
3. It is possible to add n vectors of equal magnitude and -+--+ --+--+
get zero: (bJ It is possible to have IC l<IAI and ICl<IBI
--+ -+ --+'
(aJ True (bJ False (cJ ICJ is always equal to IA[+[BI
4. It is possible to add n vectors of different magnitude _, _, _,
and get zero.
(dJ [C[ is never equal to IAl+IBI
. _, _,
(aJ True (bJ False 11. Let the angle between two non-zero vectors A and B
--+ --+ --+ --+ _,
5. It is possible to have a x b = a . b for some suitable be 120° and its resultant be C. Then:
--+ -+ --+ -+ --+ --+ -+'
selection of a and b. For example a = 0. The statement (aJ ICJ must be equa!I IAI-IBI I
is: _, _, _,
(aJ True (b) False (bJ ICJ must be less than I IAI-IBI I
_, _, _,
6.
--+ -+ --+ -+
IflAI= IBI and A,;, ±B then angle between the vectors (cJ ICJ must be greater than I IAJ-IBI I
-+
_, _, _,
--+ --+ --+
(A+ BJ and (A-BJ is: (dJ [Cl may be equal to I IAI-IBI I
(aJ 0 (bJ it/6 12. Which of the following two statements,,- is more
(cJ it/3 (dJ it/2 appropriate? '·
7. A vector of magnitude a is turned through angle 0. The (aJ Two velocities are added using triangle rule
magnitude of change in the vector is given by: because velocity is vector quantity.
(aJ l2asin01 (bJ l2asin0/2J (bJ Velocity is a vector quantity because two velocities
are added using triangle rule.
_, _,
13. Vector ais increased by/!,. a If increment in magnitude
_, .
8. Which of the sets given below may represent the of a is greater than_,magnitude_, of increment vector
magnitudes of three vectors adding to zero? then angle between a and /!,. a is:
(aJ 2, 4, 8 (bJ 4, 8, 16 (aJ greater than it/ 6 (bJ exactly it/ 6
(cJ 4, 8, 4 (dJ 0.5, 1, 2 (cJ exactly it/ 2 (dJ <I>

www.puucho.com
Anurag Mishra Mechanics 1 with www.puucho.com

F_M_OT_IO_N_ _ _~ - - - - - - - ' - - - - - - - ' - - - - - - - - - - - - - - - _10i]


c...D_E_SC_RI_PT_IO_N_O__
-, -,
14. A motor car is going due north at a speed of 50 km/h.It
20. If a 1 and a 2 are two non-collinear unit vectors and if
makes a 90' left turn without changing the speed. The --t--t --t--t --t--+
change in the velocity of the car is about : Ia 1 + a 2 = ../3, then
J the value of (a1 - a 2 ). (2 a 1 + a 2 ,)
(a) 50 km/h towards west is:
(b) 70 km/h towards south-west
(c) 70 km/h towards north-west
(a) 2 (b) 3/2
(d) Zero.
(c) 1/2 (d) 1
15. A person moving on earth's surface starts from north --t --+ --+ --+ --+ --+
pole & moves 500 km towards south and then moves 21. If A = B +C and the magnitude of A, Band Care 5, 4,
1000 km towards east and then again moves 500 km -, -,
and 3 units respectively the angle between A and Bis :
towards north and stops. The displacement of the
person is:
(a) 1000 km eastward
(a) ~os- (¾)
1
(b) cos- 1 ( ~ )

(b) -Jiooo 2 + 500 2 km towards south-east


(c) (~) (d) sin-1 (~)
(c) ~1000 2 + 500 2 km towards
(d) Zero 22. The sum of two forces acting at a point is 16N. If the
16. A person moves 20 m towards north-east then moves resultant force is SN & its direction is perpendicular- to
20 m towards west and then again moves 20 m minimum force, then the force is :
towards north-east and stops. The magnitude of (a) 6N and lON (b) _SN and SN
displacement of the person is: (c) 4N and 12N (d) 2N and 14N
(a) 2W5-2./zm (b) 20 m 23. What is the component of 3 i + 4 j along i + j :
(c) 2W5 + 2-J2m (d) None of these 00 !d+J) (b) ~d+J)
-, -, -, 2 2
17.' If A, B, C, are mutually perpendicular vectors then
which of the following statements is wrong? w ~d+b
2
~) !d+j)
2
-, -, -,
(a) C X (AX B) =0 24. At what angle the vector (A+ B) and (A - B) must act,
--t --+ --+ --t

-,
C so that the resultant is ~ A 2 + B 2

--t --t --t --t --t --t


(a) cos-1 (AA2+B2B2)2 - (b) cos~! (AA2-B2
+ B2)
2

(c) A.B = B.C = C.A =0


-, -, -,
(d) (B +C) is perpendicular to A (c) cos-1 ( A 2 - B2 ) (d) cos-1 ( A 2 + B2 )
. 2(A2 +B2) 2(B2 -A2)
18. The velocity of a particle varies with time as per the -, -, -,
--t --t --t --t -t
law V = a+ b t where a and b are two constant 25. The resultant of A and Bis perpendicular to A. What is
-, -,
vectors. The time at which velocity of the particle is angle bet)veen A and B ?
perpendicular to velocity of the particle at t =0 is: 1 1
-, -, (a) cos- (;) (b) cos- (-;)
(a) _j& (b) _I aJ2
--t --t
a.b
-, -,
--t --t
a.b .
(d) sm -1(- A) B
a.b
(c) -, (d) None of these 26. A particle moves through angular displacement 0 on a
JaJ 2 circular path of radius' r'. The linear displacement will
be:
19. A plane is inclined at an angle 30° with horizontal. The
-,
component of a vector A =- lOfc perpendicular to this (a) 2rsin(~) (b) 2rcos(~)
plane is: (here z-direction is vertically upwards)
Ca) s..!z (b) 5F3
(c) 2rtan(~) (d) 2rcot(~)
(c) 5 (d) 2.5

www.puucho.com
Anurag Mishra Mechanics 1 with www.puucho.com

108
~ ME(H~l'U(S-1
- - - - · - - - - - - - - ~ - - - - ~ ' _ , ; , __ _ _ _ __,:_ _ _ _ _ _ _-~-------'-'--'-_CJ· I
_,
(a) unit vector
2 7. If a vector A makes an angle a, ~ and y with X, Y and Z
(b) null vector
axis respectively then sin 2 a+ sin 2 ~ + sin 2 y =..... _, _,
(a) 0 (b) 1
(c) vector of magnitude between IA I and IBI
(c) 2 (d) 3 (d) nothing can be said
_, ' ' 36. Six forces are acting on a particle. Angle between two
28. The X and ¥-component of Pare 7i and 6 j. Also, the X adjacent force is 60°. Five of the forces have
--+ -+ A A

and ¥-components of P +Q are lli and 9j respectively. magnitude F1 and the sixth has magnitude F2 • The
_, resultant of all the forces will have magnitude of:
Then magnitude of Q is : (a) zero (b) F1 + F2
(a) 7 (b) 6 (c) F, -F2 (d) F2
(c) 5 (d) 13
29. Two vector A= 2i+ 3]-4k_, and B_,= 4i+ 8] + xk are =-~~N~A~C!:~
,, _,
such that the component of B along A is zero. Then the 3 7. The quantity V dt represents:
value of x will be:
J
t1
(a) Distance travelled during t 1 to t 2
(a) 8 (b) -4
. (b) Displacement during t 1 to t 2
(c) +4 (d) -8
--t,_,_,_ -+,_ A (c) Average acceleration during t 1 tot 2
30. Two vector A = 3i + 8j - 2k and B = 6i + 16j + xk are (d) None of these
_, _,
such that the component of B perpendicular to A is _, = Vx 'i + Vyj' + vz..,
38. Let V J''
i'. then ,, Vydt represents: (for
zero. Then the value of x will be :
(a) 8 (b) -4 the
duration t 1 to t 2 )
(c) +4 (d) -8 (a)Distance travelled along y-axis
31. A blind person after Walking 10 steps in one direction, (b)Displacement along y-axis
each of length 80cm, turns randomly to left or right, (c)Total displacement - displacement along y-axis
After walking 'n' steps, the maximum displacement of (d)Total distance travelled - distance travels along
person is 16-/2 .Then value of'n' is : y-axis
(a) 20 (b) 30 39. A particle has a velocity u towards east at t = 0. Its
(c) 40 (d) 60 acceleration is towards west and is constant. Let x A
_, _,
32. Two vectors A and B have magnitudes 2 and 2-./2 and x 8 be the magnitudes of displacement in the first
-+ -+ -+ -+ 10 seconds and the' next 10 seconds then:
. respectively. It is found that A. B =IA x BI , then the (a) XA < Xn
_, _,
(b) XA = Xn
value of ~
_, _, will be: (c) XA > Xn
A-B (d) The information is insufficient to decide the
(a) 5 (b) .rs relation of xA. with x 8 .
-./2 + 1 -./2 - 1 40. A stone is released from an elevator going up with an
(c) (d)
-./2-1 -./2 + 1 acceleration a. The acceleration of the stone after the
release is:
33. If the resultant of two vectors having magnitudes of 7
(a) a upward (b) (g - a) upward
and 4 is 3, then the magnitude of the cross product of
the two vectors will be: (c) (g - a) downward (d) g downward
(a) 28 (b) ../65 41. A person standing near the edge of the top of a
(c) 53 (d) zero building throws two balls A and B. The ball A is thrown
34. The adjacent sides of a parallelogram is represented by vertically upward and Bis thrown vertically downward
vectors 2i + 3j and i + 4] . The area of the with the same speed. The ball A hits the ground with a
speed v A and the ball B hits the ground with a speed
parallelogram is :
(a) 5 units (b) 3 units v 8 • We have:
(c) 8 units (d) 11 units (a) VA >Vn
(b) VA<Vn
35. The maximum magnitude of cross product of two (c) VA=Vn,
vectors is 12 units and the maximum magnitude of
(d) The relation between v A and v 8 depends on
their resultant is 7 units, then their minimum resultant
vector will be a: · height of the building above the ground.

www.puucho.com
Anurag Mishra Mechanics 1 with www.puucho.com

DESCRIPTION OF MOTION 109!


42. A body traveling along a straight line traversed one 48. The position vector of a particle varies with time as
-+ ~ -+
third of the total distance with a velocity 4m/s. The r = r 0 (1-atJt where r 0 is a constant vector & a
is a
remaining part of the distance was covered with a
positive constant then the distance covered during the
velocity 2m/s for half the time & with velocity 6m/s
time interval in which particle returns to its initial
for the other half of time. The mean velocity averaged
position is:
over the whole time of motion is :
(a) r 0 Id.
(a) 5m/s (b) 4m/s
(c) 4.5m/s (d) 3.5m/s
(b) ro / 2a
43. 'l\vo bullets are fired simultaneously, horizontally and
with different speeds from the same place. Which
bullet will hit the ground first? (c) ~r; + ~
(a) The faster one
(b) The slower one (d)~
(c) Both will reach sirnultanetmsly
(d) Depends on the masses 49. A point travelling along a straight line, traversed 1/3
44. 'l\vo projectiles ,A and B are projected with angle of of the distance with velocity v 0. The remaining part of
projection 15° for the projectile A and 45° for the the distance was covered with veloc,ity v 1 for half time
projectile B. If RA and RB be the horizontal range and with velocity v 2 for the other half of the time.
for the two projectiles, then: Then the mean velocity of the ·point averaged over the
(a) RA < RB whole time of motion:
(b) RA= RB (a) Vo+v1 +v2
(c) RA> RB v 1 +v 2 + 2v 0
(d) The information is insufficient to decide the
relation of RA with RB. (c) 3(v 1 + v 2) (d) 3v 0(v 1 + v 2)
\ 45. In the arrangement shown in v 1 +v 2 +v 0 · v 1 +v 2 +4v 0
figure, the ends P and Q of an 50. A point moves in zy-plane according to equation x =
inextensible string move at, y = at (l - bt) where a and b are positive constants
downwards with uniform
and tis time. The instant at which velocity vector is at
speed u Pulleys A and B are
1tI 4 with acceleration vector is given by:
fixed. The mass M moves
(a) 1/a (b) 1/b
upwards with a speed :
(c) l/a + lib (d) (a+ b)l(a 2 + b 2)
(a) 2u cos0 (b) u/cos0
(c) 2u/cos0 (d) u cos0 51. A particle starts from rest at A and moves with uniform
46. The accelerations of a particle as seen from two frames acceleration a m/ s2 in a straight line. After 1/a
S1 and S 2 have equal magnitude: seconds a second particle starts from A and moves
(a) The frames must be at rest with respect to each with uniform velocity u in the same line and same
other. direction. If u > 2m/s then during the entire motion
the second particle remains ahead of first particle for a
(b) The frames may be moving with respect to each
duration:
other but neither should be accelerated with =---
respect to the other (a) 2 -Ju(u - 2) (b) !!.-Ju(u- 2)
(c) The acceleration of S2 with respect to S1 may a 2
either be zero or 8mh 2 • (c) ~ -Ju(u - 2) (d) None of these
a
(d) The acceleration of S2 with respect to S1 may be 52. A particle is moving in x-y plane. At certain instant of
anything between zero and 8 m/ s2 • . time, the components of its velocity and acceleration
2
47. A train passes an observer standing on a platform. The are ·as follows. "x = 3m/s,v, = 4m/s,ax = 2 m/s
first carriage of the train passes the observer in time and ay = lmls 2 • The rate of change of speed at this
t 1 = 1 sand the second carriage in t 2 = 1.5s. Find its
acceleration assuming it to be constant. The length of moment is:
(a) ..Jf.o m/ s 2
2
each carriage is: l = 12 m. (b) 4m/s
(a) 3.3m/s 2 (b) -3.2m/s
2 (c) 10~/s 2 (d) 2m/s 2
2
(c) 24m/ s2 · (d) -24m/ s

www.puucho.com
Anurag Mishra Mechanics 1 with www.puucho.com

I 110 ; MECHANICS-I I
53. Two cars start off to race with velocity 4 rn/s and 2 (b) ( u - gt) downwards
rn/s &'travel in straight line with uniform acceleration . t .
1 m/ s2 and 2 m/ s2 respectively. If they reach the final (c) ( 2 u - gt) upwards··
point at the same instant, then the length of the path t
is: (2u gt) ·
(d) - downwards
(a) 30 m (b) 32 m t
(c) 20 m (d) 24 m 59. A block is kept on the floor of an elevator. The elevator
· 54. The instantaneous· velocity of a particle moving in starts descending with an acceleration of 12 m / s 2 •
--> A A
The displacement .of the block during 1st one second
:IJ(-plane is : V =(ay)i+(VoJj, where y is the
with respect to elevator is: ·
instantaneous y co-ordinate of th'c particle and V0 is a· (a) lm downwards (b) lm upwards
positive constant and a is a negative constant. If the (c) Sm downwards (d) Zero meter.

~ro-~:1cr· 60. A point moves rectilinearly. Its displacement x at time t


is given by x 2 = t 2 + 1. Its acceleration at time t is :
(a) 1
x3
. (b) .!_ _ _.!_
x
..
x2

, , 6B. ,~ 00
55. An open lift is coming down from the top of a building
(c)
- x2
t (d) t2
x3

61. Two particles start moving from the same point along
the same straight line. The first moves with constant
velocity' v' and the second with constant acceleration
'a'. During the time that elapses before the second
at a constant speed v = 10 rn/s. A boy standing on the catch the first, the greatest distance between the
lift throws a stone vertically upwards at a speed of 30 particles is :
rn/s w.r.t. himself. The time after which he will catch v2 v2
the stone is : · (a) - (b) 2a
a
· (a) 4 sec (b) 6 sec 2 v2
(c) 8 sec (d) 10 set (c)' 2v (d)
a 4a
56. Three points A,B, C are located in a straight line AB = a
62. A ball is thrown up with a certain velocity at angle 8 to
and· AC' = b. Two particles start from points B and C ·
the horizontal. The kinetic energy varies with height h

:,~
and move with· uniform velocities ½. and V2

~,~
• ' -->
respectively such that angle between V1 and line ABC
. -->
and V 2 and ABC is 8. If point A and both the particles
· · are always in a straight line then :
(a) aV1 bV2 = (b) avi2 = bV}
2 2
(c) a ½ = b V2 (d) aV2 = bV1
(c)~· (d)~ ·
57. A point source of light is
(Source)ro
rotating in a horizontal
plane . at a speed of OJ .. ••r\J
Jb.- .'.· ' 63. A ball is thrown up with a certain velocity 'at an angle 8
,,.· .. :' d to the horizontal. The graph between kinetic energy

: h=E:;~·-=.t2: .
radians/second. There is
a wall at a distance d - P.-··· :
M11mnuin11uim11111n11/llii11111i1111111 N
from the source. At some
instant the focus of the light is at P and LSPN = 8 (see
figure). Speed of the focus at this instant in terms ofe horizontal .horizOntal -
is : displatement displacement
(a) rod/ cos8 (b) rodjsin8 .
(c) rodtan8 (d) ro a/sin 2 e
58. A body is thrown up from a lift with velocity u relative
KE ~~
to lift. If its time of flight with respect to lift is t then (c)
, horizontal
(d)l~_I
acceleration of the lift is : -d_isplacement displacement:
(a) (u - gt) upwards
t
www.puucho.com
Anurag Mishra Mechanics 1 with www.puucho.com

- -- --- 7

j DESCRIPTION OF MOTION _____________ _ 111 '


.... , . . . _ .. _ j

64. A particle is thrown up with a certain velocity and at 68. A particle moves with constant acceleration a in the
an angle 0 with the horizontal. The variation of kinetic positive x-axis. At t = 0, the particle is at origin is at
energy with time is given by : · rest, then correct graph between (velocity) 2 and
displacement is :
(a) l~El / • (b) v2
v2
~ ,_
. ' I'
(a) 8= tan- 1 ¾ (b)

;KEl / Displacement Displacement

:~
(d)
v2

65. The velocity of a (c) 0= tan-1 2a (d)


particle varies with ,
Displacement Displacement
time as shown below. l 12m/s· Circular
The distance
:a51.Qml~ 69. At a height of 0.4 m from the ground, the velocity of
travelled by the
> :
..., . .
particle during t = 2s projectile in vector form is v = (6i + 2j) (the x-axis is
andt=6sis: 2s 6s 1ime in second' horizontal and y-axis is vertically upwards). The angle
of projection is: (g = 10m/s 2 )
(a) 2n m (b) (2n + 40) m (a) 45° (b) 60°
(c) 4n m (d) 40 m (c) 30° (d) tan-1 3/4
66. From a high tower at time t = 0, one stone is dropped
70. A point moves in x-y plane according to the law x = 4
from rest and simultaneously another stone is
sin 6t and y = 4(1- cos 6t). The distance traversed by
projected vertically up with an initial velocity. The

ili:;h
graph between distance between the particles and the particle in 4 seconds is: (x and y are in meters)
(a) 96 m (b) 48 m

:.:·
!Cl
~ ~~hl•
O ,
(c) 24 m (d) 108 m
71. A swimmer crosses a flowing stream of width 'ro' to
and fro in time t 1 . The time taken to cover the same
Time ~- _ Time , distance up & down the stream is t 2 • If t 3 is the time
swimmer would take to swim a distance 2ro in still

k
water, then :
.
(c) . ~ (d)
r~·- -
1 ffi (a) tf = t 2t 3 (b)
0
· 1ime ! i_~ Time '
(c) tj = ti( 3 (d)
72. The trajectory of a particle is as shown here and its
67. A particle moves with constant acceleration in the trajectory follows the equation y = (x-1) 3 + 1. Find
positive x-axis. At t = 0, the particle is at origin and is
co-ordinates of the point A on the curve such that
at rest, then correct graph between velocity and
direction of instantaneous velocity at A is same as
displacement is :
direction of average velocity for the motion O to A:
y

(a) 'v~~me'nt
, -----
(c) J. / . (d) 0 X
'.~· 1
_Disp!acem~nt (a) (3/2, 9/8)
(c) (3, 9)
(b) (2, 2)
(d) (5/2, 35/8)

www.puucho.com
Anurag Mishra Mechanics 1 with www.puucho.com

112
73. A bird flies for 4sec with a speed of It - 21 m/s in a along _the line y = x with such a speed that all the
straight line, where t = time in seconds. It covers a three always stay in a straight line, then velocity of the
distance of : ·third particles is: ·
(a) 2 m (b) 4 m ,.;--;;:-
(al ;rV 1V2 · (bl V1 +V2
(c) 6 m (d) 8 m 2
74. A particle has an initial velocity of 9 m/s due east and (cl (d) V1V2../z
a constant acceleration of 2· m/s2 due west. The
distance covered by the particle in the fifth second of
~vf +v~
its motion is : 80. A particle is projected from the ground at an angle of
(a) Zero (bl 0.5 ni 60° with horizontal at speed u = 20 m/s. The radius of
(c) 2 m (dl None curvature of the path of the particle, when its velocity ·
75. From the top of a tower, a stone is thrown up and makes an angle of 30° with horizontal is :
reaches the 'ground in time t 1. A second stone is (g = 10 m/s2)
(a) 10.6 m . (bl 12.8 m
thrown down with the same speed and reaches the
ground in .time t 2. A third stone is releas~d from rest (cl 15.4 m (dl 24.2 m
and reaches 'the ground in time t 3 then : 81. Two particles are projected from the ground
simultaneously with speed 20m/s and 20/../3 m/s at
(al t3 =.!ct1 +tz) (bl t3 = ~t1t2 angle 30° and 60° with horizontal in · the. same
2
direction. The maximum distance between them till
·w l=l-l · oo r:=~-r: both of them strike the ground is approximately:
t3_ t2 t1
(g = 10 m/s2l
76. A hollow vertical cylinder of radius R and height h has (al 23.1 m (b) - 16.4 m
. smooth internal surface. A small particle is placed in (c) 30.2 m (dl '10.4m
contact with the inner side of the upper rim at a point 82. A rod of length I leans by its upper
P. It is given a horizontal speed v O tangential to rim. It
leaves 'the low~r rim at point Q, vertically below P. The
end against a smooth vertical wall,
while its other end leans against the
f~~G,_·.
i"'t-
I
-_
v."·
number of revolutions made by the_ particle will : floor. The end that leans against the ',, ...
_,XI

(al h (bl ~ wall moves uniformly downward.


21tR ~2gh Then:

(cl ?-: (dl ;:ii (t) (al The other end also moves uniformly
(bl The speed of other end goes on decreasing
(cl The speed of other end goes on increasing
77. Two particles move in a uniform gravitational field (dl The speed of other end first decreases and then
- with an acceleration g. At the initial moment the increases
particles were located at one point and move "1Yith 83. A body throws a ball upwards
velocities v 1 =3.0 m/s and v 2 =4.0 m/s horizontally in with velocity v 0 = 20 m/s. The
opposite directions. Find the distance between the wind imparts a horizontal
particles at the moment when their velocity vectors acceleration of 4 m/s2 to the
. become mutually perpendicular: . left. The angle 8 at which the
(al 5 m . (bl 7..J3 m liall must be thrown so that the ball returns to the
7
(cl ../3 m - (dl 7/2 m boy's hand is (g = 10 m/ s 2 ) :
5 (al tan- 1 (1.2l (bl tan "1 .(0.2)
78. A particle is projected vertically upwards from O with 1
(cl tan- (2l (d) tan-1 (0.4l
velocity 'il and a second particle is projected at the 84. Positio_n vector of a particle moving in zy-plane at time
same instant from P (at a height h above Ol with ---> A A

velocity 'v' at an angle of projection 8. The time when tis r =a (1-cosootli+asinootj. The path of the
the distance between them is minimum is : particle is :
(a) h (bl h (a) a circle of radius a and centre at (a, 0)
2vsin9 2vcos9 (bl a circle of radius a and centre at (0, 0l
(cl h/v (d) h/2v (c) an ellipse
79. Three particles start from origin at the same time: one (dl neither a circle nor an ellipse.
with velocityv 1 along positive x-axis, the second along
the positive y-axis with a velocity v 2 and the- third

www.puucho.com
Anurag Mishra Mechanics 1 with www.puucho.com

DESCRIPTION.OF·MOTION
- - ,, .,_.,_

ss; A particle moves in :,y-plane. The position vector of


--+ ,,_ "
particle at anytimet is r ={(2t)i + (2t 2 )j}m. The rate 86. Velocity versus displacement graph of a particle
of change of 8 at time t = 2 second. (where 8 is the moving in a straight line is shown in figure.
angle which its velocity vector makes with positive Corresponding acceleration versus velocity graph will
x-axis) is: be:
2 1
(a) - rad/s (b) -rad/s
17 14
i(c) j rad/s 6
(d) - rad/s
7 5

(a) :
,~2)·.
!10
I
·····, l
:
'
. .
<
,: 12') .-
'10 ·-·--
(b) I · ·
(
,
:
'
'
· ___ _10 v(m/s) ! _ .. !0 p(mls)j

CcJ
i
110E (. m
i
· )_/.s.
2

!
I (d)
:r;·2) .
'10

I I
·_ - · - v(m/s) ' .. 10 v(rnls)'

'
I
. k.

1. Which of the following graphs cannot represent one (b) Total time for the round trip, if the wind blows
dimensional motion of a particle? 21
~:- • 7 1;l·o · --- perpendicular to the line AB, is
-v 2 .Jv 2

" (c) Total time for the round trip depends on the
(a) ~-
.
(b) /l direction of wind
I ·
L _______ . Time __ ~
1 (d) Total time for the round trip is independent of
direction of wind,
4. For a constant initial speed and for constant angle of

(c) J, ( 1 (d)
projection of a projectile the change dR in its
horizontal range R due to a change dg in value of
gravitational acceleration g is governed by the

L.~-~ lL
2. A lift of very broad - floor is moving vertically upward
- ..lime
_____11
relation:
dR dg
(a)-=-
R g
(b) dR = -dg
R g
with a constant retardation equal to 'g'. At an instant a (c) dR = dg (d) dR = -dg
stone is projected from a point on the floor of the lift at g R g R
angle of elevation 9. Then the trajectory of the stone is:
(a) A parabola in the lift-frame 5. Two particles, 1 and 2, move with constant velocities
-; -;
(b) A straight line in the lift-frame v 1 and v 2 , At the initial moment their radius vectors
(c) A parabola in the ground frame -; -;

(d) A straight line in the ground frame are equal to r1 and r2 . How must these four vectors be
3. An aeroplane flies along straight line from A to B and interrelated for the particles to collide?
--t --t --t --+ --t --+ --t --t
backs again to the same point. There is a steady wind (a) r 1 -r2 = v 1 -v 2 (b) r 1 -r2 = v 2 -v 1
speed v. The distance between A and B is l still air -; -; -; -; --t --t --+ --+
speed of the aeroplane is V, then: lr1- r2I lv1-v2I lr1-r2I lv2-V1I
-; -; -; -;
(a) Total time for the round trip, if the wind blows
2 (c)
r1-r2 = V2-V1 (d) None of these
along the line AB, is Vl -; -; -; -;
2 2
V -v lv2-v1I lr1-r2J

www.puucho.com
Anurag Mishra Mechanics 1 with www.puucho.com

,. ~.:"cl: pc!:! !
l~. .·: ·'~
hemispherical bowl. It , ·: ::< ·1
10. An aeroplane moving horizontally from west· to east
with some velocity and with an acceleration 5 m/s2
drops a food packet at some instant. Then:
passes the point A at t = ,:~:; , .,.· · -: ~ _.· '.: ': (a) The path of the packet is parabolic with respect to
O. At this instant of time, ·"·---~·-"·-- - ·---~-.:..:l ground
. the horizontal components of its velocity ate v. A bead (b) A person sitting on the aeroplane shall see the
Q of the same mass as Pis ejected from A at t =O along packet is always vertically below the plane.
the horizontal string AB, with the speed v. Friction
(c) With respect to plane the packet travels in a
between the bead and the ·string may be neglected. Let
straight line making an angle tan-1 (1/2) west of
t P and t Q be the respective time t_aken by P and Q to vertical. ·
reach the point B. Then :
(a)tp<tQ (b)tp=tQ
(d) With respect to plane the packet travels in a
straight line making an angle tan - I (1/2) east of
(c) tp > tQ (d)2- = length of atcACB
t Q length of cord AB vertical.
(e) The packet moves in a parabolic path with respect
7. Two partides ·ate thrown from the same point in the to aeroplane.
same vertical plane, as shown in figure simultaneously.
11. Two balls are thrown from an inclined plane at angle
Then indicate the correct statements :
of projection a with the plane, one up the incline and

·r~-~ :-~---~:/·~---~: 'f-----~-'


~:<.:J -·. \j
other down the incline as shown in figure (I' stands for
total time of flight):
!-;~.~-~:.--· -, .. - -.-~ ·----~::·;-;·-·1
l·. ' 1/ A'. ,'t ~,c·,.'·JJ
· .. f' _ 82 . 1 . · '· - •_ ••. · . ·

t~--~-:--2-~:{:.~~L~J
(a) Tiine of flight for B is less than that of A
/ --------~ i
I-- ---,------·
(b) Projectic:m ~peed of B·is greater than that of A
(c) Horizontal component of velocity for Bis greater
than that of A
(d) The vertical component of velocities of both ;\and (b) Ti = T2 = 2v 0 sina
B are always equal throughout the duration for gcos8
"1hkh both the particles in air. (c) R 2 -R1 = g( sin8)T/
8. A particle of mass m moves on the x-axis as follows : it (d) v,2 = v,1
starts 'from rest at t = O from the point x = 0, and 12. A particle moves in the zy-plane according to the law
comes to re~t at t = 1 at the point x = 1. No other x = asin(cot) and y = a(l-coscot) where 'a' and 'co'
information .is available about its motion at are constants. Then the particle. follows :
intermediate time (O < t < 1). If a denotes the (a) a parabolic path
acceleration of the particle, then: (b} a straight line path, equally inclined to x- and
(a) a cannot rem~in positive for all t in the interval y-axis
0 ;,, t 2' 1. (c) circular path
(b) IcxJ cannot exceed 2 at any point in its path (d) a path such that distance moved by it is
(c) Ial must be <C4atsome point or points in its path proportional to time
(d) a must change sign during the motion, but no 13. Mark correct statements.
other assertion can be made with the information (a) Two particles thrown with same speed from the
given. ·· same point at the same instant but at different
9. The magn.itude of acceleration of a particle as seen by angles cannot collide in mid air.
observer A is am/s2and that observed by Bis b m/s2. If
(b) A body projected in uniform gravitational field
m:agnitude of acceleration of A with respect to B is x
mls2 then indicate the correct statements is : follows a parabolic path
(a) la 2 -b 2 I :S x :S la-2 +b 21 (c) In projectile motion, velocity i~ never
perpendicular to the acceleration.
(b) l<i-bl:Sx:Sla+bl
(c) la-bl<x<la+bl (d) A particle dropped from rest and blown over by a
(d) O:S x :S la-bl or·x;,, la+bl horizontal wind with constant velocity traces a
parabolic path.
www.puucho.com
Anurag Mishra Mechanics 1 with www.puucho.com
V .t ,_,J e- \·1.:.·-, :."\·

\·.:-ti.\::, 1 1.hr;,.r"t~l,;
,,'" cLuivc :.c-;:1.: c. t:~, . . 1 :

· erLfr.:il d:rer.:r::;r'~
.( ... } 'i"

14. An aeroplane at a constant speed releases a bomb, As --


1··
,_
I aE
the bomb drops away from the aeroplane, !I '1B
(a) It will always be vertically below the aeroplane (c)
,.a
~

(b) It will always be vertically below the aeroplane !,..


only if the aeroplane was flying horizontally
(c) It will always be·vertically- below the aeroplane
only if the aeroplane was flying at 11n angle of 45°
to the. horizontal- •. •... · . .
(d) It will gradually fall behind the aeroplane if the
aeroplane was flying horizontally
15. Two straight lines 11 and 12 cross each other at point P. ., : ' '
The line 11 is moving at a speed v 1 perpendicular to 17.. A ball is dropp;d_ fro~. C~rtain height on a horizontal a
itself & line 12 is moving at a speed v 2 in the similar floor. The coefficient of restitution between the ball
fashion. The speed of point P is : and the floo~ is 1/2. T~e displ~cement time graph of

tl;ebr??.' . . 'Ir.~:
• -- . ·~- ·,·· . ' -····· --···1
I
I
' (a) I (b) ;, r
' -· • i. • I
i_ _ _t L. _.:. ___ ½
l2_ - ;
- - -- -· --·-J

(a)

(b) 18. The speed-time graph of the ball in the above situation
cosa
is :
~v~ +v~ + 2v1v 2 cosa (--~··v·c:·
.· · . --,.
(c)
sina (a) I
~' .
:'

' .-
'
i
i
'
. (b) !'
'k·v·
·-· --;1\
I
I
I
(v 1 +v 2 )+~v 1 v 2 cosa
(d)
cosa
:
-·~ : : ''' .t'
---- - - L..~: _ 1!
16. The velocity-time graph of a particle moving along a
(d) .
:L2'' :.- --_-,I
straight line is given as below. The displacement time
curve for the particle is given by : !· __ _; - .-· ti
. . -I 19. In a car race, car A takes time 't' less than the car B and
passes the finishing point with a velocity 'v' more than
! the velocity with whicn the car B passes the point.
-------- ________ ___ .. --·--' Assuming that the cars start from rest travels with
constant accelerations a1 and a 2 , then :
- ---- --- - - - - . (a) a 1 > a 2 · · (b) a1 < a
2
IE
• (c) v = ~a1a2 t 0 • (d) v =(a,+ a2 )t 0
'E

(a) , i
l Cl I
: I
I
i
·I
_,
:
J I
:
20. Two particles are projected with
speed 4 mis and 3 · m/s
; 4m/s 1
.____ .. -1... 2 • s . J __

l ... - - - ·..· - - - - ~ - ~ " - - - - - - ~

I I
10 1fm_e, l simultaneously' ·from same point
as shown in the figure: Then :
' \ ._k.·3mlsl
i' 5 •
(a) Their relative velocity is ·- -----------
31•

j
i~ along vertical directiol}

(b) :,~a'k-----,--.;:.•-7"'-,'-=o-
. 11me I
(b) Their relative·acceler~ticni. is non-zero and it is
I along vertical direction· ·,
,_ -1
(c) They wjll _lJ_ittn,;\ii~~ce ~imultaneously
(d) Their relat;i".~. velocicy is /constant and has
magnitude 1,4 m/Jk · , , ,
1 ~--. "·:,

www.puucho.com
•J\-) \·:";, ·-: ./'
,· \ . ., :·r,;
\ I
·-, --...._~ ...
Anurag Mishra Mechanics 1 with www.puucho.com

21: The motion of a body falling from rest in a resisting 24. A particle moves along x-axis with constant
medium· is: describ~d by .the equation dv = A - Bv, acceleration and its x-positio11 depend on time 't' as
. . . dt

!~-~1
shown in the following graph (parabola); then in
where A and B are constants. Then : interval O to 4 sec
(a) maxiriuun possible veiocity is NB mis
(b) initial acceleration is A m/s 2
.(c) ~elocicy a~:iiny time t is v = ; (1- e-B,)

_(d) velocity_ ~t_:ty time t is V = ; (1- e-At) • . I


4 -t(sec)
22. vVhich ·of -the following statement is/are correct 7
(a) relation between x - coordinate & . time is.
(a) .Average speed of a particle in a given time period x=t-t 2 /4 - -----.~-
· · is never ·1ess than magnitude of average velocity
.... (b) maximum x- coordinate is l m
(b) _it. is. possible to have situations in which ~ * 0, (c) total distance traveled is 2 m
. .. dt (d) average speed is 0.5 m/s
25. The velocity versus time of two particles moving along
x-axis varies as shown in the following two plots.·
Then: ·
't - • --+

(~) it is pcissible"to"have situations in which d Iv I * o,


.\ _'·,;; : :
,,., 4
:- dt m / ~·-v ·:it,~:iJ,.
.-
but--= 0
. dt .·
dv
r -----
t ~.mis --~-~
. 2 4.
;

(a) maximum separation between the two particles is


(d} 'fhe'.averag~ velocity of a pa!'(icle is zero in a time 2m
interval. !!' is possible that the instantaneous
(b) maximum separation between the two particles is
. ve\ocity is' never zero in the interval.
. '-. . - . 2.5 m
23. A particle is moving with uniform acceleration along a
(c) maximum separation between occurs after time ·
sfi'.aight liJ1~ Ali.
Its speed at A and Bare 2 m/s and 14
t =2 sec
ni,)'s. respectively. Then : .
(a)'·, its spied ·at the mid-point of AB is 10 m/s (d) maximum separation between occurs after time
t =3 sec
(b) its spe~d at a point P such that AP : PB = 1: 5 is 6
. m/s. :: , . .
,•:
(c) the :ti!Jle to go from A to the mid-point of AB is
·double of that to go from mid-point to B
. (d) . hone of µiese

,,·
,.

' . ',; __ , . 1:
.- ,:n!l-:r:

www.puucho.com
, .., ..;
~ _
Anurag Mishra Mechanics 1 with www.puucho.com

... , '•
._I_DE_SC_R_IPT_,IO_N_O_F_M_OT_IO_N_ _ - - - - - - - - - - - - ---- ·---· :, '" ~:,

r
IL.
1

_ 3 __ Comprehension B~~~-~~~b_i~_-~_s_·_____
J
PA s's'A G'E ,;
' ' .• ----
:. ' ' -
,,.,j '-' PASSA'lfE
A particle \4.' starts moving frorri point A with constant A swimmer wishes to .cross a river 500 m wide
velocity 4 rn/s along x-axis. Another particle 'B' flowing at a rate 'u'. His speed with respect to still
initially at rest starts moving along x-axis after (8/3), water is "v'. For this, he makes an angle 0 with the
sec after the start of A, with acceleration varying ~s, perpendicular as shown ip. the. figur~. ..
a= 4 (3-t) rn/S 2 • 1 ': •.. _B ___ ----- - -I

Based on the above information, answer thel


following questions. I : v~ 0] .
, ....::s! u
!d = sooml
I
' - - - - - - ---•·•· -·--·-··•--..-···---~~= ·- -- .. • ... I
.. -'~---- -:_ ,__ ,..____: ---....c.'_______,
1. Particle B will stop again at the position x equal to :
(a) 72 m (bl 36 m 1. To cross the river in minimum time, the value of 0
(c) 3 m (d) 6 m should be:
(a) 0° (b) 90°
2. The two particles will meet twice in the due course of
(c) 30° (d) 60°
their motion. The time interval between these two
successive meets will be : 2. For u = 3 km/hr and v = ·5 km/hr, 'the time taken to
(a) 6 sec (b} 4 sec cross the river in minimum time will be :
(c) 2 sec (d). 8 sec (a) 3 min (1,) 6 ·hr · ..
3. Position where the two particles will meet for the
(c) 6 min (d) 3 \1,r
second time is given by : 3. For u = 3 km/hr and v = 5 km/hr, the swimmer :
128 (a) can reach to Bin 7.5 min
(a) x = 72 m (b) X = - m
3 (b) can reach to B in 6 min .
56 (c) can reach to Bin less than 6 min
(c) x = 36 m (d) x = - m (d) can never reach to B
3
4, Total distance traveled by the particle B when it meets 4. For u = 5 km/hr and v = 3 km/hr, the swimmer :
the particle A for the second time is : (a) can reach to Bin 7.5 min
(Ji) can reach to B in 6 min
(a) 3~4 m (b) 936 m
(c) can reach to Bin less than 6 min
56 (d) can never reach to B
(c) - m (d) 36 m
3 5. For u = 3 km/hr and v = 5 km/hr, the swimmer can
5. Magnitude of the relative velocity of the two particles reach to B if e is :
when they meet for the first time is : (a) 37°
(a~ 16 rn/s (b) 12 rn/s (b) 53°
(c) 20 rn/s (d) 18 rn/s (c) 60°
(d) can never reach to B
6. Magnitude of the relative velocity of the two particles
when they meet for the second time is : 6. Foru = 4 km/hr and v = 2 km/hr, and to minimize the
(a) 16 rn/s (b) 32 rn/s drift, the swimmer must follow a path in which 0
(c) 36 rn/s (d) 28 rn/s should be:
(a) 30° (b) 60°
7. Variation of velocity uf the particle B with time is best
(c) 0° (d) 45°
represented by :
For u = 2 km,,'hr and v = 4 km/hr, and to minimize the
ilLv--·---1 7.
drift, the swimmer must follow a path in which 0
should be:
(a) : I

~ ___ t: (a) 30°


(c) 0°
(b) 60°
(d) 45°

(c) 'i k : "tl;


I. •
L.'___ ,_____ . ..:J
i (d) i~" '
: t www.puucho.com
[_, ·-----~-----;
ptu: s. -1..'1:..1 with www.puucho.com
Anurag Mishra Mechanics

• -, 1

·,·,-•v.iifl ~..,-:-:i ,~ 'l ~! '·{,. !:•·).I

,.-,·,~,::-:~_,.,_., .{,,, ;:-•:-;,,;·•:"'·, ,,


·, ~; {-.f.1if?J.';u_.l1 d.:~'lJ 1ic1r\1 '' - .~ .
,,;.

I .1,
..... ,. -~-
.
---- '
- ••. ,1 ' l"" \
, • L , 0 •

'
L...;..::, ....-.. .: ..;i ~
1· :

',
• ' ~ "1
.,,
,, '··
·-·... . , - ·-- '

' ..

www.puucho.com ''
. .,..'
;, j , l,
Anurag Mishra Mechanics 1 with www.puucho.com

-- ---7
~-DESCRIPTION OF MOTION 119
-- ------ -- - - - ----- - ,- - - --- -·--- --- . - .. - - ----- ---:
· - - - - - - ----r,-..---..,. c::· .: '··..: .: ',. ~- ' _.'.J \;·~~:;_
MATCHING TYPE PROBLEMS -,>.-(, 1 _:Column-2-.,:~:\',,bzt1§',
-----~--~-~-~--~£-··; -~-·.
C-
1. A dart gun is fired towards a Squirrel hanging from a
tree. Dart gun was initially directed towards Squirrel.
P is maximum height attained by dart in its flight.
Three different events can occur. (Assume Squirrel to

-~·
be a particle and there is no air resistance) .

.·~ ..· :
.... :
Two projectiles are projected
from a height such that they
strike ground at the same time.
'
(B) :u 1 > u2;81 > 82 (Q)
v/:_f--::t.~"".'Tra)ectory of dart '
0
J::-_..
d
rs"··· ...
• Two projectiles under standard'
ground to ground projection
Colurnn-1 such that horizontal range is
'-'-''------- --------· ----------------~- same.
(A) Event-1 : Squirrel drops (P) When dart is at P
itselfbefore the gun is fired. Squirrel may be at
A
(B) Event-2: Squirrel drops '(Q) When dart is at P
itself at same time when the Squirrel may be ·at
-gun is fired. ,B
(CJ :Event-3: A strong wind imp- (R) In gravity free Two swimmer starting from
arts same constant horizon- space dart will hit 'same point on a river bank such
tal acceleration to Squirrel Squirrel. that time of crossing is same. u1
and dart in addition to and u 2 are velocities relative to
gravitational acceleration. river.
Squirrel drops itself at the (S)
same instant as the gun is
fired.
(SJ :Dart cannot hit Sq-
uirrel in presence
of gravity.

2. Column-1 shows certain situations with certain


conditions and column-2 shows the parameters in
Person moving downward along
which situations of column-1 match. Which can be
slope in rain such that he
possible combination.
·observes rain vertically.

www.puucho.com
Anurag Mishra Mechanics 1 with www.puucho.com

--120
- - -- - -
3. Figure shows a graph of position versus time graph for
a particle moving along x-axis.
y
-a
'\(Q) ,....+a= .:.acos0i+asin0j
" A

X Parabola ---!<'--•:x
Straight Straight
Line . Line

t, ' (C) , a Y '


i (R) 'i:. = -asin0i- acos0j
arabola Parabola _ _!U-"l---•x:
'

(A) Slowing down (P) t1 -->t2


(D) y (
;(S)
_, .
,a·= acos0i-asin0j
.
(B) Returning towards origin (Q) t2-->t3 '
- - - - J . - -..x,
(C) Moving away from origin (R) t3-->t4
(D) Speeding up
I
(S) t4 ->ts - a
e

'(t) ,ts--, t5
6. Consider an object at point P along each trajectory
4. Trajectories are shown in figure for three kicked shown in column-1 in the direction of arrow shown.
footballs. Initial vertical and horizontal velocity Column-2 gives algebraic sign of v x, v Y, ax and ay-
components are uy and ux respectively. Ignoring air
resistance, choose the correct statement from
column-2 for the value of variable in column-1. 1
(A) .y

~
',·(P) \Vx > 0, Vy > 0, Clx > 0,_ ay < 0
I
p
'
I
I

.o. ...,1--,,~~-~-x·
;, _ Speed constant ,.
l

(B) y ,(Q) :Vx > 0, Vy= 0, a, > 0, ay < 0


' '
(A) Time of flight ' (P) greatest for Aonly p
i
(B) ,uy/ux : (Q) greatest for C only
'
(C) ux : (R) ,equal for A and B ...,S_p_e-ed-is~in-c-,e-a~si-ng•x,
- - -.
'
(D) ux !l.y , (S) 'equal for Band C
(C) y

A . _ w a vector 'i:. at angle 0 as shown in the figure


column-2. Show its unit vector representation. p

--'---'-----•x ,
Speed is decreasing'
(A) - ------ - - I
ia
'., (P) _, = asin0i. + acos0j•
' > O,vy >0,Cix > 0,ay>0
(S) .lvx

7. A particle is moving along a straight line. Its v-t graph


is as shown in figure. Point l, 2 and 3 marked on graph
are three different instants. Column-1 has fill in the
blanks, which are to be filled by the entries in
column-2.

www.puucho.com
Anurag Mishra Mechanics 1 with www.puucho.com

- - --
! DESCRIPTION OF MOTION
~---·----~--- -------~
V
(D)
->
di rl
i (S) :Magnitude of velocity
I I
. dt '

i (T) :None
--l-------r
10. For the velocity-time graph shown in figure, in a time
Column-1 interval from t = 0 to t = 6s, match the following :
.. -v(mls) .
(A) a1 is .......... a 2 (P) 'Parallel to i0
0

(B) v1 is .......... V2 (Q) Anti-parallel to


(C) I •
V3lS .......... V1 : (R) ,Greater than (in
magnitude)
'.
(D) a1 is .......... v 1 (S) ,Less than (in magnitude)
(A) Change in velocity ; (P) i- 5/3 SI unit
8. Figure shows a cube of edge length a. '
·y (B) !Average acceleration !' (Q) .:_ 20 SI unit
'
Ht,1------,,G (C) Total displacement : (R) :- 10 SI unit
'
(D) Acceleration at t = 3 s I' (S) ·_ 5 SI unit
E ,----'f---1',F
11. Let us call a motion, A when s·
N
velocity is positive increasing. '
"-'A'-_ _,__ _...J..CB'- X A -I when velocity is negative
and increasing. R when velocity
is positive and decreasing and
R- 1 when velocity is negative ··-·
D C
z and decreasing. Now match the following two tables
_,,;/ ..-., ' . ,' .:·9; for the given s - t graph :
. ...,,,r; ·.:r.olumn-1
!-----·~---~-------~---- . -..i · .. . · Columr1,2)':
----·~;....;.,;..,:.;:JL
i1:~,z,:.,_ .£'."- ..'*t~}J.l
.· -~~9.lu"!n-2
(A) The angle between AF and x:CP) 60°
-axis (A) ,M ! (P) iA-1
' ' f i

(B) Angle between AF and DG : (Q) 'cos-1 _!. (B) ~N I (Q) IR-1
3
(C) 'p '(R) 1A
J_ ' '
(C) ,Angle between AE and AG , (R). 'cos-1
(D) '.Q I (S) iR
' ' -.J3
(S)

9. Match the following :


-:::-:- Column-2
. ~-~~
. '¼~;~
_,
i (P) Acceleration
(A) I~
dt
-> : (Q) 'Magnitude of acceleration
(B) :di vi (C) Maximum height , (R) '45°
' dt
-> (D) 1Horizontal range I
;
,
I
( 1)
(C) dr
, (R) 'velocity ' (S) tan-I 2
I I
'dt

www.puucho.com
Anurag Mishra Mechanics 1 with www.puucho.com

122 ----~-~--..;....________.;.;.___M_ECHANl(S'\J
,._,.«:.;:;:,µ;;q,..w,::;>iim:s;.a:c. rn **h. ;::::.;;;;:;ut!.\.,.-.
13. In the s-t equation (s = lp + 20 t - 5t 2 ) match the
6-;:::;-· ASSERTION ~r,,!!)"REAS.£>J! -""-':.ifA,
following:
~~-·~~-
~·i~-i-;-:~·
·_~tf:c~_-r:,· ~:x_·'.'.?-ii:_<-~_~~~ ~i~_·ti_.,_,_-,f.
·:k-,~Jt}!n?n,;,•(1:±; -7:~- ·
~e. .
: 'l":r1,s. ~~'%
~_,: o umn""' Directions : Read the following questions and choose
(A) ;Distance traveled in 3si (P) - 20 unit (A) If both assertion and reason are true and the reason is
.I .
- 'i • correct explanation of the assertion.
(B) !Dispiaceinent in ls :' (Q) ilS unit (B) If both assertion and reason are true, but reason is not
I
1
-
I
i
i correct explanation of assertion.
(C) 1Initial acceleration i (R) 125 unit (C) If assertion is true, but the reason is false,
(D) !velocity at 4s ! (S) ;--10 unit (D) If assertion is false, but the reason is true.
(E) If both assertion and reason are false.
14. A particle is· rotating in a circle of radius lm with 1. Assertion : A body can have acceleration even if its
constant speed 4 m/s. In time· ls, match the following velocity is zero at a given instant of time.
(in SI units) : Reason : A body is momentarily at rest when it
'it.
Column,1f,_ reverses its direction of motion.
I
2. Assertion : A body having uniform speed is circular
(A) !Displacement
. I !
'. (P) !s sin 2
I
path has a constant acceleration .
Reason : Direction of acceleration is always away
(B) Distance ! (Q)f4 from the centre.
I J _-, '
(C) Average'velocity , (R) •2sin2 3. Assertion : The two bodies of masses M and
i ; m(M > m) are allowed to fall from the same height if
(D) [Average acceleration_ I (S) 14 sin 2 the air resistance for each be the same then both the
bodies will reach the earth simultaneously.
Reason : For same air resistance, acceleration of both
the bodies will be same
I 4. Assertion : A body is momentarily at rest when it
(A) :·co~stant' positive accele-/ (P) speed may increase
1 reverses the direction.
ration ,r '
Reason : A body cannot have acceleration if its
(B) Constant. negative accele-1 (Q) !speed may ~ecrea~e ·
' velocity is zero at a given instant of time.
l •
. . 1rat1on .
.
, _ I
I
'
I
' 5. Assertion : A particle in motion may not have
variable speed but constant velocity.
(C) !constant displacement ' , (R) ,speed is zero
Reason : A particle in motion may not have non-zero
(D) \constant slope of a-ti (S) Jspeed must increase acceleration but constant velocity.
6. Assertion : A particle in .zy-plane is governed by
/graph
I I =
x = a sin rot and y =a-a cos rot, where a as well as ro
(T) 1speed must decrease
are constants then the particle will have parabolic
16. A balloon rises up with constant net acceleration of 10 motion.
Reason : A particle under the influence of mutually
m/ s2 • After 2s a particle drops from the balloon, After
perpendicular velocities has parabolic motion.
further 2s match the following : (Take g = m/ s2 ) 10
I
(P) /Zero
I

(Q) /10 SI units


(C) Displacement of particle (R) 140 SI units·
' ''
(D) /A~celeration of particle I (S) J20 SI units

www.puucho.com
Anurag Mishra Mechanics 1 with www.puucho.com
\

[ DESCRIPTION OF MOTION ·123 i


,-·-·--· -- ----~- - --- ---- ·-··-----~~-------- ···--.-:----· 7
AN8WER9 ··_J'
-------- -·· - - ----- ---- - -- - -- ----------------- ------·

= i;;vel-1: §niy One Alternativ£is Correct.~


I
(a) 2. '' (b) 3. (a) 4, I (a) 5. (!;,) 6. (d) 7, (b) 8. (c)
1. I
'
9. : (c) 10. (b) 11.
I
'' (c) 12. I (b) 13. (d) 14. (b) 15. (d) 16. (a)
I
i' (d) I
I'
I
17, (b) 18. (b) 19. (b) 20. (c) 21. (b) 22. : (a) 23. (d) I 24
I' 28. !' (c) 30. i (b)
' '
I
(b)
I
I
25. ' (b) 26, I (a) 27. i (c) ·I
: 29, I Cal 31. (c) I 32.
'
33. ! (d) 34. (a) 35, I·' (a) ' 36. (c) 37. (b) 38. '' (b)
I'
39. (d)
I
'' 40, I
I
(d)
I
41. : (c) 42.
' (b) 43. i (c) 44, !
I
(d) 45. (b) 46. : (d) 47. (b) I
' 48.
(b) '
54. I' (a)
49,
57. ', (d)
(d) 50. '' (b) 51. I
I
59. ; (b}
(c) 52.
60.
I
(d)

II (a)
53.
61.
(d)
(b) 62.
'
(a)
55.
63. I (b)
(c)
56.
64.
(d)
(c)
I'
58. (c)
65. 1
(b) 66.
I
(c)
I
67. ' (a)
'
68. (c) 69, (c) 70.
''
(a) 71. I (a) 72.
'
I '
(~)
74, J!' (b) 75. j. (b) i
76. I (d) , 77, (c) 78. (d) 79, (d) 80. I, (c)
73.' (b)
I I i ! '
82. : (b) 84. I Cal 86. I Ca)
81.' Cal 83 . . Cd)
' 85. : Ca) '

=- L~vel-~: Mcir~!han o~~-Aii<:rn-at~'!e~ a_re Co~re


-~
'
I , i I
I ' '
I'
3. (a, b, c) 4, (b) 5. ' (b) : ,6. (a)
1. (a,b,c,d) 2. ' (b, c)
I
.I
'I
l
; i
' 8. !' (a, c) 'I·
7, I (b, c, d)
i
I 9. I {b) 10. (a, c) 11.
1
(a, b,'c) '' 12. i,I (c, d), , II
13. (a, d) 14. I (a) I 15. I' (c) 16. Cc) I 17, I (c) i 18. (b) • I
I
I 'i .I
I
I'
19. (a) 20. j (a, d) : 21.' (a, b, c} 22. (a, b, d) 23. \' (a; b, c). I 24. (~,b,c,d).
' I
I
I

25.
:
Cb, al I : t' - - I I '
I I

Passage-1:
1. (a) 2. (a) 3. (b) 4. (a) 5, (b) 6. (c) 7, (c)
Passage-2:
1. (a) 2. (c) 3. (a) 4, (d) 5. (a) 6. (a) 7, (a)
Passage-3:
1. (a) 2. (c) 3. (b) 4, (d) 5. (b)

=sMatchhl!!~!Ype P_rob~-~~~
1. A-P, Q, R, S; B-R; C-R 2. A-P, Q, R, S; B-Q, R, S; C-Q, R, S
3. A-R; B-P, S, T; C-Q, R; D-P, S 4.A-R;B-P;C-Q;D-S
5. A-S; B-P; C-Q; D-R 6. A-P; B-Q; C-R
7. A-P, R; B-P, S; C-P, R; D-P 8. A-R; B-Q; C-P

www.puucho.com
Anurag Mishra Mechanics 1 with www.puucho.com

,, · MECHANicf.f'\

8. (c)
For resultant to be zero the given magnitudes must
form a triangle. When lengths are 4, 8, 4 a triangle is
nue. Take a hexagon whose all the formed with height zero.
sides _are of unit length. 9. (c)
--,, --,, --+ --,, --,, --+ Resultant is inclined more towards vector of larger
BA= BC+CD+DE+EF+FA magnitude .
.-2;

(b)
r. _ .False., When 0 = 2mt the vector remains the same
,, 3, (a)·
10.
(b)~i
. ,\, nue. Consider a regular n-gone
4. ·.(a)
True. Consider a non-regular n-gone. and
5. ~) 11. (c)
--t -+ "--t--t -), --t --t --t-+
2
False. a x b is a vect_or quantity while a. b is a scalar ICI= IAf+IBl +2IAIIBlcosl20°
·, -+ --t -+ --t
-+ -), -+ --t
quantity. Therefore ~ x b can never be same as a. b
= IIAI-IBll 2 +[A[IB[
6. (d)
-+
(A+B).(A-B)
cos 0 =--),- - ---j,----t-
--t --+ -+
=> ICI> /1AI-I~ I·
--t -+ ,_, _,,
IA+BI IA-Bl Aliter : The miniJllum value. of IC I is IAl-I Bl and
-+ -+ 2
2
= IAl -IB1 =0 -+
--j, -+ --j, -+ this minimum is achieved when angle between A and
180°. Since 9 = 120°, /C_, /> I_, -+I
LA+BIIA-BI -+
Bis /A/-IBI
=> 0 = 7t/2
7. (b) -+ -+--t-+ -+"-+ --t'-+

-+ -+ I
13. (d) Given a+li a/-1 a/ >Ill a/=> a+lia/>I al+llial I
Let a is rotated through angle 0 to get b -), -+ -+ -+ -) --j,

~ la/ +11ial +21a/llia/cos0 >la/+llial Squaring


---t -+ --j, -+ 2 2
:. I a/=lbl= a and angle between a and bis 0
both sides.
.,
The magnitude of change in vector -+
_a is --t·~ -+-+
=> 2 la/llialcos0 > 2la/llial.

~,1 . r.~--
; lb:_~~ ~lbl 2 +1 ;l 2 +2lhl/_~ cos(it -0) =>COS0 > 1 =>0Eq,
= a.J2c1 - cos0)
= a~4sin 2 0/2) 14. vt -:~
:21_~"
= /2asi~9/2/
1
south ~west _ . l
.,• s - ----- -- ---- -___ I
,._,·.
www.puucho.com
Anurag Mishra Mechanics 1 with www.puucho.com

l DESCRIPTION OF MOTION
Change in velocity = final velocity- initial velocity.
-+
I!,. V
A

= -vi-vj = -v(i+ j)
,.. ,.. A
Component perpendicular to
the plane has magnitude 10
COS 30° = 5.J3
I~-'io
~-
·-, i>
~

...,
[ I!. V[ = v..fi. towards south-west 20. (c)
..., ...,
=SM Since Jat+ a2J = .,/3, and if _angle between them is 9,
_____ N_ _ _ · - ·
=70 km/hr towards south-west
··1 then (../3) 2 = 1 2 + 1 2 + 2.1. leas 9i.e., e = 60°
-+--+

~
-+-+ 2
(at- a2).(2a1+ a2) = 2at
'
:
'
I -+-+ ---+-+ 2
; equator!
+ (at. a2)- 2(at. ;i2)--: a2
= 2at2-- + -+ 2
(at. a2)- a2
15. (d)''
'
I
i
[ __ -- . s _____ - - 21. (b)
= 2-1.1..!.-1 = l
2 2
. ·- _,. ,
16.
22.
As obvious from the figure

(a)
4
cos e = -
5

As shown in the figure


0 8
A

'L_____. ....
B• ·-
C

xsin9=8 rjJ:
- ·-·..
xcose = 16-x i .xsin~:-s
Net displacement = St+ S 2+ S 3
..., ..., ..., Solving, we get
X= 10
~
1.... ...8. ,... . . .,
, x cos·a 16-*· ·
= (20cos45° i + 20sin45° j) + (-20i) So the required combination is
+ lON, and 6N. ·
(20cos45° i + 20sin45° j) 23. (d)
..., ..., .
= (20,J2 - 20) i + 20,J2j Component of A along .B is given by
-+ -+ -+ . ..., ...,
[St+ S2+ S3J =. ~(20(..fi.-1)) 2 + (20,J2) 2 = A.B 8
...,
= 2oJs- 2..fi. m JBJ2
17. (b) _ (3i + 4j). (i + j) (: ")
- ' ' 1+ J
-+ -+ -+ -+ -+ -+ Ji+ jJ2
A x Bis parallel or antiparallel to C. Hence C x (Ax B)
-+ -+
= O. Also A .B etc = 0. Again B +c being a vector in
-+ -+ =Zc1+Ji
2
..., ..., ...,
the plane of B and C , it is perpendicular to A . But
-+ -+
AxB. not necessan·1c.
---1s
-+ -+
y-, 1t may b e equal-C
-+
to-.
-+
-+ -+
24. (d)
( ~A 2 +B 2 r = (A+B) 2 +(A-B) 2
JA x BJ [Cj [CJ +2(A +B)(A '--;,ll)cose.
18. (b)
-+ -+ -+ -+ -+
art= 0, Vt= aAtanytimet, V2 = a+bt
-+ -+ -+-+ 25. (b)
when Vt and V 2 are perpendicular Vt. V 2 = 0
= B case

i!iJR
As .shown in the figure, A

~ ;,(;+ht)= 0 ~t = -(~~:]
cos e = -
A ....B .
a.b B J

19. (b) Required angle = lt - e ....... ~··· .... ·. ''

As shown in the figure -1(- A) !Brose . '1."I


= cos B

www.puucho.com
.. ,

Anurag Mishra Mechanics 1 with www.puucho.com

j12s. MECHANICS,( I
26. (a) 32. (b)
As shown in the figure for. --> -->
IAl=2, 1B1=2'12
angillar displacement 8 the
--t --+ --+ --+
linear displacement AB is A.B=AXB
equal co 2r sin (8/2) .
=> 8=45°
--+--+
~ - - - ---+--+
--+--+
--
A+B IAl 2+1B1 2+2A .B
27. (c) - - = ,~~~~---
--+--+ --+--+ --+--+
We know if a vector makes (al+ bj + ck) an angle a, J3 A-B IAl 2-t1B1 2-2A.B
and 'Y with x; Y and Z respectively then
=..J5
cosa. = --;=====~=,
a
~a2 +b2 +c2
b
cosJ3 = --;=====
~a2 +b2 +c2
33. (d)
Let angle between the two vectors be 8
3 2 =7 2 +4 2 +2x7x4cos8
C
cosy= ,===ea~==
~a2 +62 +c2 cos8 = -1
So, sin a.+ sin 2 J3 + sin 2y =
2
=> 8 = 180°
1- cos 2a. + 1- cos 2J3 + 1- cos 2y Cross product will be zero.
= 3 - (cos 2a. +cos 2J3 +cos 2y) 34. (a)
--t A A --+ A A

=·3-1=2 A= 2i+3j, B= i+4j


28. (c) --> -->
--> • • Area of parallelogram = IA ,x BI = 5 units
P =7i+6j 35. (a)
--+.--+ A A --t --t ' .
P+Q =lli+9j Let vector be A and B
~ A A
givenA+B=7
Q=4i+3j
-->
(when they are parallel resultant is maximum)
IQl= 5 . --+ --+ --+ --+
IA X Blmax= IAIIBI= 12
29. (a)
--+ --+ --+ --+ AxB=12 ... (i)
Given A .l B (Le., component _of B along A is 0) A+B=7 ... (ii)
--> -->
A.B=8+24-4x=O A=4, B=·3
A=3,B=4
=> X=B --> -->
30. (b) :. Minimum resultant· is A- B (when· they are
--> --> antiparallel)
Given BIIA --> -->
--> --> IA-Bl min= 4 - 3 = 1 i.e., unit vector
=> B= kA 36. (c)
=> 61+ 16j:+-xfc = kC3i+ sj-2kJ Consider a hexagon with all E F1
sides equal 0
=> k=2
=> X=-4
31. (c)
The displacement will be
maximum if he walks in
the way as shown after ,I'
/4~_;.-,/
'.. •
A F,: B
walking 20
displacement is sJz
steps ,.,,/e_....-·/ 8 ni . . --+ --+
AB +BC+ CD+ DE+EF=AF
--+ --+
(By polygon law)
--+ -:7

('" :.-··· 8m //
:. He will walk 40 steps for IA.. / Resultant of the five vectors F1 will in opposite sense of
displacement 16../2 m ··-,··.. Sm/ F2. .
Bm • ,
(
, ,10s!eps / Therefore resultant of all the given vectors
i.e. (SF1 and F2) will be F2 -F1 or F1 - F2

www.puucho.com
Anurag Mishra Mechanics 1 with www.puucho.com

D.ESCRIPTION OF MOTION - - - - - - - - - - - - - _________ fu]


·21 dl = 2x dx + 2b db
=KIN§Atj~ =>
dt dt dt
39. (d) => 2l(u) = 2x(:) + 0
Let magnitude of acceleration be a. Also let west to
east be positive direction. Hence
xA = Ju(l0)-½(a)(10)
2
I= JlOu - SOaJ
=(:)
l u
=-U=--
X cos0
Xn = Jcu -10a)(l0)-½ (a)(10)
2
I= JlOu- lSOaJ Alternative :
Let the veloc/ty of the block be v
Let (lOu - 50a) = 11
upwards. Hence velocity of the
xA = l11I, Xn = 111 -lOOaJ block along the string is v cos 0
Hence x A may be less than equal to or greater than x B and perpendicular to string v sin 0. ;_u_ --------:_i
depending on 11 and a. Hence
[For example: If O < 100a < 211 then xA > Xn, if u
VCOS0 =U =>V = --
100a = 211 then xA = Xn, if 100a > 211 then xA < xB ; COS0
under the condition 11 is a positive quantity. However if 11 46. (d)
_, _, _,
is a negative quantity then xA.< xB] Let a p, a,1 , a,2 be accelerations of the particle, fr3.!11e
41. (c) S1 and frame S 2 with respect to ground. Hence
Let both the balls be thrown with speed v O and let _, _,
height of the building be h. Hence vi =v 2gh and 5- aP- a,1 = 4ft 1 ... (i)
_, _,
v~ =v5-2gh =>vA =Vn, aP- a, 2 = 4ft 2 ... (ii)
[Note that v A = v B also follows from law of where ft 1 and ft 2 are unit vectors.
conservation of mechanical energy] Subtracting (ii) form (i)
42. (b) _, _,
as,- a,1 = 4ft 1 - 4ft 2
Suppose the total distance be d.
Time taken for first d/3
d d
I;,,-;,, I= ~4 2 + 4 2 + 2(4)(4) case
m=--=-sec
3x4 12 = (4-Jz).Jl + case = 8 cos~
Let body travels for next T sec then . 2
T \ T 2d d
-x2+-x6=-=>T=- =>
2 2 3 6
So average velocity=~ = ,4 m/s Here 0 is the angle between ft 2 and-ft 1.
-+- 47. (b)
12 6
12 = u(l) +~(a)(1) 2 = u +~ ... (i)
43. (c) 2 2
Since both have same initial vertical velocity (zero in
this case) and displacement along vertical axis is also 12 = (u+a)(¾) +½(a)(¾r
same for both when they strike the ground therefore
time of flight is same for both. 3u 21
=-+-a ... (ii)
44. (d) 2 8
2 . 300 2 . 900 Solving a= - 3.2 m/s2
R - UA Sill d R - UB Sill
A - 2g an B - 2g 48. (b)
__, __, _,
Hence RA and RB depends upon initial velocity of r = r 0 (t -at 2 ). At t = 0, r = 0. Hence the particle
projection which is not given Le. , information is returns back to initial position if
insufficient.
velocity of the particle = dr = r0 (1 - 2at)
45. (b) dt
z2 = x2 + b2 So, particle will come to rest when v = 0, i.e., after time
www.puucho.com
,,
Anurag Mishra Mechanics 1 with www.puucho.com

1
t=-
2a => ¾[(u +~u 2 - 2u J-(u-~u 2
-2u )]
· Positio11 of particle at this m9ment = 3.,Ju(u - 2)

= r0( !-a x (~)2 ) =: 52. (d)


-+
a

·-: ':: '-+ /2 2


\,so distance travel = double of the above distance V =Vxl+VyJ =>lvl=\/Vx +Vy
= _:9_
2a dl;I = ( 2vx ~+2vy~)
dt .,fv2+v2
~ X y
"7
div I= Vxax +vyay
dt fv2+v2
\I X y

=3x2+4x1= 2 m/s 2
.JJ2 +42
53. (d)
s=4t+.!.(l)t 2 =2t+.!.(2)t 2
50., (b) 2 · 2
,_,' (dxJ·'i (dyJ· '
v = dt + dt j = ai + a(l- 2bt)j '' 4t + 0.St 2 = 2t +t 2

"7 ' '


Solving we get, t = 0 and t = 4s.
A,= 0 i + (-2ab)j
So, s=4x4+.!.(1)4 2 =24m
... ,_ ' - -+ . 2
·· · Hence acceleration A is along
. ·, ·, .- ·, ··:' . ,-;, -54. (a)
'negiitive y-axis. Hence when A "7 ' '
"7 v = (ay)i + (V0 )j
,,and· v enclose it 14 between
them the velocity vector makes Vx = ay and VY =.V0
·.angle ·It/ 4 with negative y-axis. Hence dx dy.
-=ayand -=V0
dt dt
· tan 2: = a => [1- 2bt[ = 1
. · 4 1ac1-2bt)I . dy V0
-=-=>· f aydy= s·V dx 0
<.~-:- ' . 1
dx ay , . · .
=> ' 1- 2bt = ±1 => t = - or 0 1 . .2
b -ay =V0 x+c
2
But when t = 0 the y-component of velocity is along
positive y-axis, hence t = 0 rejected. 1 ay 2, = "v x, (·: (0, 0) satisfies)
. 51. (c) 2 0

' . 21'
. Let at .~y time t the displacement of first particle b~ y=± __o_x
S; and that of second particle be S 2 • ---!!,_,,
negative
2
·. S1 =½at and S 2 =u(t-~)
-~2V
y.- -- 0x ·
,
For required condition S2 > S1 a

.=> u ( t--1 >-atJ 2u 2u ,


1 2 =>t 2 --t+-<0 · Also for y to be real x must be negative.
a 2 a a2 55. (b)
2
=> ¾(u-~u -2u) < t < ¾(u+~u 2 ~2u) 0= 30t+.!.(-10)t 2 =>t = 6
2 '
Hence the duration for which particle 2 remains ahead
of particle 1

www.puucho.com
Anurag Mishra Mechanics 1 with www.puucho.com

"
. · ~~-- .. , · <:~~~~-----··
_.-F:'o'ES(RiP.Tfo":q~LM9YI~'.,.,,...___.....,. .... _· ..

r
· 56. (d) 62. (a)
ABP and
triangles.
ACQ are similar
KE= ½m( ~V} + v;
Hence
V t .a
-1= - = :!m((u cos0) 2 + (u sin0) 2 - 2gh)
V2t b 2

~ bV,,1 = aV2- = :!m(u 2 -·2gh) = (-mg)h +:!mu 2


. 2 2
.. · 57. (d)
The graph will be straight line, which will retrace the

OJ
a(~-e) d0
=--
same graph after it reaches its maximum height. Also
kinetic energy is not zero at the highest point.
·dt dt

r
63. (e)
Speed of focus
. = ldxl = d[(d) cot0] KE= ½m( ~V} + VJ
dt dt
= :!m[u 2 cos 2 e + (u sin0-gt) 2]
= l-dcosec 0
2
:~1
OJd '
2
= :!m(u 2 + g 2t 2 -2ug sin0t)
2 2
= idrocosec el = - -
, sin 2 0 2 2
= (½mg } - (mug sin0)t +½mu
2
58. (e)
But horizontal displacement x = (u cos0)t. Hence
-> 1 (->g-a->) t 2
0=Ut+
2 1
KE=-mg 2(-2 - -
X
2
2
) -(mgusm0)-.
• X 1
--+-mu 2
2 u cos 0 ucos0 2
-> 1 (->g-a->) t
0=u+ 2
2 2
=( ~ 2 )x -(mgtan0)x+:!mu
2
-> ->
->a= (gt+2u)
2u cos 0 2
64. (e)
t 2
2
a = ( u ; gt ) upwards.
KE= :!m( lv.2 + v.2 ) = :!m(V.2 + V.2)
2 ~ X y 2 X . y

2 2
= :! m[(u cos0) + (u sin0-gt) J
59. (b) 2
·_1/vith.,espect to: ~ievator the initial velocity of the block
= :! m[(u cos0) 2 + (u sin0) 2 + g 2t 2 + 2ugt sin0]
i~ ·zero ·and the block stans accelerating upwards with 2
anacceleration of 2 m/s 2. Hence
1 .
1 .
KE= -m[u 2 + g 2t 2 + 2ugt sin0]
S = 0(1)+- x 2x 12 = lm upwards. 2
. 2 Parabolic graph.
60. (a) 65. (b)
x2 =t2+1~ dx = t Distance traveled by a particle is equal to area under
. dt ~t2+1 speed-time curve. Hence
d2x 1 1 d = lOx 4+:!1t(2) 2 = (40+ 21t)m.
' 2 .
dt2 - (t2 + 1)3/2 = xs
66. (e)
. 61. (b)
At any time t the distance d between the
Let _x be:the distance between the particles after t sec.
particle is :
Then
,. 1 2
x = vt--at
2
. d = l.(h-½gt )-( h -½gt
2
\~t J[
2
=i(-u)tl =Ut \: -
For x to be maximum Alternative : · ,''-;._ ·
. dx V
-=0 ~v-at=0ort=- Let us take particle 1 us observer.. Hence till both the
dt · a.
particles are' in air the relative a~celeration is zero.
Substituting the value of x, we get I

. v2
Also the relative velocity of particle 2 with respect to
X=- particle 1 is u. Hence d "c ut
2a
www.puucho.com
\\
Anurag Mishra Mechanics 1 with www.puucho.com

~;:;. ·\:;::~0:;:
,it ::S.~·
=~~--~z•/'~;,j:,

67. (a) => (2X1 + l)(X1 -1) 2 = 1


v 2 = 2as => v = ±,/2as => v = +,/2as => 2xf - 3xf + 1 = 1 => xf (2x1 - 3) = 0
68. (c) 3
v~ = 0 2 +2.(a)s => v 2 = 2as. => X1 = 2
69. (c)
Let u be the initial speed of the particle
:. A is (~,i)
v2=u2-2gh 73. (b)
U2 =v 2 +2gh
u; +u; =v; +v; +2gh Cvx =ux)
d =I I-;; Idt =I: It - 21 dt
· u y2 =v y2 +2gh
u; = 2 2
+ 2(10)(0.4) = 12
= I: (2-t)dt + I: (t - 2)dt = 4 metre.

74. (b)
= ..f0.m/s
uy
ux = Vx = 6ml s d --I',, 1
1..,1
V dt= Is I9-2t Idt
4
1
Uy ..f0,
tan0=-=--=-
ux 6 ../3 =I 4
45
.
(9 - 2t )dt + Is (2t - 9)dt = I_ m
45 2
so, 0 = 30° 75. (b)
70. (a) Let h be height of building. Hence
Distance = )(dx) 2 + (dy) 2 1 2
-h =ut 1 --gt 1 ... (1)
2
= (dy)2
dt
+ (dx)2 dt
dt
1 2
-h = Ut2 - -gt2 ... (2)
2
= ~(24sin 6t) 2 + (24cos6t) 2 dt 1 2
-h = --gt3 ... (3)
. 2
= 24I: dt = 96m From (1) and (3) :
71. (a) 1 t2 g
' -g2-=-u+-t 1
Let v be the river velocity and u the velocity of the 2 t, 2
swimmer in still water. Then From (1) and (3) :
t, = 2( ro ) 1 t2 g
-g2.=u+-t 2
.Ju2-v2 2 t2 2
ro ro 2uro Adding above two questions :
t2=--+--=~-~
v+u u-v u 2 -v 2 76. (d)
2ro
t3=- 2
u Time taken for one complete rotation = rrR
And It is obvious from the above that

72. (a)
2
t1 "'t2t3 Total time taken to reach the bottom = t Vo

Clearly A is the point such that OA So, number of rotations = ~~


is tangentto y = (x-1) 3 + 1 at the fg ·2rrR
point A. Let point A be (xi,Y1). 77, (c)
y = (x-1) 3 +1 Let the particles move perpendicular to each other at
,' ---·--" ·-·--·"
=> '[d~]
cJx X=~l
= 3(x1 -1) 2
'
time t.
( · - - - y·---·
l '
-···7
;

Hence equation of tangent at (xi,y 1) is


,'3ml
! .-···
.- ·.,•. . mis
. ·
.' I 1

(y-y 1) = 3(x1 -1) 2 (x- x,) ! '! h ·:\ i


i '- xJ
But this tangent passes through origin. Hence '- •., - . Q_'~--•·--~ " ,.I

-Yi =-3x1Cx1 -1) 2 =>y 1 =3x1(x1 -1) 2 Hence


=> (x1 - 1) 3 + 1 = 3x1 (x1 -1) 2 (4i - gtj).(-3] - gtj) = 0
www.puucho.com
Anurag Mishra Mechanics 1 with www.puucho.com
,- " { .

DESCRIPTION OF MOTi_O~N_ _-~··_·_-'_'~·-·-~--~--:i,~·:_;', .. · f 7_·-:.:::2,~;~·-··-'-,_,.,_"i'_'___


1 +.,J··---,_.-~------•--_-.>-'t"-3-'-"1~

=> -12+g2t 2 = 0 82. (b) . !. ••.\ -~) :;


,.
x2+y2=z2,
=> t = r12
·) => t = ../3
vtjociJ s =>
dx dy.
2x-+2y-. =O
dt dt
dx = -yvy = -yvy
=>
dt X ~12 -y2

1 1:1 = J1 2·;~2 _

78. (d)
Smee y is decreasing ~ I / y - 1 is mcreasing
2 2
Relative acceleration between the
particles is zero. The· distance between continuously. Hence is decreasing
them at time t is
s=~r{h=-~(=v-_-v_s_m_0_)t-}~2-+_(_v_c_os_0_t~)
2
continuously.
or s ={h-(v-vsm0)t} 2,+(vcos0t)2
2 83. (d) ·
s is minimum when The ball returns· back to,boy's. ·
ds2 hand only if the path of the I _ ,fZOm/s
-=0 ?~11- is a strai~ht liJlt, 1-le_nce , _IAm/s2 a_/
dt
2{h - (v - v sin0)t}(v si~0 - v) + 2v 2 cos 2 0t = O minal velocity and net 1- - - - - - •
acceleration must be albng' .- i: .·
h the same line. Hence · · : !· / 9
t=-
2v : , •• · 10m/s2
tan0 = ..±. =>0 = tan-1 0.4. t~. - - - - - - - -
79. (d) . 10
At time t the positions of the 84. · (a) _, - .l"
particles are shown ' in the '
=-t - -~ " ...
figure. r = aO.-_msrotJ i-i;P sincotj
Slope of AB = Slope of BC => x = a(1- coscot) andy = a sin cot
v,t- v 3 t 0 _ v 3t => (x- a):=, -a cos cot andy = a smcot
' C •

=> --~- ../2 ../2 => (x- a) 2 + y 2 =·a 2 '.


_ v 3t t · ~. j.. , 2:f J. :; ,, '

0 85. (a)
../2 ,,,_,., 1·:;r dx
. ·x = 2t;=> V = - = 2
X ' dt
=>
y 2·2t 2 =>V "'dy = 4t
· .•
...._~· ::._.Y
,-,
-~dt
80. (c) \

4t
Let v be the velocity of the particle when it makes 30° · - , · tan0 =
·,
- -· 'V
+-='--2 = 2t
·v;
'

with the horizontal. Then


v cos 30° = u cos 60° => v = 20/ ..J3m/s Differentiating with respect to time we get,
v2 d0 •· .-
Now gcos30°=- (sec2 0)-/= 2
,__ dt ,, .. . ,.
R
2 ' d0 . d0
v2 =>. (l+tan 0)-=2=>(1+4t 2)-=2
So, R=--- 15.4m dt -· dt
g cos30° , ._ ,, . dB-;;· ... z' .'
81. (a) =>
· ,r,·" ;. ·dt · • 1;+:4t 2
Components of the velocities of both the particles m
p ,[::1:2 ~r+ ~2) 2 17
2
vertical directions are equal. Therefore, their time of => -rad/s
11
flights are equal and their relative motion is in
, .,~ ·;.~ ·-::: .... l'. l11;_' '
horizontal direction only. Thus the maximum distance f!6. C~? ,:.. ~- 11 ·,1·L ...:·nn 1 ;__. 1 ... ,
between them is the difference between their From V·S gti;'PP. '
1
'. .. .
horizontal ranges. "' '·duds
·• V =·S => ....,.....-= - =>a= V
l • ···~t'' "dt dt
l

www.puucho.com
Anurag Mishra Mechanics 1 with www.puucho.com

; MECHANIC5:~ .
--+--+ --+ --+
r1-r2 = Vz-V1

'.!.· ca, b, c, dJ
11, - 121 ii'2- ii', 1
1

Graph (a) ~dicates two c\lsplacements at a given time, Alternative :


which is imposs)b!e. . '. · ' Let us assume that the· reference
Gr~ph ·(b) 'iridj~tes. two velocities at a given time, frame is rigidly fixed with particle
which is impossible. 1. Hence
-, -, -,
<;rraph; ,(~) hi.di~i!'t~s speed can ·be negative, which is r2 w.r.t.1 = r2 - rl
_impossible.·_ . ' -, -, -,
.Graph. (.d) _inµic~tes distance-travelled increases then Vz w.r.t.l = Vz - V1
'ciecre's,.ses, whi~ 'is impossible, Fpf the particles to collide with each other, the
2. 'o;, ~] :'' ' .
,.;. ;~ . <...:,. . • ' •
' ' particle 2 must be moving towards particle 1. Hence
Tii~:lift is /ICC~lerating downwards with acceleration g. 4 --+ --t --t
Cr2 -r1 ) _ (v 2 -v 1 )
. · !ieic~. a~~~ler'!,tion of sto~e in lift frame is g - g = 0.
--+ --+ --+ --+
3. [a,'~, ·er . . . /r2 -'r1 / /v 2 -vd
· (i) t~¥,~rd is blowing alongfIB. Hence total time T for 6. [a]
the f<;!\l!ld h:iP is Since Q moves along a smooth
·:.' . l l 21V ; '""ijJ.';--. ----·:--· '
T=tAB +tBA = - - + - - = ~ -- horizontal rod its velocity , i -- •• - ~ - ~ .
. . V + V V - V v2 - v 2 remains constant. But as P '.•
• -- ------>vsin B .
.

(ii),):.eft,· >}\7\nd,,:. is blowing r-·,,··-:~ moves downwards its speed l . •• .·'


I 8"· ..... .!

perpen#i!;U[ilr to AB. Hence ; windv cos increases. Therefore its __v.cos.B_! _____ _J
8 horizontal component of
ii=Vsin0=>sin0=~
V
;,A
I
Bl velocity v sine increases and becomes maximum at
.l LY§in~Q-~·-' lowest point. Afterwards it decreases gradually &
tAi, =-.-·- becomes minimum at B; but at B, the. ho.rizontal
' Vcose
' l component of velocity is equal to that at A. f!ence
horizontal component of velocity of P, is never less
~. V·I--
~2
.Jv2 -v2 than velocity of Q. Since horizontal ·displacements of
.... v2 both are same, therefore, P takes less time o'r t p < t Q.
Similarly . · .
l Hence (a) is correct. · .,.
. ' ' .tBA =-;====
.Jv2 -v2 7. [b, c, d]
21
Hence· T=tAB +tBA =-;==== Both A ·and B have same hmax· Hence
. .Jv2 -v2 (uA siir0A)
2
= (uB sin0B)
2
4;. [b]
2g ' 2g
R = l!2 sin20 => dR =·(u2 sin20)(-l)dg
' g g2 => UA sine A = UB sjn0B => (Uy)A = (uyh
Hence option 'd' is correct.
2
Again time of flight ,
=> dR=-(u s:20)(~)=>~=-~
= 2 ( vertical velocity of projection)
s. [b] g
If the partic;les collide at time t then Hence time of flight of A is equal to that of B. Hence 'a'
i)+v; t =i-'2+1½ t => (ri-r;)= (v-;-v;} ... (i)
is wrong .
Since range of A is less than that of B and time of flight
of A and B are equal, therefore
... (ii) (ux)A < (uxh·
Hence 'c' is correct.
Speed of projection= ~Cux ) 2 + (Iiy) 2
Since uy is same for both and (ux)A < (ux)B,
therefore, speed of projection of A is less than that of
B. Hence 1b' is correct.
www.puucho.com
Anurag Mishra Mechanics 1 with www.puucho.com

~ DESCRIPTION OF MOTION

8. [a, c] => JA-BJ= Ibft~ +ac-11 1 )[°


Since the particle starts from rest and finally comes to
rest therefore the particle first accelerates and then => X = -Jb 2 + a.2 + 2ab COS0
retards. If it is assumed that the particle accelerates => la-bl:,x:,Ja+bJ .
uniformly only once and retards only once in the Hence 8 is the angle betwe~n ft 2 &·-ft,.
entire journey then the velocity-time curve is two 10. [a, c]
straight lines forming a triangle with time axis. Also
If the velocity of 1 · !

rs
area of each of these triangles is one unit since · pacl<et
displacement of the particle is 1 m. Hence in each of
the above
aeroplane is u m/ s when
the packet is dropped g .
I
I
,- - - ____,_ -- - -- --~- - -·- ' then path of packet is IWest•-····:. ground- ···>Eastj
_
mentioned
motions the
! parabolic with respect to
I ground as shown in figure. .
maximum velocity
of the particle is 2 With . respect to
aeroplane the initial 1',· ..
rs, . ,
4m~s 2
. ···-·>4m/sZ-7
2 . l
m/s. Again, if it is ••• O~s \
assumed that the 1s A velocity of the packet is I' • I
1West
,- . c-'- -----·-----······>East
gro~risJ._ _ ___,
particle zero and acceleration is
accelerates for.! s and retards for .!s then the v-t curve as shown in figure.
2 2 -1 5
8 = tan - = tan -
-1 1
west..of vernc . a]'.
is represented by the triangle OQA and magnitude of 10 2
slope of the lines OQ and QA are both 4. That is the 11. [a, b, c]
acceleration and retardation are both of magnitude 2
h _ (v 0 sina)
4 m/ s2 • Also from the figure giyen below it is obvious lmax - 2g cos8
h2max => (a) is correct
that if acceleration has magnitude less th~n 4 m/ s2
2v 0 sin a (b . · ·
then retardation has magnitude greater than 4 m/ s2 T1 = - ~ - - = T2 => ) 1s c.orrect
g case .
and vice-versa.
Again if acceleration is not uniform · still then the R, =(VoCOSa)T, _.!gsiilBT,~
2 '
magnitude of acceleration or retardation of the
particle has to be R2 =. (v 0 cosa)T2 + .!2 g sinBT,,2
greater than or Q
equal to 4 m/ s2 (R 2 -R1 }= g sinBT,2
tane =4
at some points in => (c) is correct
the path. One of v ,, & v ,2 are the velocities of the particles at their
such possible maximum heights. Let the particles reach. their
motion is shown maximum heights at time t 1 and t 2 respectively. Hence
in the given below. 0 = (v 0 sin a)- (g cos8)t 1
Note that it is not necessary that v max should be 2 m/ s . v 0 sincx
t, = ~ --
The only essential condition is area under v-t curve gcos8
should be 1 unit. v sincx · ·
9. [b] Similarly t2 = 0 . Hencet 2 =t1
gcos8
Let accelerations of the particle, observer A and
Hence v,, =v 0 cosa+(gsin8)t1
observer B be p, A, Bm/s 2
with respect to ground. v,2 = v 0 cosa+ (g sin8)t 2 ·
.
Hence as per question =>
...,p-A
..., -
= an 1 ... (i) 12. [c, d]
vt1 -:t=vt2·

p-B= bn 2 ... (ii)



smrot =-Xa and cosrot
·-
=1..-· ·a-y
·Hence ft 1 and ft 2 ate unit vectors depending upon
direction of acceleration of the particle with respect to
respective observers.
Subtracting (ii) and (i)
=> x2+(y-a)2 =a2,
which is a circle. Hence (c) is-correct.
A-B=bn 2 -aft 1 www.puucho.com
Anurag Mishra Mechanics
Ul-'· 1 with www.puucho.com
, '7._,>;;:,~ (n .-
• -,ci

"
1
1
,•,·. c1.1 ~fJ' \' L..,.,,.

·c1x ., ' dy . dv
vx = - .. = arocosrot and vy = -.-·=
1
arosmrot t:0~ 1~ v > 0and-= 0
dt. ~<--;l?-:-,1 dt ' ' dt
· I 2 2 ·· ds · d 2s ·
. v=vvx.+-py =aro ->0and-=0
dt dt 2
Hen~~ particl~ 'is in~~ 'b'i(/i\cirtular path with
constant speed am. -Hence distance. travelled' by it on => s-t curve increasing and a straight line.
circular path _in time 'i is"a,~t; _Heft~_e.,(d) is· con:ect. t:1~2~v>0and-·· <0
· dv
. dt
13. [a, d] _ : f'='i--..:_Ji/1~t•'"',;:-i-1 ·-,\·:, .
2
ds , d s
(a} is correct because. . . ', .. - have a non-zero
. , particles\ .=> - > 0 and - 2 < 0
relative velocity (always) and acceleration . dt dt
iela_tive lo one anoth~r
is 1Zerci (both are falling => s-t curve is increasing _and lies below its tangent.
withg). !--) T·;; ·· , . dv
(b) is wrong because if a body is psqjected vertically, . t:2 ~ 5 ~ V < 0 and - < 0
' dt
it wil\ not follow•pataboliclpat4, ds · · d 2 s
(cl is wrong because at higl!e~t point ,of projectile -<0and-<0
dt dt 2
velocity is perp~~<liC1!1.\lr t_q,a~celeration.
(d) is correct since particle has, uniform horizontal => s-t curve is decreasing and lies below its tangent
velocity._and constant'<i\cce\eration (down~ard) · dv
t:5~ 9~v < 0and- < 0
Le., it.will trace.a par~bolic<paJ]i. . · . dt
14. [a] ·'-'(: ,:: :1;,P.q ::~Jr"l,, ' ds < 0 and d2s > 0
Only (a) is correct because aeroplane wilf'provide dt dt 2
same velocity with which it is flyingand·in·the same => s-t curve is decreasing and lies above its tangent.
direction of its flight. c ' · · ·, . dv
15. [c] :nu ,,., -:i " · t:9~. 10~ V > 0 and->. dt
0

·--·
· At time t, position of.lines are shown in the diagram.
r"-;-..--~il;,''",_,i'i'~',;>-=.-~-:-., ,, • =>
ds
->0and->0
d 2s :
. ';·~:·:~_~N:;:A .~~~·:1_ :--~~~- ~ dt dt 2
···v2t •. t => s-t curve is increasing and lies above its tangent.
!-
.1 - , :
, , __
',F~t, '
:_::·__ - - · ~ - " ; . } : ·

. a_ ·z::::-t:::<.t.-i;.: ...: -
1
.
;··1: .....

I 17. [c]
· i- ,__·-.{ .....tv 1_t . a ·,>"ir,'',.,' ',· >· The ball will stop after a long time. The '.final
·-•• -:."9: . ~: ·...... < • '
displacement of the ball will be equal to, .the height.
L~
··
· ·:~.2:~vt-~:. ~~--:t,'--vt··
f_~-J~:~1~ _j· The motion is first accelerated, then retarded, then
accelerated and so o~.
2 1
PA= - - PB'= - -·!'Hence 18. [b]
· sin a' .. sin ex
The velocity of the particle first increases linearly and
PP'= ~~A 2 +:P~;:+ 2PA.PBcosa-; then at the point of collision it suddenly changes its
= ~v; +v~ t2111v, 2 coscx(~~J direction and then starts decreasing·in magnitude and
· s1na) the pro~ess is repeated again and again. Also every
r·. - ,• ._-:-~ I • '

Velocity of point P' = p_p collision decreases the speed to half its value before
t . collision. Hence graph given in option 'a' is v-t curve
~rv~f_+_V~~-+-2-'-V-1_V_2_C_O_S_CY. and th.at given in 'b' is speed,time curve.
19. [a]
sinu
v 1 =a2 (t:t-t 1 ) (forcarB)
16. [c]
V +v
1 =1- -1- a t
For case A we C'l,Il write . , , ---
Vi a2 t + t 1
vfinal =v.+v1
a1 .. ·(V+v 1)(t+t1 )
T = t 1 , distance =s • -= - > 1 ==>a1 >a2
U2 . v1 t1
For case B
=V1,T=t1 +t, distance=s
. ,vfinal 20. [a, d]
(V+v 1 ) = a1t 1 · (forcaseA) So, velocity of first particle
' ..:.::' '

www.puucho.com
Anurag Mishra Mechanics 1 with www.puucho.com

I DESCRIPTION OF MOTION ,,. ., ,~ . ' t-,. r

13sl
= 3cos30°1+3sin30°j - •.·' d ' .1
23. [a, b, c]
12: 9:
=-1+-J IJ. 'A- - -'p
~ - " - - ~ -B I
j
5 5 ~sec 14 in/secl
velocity of second particle
142 =2 2 +2xaxd
= 4cos 53° i + 4sin 53° j
at mid0 point let velocity is v
12: 16:
= s 1 +sJ => v 2 =2 2 +2xax~
2
So, relative horizontal velocity is zero. So their relative
velocity is vertical only. Since both particles are
=> v2 = 4+ 142 - 22
2
moving under gravity, so their relative acceleration is
192
zero. v 2 =4+ =100
2
. 1 .
Therrreanvev 16 9 7 14 m/s
el oCity=---=-=.
.
5 5 5 v = 10 m/sec"""7 if AP=.! AP=~
· PB S 6
21. [a, b, c]
dv Let velocity at P is v 1
a=-=A-Bv
dt 2 2 ' d . 142 -2 2
v 1 =2 +2xax-=4+---
"""7 max. possible velocity is terminal velocity (i.e., 6 6
when a= 0) => v 1 = 6m/sec
=> A-Bv=O Let time taken to reach mid-point from A is t 1 , and t 2
"""7 initial acc. is when t = 0, u = 0 be time taken to reach B from mid-point.
a=A-O=Am/s 2 6= 2+at1 ••• (i)

dv
-=A-Bv dv- = dt
=> v- - f f, 14 = 6 + at 2
t 4 1 ·
.:. (ii)

dt OA-Bv o
...!. = - = - => t 2 = 2t,
..!1nA-Bv =-t =>1-.!l.v=e-"' t, _8 2
B A A 24. [a, b, c, d]
A(l
- -e -Bt)-
-v Since the graph is like a· parabola
B :. let x(t) =At+ Bt 2 + C
22, [a, b, d] ·
From graph x(O) = O => C = O
Total distance
Average speed = x(t) = Bt 2 + At
Total time
. displacement x(4) = 0=> 16B+4A = 0 ... (i)
Average velocity = - ~ . - - -
nme
distance > displacement
(dx)
dt o
=1 =>(A+ 2Bt),=0 =1
'
:. Average speed > Average velocity 1
Put in (i), we get B =-- -
_, 4
dj vi = tangential acceleration t2
dt X=t--
_, 4
dj vi = net acceleration max. x coordinate·= 1 (from max. and min._)

[Q]
dt """7 Since motion is a straight line motion
"""7 In uniform circular motion """7 total distance traveled = 2 x 1 = 2m(
_, _, . 2 .
d!vl =0 dv *0 Average speed= - = O.Sm/sec
4
dt ' dt
25. [b, d]
"""7 In circular motion from pt. A to Pt. A again Separation between. them will be maximum when
both particles have same velocity. This situation come
Average velocity = 0
at t = 2 sec, but just after it, first particle comes to rest
lnstaneously velocity ;e 0 (at any time)
and second 1 m/s. So first particle will again gainthis
velocity in next one second. So, maximum separation
will ocCIIr after 3 seconds.
www.puucho.com
Anurag Mishra Mechanics 1 with www.puucho.com

j'---1 c.:3~6-·-------~---~_;_________ ~._ __,___ __o..',.,_.-...;·.__..;___._··...:.M_EC'--H-'-A.,__Nl~Cs_:1..:J! .


Maximum separation 5. [bl
= Displacement of seconds particle - Displacement At t=2sec (from Q. 2) JA ~ 4 ,m/sec .\
of first particle in first 3 seconds They meet for 1st time [!~16m/~cj

= (2x 2+ lx 1)-(.!. x 2x 2+.!.x1 x 1) v(2)=24-8=16


· 2 2 Relative velocity = 12 m/sec
=5 - 2.5 = 2.5 m 6. [cl
• . ·&;e1.'i;:c;;;;"'eh=e=n~""i;-n""'"ii=--se"'"d""''i,-;-;;-b""j;=;"'~,;------. At t = 8 they meet for 2nd f A.,_-.,:4m(sec;
lll,::e::--z:r-,rc::::::e:r::!::'t·-n· • 4 c;-=ur ~ j@&s, time !32.m/sec I
B 1
Passage-1 v(8) = 12 X 8 - 2 X 8 2 1

·
a= 4(3-t) =>- = 4(3-t)
dv = 96-128
dt =-32
=> f:dv= J;4(3-t)dt Relative velocity = 4 - (-32)
= 36 mjsec
=> v=l2t-2t 2
7. [cl
2 2t 3 V = 12t-2t
2
=> x= 6t - -
3
= 18 - 2(t 2 - 6t + 9)
1. [al
For particle B to stop v = 0
Passage-2
=> 12t - 2t 2 = 0 => t = 0,6
1. [al
2x 63
x(6) = 6x 6 2 - - - 500
t=--
3
vcos8
=63(1-¾)=6: =2~6=72m 5
· = 00 for 8 = 0°
t mm
V
2. [al -
2t 3 2. [cl
For particleBx(t) = 6t 2 --
3 For u = 3 km/hr, v = 5 km/hr
For particle A x 1 (t) = 4(t + J) mm
500
t . = - - ' - - 360 sec = 6 min.
5x1000
For particles to meet x(t) = x 1 (t)

6t
2
-3
t=8,t=2
3
2t = 4 ( t +3 8)
3. [alu = 3 km/hr, v = 5km/h
to reach exactly Pt. B
vsin8 = u => 5sin8 = 3
. 8 3
Slll = -
GZJ,
I .
r-~ ~ ·, .· :,
··.··11

5 ~¥-·- --- - -··· -- - ....,. . .
time interval = 8 - 2 = 6 sec
500 0.5
3. [bl t=--=--
vcos8 5 xj
Required position
5
x(t) at t = 8 or x 1 (t) at t = 8
= 0.5 hr = 60 X 5
1 8
=4(8+¾)= ~ m 4 4
t =30
- =75 .
. mm
4. [al 4
For particle B V = 12t- 2t 2 4. [dl
x(6)=72m U = 5 km/hr, V = 3 km/hr
x( 8) = 128 again to reach Pt. B
3 vsin8 = u => 3sine = 5
Total distance;;,'d + d1 = 72 + ( 72 -
1 8
~ ) sin 8 = ~ not possible
3
304 :. The swimmer can never reach to Pt. B
=-m
3
www.puucho.com
Anurag Mishra Mechanics 1 with www.puucho.com

[_•._DE_SC_R~IP.J~l~~-O~F_'~~O~T!~O_N_._ _ ~--·-·-----------------,____-·_··~~--,3IJ
5. [a] 4. [d]
From question 3 required angle '0' is At the top of trajectory speed = u cos0
. 0 =-
sm 3 = 5../s m/sec
5 5. [b]
i.e. 0 = 37° u 2 sin20 (10v'5)2 sin 120°
Range=---
6. [a] g 10
U = 4 km/hr, V = 2 km/hr

=
lOOx sxF3
d = 25.)3 m
t=-- 2xl0
vsin0
drift = (u - v cos0) x _d_ Matching Type ·Pr~em?i;;--:--_,
vsin0
11. (A) Corresponding w graph will
x=(4-2cos0)x~

. . .
vsin0
For x to b e maximum or m1n1mum -dx = 0
d0
n/C',u,.l
l
=> Speed time curve will be
From this we get sin 0 = .!. => 0 = 30° ,- . -----7
~
7. [a] l~Ol ·
I 1,i,(1 1,; I,, i speed decreasing in·(t -t 4 )
Here v>u ------ u=2I 4
· - - _:,______:__i
3
'
Minimum drift = 0
=>

Passage-3
4sin0-2= 0
0 = 30° !·~
. - ----· '-··
(B) (x) ( : ) < 0 to return (P,S, T).

(C) x(:) > 0, i.e., in (Q, R)


1. [a]
Let speed at P = v (D)Speed in increasing in (t 1 -t 2 )(t 4 -t 5 )
15. (A) Slope of a1 and a 2 is +ve parallel
Range PQ = .J15 2 + 20 2 = 25
Slope of a1 > slope of a 2 R
v 2 sin 2 900 v 2
25=----=- (B) Both v 1 and v 2 are +ve parallel
g 10
v 1 < v 2 (obviously)
v = -J250 = 5-Jio m/sec (C)
2. [c] 15'[5-·---··-- ·'" usin0 -- r··--·1 Slope of v 1 is+ve }
anti-parallel
Slopeofv 3 is-ve
!t t /
_,s-[fo " !
i~~~. '---~~-·
(D) Slope at sis +ve a1 > 0 :. parallel
V1 > 0
-----~_:'.'.~_~j a1 and v 1 cannot be compared.
u 2 sin 2 0- 2x !Ox 12.5 = (5-J5) 2 -+AA-+ AA
16. AF=ai+aj+ak DG=ai+aj-ak
u sin0 = .J125 + 250 = .J375
b= i
and u cos0 = 5../s = -/125
(A)AF · b = AFcosa
tan0 = ~375 = F3 1
. 125 => a=..ffacosa cos a= -
F3
0 = 60° -> . ->
3. [b] (B) DG ·AF= (AF) (DG) cosp
ucos0 = 5../s
· and
a 2 = F3a F3a cosp cosp = -1
0= 600 . 3
-> • ,. -> • •
u= 5../s = lOv'S m/sec (C)AE =aj+aK AG=ai+aj
1
-> ->
2 AE ·AG= (AE)(AG)cosy
www.puucho.com
Anurag Mishra Mechanics 1 with www.puucho.com

...,_

Tl \,~

\\;@
r ,. -· " , ~ \:
\
\
}
,>':'~=r-·
: .-.:":;: I
\\.,(''V.:'_y ::~--- 1
I
~:b"'"."'t:' · ..;.;..._.
'iI{ff,!j{{fliil:;;,
<~~ ;( , -.:
d u ..
>:
~ :OI

' "G:l~~-...
""" ~, i
M •
FORCE ANALYSIS /

....... ,

THE CONCEPT OF FORCE Important Concepts


Force may be defined as action of one body on another. 1. The various interactions known in modem physics
In order to completely specify a force its magnitude, can be classified under four headings.
direction and point of application should be specified. Effect (a) gravitational interaction appearing between all
of force depends on magnitude of P, the angle 0 and the bodies in accordance with the law of universal
point of application as shown in the Fig. 2.1 gravitation.

[ : ~ ...
1
,,,,,
·::.~~--n -
~
:I
(b) Electromagnetic interaction-between bodies or
particles having electric charges.
(c) Strong interaction existing, for example, between
' p the particles which atomic nuclei consist of, and also
+ between mesons and hyperons and
-----~I'.,.,_-~ (d) Weak interaction characterizing, for example, the
processes of transformation of some elementary
particles.
2. In problems of mechanics, gravitational forces (forces
Fig. 2,1 of gravity) and two varieties of electromagnetic
forces - elastic forces and friction forces are taken
Forces can be generated through direct physical contact.
into consideration.
They may also be applied through distant action of fields,
3. The forces of interaction between portions of a
e.g., gravitational force keeps objects bound to the eartb, a system of bodies being considered are called internal
bar magnet exerts force on a piece of iron, etc. forces.
Force is a vector physical quantity that is a measure of The forces exerted on bodies of a given system by
the mechanical action exerted on a point particle or a body bodies not included in this system are called external
by other bodies or fields. A force is defined completely if its forces.
magnitude, direction, and point of application are given. A system of bodies on each of which no external
The straight line along which a force is directed is called the forces act is called a closed (isolated) system.
line of action of the force. 4. If several forces act simultaneously on a point particle
(F1 ,F2 , ... ,Fn), they-can be replaced by one force F,:
The action of a force results in a given body changing called the resultant force and equal to their sum :
the velocity of its motion (it acquires acceleration) or n
deforming. -F,: = IF,1
i=l

www.puucho.com
Anurag Mishra Mechanics 1 with www.puucho.com

·- - · " ___ , ____ I

FORCE ANALYSIS 1391

The components of the resultant force onto the axes Concept: 1; In all systems of reference which are in
of a Cartesian coordinate system equal the algebraic uniform rectilinear motions relative to each other the
sums of the corresponding components of all the acceleration of a moving body is the same.
forces: 2. Experiments show that the forces acting on the bodies
n n n
LF, =IF·1r·
w
LF
y
=IF..,,,
w
LF, =IF;,
w
and the mass of the bodies are independent of the choice of
any of these systems of reference relative to which the motions
5. Mass is a measure of the inertia of a body; i.e., the of the bodies are considered.
mass of a body is a measure of the body's resistance 3. The forces depend on the distances between the bodies,
to acceleration. Mass is a fundamental property of on their relative velocities, and on time, all these quantities
matter just as length is a fundamental property of not varying when we pass from one system of reference to
space and time is a fundamental property of another system of reference which is in a uniform rectilinear
existence. motion with re~pect to the former.
6. Every object on or near earth's surface experiences at
least one force acting on it, its weight mg. If we choose an arbitrary set of frames of reference
7. In order to study motion, we must specify system which are all in uniform rectilinear motion relative to each
first. A system is a collection of bodies or a single other and if, in addition, it is known that the laws of
body under consideration, whose motion is to be dynamics hold for one of these frames then the first and the
studied. second laws of dynamics are stated in the same manner for
8. A system in mechanical equilibrium has zero all the frames of reference we have chosen. All such frames
acceleration. Acceleration is rate of change of are referred to as inertial (or Galilean) frames of reference
velocity, hence zero acceleration implies the system and the Galilean inertia law is valid only for such frames.
has constant velocity, whose magnitude and This is the proposition we call Galileo's relativity principle;
direction do not change with time. the transformation from one inertial system of reference to
9. Total force, net force, resultant force mean the same another is called a Galilean transformation.
thing. A system in equilibrium has zero force on it.
System in equilibrium Concept: A frame of reference which is in an
accelerated motion with respect to an inertial frame of
a= om/s 2
reference is spoken of as a non-inertial frame of reference.
<=:> Zero total force on the system
-> Which of the systems of reference we deal with can be
F1otal =0 regarded as inertial one ? However, the investigation of
System not in equilibrium motions whose velocities are small in comparison with the
3;e om/s 2 velocity of light indicates that the coordinate system whose
origin is connected with the centre of mass of the bodies
<=:> Non-zero total force on the system
-> forming the Solar system and whose axes have invariable
F1ota1 ¢ 0 directions relative to the "fixed stars* can be taken as an
Reference Frame inertial frame of reference. The experimental data obtained
The laws of dynamic can be stated the same way only both in the study of the motion on the Earth and from the
astronomical observations confirm the validity of this
for the inertial frames (system) of reference which are in a
assumption.
uniform rectilinear motion relative to each other. Suppose
v9_,•..........-......... v,
that there are two frames of reference (see Fig. 2.2) one of
which, denoted 1, is regarded as being at rest (Le., as being
fixed) while the other, denoted 2, moves relative to the
z'
~----·," ·,
former with a constant velocity v O. Then all the bodies
which are in a state of rest with respect to the latter frame of
reference will move with velocity v O relative to the former y'
and the bodies moving with velocity v 1 relative to system 2
will obviously have the velocity v = v 1 .+ v 0 with respect to x' y
system 1 (assumed to be fixed). The velocity v O being
constant, the acceleration of a body relative to the moving X
frame of reference coincides with that relative to the fixed
Fig. 2.2
frame of reference and vice-versa.

www.puucho.com
Anurag Mishra Mechanics 1 with www.puucho.com

.
( ·-~;,,
'.'

As to the frames of reference connected with the Earth, The bar can be considered as a free body because the
· they can be considered inertial only approximately. This force of gravity of the bar is balanced by the buoyant force
·. approximation involve§ some errors which will be analysed while all the ,other actions on the bar may be ignored.
later .on. · It is known from experiments that when the cart moves
Newton's first law· of motion 'is related to the state in this way the bar will approach the left wall of the vessel.
·of equilibrium. If a system is in a state of eqtiilibrium it will In the first case, the behaviour of the bar is interpreted
remain in equilibrium unless compelled to change that state on the basis of Newton's first law: the free bar continues in
by a non-zero force acting on the sy~tem. its state of rest (its unchanged position in the coordinate
• 1 '\,·, . • . -- . • , . • ··7:-::··'"1 system XOY), whereas the cart together with the vessel
Concept; 1. N~to!l's first law; establishes thefqct oft, travels to· the right (the lefr side of the vessel approaches the
the existen,e J!f inertial reference :fr;ames an:d describes the\ bar with acceleration).
11ati.tre of,th¢tnotiort of a free poi~t'p(lrtic/~ in'.ari iner)ti~f! •. ·- ·-"':""-·"·-------~-----,,..- .---..----·:' . ·,,, _1,7
. riiferenr;!?.fra,rne. . . : '. '. . . . . . . Ct>nc!lph In the second,. the bClrmoves iyith acceleration
2. Referehctframes in which dfr/er pointparticlds;i~ its (non-/nertiallyJ to the left without 'any actions whats¢eyer on
state pf rest or ofµniform moti.on/n;'d'straightline are defined it in this.dire,tiort, while the carf\,,ith' the vessel is ai·i~t'.iri ·
as inertial'refeferice frame. . ·• :. ' . · · ' . the coord,inate system X'OY'. Here.Newton's fi~st latv 'is "not
- a: ke.te~ert¢j frdmes in ~hich lifree point particle or.fre~
observed for 'the bar (the bµr performs non-inertialn1ptiori
although_ii'mqy be 'considered. as ajree body).. . j
body.does.not
f..,
retain a constant.veloeir:j
' ' ' \• ,, ' . .: ' .-.
(i11.insinertial
' • ..
motiori). ·,
~are ~fifre~ ~ rt/!ll·iner:tiiI{ referertfe:-frqmes. · . • . ..\ · . · Newton's second __, law : Acceleration of a system
·e
i.• ·_,_1: 4. :" r!'f1_"_~n. ·.ce fr.wn. trµyell.~g_ _;w_J.th.·_a._c_ ·c.el.er~tio. n_ r_ei_ci.··tt·
to. an merµal reference frame is .a• tt.on-merttal one; ;In.
I1norl'iilertialfrq,!lf4,
-~~ral depends on total force F1o1at acting on the system. According · ·
to Newton's second law of motion a system of mass m, .
·even ajree bogY,,cq.n.perform nori!irierti.al, __,
f!!:.£.t!en, Le,. tratel with. acceleratfoni:, · , J subjected to force
is given by
F1o1at experiences an acceleration 1.which .
CONCEPTUAL EXAMPLE-1: A stationary cart carries __,
a vessel with water in which a wooden bar floats (Fig. 2.3). ~ Ftotal
a= --
Describe the behaviour of the bar in accelerated rectilinear m
__, __,
motion of the cart to the right using two reference frames: . i.e., L F,x1 =ma
(1) a stationary inertial frame associated with the surface
over which the cart travels [the coordinate axes OX and OY Vector sum of forces on the system (action taken by
of this frame are shown in Fig. 2.3 · (a) and (b)l a external agent) =Response of system

, fill-. 1
non-inertial reference frame associated with the Newton's third law : According to Newton's third
~ccelerating cart. [axes O'X' and O'Y' in Fig; 2.3(c)] law; if a system A exerts a force on another system B, then B

-~~Iro-----:·i .. '.:'7
exerts a force of the same magnitude on A but in opposite
Y. direction, which implies that forces always occur in pairs .
•, , ___ -- ~ - , I , ,V. _8 :
:. ' 0 ", T~•,••• ••• •••• •.·, ... ~
(a) Forces that constitute a pair act on different bodies: ·
· , __ ' . n 1nmii,71111fiJ11,piFAm1m ~ ·: : .\ The two members of a given. force pair point in opposite·
·~"
,.·,,.!',•.
'.;'."' -'f,.
.. . •" <•1 .. !~, . ·I directions. ·
(b) Each member of a given pair of forces has the same
• -~ I magnitude .
. · er· v
!===-="\"···
·i
! (c) While applying Newtonls second law; consider force
o,-- L • • •' _ .- - 1·
exerted on any system by other bodies. Thus only one force
,x· #· •. of the pair is involved in applying the second law of motion,
e.g., if we are studying system A, then the force on A by B is ·
', relevant. Force of A on B will try to accelerate B.
·'

www.puucho.com
.~., ·. ',!:
: ,.:_
Anurag Mishra Mechanics 1 with www.puucho.com
.-

L~CE ANALYSIS --- ---- 141


: ----- --------7

: One system:
;you GravJta.tipOal f9fce: •
!.
I
1

Force of the • of t~e E•rt_h on you 1


/
surface on yoti.;',--,f-11~
.------ .. - Gravitational.force
of you on the Earth /

Force of you - - = . i Another system , .


on the surface N the surface
(b)
(a)

L.
Fig. 2.4 I
- . - ---- - ---- .. - ----·· ---·--· ·------- . .. .. "" ------- ~---~-- -·
(a) Collision (b) Boxer (c) Tennis (d) Attraction (e)Gravitational (Q Block
of billiard struck by ball struck between attraction attached
balls opponent by racket two magnets between to stretched
skydiver spring
and earth
,
C
0 A

~
~ A1B ~
:;:, A•B A B
" G}G
~
-
Q)
ll'CSDE:S
i "iiiiii~"-il ii :

~r ~
.!:
<(
C A

~
0 A A
Q)
!:! ~ ---e NI::$-
0
LL

- r
m B


C
0 B B
B
Q)
!:!
lll," ~ -N:=l! +{]
0
LL
I

Fig. 2.5 1
____________ ._. - , I

CONCEPTUAL EXAMPLE-2 : Let us consider a Besides the force Fwp with 1- - - - - - F--,,-- ·7
weight lying on a man's palm (Fig. 2.6). The force exerted by which the palm acts on the weight, i · wp i
the palm on the weight is FWP; it is applied to the weight and the weight is acted upon by the· ;
l
is directed upwards. The weight, in its tum, acts on the palm force of gravity, that is by the force i !
with the force Fpw which is applied to the palm and is generated by the interaction '
directed downwards. Now imagine that the man lifts his
palm or lowers it. By the third law, in all the cases, we have
between the weight and the Earth;
we denote this force as Fwe· Now
.. . " .. ,IJ.•
; -:, .
I
Fwp+Fpw=O • we can determine the resultant
·' .. , ~·
.,.l~
This equality always holds irrespective of whether the force acting on the weight and find
palm supporting the weight rests or moves. the acceleration of the weight
The third law does not characterize the magnitudes of which is the sum of the two forces
the forces and only asserts that they are equal. It is also
important to stress that the forces of which the third law
Fwp and_ Fwe. According to the
resultant is equa\ to the product of
I
speaks are always applied do different bodies. the mass of the weight by its - - - - - -Flg.2.6
- - - - - --- _J
Let the palm move in a certain way. It is required to acceleration:
determine the forces acting on the palm and on the weight ~+~=~¾ . •.
_.and find the acceleration of the weight. Hence, if the magnitude of the force Qf gravity Fw, is .,, ,t\:~
..
greater than that of the force of the palm Fwp, the ,1·
acceleration of the weight is directed towards the Earth; if I
www.puucho.com
Anurag Mishra Mechanics 1 with www.puucho.com

________ ,
MECHANICS-fl

17,.
otherwise, that is if the magnitude of the force of the palm
exceeds that of the force of gravity, the acceleration is
directed upwards.
Concept: The magnitude and the direction of the acting,
'
force determine only the acceleration but not the velocity and; F,p
,therefore we cannot find from the direction of motion of the:
weight. For instance, when Fwe > Fwp the weight can be eitherl
in a downward accelerated motion or in an upward: .
decelerated motion. More precisely, when the acceleration is·
directed downwards the velocity can have an arbitrary,
'
'direction; it can go upwards or downwards or even form an
angle with the vertical. The direction of the velocity at a given; ~

moment has no direct connection with that of the acceleration:


,while the acceleration itself is completely and uniquely
:determined by the acting forces.
If the acceleration of the weight is equal to zero, the sum
of the forces acting on it must be zero; in other words, in this
special case the force Fwp of the action of the palm on the
weight is equal in its magnitude and opposite in its direction
' '" " nmnmm mmmm
to the force of gravity Fwe. In these circumstances the weight
can be in a state of rest or in a uniform rectilinear motion Fig, 2.7

with any constant velocity.


CONCEPTUAL EXAMPLE-3 : A weight is suspended 1
Concept: Thus, an "inertiafree 11 spring 11 transmits 11 a
from a spring attached to the post placed on the table, we force without changing the later irrespective of whether that
'spring rests or moves. Any body whose mass is negligibly
consider the interaction of three bodies: the weight, the
small possesses this property; for instance, in our discussion
spring, and the Earth (as has been said, the Earth together
we tacitly imply that the threads connecting the bodies in
with the table and the post form one body). The forces ,question are 11 massless 11, 11 inertia-free" and possess the
1

taking part in this interaction are shown. The earth acts on indicated property. That is why when speaking or a tension of
the weight with the force Fwe (the force of gravity of the a spring or of a thread we mean the magnitude of the
weight) and on the spring with the force F,e (the force of ,stretching force which is considered the same for both ends of
gravity acting on the spring). The weight acts on the spring the spring or of the thread.
with the force F,w and the post (considered as one body
The force Fwe with which the Earth acts on the weight
together with the Earth and the table) acts on the spring
(the force of gravity of the weight) is no longer equal to the
with the force F,p. According to the third law, we always force Fw, with which the spring acts on the weight. The
have the equalities difference between these forces determines the acceleration
Fwe +Few= 0, F,w +Fw, = 0 and Fsp +Fp, = 0 of the weight. It should be noted that if Fws > Fwe at a certain
Assuming that the magnitude of the mass of the spring is time this does not necessarily mean that the weight moves
negligibly small (and only under this assumption) we can upwards; this only implies that the acceleration of the
write, on the basis of the second law, weight is directed upwards. The force of the spring Fws and
F,p +F,w = 0 the force of gravity Fwe are not equal to each other
Condition shows that the force of tension of the (according to the second law). It is the difference between
these forces that produces the acceleration of the weight.
("massless", i.e., 11inertia-free11 ) spring is in all the
circumstances the same at both ends of the spring. In this When the weight and the spring are at rest their
accelerations are equal to zero; aw = a, = 0. Then the force
approximation the magnitudes of the forces acting on the
Fw, with which the spring acts on the weight is equal in its
ends of the magnitudes are equal to those of the forces
magnitude to the force of gravity Fwe and, by the third law,
acting on the ends of the spring are regarded as being to the force Fsw with which the weight stretches the spring;
precisely equal. Further, by the third law, these magnitudes in the state of rest the force Fsw coincides with the force of
are equal to those of the fores Fp, and Fws with which the gravity of the weight. Thus, in the state of rest the absolute
spring acts upon the bodies stretching it. values of the three different forces Fwe (the force of gravity
of the weight), Fw, (the force of tension of the spring) and
www.puucho.com
Anurag Mishra Mechanics 1 with www.puucho.com

Fl FORCE ANALYSIS
------ -- -- ---
F,w (the force with which the weight stretches the spring) Nonna!
are the same. The force of gravity of the weight and the force component
Fsw are simply equal to each other: N ---·-

Fwe = Fsw
: Horizontal
Force of the string
on the wall
Force of the
string on girl
. - . ----- : component
Ffric.
Fig. 2.10

push is the normal force of the table upon your hand. The
The system The system~ component of force parallel to surface is friction, discussed
is the wall is girl later in this chapter.
'' + Normal force in perpendicular to the contact surface
as shown in Fig. 2.11
(a)

Force of the wall The system Force of your friend

:n thi strtn~- -----.--_J_ _------~~-t_h_·-~rg 8


.'!\: _- - - - - - - - - - - . - - - - . - - - .• '
(b)

Fig. 2.8
A A
Ideal String
An ideal string is considered to be massless (negligible
mass), inextensible (does not stretch when pulled), pulls at
any point in a direction along the line of the string, can pull
but not push. The force with which one element of the string 8
B l!---+---+-+Ns ----"t---t--+-+Ns
pulls on its neighbouring element is called tension in the
string.
A girl pulls a string tied to a wall. The string will exert a A A
force on the girl in a direction opposite to the force the girl Fig. 2.11
exerts on the string. The string exerts a force on the wall in a + If direction of contact force cannot be determined, it
direction opposite to the force exerted by the wall on the should be shown as two components (Fig. 2.12).
string (Fig. 2.8).

~~
Ideal Pulley
An ideal pulley is assumed to
be massless, frictionless. Action of
the pulley is to change the 7B;•,Tlcieal pulley --~
direction of force. The ideal pulley .
does not change the magnitude of Fig. 2.12
tension in the rope. Tension is
same in the string on both sides of Fig. 2.9
+ When contact between two bodies breaks, the
the pulley. If there is no stretch in
normal reaction vanishes.
the string, the speed at which rope comes onto the pulley is
equal to the speed at which it leaves the pulley (Fig. 2.9).
+ The weighing equipments measure the normal
reaction.
Contact Force + Normal force is a variable force; it can very in
Whenever two surfaces are in contact they exert forces magnitude as well as direction. In Fig. 2.13, normal
on each other. Such forces are called contact forces. We reaction passes through centre of gravity of body in
resolve these contact forces into components, one parallel to the absence of any external force. Line of action of
the contact surface, the other perpendicular to that surface normal reaction shifts to the right when an external
Fig. 2.10 shows contact force on finger by a tabletop as it force is applied, as shown in the Fig. 2.14. At the
slides on it. The component of force perpendicular to the instant the body is about to overturn, it passes
surface is called normal reaction. The force resisting your through the edge of the body about which
www.puucho.com
Anurag Mishra Mechanics 1 with www.puucho.com
'

L..f1~44_,~'--'_·,_·'_~'------------~"-~'--'~~---~----ME_C_H~
overturning talces place. For a block kept on an earth to be separate systems, the weight -is, external
incline, N = mg cos0. If angle of incline is gradually force on both the .bodies.
. · increased, the normal !eaction decreases. + Internal forces always act in pairs.
+ Vector sum of all the internal forces on a system is
;:c '~.~---,~ ,,'
zero .
. :·1 .N 7
L, Fiilternal
System
=0
Problem Solving Tactics
By Applying Newton's Second Law
mg mg: L Identify the object you are considering; make a simple
, (a) (c) sketch ofthe object.
Fig, 2.13,, 2. Draw arrows on your sketch ·to show the direction of
r--
.., ' each force acting on the object. Arrows are drawn to
represent direction of forces acting on' the body. This

. ·_ _ m!!
diagram is called direction of forces acting on the body.
This diagram is called a free body diagram. Only

L 0
external forces (forces exerted by the other bodies)
acting on a body are shown in the free·body diagram..
_· Fig. 2.14 .·,··:,-·

Concept of External and Internal Force


- ,I
:<,_YL
<., ; X ·, '
'c;E'NA A -
Consider a boy pulling two toy cars A and B connected
,.. p , f '
through a string. In Fig. 2.15 (a) our system includes A and - . A " ' ,mAg
B; the· pull of the boy comes from outside the system, A" ' ' ~' w'> \ C '

. Wsyre~·-.-.
-'., . >.~~\::!.~¼~.' ,. Pull_

-c~i"'~-_-:•.'!;.:.--:, . .
'",

,
' .

''.Ns} .. _-
, . msg
B
. ,,
~-

.. [

/
-1,

.. ,
"•'
,. '. (a) (b) Groun!l,.-"..,.,- . - - -
•,,, ,,
.•0 -,..
• _F_i9~·_'2_.1_s~--------~
. r -· :_:- '

' ' . (b) (c)


.fig.
:,
2.11 ·

In the Fig. 2.17 (a), force Pacts on block A; it.must


be shown only on A. Block A presses the body B
with certain force, which is represented by a normal
internal force. Note that this tension is paired and acts on reaction NA , which acts on both the bodies. Neither
both the toy car as well as B. In Fig. 2.15 (b), the pull of the weight of A nor force P should be shoWn on B.
string on the toy car B is external force, because string is not Whatever force A exerts on B is communicated
part of the system. through normal reaction. Similarly body B presses
Now .. consider. a ball projected upon the .surface of the ground with normal reaction NB downwards. ·
earth. If we include the ball and earth
in our system,
then weight is internal force. If we consider ball and·

www.puucho.com
Anurag Mishra Mechanics 1 with www.puucho.com

C
FORCE ANALYSIS 145·
---·- - --··. -- ··---
3. Assign a coordinate system to your free body The force diagram ,, vL
diagram. Coordinate axis is assigned according to for the lower section of :~.,, __/~
convenience in resolving forces and accelerations
into components. For exampk in Fig. 2.17 (a), x
the rope is shown in the
figure. The section is r
and y axes point in horizontal and vertical direction
respectively. In Fig. 2.17 (b) it is along incline and
normal to incline. For a particle moving along
pulled up by a force of
magnitude T(x), where
' T(x) is the tension of x.
1 L
T(x)

curved path tangential and normal axes are


assigned as shown in Fig. 2.17 (c):
4. Resolve all the forces acting on a body into its x aod
The downward force on
the rope is its weight
W = Mg(x/L). The total
rJ 1
j_
X
y- components. force on the section is
5. Apply Newton's law in component form as zero since it is at rest. j
LFx == max, LF'y == may, Hence
.EFz == maz w
6. Solve the set of equations for any unknowns. T(x) = Mg x Fig. 2.20
L
'y
At the bottom of the rope the tension is zero, while at
l:F v... the top the tension equals the total weight of the rope Mg.

:_dL-= o·-·;:r;-x _Reaction Force on a Pulley


When a pulley is used to change the direction of a rope
under tension, there is a reaction force on the pulley. The
Fig. 2.18 force on the pulley depends on the tension and the angle
through which the rope is deflected.
Pulley System A string with constant tension T .is deflected through
A pulley system allows you to lifr an object while angle 28 0 by a smooth fixed pulley. What is the force on the
exerting a much smaller force in a more convenient pulley?
direction and with greatly improved control over the object's
motion.
In a single pulley system (Fig. 2.19), the rope exerts
equal tension force at its two __,ends. __,At one end, tension
balances the force you exert: T = - F. At the other end,

T
Block Element of ......
rape at hand . /18/2
(a)
(b) Fre"i9:-body diagrams,'
Object being lifted with
the aid of a single pulley.
Fig. 2.19

-+ -+ ---+ -+
tension balaoces the objects, weight, T = - W. Thus, F = W.
Fig. 2.21
The single pulley is useful because it allows you to pull
downward rather than upward, but it doesn't reduce the
necessary force. Consider the section of string between 0 and 0 + t.0. The
Tension in a Hanging Rope force diagram is drawn below, center. t.F is the outward
force due to the pulley.
A uniform rope of mass M and length L hangs from the
limb of a tree. Find the tension at a distance x from the The tension in the string is constant, but the force T at
either end of the element are not parallel. Since we shall
bottom. www.puucho.com
Anurag Mishra Mechanics 1 with www.puucho.com

. MECHANICS-I

shortly take the limit t.e -, 0, we can treat the element like a 2F - (M + m)g = (M + m)a
particle .. Fo.r equilibrium, the total force is zero. We have (i) Minimum force is required when box moves
with constant velocity, i.e.,
Af/ - 2T sin t.e = 0 For small t.e, sin(t.e/2) = t.e/2
2 a= 6, thus
and Af/ = 2T t.e = Tt.e (M+m)g
2 Fmin = 2
Thus the element exerts an inward radial force of (ii) If F > FmJn, then acceleration of the system is
magnitude T t.e on the pulley. 2F
a=---g
The element at angle e M+m
exerts a force in the x (iii) For calculation of normal reaction will have to
direction of (T t.e) case. The Tt.0 consider FBD of man. Considering the free body diagram of
total force in the x direction is the man, we have from Newton's Second Law,
LT case t.e, where the sum is .... ~- .... F+N-Mg =Ma
over all elements of the string
which are touching the pulley. or F +N - mg= m[_l!__-g]
M+m
In the limit t.e -, 0, the sum ---~ig.2.2_2_ __
becomes an integral. The ' -- - or N=[M-m]F
M+m
· total force in the x direction is therefore

J-•o•o
Tcosede = 2Tsine 0 • b~§~~P{!?:
1· ..... ... .. .
,12-le>
. ... . .. .. ·- ..
[~~2ci'iR!~~.J> ,A heavy block of mass M hangs in equilibrium at the end of a
:rope of mass m and length l connected to a ceiling. Determine
;A ma~ of mass.M stands on a.box of mass mas ; 1tlie.. temiq_n in_ the rope qt aAisJance xfrom the _ceiljng. ....
1shown in the Fig. 2E. l (a). A rope attached to Solution : Procedure: When a rope has mass, due to
'the box and passing over an overhead pulley : force of gravitation it tension in it will vary, separate the part
,allows the man to raise himself and the box by , of string and block on which tension is required :
!pulling the rope downward. ' . -- . - - - - ' ~-- ~, -- )

°(i) With what minimum for¢e should the '


man pull the rope so as to prevent himself ,
fromfalling down. · X
(ii) If the man pulls the rope with a force F : '---'"'--""' m
greater than the minimum force, then : Fig. 2E.1 (a)
e
determine the acceleration of the ·- -· --- -· ·
. (man + box) system. .. \ T
'.(iii) Determine. the normal reaction between the man and the I
.. trqlley. . . . .. ·- .... • .. .
Solution: Procedure: Draw free body diagram of
box and man apply Newton's second law separately to them.
l~ (e-x)g

Let the whole system moves upward with an acceleration a.


Applying Newton's Second law,
. f'
·
'j, ..
lMg
Free body diagram of the block anii rope of lengtt, (f- x)!

,j \ · / m
T--(l-x)g -Mg= 0
Fig. 2E.2 (a)

Using the condition of equilibrium,


:EFy =0
'

(M + rn)g
or T=Mg+mg - -.
1 (/- X)
(b) (c)

www.puucho.com
Anurag Mishra Mechanics 1 with www.puucho.com

r FoRce ANALvs,s
I-·----- - - - •-----
- - -- --
--- - - - - - · - - • - - - - -

T
Therefore ITx = - Ta cos 8 + Tb cos <j> = 0 ... (1)
T
ITY = Ta sin 8 + Tb sin <j> - mg = 0 ... (2)
_Tacos8
From eqn. ()1 , Tb - ~ - -
Mg 1 - - - - - - ~ cos <I>
On substituting Tb in eqn. (2), we get
Ta cos 8 sin <j>
Ta sin 8 + - - - - - - - mg = 0
0 x•l X 0 I X cos <I>
Tension is constant along mg
Variation of tension the length of a massless string. or Tb = - - - - - - -
T as a function of x. sin 8 + cos8 tan8

---
(c)
(b) ,----

Fig. 2E.2 4 -
-- -
Tension in the rope is minimum at the bottom, at I = x Fig. 2E.4 (a) _shows a block of mass m1 sliding on a block of
i.e., T = Mg, mass m 2 , with m 1 > m 2 • Find
(a) the acceleration of eadz block;
and the tension is maximum at the ceiling, at x = 0
(b) tension in the string;
i.e., T = (M + m)g (c) force exerted by m1 011 m2 ;
Let us consider an idealized case of massless suing (d) force exerted by m 2 on the incline.
When the weight of the string is already small compared
with the other force involved, we consider the suing to be \;:\
light.
For a light suing, tension is constant throughout its
length.
T = constant O :S: x :S: 1
If the block would have been suspended from a light Fig. 2E.4 (a)
string, then the tension would be
Solution : Fig. 2E.4 (b) shows free body diagram of
T = Mg, constant everywhere. each block. We will apply Newton's second law along x- and
y-axis shown in free body diagram. Block m1 is heavy, hence
it slides down whereas m2 slides up.
A bucket is suspended by two light ropes a and b as shown in
Fig. 2E.3 (a} Determine the tensions in the ropes a and b.
- -
y

Fig. 2E.4 (b)

mg
Block 1:
(a) (b) ITx = m1g sin 8 - T = m1 a ... (1)
Fig. 2E.3 ITY = N 1 - m1g cos 8 = 0 ... (2)
Block 2 :
Solution: Light rope implies that weight of rope is ITx = T - m 2 g sin 8 = m 2 a ... (3)
negligible as compared to the force it exerts.
ITY =N 2 -N 1 -m 2 g cos8= 0 . .. (4)
Since the bucket is at rest, its acceleration is zero. Thus
From eqns. (1) and (3),
Newton's second law gives
. m1g sin 8 - m 2 g sin 8
ITx = 0 and ITY = 0 a= --'-"-------"'CC---
m1 + mz
And T = m 2 a + m 2 g sin 8
www.puucho.com
,"~
Anurag Mishra Mechanics 1 with www.puucho.com
,+,.
\.
i
I

= m2(m1g sin. e - m2g sin 8) + m 2g sm


.
8
From eqns. (1) and (3),
m 3 a= m 2 g
I m1 + m2
2m1 m 2g sin 8 or a= (m 2 /m 3 )g
' =
m1 + m2 From eqn. {4),
I,' From eqns. (2) and (4), F=T+M 1 a+N2
'
= mzg + m1 x (m2) g + m2 (. m2) g
. m3 m3
, m
= (m1 + m2 + m3) -2 g
m,
Constrained Motion
In unconstrained motion the moving body follows a
path determined by its initial motion and by the forces
which are applied to it from external sources .

.'

In constrained motion, the moving body is restricted to a,


specific path i.e. the path of the •body is governed by the·
restraining guides e.g. a train moving ,tlong its track; a ball
tied to end of string and whirled in a circle a lead gliding on
'' . '
a fixed wire frame.
Equations of block m3 : Kinematic Constraints: Kinematic constraints an
T=m 3 a ... (1) equations that relate the motion of two or more· bodies. B)
iv~= m 3 g ... (2) differentiating the kinematic constraints for the position 01
Equations of block m2 : the particle in a system, the corresponding kinematic
T=m 2g ... (3) constraints among the velocities and accelerations of th,
... (4) particles may be obtained. ·
N 2 = m2 a
Equations of block m1 : . lllustration-1
F-T=m 1 a-N 2 ... (5) In the figure shown the masses are
N 1 =N 3 +m 2g+T ... (6) attached to the inextensible string. At
Remark:------------------- any instant, let the positions of m 1 and
If m 3 has to be at rest relative to m1, they must have m 2 be x 1 and x 2 respectively as showri
same acceleration. in the Fig. 2.24.
-> -> ->
am:,m, = am, - am, = O then, x 1 + x 2 + 1tR = l .>\;
'

_, -> (length of the string) = constant


an73 = elm, m,!i ~
Differentiating with respect to "<::_,,;_"<',< ·.---.~'.-:·j·.. )
!

tirile, we 'get- ~ -"..c/.!!11::'.:·24·_, :,,


dx1 + dx, = 0
dt dt
.I v 1 +v 2 =0 or v 1.=-v 2 ... (i)
I. Again differentiating w.r.t. time, we get
... (ii)

I www.puucho.com
.-,~I
Anurag Mishra Mechanics 1 with www.puucho.com

[!:ORCE ANALYSIS -
. - r-1-.
---- - ----- -- ___,.1___
1491
_,_
The equation (i) and (ii) are constraint relations for
velocity and acceleration. Negative sign denotes that their L ~~~!'.DP}":::!~_>-
directions are opposite to each other.
llfustration-2 A block of mass m1 on a smooth, horizontal· swface is:
connected to a second mass m 2 by a light cord ovbr a light,:
In the Fig. 2.25 the blocks 'A' and 'B' are connected with frictionless pulley as shown. (Neglect the mass of the cord and'
an inextensible string. The block 'A' can slide on a smooth
of the pulley). A force of magnitude F0 is applied to mass m1 ·
horizontal surface.

X9
VB

IB
J_7 as shown. Neglect any friction. '

'
{
h {
Fig. 2E.6 (a) /

(a) Find the value of force F0 for which the sy!tem will be in
a
equilibrium. '
Fig. 2.25 (b) Find the acceleration of masses andtensio~ in string if F0 :
has a value which is double of thatfoun,d in p_art (a).
Since the thread is inextensible, its length remains
constant i.e. Solution : (a) F0 = zr = 2m 2g
)xi+ h 2 + xB = constant i.e.,

Differentiation w.r.t. time, we get, (b) Concept: Movable pulley is massless therefore
XA dxA + dxB =0 forces on either side of it must be equal.
)xi +h2 dt dt r--r-:,.,___
T'~T T-Zf'= Oxa
As the ball moves, xA increases and xB decrease with
although pulley is accelerated _
time.
Therefore
dxA
--=VA--=-Vs
dxB For
dt dt 2T
XA F'o
and --;===== = cosa
)xi+h2
T=m2g
hence v 8 = v A cosa.
Concept: If blocks are connected by an extensible string,
,component of velocity along the length of the thread of the
any two point of the thread must be same, otherwise either m,g
Fig. 2E.6 (b)
length of the thread will increase or thread will get slack.
For m 2 : T - m2g = m2 (2a) ... (1)
Component of F0 - Zf = m1a
velocity perpendicular
v, For m 1 :
e, 4m 2 g - 2T = m1 a ... (2)
to the length of the
thread changes the Solving eqns. (1) and (2), gives
angle of the thread. T = m 2 g[m 1 + 8m 2 ]
m1 +4m 2
If the thread is
attached to a sliding 2T
constrained body then
at the point of
attachment of the Fig._2.26 T
thread, component of
velocity of the body along the length of the thread is equal to
the component of velocity of every point of the thread along
its length.
v 1 cos8 1 = vb sin8 2 Fig. 2E.6 (c)
1· 1 sin 01 changes the angle of the part 'AC '?f the thread
and vb -:.,0 2 changes angle of the part 'BC' of the thread. www.puucho.com
Anurag Mishra Mechanics 1 with www.puucho.com

\
\
ITTo- --\ ---- -- .. .

~
am, = 2a = 4m24g
m1 + m2
2
ace ~ration of m1 =
m 2g
i m1 +4m 2
Fig. 2E.B (a)
4
acceleration of m 2 = mzg
\ m1 +4m 2 Solution: ·-,------------~--- '-----!
'. . . _m___,"'g.:.[m-'1--_+_B_m~2""]
Tens10n m strmg = 2 Concept: If a.body slides on another accelerated .mrjacej
1
· m 1 +4m 2 ;.app ly equatwn
.
S rel = u rel .+ -I arel t i I
-- ....., . . -~·-1
I. Exci.m::JP. l.e
L:T- .- .•..
I
I 7 · __.. . ~--
2
-~reJ_::= Oi are1_=:__~_- °:.
·
I
·--'
7 S9S 3_7_0 =-: N__ __
A sm~;l ~:b~ca\~l~ck is p~ac:n a triangular block M.so thati ma
,they touch each other along a smooth inclined contact plane: N

'as shown. ThJ, inclined surface makes an angle 0 With the I


horizontal. A hqrizontalforce Fis to be applied o.n the block mi
so that the two\ bodies move without slipping against each
other. Assuming the floor to be. smooth al.so, determine the ,,.l·
m(g- a) cos 37° mg
37°
ill~~// .... Fig. 2E.. 8 (b) __
Fig. 2E.7 (a)
4
or N=7x- =5.6N; lxarel =7sin37°
.(a) normal force with which m and M press against each' 5
other and 3
or arel = 7 X - = 4.2
(b) the magnitude of external force F. Express your answers; 5
in terms of m, M, a and g. _i 1
2.l=-X2.1Xt
2
2
Solution:
or t=lsec.
Concept: When' there is no sliding at any contact
lsu,jace we may take c~mplete system as a single body.
Considering motion of the system A particle of mass 10 kg is acted upon by a force F along the I
1line of motion which varies as shown in the figure. The initial'
~F---,a ;velocity of the particle is 1oms·1. Find the maximµm velocity'
£mg ;attained.by the particle before it comes to instantaneous rest.
FBDofm
.,. =-·-------- ---,
: F(N) • :
Fig. 2E.7 (b)
F = (M +m)a ... (1)
,, 20---
From FBDofm
N cos0 = mg ... (2) :' (0, 0)1---l--- - - -...
, t (sec) I
10
and F -N sin0 ~ ma ... (3) '15N--1---'-----
From eqn. (2) N = mg/ cose
Solving eqns. (1), (2) and (3), we get Fig. 2E.9
F = mg (m+M)tane
M Solution : . F = 20 (0 ~ t ~ 10)
:---1 a=F/m=2m/s
~~~gmplg ! a : ._.-__,.
f
=======->\._____ ~ Max. velocity will be attained at t = 10sec. because
after that force stan acting in opposite direction
.A block of mass l kg is kept on the tilted floor ofa lift moving; dv =2
jdown wfrh 3 m/s 2• If the block is released from rest as shown, dt
.what will be the time taken by block to reach the bottom. V 10
'¼'hat is the normal reaction on the block during the motion? ; or fdv=f2dt
- ·' - -. - - . - . .. .'
10 0
www.puucho.com v = 30m/s
Anurag Mishra Mechanics 1 with www.puucho.com

r-Foiice ANALvs1s
L - . - -
- - - 151'
___lj

!J~~.Gtnf:>!·~ r: 10~'->
A homogeneo!'S and flexible chain rests on a wedge whose side T
·edges make-angle a and p with the horizontal [refer Fig. 2E., Tsin a Nsina

'="·A
,lO(a}]. The cehtrarparcuf the chain lies on the upper tip o/
the wedge. With what acceleration should the wedge be pulled
,to the left along the horizontal plane in order to prevent the,
·~
P1 cosa ..o
displacement of the chain with respect to the wedge? P, .....B..+ J',s,0
[Consider all surfaces to be smooth] 'I-

Fig. 2E.11

Solution:
, Concept: Draw neat and clean FBD of fixed wedge and
. I Fig. 2E.10 (a) •blocks. Let reaction at comer on wedge is R.

Solution: Equation of wedge:


R+Tcosa=Nsina ... (1)
Concept: Consider the parts of chain on either size of. R =Nsina-Tcosa ... (2)
incline as two different element, draw FBD. Apply Newton's Equations of blocks :
law or these parts separately. N =P1 cosa . .. (3)
P2
T-P2 =-a ... (4)
g
. P1
- T + P1 s1na=----:-a ... (5)
g
mg/2
(J, -1)+(~ -sina)=o
Fig. 2E.10 (b)
T = p1p2 (l + sin a)
Taking comp. along incline P1 +P2
mg sina-T = m acosa R =Psinacosa- PiP2 (l+sina)cosa
2 2 P1 +P2
T- mg sinp = m acosP = Pi cosa[(P1 +P2)sina-P2 -P2 sina]
2 2 P1 +Pz
g[sin a - sin Pl R = P1 cosa(P1 sin a -P2 )
on, solving we get a=~---~
cosp + cosa P1 +P2

L~~~a~~~!~".f12 [>
A body A weighing P1 descends down inclined plane D fixed of The pull P is just sufficient to keep the 14 N block in,
·a wedge which makes an angle a with the horizontal, and, equilibrium as shown. Pulleys are ideal. Find the tension (in
'pulls a load B that weights P2 by means of a weightless and' .N) in the cable connected with ceiling.
inextensible thread passing over a fixed smooth pulley C, as: Upper
,shown in Fig. 2E.ll. Determine the horizontal _component of, cable
:the force (in Newton) which the wedge acts on thef/.oor comer
E. ,

p '
Fig. 2E.12 (a)

Solution: T1 =P
T2 = 2T1 = 2P
www.puucho.com T3 = 2T2 = 4P
Anurag Mishra Mechanics 1 with www.puucho.com

MECHANICS-I j
Upper
cable
Tw~bl~1k.A ~;d·B h~ving ·;;:.;;;s-;~-~1-kg, ~: =4kg a~ei
arr.anged as shown in thefigµte. The pulleys P imdQare light,
~and frictionless. All the blocks are resting on. a horizontal\
T, 'floor and the pulleys ate held such that strings remains just!
' • t, '
taut.
p . At.momentt = OaforceF = 30t (N)starts acting on thei
Fig. 2E.12 (b)
ipulley p along .vertically upward direction as sho,.;m ii'). thei
[figure. Calculate. · · · j
For equilibrium of block '(i). the time when the blocks A and B loose contact with!
T1 +T2 +T3 =14 I ground. , '
= 7P = 14 (ii) the. velocity of A when B looses contact with ground.

..
=
--: .. - . . ~7-.
P~2K
1
(iii) the height raises]. by A upto.this instant. ·

~"-~~~~R!c~ ·J 13 v· (iv) the work done by the force F upto this instan(.
1 r"
'
-------------- ·---·-
F -30t(N)

,For the equilibrium situation shown, the cords. are strong,


enough to withstand a maximum tension 100 N. What•i$ the 1 i
largest value ofW (in NJ that J/)!lY can support as slwwn. ?.
. ' =---- ·, t
}, .
53°
!
I
j
I Flg.2E.14(a) i ,
~.:.~-"'-,<'--"""''",· .,...~----·'- - _, ·--' --- - !
Fig. 2E.13 (a) __.,...:. • _, .. <
- ·· Solution:
- ,,w

Solution:
- - " ' .. \<.-· ,,-,;~ ... rft, :.-·~- ·- ?_....-.i~J~~ ~~-½'·I
1,--~~~ll~;;;:- Mien bldcks loose--contact w:;t;;-;~;;~
Concept: Consider the Mint at the function ofstrlngs; _normal rea':~?~_'?..n _t_1!_em bec;ames ze!:.o:._., ___ .···- . . _J
'as string element at function is massless and in e'lµilibrium,I (i) When A looses contact T = 10 N • • - · · "301 = F ,
sum of forces in x · and :l .direction . must .be ,-e9uaL , F= 3T
:EF,.. = 0 ' - , ..., ......... , .- -. 1
I " .... -" ---. - ___ ,,.,_" .,,--"'. or 30t = 30
1~0N : . ··:YL:·: t = 1 sec
p

When B looses contact


,.
.
·~ .·
53° :'' x,;
2T = 40
. . .~.~:-·; i or T=20, F=3T
T T T

w
..
.'
I or 30t = 60
or t=2sec
.... Fig. 2E.13
_,.;:;;-
(b)
---- ------·
(ii) T-10 = a
Tsin53°-100cos53°= o
and 3T = F = 30 t
or T = lO0cot 53°= 300/4 = 75 N
For getting velocity we have to use
:EFy =0 calculus because acceleration is variable
100sin53°-W-Tcos53°= 0 V 2
W = 100sin53°-Tcos53° V = Jdv = 10J(t - 1) dt = 5 m/s
O I
= 400 _75x~
5 5 (iii) J· dx = J10[.c-r +~]dr
= 80-45 I 2 2
=35N
= X= 5/3m
(iv) W = fFdx
www.puucho.com
Anurag Mishra Mechanics 1 with www.puucho.com

--
' FORCE ANALYSIS
2
= J30txl0
J
[t 2
--t+- dt=-J.
2 2 6
l] 175 · i-'=X~9Dl';Pl_!2- [ul;>,
In the figure shown, friction force between the bead and the
15 ,_.-
' ·- light string is mg. Find the time in which the bead loose
4
In the given figure find the velocity and acceleration of B, if contact with the string after the system is released from rest.
instantaneous velocity and acceleration of A are as shown in
the Fig. 2E.15 ( a)


Im
I
I
Fig. 2E.16 (a)

Fig. 2E-15 (a) Solution:


Concept: Only interaction force between string and
Solution: bead is friction. Tension in string is due to friction.
Concept: We use the fact that string is inextensible and
Tension in the string, T=f=mg
length of string is constant. 4
11 + 12 + 13 + 14 = constant_ mg
mg--
..
differentiate to get -
d1 l d2l d l3 d l 4
+- +- +_ =o Acceleration of the block, a2 = 2
m
dt dt dt dt
vA +vA +(-vc)+(-vc)= 0 a2 = II_ .1. [downward] mg
2
4
Now acceleration of string, a, = 2a 2
a, = g 1' [upward]

--
vA=-1m/s
[where bead is placed]
Thus equation of bead
mg
mg
Fig. 2E.16 (b)
aA= -2m/s 2 mg--
3
ab = - - ~4- = g .1. [downward]
f" m 4
Relative acceleration of bead with respect to string
. 3g
=-+g
Fig, 2E.15 (b) 4
or V C = VA => V C = l m/s 1' Now, a,,1 = ab, =
7
!
where Ve is velocity of pulley C _ 1 1
Similarly ac = a A
Now, apply eqn. Sret = -a,,
2
2
1t ; l = -ab,t
2
2

2m/s ~ 21
2
ac = .1. or t = = /]f _
Now, 14 + Is = 16 ab, 1J7g
dl dis d!6 -~-
-4+ - = -
dt dt dt L,S-~fil-t,TIP J,!=!-_ I 17 1.;:>
-Ve+ (va) = -va In Fig. 2E.17 (a) shown, both blocks are released from rest.
Ve Length of 4 kg block is 2 m and of 1 kg is 4 m. Find the time
Va =2
they take to cross each other? Assume pulley to be light and'
va = O.Sm/sl string to be light and inelastic.
. Similarly, :. aa = lm/s 2 .1.

www.puucho.com
Anurag Mishra Mechanics 1 with www.puucho.com

154 MECHANICS-I j
~---------- ---- ··· 1 -=c..=-= =-=--1-- ·-- -- -- --- -·1
!2 ,11
! mjQ0!4m _,
_, _,
v1/,+v~,
or, v p/g =
--- --··- ---·-
Solution :
L 1kg
ig. 2E.17 (a)
•-,.,----_.,..- --=-~----....,__ ____ --···- -- Lltxamr.:.l·e
. - -r,;_=~1--~_':---..
its I ~
2

r--- -- - ----- --- C


¼bead c~n ;;,ove fre~ly ~n a h_o_nz_·-o-n-tal ro~Th;bead ~
I.connected by blocks B and D by a string as shown in l:he/

r
ifigure. If the velocity of B is v. Find the velocity of block D. /
T
I
I

i. :9
T .a, I
!•l 2m! -·
a
t
I
I
4g
4~g
4m
I' A
1g '
! Solution:
I Fig. 2E.17 (b)
L_ - - - - - · - - - - - - - - - - - - - - - - · - · - ----- -
.)
I
. [---~o.:~e_p_t_:_a_lo_n_g_thelength of s;ing ~~ocity component/
From FBD of blocks A and B solve acceleration of each _is sa'!!:.f!'!..!11!..!1!:.J!Oi!_1-ts_onstrl11g. ~-- __ A
block
4g-T=4a
T-lg =lxa
3g
After solving eqns. (1) and (2), a=-
... (1)
... (2)

5
.I ~. l
.I
, Ve
,. ,. ...-o
I.
~
Vp

tD
i!0'······.....'•,,
~.,.
lI
acceleration of A w.r.t. B 6g
aA/B = - = 12m s2 I I
I
t"" Ve COS 37°
5 l I
! (b) (c) I
If A will cross B then distance travelled by A w.r. t. B is
6m ·i
'---
Fig. 2E.18
--- -------- -- --·-----
____ _J
6= o+I.xl2xt 2
, t = 1sec = Ve COS 53°
VB ... (1)
2 .
Ve cos37°= v 0 ... (2)
PULLEY CONSTRAINT from eqns. (1) and (2) we get
· lllustration-3 · vB cos37° VB(4/5)
Vv =
In the Fig. 2.27 shown pulley moves 1 -- - cos53° (3/5)
v
with acceleration P. Let acceleration of 4
VD =-VB
_, 3
blocks m 1 and m 2 w.r.t. ground are v 1 i _,
and ! v,f i;:g~me1~ai::>
i m,
I m2
¼-ii{; goes ~~-with lOmj~.A-pulley P ~-fa<;d~~ th;·;;m,;;J
Jthe lift. To this pulley other two pulley P1 'and P2 are attach~i:l.
Il. _ Fig. 2.27 __ _
·p1 moves up wi-th velocity 30m/s. A moves up with velocity
--------- ------ .. ------ - ---- --------- ----- ..I
Concept: According to string constraint for.an observer
l10 m/s. D is moving downwards with velocity 10 m/s. at same
!instant of time. Find the velocity of B and that of C at that
on pulley the length of string that approaches pulley must bel ~t~_n!:_~_s~:1!_~ that all :_'~l'!..C!_t!es are relati~e to the gr<_J'!_nd.
released way form the other end of pulley, Le., relative to!
pulley velocity of both the blocks should be equal in\ '
magnitude but opposite_ in_ direction. _____ _ _ _____________!

www.puucho.com
Anurag Mishra Mechanics 1 with www.puucho.com

I FORCE /\NALVSIS 1551


r---- . .i
-1 i
[
a,
I
II
I

I
2

a,
.,:re'
3
.
s\fi-~.-
'2:.-···
a, hl
&a 2 cosa

\
\
\
I
I
'
a1 case\ ·
a1
I
I
!

I 'imrrln1TT1rmr!rn'11111TT1TTT \ a, sine :
~ - - - - - - - j _____
F1_g.2,_e_.1_9_ _...,__ _ __

Solution: Apply constraint on pulley P


_J
'----------------------
Fig. 2.29
J
If wedge (1) and (2) are to remain in contact
-> ->
V P1 /P = -V P2/P component of acceleration perpendicular to contact surface
must be same.
--+ --+ -t -t
Vi,-Vp =-(Vp -VP)
2 a1 sin"B = a 2 cos9
_,
v Pi , v
_, _,
P, v P
2
are respective velocity w.r. t. ground, = tan9 a,
a1
_, _, _,
· V
P2
= 2Vp-V "1 ·mustration-5
= 2[10Jl - [30.fl In the Fig. 2.30 shown plank 1 and wedge 2 are free to
= -lOj more obtain relation between their acceleration procedure is
Now apply constraint eqn. on pulley P2 similar to that of previous illustration.
--+ --+ --+ --+ a1 = a 2 sin El
Vc-v,,, =-(VP-VJ>,)
_, _, __,
v c = 2v,,,-vD

__,
Vc
= 2[-10JJ-[-10J1

= -10j
'
=tJll- ~~Q:,=I
Apply constraint eqn. on pulley P1 to get
--+ --+ --+ --+
M 7[9ne I
VA-Vi, =-(v.-vi,)
__,
Va =2Vi,-VA
_, __, ~cline i
---- _,
Fig. 2.30
= 2[30jJ - [10J1 = soj ~-------- 1

Normal Constraint lllustration-6


Consider two blocks moving on a Fig 2.31 shows three identical cylinders, cylinders are

~;~--,7
released, find relation between accelerations of cylinders.
surface and always remaining in
contact. In order to maintain contact
component of velocity vector I,~e;J~C
I,;:
perpendicular to contact surface must
be same.
i.e.,
__, __,
V1 =V2
L Conta._ct ~u,rface
_!:!g. 2.28

sinlilarly
_,
a1
_,
= a2
Frontview ,....
60° •.

82COS 60°
lllustration-4 , ___ Fig. 2.31 _J
In the Fig. 2.29 shown find relation between - ------ ------ ---
Constraint equation relates component of acceleration
acceleration of wedge 1 and 2
perpendicular to contact surface shown in figure.
a1 =cos 30° = a 2 cos 60°
a1 1
a, = ..J3

www.puucho.com
Anurag Mishra Mechanics 1 with www.puucho.com

r1ss - - ·- MECHAN!cs:1 -I
L-- ----- - --------- ' - - - - - - - - - - · - - _,;__j

k~$~~~J~,T-wl>
' --·- --- -·- ,-, --- ---- --- - ··---,-,
1n the situation given, all surfaces arefrictionless, pulley is/
;ideal and string is light. If F = Mg/2,find the acceleration ofl
,both the blocks in vector fonn. ,
F I,
I
· ~ r':
. :~ y l Fig. 2E,21 (b)

Now write constraint equation for pulley to get


Fig. 2E.20 (a) I -+ -+ ~ -+
vn-Vp = -(vc-Vp)
Solution: First consider both
the blocks as system force that we or,
apply at one end of string is tension
'.~~Lsysterii
---~=Mg~,] --> -->
also Vp =-VA
in the string. · B A I
I --> •
For system block (A + B) l , I Thus we get vA = (37. Sj) m/s
Mg =2Ma : J --> .
2 1
_ Fig:..2E._2_0_lb)_ VB= (-12.Sg)m/s
a=g/4
Thus, a= g/4i
- -.---- -- - -

:.1$-,
For system block A:
Mg-Mg =Ma r·-· - ---- - --- -· . ,
2 ' iSystem is shown in figure. All the surfaces are smooth. Rod is
-->
a,= g/2j
• lmove by e.x:temql agent with acceleration 9m/s 2 vertically
I mg :downwards. Find the force exerted on the rod by the_ wedge1 ;
1

Thus, llA =g/4i+g/2] Fig~ ~E.20 _(c) __ i


__j
l
I
"'cn
E
1

a,
--> •
a, = g/4i I
10kg
[,"sng!'.l?Rl,~J21l>
! . - -·. ·- ' --· - - - -- - - ----- --- . -·--
37'

Fig. 2E.22 (a)


•Three blocks shown in figure more_ vertically with constant!
!velocities. The relative velocity ofA w.r.t. C is 100 rri/s upward! Solution : Constraint equation
'.and the relative velocity of B w.r. t. A is 50 m/s downward. 1
a2 sin37°= a 1 cos37°
:Find
I
the velocity. of C w.r.t.
.• ·- •
ground. - All.l the. string are ideal.~
or, a 2 = a1 cot37°
I
= (9 x 4/3)m/s 2 = 12m/s 2

I'I
I

I,
I
~ · ~. A~a 2

cos3r- -

!-
i
I
I~
!a 1 sin37° a a1 cos 370
~ ·:ct a sil137"
2
Fig. 2E.21 (a)
--- - - .=-,e_-,.-1_ _ _ !ig, 2E.22_ (b)

Solution: From FBD of wedge we can see that


Let velocity of blocks, A, B and C are N sin37°= Ma 2
--> --> --> Thus force enerted by rod on the wedge is
vA,vBandvc
--> --> •
N= Ma 2 10x12
VA-Ve= 10Qj ... (1) sin 37° (3/5)
--> --> • =200N
Vn-VA=-50j ...(2)

www.puucho.com
Anurag Mishra Mechanics 1 with www.puucho.com

; FORCE ANALYSIS
L.-.
15_!i
On solving equations we get
T 4mg
1 = 3,,J3
Find the tension T needed to hold the cart equaibrium, if there
.is no friction. T = 2mg
2 3Jj
a=__!_
3Jj
T 30° Concept: What is cause of a acceleration of bob?
• • ,Resultant force on ball in x direction is (T1 - T2 ) cos 60° it
cause acceleration in bob.
Fig. 2E.23 (a)

Solution:

Nsine

Rl*N T
-t'-i'(:-
_•• J;,:.· w case
·A block of mass 10 kg is kept on ground. A vertically upward
force F = (20 t )N, where tis the time in seconds starts on it at
t = o.
(a) Find the time at which the normal reaction acting on the
··)B..... w T block is zero.
Fig. 2E.23 (b) (b) The height of the block fr~m ground at t = 10 sec.

N =Wease Solution:
Nsin0=T (a) When N =0 F =wt
20t = lOxlO N
Wcos0sin0=T or, T=W[.}372x~]
2 or, t == 5 sec
T=Jjw (b) from t = 5sec to 10sec
9 mdv
20t-mg = - - mg
- r- - - ... dt
[J~~fl~J?..I e . 2~J> v(t)
Jdv = J(2t -
C

lO)dt
Fig. 2E.25

A steel ball is suspended from the ceiling of an accelerating, 0

carriage by means of two cords A and B. Determine the v(t) =lt 2 -10tl~
'acceleration a of the carriage which will cause the tension in-A v(t)=t 2 -10t+25
,to be twice that in B. h 10
60° 60' f dh = f (t 2
- lOt + 2S)dt
0 5
B A .il.,. 10
h= -t3
- - 10t2
- + 2 5 t1
l 3 2 5
125
Fig. 2E.24_ (a) =--m
3
Solution: Concept: When force is variable always apply calculus .
- .
T1 cos 60° - T2 cos 60° = ma .. . (1)
T1 sin 60° + T2 sin 60° = mg ... (2)
T1 = 21'2 ... (3)
y 'lwo mass A and B, lie on a frictionless table. They are
T1

Lx·
:attached to either end of a light rope which passes around a
,horizontal movable pulley of negligible mass. Find the
,acceleration of each mass MA= lkg,M 8 = 2kg,Mc = 4kg.
:The pull_ey P2 _is vertical._
mg
Fig. 2E.24 (b)

www.puucho.com
Anurag Mishra Mechanics 1 with www.puucho.com

ffsa
,---
MECHANICS:::i-J
its equilibrium position. If we hold the block in position x,
B form Newton's second law,
1·-- _ .............. --- -··· ···-··· . -· . -- ....... - - ·r
A

System
JL

i
I
1

__Flg'._2E~6 !•] __ '-...···


: •
··,.... --¼
(AJ Constraint equation for pulley A is : ' ': Fexternaf
-+ 4 -+
(a) ap = aA + a 8 \.. / Frictionless
•·· -· surface
-> -> -> -> -> ->
(c) ap = aA- a 8 (d) a, =2(aA+a 8 ) Equilibrium
position
(BJ Acceleration of A is :
(aJ 3g (bJ 4g (cJ 2g (a)
5 5 5
· (CJ Acceleration of B is :
(a) 3g (h) 4g (c) 2g
...
Fspring
System
j x>o, ,
5 ' 5 5 "t.pnng /.. ··-\ ,--f·1 I
(DJ Acceleration of C is : : , , , ,'t.xtemal
2 . , ,. .1 J a .£me· I
(a) 3g (bJ 4g (cJ g (dJ ![ ........... EqJ_ilibrium
-· -· .. 5- . . ---~ --- __ 5 ________ _5
position
Equilibrium
Solution : position
mcg - T = mcac ... (1)
(b) (c)
... (2) Fig. 2.32
... (3) -> ->
F external + Fspring =0
->
Robert Hooke experimentally found that F external is
proportional to x.

...
Fe?(ternal

Constraint equation is x<O x>O


,, Spring compres$ed Spring stetched. _ _ _~:---+x
-+-+ -)-+
aA-a, =--(a 8 -a,)
2ap = aA + aa ... (4)
on solving eqn. (1) to (4) we get
~ig.3,33
4g 2g 3g
aA=- aB=- ac=- ->
.5 5 5 Fexternal = kxi,
T= BN
Where k is called spring constant and has unit N/m
Elastic Force of Spring ->
Spring shown in Fig. 2.32 (a) is stretched or compressed· Fstring = -kx i
by applying a horizontal external force on spring. We choose Therefore force of spring on block is proportional to the
origin of coordinate system at equilibrium position where amount of stretch or compression of the spring. It is always
the spring has its normal length. In horizontal direction directed towards mean position. It is independent of mass m
there are two forces acting on the system: attached to spring. An ideal spring has negligible mass as
-> -> compared with mass m attached to it.
(1) Fextemal (2) F,pring •
When we pull the block to stretch the spring, force of the Series Combination
spring is opposite to out pull [Fig. 2.32 (b)]. Ifwe push the Elongation or compression in different spring may be
block to compress the spring, force of the spri_ng is again same or different but tension in each and every spring is
' .
directed opposite to our push [Fig. 2.32(c)]. Force of the same.
spring is restoring force since it acts to restore the block to

www.puucho.com
Anurag Mishra Mechanics 1 with www.puucho.com

\FORCE ANALYSIS ---159 1


~- --- - - _,

The system of two weights with masses m1 and m 2 are


connected with weightless spring as shown. The system is
Fig. 2._34 resting on the support S. The support S is quickly removed.
x represents is the total extension produced in all Find the accelerations of each of the weights right after the
springs. support S is removed.
x 1 , x 2 , x 3 , ... Xn are extensions produced in individual
springs.
If this spring is replaced by a single spring and same
elongation is produced and tension developed is then this
single spring is equivalent to combination and its force
constant is equivalent force constant of combination. Even
total energy stored in combination will be equal to energy
Fig. 2E.27 (a)
stored in this single spring for same deformation.
X1 + Xz+..... ... +Xn = X Solution:
k1 X1 = k2X2 = .... : .. . = knxn
Concept: Force of spring does not change
. . . _ 1 . 1
X1,X2, ......... Xn - - . - .....•... -
. 1 instantaneously so find spring force at initial instant,
.kl k2 kn
1 Initially m1g =kx
k1 When support is removed, spring force does not change.
X ----c--~---,---X
I - 1 1 1· kx kx
-+-+........+-
k1 k2 kn
T = k1X1 = k,qX
1
M2g
k1 FBD litially
k1 X l l l X = k,qX FBD when support
is removed
- + - ........+-
k1 k2 kn (b) (c)
Fig. 2E.27
1 1 1 1
- = - + - ........+-
k,q k1 k2 kn NewFBD

1 -:E(l)
k,q k
For m 1
or
: m1g -kx = m1a1
=0
a1
Equivalent force constant is smaller than smallest
For m 2 m2 g + kx =m 2 a 2
individual force constant. (m1 + m2lg
or az = - - - - -
m2
Parallel Combination
Tension in different springs may
be same or different but direction of ·
tension in each spring is same. Even· An object of mass mis suspended in equilibrium using a string
elongation or compression produced of length l and a spring of constant K(< 2mg/!) and
in each spring is same. unstretched length !/2. Find the tension in the string. What
Total tension in this combination k,.x happens if K > 2mg /! ?.
and that produced in single
equivalent spring must be same.
k,q x = k1x + k 2 x+........ knx Fig. 2.35
k,q = k 1 + k2 +..... , .. kn
k,q =:Ek Fig. 2E.28 (a) ·
Equivalent force constant is greater than greatest
Solution : The string is under tension and the system
individual force constant.
is in equilibrium, if Kx < mg
www.puucho.com
Anurag Mishra Mechanics 1 with www.puucho.com

l
i.e., for, K(½) < mg [herex = -]
2

i.e., if, K< 2mg


l
T=mg-Kx
l
=mg-K-
2
2 acceleration of 3 m will be zero.
If K > ~g, the spring force is more than

mg, for x =.!. Thus the system will ai:c~lerate


2 ! - ·· • #- _';-~ -;_::-,-~~ --- . --;.",-f0:~1
[The mass in the Fig. 2E.30. can slide on a fr.ictio11/~sJ
lsurface.TIIe mass is pulled out by,a,distance x. The ..sp,;ifzgj
)constants are k1 and k2 · respectively. l:i".d the force. pulluj;
. . · .l

-~\·
•back on the mass and force on the wall.
I

; ·)
. -·· ___ ..•. ... Fig. 2E.3~ ·"' .J ._ ''
Solution : Springs are in series
Hence k = k,k 2
eq k1 + k2
and

Solution:
/. •.:<;:oJ?-Cept: Sp~ngforcedoes 710t,~haMe insta~;~ne~~;~J
~:c.afirststepfi'i!!:;,:f:'.'!frm in all the springs'._ : ___ :__J
· r+·i<,;,0:~-: .· Kx; • Ki, ··T
,rb:\::. ·: J; ' . rt .
,r··
13~~ . .,·,--r
'· Kx3+2rng
t , !
: Kx2 +rng:
L: • .Fig, 2E.29 (b) ,c.~___J
Form FBD of blocks we get
Block C 3mg = Kx 3 ... (1)
BlockB 2mg+Kx3 =Kx2
2mg + 3mg = Kx 2 ~.Smg = Kx 2 ... (2)
Block A ·Kx1 = Kx 2 + mg. ...(3)
when spring 2 is cut spring force in other two strings
remain unchanged, at that instant.
Kx1 -mg= ma 3
~ aa = Sgt .., .., ..,
Kx 3 + 2mg = 2ma 2 ail/•· = aA/8 + a 81• _ ... (1)
[aA/g]x = [aA/B]x +[aB/g]x ... (2)
www.puucho.com
Anurag Mishra Mechanics 1 with www.puucho.com

FORCE ANALYSIS 1s1:


---------- --"··-_j
From FBD of A it is clear that Block A cannot accelerate Solution: Constraint relations ;
horizontally.
LetX A•XB and Xe ate the positions of pulleys A, Band C
i.e., in x-direction because there is no force in respectively at any instant with respect to a dotted line
x-direction. shown in Fig. 2E.32 (b). The total length of string
Block A can accelerate in y-direction only.
[aA/,lx = 0
Therefore [a A/B ] x -- - [aB/g ] X
That means for an observation on wedge block moves
only x > 0.
For block A; mg-N = m(asin8) ... (3)
ForblockB; (N + mg)sin8 = ma ... (4)
On solving eqns. (3) and (4), we get
2gsin8]
.:·;·w···.·····~··1
: A
'
:
a= [ l+sin 2 8 '' ''
'' ''
The acceleration of block A, : 1g+ T :
aA = asin8
2
TGJ'.
....~-··;·····:
= I 2g sin8]sin 8 = [ 2g sin 8] 0
l_l+sin8 l+sin 2 8
Displacement of block A in 1 s is
1
_19't_T ···-···········
1
S = O+-aAt
2
2 :T T:
'
2
C
= .!:_ x [ 2g sin 2 8] x (1) 2
2 l+sin 8

= [ /+s:;:288]
1g
---- - --. ------. -- -.,. --·
[.. §.X-Ql}JPI~ i 32 :_> Fig. 2E.32 (b)

In the pulley system shown in Fig. 2E.32 (a) the movable,


pulley A, B and C are of 1 kg each. D and E are fixed pulleys. 2XA + 2Xu + (XB -XA)+Xc +(Xe -Xu)+ lo= l
The strings are light and inextensible. Find the acceleration of or XA +Xu +2Xc+l 0 =l ... (1)
the pulleys and tension in the string. Where 10 is the length of pan of string over the pulleys,
which is constant
Differentiating equation (1) w.r.t. time, we get
dXA + dXB +zdXc =o
dt dt dt
or ... (2)
l-).
vA+vu+2vc=O
also aA + aB + 2ac = 0
Let aA = a upward
and aB = a upward
then ac = ( a A ; au ) = a downward

Since string is same throughout and uniform, the


tension in it will be same every where. Thus
For pulley A :
Zf-(T+lg) = la ... (!)
Fig. 2E.32 (a)

www.puucho.com
Anurag Mishra Mechanics 1 with www.puucho.com

For pulley B::


2T-(T+lg)=la ... (2)
From pulley C :
lg-2T=la ... (3)
Solving above equations, we get

-g
a~
'
=-t
g
aB =-3' ac .=-
3 i ------·- ."'-,,-··----·----- -- ----- ·---·- -- . -- ·-· '
:Three blocks" 4, B & C are arranged as shown. Pull'!Yf andi
and T~2g
3
!strings are· idea( All surfaces .are frictionless. If block C is:
bL~mi:;,J~,]
----;;:,;-~__,:,
33
.---··-w<7
r~
~'
----~ 1<:"M~~--
observed.mbving down alongthe:incline at 1 m/s 2 .}',ind, masst
of block. B, tension in string and accelerations of A. B as the
system is ~ele(!5ed from rest.
I 6 kg ·-- --- ...... • - • .....
'lwo identii:ill, blo_cks each hav.ilig a mass of 20 kg are
connected to each other by a light znextensible string as shown
and are pla~er! _over a rough ·surface. Pulleys are connected "to
the blocks:· ·,:! · · . : , . . .' . !
1lll/771i7lli'7i'C.'fiL;,'fi-'7i-'T,-'T,-rd~i~
A
~/s2:
3 kg !

l
Find :a"'eleration of the blocks ;ilfter one second,' after the

e~ i
applicqtibn of·the time va,yingfotteof 40t N, where t'is- tn B _cc.....:,. ;

,.· J:
second. · •. ~-~

u=O.
~~kg
· ··

---~
kg ---F = 4
~~--··-~-~O

·
~:1, ,. _l=<-~=-™ -
u=0.4 '· ·
Solution:
11 + 12 + 1, + 14
==-~~-

=0
~=:~~(~)- ~- ~~ -=-' - -- -" - ' ---~, # ~
l 4 \ \ \ \ \ \ \ \ ~ \ \ \ \ ~ \ t .: Z1.,+ Z2"+Z3"+Z4"= o
'----'"'--.-:."~-- ~ig.:;;..:.~ . __j_:.c· -aB-2aA+l=O
Solution: aB + 2aA = l ... (])
· a1 · • a1 ;:·7 3g sin32°-T = 3 ... (2)

;L·=_-.~"-4-,·--i·~--+~-7 : a~:I 3
3gx--T=3
5
18-3 = T
T = 15N ... (3)
Fig. 2E.33 (b) · ... (4)
-~---'-·-·. ·--· -· -··--- T-MBg =MBaB
2T=MAaA ... (5)
2a2 = 3a1
Considering. block I 2T = OOA
F - 3T - f = 20a1 ... (1) Put T = 15N in eqn. (5)
Considering block Ii 30 2
= aA = 5m/s
2T- f = 20a 2 = 30a1 ... (2) 6
Solving Eqns. (1) and (2) Put aA = 5 m/ s2 in eqn. (1)

a
F- 3( i + 1501 )- f = 20a1
from eqn, (4)
2
aB = -9m/s •

. ' 5
F = 65a1 +-f ... (3) T-MBg =MBaB
2
15-M8 X 10 = M,; X (-9)
When motion ;;tar-r's t = µmg = 8 N and a 1 > 0 15 = MB(l0-9)
•, 40
1 MB =15kg
Solving we get, t = -s
2 MB = 15 kg, T =15,
1 2 2
Motion of blocks will begin at t =- s aA = 5 m/s -c>, aB = 9m/s .J,
2

www.puucho.com
Anurag Mishra Mechanics 1 with www.puucho.com

L~~~CE ANAL-'-)'S-'-1S_ _ _ _ _~ - - - - - - - - - - - :f~iJ


1
}i::q~.@!\')f?J~j35l:':> Bloc:ic B : ~o acceledrationsd~re sudpealrpose~ clino':1 it: itsd
·· - · · · - · · ..___,,,_.- acce1eranon re1anve to we ge aBA rrecte oni:jm e an
The -;,;;e-m-sh~-:Vn. i~ --;h;
Fig. 2E.35 ( a) is given an acceleration of :edge.-+ -+ .
;acceleration 'a' towards left. Assuming all the surfaces to be aBA = aBG - a AG·
jrictii°':l_ess_,_ji~d_t~e_J!)_rc~ _on ~~1!'_5_P.~"!_e._____ __ ·--. where B stands for block B, A for wedge and G for

i
I
g ':_N\
~
gro~~s ;BG = ;BA + ;AG
For sake of simplicity we drop the subscript G. Therefore
i
I +J!. N,·.. resultant acceleration of wedge is vector sum of its
I acceleration relative to A and acceleration of A on ground.
i i ....,,,_.,..(a_)_._.i., · Jj Takethe~~~~:t;~in_Fig._~:~~-~)~.

i
I
I: Fig. 2E.35
(b) N,

-- ~olu~:;~~-F~rces actin;:n -;~er~ are ~ho_w_n i~-~e N,cos AN, -+--+•A=A. A·mgc~·se
Fig 2E.35 (b).
:r.Fx =N 2 -N 1 sin 30° = ma ... (1)
1 8
mgsn mg
:r.Fy = N 1 cos 30° - mg= 0 ... (2)
Thus N, = mg
cos 30°
l_ .. _. -----~ 9
__Fl~g. 2_E~~J~L--------'
-+
= '1.15 mg x-component of acceleration

aB, aBX = a - A cos 9
From eqn. (1), N 2 =ma+ N 1 sin 30° -+
y-component of acceleration aB, aBy = A sin 9
=ma+ (1.15 mg) x (1/2)
= m(a + 0.58g) Wedge A: :r.Fx = N 1 sin 9 = MA ... (1)

lcoJ:E~p:~,w,~·J36~ Block B:
:r.Fy =N 2 -N 1 cos9-Mg = O
:r.Fx = mg sin 9 = m(a - A cos 9)
...
...
(2)
(3)
!TI1e block B st-;;,;.fr~~.~~; and slides ;n-th~ ·,:;~e A~hich1 };FY = mg cos 0 - N 1 = mA sin 0 ... (4)
ican move on a horizontal surface. Neglecting friction, N,=--
MA
sin 9
1determine (a) t/ie acceleration of wedge, (b) the acceleration
;af! t!Je _bJoctrelati)'Ll'Q tli<!....IY~ggg._,__ ··;;·\. __ 7 Substitute N I into eqn. (4) to get A.
1
mgcos9- MA =mAsin0
sin 0
' ~ I
or mgcos0=mAsin0+ MA
-> sin 0
-+--+aA
A A= mg cos0sin0
, 8 eA , or ~:
! ...,,.,...,...,.__....,
: -- . ~ I
' !
From eqn. (3),
msin 2 0+M . a
I __ Fig. 2E.36Jc)
l
.
:--···----~~--~:61~!--~-----·- . .J ma= mgsin0+mAcos0
Solution : This problem involves two branches of
mechanics: kinematics (which deals with motion) and vn
a=g sin0+A cos0
dynamics (which deals with cause of motion).
. mg cos 2 0sin0
>,\;--. First we shall analyze the. accelerations of wedge and = g sm 0 + -''------
"·'.',·block.::·.·.. · · m, sin2 9 + M
Wedge A : It moves on horizontal surface; we assume (M + m)g sin 0
=
its acceleration towards right. M+msin 2 0
.. Note that axes of x and y can be assigned in another
manner, as shown in Fig. 2E.36 (c).
www.puucho.com
Anurag Mishra Mechanics 1 with www.puucho.com

:_164___ --- . --- --- ·-


x-componenc of acceleration of B = a cos 0 - A On substituting expression for Nin eqn. (4), we obtain
and y-component of acceleration of B = a sin 0. mg cos0- MA = mA sine
Now force equations for block B are sin 0
D'x = N 1 sin 0 = m(a cos0-A) ... (5) . 0+-
mg cos 0 = mA sm MA-
D'y =mg-N 1 cos0=masin0 ... (6) sin 0
We can arrive at tbe same result by considering eqns. (5) A= mg sin0cos0
or
and (6) instead of (3) and (4). m sin 2 0 + M
- -- - --·--- - -- -- r - r
LJ=,~t;pHJ~J 37 l__>
'' In the F~. 2E.37 (a) shown, mass ;m, is being pulled on the A rod 'A' constrained to move in vertical direction rests on a
incline of a wedge of mass M. All the surfaces are smooth. wedge B, as shown in the Fig. 2E.38 (a) Find the accelerations,
Find the acceleratiqn of the wedge._ . of rod A and wedge B instantaneously after system is released,
from rest, neglecting ftiction at all th_e contact surfaces. '

M
m

B F
. _JmL
A
Fig. 2E.37 (a)
B
I Solution : Fig. 2E.37(a) shows force diagram of tbe M B
wedge and the block. Let acceleration of block relative to
Fig. 2E.38 (a)
wedge be a'.mM = a'. and acceleration of wedge on ground is
Solution: In the Fig. 2E.38 (b) dotted line shows initial
position of rod and wedge. If the rod is displaced vertically
-+ -+ -+
amM = am - aM through y, then tbe wedge moves a distance x.
-+ -+ -+ y=xtan0
or am = amM + aM Therefore tbe relation between accelerations of rod and
or (amlx = a - A cos 0 wedge is
and , Cam\ = A sin 0 a=A tan0 ... (1)
N,
+-·f
·_r
X
<ill-. - - - - - -
N sin 0
o:• 000
:8
Y~~x~•
c.Pe:, ,· ••
X
A
N'

N
N sin Oco
8
'OZ <i'°' ,·' 0 ,i·.. F z
/JI. '9,s,,.
~0

AcosO
mg
"·. N cos B

?,..·· A sin B
'\.a y r xf :
N

Fig. 2E.37 (b)

Equations of wedge: img


D'x = N sin 0 = MA ... (1) a
D'y = N ' - N cos 0 - Mg = 0 ... (2) Fig. 2E.38 (b)

Equations of block : Equations for wedge:


D'x = F + mg sin 0 = m(a - A cos 0) ... (3) D'x =Nsin0=MA ... (2)
D'y = mg cos 0 - N = mA sin 0 ... (4) D'y = N' -N cos0-Mg = 0 ... (3)
MA
From eqn. (1), N = -.-
sm 0
www.puucho.com
Anurag Mishra Mechanics 1 with www.puucho.com

[ FORCE ANALYSIS -1651.


Equations for rod : When one body moves on another the asperities of one
:r.Fy = mg - N cos e = ma ... (4) surface will interlock with those of the other. Causing their
deformation. As a consequence, tangential as well as normal
From eqn. (2), N =-MA- forces will develop at the surfaces in contact, as shown at
sine
one of the points of contact in [Fig. 2.36(a)]. The friction
On substituting expression for Nanda in eqn. (4), we force is the resultant of these tangential forces. If the
obtain asperities of the surfaces are in direct contact. We have dry
MA case
mg----=mAtane friction. When the surfaces are lubricated, it is fluid friction
sine [Fig. 2.36(b)]. Fluid friction is always much lower than dry
or A= mg sine case friction.
m sin 2 e + M cos 2 e The Laws of Sliding Friction I

mg tan e
=--''---~-
M +m tan 2 e
Friction depends on a series of complex mechanical,
chemical and other phenomena. The laws of sliding friction
1
and from eqn. (1), are the result of generalization of a great body of
experimental data. The basic laws of sliding friction are
a=Atane= mgtanze presently formulated as follows:
M + m tan 2 e 1. The friction force is pmportional to the normal
FRICTION pressure.
A friction force arises when one body moves on another 2. The coefficient of friction depends on the nature of
and is always opposite to the motion. Friction plays an the bodies in contact and the physical condition of
important role in many transmission mechanisms, such as the surfaces in contact. ·
belt, friction, rope drives, the motion is transmitted with the 3. Friction between similar bodies is generally larger
aid of friction. In other cases friction opposes the motion and than between dissimilar bodies.
leads to a useless expenditure of work. 4. The friction force does not depend upon the a\ea of
Two types of friction are distinguished, depending upon contact, except at high unit pressures.
the form of motion: sliding friction, kind, and rolling friction. 5. The static friction force is greater than the kinetic
friction force for most bodies.
As experiments show, friction is a complex phenomenon.
Here is a simplified explanation of sliding friction. 6. The friction force depends on the relative velocity
the bodies in contact. In practice the friction force is.,
The surfaces of any contacting bodies have irregularities often assumed to be independent of the velocity is
[Fig. 2.36 (a)] the range of velocities encountred usually.
7. Coefficient of static friction depend on the material
of the bodies in contact, on the quality of machine
of contacting surfaces.
Analysis of Friction Forces
Fig. 2.37 shows a block of mass mg resting on a rough
surface. A horizontal force 'P' is applied to the block force P
is gradually increased from zero.
I * When applied force P is very small, the block does not
I- (a) . -
move. From condition of equilibrium,
l:F'x = p - F friction = O; Ffriction = p
:r.Fy = N - mg = O; N=mg
I_ _
: I
I !
I ~-...;::: ~ -

iW!
m_ p (applied force) [
F'--'---<'---'--- I
rough ]
J (Fraction force) I
mg
I (b)
I 1

L__________ _:~9: ~-36 __ ____ _J


Fig. 2.37

www.puucho.com
I
Anurag Mishra Mechanics 1 with www.puucho.com

166,, - .. MECHANl~S-1 l
Friction force counter balances external force, till · ¢,, is maximum angle of friction if If>,.; ¢,, then block is
the block is static. This friction force is referred as static.
static friction (F, ). As external force is increased, + If applied force P is greater than F,max (µ ,N), the
static friction also increases to its maximum value block will have a resultant force F - fx on it.
f,max. Where A is kinetic friction force.
+ As applied force P is gradually increased, a limiting A =µkN
point is. reached where friction force F, (maximum µk ~coefficient of kinetic friction
value fsmax.) is not sufficient to prevent the motion N ~Normal contact force.
of block. When the block is about to move, the state The block will accelerate in the direction of
of motion is called impending state of motion. resultant force.
At this point friction force has maximum value. Fig. 2.40 shows a
N
F,max. = µ,N block of mass m,
where µ, is defined as coefficient of static friction, kept on an incline
N . is, normal contact plane whose angle
of inclination can be
+ if~~:~e Pis greater than I> IF a
varied: At certain
Fmax., the block will have I value of ¢,, just
a resultant force "· sliding of block
P - Fmax. on it. The block -----··;1.·,~,----- starts. At this instant _ _ _F)_g. 2.40
will accelerate in the iL Dynamic friction force at its
direction of resultant ~· ta· maximum value F,max. the equilibrium equations
force when · sliding 45°
"-'-'--~--..,.P are:
mdtion ensues. r.Fx = mg sin¢,, -µ,N = 0
Fig. 2.3s
· F=µk N + Friction force opposes relative motion between two
where µ 1c is d¢fined to be coefficient of kinetic surfaces. In order to decide the direction of static
friction. · · friction, try to imagine the likely direction in which
Fig. 2.38 shows variation of friction force versus the· body will tend to move; friction force is opposite
external force graph. to it. In the figure, force P pulls block B towards left
+ When condition of impending motion or sliding is and A is pulled towards right. Friction force on B is
not known. To determine friction force we assume towards right and on A is towards left. Important
static equilibrium and solve for the friction force F,. point to notice is that for two contact surfaces
The possible results are: friction force is in opposite direction. It is intern.al
(a) F, < µ 8 N (maximum value of friction): Body is force for two contact surfaces, so it must be an
in static equilibrium. The value of F, can be
determined from the equations of equilibrium.
Jb) F, = µ, N: Body is in impending state or about.to
move assumption of static equilibrium is still valid.
(c) F,,> µ, N : This condition is impossible. Friction
force cannot be greater than F,max. (µ, N). ·
+ Normal contact force N and friction force F are two .. , '
components of the resultant contact force R of the
surface on the block. Angle between resultant .-,.~-"--IA
contact force R fill.d contact force N is called angle T
...._+-_,_- fe .
of friction(¢,).
tan¢,='~; R=~f 2 +N 2 ma9'
_Fig. 2,41_;_____ · - - - - - - "
When block is in impending
+

~
state maximum value of Consider a conveyor belt moving with velocity v A. A
static friction force is acting small block is gently lowered on it.
on block.·· -+ --+ --+ -+
VBA =VB -VA=O-VA
f=F,,;, .._ =µ,N Velocity of block B relative to surface A is towards left;
f
,___ _F_;;ig. 2.39
friction force is opposite to VBA, i.e., towards right.
Due to this friction force, the block accelerates
i
' www.puucho.com
Anurag Mishra Mechanics 1 with www.puucho.com

- - . ----.;:::;-,
FORCE ANALYSIS 1671
towards right and the belt retards. Finally the block Solution: Force of friction = 0
acquires the velocity of belt and moves with it. => N =0
.. ·1-:-7
~ J~='~g~_J;~-l~,cl 39 1..> I
=> f=tan0 =>a=gcot0
a
A block weighing 20 N rests on a horizontal surface. The: F = ma = Mg case
coefficient of static friction between block and surface is 0.40 k.J~.x:g_t;.ti.l!i!l:;;;.--::Q_':-,..._
and the coefficient of kinetic friction is 0.20. ; = ·- . ~~~
·····- -- - -··7
( a)How large is the friction force exerted. 011 the block?
A black of weight W rests on a rough horizontal plank. Thel
(b) How great will the friction force be if a horizontal force of
slake angle of the plank 0 is gradually increased upto 90°. ;
5 N is exerted on the block?
(c) What is the minimum force that will start the block in: Draw two graphs both withe along x-axis. In graph show the:
motion? ratio of the normal force to the weight as a function of 0. ,
In second grapl~ -show ·the ratio of the friction force ta the!
(d) What is the minimum force that will keep the block in.
motion once it has been started? weight. Indicate the region of 110 motion and where motion,
exists.
(e) If the horizontal force is 10 N, what is thefrictionfprce?
Solution: (a) Solution
N I
When - I . I
Fmction =0
From condition of equilibrium,
i
angle of incline is
:
I
P=F=O being gradually
mg cos a increased I
(b) First we calculate
----mg
F=,.=µ,N
_F_lg. 2_1=:~1 (a)_
= (0.40 X 20)
=SN mg Till block is static mg sin 0 = f,; ~ = sine
Since P < F max., block is in static Fig._2_E.39 mg
equilibrium, i.e., As incline angle is increased, if block does not move
P=F=SN friction force has balanced component of weight down the
(c) When the block just starts to move, it is in incline
impending state. From condition of equilibrium, In impending state of motion
:r.Fx=P-F=x_=O mg sin0 0 = µ,mg cos0 0
or .P=F=x_=µ,N=BN tan0 0 = µ,
(d) When block is in motion, F = µ kN. ..!'!.
mg
Minimum force will be required to move the block with
constant velocity.
From condition of equilibrium,
r.Fx = pmin. - Fkinetic = 0
or Pmin. = F!cin,ti, = µk N = (0.20)(20 N) = 4 N
(e) Since P > µ, Nin this case, the block accelerates.
From Newton's second law,
:r.Fx =P-µkN=ma
Therefore F = µk N = 4 N. - l f - - - - ~ - - - . . . 1 . . . ._ _.e 1,'

goo
Fig. 2E.41 (b)
lc~S~-S¼W,B!iJ 40 [> - - -. ----

A wedge of mass M m~es an angle 0 with the horizon~!. Th~


wedge is placed on horizontal frictionless surface. A small'
block of mass m is placed on the inclined surface of wedge. ,
What horizontal force F must be applied to the wedge so that!
the force of friction between the block_ and wedge_(s ,._ero ]__ ;

www.puucho.com
Anurag Mishra Mechanics 1 with www.puucho.com

j168
-·-·· -- . - --- ·-···· - - -
MECHANICS-I '
--------------- - -------------- ----
...!... Impending state , Tmin=40N
mg
'/ µsCOS8o ,i---·-----:a,,/
:50N-[I
'
I µ,cose 0 1----,,L-'--4~
I
r (f,)max•40N

. mkcos00 j (c)
Static T•40N
sine
i
motion 50~
occurs And friction
force is (10~ N
f, •10N
90° e
(d)
Fig. 2E.41 (c)
I. Fig. 2E.42
When block begins to slip
fk =µkmg cose Thus block A remains static
fk Force F can not pull block A
- =µk case
E._f;~~'~'~,P.-'~- i 43 1>
w 1-····-- -------r-i-

[~>f~'L'gfg~J42!>
r --- - - - ---, ;11ie 10 kg block is resting on the horizontal surface when the
:Find the acceleration of the block and magnitude and: force 'F is applied to it for 7 second. The variation of 'F with
'direction offrictional force between block A and table, if block I ;time is shown. Calculate the maximum velocity reached by the
:A is pulled towarq~ !eft iyith a forq, pf !jO /'{._ lblock and the total time 't' during which the block is in1
. I 1'
!motion. The coefficient of static and kinetic friction are both,
I }--X ! ;a.so.
' F(N)
µ•0.8
g•10m/s2 B 100 ...... .
4 kg
I Fig. 2E.42 (a)
L -- --- - -·-'
Solution: Case (i) If block moves down, maximum
possible tension T = 40 N is attained when it moves with
constant velocity. In this case
---·· / '---'---'------+ t(s)
4 7 l
N [
50 N Fig. 2E.43 (a)
I_
A 40N=Tmax'
Solution : Block begins to move when
0 40N F=µN
= O.Sx lOx 10 = SON
50 N From t =O to t =4sec
i . - --- --
Fig. 2E.42 (b) F = 2St
Tmax can not over come apposing forces of 90 N, From t=4tot=7sec=40N
Block begins to move at t =2 sec.

:41:NF:
therefore it is not possible.
Case (ii) after that
If F=SON force can pull block A to left, mm1mum F-µN= mdv
dt µ '
tension in string Tmin = 40 N if B moves with constant ' '
velocity. 2St - SO = 10 dv mg
dt
Fig. 2E.43 (b)

www.puucho.com
Anurag Mishra Mechanics 1 with www.puucho.com

' - FORCE ANALYSIS 169


V 4
or, f dv = f (2.St - 5)dt Concept: Kinetic friction is opposite to relative velocity
it opposes relative motion. When horizontal component force
0 2

V -0 =12-~t2 - 5{ is reversed, relative velocity is not_changed therefore, direction


of kinetic friction does not change.

After t
force
=
= (2.5 X 8 - 20) - (5 -10)
= 5m/s
V

4 sec.; block retards due to greater friction


Stage 2: Fcoh-'
µ,N
Fsin37°

v = .Jl 12 m/s

a
,._ ----+Uinitia! = 5m/s
F cos 3 7° is reversed, block continues along original
- - 0 - F a 40N direction, but due to retardetion created by µ kN and
F cos 37° block travels till it stops.
µ,Na 50N
-(Fcos37°+µkN) = ma
Fig. 2E.43 (c) -(20x .:': + (0. 25 x 8) = 2x a or, a= -9m/s 2
5
a= 50- 40 = lms-2 Displacement of block in this phase
10 v2
0 =v s =-
2
- 2as;
Velocity of block at t = 7 sec 2a
(112) 56
at t = 4sec, v; = 5m/s =--=-m
v 1 =v;+at 2x9 9
Stage 3: Which block returns its a acceleration is:
= 5 - 1 x 3 = 2 m/s
Fcos37°-µkN = mg
LE~ff~J!tl:?l~ .~
' ~
44j;._> a= 7m/s
2
Fsin37°
Velocity of block when it
A force of 20 N is applied to a block at rest as shown in figure. returns to original position Fco~
After the block has moved a distance of Bm to the right the
~
v2 = 2as
direction of horizontal component of the force F is reversed in·
56
direction. Find the velocity with which block arrives at its. =2x7x(s+ ) mg
starting point. 9
Fig. 2E.44 (c)
v=--m/s
16-.fi
-~7' 3
µ•O.~
- - - - .-~
lE?ffl~BL~ ! ~__;>
Fig. 2E.44 (a)
,Find the contact force on the 1 kg block.
Solution: Stage 1: Motion till force reverses its () ':,

direction '<
N = mg -Fsin37°
= 20-20X~= SN
Fsin37° .
• • Ijj Vs;;.;
5 Fig. 2E.45 (a)
Fcos37°-µkN = ma
. µ,N Solution : fk = µN
2ox .:':- 0.2Sx 8 = 2x a ' N
5 mg•20N 4
2 =0.Sx10X-=4N
a= 7m/s Fig. 2E.44 (b) 5
Velocity of block after N=lxlOx.:':=sN
displacement of 8 m 5
v = .J2 x 7 x 8 = .J112 m/s Contact force = ~ fk2 + N 2 37'
10

= .J16+ 64 Fig. 2E.45 (b)


= 4-JsN
www.puucho.com
Anurag Mishra Mechanics 1 with www.puucho.com

MECHANICS-i .
- - - - - - ,M~-----<•-
< '"s

Concept: Contact force is res.ultant of normal reaction 1


and friction force. m,,C:J::F
l~~~q~Fl~,·~ Fig. 2E.47 (a)

Solution : In the impending state


,Blocks A and Bin the Fig. 2E.46 .(a) are connected With a
'string of negligible mass. The masses are placed on an inclined N .F
plane of inclination 30° as shown inFig. 2E.46 (a) . If A and
B each have mass m and µ A = 0 and µ n = J1, where µ A and
f,
50 N '
µ n are the coefficient offriction between plane and the bodies Fig. 2E.47 (b)
}"espectively, calculate the acceleration of the system and
tension in the string. F = kt =µ,mg
2k=0.4x5xl0
or, k = 10 N/sec
When force F is further increased, block accelerates
kt -µkN = ma
Sa= lOt -15
Fig. 2E.46. (a)
or, a= 2t - 3m/s 2
Solution : If system is moving down with acceleration a
a for block A
. f=-yf'mg

I~~
~ o s 3 0 ° = l.mg
~ mg
2
300 ;/2
1 m/s2 t----,

t= 2·sec

Fig. 2E.47 (c)


l
Fig. 2E.46 (b) Concept: Static friction: The direction and;
magnitude both are self adjusting such that relative motion is 1
mg -T=ma ... (i) opposed. !
2
mg +T_mg =ma ( a) Direction: It acts always tangentially to the contacti
... (ii) surface.
2 -.fz
solving eqns. (i) and (ii)

a=½(1- Jz)
Fig. 2 42 (a)
T= mg 1 . l,
2-.fz This example is to show that friction acts against _the:
tendency of relative motion._ . '
~J~~f!-~~J~1~
iln the Fig.' 2E;47 (a) shown a time dependent force F
,~,lg®:
·expressed as F = kt is applied on a block of mass 5 kg. f =~F2+p2
.Coefficient of static and kinetic friction is µ, = 0.4 and
' ' y
Fig. 2.42 (b)
·µ k = 0.3. Motion begins when t = 2 sec draw a acceleration·
'vs _time graph for block. (m = 5 kg,µ, = DA µk = 0.3)
www.puucho.com
Anurag Mishra Mechanics 1 with www.puucho.com

- -
FORCE ANALYSIS 171

(b) Magnitude: Maximum strength of the joints formed is


directly proportional to the normal contact force because
higher the normal contact force higher is the joint Find unit vector in direction of friction force
strength i.e., f, max ~ N
It al.so depends on the roughness of contact surface.
acting on block ~-
-) AA-+ AA ~Vp
f, max (al.so called fumufr,g) =µ,NJ · Vp=7i-2j, VB=3i+j
Fig. 2E.49A
Magnitude of static friction is self adjusting such that
relative motion do not start. -+ A A A A

Solution: v 81 p = 3i + j- (7i - 2j)


• • 4: 3:
rk = -V B/P =- - l +- J
5 5
m = 20 kg,µ, = 0.5,findftiction on block. It is not self adjusting as in static friction. A = µ kN. It is
fixed.
/4100
Concepts:
~-- (1) Value of µk is always less than µ,(µk < µ,) from
Fig. 2E.48 (a) experimental observation.
(2) If on(y coefficient of ftiction (µ) is given by a problem,
Solution: N+60-mg=0
N = 140
f,max = µ,N
f, max = 70, hence answer is 70.
J-so
itN
(3)

(4)
(5)
thenµ, -µ k = m (assumption for)
Value of µ, and µ k is independent of surface area it
depends only on surface properties of contact swface.
µ k is independent of relative speed.
µ, and µ k are properties of a given pair of surfaces i.e.,
mg for wood to wood combination µ 1 , then for wood to iron
Fig. 2E.48 (b) µ 2 and so o_n.

, --E-xample i 50' -~
~---' --:_: ,:_: -" -~-' '··-·---·~
Find 0 at which slipping will start. µ, is coefficient of static
ftiction. (Angle of repose) _ Blocks are given velocities as shown at t = O,find velocity and


Fig. 2E.49 (a)
position of 10 kg block at t = 1 and t = 4.
4-12m/s
10
g=10m/s2
l =0
Solution: N - mg cose = o µs = µk = 0.4
f, max =µ,mg cos0 Fig. 2E.50 (a)
when slipping starts f, = f, max
Thus mg sin 0 = µ mg cos0 Solution: How a student will approach making FBD.
tan 0 = µ,
tan-1 µ, is called angle of repose.
Fig. 2E.49 (b)
-· Jii.
Direction of Kinetic Friction
It acts when there is relative motion between two
__Fig. 2E.50 (bl__
~
surfaces in contact.
Direction: It acts always oppositely to the relative 40+T=l0a; 50-T=Sa; a=6m/s 2
velocity.
-
u = 12; a= -6
v = 12-6xl = 6m/s; S = 12xl-3xl = 9 m
mis !.-..0._fk
w.r.t.B ~
But it is wrong. Since velocity has changed the direction
5m/s ~ during motion friction would also have changed thus
Fig. 2.43 direction and acceleration will change.
u = 12; a = -6 (till velocity becomes zero)
v=0 => t=2sec; S=2x2-3x4=12m
www.puucho.com
Anurag Mishra Mechanics 1 with www.puucho.com

MECHANICS_;i-j.
NowFBD
50-T= Sa
T-40=l0a f = 40 •

10 ~T JB:oN a= 30 = 2ms-z
15
a=~m/sz Fig. 2E.50 (c) Fig. 2E.52 (b)
3 (4) Now check if this acceleration is possible by
2 4 verifying f ~ J1 i.e., make FBD of
u=0,a=-,t=2,v=-
3 3 -2 f •
S= .!. x ~ x 4= .'.!; Total displacement= 12-.'.! = 10~ f ~ 30 I rn_l~2:
2 3 3 3 3
30- f = 5 x 2 or f = 10x2= 20
Concept: Friction oppose rela_tive velocity not relative f = 20 < 25

L~~~~~:~-~- r-;;-t>
acceleration.

Find acceleration of blocks F

µ 1 =o.s-

a .
, µ, = 0.2 .........
= 40 N

. --~·
0

Solution: Assuming same acceleration


60 N

a =4 m/s2
Fig. 2E.53 (a)

Fig. 2E.51 (a)


.~(
Solution: (1) First of all find values of limiting friction
at all contact surfaces. CJ, max) 60-f=Sx4 or J=l0x4=40
f = 40> 25
0 fsmax=25
hence our assumption is wrong.

"'l"~~,J~,"
Fig. 2E.51 (b)
= 30
'
•2E • \
5
~ 25
\
,, 60

a1 = 7, Oz = 2.5
-

(2) Maximum force upper surface of 10 kg can Fi!!: 21t,53 (b)


experience is 25 N so it will not more relative to ground. t - -· - -· ·- - - - ~

(3) Hence only 5 kg will move. k:~0:9"''22,~l~ .l~~-•>


25____[D--- 40 Find m(l,!Cimumforcefor which they can moyf_tgget!Jer..
1.:7 . aA=3,a 8 =0
Solution:
I 10 25 ·
25••-'-----'1
Fig. 2E.51 (c)

:: :
25-m

l' !e;?~f~-riiel~. r;-27> _30~~!;:t


Fig. 2E.54_ ....
A

, µ, = 0.S -ill- 30 N
if they are moving together a 1 = az = a
F-f-30=l0a; f=5a; F-f-30=10x1. ·
smooth~ , 5'
Fig. 2E.52 (a) F = 30 + 3/ maximum f is 25.
F=30+75=105
Solution : IT] /1 max = 25 Two Block Problem
~ fzmax =0 - .. - ------.
LJ=:;~~g.fD,~J~ i 55 L->
, ,.

(1) 10 kg block must move because some force on upper


surface will act on it.
(2) B can either move with same velocity and _<;:onsider two b.locks with friction coefficient and mass as:
acceleration as A or it can move relative to A. shown in Fig. 2E.55 (a). . I
(3) Always assume it moves with A and solve.
www.puucho.com
Anurag Mishra Mechanics 1 with www.puucho.com

I~---------------------
FORCE ANALYSIS - ~_71]
r-- -_ .:·""""·-""'"' ""'·: -_ - ----- --·- -
I
I
I
' I
SE 13
.,----µ,
I
a 0.2, mA a 2kg
.,----µ•0.3,m 6 •3kg,
'
1
F-6=2x-
3 i ~ F-;
! 6 I

I · mmm mh,fir , or, F= 20N Fig. 2E.56 (b)


3
l II I. -- -----

I______ __!,._,=,.. ::u=:,__--,


Fig. 2E.55 (a)
_,__,..,,_,.,_._,,""=--. __,.,._---=-~"--'-

Solution: Force is applied on blockA


I~E:~F~t11-~l.~ J57 [>
.I.11_exqmp/e !i!i_fil_lg.frictiol_l_fosce b~tw~~n_ !,locks if F = 6N._
!qfl--A
I A F
NqfJ!
B
Na Solution : For this force both the blocks move in
I combination acceleration of system
'
I = 6-5 = "I.m/s 2
mAg meg
5 5
I Fig. 2E.55 ( b) From Newton's second law on blocks A, we get
L_ - - -- . - ··- -
Find f max that exists at each surface and NA and N 8 6-/=2x"I.
5
f1 max =µ,mAg=4N or, f= 6-.~
5
f2 max = µ2Na = µ2CNA + m 8g) = 15N = 28N
(i) If motion of B is to take place .-"--~ ·- .. 1 5
f 1 > Ji which is not possible in this : 12 , 8
j I f-+t,
case. Therefore block B cannot be : Fig. 2 E.SS (c) c:tt->5!im,lilg ~i58l>
moved by applying any force on A. • - - - -
Thus only A can slide, it just begins to slip when F = 4N Coefficient of friction between 5 kg and 10 kg block is 0.5. If:
(ii) Let µ 1 = 0. 3 and µ 2 = 0.1 'friction between them is 20 N. What is the value offorce being,
now f 1roll = 6N !applied on 5 kg. The fio.9r is frictio11le_s~, _ ·
f2 max = SN Solution : First compare friction with force with µN;
Now, f, mM > f 2,,,,,; blockB can be moved find force F for f < µN implies.
which slipping occurs at any surface.
' - -

I
----
1 A f---+F :
B J '
20 N is static friction so there will be no relative motion
between blocks and acceleration of both will be same
----~
20•41~--~
I~~,
I Fig. 2E.55 (d)
10kg 110kg , • : 2 m/s2:

Fig. 2E.58
As force required to cause slipping of A is more than that I
at B, slipping starts at B. Blocks A and B. move in F-20=5X2
combination. For slipping to start
F = 10 + 20 = 30 N
F=f2max =SN

~~~111riJ~~v LE,~o.r,q~~~J 59 b>


1- - - - -·
!An object is given a quick push up an inclined plane. It slides:
ta) In example 55 what is maximum possible acceleration of, :up and then comes back down. It is known that the ratio of
·the ascent time (t up ) to the descent time (t dawn) is equal to the I
j(b) fn°example 55 what is mcu.imumforce F for whichblocks1 ·1coefficient of kinetic friction (µ). Find the angle e that the,
L_rr,pye)n_cqm/1iJ'!..a_t(on.___ _ ______ _J inclined plane makes with the horizontal Find also the range!
1
Solution: (a) Maximum force that A can exert on B is ,ofµfor which the situation described is possible. Assume. that;
/1 = 6N -the_ coefficients_ of static_and_ kjneti~ fric_tion a~e_equal. _i
max
a8 = 5 - 5 = "I.m/s 2 Solution : aup = g sine+ µg case;
Thus,
max 3 3 ad,wn = g sine - µg cose
Till this moment blocks A and B more in combination. L = "I.[g sine+ µg cose]t~;
(b) Fmax can be obtained by applying Newton's second 2
law on upper block
www.puucho.com
Anurag Mishra Mechanics 1 with www.puucho.com

-MECHANiC:S:fj
'"~---,-'

L = .!_[g sin0 - µg cos0] tJ,wn So, Resultant force = ~ fk + N 2 2

2
(sin0+µcos0)µ 2 = sin0-µcos0 =)(µN)2+N2 =N)1+µ2
µ(1+µ2) = mg cos0~1 + µ 2= 12-F, N
tan e = '--'--'--c--'-
(1 - µ 2) . . . ,---,,
. . .....

µ <1 iJ;;lff½~:EiJ~ I. 62 J.>


;E . . ,- . r.:7 ...._ .
A car has headlight which can .illuminate a horizontali
hi,~~~m?Ii?i~ .1 60 f ~ straight road in front upto a distance L If coefficient offriction
A time varying for F = ·10../2 t starts acting on the between. tyres & road is µ. Find the maximum safe speed of the
3 kg block kept on a rough hori,;ontal surface (µ = 0.2) at car during a night drive neglect the reaction time of the;
t=0.Find 1
driv.q. . ._ . . . . I
(. a) the moment of time when the blocks leaves the surface
.
I' Solution:
'(b) the moment of time whm.the<horizontal motion begins. :
Concept: Car must stop within the maximum ·vi.sible!
' ·~;._ ' ! ,safe distan_ce.
,µ=0.2~ ' a= -µg, v f = 0, s = I
2 2
VJ-V;= as
2
Fig. 2E.60 (a)

Solution: From FBD of


0
2
-u!ax = -2µgl
block calculate N '. ~ N Fsin45°=10t i Umax = .J2µgl
N = 30-lOt I •
(a) The block leaves \ Fcos45~=10t . - - - -
the contact with surface, A block of mass 2 kg is placed on the floor (µ = 0.4). Ai
'. f l
when N=0 .
W=30
I
I
;horizontal force of 7 N is applied. on the. block. The force a/
:.frictiqn};,etweeri t~e bJgck & floqr 4 J,. Find .the.J, ~- __
(b) The
t = 3 seconds
block
horizontal motion, when
begins

F cos45° = f max
Fig. 2E.60 (b)
Solution: External force = 7 N
Max. friction force= 0.4 x 2 x 10
=BN
·--
• ·o-!
; f5
7N
•mm, mm~.
Fig, 2E.63
l0t =µ(30-lOt) External force is smaller than '--•'

lOt = 0.2(30-l0t) maximum friction force.


1 Hence, f, = 7 N
t = -sec

~;i;;~!i~~L~ ..1617::__>
2 I . .. . .. . . .

. .
·r,-:i-
t:. ,_!;=..~,f¼.~~i~'j 64 1> - . .. ···- - I
• 1 Block 1 sits on top of block 2. Both of them have a ma,ss bf 1'
·A block of mass 3 kg slides on a rough fixed inclined plane of kg. The coefficient of friction between blocks 1 and 2 are
1
37° angle having coefficient offriction 0.5. Find the resultant, µ, = 0.75andµk = 0.60. Thetableisfrictionless.AforceP/2[
force exerted by plane p_n the blocf5.. / is applied on block l to the left, and force Pon block 2 to the.
N : right. Find the minimum value. of P such that sliding occurs:
,,-'."> f
' ¥) between the two__blof.¾.?.,. ..
, mg sine
37° mg case Solution: P- f = la
f-P/2= la
Fig. 2E.61
P/2 = 2a
Solution as f is static
f = 3P/4 5 µ,mg Fig.2E.64
' Conc!!pt: If angle
of incline is greater than angle of
µ,mg= 0.75 X 10 = 7.5
:repose friction force is kinetic in nature. I J 4
Thus P 5 -µ,mg
' • ' - - - -- · - & " " " " ' _ ,_ _ _ _ -- --

tanq, S: µ, 3
A =12N P=lON
www.puucho.com
Anurag Mishra Mechanics 1 with www.puucho.com

FORCE
____ ANALYSIS
,...,,_,·"·~------
1
".,,.__. - .. ··--- -·-·- ---- -----·----
The vector triangle of forces is shown in Fig. 2E.65. The
minimum value of P will occur when the lines of action of P
and Rare perpendicular to each other, as shown by the force
A block of mass m rests on a rough fl.oar. Coefficient offriction P".
,between the block and the fl.oar isµ.,
(a) Two boys apply force Pat an angle e to the horizontal.
One of them pushes the block; the other one pulls. Which, and
one would require less effort to cause impending motion· µ,mg µ
P=-------
of the block? cos0m +µ, sin0m ·
,Cb) What is. the minimum force required to move the block byj 1
pulling it? i cos<j>, = - - - -
(c) Show that if the block is pushed at a certain angle 0o, it': J1 +µ;
_ . canJ!pLb.e _rr,Qved_wh<I_teyq _tltf _yalu~ of_l' be. _ µ,
and sin <I>, = -~'--
Solution:
(a), _Equation~_ f~r_ pullin; _!o~ce : Ji+µ;
P sin Bp P=· µ,mg
N ... Therefore
(l+µ;)/J1+µ;
! aI
_...,__.,_p·ro: r
µ,mg
= -"======
YLi J1+µ;
mg mg x I (c) From (a), pushing forcePrequired to move the block
Fig. 2E.65 (a) is
µ, mg
P=-~~~--
From conditions of equilibrium, cos0-µ, sin0
Ux = P cos 0 - µ, N = 0 ... (1) When cos 0 - µ, sin 0 = 0 or cot 0 = µ,, the force P
Uy = P sin 0 + N - mg = 0 ... (2) tends to infinity, i.e., the block cannot be moved. Secondly,
On eliminatingN from equations (1) and (2), we obtain force P must be positive so that it remains the pushing force;
therefore cot 0 :c, Ois the required condition.
p = -~µ~,'-m-'g'--
cos 0 +µ,·sin 0 ~.J;,~91E:(3;1g~
Equations for pushing force : From conditions of
equilibrium, A block of mass m rests on a b;~~ke; ~J-mass ~ -Th~;
Ux=Pcos0-µ,N=O ... (3) coefficients of friction between block and bracket are µ, and
·µ k . The bracket rests on a frictionless surface. What is the
UY = N - P sin 0 - mg= 0 ... (4)
maximum force F that can be applied if the block is not to
On eliminating N from eqns. (3) and (4), we obtain slide on the bracket?.
1
p = -~µ-','-m-'g'-_
cos0-µ, sin0
which shows that latger force is_ required to push the
block. Note that normal reaction in case of pushing is
greater than that in case of pulling force. Consequently
friction force is increased. F!~ 2E.66_ (_a)_ _ _
(b) The body is in equilibrium under the action of three
forces: applied force P, total reaction R and weight mg. Solution: Block and bracket must have common
acceleration in order to move in combined form. For the
Pmin I

_fil
I system shown in Fig. 2E.66(b) , Fis external force; therefore
block and bracket will have acceleration in the direction of E

~-~:·:
',~ I

'
" P><<!,,mg I,~sR -+
mg
µsN ~ ,, I - '
\. I
mg Direction of R ·~ ,
\
'
m
M ,,
..,,"II,
Fig. 2E.65 (b) - mg N mg '-
~----·-- I

www.puucho.com
_ Fig.2E.66(b) ________ ___ --·-·-
------
Anurag Mishra Mechanics 1 with www.puucho.com

From Newton's second law, or P = mg


Equations for block: max. sin e - µ cos e
LP,·= µ,N -F =ma ... (1) Case (ii) Impending motion downward : In
. LPy = N - mg= 0 ... (2) this case friction force acts upward.
Equations for bracket: :r.F, =N-P cos8= 0 or N =P cos8
LP, = 2F - µ,N =Ma ... (3) LPy = P sin·e + µN - mg= o
. , LPy=N'-N~Mg=O ... (4) or P sine+ µP cos e - mg= o
On solving eqns. (1) and (3), we obtain or p. = mg
F mm. sine+µ cose
a=---
M+m Therefore the block will be in static equilibrium for
From eqn. (2), N = mg , mg ,;;p,;; mg
On substituting a and Nin eqn. (1), we obtain sine+µ case . sine-µ cose
F = (µ, mg)(M + m)
(M + 2m)

l:,E,x(l:_~m:J e~ 67 ~d>
.....:::.:.::~ > )~£?~~! ~
· a
l..<1 blockis ke'pfiln rough mcline \.I/hose a~1e ofinclinatTa;;
~- lg,:eater· thcin;ang;le of repose'. . ' ·. · ·
lA block of mass 'm' Is supported oit arough wall by aj,p(yjrig a '(a) Find the minimum and m<Vf mum fore~ F applieq pc1rnllel
r
. ,~~~im::~•' · . •.
!,ore~ P asshov,m inFig. 2E:67 (a/Coeff!.dentof static.fri,ctioTJ, to incline. "t.hat will keep. if i!} ~quilibrium. . . ·. ;) ,,
!between blocJs:,and iyall isµ,. Eopyhµtrange ofvables,of:P; fb) What is the ,required force if it is applied 'non71alto_<the
incline~:";,. · '\ ·- ,,.· '}
(c) vyhat !s,the ~ange ofF if zt. is~applfed horizontally 9/Jc/he
· · block?:cL:. .. . ·· · '·' · ____ , '\' ·

,; ·in
i
".·1,
p
Fig. 2E.67 (a) , ~
.(c)

~
F/4,s
µN

.
--------·- ""'-~:..-~==-- ----------·· -,.··1,,J
Solution: Impending state of motion is a critical . /4~f
-~e mgcose
border line between static and dynamic states of body. The i (•~~e_ _ ___,
block under the influence of P sin e (component of P) may iLl-.-~--Fi_g._2_E--6~----L
I ,
.
have a tendency to move upward or it may be assumed that
P sin 8 just prevents downward fall of the block. Therefore Solution: According to condition of problem, the angle
there are two possibilities: of incline is greater than the angle of repose; therefore the
Case (i) Impending motion upwards : In this i:ase block will slide downwards. An external force can keep it in
force of friction is downward. equilibrium. We will consider two cases:
- . . . . • ,.'
I ~' (a) Case (i) Impending motion downwards : In
l
! this case force F first prevents the· block c--::--
y ~ . 'Jfx_-., -.

~
from slipping downwards. This is the N ·V ·
· _.··•~ N
. minimum value of required force ·R

l.·
~Y-
Friction force acts · upwards. From
IP cos o x . -P-c~qs°"·~··. l. mg conditions of equilibrium,
I , -mg,
, µN I LP, =F+µN-mgsin8=0
I P sih,{)
... (1)

l ---~------~~~---··--
Fig. 2E:sr (bl
LPY = N - mg cos e = o ... (2)
..~ From eqns. (1) and (2), we obtain
From conditions of equilibrium, Fmm. = mg (sine - µ cos 8)
:r.F, = N - P cos 8 = 0 or N = P cos 8·
LPY = P sin e - µN - mg = o
or Psin8-µPcos8-mg=0

www.puucho.com
Anurag Mishra Mechanics 1 with www.puucho.com

.. - . ·1
_-- _,.177 _,
Case (ii) Impending motion r· -- - - - -· - · I Ll'x = Fcos e + µ,N- mg sin 8= 0 ... (1)
upwards : In this case, force F is large =N =o
enough to just push the block upwards.
. ~YVf: LFY + F sin e - mg cos e ... (2)
This is the maximum value of required : ~
': From eqns. (1) arid (2),
force F. Note that friction force will I c}<:.-- ~ .1' F . = mg
(sine-µ, cos.8)
reverse its direction. From conditions iI' "'"' . ~ ':
'I, ' mm. (cos 8 + µ, sin 8)
of equilibrium, l_ Fig. ~-68 (c) I Case (ii) Impending motion upwards : When
Ll'x =F-µN-mgsin8=.0 ... (3) force F is increased the block has a tendency to move
upwards. Therefore friction force changes its direction to
Ll'y = N - mg cos 8 = 0 ... (4)
downward. From conditions of equilibrium,
From eqns. (3) and (4), we obtain,
Ll'x =Fcos8-µ,N-mgsin8=0 ... (1)
F"""' = mg(sin e +µcos 8)
Ll'y = F sin e - mg cos e + N = o ... (2)
Therefore the block will not slip if
From eqns. (1) and (2),
mg(sin e - µ cos 8) ~ F ~ mg(sin 8 + µ cos 8)
Note that when force F is increased from its minimum F=mg (sine+µ, c~se)
case-µ, sme
value the friction force is reduced from its maximum value
µ,N .. When F equals mg sin 8, friction force is zero. Block Thus the range of force P for which the block remains in
will have a tendency to move upward only when F equals its equilibrium is
maximum -value. Static friction is a variable force; its mg (sine-µ, cos8) ~F~ mg (sine+µ, cos8)
magnitude can change and, as the example illustrates, even cos8+µ, sine case-µ, sine
its direction can reverse.
(b) From condition of equilibrium, ~~~E,Kq.ta~C:>
Ll'x=mgsin8-f'."0 ... (1) ' -;--·-····------"· '
'A wooden block" slides down the right angle channel as shofvn
Ll'y=N-F-mgcos8=0 ... (2) )in Fig. 2E.69 (a). The channel is inclined at an angle 8 w.r.t.
Note that, due to external normal force the normal :the horizontal. The,angle a is 45°, i.e., the channel is oriented
reaction increases, thereby increasing friction force. •symmetrically with the vertical If the coefficient of friction.
Therefore for minimum force F we must have maximum ,between the block and the channel is µ k, find the acceleration
friction force µ,N. ;of the block.
Thus mg sine= µ,N
or mg sin 8 = µ, (F + mg cos 8)
or F n,;~ = : (sin 8 - µ cos 8)

(c) Case (i) Impending motion downwards: Fig. 2E.69 (a)


Block has a tendency to slip downwards and external force
just prevents it from sliding. In this case Fis minimum. From · Solution: The block is kept symmetrically in the
conditions of equilibrium, ____ . _ _ channel, therefore normal reactions on both its surfaces are
equal in magnitude. If the channel had been on a horizontal

~
Fnction force
-<'.J';· N is upwards surface, the reaction would be vertical_ [see Fig. 2E.69
"(".)..., N µ for i~pending l
'- F~ motion down , (b)(iii)J, since the charmel is inclined to the horizontal

~
surface. Net reaction is normal to length of channel AB.
I mgcos0 ; Since the charmel is symmetrical,
I '< ,},,."' µN I N 1 =N 2 =N
I ,,p, 0
Net reaction 2N cos 45° = mg cos 8
'I
I Frtctioll force is dowOwards
I for impendiilg motion up

I Fig. 2E.68 (d)

www.puucho.com
Anurag Mishra Mechanics 1 with www.puucho.com

-MECHANICS~ij
From conditions of equilibrium,
y

x.J

mg Fig. 2E.70 (b)


(I) - (Ii)
For blockM:
N2 ,-·-N.1 cos'45°t N2 cos 45°: Il'x = T - Mg sin e - µ ,N = O ... (1)
N1 N j ITy = N - mg cos e = o ... (2)
For block m:
ITY = T - mg = 0 ... (3)

B
From eqns. (2) and (3) we substitute values of N and T
A in eqn. (1) to obtain
mg hlg cos B mg=Mgsin0+µ,Mg case
Therefore maximum value of m = M(sin0+µ, cos0)
(iii) (iv)
Case (ii) Impending motion downwards of
Fig. 2E.69 {b) block M : In this case friction force acts up the incline.
N=mgcose From conditions of equilibrium,
For blockM:
-/2
From Newton's law, Il'x =-Mgsin0+T+µ,N=O ... (1)
mgsin0-2,!N=ma ITy =N -Mgcose = 0 ... (2)
. (mg cos 0) For blockm:
mg sm0-2µ----=ma
-/2 ITY = T - mg = O ... (3)
or a= g[sin e-:- -/2µ cos 0] Now we substitute N and T from eqns. (2) and (3)
-E-xa -:. ·.-e.- r-;;;1--.,
b~,__~--,_,-i':,@J~-±c::: "; - ~
respectively in eqn. (1), to obtain
mg+µ, Mg cos e = Mg sine

~
1
~:~~:r~;ti~y.-~-~-- -,,a-l~~cd~~:e~~~!hi:~u~~~~
an angle e ii,tth the hortzontql and.'l(l' ~ luinging vertical()> asi
or m = M(sin 0-µ, cos0)
Therefore the blocks are at rest if
M(sin0-µ, cos0),-;m,-;M(sin0+µ, cos0)
shown tn .Eig. 2E.70 (a). 11,e
co_effic/ent'of static friction' ··- - --· ,_ .
~J=~.2€5i!,~P,_\~_ ,:
~?-,.._
71 Ji,-->
'

between 'M' and _the tncline isµ,. 'Find the minimurn an<i 0

_maximu_myqlues of/m' so thatt_hef____sys(e_'f'"is at rest. ·


I
!r;;,o bl~cks-arekept 0~ ~n incline in contact with ea;;, othe;.:
,Masses of blocks are m1 and m 2 and coefficients offriction are,
'µ 1 and µ 2 respectively. The angle of inclination is e.;
"Determine:
m I
/ '(a) acceleration of blocks, and
·! '(b) force F w1th which the blocks press against each other.
e I

L_ --·----·- .__ Fi~~~-~-- _L ·---- ~-----j


Solution: The block of mass M can have a tendency to
move downwards as well as upwards. It depends on relative
values of masses m and M. If M is heavier it tends to slide Fig. 2E.71 (a)
down, and if m is heavier it tends to move down.
Case (i) Impending motion upwards of mass Solution : It is not clear whether the blocks slip or not.
M: In this case friction force µ,N is down the plane. So we arbitrarily assume that both the blocks accelerate
www.puucho.com
Anurag Mishra Mechanics 1 with www.puucho.com

FORCE ANALYSIS -- - --. - --- 179 I


~--------------------------------- - -------- - - " - - - -----~
downwards. Contact force between m1 and m 2 is R; it should Solution : Step 1: Draw free body diagram of all the
not be negative or zero. Contact force between two bodies blocks.
reduces to zero when the bodies are separated. Step 2: 1iy to identify the cause of motion of blocks on
1··-----·- - - ----------
·y which force is not applied.
Block A moves due to static friction. When slipping
starts it is f, mu = µ,mg. This force must be greater than
µ,N1 \ tension T, only then it accelerate forward block C moves due
to tension, Twhich must be greater than/the static friction
between C and D. Block D moves due to f
- - -
--~ µ,½_I
!Trµ,mg . Lf--J -F,

Fig. 2E.71,(b)
mg Mg !
·--- - ·- --- .
From Newton's second law:
Block 1 :
Ll'x =m1gsin8-R-µ 1 N 1 =m 1a ... (1)
Ll'y = N 1 - m1g cos 8 = 0 ... (2)
Mg
Block 2:
Fig. 2E.72 (b)
Ll'x =m 2 gsin8+R-µ 2 N 2 =m 2 a ... (3)
Ll'y = N 2 - 11 2g cos8 = 0 ... ( 4) From FBD of block B
Now we substitute N 1 and N 2 from eqns. (2) and (3) in F-,t,mg =Ma .. '(1)
eqns. (1) and (2) respectively. Now add eqns. (I) and (2) to From FBD of block A
obtain µ,mg-T=ma ... (2)
a= (m 1 + m2 )g sin 8- (µ 1 m1 + µ 2 m2 )g cos8 From FBD of block C
m1 + m2 T-f=ma ... (3)
From eqn. (1) we obtain R. From FBD of block D, f=Ma ... (4)
R = (µ 2 -µ 1 )m1 m 2g cos8
from eqns. (3) and (4), T=(m+M)a ... (5)
m1 +m2
which shows that ifµ 1 > µ 2 then reaction R comes out a=(m:M)
to be negative, which is impossible. It also implies that
putting T in eqns. (2) from (5)
blocks have separated.
µ,mg-(m+M)a=ma
[ -- - -- - - ° C •• i-·-1 ·--
;_~~?1'~~pJ~.;, 72 ~ µ,mg
=a
----- - - --- - -- -- ------·1 (M+2m)
Four blocks are arranged on a smooth horizontal surface as!
1shown. The masses of the blocks are given (see the diagram). I putting'a' in (1) F-pmg = µ,Mmg
(M +2m)
_,The coefficient of static friction between the top and the,
·bottom blocks i§ µ ,. What is the maximum value of the', F = 2µ,mg( m+M)
/horizontal force -F, applied to one of the bottom blocks .as; 2m+M
:show!', thilt makes all four blocks move with the same! (,E,Xei'?'c',l".".f;e•t_7_3 ', ,.__-_
i
\acceleration ? __ _ ___ _, ____ · ____ -, ~,=-,-~'!3!,le;,,c,,.s,
,- - -- -, - -
~
. . -- ------,
/ :MBJ MB=r: ,
. . . ..
'A car begins from rest at time t = 0 and then accelerates cilongl
la straight track during the interval O < t ,:; 2s and thereafter 1
1
'.with constant velocity as shown in the graph. A coin is;
! 'initially at rest on the floor of the car. At t =1 s, the coin I
Fig. 2E.72 (a)
- •• _J
i '·begins w slip and it stops slipping at t = 3s. Find the!
.s-.oefficient of static friction bctwren the floorpnd tlze cqi11•. J

www.puucho.com

Anurag Mishra Mechanics 1 with www.puucho.com

11ao ~-~··-------

,.
l1s~--- N

j~ Parabo
. ' z.
lj co."'o'-J-;;--2;--<3i---'4-+ icsJ
I Fig. 2E.73 (a)
~ - - - ~---···--'<.'--·""'···- -- -,.,-· (b) (c)
Solution: Fig. 2E.74
r·-·-- . ·-· ---------·
i Concept: What IS ':~11_se_ of acceleration of coin? (B0-72)g = ma;;
Friction force accelerates it, when slipping starts.
µ,mg=maora=µ,g
Given th~t graph is parabola having
vertex at origin then function of velocity
w- a'
now
or
ay = 1mJs2
ag = asin37°= lm/s 2
a= 5/3m/s
2

Now apply Newton's second law on man is direction of


is. Fig. 2E.73 (b) acceleration. Note that x component of acceleration of man
V = ~t 2 is due to friction.
at t = 2sec.; v = Bm/s we have mg sin37°-µmg cos37°= m x (5/3)
B=k-4 :; k=2 6-8µ = 5/3
dv 6-(5/3) = 8µ
V = 2t 2 =} -=4t
dt µ = 13/24
the coin slips over floor if

Thus,
ao = µg
µ = ao = 4 x 1 = 0.4
. . . . . . . ,---:::::7
~-.'.fil~~!I'PJ~ j 75 1>
g 10 -In the figure shown, the static friction .coeffici;n; betwe~;~u]
contact surfaces is 1/2. What minimum force applied leftward;
~~~?~fr 'on block 1 will move the system ? Repeat problem if tbe force)
is now applied on block 2; _______ •
i - - - - - - . - - - ·', - - · . - - - - - -

iA man:;of m~§ •eyo kg, stands on a hqrizontal weighing.


machine, ofi'!eg{lgible mass, attached to a massless platform P.
-~i
that slides do111ri ·at 37° incline. The weighing machine read ~
72 kg. ,:nan is ci/w;;ys~_
a£. r_esP_t _w.r. t. ·weighing machine. Fig. 2E.75 (af
\ . "'
Solution:
I , Step 1: Calculate maximum friction force that acts on
all the rough surfaces.
' ;:, _. ·1
l' Step 2: Check the tendency of motion of each block,

!l . l •
Fig. 2E.74 (a)
static friction opposes that. When slipping just begins f, is
maximum.
!Calculat,e : , . Note that due to string constraint both the blocks'will b~
at the verge of slipping simultaneously.
( a) The vertical- acceleration of the man ,
'(b) The coefficient of kinetic frictionµ between the platform Case I: f 1,- =0.5x3xl0=15N '
l_____q_n(IJnriline,_:__ . _ ___ _ · f 2smax = 0.5x2xl0= ION
Solution: Weighing machine measures normal from FBD of 1 kg
reaction. Draw FBD of man. System of man and platform F = f1 smax + T + f2 smax = 15 + 5 + 10 = 30 N
h& e acceleration at an angle of 37° ax and ay are x and y from FBD of 2 kg
components of ·acceleration. What is cause of vertical zr ". f2,m~ = 10
acceleration and horizontal acceleration. T=5N

www.puucho.com
Anurag Mishra Mechanics 1 with www.puucho.com

I FOiCE ANALYSIS -- -- .. --- . - '" .


181]
Note: _ _:__:__ _:_--_:_·-:..:-:.:·:::.- ..:·===========
First decide whether there is slipping between blocks or not.
2kg If blocks have same acceleration then friction force
f2,
1kg between blocks must be less than µ,N.

f1, Let blocks move together with common acceleration a


(b)
6mg - f 2 6mg - 3mg
a= =-~-~=g
N1 3m 3m
2 kg f2, then for upper block
f2, T T-f1 =ma => 3mg-f1 =·mg
f1,
20N 10N => f, = 2mg
(c) but f 1 ,;; limiting static friction but here f, is coming out
Fig. 2E.75 to be greater than µ,N. ·
Case II: From FBDof 1 kg T = f 2 smax + f 1smax = 10+ 15 Assumption of no relative motion between blocks is
incorrect that means there is relative motion. Therefore f 1 is
Fmin = ZT+ f, ,m~ = 2x 25+10= 60N kinetic friction.
3mg-µmg --2g
~-~T-;;, N1 (towards right)
F m
2T 3mg+µmg-3µmg .
-~~~~~~ = g/2 (towards nght)
12, 2m
Applying pulley constraint to get acceleration of hand
+
ap = a1 a2
2
acceleration of pulley= acceleration of hand
am +a2m 2g+g/2
ap =~-2~= 2
A block of mass m rests on top of a block of mass 2m which ls
·kept on a table. The coefficient of kinetic friction between all = 5g / 4 (towards right)
,surfaces ls µ = 1 A massless string ls connected to each mass
.and wraps halfway around a massless pulley, as shown. r- ~.~p~e} .!:: ..:-1?!;.>
'Assume that you pull on the pulley with a force of 6 mg.
What ls the acceleration of your hand ? A 4 kg block ls placed on top of a. long .12 kg block, which is
F=6mg
accelerating along a smooth horizontal table at a= 5.2 m/si:
µ=1[mJ - under application of an external constant force. Let minimum;
'µ=11 2m ~ coefficient of friction between the two blocks which Willi
prevent the 4 kg block from sliding ls µ, and coefficient of
Fig. 2E.76 (a) friction between blocks ls only half of this minimum value. of,
(i.e., µ/2).Find the amount of heat (in joules) generated due•
Solution : The free body diagrams both the blocks are: to sliding between the two blocks during the time in which. 121
kg block moves 10 m starting from reg,

~~.
N2~
~
f11
11
= 5.2m/s
r"l'.'.":J'---'-, a 2

12 T I 12kg 1--
smooth.___,__- - - - ' · - -
mg N 1 2mg Fig. 2E.77 (a) ___ j
(b) (c)
Solution: First assume that blocks have common
Fig. 2E.76 acceleration, for both block to move together acceleration of
4kg block must be 5.2 m/ s 2
f 1 is force of friction between blocks
f 2 is force of friction between block and 4kg 1 ·
ground. 4
I' ,'-"=1--1-2..::kg....=ll:;-b+f a = 5.2 mis~-
from FBD of pulley, we get
T = F/2 = 3mg Fig. 2E.77 (b)

www.puucho.com
Anurag Mishra Mechanics 1 with www.puucho.com

- a= 5.2m s 2
f= 4x5.2
I rt:;~·:·~~? b··:;· ·-,~. --.-- ·7 ·
µ 0 mg = m(S.2m/s 2 )
· µ 0 = 0.52 I.~--. .:--r·, .,.., '· · ,. '·1 !I
Ifµ= .!co.52)
2
due to friction
.
= 0.26 the

.
.
acceleration of 4kg block is

' ' 2
L
11 + 12 = constant
_:c!''!l· ~~,,?~_!b!
a
. :

a 1 = µg = 2.6m/s
As there is relative motion.between blocks we apply s,,1 Ii+ 12 = 0
Sre1 =u,,it + -1 a,,1t 2 z;+z~=o
2
a;<I = a 1 -a= 2.6-5.2 b+ /4 = 0
= - 2,6 m/s 2 , From which we get .a = b = c
1 2
Applying Newton's Law on.block A
sre, = -(-2.6)t Mg-T=Ma ... (1)
2
Time of motion can be determined from motion oflower on block B
block· T-T1 -µmg= ma ... (2)
For 12 kg.block ~ = .! (52)t 2 = 10 (given) on blockC
2 T1 ~µmg= ma ... (3)
=-Sm sre,

~~:,'i ,~
solving eqns. (1), (2) and (3), we get
work done by friction is given by.
w1 =µmgS,., 1 = 0.26x4x lOx (-5) = -52J :r ~~
l
µ~ ",. .•. -:.,, ·:. -~~-o,
Heat generated = 52 J ·
~,,.,~..,--.-rat·-,.,
e-e><.~~:~e ~ 78 ;;;'1>
!:Er"'=~,.::.,_,,;_ ·;;;_~·,".....-""'"-V
*A
I . ,
a•T
, ~ · T ~ , ,"
,..,__
,,T, , ~ , ~ µ m g
r:' ,·, ' '"• a__ J
an{c\~;;J
• ,

Given t/J7.Ia;·sho;v;, .in °the figU;,/


Bliicks~A, B [: t,19, ' . . • ' . ' ; .
masses m,1. "'M·& mB = rnc.1" ni. TI1e strings are q5symedi L.._.~.:.-~-· fl~:.;~E-?8_\':l, ~- --~-.:.. .
massless and 11v.itretchable, 'iJ.11il t/iii pulleys frjctionless:)'11.efe ·
is no frict;.on l>'.etween blocks B qri:d (he support table, bu"ttherel a=(M-2µm)g
0 M+2m
is frict/o~ /zetivee,; qlocks -p:~ncl,\i/denoted; bY,,<1::giveni
coei.;
'.JJ ienfµ. • . . ,.
0
U., ,
.· • . ; , ·- . •: «-
_w/-;~ - - -·~ • --- •
,. .·- • ~ . ;·. ,·:i.
w eC ;,\: , , , ··1 j
putting 'a' in eqn. (1), we get

·, • ·. ,·.:]
.., ~{~·
mC . !::,::;~::::.: '.
;j ··,: -,.--.\q,:-··,1·
Mg-T =M(M -2µm)g
M+2m
, m B ';:: .. , ,''
,' •:,_ \· r ::···,; _·: ' :·
T = 2.mMg(l +µ)
• . .: 1 . M A I ·. : ' • :J (M+2m)
(b) As there is relative motion between blocks we
apply

ta) ~:~:io·~.$.,·~.t~:!~~.~t?E.i~.-.·.·~~.-.i·.:t~::~~~i.!.·.·~.R.~.~·;~
'(b) SupJJOS~ ~he syste(n 1..< relea.sedfror1gestwith b!ockC heajJ
v~l = v:el + 2arel srel
If system is released from rest, u,.1 = 0
I .. _the/,g.ht end of blockB,.as·s1ibik,rin the above'figjlre. Ifj v;el- =

!,
2arel.srel
the.le,ng~.1.1'+ of. blb.ckB. is give·n·; w·h·.··a.tis tl.,e· sp_.jed. of·,.·. .b!iJc.k Of, Vrel = ~2arel Srel
· · C. 1;1.s' i( '!'~aches th~ (eft end.pf-block B? Treat,,siJ!~,oj\C
i. 'small: , J . : · ,,; ' , · ·. · , ,;;,,· , arel, ·=·.2a·
'(~) if t/zci Fri~$ of blq~k ;t';:, z;;;ss th~,.; some critical vi:zltii!, the S,.1 = L- - - - -
I ..blocks will not.'accelerate1whefr·relea.sed from" rest.,Write . v = , I,4g--"-L('-M_-___,2µm--")
t__.<;/9.wJ1.q ezyr_i§~jqn_for_thaf41nca,l_111C1§~.~ ·. ,~ ~- ..2:"· ·· (M +2m)
Solution: AppJy·constraint equation on strings, length (c) If blocks will not accelerate, then
of strings is canst. Differentiate. twice to get relation Put a = 0 in express in of a to get
between acceleration. Let acceleration of blocks A, B and C
M=2µm
be a, b and c respectively. www.puucho.com
Anurag Mishra Mechanics 1 with www.puucho.com

FORCE ANALYSIS 183'

l2c.S-=K9.~R.1.C?- , 79 :__.;-

Board A is placed on board B as shown. Both boards slide,


without moving with respect to each other, along a frictionless
horizontal surface at a speed 6 m/ s. Board B hits a resulting
board C "head·on". After the collision, board B and C stick
together and board A slides on top of board C and stops its
Fig. 2E.80 (a)
motion relative to C in the position shown on the diagram.
What is the length (in m) of each board? All three boards Solution: For equilibrium of block B
have the same mass, size and shape. The coefficient of kinetic
friction between boards A and C and between board A & B is y
0.3. ·~"71,Y ftl:gN
L . ~ 4 s · Lx ... ··• Lx
--+-
Before
v=O

Fig. 2E.79 (a)


-After
and
Fig. 2E.BO (ti)

IFy = 0
µN

Solution : mg+ µN = .!!_


..fz ..fz
Concept: Initially block A slips on block B and C. N = ..fzmg
Finally A and C have common acceleration apply 1-µ
For equilibrium of block A
IF = F-.!!_- µN = 0
X ..f2 ..f2
or F=!!..[l+µ]=mg(l+µ)
Block A: µkNk = mAaA o-v ..fz (1-µ)
Block B: µkN A -µkNC = mag, ~~~f-+v/2
= 0.6 X 10 (1 + 0.4)
Thus are1 = 4.5 m/ s2 (l-0.4)
Fig. 2E.79 (b) =6xl.4=14N
VreJ = O; 0=~-4.5t
2 0.6
6 2
t=--=-sec.
2x 4.5 3
For block A, aA = -3m/s 2 ; A carriage of mass M and length l is joined to the end of a
v 1 = 6-2= 4 m/sec slope as shown in the Fig 2£.81 (a). A block of mass m is
Apply V~t =u!1 +ZaretSret;
released from the slope from height h. It slides till end of the
2
carriage (The friction between the body and the slope and also
0 2 = ~ - 2 ( + 0.45) (L) friction between carriage and horizontal floor is negligible)
4 Coefficient of friction between block and carriage is µ. Find
For v = 6 m/sec ,minimum h in the given terms.
L=lOm m
r:..:·l;;~gmp)~i
-·-· ·. --.BOV
A side view of a simplified form of vertical latch Bis as shown.
:F. . . . "'=,---,
M
,...,_~~..-, smooth
The lower member A can be pushed forward in its horizontal
channel. The sides of the channels are smooth, but at the Fig. 2E.81 (a)
inteifaces of A and B, which are at 45° with the horizontal,,
there exists a static coefficient of frictionµ = 0.4. What is the'
minimum force F (in NJ that must be applied horizontally to
(a) µ(1+:)z (b) 2µ( l+ : }
A to start motion of the latch B if it has a mass m = 0.6 kg;
question ·
_ (c) µ(2+: )z (dJ.µ(1+~J1
www.puucho.com
Anurag Mishra Mechanics 1 with www.puucho.com

·L184 MECf,f/\Nl(Ssl
----..---~·--· ----· -------'-----~ '

Solution: Solution: Most important concept here is that man


Concept: Block slips relative to carriage, use relative, ' moves slowly. Slowly means, always in equilibrium
For the man,
motion equations of kinematics.
N+Tsin8-Mg=0 (vertical)

~
+-·,mg

Fig. 2E.81 (b)


F-Tcose = O (horizontal)

velocity of block, just before reaching carriage


_Fig, 2E.~3_ (bL__,
j Vo =~2gh
Now acceleration of block Maximum extension is obtained when static friction on
µmg man is maximum
a, =---=-µg
m For maximum extension, f = µN
acceleration of carriage For spring, T cos8 - kx = 0 (horizontal)
µmg
a2;;::-- Solve for T : T case = kx
M
~ T = kx/cos8
considering this moment as t = 0, motion of block as
seen from carriage Substitute for F and solve for N
U,el = Vo = ~2gh µN-Tcos8 =0
a" 1 =a1 -a 2 =-µg(l+ : ) or N = T cos8/µ = kx/µ
So, kx/µ + kx sin 8/ cos8 = Mg
Relative velocity of block when block moves through
or kx(l +µtan 8) = µMg
distance x with respect to carriage
2 µMg
2 2
Vrel = Vrel + arel X x=-~~~-
k(l+µ tan8)
when x=l,vrel =0

2gh = 2µg( i +: )z ~ h=µ(1+:)z -- '... - - --· ·-·------- --------"-~~--~.-----;


!Find minimum normal force to be. applied by each hand toj
;hold three identical books in vertical position. Each booJi hasi
---, :mass' m' and value of coefficient offriction between the l,ooks1
A man with mass M has its string'.attached to one end. of a: .as well as between hand qnd the bpok is µ. !
'spring which can move without friction along a horizontal, I
overhead fixed rod. The other end of the spring is fixed to a,
wall. The spring constant is k The string is massless. and:
'inextensible and it maintains a constant angle 8 with the,
;overhead rod, even when the man moves. There is friction,
iwith coefficientµ between the man and the ground. What isl Fig. 2E.83 (a) j' .
,the maximum. distance (in m ) that the man moving slow[y; '-------- ----~"'~"',-- - - ------ -·-=~
;can stretch the spring /?eyqnf!Jts_J!atyral length? Solution:
k '

--:~.M
I From FBD, for center book
2f, = mg
M f ,,; µN ~ 1,1N > mg i
- • 2 iN

µ
N>mg

... (1) 1J, '
(friction coeff,) 1, f,
For side book f - f, = mg __ Fig.2E.83(b) .,,_,J
Fig. 2E,82 (a)
f= 3mg :o;µN
2
www.puucho.com
Anurag Mishra Mechanics 1 with www.puucho.com

f-FORCE ANALYSIS
N;;, 3mg ... (2)

B / ~a :

By eqns. (1) and (2) --A--/~~i


. . N 3mg
Thus, M1mmum =- -
2µ Fig. 2E.85 (a)
..... ''
- -----·--··'
~>
. ..:-- _ - ........ -.-~~,

~E~a"~PI':- Solution: During upward acceleration


N , = a.1 .sin 37' • . a1
,When the system shown in the diagram is in equilibrium, the
,right spring is stretched by 1 cm. The coefficient of static:
'
::Jriction between the blocks is 0.3. There is no friction between[
I µN L0.!.:. 4a !
I

\the bottom block and the supporting surface. The force) mg a 1 cos 37q =~ \
,constants of the springs are lS0N/mand 450N/m (refer Fig,; Fig. 21:.~5 j~). .I
2E.84). The blocks have equal mass of 2 kg each. :
= m(
3 1
Find the maximum amplitude (in cm) of the oscillations of1 N - mg ; )
·the system shown in the figure that does not allow the top:
'block to slide on the botto111. ; µN = m( ~ 1
} on solving we get
450N/m
150N/m a1 = 15g m/s2
31
Fig. 2E.84 . Concept: When lifting arms accelerate up, caus.e ofj
Solution : Suppose origin is at the equilibrium position ,acceleration a1 cos37° is friction µN. And resultant force up1
and the direction of increasing x is towards the right. If the :is .(N -c 111g):,1chich causes acce!er;it!~n,J{i.sj.n}J_0,. _.
- -- - - - - --·· ''f
blocks are at the origin, the net force on them is zero. If the a2 cos37°=5
4a~ 1
·
blocks are a small distance x to the right of the origin, value
of the net force on them is -4kx. Applying Newton's second
law to the two-block system gives 37° ··71 I
82 +
-4kx= 2ma
Applying Newton's second law to the lower block gives
a2 sin 37°=,
Fig. 2E.85 (c)
k(x1 - x)- f = ma
where x1 = initial stretch and f is the magnitude of the FBD when Arm is in Deceleration
frictional force.
. Concept: During deceleration direction of friction force
f=k(x 1 +x)
,is towards left. Student is advised to ponder over a simple
The maximum value of x is the amplitude A and the question.
maximum value for f is µ,mg. Thus, µ,mg= k(x 1 + Amrucl· I.----- - .

Solving Amax gives ''Which force is cause of component of acceleration


A =µ,mg -x =3 a 2 cos37° parallel to surface."
max k 1

mg -N = m( 3; 2
)

'In the manufacturing process disks are moved from level A to:
B by the ·lifting arms shown. The arms start from level A withi Which on solving given a2 = lSg m/ s 2
lno initial velocity, moves first with a constant acceleration a,! 4a
'·-------..---···-···-~ ~
6-,~~~.~~~-~-·~
'as shown and then with a constant deceleration a 2 and comes:
,to step level B. Knowing the coefficient friction between disks
"- - --- --- - --- ·- - -- - --- -------·--- -· - 1
'.and the arm is 0.30, determine the largest allawable.
;In the Fig. 2E.86 (a) shown a constant force Fis applied on,
:acceleration a 1 and the largest allawable deceleration a 2 of
:lower /:,lock, just large enough to make this block sliding.outi
·the disks are not to slide. ' from between the upper block and the table. Determine the i
'force F at this instant and acceleration of each block. Take:
g_= l()_m/s 2_. • • •.. ______ - - - - - - - - · - · · _______ ... .!
www.puucho.com
Anurag Mishra Mechanics 1 with www.puucho.com

11as , / , f, >'

r:;1·;;,d ~2 (11)(~2>
<,,
II ~.s7 r·- - - - {
"
'
'
.•

I ' I '
i
I •
3
_:_S_o-lu-tio~-lt i-n-s;;:~::~~:~:::;:e_F_w_e-ap;y l 0
!
time force F = (2Ot) Newton. Plot a graph between
.
l
I, 5 7
t (sec)
acceleration of both the blocks .and time. I.(. , Flg::1E:as (c)
Let f1 =·force of friction between 5 kg and 15 kg block ,.. , . ______ -··---·-----~
and h = force of friction between 15 kg block and CIRCULAR MOTION
gronnd.
· Consider a string of beads whirled in a circle as shown in
Then, (f1 lmax = maximum static friction Fig. 2.44. Each bead moves along a different arc but sweeps
= (O.3)(5)(1O) = 15 Newton the same angle. If the arc length traced by a bead at a radial
(f1 ) k = kinetic friction distance r is I, then we define 0 as
= (O.1)(5)(10) = 5 Newton 0 = 1/r
similarly (f2 lmax = (0.5)(15 + 5)(10) = 100 Newton When 360°
I = r, 0 = 1 radian, 1 rad= - -
and (f2 h = (O.4)(15 + 5)(10) = 80 Newton 21t
Now when F $ lOON, the -,. ------------····""·- = 3600 = 57,30
system of block will not move. i, : ,' : @5) , 15 Newton 6.283...
In this case f 1 = o," i.e., f 1 [ 1 5 N e ~ _ ~ • Any angle 0 can be transformed in_to degrees by ·
starts acting for F > 100 I ~F ·e (radian) = --'---"----'-
--,--,--,- e(degree)
Newton. At the time of lso Newton ,....... a 2it (radian) 36O(degree)
slipping between 5 kg and· 15 l' " Fig. ze.as (b)
kg block f1 will be Cf1 lmax and - - - - - - · ........- . lllustration-7
f 2 will (f2 )k and obviously F > 100 N. Diameter of moon, D ~ 3.4 x 10 6• m
At this instant both the blocks will have the same Distance from earth, r = 3.8 x 10 8 m
accelerati,on. ,
~quations of motion are as u11der :
15= Sa
a=3m/s 2
F-95 =15xa = 45
F = 140 Newton
We saw that· a1 = a 2 = 0 upto the instant when
F = 10ON or t = 5sec.
Both the blocks move· with same acceleration,
a = F - 80 = 20t - 80 = t _ 4
. 20 20
till force becomes 140 Newton or -.7 second. After 7 If we approximate its straight line diameter as an arc
seconds acceleration of upper block a 1 becomes constant i.e., · length, then the angle 0 subtended at the earth l,y the-moon
1 m/ s2 while that of lower block is
F-8O- 5 0=i=D
a2 =
15 r r
.
2O 85 6
= t- = l,33t - 5,67 = 3.4 x 108 m = 0.009 rad.
15 3.8 X 10 m
The corresponding graph is as shown in figure. Diameter of sun (l)=l.4x10 9 m
www.puucho.com
Anurag Mishra Mechanics 1 with www.puucho.com

IFORCE ANALYSIS- -
Distance of sun from earth (r}= 1.5 x 1011 m
Angle subtended at the earth by sun is
9
S = _! = E_ = 1.4 X 10
r r 1.5 'X 1011
=- 0;009 rad. X
That is why the sun and moon seem to be of same size. '-------F_l=.g;__2:~6 '________ J
Average and Instantaneous Angular Speed
The angle 8 is measured w.r. t. the x-axis ..
When the beads move in a circle of radius r, the radius
sweeps angle 8, we refer to it as' angular displacement. After Acceleration in Circular Motion
taking 3 complete counter clockwise turns, 8 = 3 x (21t) rad The position vector of velocity and angular
rather than 8 = 0. Arc length 1 is different from vector velocity for circular motion : Position vector i(t) is
displacement, but we can take counter clockwise l positive ---- ---y -- -- - --- ' - ----·1
and clockwise negative. We can call it curvilinear ' . '
displacement. , v(t) - I
[ I
I

;, ~ l
'
B(t)
x(t) i
Particle

y(t)J r sin B(t)_ I

· _ ·x I

From figure,
where 111 = 11 - l; and ,118 = 8 f
If time duration is /J.t, then
- 8;,
... (1)
r
----~:::_ J
(t) = [r cos 8(t)]
Velocity of particle is
i + [r sin 8(t)] j
111 /18
-=r-
!J.t !J.t
... (2) ,I (t}= r ~ [cos 8(t)]i + r ~ [sin 8(t)]j
' dt dt
118
v avg. =· rco avg. where CO avg. = ~t = r[- sin 8(t)] dS(t) i + r[cos 8(t)] dS(t) j
dt dt
We may call 118 as average angular speed, angular speed The velocity vector is tangent to the circular trajectory.
!J.t
=
of 1 rev/s 21trad/s Velocity vector ,I (t) is perpendicular to the position vector
Instantaneous angular speed i/(t) at all times. Students can verify it by scalar product
00 = Jim 110 = _de ... (3) 1(t) - ,l(t), which is zero, independent of time t .
M...;O /J.t dt
and eqn. (2) becomes
dl d8
----,-=r- In circular motion the three vectors ,l(t), oo(t) and
dt dt
v = rm
r (t) are related to each via the vector product
Angular Velocity Vector ,I (t) = 00 (t) X t (t)
· Angular speed ro is the magnitude of vector called the In magnitude, v(t) = rro(t)
angular velocity oo of the particle. Direction of oo can be Acceleration of particle at any
determined from circular motion right hand rule. instant of time t,
Curl your fingers of right hand in the sense of rotation of
particle, then the extended thumb points in the direction of
...;
ro.

www.puucho.com
Anurag Mishra Mechanics 1 with www.puucho.com

·. MECHAN'iS~lB
Since the particle is in circular motion, the radius r is
Hence first term is cit(t) x i1 (t ), it is termed tangential
constant. If the particle is undergoing uniform circular
motion, acceleration.
de ' ~_~;:?/. '' . ' ·;.,~ ' '--+ ;~~:~.,]
-- = OJ = constant Concept: 1. The angularucceleration vector a(t)pointsl
dt .
in the direci{on of the change in tile angular ve(ocit;yye~torl
li(t)=-{· rdO[cos0(t)] de}i+ {r dOdt [-sinO(t)]- dO}J ,- . ~ :· ". ,. .
i6(t). If speed of particle increases or de~reases, the angllfor
:' ·-· • . I
dt dt dt 1
=-(d 8
2
) {[r cos ·e(t)] i + [r sin O(t)] J}
~locity vectqt-falso increases or decreases. - ·. ir ' !
dt . [ Dfrection 'of angular velocity vector is always normal tc;,
lane of rota'i;ion. Therefore the -angular acceleration vecfor
2
act)= - 1ct)
00
lot
I
ct) is.- direct~/i
' , <:
either' parallel to :ol(t)
"
when the part(~le
,
is.
Note.that term in { } is position vector i(t). Negative
sign indicates that the acceleration is antiparallel to the
~p
!speeding dl~ng t~e circle; or antiparallel to c6(t) w,he~;the-1
fparti_cl_e _isc5lp}ll_ing"'.'-:______ •.____.____ -----..-----, .
position vector i(t). That is a(t) is directed towards the
centre of the circle. This is called centripetal acceleration.
The 'magnitude of centripetal acceleration
. a, =ro 2 r=(v/r) 2 r=v 2 /r
Alternatively, centripetal acceleration can be obtained
by differentiating the expression
·. A )' c'"tJ;: ~ 4
,

a,=a(t)xr(t)
4
·-
I
Fig. 2.49
v(t) =0) X i(t) '-----:~---. ·-c . >"'·,~----, _ .. _ ·- l
2. At" ~n( instant tangential: _acceleration _is. ~-lwaysl
d--+--+ --+--+
a(t) = - (OJ x r(t)) = OJ x v(t)
tdnl!;ent to t/tecircular path andpa\dllel to v(t) ifthepdr,ticle
dt
or 'it,(t) = ol xv(t) = -00 2 1 (t) is speeding 'up and_ antiparallel to 'v(t) if the partic[; is
. 2
slowing dowij; .... ______ ..____ ----·--"··--· ___ ..
and 1a,Ctl I=~= ro 2 r 7
. r
Non-uniform Circular Motion and Angular
Acceleration:
The acceleration of a particle is rate of change of
velocity.
c
1
1l~ r;;}~
ac...___
For a particle speeding
_ .,_v_ , at a
~

~ For a particle slowing


up in circular motion
act)= dv Ct) I (a)
down in circular motion
(b)
dt
_, _, _, ____ Fig. 2.50
Since v(t) =ro (t) x r (t) ,..,
3. a, ,;,resultant acceleration • ·
a (t) = d OO (t) X r (t) + 0) (t) X d t (t) ~- .
I
Hence
dt
Note that second term in this equation is
dt .a, =' taT1gential acceleration
~ : '' .
_ a, ;"' centripetal accelemticin
- ' ,.'
,, I
,r-·- - ..::; ...- ........ ..
a, =oict) x vet)
,1 ~ V

where d1 Ct)= v(t).


dt
I .,
\
d ol (t)
In the first term is defined as angular ~,
dt - - - • a,
acceleration.
, Angular acceleration is rate of change of angular
· velocity. "
o1(t) = di6 (t)
dt

www.puucho.com
Anurag Mishra Mechanics 1 with www.puucho.com

Ir FORCE
L....
A1{ALYSI$ . ·.
. - ,,1., --
· :
::::::::-¢=-;:::::;:~~~;;:;.;:;;~~~:-:::::'':::-'::::'
: • : . ci .
·-=~~-_;.•"..:'..=· '-~-'---.;_:_c
e, = (cose) i + (sine)j
and e, = (-sin0)i+ (cose)j
Radius vector of the particle at time 't'.
-> • •
r = r[(cos0) i + (sin0)j ]
Differentiating both sides w.r. t. time, we get

dt= r [ -sm
- . 0 -1+cos-J
de • de "]
. ; dt dt di
II' ->
a,
i = roo[ (- sin0) i + (cose)j] ... (1)

, Fig. 2.51 (b)


-2+ -- . ----~--·-- [,
When a,. is )n direction of motiol! i.e. parallel to velocib-
vector speed of object i'!creases. :,
Centripetal acceleration chrmges direction of ,;elocity
vector. ,_ 1_,: j

' ' ' '


When ta11gential acceleration is opposite ·to veJoi:ilX
vector speed ·of_ o_bject decreases. · · · .. :
· Note that aligular velocity> vector, position vecton: cmd
tangential ac;celeratio/1 vector are -rtormal to each other. . ··
Total acceleration of particle a(t) is ;
< 4 ~ '~ ' ~ -:+
a (t) = ex (t)Xl'.. .(t) + OJ (t) xv (t).
Again differentiating (i} w.r. t. time, we get
-> ->
= a,(t)+a, (t)
di =ro>~{(,-sin0)i+(cose)j}
Total acceleration is vector slim of the two mlltu~llyj dt dt
P!,yen_d_icu_la!L!'!E.g~ntiq1: '!'!_d centripetal acceler'!.U.~:-~-~-J doo • . •. I
+r-{(-sin0)i + (cos0)j} . ,
dt
To Find the Angular Velocity of a Particle 2 do>
=--{J) r{(cos0)i+(sin0)i}+rdte, :I
i
with Respect to the Other as Shown in the
I
Fig. do> •
=- ( co 2 r)er+r-e,
'
i
. dt
From the Fig. 2.52, angular velocity of B with respect to
particle A is: Hence anet = -(oo 2 r)e, + (exr)e,
i,

r· '"'JL'·
j
I
A
.
. / :_
.
½·ll
.. B ,
I
where ex r is the tangential acceleration and
radial or centripetal acceleration.
lllustration-8
Consider a particle moving in the x-y plane according to
OJ
2
r is the · I
· I

; ____. _ Fig.2.5~ _______,' r = r(cosooti + sinooti), where rand OJ are constants. Find the
linear velocity of B w. r. t. tra~ectory; the velocity, and the acceleration.
A .l to the line joining them y
separation distance between them
(v 2 sin0 2 -v 1 sln0 1 )
-- ',,.,, :I;

l x=.rcoswf '1.v_
----~-~- '· \•, ' ·l
Unit Vectors along the Radius and the Tangent
Let us consider that a particle P is moving in a circle of
radius 'r', at any time 't' the particle's angular_position is 0. •.
'
oot
. .y~ r sin,oot
L..l:'_
I \'
I
) .. x)
Let e, and. e, denote the unit vectors along the ,a<!ical i • ' !
and tangential directions then from the Fi.g 2.53. ~----F_lg_._c·~~:~_4_ _ _~
www.puucho.com
If
Anurag Mishra Mechanics 1 with www.puucho.com

I190 MECHANICS-I I
Irj = [r 2 2
cos rot+ r sin rot]1/ 2 2 2
The acceleration is directed radically inward and is
Using the familiar identity .sin e + cos e = 1, 2 2 known as the centripetal acc~leration.
Irl= [r 2 (cos 2 rot+ sin 2 rot)J1/2 lllustration-9
= r = constant. "Iwo particle 'A' and 'B' are moving on the same circle
with angular velocities ro 1 and ro 2 respectively w.r.t. the
The trajectory is circle.
centre of circle. Find the angular velocity of 'A' w.r.t. 'B'
The particle moves counterclockwise around the ~ircle, when, ·
starting from (r, OJ at t = 0. It traverses the circle in a time T
(i) their sense of rotation is same,
such that roT = 21t, ro is called the angular velocity of the ' .
motion and is measured in radians per second. T, the time (ii) and their sense of rotation is opposite.
required to execute one complete cycle, is called the :
:
.,,, . ;
ro 1r sine/
'~, A :
period.
,-··--···- ·-- ' 'y
' !,·/\ .
,.,., ... - ---....,r
'\\
.~0/2 :
•:\J,e/2·r·
~-.. ':~
oj
2
}rsin0t2!

r . _
r sine12j
I

' ,/ '•.' I
,
I ro I I
1 ...____...- iB ro,r sine/2 J
1
·x . I
I
I I ___, ___ _ Fig:2.57 ______ !
\ I
,.._
.,..,/ Angular velocity of 'A' .relative to B (ro A/B)
......
-- --
(i)
VA/B
ro A/B =
separation distance between A and B
., , Fig: 2.55. · v A/B => Velocity of'A' relative to 'B' perpendicular to the
~- dr · ~-- ~ - - ~
v=- line joining them.
dt ro 1 r sin(0/2) + ro 2 r sin(B/2)
= rro (- sinroti + cosrotJ) ro A/B =
2r sin0/2
We can show that v is tangent to the trajectory by
calculating v · r : .

v · r = r 2ro (-sin rot cosrot + cosrot sin rot)


-------., , ~--.·-·--·--7
• •A .
(anticlockwise)

and
=0.
Since v is perpendicular to r, it is tangent to the circle

Iv I= rro = constant.
It
J
ro 1r sin0!2/.:
r/ :
·/J,i+-- ' . '
r··.. :
} i
r sin0/2 ;

r sinB/2
dv
a=-
J oo 2 r sin012·· •• :
dt I! •'B
= rro 2 [ - cosroti- sinrotJ] } ·
,.
ro2 r ~-
a,
= --co2r I ________F_:lg-J·~'!__ _______ __
. .r L
y I: ·y V .

--
I •
(ii) ro - .A/B
,/
. . ,. ,,,...--
" A/B - 2r sin(0/2)
I '\\ ,
. I ro 1 r sin(0/2)- ro 2 r sin(B/2)
I I
I ., t 1 ' .I
,• I
ro A/B = . 2r sin(B/2)
I I

I 1
\ I
I X I 1
I I
I ,x
\ I
, i \
,.._ I
....... __
I ' ---- ,./ 1,
I
!
......
-- -- _,./ If ro 1 > ro 2 , then ro A/B is in anticlockwise.
If ro 1 < ro 2 then ro A/B is in clockwise.
If ro 1 = ro 2 ,then ro A/B = 0.
1_.[ _-.~-_ _F_lg_.2_,s_s_(a_J_ _~J
1
_:_ Fig. 2.56 (b)
1
,..
www.puucho.com
Anurag Mishra Mechanics 1 with www.puucho.com

(!oRCEANALYS~ 191'
-------~
Equations of Motion Vy
and tan0=-
Case I : Constant angular acceleration vx
ro = ro 0 ±at (u sin 0 - gt)
= - - ~0~ ~
1 2 u cos 0 0
0 = ro 0t ±-at
2 Similarlyvand0can be determined in terms of0 0 andy.
2
(J)2 = (J)~ ± 2a0 Vy= (u sin 0 0 ) - 2gy

ro 0 is initial angular velocity vx=ucos0 0


ro is final angular velocity I; I =Jv; +v;
a is constant angular acceleration = ~~(u-s1-·n_0__)_2___2gy
__+_(_u_c_o_s_0_)2-
0 0
0 is angular displacement in time t 2
Vy ~(u sin 0 ) - 2gy
Case II : When angular acceleration a is variable tan e = - = ~ - -0- - - -
(a) if a= f(t) (function of time) vx u cos8 0
dro
a=-
dt
or J dro= Ja.dt= Jf(t)dt ,- -- -- - - -- --
;A stone is thrown horizontally with the velocity v x = 15 m/s. ,
(b) if a= f(0) 0 or f(ro)(function of 0 or ro) !Determine the normal and tangential accelerations of the i
rodro ~t9_n!l_ (11_1 s_ecol)_d_ after it 1,_egbls to_ mo_ve._ ___ __ ___ '
a=--
d0
Solution : The horizontal component of acceleration is
Jrodro = Jade zero. The net acceleration of the stone is directed vertically
Radius of Curvature at any Point on the Path downward and is equal to the acceleration due to gravity, g.
of a Projectile Thus a= g = ~a; + a;
Consider a projectile at any instant t with its velocity From Fig. 2E.92 we can see that X
vector v at an angle 0 with the horizontal. We choose
tangential and normal axis as shown in Fig. 2.59. - cos e;:::: vx = an ;:::: an
V a g
Component of g towards normal axis provides centripetal
acceleration. and
,.y I ' 8
-~ .
'- l \,
I \, V

Hence Vy. \ a::


n-axis ',, ~/
\
\
\
t-axis
v/
''
' ', , , , .
,, '
j, ', ~Ir--~ a
I gsin8
I ..-; a,
\igcosO
and ____ Fi~_-~E-~!- __
9 a =g!!.!_= gvx
1
I . . X'
n v ~v; +g2t2
L__ -- -- ___ Fi?:.2~~9. _________ ! On substituting numerical values, vx = 15 m/s, g = 9.8
v2 m/s 2 , we get a, = 5.4 m/s 2 and a,, = 8.2m/s 2 •
=P
a,
where p is radius of curvature of the trajectory at the
~~2$...,.~~J 88 ~
instant under consideration. Thus, ltt~lloon -s~ar~ risi~g fro,;_ th~ ;~rface of th~- ,"~,;h~itiil
v2 v2
p=-=--. !vertical component of velocity v 0 • The balloon gathers ai
a, g cose ihorizontal velocityvx = ay, where a is a constant andy is the!
v
where and 0 can be· determined in terms of vand 0 0 ;height from the swface df the earth, due to a horizontal wind. i
(velocity and angle of projection) and time t. :netennine I
vx =u cos8 0 (a) the equation of trajectory of the balloon.
vy=usin0 0 -gt. (b) the tangential, normal and ,tqtal accelercation of the:' ·
l;l=Jv;+v; [_ l?ailg_O_I! ~fun~tion ofy. · "': __________ -· ____:
Solution : (a) Balloon's vertical velocity is constant
= ~(u cos 8 0 ) 2 + (u sin 0 0 - gt) 2
and horizontal velocity is variable w.r. t. height y. So we have
www.puucho.com
Anurag Mishra Mechanics 1 with www.puucho.com

.. , ' .
·:·-v ::,. :f> i' ::.•'

Vx = ay; Vy =Vo Concept of Pseudo Force


dx dy
-=ay· -=·vo Newton's laws of motion are applicable in inertial
dt ' dt reference frame but not non-inertial reference frames. In
·:dy dy/dt '!o this section we will see show Newton's law can be modified
As -=---·=- ... (1)
dx dx/dt ay so that they work in non-inertial reference frame too! !I
On rearranging eqn. (1), we get In the Fig. 2.60 two observers, one on ground and the
ay dy =v 0 dx ... (2) other in a balloon moving with constant velocity, observe an
On integrating eqn. (2), we get trajectory as airplane.
ay2
--=VoX
1 : Position vector of plane in gro~nil reference frame.
2
1• : Position vector of plane in balloon reference frame.
or y 2 -(2v
- - 0) X I
I
a
(b) ·x-component of acceleration,
dvx dy
ax =~=a dt =avy =av 0
l
. dvy
y-component of acceleration, ay = - - = 0
dt
->
Resultant acceleration a = a)+ ayj = av 0 i
, From Fig 2E.88, : = tan 8; therefore
Flg.2.60
,, ->
R : Position vector of balloon.
,-+ -+ -+
f =R+ r'
-> -> ->
dr dR dr'
', .
',
or -=-+--
dt dt dt
... (1)
',
'.,.,a~'. L,
We- assign letters to each body: P, airplane; B, balloon; G,
·~ ..... ~ ground.
e· ......~..-n-axls -> -> ->
Vpa = Vpa + VaG ... (2)

If we differentiate above eqn. (2) again,


... -> ->
= ap8 ,
->
= constant .
1+(:r 1 1 a PG since v BG

cos8= ~ Therefore accelerations of a particle with respect to two


coordinate systems that are moving at constant velocity with
respect to each other are same .
ay . Now consider two reference frames as shown in Fig.
=-;========
~(ay) + v~ 2 2.61. .
Y Inertial y Non-inertial
sin8 = dy/dx = v 0 /ay reference r'efereflce
frames·
~1 + (v 0 /ay) 2 ·
2 !rams S •
~1 + (dy /dx)
= Vo Origins coincide
when r= Os
~(ay)2 + v~ o,,,--~-o.x
2
Tangential acceleration, a, = a cos 8 = a VoY
, . ~(ay) 2 + v~
z ·zi
- av 20 A non-inertial refereflce fr8me' S' a~elerating with '
Normal acceleration, a =asin8= respect to !ne_i;tlal frame S.
n . ~(ay)2 + v~ . ·,.,~. ' •'
Flg.2.61

www.puucho.com
Anurag Mishra Mechanics 1 with www.puucho.com

F'·FO~CEANALYSiS1' 1~~'":;'.",~;[t~,-. - ; , . .
-~-::::.fr, ~--::1 ''" --·: -~-"'.-~~~: "": ~ ~ ;.-;_.
- . , ..
-;1,,-"' ';;,

The tr~nsformati~n equati~ns .relating :the coorc:iinat~s According to the observer (non-inertial) riding in the car
of the particle in each reference frame are · · . the pendulum bob is at rest. The thinking point for him is:
1 2
...,
x = x' + vxut + - a,,,t which force has balanced horizontal component of T.
2
Equations for non-inertial observer:
y =y"
z =.z' E~--~ (:~"~~~
Acceleration of particle in inertial refe~ence fr~me Sis I·'
~ ' lnerllal .
_, d 2 X • . d 2 y o d 2z i'
a =--i+·--J+--K
I . ·obsel'VSri
dt 2 dt 2 . di 2 L.
. d2 · r 2 , d 2 y' , d 2:z'
=-

2
2
dt . .
(x'+vx 0 t+-ax 0 t )1+-2-J+--.k
2
2.
d x', . · , d y' • d z' 2
dt dt 2 ' [f ,•, .
= --. 1 + a, 1+--j +--k (al,
dt 2 O . dt 2 dt 2 Non-iriertial': 0bS8~er:1 ,
--+ . --+. -t . "
•s . ~ :.'
Therefore a =. a' + a_0
For inertial observer, ~xpression for.Newton's \ . 'l._' ,-
_, _, _, . . . .law
. is . . ·:0,",t,;,.:_ i-J't~
'- r a:
F,0 ta1 = ma + ma 0 ... (3)
', .::
··: f/ftictiiioUs:, ·:• ·
.
_c,:;'.-f:! .. ;; · • . JnQ,
For non-inertial_, observer
. _,expression will be_·
Ftotat = ma' ... (4)
~ .....
which is wrong, There is an additional term 'mao iri . ;;-·:, C!>i ,. : d
equation (3). ·
.'
· The additional term on the left hand side {-m a:i) is· T sin 0 - Fpso.,,io = 0
. l:Fx ".'
called a pseudoforce. ITy = T cos 0 - mg = 0
Ho;_.,ever, if we rearrange c (3) in the form · The non-inertial observer must obtain same
--+ .-t -t•
F1o1a1 + (-ma 0 ) = in a mathematical result as the inertial observer does, whtch is
.__.-, possible if
Real force Pseudo force
·~pseudo = _mainertial ;;;;; ma.
therefore in a non'inertial reference· frame, Newton's
Illustration 11 : Consider a block kept on a
law can be written as
_, _, frictionless turntable, connected to centre by a string.
F real-+ pseudo = m Ji' According to ground. observer (inertial. reference frame)
i.e., vector· sum of real forces and, pseudo forces on the block moves ;,long a circular path. Therefore it must have.a
centripetal acceleration provided by tension of the string.
system is m °ii' where. a' is observed acceleration of mass in From Newton's second law, ·
non'.inertial reference frame. Non-inertial i
!:·,· . / "-n,·':· 'r-~1:t"'..-'", i'..:.: ,;··~:.,.~:: h
Newton's second law_ can be applied ·by considering an obseiv6r' -'. ' '

pseudo (imaginary) force -m ~ o on the left hand side- oflaw.


Non-Inertial Reference Fram~
r~
,,,,--,
i r
! ;
...
Illustration 10.: Consider a pendulum bob in an
accelerated train car. Pendulum is inclined co vertical at an
·,
angle 0. According to an observer on_ ground -(inertial
··'.",9 l
·. ~ ~5-' ' " " ' . • ' . !
observer) the forces acting on the bob are: tension of string (·::~ ."1t ' • ,, ,:, , Inertial observer, ,,.~ •

'i and weight of the bob mg. ·The :icce1eration a is provided I,"· ~ (a) .., ' ' · , '(b) !
L ,, \.-'.·:,":. ,_ -Fig.'2,63:· ___ -,.,,.,;~~ ------~-~-
to the bob by horizontal· component of T, and vertical mv2
component of tension balances weight.;. T=-
r
l:F'x = T sin 0 = ma ... (1) According to observer ·(non-inertial) on the block the
l:Fy = T cos 0 - mg = 0 ... (2) · block is at rest. Since observer and block turn through same
On solving eqns. (1) and (2) simultaneously, we obtain angle, the observer will always see the block in front of him.
a=gtan0 In order to explain equilibr(um of- the block, ·the observer
www.puucho.com must imagine an outward force to balance tension, i.e.,
Anurag Mishra Mechanics 1 with www.puucho.com

1194
mv 2 (b) Minimum contact force between two bodies is zero.;
r.F=T--=0 at this point contact between two bodies breaks.
r
This outward acting pseudo force (imaginary force) is mv 2
From eqn. (1) 0 = -·-' - mg
termed centrifugal force. r
or v, =-..fir'
l~Exi&.~\~~~
E=.=,E__ ' -~.....- (c) At the topmost point, IFy = may

I A pall of }'later 1s whirled ''L.i: circle of r~di~f,,,;r1:.i&J


1topmosfpoirit the speed of the pqi[is·v, : .,· , 'L, .i{,,.i/·I
· mv 2
Fp -mg = - - b
r
'(a) I)eterrnine•the force exerted,on:\vater by the'.pa/Uq(the1 or Fp = mvi + mg
l top of:the'drcle . .• " : . . . ' ·• "' :· . r
(b). Pihgtke•;mini111~m value ofv, jqr the .,;at,?ta r~#taj~
in Remark:

I 'the.pair .• ·. . · · · . , _ . . · . . :,:~ .. _'. When a particle moves along a curved path, no


particular force can be said to be centripetal force, It is
(c) Find th.e force· exerted by th¢ pail at: the ·gottomtoj;'the
the name for resultant force that must be directed ·
l_ circle ,,/;hiJreCspeed is Vi, . .£:.:"' . , _' .. ::; >, ; ', towards of circular path,
Solution : Forces acting on water are weight mg and
' ........
the force of pail on water Fp . Fp may be termed reaction of
pail on water. Same force will be exerted on pail by·water.

.~·--c,:- .·- i
~·,,,...,.··---- .
r,:-"1 ~-t :<'"'"' _ _ _ --~---·--,,...,,_., ' ". ····-~·- ··,· - --~~--

0 ball of'rnas~ m is suspended from· a rope oflength r . rtl


l'd~rib.ei 1'hcrriiontal ~irele of raqius r with speed v. T(t(ropel
!makes aniingle8.withverticC1lgivenby sine= rf 2,Determine'1
·Y
.··: ''", '(a) 'the, (ension}(I the rope,.,and (b) the speed .of the'balZ: j
t-=-~~ 1
(c):Ji,111e,per;ip_cl,_of b_gl[._ , .. ...: .. • . ... .::___, __ .. J·
L.:,;'
• ---,
Solution : Forces acting on the ball are: weight mg .
and the tension in the string.
Note that component of tension T cos 8 towards centre
of the.horizontal circle is the required centripetal force.
t
L· r.FY = T cos 8 - mg = may = 0
2
... (1)
(a) At top of the circle, . mv
r.Fx =T sm 8 = max =-r- ... (2)

r.Fy = may = m (- v!)

-Fp -mg=m -~,.


( '2)
2
mv
or Fp =--' -mg ... (1)
r
Note that there are two ways to write a force equation:
(1) Assign positive and negative x, y axes; e._g,,
centripetal acceleration is towards centre of circle. At the J
topmost point it points in negative y direction; F; and mg v2
From eqns. (1) and (2), tan0=-
also point in negative y direction. · rg
· mv 2
(2) Set net force towards centre equal to - - , or V = .Jgr tan 8
r (c) v = rro
mv 2
i.e., Fp +mg=---' ~
mv
r
2
(J) -
v~
or Fp =-.-' -mg T = 21t = 21tpcos8
r' (J) g.
www.puucho.com
Anurag Mishra Mechanics 1 with www.puucho.com
'

FORCE ANALYSIS 195 I


... -1
WHIRLING ROPE Car Negotiating a Circular Bend:
A uniform rope of mass M and length Lis pivoted at one A car, travelling along a level road, enters a tum with a
end and whirls with uniform angular velocity ro. What is the
radius of curvature R.The coefficient of friction between the
tension in the rope at
distance r from the pivot? road and the tires is µ.What is the maximum speed at which
Neglect gravity.
(l) - - - -.... the car can negotiate the turn ?
.________ Ii ii ii // I :7n,
Consider the small ~ I T/1 rc.)'1

t
L '1 Concept: When a car turns a comer on a level road,
section of rope between r , _/ friction is the only force acting horizontally on the car. It is
and r + l!.r.The length of therefore the friction exerted by the road on the car that
the section is l!.r and its accelerates it around the turn that is provides necessary
,_j
ii/Iii/I
mass is t.m = M l!.r/ L. centripetal force required for circular motion. Because the
Because of its circular tires roll without slipping, friction force involved is static
motion, the section has a r+M ___.
friction, and it is the limit on static friction that sets a
radial acceleration. maximum speed for rounding the turn.
Therefore, the forces
pulling either end of the T(r) T(r+l>r) Figure shows two views of the car. Since the car is not
section cannot be equal, accelerating vertically :
and we conclude that the Fig. 2.64 0=IFy =N-W
tension must vary with r. => N-W= 0 or N=Mg ... (i)
The inward force on the section is T(r), the tension at r, ...
and the outward force is T(r + l!.r ). Treating the section as a The top view shows the horizontal force f, acting on the
particle, its inward radial acceleration is rro 2 • car. Since friction is the only unbalanced force acting, it
The equation of motion for the section is equals the ca(s mass times its acceleration :
T(r + l!.r)-T(r) = -(/!,.m)rro 2 f, = IFx = Max = Mv 2 /R ... (ii)
Mrro 2 !!.r The maximum speed is that which requires maximum
= L possible friction f max = µ ,N. Combining this result with
However, by dividing the last equation by l!.r and taking eqns. (i) and (ii), we have: Mv~,,jR = µ,N = µ,Mg
the limit l!.r ~ 0, we can find an exact expression for dT/ dr.
dT = lim T(r + 1!.r)-T(r) y
dr or-,o l!.r
2
-,
Mrro N
=---
L ...f
To find the tension, we integrate.
Mro 2 X
dT=---rdr
L
2
dT=-J'Mro rdr
To J T(c)
o L '
where T0 is the tension at r = 0. (a) end view (b) Free-body diagram
Mro 2 r 2
T(r)-T0 = - - - -
L 2
Mro 2 2
or T(r)=T0 - U r

To evaluate T0 we need one additional piece of


information. Since the end of the rope at r = L is free, the
tension there must be zero.
We have (c) Top view
1 2 Fig. 2.65
T(L)=0=T0 --Mro L
2
The mass of the car cancels out,
1 .
Hence, T 0 = -Mro 2L, and the final result can be written and V max = ~µ 5 gR
2
2
T(r) = -Mro
- ( L2 - r 2 ).
2L
www.puucho.com
Anurag Mishra Mechanics 1 with www.puucho.com

'"''""·----· -·-~~ ' · - :-7


119s "----··-··- ......... MECHANICS-I
· - - - - ._ - - · · ~.,u,. s· '
1
___ }

The maximum speed depends on the. road conditions via Motorcycle Stunt
the. coefficient of friction. On a wet road, the coefficient of
friction between the tires and the road is reduced, and the ~·
f
car cannot turn as rapidly as on dry pavement.
Roads designed
for high-speed traffic
have banked turns
(Fig 2.66). Then both _, _,
the friction and N N
_,
normal forces exerted w
by the road on the car
have horizontal --,_ --; I
components that
w w:
.,.__to center of turn R Fig. 2.68
together ~ause the
Fig. 2.66
necessary Fig 2.68 is a free-body diagram for the motorcycle and
acceleration: No rider, modeled as a single particle.
friction is necessary, a11d you can round the turn even on an
icy road at the proper speed for a given bank angle. Example Concept: Static friction, exerted by the cylinder walls·
96 for motion along banked road. on the motorcycle tires, balances-the weight of cycle and rider.
'The normalforce acting on the tires causes the centripetal·
Lift Fore~ on an Airplane iaccel_':ration of cycle and rider.
Airplanes also make turns by banking. The lift force, due
to contact forces of moving air on the wing, acts at right If the rider tries the stunt at too low a speed, the normal
· angles to the wing chord when the aircraft banks, the pilot force will be correspondingly small, and the maximum
maneuvers to obtain greater lift than necessary for level possible friction will be too small to balance the weight. (On
flight the vertical component of lift balances the airplane's a straight wall, there is no horizontal acceleration, no
· weight, and the horizontal component accelerates the plane. normal force arises no matter what the speed, and the stunt
cannot be done.) The minimum speed for the stunt is that
Concept': What does it mean t~ feel heavier? In level for which maximum friction can just balance the weight.
'flight, ea~h pass.enge~'s weight is balanced by the normalf~rce Vertical Components Horizontal Components
!exerted by tlie"se(lr; -exactly as if the person were at rest on the 'I.Fy = 0 'I.Fx = Max
,ground.' fhe person's muscles tense to maintain an upright f~Mg=O N=Mv 2 /R
,posture. This 1m1scle tension· and the pressure on our l:>ottoms
lis what ~e ·se!1s~ )Vhen we speak offeeling our weight. Wlien At the minimum speed, friction is at its limit;
!the airpla11e:biinks, the seat has. to exert enough normal force µ,N = fmax =Mg.Thusµ, Mv~;n/R = Mg; so:
ito·balance weiglit and to accelerate· the plane. Non-uniform Circular Motion on Horizontal
• -·-- >•· ---· - •

! -+
L,
Plane
! Let us consider that a particle of mass 'm' is moving in a
horizontal circle of radius 'r' with velocity' v' and tangenti;tl
,. 0 0
acceleration a,. We will solve problem in reference frame of
car. To oppose the tendency of skiilding of the particle
-+ (body) in the direction of net force F,er, a static frictional
w force F, is developed as shown in the Fig. 2.69 .
• (aj.
(b)
To avoid skidding,

rnv 2
r
\ 8b:;; bank angle
~1 -+
I w· Fig. 2.69'
'
·(c) ,
L__C_. - - - - --- ' - · · · ·- · - - Fig. 2_.67 , --

www.puucho.com
Anurag Mishra Mechanics 1 with www.puucho.com

FORCE ANALYSIS
- - - - - - - ' - ' - - · - - - ____________ ------~-'-----'-'---'---'1~9.:-111
. mv 2
LFx = N sm 0 = max = - - ... (1)
i.e., F, = (m;2r+(ma,)2 ... (1) r
And • LFY = N cos 0 - mg = may = 0 ... (2)
Since F, <; µ ,N ... (2)
Thus from eqns. (1) and (2),
v2
Where N is the normal reaction (N = mg) and µ, is the tan0=-
rg
coefficient of static friction between the body and the (b) If driver goes faster than designed speed v m/s, a
ground. frictional force must act parallel to road and inward toward
From eqns. (1) and (2), we get the centre of road.
2 mv 2
mv ) · 2 Il'x =N sin0+Ffri,. -cos0=max = - - ·
( -r- +(ma,) <;µ,mg r

( vr2 r + a; <; µ ,g
v <; [r2(µ;g2 -a;)J1/4
LFY = N cos 0 - Ffri,. · sin 0 - mg = may = O
When a rolls without slipping, there is no slipping
between the road and point of contact with road. Therefore
static frictional force comes into play. Since we require
maximum speed with which the curved road may be
negotiated, we will require maximum frictional force. ·
Ffri,. = µ,N
Hence the maximum velocity, so that the body with Thus our equations are
tangential acceleration a, in a horizontal circle of radius 'r' mv 2
N[sin e + µ, cos 0] = - - ... (3)
can move safely without skidding is given by, ·- r
vmax = [r2(µ;g2 - a;)J1/4 N[cose -µ, sin 0] = mg ... (4)
For uniform circular motion (a, = 0), the maximum We can eliminate N by dividing eqn. (3) by (4).
velocity with which a body can perform·a horizontal circular sine+µ, case v 2
motion safely without skidding is given by case-µ, sine . rg
Vmax = ~µsgr ~-------
or v = (sine+µ, c~s e)gr
r :Etxcii,;:.;~,ef9iT---,,
f~
[~-,.~~::;,,,,~~-iL-i~~::,i.~
cos0-µ, sU18

A section of a hilly highway is a circle with ·radius r. : ~,;~~H~JIJ.~J92!p


(a) What should be the banking angle e of the roadbed sol . - . - . - -- -· - . --- ---·;" - · - 7
that cars travelling at v mis need n_o frictional force from 1 A small block B is supported by a tum-table. The friction
the tyres to negotiate the tum? coefficient between block and'suiface'is µ: '. ,-: .'" . :
(b) The coefficients of friction are µ, and µ k • At what' (a) If tum-table rotates at constant_'angular'speed_ OJ, what,
maximum speed can a car enter the curve without sliding: can the. maximum angular speed OJ be fo·r w/tich the block
towards the top edge of the banked;curve? ' doesnprslip?- ' •. : -- .. , .-. - .. -
• :-~· ·--~... -- - -·· ._,. - .. 1-
(b) ~f the ang,;la,· speed is increased.uniformly from rest'with
, N Ncose
an angular acceleration a, at ivhat 'speed will' th'e block
slip? ' · · ".: . · ·' '· ' " · i
X (c) Of the tum-table rotates in such a way that the block'
undergoes a constant tangential acceleration,· what is. the
smallest interval of time in which the block can ·reach the
e speedv? ' ·
mg mg - I --- • j '
I
I ~r-+
• 'i
1
N 81§'Ny~
cos -1
II
.· I N sin 8 I
I
Fmcsin8l cCOS9 !
mg i
.·',':'·.
Fig. 2E.91
--------
'
Solution : (a) Fig. 2E.91 shows front view of car. We
assume no friction, hence the only forces· acting on the car
are normal reaction and weight. From Newton's second law,
www.puucho.com
Anurag Mishra Mechanics 1 with www.puucho.com

1198 MECHANICS-I
Solution : (a) In Fig. 2E.92(a), the circular path of the
block is shown. The only force directed towards centre is
Ffriction•
Since angular speed is · constant, the bloc!< has
centripetal acceleration only.
Toµ,vlew~··
. N
From Newton's second law; 1
I
, mv 2
LFx = Ffri,tion = max = - r - ... (1) . .
Ffric:tion ..
r

·side view mg
r.Fy = N - mg = may = 0 ... (2)
Since F motion, ""'- =µ ,N from eqn. (2) we substitute N Fig. 2E.92 (c)
. into eqn. (1).
(c) From equation of kinematics,
mv 2 v =v 0 +at·
µ,mg=--
r if block ~tarts from rest, v O = 0.
Thus V nnx. = ~µ ,gr = r 0) nnx. So · t = !:_
a
or ffimax. = ~µ,g/r where a = a, = ra
(b) When the tum-table rotates with angular When the block is on the verge of slipping,
acceleration, the block has centripetal as well as tangential
acceleration.
V = ~(µ,g) 2 - (ra)
2

r-, - -- - --- ~(µ,g)' - (ra)'


: Therefore t =- - - - - - -
(ra)

k,_g~~~~
Frriction mg r-· , · . -~---"'~---~w-:·-7
IA.50 kg wo'man.is on a large swing (generally seen inft;Iirs)of1
- Tapvi8w Side view radius 9 m that rotates in a· vertical circle at 6 reve.1/min. i
Fig. 2E.92 (b) What is ·the magnitude of her weight when she has movedi

~~~, ~
..., ..., ...,
Therefore aR = a,+ a, . 1
I~aR I = '\/Ia,2 + a,2
= ~(ro 2 r) 2 + (ra) 2
Resultant acceleration of block is parallel to surface of
tum-table. The only force that is parallel to surface is force
of friction.
., I.
N,-f N, jy ;l
mgX,
So LF'r = Ffriction, m:ix. I
Flg.2E.93
=µ,N = max
-- mVla2C + a2l Solution· : The woman experiences three forces: mg,
her weight acting vertically downwards; N 1 , reaction due to
= m~(ro 2 r) 2 + (ra) 2 ... (3)
her ·weight; N 2 , horizontal reaction whlch provides the
and LFY = N - mg = O .... (4) centripetal acceleration.
From eqns. (3) and ( 4) From Newton's secon(i la.v,
mi,2
µ,mg= m~(ro 2 r) 2 + (ra) 2 LFX =N, = - -
r
or (ro 2 r) 2 = (µ,g) 2 - (ra) 2
r.Fy=N 1 -mg=O
2 ]1/4 v = (21tr)v (wh'erevis·frequency)
or ro = [(µ;g) -a• ;. (2!t X 9)(6)
= l.81t m/s
Therefore,
N
2
= (SO)(l.Bit)' = 178 N
9
www.puucho.com
Anurag Mishra Mechanics 1 with www.puucho.com

i--FORCE ANALYSIS. . . 199:


1-- -- -

N 1 =mg= 490N Solution:


The magnitude of her weight is the magnitude of the
resultant force exerted on her by the chair. Concept: Detennine tangential and nonnal component
/
N=-yN 1 +N 2 2 2 of force E Apply
IFn--mRro 2
=~490 2 + 178 2
LF, =mRt.
= 521 N
, r--·. F case = mro 2R ... (1)
i s.-"t:'SJ?·~P r.e i 9_4 _i> F sine= _ma, ... (2)
Angular velocity ro of Line joining P and C is
:In amusement parks there is a device called rotor where ro = d(20l = 2 de
people stand on a platfonn inside a large cylinder that rotates. dt dt
about a vertical axis. When the rotor reaches a certain
de = (~) and tangential acc. of particle about
angular velocity, the platfonn drops away. Find the minimum· dt 2
coefficient offriction for the people not to slide down. Take the,
Ca =Fsine
radius to be 2 m and the period to be 2 s. ' ' m
...:>
ar=Ra=(F:::'e} a=F:e
' f y
: N t
; ...... ~
a=dro= d[2~]=2d2e d2e=~
1---1mg dt dt dt 2 ' dt 2 2
'
Fig. 2E.94

Solution : In this case normal reaction of surface


provides centripetal force and friction force prevents the
man from sliding vertically. C
From Newton's second law,
mv 2
Lf'x =N = - - ... (1)
r Fig. 2E.95 (b)
Lf'y =f - mg = 0 ... (2)
where f = µN = µmv
2
d 2e Fsine
,.,(3) -=-- ... (3)
r dt2 2mR
µmv 2 From eqn. (1)
From eqns. (2) and (3), - - = mg
r 2
de = ~ = _! x (F cose)1/
or µ = rg = gr = 0.5 dt 2 2 mR

la~~R~P Ie i_~5,:y
v2 (2nr/T) 2
(:~r ::(~r =¼F:e ... (4)

- ·: d 2e

(:~r
A particle Pis moving on a circle under the action of only one,
~ = Fsine x 4mR = 2 tane.
force acting always towards fixed faint O on the:
2mR Fcos8
. ,,
czrcumJerence. . oif -d2e & (de)
F'md ratw - .
dt 2 dt 1···-- . . . . . r-c.
p
96 l >
ks~q_!TI__p!~J -~:.----
' .
IA car is moving in a circular path of radius 50 m, on a flat.,
!rough horizontal ground. The mass of the car is 1000 kg. Ata•
;certain moment, when the speed of the car is 5 m/s, the driver'
,is increasing speed at the rate of 1 m/
s 2• Find the value of
Fig. 2E.95 (a) •sta_ti,frictio!J.. on tyres at this moment, in Newtons,

www.puucho.com
Anurag Mishra Mechanics 1 with www.puucho.com

1202 - -- .-- - -- -- --- -MECHAN·1cs:1 i


~-------------·-·--- --·-----------··- ·---- __.
,.-·· . 5m ···IIJ
:......._____~-----··.-· '
B Fig. 2E.103 (a)
'(a) Tangential acceleration of the block.
(b) Speed of the block at time t.
Fig. 2E.102 (a) (c) Time when tension in _rope becomes zero.
Solution: Radial direction: Solution: (a) Tangential acceleration is the retardation
T1 sin60°+T2 sin60°= mco 2 r produced by the friction
a= -Jim= -µmg/m
a, = -0.2x 10 = -2m/s 2
dv
11:......... . (b) - = a, =-2
dt
V C
1 ••.•------·:· ......
•/ 5m
p1Qm/s:. ·

Fig. 2E.102 {b)


f dv = -2 f dt ·'·
: ·~········--------------·· ·,
I
'.
10 0
Fig. 2E.103 (b)_
(T1 +T2 )sin600= mco 2 Lsin60° v-10=-2t
V = 10-2t
T1 + T2 = mco 2 L ... (1)
(c) Tension in the rope will become zero when
Vertical direction : centripetal acceleration becomes zero i.e., when speed
T1 cos60°-T2 cos60°= mg becomes zero
T1 -T2 = 2mg ... (2) v=0 => 10-2t=O => t=Ssec.
adding eqns. (1) and (2),
2T1 = 2mg + mco 2 L
L.s,dq~p,,e;__[W41~
mco 2L ·A ball of mass M is swing around in a circle around on a lighti
T1 =mg+--
. 2 ·spring which has spring constant k The ball describes a
:horizontal circle a distance h above the floor. The stretched

_____ 60° 60°!


.......... .
,::·..··.·.·.::·"·..--+-.C...............
spring has a length I and makes an angle ewith the vertical as,
,shown in Fig. 2E.104 (a). Neglect air resistance. :

M
Fig. 2E.102 (c) <:~.·-·.·...........
(b) Tension in lower string= zero
Tcos60°= mg ... (1)
/illlllllll/111/JIIIIIIUIIIIJ/J
T sin 60° < mco 2 r Fig. 2E.104 (a)
T sin 60° < mco 2L sin 60°
(a) In terms of only the given quantities, what is the·
T < mco 2L ... (2)
substituting eqn. (2) in (1)
magnitude of the force F that the spring exerts on the I
mass M?
mco 2L cos 60° > mg ,Cb) In terms of F, k and i what is the natural length 10 of the:
(02 > 2g => spring, i.e., the length of the spring when it is not:
co>Ff
L stretched?
'(c) In terms ofF, l,M and 0, what is the speed v of the ball?
~~mpJ~f 103 )> (d) At same instant aftime, the spring breaks. The ball moves'
a horizontal distance x before it hits the floor. In terms al,
~-block of~ass 25 ~-res~ on a hor~o~taiflo;r (~- = 0.2). It; l .v, Ii, ,ind g, what is x_?
lis attached by a 5 m long horizontal rope to a peg fixed on,
,floor. The block is pushed along the ground with an initial.
\;L~f~ of 10 ~/s so that it mov~ in a circle_ aroun_d ~he_ ~eg. j

www.puucho.com
Anurag Mishra Mechanics 1 with www.puucho.com

l_Fo~c~_~N_A~s~ -- · ·--- · ·__ .. __ -·--~ .::_:_ ~~- -- ------ - ·---- .. - - - - - - - ~------ -- --- --
Solution: friction between the shoes and the drum is µ, find the power
Concept: When a particle moves in a circle, required in watt to tum the governor shaft.
,perpendicular forces along y-axis balance out. Towards of
horizontal circle centripetal acceleration acts therefore that
must be a resultant force. Note that along the length of spring
forces are not balanced because this direction has component
of acceleration.

component of
~~ +y-axis acceleration
Fig. 2E.105 (a)

"\:T··.0,
tTcosB
Solution Centripetal
(~·.·.·.·-.-,-~~---5t
--~···· ,.,
force for rotation of brake shoe
8 ! \._ comes from normal reaction
+.
mg
between brake shoe and drum.
N = mrro 2 = mr(2rrf) 2
(b) (c) Friction force
Fig. 2E.104
(F) = µmr(2rrf)2 Fig. 2E.105 (b)
Power required to
overcome friction force on both the brake shoes
(a) F,p cos0 = Mg => F = Mg
sp case P=2Fv
(b) F,p = K!il = 2 x [µmr(21tf) 2 ] x r(21tf)
Iii = F,p = ~ = l6mµ1t3 f3r2
K Kcose r --- - · - - ---- --r--,..
10 = l - Iii ~ ~~gm~,!,~ }106 I >
10 = 1 - ~
Kcose
,--~,. A particle suspended from the ceiling by inextensible light
'-................... ;r=l~_;_~~---· , string is moving along a horizontal circle of radius 1.5 m as
2
. mv
(c) F,p sme = - r - Fig. 2E.104 (d) shown. The string traces a cone of height 2 m. The string
breaks and the particle finally hits the floor (which is zy plane
mg . mv 2 5.76 m below the circle) at point P. Find the distance OP.
--sme=--
cose I sine
--~
(d) h
v=.1=---

1 2
= -gt
2
glsin 2 e
cose

=> t=f! ~
I _
1.5m , ,./
.--
112m
5.76m
/ Q
X = Vt X

.! --------
x=vf! y
p[_..-·····

l.,S:~R\J\.i:?J? !1057> Fig. 2E.106 (a)

Solution : Let the string breaks when the particle is 1.5


The essential elements of one form of simple speed governor m right of point O and direction of its velocity v is along
are as shown : to a vertical shaft a horizontal rod is mounted y-axis.
symmetrically and on the horizontal rod are freely sliding' . mv 2
Tsme=--
brake shoes, r
' .
When the shaft turns at a frequency of rotation f the brakel
shoes press against the inner surface of a stationary, and Tcos0=mg
cylindrical brake drum. If the brake shoes are each of mass m! ---
v = .Jgrtan0
and their thickness dimension is negligible compared to the Now time to reach the floor,
inner radius of the brake drum rand the coefficien~ of sliding'
www.puucho.com
Anurag Mishra Mechanics 1 with www.puucho.com

f 204 . ·- . -. - --
I _____ -- ------- ---- . - - - - ______ M_ECHANICS-1_i

t = ~ =; v=~=Sm/s
Before it hits the floor, T
(b) Tangential component of
l'>.y = vc:t,---c: 0 force
= ~2h 2 rtane
where cane= -
r --,- =k(3:)sine
h1
dv 9kR
Fig. 2E.106 (b) , m-=- Fig. 2E.107 (b)
l'>.y=~ dt 25
Rate of change in speed
= 2xl44x(l.5)2 dv
-=--
9kR
·=; 24m/s 2
. 25 . 2 dt 25m
18
=-m=3.6m
5
Its position from 0, when it hits the floor = LS i + 3.6]
OP= ~(1.5) 2 + (3.6) 2 = 3.9 m An inclined plane of angle a is fixed onto a horb:c,ntal.
tum-table, with its line of greatest slope in same plane as a
'diameter of tum-table. A small block is placed on the inclined
plane a distance r from the axis of rotation of the tum-table
and the coefficient of friction between the block and the
A bead of mass m = 300 gm moves in gravity free region.
.along a smooth fixed ring of radius R = 2 m. The bead is inclined plane is µ. The tum-table along with incline plane,
'attached to a spring having natural length R and.spring, spins about its axis with constant minimum angular velocity
constant k = 10 N / m The other end of spring is connected to;
~- '

6 : r :
,.__,.;
a fixed point O on the ring. AB= R_ Line OB is diameter of ! !
-5
1ring:

Fig. 2E.108 (a)

Fig. 2E.107 (a) (a) Draw a free body diagramfor the block from reference of.
ground, showing the force that act on it.
_Find (a) Speed of bead at A if normal reaction on bead due to (b) Find an expression for the minimum angular velocity, oo,,
,ring at A is zero.
to prevent the block from sliding down the plane, in terms
i(b) The rate of chqnge_ in SJJ.eec:( qt this irJ5.tanf. of g, r, µ and the angle of the plane a.
Solution: (c) Now a block of same mass but having coefficient of
friction (with inclined plane) 2µ is kept instead of the
Concept: Spring force has component in radial as well original block. Find ratio of friction force acting between
_as tangential dir_ection.

r
block. and incline now to the friction force acting in part
(b).
6
(a) Elongation in spring= (2R) 2 -( : -R Solution: (a)
N
.

k
=SR_R=3R
5 5
Radial component of spring force
= k(3R)
5
case= 12kR
25 Fig. 2E.10B_(b),•

As normal reaction is zero


Concept: Not force along vertical axis is zero and along
12kR mv 2 radial axis provides centripetal acceleration vertical axis.
--=-- . . .
25 R
www.puucho.com
Anurag Mishra Mechanics 1 with www.puucho.com

[ioRCE ANALYSIS .

~~~~
(b)

Ct. stone is ia~~ched upward at '45° with speed v~. -fl beej
[follows the tra;ecto,y of the stone a_t a constant speed equal to/
\the initial speed of the stone. ·· . ;

(a) Find•.
the
.
mdius of curvature .·
at the top point of thei 'I
I traJectory. · · . · . . i
'.Cb) What is t/ie acceleration of the bee atthe top po/nt of, the1
µN sina+N cosa- mg= 0 L_. trajectorxLf.QrJ/Je store,J1_egle_c_t.thLair.J:.<J,~tg.1J<;.e; __ ... .:
N sina-µN cosa = mro 2 r Solution: (a) At the topmost point of trajectory weight
mg (sin a-µ cosa) mro•r mg acts as centripetal force. Thus,
(µ sin a+ cosa) Radius of curvature,
g(sina -µ cosa) 'IJ 45° ~,
(J) = .I . _ 0cos =.p..
r(µ sin a+ cosa)
(c) As block remains static at same height and
radial distance, requirement of friction is same as in pact
iv~al=g"""
1~-~- .,."-.

(b).
i__ ___ .. ~i_!!::e.~10_ _
Force of friction will remain unchanged.Hence ratio is 1. ·an =g
kif ...., .. '.. ··--r-t·- R=-=_o_
vz v,2
..::_~~~~~,~t::1,10~~ an 2g
1A circular r~ce. track is banked at 4s
and has a radius ~},
0 (b) For bee speed is u 0 and radius of curvature
:40 m At what speed does a car have no tendency to slip ?If tlie ! trajectory is same as that in part (a) :
vz v.2
;coefficient of friction between the wheels and the track is .!.
: · ' . .
,!
2 I
a =-=-o_
n Re Vo2 /2g
find the maximum speed at which the car can 'travel round theJ
'trg.ck.without ~l,idding. , · • . . . __ . . ........ J . an= 2g
Solution : (a) Banking angle is given by t:Exam,.,; I
~CE===~tl!?=s·....~
~:::fml:>:;:,,,,.
tan8=-
v• '.
---·----- - --- -----~---··- -·-
'" . " . . l

rg ;A .rock .is· launched upward at 45°; 'A bee moves along the;
v 2 = -Jgrtan8 = .J400 = 20m/s itrajecto,y of,,the rock at a i;onstant speed equal to the_ initial!
!speed of thejoi:k. What is the magnitude of acceleration (in!
mv 2 !m/s 2) ofthe bee at.the top point of the trajectory? For the'
(b) Normal to plane N = mg cos45°+--cos45°
r kos(<, _nfg/iftth_e q_ir..resistaric;e.... ., _ __ ____ _ _ .
2

= ;(g+ vr )
Solution : From previous problem we have at highest
point a,= g
i . - ••
f,maxµN = ½;{g+ vr2) I

Along the plane


2
friction + mg sin 45° = mv cos 45°
r
2 2 !I ' '
j
m ( v ) mg mv c · Fig. 2E.111 (a) \
2.Jz g+7 + .Jz = .Jzr .,.__. ~-----------~----·-- -- .__ --
·g v2 v2 g
-+-+g=-
2 2r r
v 2
3g [R, = Radius· of curvature]
-=- u2
2r 2 -=g ... (1)
2
v =3gr=3xl0x40 2R,
.:,",
= 1200
= .J1200
www.puucho.com
V maxc
Anurag Mishra Mechanics 1 with www.puucho.com

--- ------·- - --------- -------...·-------·--- ___________·:.~.- _. _- _ MEfiiAN!CS-i i


Now when bee moves along the same path with 2
:i:.Fx = mrm cos 0 + µN - mg sine= 0 ... (3)
constant speed u, then at top point, since radius of curvature
(R, ) remains same :i:.Fy = N - mg sine - (7lrro 2 sine= O ... (4)
u2 Substitute N from eqn. (4) in eqn. (3) to obtain
R=a, ... c2i
'
g(sin0-µcos0) ]1/2
Olmin = [ Rsin0(cos0+µsin0)
Therefore the block will remain stationary relative to
bowl if its angular speed lies in the range ro mm.
. < ro < ro max.
Students are advised to write the equations for block in
ground reference frame also and verify the similarity of
Fig. 2E.111 (b)
results in both the approaches.
From eqns. (1) and (2), we have
1 g
-2=- => a,=2g
a,
a,= 20m/s 2
-- -----· ---- -- -- -. ~-~

kF.~P-ID;PJ:.c~,! 112 j~ •
'A wedge with mass M rests on a frictionless horizontal
;A block is kept inside a hemispherical bowl rotating with; ,surface. A block with mqss (1l is placed on the wedge. 111ere is,
,angular velocity Ol. Inner surface of bowl is rough, coefficient :no friction lietween the block and the wedge. A horizontal·
'of friction is µ. The block is kept at a position where radius force F is applied to the wedge. What (Jlagnitude F must have
,(Jlakes an angle 0 with the vertical. What is the range of the' .if the block is to re(Jlain at constant height above the table
'angular speed for which the block will stay at the given. top?
position?_ ·
vvx
,w:
;_
'

. I
I8 ,
I
I
Observer N

----1---
~--
18

r = R sin B
------/
,t
1
I
/
jf mrro2 cos 8

mrro2
(Pseudo force)
mrw2 sin 0
'll
mg cos 8
Fig. 2E.113 (a)

Solution : This problem can be solved very easily if we


analyse the block in the reference frame of wedge instead of
mg sine mg i
I analysing it in ground reference frame. Reference frame of
(a) 'I' wedge is non-inertial, therefore we must apply a pseudo
(b) I force on block m.
Fig. 2E.112
.I According to condition of problem the block m remains
Solution: We analyse this problem in the reference at constant height h, i.e., it does not slip downwards along
frame of bpwl. As angular ·velocity is increased the the incline. For an observer on the wedge the block will be
centrifugal force will increase. When the component of stationary.
centrifugal force, tangential to surface, (7lrro 2 cos e will
N
System,,. ... -"?,-.. . ,,
increase, the block will have a tendency to slip upwards. In
this case friction force will a~t downwards. Similarly at low "'-../
I\
., ' ,
"
Pseudo force 8~ ,
angular speed the block ha_s a tendency to slip downwards \f m \ mA+-cc'-i'...;,;.
and friction force will act upwards. \ M ~A ~c::,.~I /a
Impending motion upwards:
2
',..__ _8~ I
....... .,,'
,I"' ~'<'/ 0~
:i:.Fx = mr ro cos 0 - (Jlg sin 0 - µN = 0 ... (1) ----
l'0 mg
:i:.FY = N - mg sine - (7lrro 2 sine= o ... (2) Fig. 2E.113 (b)
Substitute N from eqn. (2) in (1) to obtain
:i:.F, = (Jlg sin e - mA cos e = o ... (1)
Ol =[ g(sin0 -µ cos0) ]1/2
max R sin0(cos0-µ sin0) :i:.FY = N - mA sine+ (Jlg cos e = o ... (2)
From eqn. (1), A = g tan e
Impending motion downwards:
From eqn. (2), we may obtain
www.puucho.com
Anurag Mishra Mechanics 1 with www.puucho.com

·[FORCEANALYSIS _ _ _ _ _h--~----'------'-------'---------·2~0__.7 !
N = mg/cos0 [E.1!S:am,mte·l 115 ~:~
i§==--- -·· -,~~~=-~:.:~l-.J~
If the block is kept on a scale its reading will be
N = mg/ cos 0. We may consider block and wedge as a single ~ small-bead-of-,,;~;·;;; ~.-g-i-ve_n_. ;~i;id~l-velo;iryefi
· body (because block does not slip).
jmagnitude· v O on a horizontal circular wire. If the coefficienti
Therefore F = (M + m)A of kinetic friction is µ k , determine the distance travelled I
= (M + m)g tan e 1
[QefQ[e.£he_collfl[_COm~ tq J:est,_ ____ __ ..... _______ , ___ •.l

~~g~fiJ~~ Solution : Reaction of the wire on the bead is


unknown. We assumes it to be N at a!' angle 0.
;;,; ·smo~~;;· se~icir;u/~;-~if~-;;;,,;, of radi;,; ;_· is fix;d-in a\ From Newton's law,
/vertical plane (Fig. 2£.114). One end of a massless spring ofi mv 2
LFx =N sin0=-- ... (lJ
·natural length 3R/4 is attached to the lowest point O of the,· r
1wire track. A small ring of mass m, which can slide on the LF'y = N cos 0 - mg = may = 0 ... (2)
:track, is attached to the other end of the spring, The ring isj Eliminating 0 from eqns. (1) t ~ · . l

r
iheld stationary at point P S[!Ch thdt the spring makes an angle;
'of 60° with the vertical. The spting constant .K = mg/R J
and (2), we get
----~-
' I
!I '
r
,.,_
m l
!Consider the insta7.1t when the ring is released, and (i) drawi
:the free body diagram of the ring, (ii) determine thei
N = (-m-;_2 + (mg )2 , I Vo
' I
tangential acceleration of the ring and the normal reaction. ' ' '
, - - · -·--- -·- -- - ---------- ·-1 l Friction force is tangential force _ _Fig. 2E.115 (a) ___ _
.0 ,·
R'
!
l on bead .
fl>', l!
~,, From Newton's law,
n-axis ' ------
.................... ' '' dv
=-mv-
'
Ct----+-- ----+-'~- '' l
ds
.............. I
,,,
........'IJ .Sil]
I1- or
....... Joo'

N
N tNcos0
81
Nslnl
+--- y
(a) (b) .

mg
XJ i
mg I
Solution : Note that the ring slides along a circular Fig. 2E.115 (b)
.I
wire. It starts from rest, hence centripetal force is zero. From
Newton's second law,
LF'n = N + kx sin 30° - mg sin30° = 0 ... (1) .or _Jo d(v )
2
2µk r' as
vo ~r2g2 + (v2)2 r Jo
LF, = kx cos 30° + mg cos 30° = ma, ... (2)
3 [1n(v2 +~g2r2 +v2lJ:, .= _2µ_/_s
Natural length of spring is R ; therefore stretch in or
' ' 4
spring=R- 3R=~ v2 + /r2g2 + v• 2µks
4 4 or In o 'I/ o
rg r.
From eqn. (2),
ma = (mg · ~) ../3 + mg ../3 = 5-,/3 mg
' R 4 2 .2 8
Thus 5= _r_ 1n
2µk
[v~ + ~r g
2 2

rg
+ Vci]

5,/3
or a, =8 g

www.puucho.com
Anurag Mishra Mechanics 1 with www.puucho.com

For the limiting case when a = ,i,, eqn. (3) gives P = 0.


Also for a = 0, the inclined plane becomes a horizontal
v-:-, . ' ,; ' ' ,-,--,~ ,'' , , ;-1 -)1- plane. Eqn. (3) gives P = µMg
.,rert.angu.la.,rb,lpck ofm.ass,M
:,A· ... r~ts.. 011.· .an. inclinedplanel. .h.
miakes an angle a.with the,horizontal as shown in'Fig. 2E. 116
qtl In the first limiting case all the available friction is used
to resist sliding of the block down the plane; then there is no
fca). Find th~ m~itude of a ?O~Ontal foq:e Papplied io thej
resistance to lateral slipping. That's why a rear wheel drive
kentre of _the 1:i/q.ck and acting, m: a plane parallel to ;t~el
automobile can skid so freely from side to side when
!inclined :plCf1!e,:thb;t "'.ill cause th,e· riwtipn of t~e. q]oc~ ;toi
climbing a wet or icy pavement. For the same reason a car
1irnpenci ·, /·· ·. . . . . ., .: . ,i
loses lateral stability if the brakes are too suddenly applied
iAssume that tM dngle offtictioh ~/or the surface of contact is 1

,'""r·"" ..i,,, "''"" """ ~'""";'-- . : :.


so as to cause the tyres to slip.
re-·-~ -~···· ···-:--~'-
~le~~,~~~~

rI
I
i
'. ! I
~ ;~n hangs ;;;~~h~·-;;d;;;~~ ~j a'.);p~·-1;;,· l~~ih~ e~ll
0

6fwhich are tied to two light rings which are free to.m6v{!J,ni
a horizontal rod (see Fig. 2E.117); ,Wluit is the maximttm i .
! "''..l' X ;,. ,
i, j' possible sep4raticin d of the rings y;hen the man isHapstngiiiJ
!equilibrium,. if the relevant coefficient of static. ft'fcti9r't}isi
i !o.335? · · - ·, -, ·-:t ..,,, ,_
·J
..·. I I'!, '. I
1----'-=--~
Fig. 2E.116 (a)
L~~L.._.._ 1 . '

Solution : At .the. instant of impending slipping the


block is in equilibrium under the action of three forces:
I
weight, Mg; the external force P; and a reaction, R, exerted
by the inclined plane. These three forces mu.st intersect in
one point and also lie in one plane. When sliding impends, Solution : Since the man hangs from the midpoint of
the.reaction Ris inclined to the normal to the inclined plane the rope, by symmetry the tensions in the two portions ofthe
by angle of friction qi. rope must be equal and have _magnitude T, and each portion
For the equilibrium of the block will be inclined at the same angle 0 to the vertical. Thus.the
LF2 = R cos qi - Mg sin a = 0 ... (l) system of forces .acting on each ring will be the same.
Fig. 2E.116 (b) shows the system of coplanar forces in Now consider one of the rings. Three forces are acting
equilibrium in the plane of incline. So we have on it: the tensional pull on the ring due to the rope, the
,,~-----:--~-- ' --- -!
h-~-'-"-----,, I
normal force exerted upward by the rod, and the frictional
, ! force attempting. to prevent motion of the ring toward its
fellow. Since the ring is light, its weight may be ignored. If
'I' the ring is too far out, slipping will occur. At ·the maximum
ot:-+-_r-r_.. , distance apart, each ring is just on the point of slipping.
v; Hence F = µ,N.
_,
''. .I
p
'i When we resolve T into its horizontal and vertical
·: '','
components, the equations for equilibrium become
X
'•/ · Uy = N - T cos 0 = 0
i Lf'x =Tsin0-F=0
,fig; i!E._1_1~_(_!1~)--~' where we take the positive perpendicular direction as
R 2 sin 2 qi= P 2 + (Mg.sin a)2 · ... (2) pointing upward and the positive parallel direction as
pointing to the right. Then ·
Eliminating R from eqns. (1) and (2), we get
N=T.cosa and F=µ,N=Tsin0.
· P=MgJµ 2 cos 2 a-sin 2 a , .. (3) · _ ·µ ,N _ T sin 0 _ _
µ, - - - - - - - - t a n 0 - 0.35
where µ = tan qi = coefficient of friction. N T cos a
or 0 = 19.6°
Finally, we solve for d :
www.puucho.com
Anurag Mishra Mechanics 1 with www.puucho.com

[_FORGEANALY.,$~_-·-_ __ ---'.,__:.···=20!]
--~-~--"---'----~--k·~-------''~~
'~'

sin0 = sinl9.6°= ~ = dm-1

0.33 = d m·'
1/2m
------ 1'- ' ---
In Fig. 2E.119 (a) shown two,b}ocks are kept on a. rough
d = O.33m table, where nfa = 0.9 kg, m 8 =I.'i"kg, r = 13 cm,µ, = 0,').
which is the maximum separation permissible. Consider fticti.on between all the contact swfaces, pulley is
Note that 0 and d do not depend on T and therefore the (frictionless. JJetennine the angularspeed of the turri-tab/efor
ring separation is not dependent on, mass hanging from the !which the blo~15sJust begin to slide_._._ _ _ · ---'-~
midpoint of the rope.
Iiii!(·Exa-tn.t:r!e ~118,....__,__
' - ••. - ·.·· ~'LWT. ~ : . ' : . . ~
C

- ·- ... -··. ··-------··-··--··---·-----~ B


B w

~-~
;Figure shows top view of a circular rotating table, rotating
a
,with speed o). Thto particles connected by string are kept,(in :r=13cm
i two mutually petpendicular radii. Coefficient of friction. i~ i!,,
•What can be the maximum angular speed of the table so that
/the.particles do not slip on it?
. --·· --------- ---
.
. -----·-1. I (a) Side view (b)Top view

l ~-Var
!
m _r_J '" mrco2
T
J{~ f
: 84~• e f X
1
1



I
(a)

(Pseudo force)
msrw' ~ 0......__..,: TYL
(b)

- (Pseudo I X
fmax = µs (mA + ma)9
force) j·
(Pseudo force)
mArw~
2
[Al......__.. ,_ T YL
· ·.... ' .' .. --"'·~,'!!,..__~J
Solution: We will solve this problem in the reference
fmax = JlsmAg X

frame of table. Friction force is static, therefore it is variable. Solution : We will solve this problem in the reference
Letfrictionf act at an angle0as shown in Fig. 2E.118 (b). of tum- table. Due to larger pseudo force on B it will move
fmax. = µN outward, and A will move.inward. When blocks just begin to
In the impending state of motion, slide, the static friction force is maximum.
:i::FY = mrro 2 - (J cos 0 + Tcos45°) = 0 ... (1).
Equation for block B:
2
:l:F, =T+µ,(mA+m 8 )g-mnrro =.O ... (1)
:l:F, = f sin 0 -Tsin45° ... (2)
Equation for block A:
From eqns. (1) and (2) we eliminate T, to obtain :i::F, = T - µ,mAg - mArro 2 = 0 ... (2)
2
~rro = f(sin 0 + cos 0}= f.Jz ( J'z sin 0 + J'z cos 0) From eqns. (1) and (2), we eliminate T to obtain
2µ,mAg +µ,(mA + mnlg
= µmg.Jz sin (45° + 0) = Cmn - mA)rro 2
2
or ro 2 = ,Jzµg sin(45° + 0)
r
or ro = [µ,g(3mA + mB )]1/
r(mn '-mA)
Since.maximum value of sin(45° + 0)_= 1, which on substituting numerical values yields ro = 6.4

therefore ro max. = ~ .Jz: g


rad/s.

www.puucho.com
Anurag Mishra Mechanics 1 with www.puucho.com

-~~::~~-~--,-~~-;;µr:·::; _ ,t:~:::-~1
On(!iOne Alternative· is Cortept ; ; . ·· <Y ·i
___ L:.:.:i~ -~
7
.· ~ 1:-•• ·'"~-2~~·0: · · 1 , 'i • ,,". -~----~---_ _ _•

1, ,A cyclist move~.with uniform velocity down a. rough 5. In the•.situation shown


-.inclinecJ; plane of inclination a. Total mass of cycle & in· · figure the
cyclist is ~.Then the magnitude and direction of force magnitude · of total
acting on .the :cycle from inclined plane is : external . force acting'
(a) mg cosa perpendicularly into the inclined plane · on the block_A is (all the surfaces are smooth/::,
(b) mgcosa perpendlcu\arly outward·ofthe i~cjined (a) 21 N (b) 1_4 N
plane . · (t) TN (d) Zero .
(c) mg perpendicularly outward of the inclined pla11e 6. In the figure_ a _ro.pe o.f m_.a_ ss m !. · .._ ;. , ._, ·._:_:_:_ .;::,_?_;_,,J
. and length Z1s such that its one I .... ,;,•,t;, ;:''·: • ··--1-
(d) mg vertical upwards
end is fixed_ to a ri~d wal_l_ and !'·
fixed to. .ih.·e_J:_1g[ti'w_"_I_I. ;
2. A block of in:ass s.kg is dropped from top of a building.
Then the' mamiitude -of force applied by the block on
the. other IS applied With a. U-'·--·-··:.Jrfuj ·
honzontal force F as shown · , ·
the earth whne falling is : . below, then tension at the mi<ldle of the string.is.:
· (a) SgN .upwa,ds. . (b} Sg N downwards (a) F (b) 2 F
0

Sg
N down~ards (d) None of these

'.@
--·A,;· . (c) Zero (d) F/2
(~).

3 .. .In•a.vertical _disc two grooves-are made , 7. the sum of all electromagnetic force between diffe~ent.
as shown in figure. AB is a diameter. particles of a system of charged particles is_ zero :
Two balls.are dropped at A one in each- .' ·. . (a) Only if ~ the particles ar~ n~gatively charged
~oove;·;in'i.ultaneously. Then: · ·:, · (b) Only if half the partides are positiv~ly _charged &
(a) ,Time to. each.at. C is less than t:µat c '· • : half are· negatively charged · · ·,
to reach at B ·6_,_.l (c) Only if all the particles are positively charged
(b). Time.to reach atC_:is greater than that to reach at (d). ,Irie_spective of the signs of the charges
B
8. Figure shows a light spring ' ['..,- , ,_ -,-;~--:-1
. (c) :'Dille to r_each at c; _is equal to that to reaclT a! B balance connected to two . · .,
· (d) The difference in time to reach at C and to reach blocks of mass 20 kg each. ~·. ,• a 'I
, . . at B may be positive, negative or zero depending
•' I•., .• •" , •
The graduations in the f,-J, 20 kg · • • -· db t;· '1
ona,, .. · balance measure the tension· I!:::!:-~~~ • ---1
4, With ~h~t ·f;~ce mu~t.a man. pull in the spring. The reading- of the balance is :
on the rope to hold the plank in (a) 40kg .. , '
position if the man weights 60 kg ? (b) Zero kg
Neglec_t the wt. of ~e plank? rope (c) 20 kg
and pulley. [Take g ~ 10 m s 2]
. - (d) Depends on mass of spring balance .
', ' '
(a) 100 N
(b) 150 N
(c) 125 N
,.
. ' '
(
.'
(d) None _of these ' . ' . '.'. :, ,,
www.puucho.com
Anurag Mishra Mechanics 1 with www.puucho.com

9. A block of mass 10 kg is suspended (a) t 1 <t 2


through two light spring .balances as. (b) t, > t 2
shown below :
(c) t 1 = t 2
(a) Both the· scales will read 5 kg
(d) t 1 < t 2 ort1 >-t 2 depending on· whether the lifr is
(b) Theupperscalewillread lOkg&the going up or down.· _· ·
lower zero
(c) Both the scales will read 10 kg 15. Three blocks A, Band Care suspe~dedl- -·---·--,,i--

(d) The readings may be anything but as sh<>~
A and Bbelow
is .m.
....
If ofsyst~m·· ,,eh
ofhloek
is in m ',. '.\_ ....
: ·,, .._ ,.,
,_-
·-._.- ·-
their sum will be 10 kg.
equilibrium, and m.ass ofC is"'!' then: -A .•/,c:: ':
10. A force F1 acts on a particle so as to accelerate it from
' ', .-:f-__,~;,A
rest to a velocity v. The force F1 is then replaced by F2
which decelerates it to rest: (a) M<2m (b)"M>2m
(a) F1 must be unequal to F2 (c) M = 2m (d) M !> 2m
(b) F1 may be equal to F2 16. A light spring is compressed and - - - r '""' "l
(c) F1 must be equal to F2
(d) None .of these'
-e~~:~1 ~~~z;~~!d :~:;,nfre!
to slide over a smooth horizontal
l~~
11. 1\vo objects A and B are thrown upward table tcip as shown in the figure. If the system is
simultaneously with the same speed. The mass of A is released ,from rest, which of the graphs· · below
greater than the mass of B. Suppose the air exerts a represents the relation between the acceleration' a' of
constant and equal force of resistance on the twci
bodies : the block and the distance 'x' traveled by it ?, ·
(a) A will _go higher than B
(b) B will go higher than A
(b) ~K _-_7-
(c) The two bodies will reach the same height ~
o_
·x~

,~~
r- 7 t1 ·1,j
(d) Any of the above three may happen depending on
the speed with whiclt the objects are thrown
r~-------.
12, A smooth wedge A-is fitted in a chamber hanging from (c}
a fixed ceiling near the earth's surface. A block B [d)
__ - __ · __ -..,"~.!
placed at the top of the wedge takes a time T to slide
down the length of the wedge. If the block is placed at 17. A steel ball is placed on the surface of water in a deep ·
the top of the wedge and the cable supporting the tank. Water exerts a · resistive force which is
chamber is .broken at the same instant, the block will : proportional to the velocity of the ball. The steel sinks
(a) Take a time shorter than T to slide down the into the water : · · '
wedge (a) with decreasing acceleration and finally attains _a
(b) Remain at the top cif the wedge constant velocity · ,i,
(c) Take a time longer than T to slide down the wedge, (b) with constant acceleration equal, to the
(d) Jump off the wedge gravitational acceleration ·..
13. In an imaginary atmosphere, the·air exerts a small (c) with constant acceleration less than the
force;'=.'."'~any_particle in the direction.of.the particle's gravitational acceleration
motion, A p~rticle of mass m projected upward takes a (d) with acceleration detreasing _initially and
time t1 in reaching the maximum height and t 2 in the reversing _after a finite )ime. ' '' ' '
, .return journey to the original point. Then: 18. In the arrangement, slj.ownbeic\w,p,ulieys are'massless
(a) t 1 > t 2 ' and m_monleSS an~
threads ~re
II~9:1'illlf..bJock Of
1.

(b) t, = t2 . mass m1'win remam at rest 1f :,. , _ ·


4 1 1 .
(c) t 1 < t 2 (a)-=-+-
(d) The relation betwee\1 t 1 &t 2 depends on the mass m1 m2 m3
of the particle. (b) m 1 = m 2 = m 3
14. A person standing on the floor of an elevator drops a 1 1 1
(c) - = - + -
coin. The coin reaches the floor of the elevator in a m1 m2 m3
time t 1 if the elevator is stationary an~c,time t 2 if it is 1 2 3'
moving uniformly. Then: (d) - = - + - ·
m3' m2 _mi
www.puucho.com
Anurag Mishra Mechanics 1 with www.puucho.com

I 212 ~ ,MECH,\Nl~S-1 j
19. A fireman want to slide down a rope. The breaking 25. In order to raise a mass of 100 kg a man 60 kg fasterts
load the rope is 3/4 th of the-weight of the man. With a rope to it passed the rope.over a smooth pulley. He
what minimum acceleration should the fireman slide climbs the rope with acceleration Sg/4relative to rope.
-down?, ' · The tension in the rope is: (g =.10m/s 2 ) ·. ,
_(a) g/6 (b) g/4 (a) 928 N (b) 1218 N
(c} J/43 (d) g/2 (c) 1432 N (d) 642 N .
20. An einpty pl1!5tic.box of mass Mis found to accelerate 26. A ball is held at rest in
UIJ _at the r~te of g/6 when placed deep inside water. position A by two light
, How much ~and should be put inside the box so that it cords. The horizontal cord is
may accelerate down at the rate of g/6? now cut and the ball swings
(a) , '2M/5 (b) M/5 to the position B. What is the

21.
(c) ZNf/3

A m"'i ;>i.~
· · (d) 6M/7
is hrn,g by a-
,
fu<ol W
aw.al): ..Tjie fo.rces acting on the sphere are '~ .
shown ·in figure. Which of the following N .
~ T, t ·ji
ratio of the tension in the
cord in position B to that in
position A?
(a) 3/4 . (b), 1/2
statement 'is/are wrong ? .' . · · ··w · (c) 3 (d) 1
(a) T 2 =N 2 ·+ W 2 (b) T = N + W ·-·--·-- 27. In the shown figure two beads
--+, -t --+ slide along a smooth horizontal
(c) N+T+W=0 (d) N =Wtan8 rod as shown in figure. The
--+ --+ :.+ relation- between v and v O in
22. A force F = vx·A is exerted on a particle in addition to
. .... the shown position will be :
the force of gravity, where v is the velocity of the (a) v = v 0 cote
....
particle and A is a constant vector in the horizontal (b) v,;, v 0 sine
direction. The minimum -speed of projection for a (c) v = v 0 ,tan8
particle of inass m so that it continues to move with a
constant velocity is given by :
(a) mg
3A
(b) mg
A
(d) v = v 0 case
28. Two masses each equal to m· ,~,--·--.-;..,+;
are constrained to move only
along x-axis. Initially they m
y~d.
· >,..f · ' ., · ·
' '· m x
(c) mg (d) mg are at (-a, 0) and (+a, 0). (-a, O) i. (~. of .
2A They are connected by a light
23. ,A pa,~icle of small mis joined to a very heavy body by string. A force F is applied at the origin along y-axis
a lig~t string passing over a light pulley. Both bodies resulting into motion of. masses towards each other.
are'·f'ree to move. The' total· downward force on the The accel~ration of each mass when position of masses
Pl!lley is : . at any instant becomes (-x,0)_and (+x, 0)is given by:
.. ,-1 ' .
., (a) 2 mg
.. . i: '
(b), 4.mg F.Ja 2 -x 2 . Fx
Jc) ,ng (d) ·>>mg (a) --- (b)
m X 2m.Ja2 -x2
24. Blocks A & C starts from rest & inoves to the right with
acceleration aA =12tm/s 2 & ac = 3m/s 2 • Here''t' is (c) F x (d) _!_ ~
in seconds. the time when block B again comes to rest m:J 0 2_x2 ,2mV~
is : 29 . All surfaces shown in figure are smooth. System is
.I . A.
I,
,.....+
released with the spring unstretched. In equilibrium,
compression in the spring'will be :
,_ -:

(a) 1 s
[---'---·--·-~II
3
I
1
~! ·I
(b) -s
2 2mg (b) (M +m)g
1 (a) k
(c) 2·s (d) -s ..J2k
2 mg (d) mg
(c) ..J2k k
www.puucho.com
Anurag Mishra Mechanics 1 with www.puucho.com

I FORCEANA~iL:s.L'..;..,.---'-"~-
30. Find the ' maximum · ('."". . -- ""- 36. A spring of force-constant kis cut into•two pieces such

/ ~;~fi~!~~~ l~~~~~:
with acceleration 'a'. All
the surfaces ate smooth :
kufu!::::e~~ ,, that one piece is double.the length of the. other. Then
the long piece will have a force-constant· of :
00 ~k
3
(b) ~k
2
(c) 3 k (d) 6k
ma 2ma ' . '
(a) (b) 37. In the arrangement shown in figure -----,
2k . k tlie wall is smooth and friction
ma 4ma coefficient between the blocks is
(c) (d)
k k µ =0.1. A horizontal force F =1000
31. A block of mass M is sliding down
the plane. Coefficient of ·static
friction is µ, and kinetic friction is
µ k. Then friction force acting on the ,
·. _•.-·.
[E 0 -- ,
N is applied on the 2 kg block.The
wrong statement is :
(a) The normal interaction force
1:>etween the blocks· i~"lOOON.
block is : =-
(b) The friction force between the blocks is zero.
(a) (F+Mg)sin8 (c) Both the blocks accelerate -downward with
. (b) µk{F +Mg)cos8 acceleration g m/ s 2
(c) µ,Mg cos0 (d) Both the blocks remain at rest
(d) (Mg +F)tan8 38. 1\vo blocks are kept on an -r--,~-
32. The displacement time curve

lI . lj
inclined plane and tied to
of a particle is shown in the each other with a mass-less
figure. The external force string. Coefficient of friction
acting on the particle is : ' a.
I-~ , between m1 and inclined
(a) Acting at the beginning ,o~---~--.. plane is µ 1 & that between
0 Tim~~,
part of motion m 2 & the inclined is µ 2 .
(b) Zero Then:
(c) Not .zero (a) The tension in the string is zero if µ 1 > µ 2
(d) None of these (b) The tension in the string is zero ifµ 1 < µ 2
33. A block of mass 'M' is slipping down on a rough (c) Tension in the string is always zero irrespective of
inclined of inclination a with horizontal with a µ, &µ2
constant velocity. The magnitude and direction of total (d) None of these
reaction from the inclined plane on the block is : 39. A block kept on an inclined surface, just begins to slide
(a) Mg sin a down the inclined if the inclination is 30°. The block is replaced by
(b) less than Mg sin a down the inclined another block B and it is just begins to slide if the
(c) Mg upwards· inclination is 40°, then :
(d) Mg down wards (a) Mass of A > mass of B
34. A block of mass 0.1 kg is held against a wall by· (b) Mass of A< mass of B
applying a horizontal force of SN on the block. If the (c) Mass of A =mass of B
coefficient of friction between the block and the wall is (d) All the three are possible
0.5, the magnitude of the frictional force acting on the
block is :
(a) 2.5 N
(c) 4.9 N
(b) 0.98 N
(d) 0.49 N
40. A force of 100 N is applied on
a block of mass 3kg as shown
below. ·The coefficient of .
friction between wall and the I .. 1
,:--
.,' '~-~·-·_.
h..,,,, .
0
"d
F = 100N
35. A body of mass Mis kept on a rough horizontal surface block is 1/ 4. The friction force l
Fixed vertical'wan: __ _
(friction coefficient= µ). A person is trying to pull the acting on the block is :
body by.applying a horizontal force but the body is not
moving. The force by the surface on the body is F (a) 15 N downwards (b) 25 N upwards
where: · (c) 20 N downwards (d) 20 N upwards
2
(a) F = mg (b) Mg ,,;p,,;Mg~l+µ
(c) F =µMg (d) Mg?. F?. Mg~l-µ 2
www.puucho.com
... _
Anurag Mishra Mechanics 1 with www.puucho.com

- ' ...... :,., .. · 1.• ME~HANICS-1 j


41.
An insect surface
hemispherical crawlsveryup
slowlya
.(see the figure). The coefficient of
iw·
_ ·-. I(X ·• _ •
46. A stationary bcidy of mass m is slowly lowered onto a
rough massive platform moving at a constant velocity
v O = 4 m/s. The distance the body will slide with
f'r!cti~n betwe_en the insect and respect to the platform µ· = is : · :o.~
· ):lie surface is 1/3: If the line
joi1Jing the :centre of the hemispherical surface to the
insect · -makes· an" angle a with the vertical, the
(a)
(b)
4m
6m
i
I\ " ''.. Platform
1,m,J,,,;;;;;,t;,,, I
',.,
v0 =-4m/s1
I
(c) 12 m • Smooth surface
· maximum possible value of a is given by :
(d) 8m
(a) cot a=' 3 (b) tan a= 3
(c) seca = 3 · (d) coseca = 3 47. In the diagram shown the ground is smooth and F1 &
F2 are both.horizontal forces. The mass of the upper
;ri,1ocl(of mass 2 kg is held. at rest. against a rough block is 10 kg while that of lower block is 15 kg. •The .
vertical wall by passing a horizontal (normal) force of correct statement is :
45 N, Coefficient of friction between wall and the block r-· --;:=-=·::;;=;-::-::------7
is equal to 0.5 .. ·Now a horizontal force of 15 N i<'·. , ·
1 m =10kg F , . ·
, 1_ µ·::o 0.1 between the blocks
(tangential to wall) is also applied on the .block. Then fF m2:::::15kg , (µ:-coefficientoffrl,ction)
the block will :. . .
·c;) M~;e horizontally with acceleration of 5,m/s2
F'·' .. (Smooth ground) ' WeSI Eas\
. '
(b) ·.. Start to move with an acceleration of magnitude . (a) m1 experiences frictional force towards west only
:· .L25 '!'( s2 · . • iJ; F1 > F2
(c) .Remain stationary (b) If F1 '# F2 then it is possible to keep the system in
(d) 'Start to .move horizontally with acceleration equilibrium for
certain suitable values of F1 &F2
.,• .gte~t~r than 5 m/ s2 (c) · If the system is to remain in equilibrium then F1
must be equal to F2 & F2 :s-10 N ·
, 43. Astring;?f h¢gllgible ~a~s going over a
clamped'-'p~lley.' of mass m supports ·a (d) If .!i = !-!,_, then frictional force betwe'e~ the
block of mass M as shown in the figure. m1 m2
The force on the pulley by the clamp is blocks is zero
givel},,by : , · · , 48. Consider the system as shown.
(a)
(b)

(c)
J.'i. Mg.
42 mg
0CM + m) + m )g
,, ..
' .
2 2
The wall is smooth, but the ·
surface of block A & B in contact is
rough. the friction force on B due
to A is equilibrium is: ...
Es]
(d) (~(M+m) 2 +M 2 )g (a) Zero
~:~. ~ ' '
(b) Upwards
44. The pulleys -and strings shown in
tit!! ,figure are smooth and of (c) Downwards
negligible mass. For the system to
remain in equilibrium, the angle e
should be : m
(d) The system cannot remain in equilibrium
49. Given mA = 30 kg, mB = 10 kg, .
m, = 20 kg. Between A&B µ 1 = 0.3, ~
between B&C µ 2 = 0.2 & between
A ,-
B · •
. F
rnw·.
(a) ·_Zerp ·. (b) 30° C & gronnd µ 3 = O.L The least . c ·
horizontal force F to start motion of · ·· ·
cc1
45~· · (d) 60°
any part of the system cif three blocks resting upon one
45. The',force· F1 required to just moving a body up an
another as shown below is:
incljned plane is double the force F2 · required to just
preyertt 'the body from sliding down the plane. The (Take g = 10m/s 2 )
coefficient of friction isµ. The inclination 0 of the plane (a) 90 N (b) 80 N

.
is·: '

ca)
'
truJ..:1
'
µ
·

(b) t an -1 -µ
(c)' 60 N (d) 150 N
SO. The coefficient of friction
2 between the block A of mass m
(c) tan~' 2µ (d) tan-1 3µ & block B of mass 2m is µ. I ~
· B .
There is no friction between . 8 Fixed'
blockB & the inclined plane. If
,.
•'
' ',., www.puucho.com
''
Anurag Mishra Mechanics 1 with www.puucho.com

~1-_FO_RC~~-·AN_A_LYS_l_s'_-~:_._ _ ~;~---'_·,~-----------~------'-----~~-"''~~15....JI
the system of blocks A &Bis released from rest & there 53. What is the maximum
is no slipping between A ~ B then : value of the force F such ,. F" - 1
~ m-"3k1/µ =~
(a) 28~ sin- 1 (2µ) (b) 0 ~ tan-1 (µ) that the block shown in
the arrangement, does not
,m,;;m,D~ .
-~-~
(c) 20~ cos-1 (2µ) · (d) "28 ~ tan-1 (µ/2) . ·- _., ~

move question:
51. The system is pushed by the (a) 20 N
force F as shown. All surfaces
(b) 10 N
are smooth expect between
B&C. Friction coefficient (c) 12 N
between B&C is µ. Minimum (d) 15 N
value of F to prevent bloc!< B from downward slipping 54. Two beads A &B of equal mass m A
is : are connected by a· light
·inextensible cord. They are ' ·
Cal (2:)mg .(b) (!)µmg connected to move on a
frictionless ring in. vertical pla1?:e. · _,8 . ·----····· ·······•··••
(c) ·(..!.)mg

(d) (;)µmg The beads are released from rest ' · ·
as shown. The tension in the
1
_...::::::-d:::::::::___.J
LI

52. A block A is placed over a long cord just after the release is :
rough plank Bsame mass as shown (a) ../2.mg (b) mg.
below. The. plank is placed over a 2
smooth horizontal surface. At time (c) mg Cd) mg:_
t = 0, block A is given a velocity v O in horizontal 4 ,./2

,-,---,
dqection. Let v 1 and v 2 be the velocity of A &Bat time 55. A bead of mass 'm' is attached
't' . _Then choose the correct graph between v 1 or v 2 to one end of a spring of
'l"d t: natural length R & spring
jvr&'v2. (-fl+
constant k = ---~. The
l)mg
(a)
·v,
V2
'
I other end of the sp!ng is fixed \
at point A on a smooth vertical L 1 _ _:::::::=C:::::..-_J
ring of radius R as shown. The normai reaction at B
-+-------;;,. 11
L just after it is released to move is : '
(a) .fl mg (b) 3,.J3 mg
r·1 & V2 (c) mg (d)·,3,.J3mg.
2 2 ...
(b) 56. In the above question 55 tangential acceleration of the
I
bead just after it is released is .?, ·
U2
I-_ . (a) !
2
(b) ~ g· .
4
V1 & U2 (c)
4
! (d) ig
(c) 57. If you want to pile up sand onto a circular area of
radius R.The greatest height of the sand. pile that can
v, be created without spilling the sand onto the
surrounding. area, if µ .i's the coefficient of friction
between sand particle is :
v1&v2 (a) µ 2R (b) µR

(c) R (d) R
·(d) µ

www.puucho.com
Anurag Mishra Mechanics 1 with www.puucho.com

,;r-~·
58. A man of mass 60 kg is pulling a hanging icleal string. The : 50kg
mass 'M, by an inextensible light maximum possible
rope passing througlf · a smooth tension in the string is
& mass'.less pulley as shown.
The coefficient of . friction
1000N. --The minimum
time taken by the man to i 20m . : 'jto~-J ·
between the man & the ground
isµ = 1/2. 'Ihe maximum value
reach -upto the pulley :
. r,L-----~
. - _,,_ '"--··
,. ...·___I,
* ·. 4

. of M that can be pulled by the man without slipping on


the ground is approximately :
(a) m (b) 1
(c) .Ji (d) none of these
(a) 26 kg · (b) . 46 kg ·
64. In the above question 63 distance between the man ·
(c) 51 kg (d) 32 kg
and the block' A', when man reaches the pulley is :-
59. A weightless string passes through a slit (a) 10 m · (b) 2 m
over a pulley. The slit offers frictional
(c) 20 m (d) None bf these
force 'f' to the string. The sp-ing carries
two weights having masses m 1 and m 2 65. The force acting on the block is give1_1 by F = 5 - 2t.
where in 2 . > m1 , then acceleration of The frictional force acting:1m the block after time t = 2
the weights i~ : seconds will be : (µ = 0.2)

l ; l;;;;, ~-
,------. ·""r ¢•··'"· - · - - · - --.-

(a) f :- (m2 - -m1) g !~-~


\
0.2 kgl;?
di/II 1/IIJJJJ
(5 - 2t)N il
m1 +m2 --·---·---· ~ ~ - - · · - · - - - - - ~ - - J

(a) 2 N . (b) 3 N
(c) .Cm2 - m1Jg- f ·
m1 +m2 ,
(c) 1 N · (d) Zero
66. The acceleration of small block m with respect to
60. A plank of mass 3 m is
ground is (all the surface are ·smooth) :
.placed on a rough inclined
plane and a man of mass m (a) g . ,(b} g/2
walks down the board. 1f (c) · Zero · (d) .fig
the coefficient of friction 67. In the above question 66, if the same acceleration is
between the board and towards right the frictional force exerted by wedge on
inclined plane isµ = 0.5, the minimum .acceleration of th,;, block will be : (Coefficient of friction between
wedge & block = ../3/2}

l
does not slide is :
(a) 8 m/s 2 (b) 4m/s 2
(c} 6m/s 2 (d) 3 m/s 2 ! fig ' ~- .•
61. A small block slides without friction down an inclined
plane starting . from rest. Let Sn be the distance
traveled from time t
.
(a) 2n - 1
~ n -.1 to .t =·n. Then ..!!.E_ is
(b) 2n + 1
sn+l
:
(a) mg
t -
30°
~-~- -· ....,.._ __ I
(b) 3mg
2
I
;

2n 2n-l (c) 2mg (d) mg


(c) 2n,- l (d) 2n 2
2n+l · 2n+l 68, A· block of mass 'm' is held
'•
62., A· wedge of mass 2 m and a cube of stationary against a rough wall by
mass m are shown in figure. Between - :1.' applying a force F as shown. Which
cube and. wedge, there is -no friction.
":' '
.
,_--~---. one of the following statement is
The minimum coefficient of friction incorrect ? '
45°
between. wedge and ground ·so that
wedge does not move 'is :
~
--- (a) Friction force f = mg
(b) Normal reaction N = F
caJ 0.20 CbJ 0.25 (c) F will not produce a torque
(c) 0.10 (d} 0.50 (d) N will not produce any torque
63. The figure shows a block 'A' resting on a rough
horizontal surface with µ = 0.2 A man of mass 50 kg
standing on the ground surface starts climbing the
www.puucho.com
Anurag Mishra Mechanics 1 with www.puucho.com

[}011.cEA_NA_-LY_s1s______________~ - - - - - - - - - - ______ 31?]


69. Two blocks A and B of masses 2mand m, horizontal circles. Then, the ratio of tensions in the
respectively, are connected by a massless two parts of the string is :
inextensive string. The whole system is (a) m,/(m1 + m 2 ) (b) (m,. + m 2 )/m1
suspended by ·a massless spring as shown (c) (2m 1 + m 2 )/2m1 (d) 2m 1 /(m 1 + m 2 )
in the figure. The magnitude of 74. A small particle of mass 'm' is
acceleration of A and B, immediately
after the string is cut, are respectively : released from rest
inside a· smooth from point
hemisphere bowlA ii~-A~ \ _,.,___, ,
:_:·::~--~::_:~-~----·~;;-.
(a) g,g/2 (b) g/2,g as shown. The ratio (x) of 8

(c) g,g. (d) g/2,g/2 magnitude of centripetal force &

r
70. Two particles of mass m each normal reaction on the particle at any point B varies
are tied at the ends of a light 1 withe as:
string of length 2a. The whole
system is kept on a frictionless
horizontal surface with the
nJ
o- O
p '',m
,., :I i . Jc· I
[ l
. i
string held tight so that each : Jc a >Jc ., a >j
mass is at a distance 'a' from
the center P (as shown in the figure). Now, the
+=-.er
r-.·- ---- ~--1
(b)_

.
.

r----·
-- ~I

, , h-..!
mid-point of the string is pulled vertically upwards
with a small but constant force F. As a result, the
particles move towards each other on the surface. The
magnitude of acceleration, when the separation
between them becomes 2x, is :
----------'
{d) : ~ .J
____t

(a) F a (b) F X

71. A
(c)
2m )

F x
2m a
0 2 -x2

particle moves
(d)
2m )

2m
0 2 -x2

F )a 2 -x 2
X
75. A particle of mass' m' oscillates
along the horizontal diameter
AB inside a smooth spherical
shell of radius R. At any instant
KE. of the particle is K. Then
~@i-A--- --------.;......... ••· B
1··-~-
i
_____ ·;_ _ _ _ _j
along on a road with force applied by particle on the
constant speed at all shell at this instant is :
points as shown in K (b) 2K
(a)
figure. The normal R R
reaction of the road on the particle is :
(c) 3K K(c)
(a) Same at all points R 2R
(b) Maximum at point B 76. A particle of mass m is moving in a circular path of
(c) Maximum at point C constant radius r such that its centripetal acceleration
(d) Maximum at point E a, is varying with time t as a, = k 2 rt~ where k is a
72. A particle of mass m rotates about Z-axis in a circle of constant. The power delivered to the 'particle by the
radius a with a uniform angular speed ro. It is viewed forces acting on it is: -
2 2 2 2
from a frame rotating about the same Z-axis with a (a) 21tink r t (b) mk r t
uniform angular speed ro O• The centrifugal force on (c) (rrik 4 r 2ts)/3 (d) Zero
the particle is :
(a) mro 2a (b). mro~a
77. A long ·horizontal rod has a bead which can slide along
its length and is initially pl~ced at a distance L from
00 one end A ofthe rod. The rod·is set in angular motion
(c) m( + 000 ) \ (d) mroro 0 a
2 about A with a constant angular acceleration, a. If the
73. A particle of mass m1 is fastened to one end of a coefficient of friction between the rod arid bead is µ,
massless string and another particle of mass m2 is and gravity is neglected, then the time after which the
,fastened to the middle point of the same string. The bead starts slipping is :
other end of the string being fastened to a fixed point
on a smooth horizontal table. The particles are then (a) fa (b) µ
../ri.
projected, so that the two particles and the string are 1
always in t:lie same straight line and describe (c) ../µa. (d) infinitesimal
www.puucho.com
Anurag Mishra Mechanics 1 with www.puucho.com
,·,

-.
-',"
, _1, .. Cl",
. -, ', ', ME~f!{INICS-1 :I
·1
a
78. · In gravity-free space, a particle is in constant with the witho1;1t fri_ction thr~ugh it. B is . r.;· .. .
· hii,er- surface• of. a hallow cylinder and ~aves in a
circular path aJo!'g the surface .. There is some friction
·between the particle· and the -surface, The.retardation
of the particle is: ... : .:· · · .
Dunng. •CA
'"""
will:
the moaon from. A, to
. •,d ru,,i.""1
•'
C if
""'""'
.
(a} Alw~ys be in contact witlr the ... ' .,:,.~:~:-'.· . 9,
1~-
_· .- .·.-
. · ·+::: .B .•..
: '' ·::·

· Ca) Zero · . .' inner wall of the tube


(b) Independent of_the velocity .(b) Always be in contact with the outer wall of the
Cc) Proportional to . -.
its velocity
,. '. , .tube : · _ · .
Cd) .Proportional tQ'the square of its vel99ty Ci)- Initially be in contact w:it:li. the inner wall and later
79. A curved ·section of a road is banked for a speed v. If 'with the outer wall
there is no friction Between· the road and the tyres (d) Initially be in contact with the outer wall and later
then: · · with the inner Wall ,
Ca) .a car. moving with speed v ·does noFslip on the 85. A particle is ~i>ving iri. ·a: 'circle ~t radius R in such a
.road way .thilt at
any instant 'the normal. and tangential
(b) a caris more likely to.slip on the road at speeds components ·a( the acceleration '!i:e· equal. If its speed
higher than ·v, than iit speeds lower .than v at t ·=· 0 is ·u 0 ,. :the tim_e taken to complete the first
Cc) a car is mqre likely to slip on the road at speeds revolution is :
,, _lower thaji v, t!i~. at _speeds. hlgher than v ·,- , Ca) R/u 0 (b) u 0 /R
Cd} ,a ·car can remain stationary on. the road without Cc) ~(1--e"2lt) Cd) ~e-2'
slipping . -. ' .' _ · Uo - 'Uo
80. In' a, circular :mbti~n- of ~ particle the tangential 86. A ·particle P is inoving in a circli ~f radius r with a
acce)eration of the particle is given by a,·=
2t m/s 2 • uniform speed u. C is the center of the circle and AB is
The radius of the circle described is 4 m. The particle is diameter. The angular velocity of P about A and C are
initially at rest. Til)l.e• after Which-total acceleration of in the ratio·: ·
the' particle makes,45° with'radial acceieration is : (a) ~ : .2 (b) 2 : 1
.i
Ca): sec · ... -· .r,'. ,_ (b),'2 sec (c} 1: 3 Cd) 3 : i
Cc}"3 sec -Cd)' '4 sec 87. A small body of mass m can
81 .. A partide travels along the arc ofa circle bfradius Its r. s_lide without friction along 1 ' ]

· speed depends on the distance. travelled l as v = a.ff.,
where 'a' is a -constant. The angle a; between the
vectors of total, accejeration. and the velocity of the
a trough bentwhlch is iri the . ·1
form of a semi-circular arc .
of radius R At what height h i · ~ h ; ·
0~ ·_· ·. .
7 _· • • . - ; ''.. ,j' ·

particle is : , ,_ will the body be at rest with " - · ~-----


1
(a)_.g = tan- C2l/r) - . Cb),. a= ~os- (2Zjr) ·., 1 respect to the trough; jf the trough rotates with
uniform angular velocity OJ about a vertical axis.:
Cc} a; =·sin-1 C2Z/r) · ·(d), ·a= cot" 1{2Z/r)
Ca) R · (D) R -· 2,g -
82. i p~~le of ·mas~ m is atta~~ed to· ~ne end of a string OJ2.
oflength 1while the othe'r end' is fixed to point Ii (h < l) '
· above a horizontal table. The particle is made to
(c) 2,g- R_; · -Cd)· R-.L ·
- 2
. QJ2 OJ .
revolve ·in a circle ion the table so as to make p
revolutions per second. The maximum value of p, if 88. A car moves, along a horizontal circular road of radius r
the particle is to be in i:ontaciwith the table, is : CZ > h) with constant speed v. The coefficient of friction
between the wheels and the road is µ. Which .of the
(a}-• 2rc.Jifi · Cb)' ..jg/h
; -- . '. ' . - '.I: ' following statement is not true ?
Cc) 2rc..jh/g Cd) -, ..jh/g Ca) The car slips if v > .Jµii
· 2rc
83. A. sto~e is thrown horizontally with a velocity of 10 mfs Cb) Th~ car slips ifµ < (v 2 /rg~
at t = 0. · The radius · of curvature of the stone's Cc) The car slips ifµ_>(v 2 /rg)
trajectory at t = 3 s is : [Take g = 10 m/ s 2 J Cd) _The- car slips at a lower speed if it moves with
·cal 1oJio m . Cbl 100 m - some tangential acceleration, than if it moves at
Cc) 10oJio ~ Cd) 1000 m constant speed -
84. T)Ie narrow tube· AC forms a quarter circles in a 89. A smi>otli liollow cone whose vertical angle is 2a; with
vertical plane. A ball B has an area of cross-section its axis vertical and vertex downwards revolves about
slightly smaller than that of the tube and www.puucho.com
can _move i~ axis 11 tl!ne p~r seconds. A Particle is placed on the
;· . '' ·, . .
,,
Anurag Mishra Mechanics 1 with www.puucho.com ', ,·· '.,
•• ..
'

.
•.

inner surface of cone so that· it rotates with same


speed. The ,radius of rotation for the particle is :
(a)gcota/4rr 2 T] 2 '·, (b) ·gshl'a/4rr 2 Tj 2
(c) 4rr 2 TJ~/g (d) g/ 4rr 2 TJ,2 sin a
90. A particle is kept fixed ~n a turntable rotating

It
(a) 20_,sec (b) 10 se{
uniformly.As seen frpm the. ground the partjcle goes in
a circle, its speed is 20 cnys & acceleration is 20 cm/s 2 • (c) 40 sec (d)'-'s tee'·,

~
The particle is now shifted to a new po~itiol) to make 96.
the radius half of the original.yalue. The new values of !1~:S~~cl~o:ti;ll~s :~::_ • '~;;7
the speed & acceleration will. be : · figure'. .The , · · approximate· ~ · ./" = 01
(a) '10
cm/s,·'10 cin/s 2 , (b) '10, ~/s, 80 cm/s 2 variatio~ · of· direction . ·of · · t :· . ·· ·.::'.,;,·{. •, i' j
resultant acceleration . as·· B ·····;,; >\J:~:.J
(c) 40 emfs, 10 cm/s 2
(d) 40 cm/s, 40. cm/s\::, particle µioves 'from A to B is : .. ' . .
'91. A particle· of mass in is suspended (a) clo~se ,.
from a fixed point O by a: string of ~o

rn
(b) anticiockwise '. ,
length' l. At t ~ 0, it is displaced from . ··'
(c) · direction· does not changes.'
its equilibrium position and released. . e :.
The graph which shows the variation
of the tension T in the string with
· ~. . . .1.. ,:,-,:·
I ' , (d). no~e o~ the~e ,., .· . . .
97. In the above question 96 the net acceleration of
.

time t is : =
particl~ is h«;>rizont'al only at 8 (8 is acute angle made
by string.~th liµe OB)':. ' ··

(a)
. (b)
f,~-
r, ,; ·~.·
:;• .· .•..·._·J·
:j
•., .• ('1)·
./3
' ·. (a) cps-' . · .·' '(1)
.'J3 . (b{si~~' .
,I_,_
1-. ---·-.
1· :,,,
-·-L.:J ·c· )..
c
2.
.i)
r,; . . , .
-rr:. -.sm
. ~'(
· .. ~;(·.1r,;:J.
(~)_:rr:
;c-.,-cos_.
'!3: . . ,_.,,.,.2 ',,: . ....,3 '·
~~-1·;r~,·
.
98. Two similar trains are moving along the equatorial line
r~I .,.
(c) l : , . ~ ( d ) with same.speep_but in·«;>pposite dire~tion. Then:
(a) they'.will exert 'equaH9rce ,6n rails ,: :-,
92. A rod of length Lis pivoted at one end is rotated with a (b) they ~ :not exert 'any f~rce. as they are on
uniform angular velocity in a horizontal plane. Let equ!'ltorial line · · ·. ,
T1 &T2 be the tensions atthe pointL/4and 3L/4away (c) on~. of them will exert.'zero ·force.
from the pivoted ends. (d) both exert.different forces _... ' . . ·. •.
(a) T1 > T2 99. Two b~· of'mass m and 2111 are attached with strings
(b) T2 > T1 of length 2L and L respectively They ,are released from
(c) T, ~ T2 of
horizontal position. Find ratio tensions in the.string
(d) The relation between T1 &T2 depends on whether when the accelerati9n ·of b9th, is only ,in vertical
direction: · . · · · ·. _. ' ·. ··
93. The
the rod rotates clockwise or anticlockwise
driver of a car-travelling at speed V suddenly sees
(a) 3·: 5 · . , · '(b). ,t
5
a wall at a distance r directly infront of him. To avoid ci:). 2 ,.s,·:·_ . ca) -i.i~- . ...
collision. He should : · 100. Indicate.the direction offrictional f~r~e·6l' a car which
(a) apply the brakes . is movhlg along. ,the.· ctJrv~d. path- with .. ,non-zerf>
(b) tum the car simply away from the wa:11
(c) do any of the above options
(d) none of these ·
94. A body is undergoing uniform. circular motion then
. (a) ,~~r~. . .. _: ~) -F~,,J· :,: . .;.-
tangential acceleration; ih a,nti-clock' directioJi': . . .

[0_·.LL,
.·,.-< _:I · . _,
which of the following quantity is constant :
(a) velocity (b) acceleration • (d) . ['\I ,.
A
(c) force
particle is resting on an
(d) kinetic energy
inverted cone as shown. It
'il -
. lil$.J
95. . ·'
is attached to cone by a thread of length 20 String cm.
remains parallel to slope of cone. The cone www.puucho.com
is_ given
Anurag Mishra Mechanics 1 with www.puucho.com

I 220 M_ECH~N.lCS-1 ]
---~-Cc-------~------ -------c:,-------:.....-....J.
j • ,, ••• • ,. •

10 I. If a particle starts from A along the curved circular 103. A simple pendulum is oscillating without damping.
path shown in figure with tangential acceleration 'a'. When the displacement of the bob is less 'than
Then acceleration at B in magnitude is :
r·--···-s-··_ . 7 maximum, its acceleration vector a is correctly shown
in:

l' C'\!.
.:f':.__ .• .c•
I
(a) 2a~1+1t 2
(c) a~1t 2 -1
(b) a~l +1t 2
(d) a1t~l + 1t 2
102. A small block is shot into each of the four tracks as
shown below. Each of the tracks rises to the same
(a) !~i (b)

······et 'i
height.The speed with which the block enters the
tracks is the same in all cases. At the highest point of
the track, the normal reaction is maximums in :
(c) r ·...: .~ . -·-1.
'· . . ~. lI
(d)
,'' '~ .. ··
': a . '
cai I ~ ; (b) ,: ........·· !
: -
L-,--." -,- ,_ · - - - - ~

, , G'. ~ ) "'
l : ¾,~ ~ : __ -----1

':,:'

www.puucho.com
Anurag Mishra Mechanics 1 with www.puucho.com

[ FORCE ANA~YSIS
···- ----

2 __ --~~~~ ,t~-~~-~~~-~ltern~ti~=-~~~~~~ -~or~ect -~

1. A particle stays at rest as seen in a frame. We can when seen from S1 and S 2 respectively. Which of the
conclude that : followings are not possible ?
* *

:LI
(a) Resultant force on the particle is zero (a) F1 ,;, 0, Fz = 0 (b) F, 0, F2 _o
(b) The frame may be inertial but the resultant force (c) F1 = 0, F2 ,;, 0 (d) F1 = 0, F2 = 0
on the particle is zero 7. In the arrangement shown pulley r- ······
(c) The frame is inertial and thread are mass less. Mass of
(d) The frame may be non-inertial but there is a plate is 20 kg and that of boy is 30 I . ·:
non-zero resultant force kg. ' :
Then: . . ' ' '
2. A particle is found to be at rest when seen from a
1
-.--- ---·-"~ -
1~1a _i
~.
frame S1 and moving with a constant velocity when (a) If normal reaction on the boy is
seen from another frame S 2. Select the possible equal to weight of the boy then the force applied
options : on the rope by the boy is (lS0g/7) newton
(a) Both the frames are non-inertial (b) If the boy applies no force on the string then the
(b) S 1 is inertial and S 2 is non-inertial normal reaction on him is 30 g.
(c) Both the frames are inertial (c) If the system is in equilibrium then the boy is
applying 125 newton force on the rope
(d) S1 is non-inertial and S 2 is inertial
(d) None of the above
3. Figure shows a heavy block kept on a frictionless
surfaces and being pulled by two 8. A smooth ring of mass m can slide on
ropes of equal mass m. At t =0, , · ·· - - · ····-- - ·1 a fixed horizontal rod. A string tied m

the force on the left rope is ' . ~ .i to the ring passes over a fixed pulley
. 1 m
withdrawn but the force on the 2@JN _ ! .
m Fj B and carries a block C of mass 2m as
right end continues to act. Let F1 shown below. As the ring starts '
and F2 be the magnitudes of the forces acting on the sliding:
block by the right rope and the left rope on the block
respectively, then : . f h . . 2g case
(a) The acce1eranon o t e nng 1s --"--'--
(a) F1 =F2 =F + mg fort < 0 1+2cos2 0
(b) F1 = F, F2 = F fort > 0 (b) The acceleration of the block is 2g
(c) F1 = F2 = F for t < 0 1+2cos 2 0
(d) F1 < F, F2 =F fort> 0 c . m
e tension 'the stnng. .1s - -2mg
-"--
()Th
4. The force exerted by the floor of an elevator on the 1 + 2cos 2 0
foot of a person standing there is more than the weight (d) If the block descends with velocity v then the ring
of the person if the elevator is : slides with velocity v cos8.
(a) going up and speeding up 9. A block of mass mis kept on an inclined plane of mass
(b) going down and slowing down 2m and inclination a to horizontal. If the whole system
(c) going up and slowing down is accelerated such that the block does not slip on the
(d) going down and speeding up wedge then:
(a) The normal reaction acting on 2m due to m is
5. If the tension in the cable supporting an elevator is
mg sec8
equal to the weight of the elevator, the elevator may
be: (b) For the block m to remain at rest with respect to
wedge a force F = 3mg tan a must be applied on
(a) going down with increasing speed 2m ·
(b) going up with uniform speed
(c) The normal reaction acting on 2m due to m is
(c) going up with increasing speed mg sece
(d) going down with uniform speed (d) Pseudo force acting on m with respect to ground is
6. A particle is observed from two frames S1 and S 2. The mg tan a towards west
frame S 2 moves with respect to S1 with an acceleration
a. Let F1 and F2 be the pseudo forces on the particle
www.puucho.com
Anurag Mishra Mechanics
'
1 with www.puucho.com . ! •

l--~-,:~22,.,:.'__·,f;~~~:~~~:>,~~4
. "~ J-!:= :,~:1\·>f·:
1
·.:.· ;~~~./: }'::~~~-1~~,f.
. ·i,, ~._:-~<;,.:, -.~: .°'.--,.3-::.1,:~S,..;;·0.:_ ..__. __ --'. ~.·: ._
_ · - ·-- "'~--.i:1('
. ·. -;>,-. :: ,.,-~
f' ;,::.~:-,~-~, -·-··.·MECt1AN1~s~:J-
. -·-"-"''----C·-:.u.:~Li~....:.::..-·,_... _.~~w
io. pie ca-~i~ tile,glven-figure mov~s. ·--··":::7··---:;:•·7
r-c . , . (b).
The maximum force, which the man can exert on
with ,co~gant vel9city v, When,' t .· ,·: ·- 2\_._~· .5 the wall is the.maximum frictional force which
x = 0 ends A and B ..were l ·, · ·" l, exists between his feet and the floor
.i6incident'.at c;· Then whicli ~f ~ t;:·:· _;,t.,J:1
i'+::c, , ,B" ~ ·! ' ··
Cc) 'rhe man can never exerts a force on ,the wall
'tlie foil~~tig -s~nterices · is/a~e ,' °&hich exceeds his weight
corr~f~.: · ' ·, ,- L;:,:::.~~ ,. ,I ,.(di .. The.man cannot be in equilibrium since, he is
· -;-·· ·, '. :.·. '- , :_;· :, . :: .'i' ·. --:' :~ · exerting a net force on the wall ·
·:,-: (a)° ·The velocity of the block is ·· 15. A block of mass mis placed on a smooth wedge of
2
. :: _· . .,·:,',,/::.::.: :block
(b) Accelerattoh
: ·_::.v~:t\ ·:· ' :~~;=~*re~~~~~!Jt:a:~:~e~s::~~~~~~~t::
, · , , ·, of the
, . , ·is- (H2..- , . '_ v 2)' .3/2 : ' from .the .grou.nd: ·. ' . . ' '

I ··· · · ··
· --. ' . ._. ::; ' · - · ·_ ·.· ·· + ~· · · · (~) (M'.t- mig Cb) ,(M + 171sin8)g -_
(c) Ac'ce!~r~~oh'cif.block_A, is i:¢ro, ~ '.: ,_.,, (c) . Mg (d)• .('M_ +msin8)gcose
.(d)-Velocity'· ofthe'blockisti. . ···:,. .:!,· ·
' ' .. , · M+m· '
i1, .Two !Il.~n.'.01'f.:un~qual).~as,ses lio.ld'oo ~.di,
0 f Ii 'h · · · · ! ·· J 16. A block of mass m is placed on ,a smooth Wedge of
secti_o.n~. a· ·g t ~.~p
..e. assip-·g·
smo~~. ljght pulley: Which ~£,othe (ollo'Vll1g f ;,, :•.·...'...•
tw.·· .. -.c,. , ;P·. ,oyeta [·.·f. ·•·. inclination e m,·th the horizontal..J'he ~hole system is
I ,.., ,
:f 1i!\ :a~~~-
., .. - 'bl· ?". , -- . -
1 ~~·;~ :~t:~oriaj,:
·• , .. ,

? ·.- ···" _.,,,,. ;-·· _·. · -· .... - ; 1 _ ·• ._, , • •• ,_ ·~·


~~~
- .
the. 1: ,:;;,,l]
accelerated so, that the_ block does not .slip on the
wedge. Theforce exerted by the· wedge on the block
has a magnitude:
": lighter miili-fui>ves wlth:some·acceleraticin
~) _th~)iiO:tet~~ sti;i ha;:y_ w!ill~--the h~~~er ls 9 Ca) mg/cose . Cb) mgcose
.... man·moveswitlisciineaccelebition:'. , · (c)· mg (d). mgtan0,
cd:'rhe light~r:in~n _ls stati\)hazy,~hile,the heavier 17. In ,arrangement .shown below, the
-_,: . •m·.an.·,m,,9.."e.'Yi,\li, some,,ac.~e\e,ra.t_io_h .. .-·-'.. , _ thre~d ,pulley :and spring, ~e .. all r,

td)< The twii' . men move. wiih acceleration: cif the· same massless and there is no friction
.. ·(, \•: -~,amJf.U~e ll! O~po~it<s,4/i:ecti?nf . ':':·,
12. In, the situation' shown · iri '.figure · · : ·
:::t;~:~. I~eadsi;ri:~ctin~ in4 ~ m.,·r,J'-;,-'--\_;"·1
cut then just after thread· is cuf: ·, '
F = soo ~evJton appJied, oft· t)le 'pulley.- : .-
m,· = s··J<g'and in~ 10 kg and' pulley ·. ·:. =·
and . -. strings ', are·· '·massiess .~
. frictionless. ,Then,. the- acceleration ·of ;rn
' .
, and , ·; • '·. •
(a) a<;cel~ration of m4 = 0
tJje p~lley is'.: fg =: 10.ir!./s2 ] · .'.,~~"'"-3 (b) acceleration of m1 = m2 = ~ 3 =, m4 = 0
(~) 1s:1;1Js\0::', . :/, :_': (b) 27:s:n;s 2
' '. (c) acceleration of m1 = m2 "' m3 = 0
(c) 40 rfi/s
. . ', '.· i ·,;l : ,.
2
r - ::·. _; : (d)
. ,... -~
7.S·m/s 2 : .: •
' ' ,·, ' ' '• '' ' -
(d) ,,acceleration ,of m
4
=
[(m, + m2 )-,(m 3 + m4 )lg
13. !Ii.th~ figur,e; the puµey·P·mov:es· td -7>~ f -. . m4

the right-with 1i·const1!-'!t s~eed u. T_he ~ -"' 18. A,' trolley C can run on a smooth: ... :.. -~--"·;;--
dOw,11\\'.ar~, speed M, 1ps VA, and tl}e' l:-' ,/• , , ' ,· but
h~ri:i:ontal ta.. ble. 1\vo, much smaller 1J[.,,,·····[!l·J
-. speed of B to,the tjght is vB: , ,· L , , AJ equal masses A .and B- are hung l . ::::;: '; 8 J
(J) VB =h+~~'· <:, ' ' i ' .• • ,, ,. by strings which pass over smooth A •.
• • ' r.
,i' <j
)
i. <
(b) VB tu,{v:;,. ;. _, ,: ' pulleys: The stnng are long enough --- 0 - - • - • - ~ 0 , ... ,

' . ,,, . ' • ·.,, .., . I. ~ '. •_-, that when C is in equilibrium. A and B both are just on
(c) VA "" VB : , ' ' ' , '. · . , .
the ground. The trolley is pulled slow to one. side and
~ • ' _,, ·,-,,,, •, ' ' - • - • ' ''-• ;.l . . . . ~ ' ,, ,_
(d)· The two·
< ,, .
blocks'.• have
. i
'acceleration
- .
of the same < ., .,• released as shown below. The graph of its velocity' v'
, ' magnitude · ·- . ,, ,·; · ·. . , · against ' t' ~II be as :.
14. A man pusiles. against ~ rigid"fixed vei-iicaj wall'. Which.
of the folio~g .is · {!ll'e)', tjie most· accurate -~ t·'.:,:\l .
f". •n-:'!"7'7

statemerit(s) related 'to the·.siruation ?, ,


(a) Whaieve;· force the niari ·ex~it:s 'on th~ wall, the
. wa1La,i~o e'!'e,-u, an e<'J.ua1 and.opposite force on
· , (a)
! o , ::. ~
·,,, ·
i ·.•Is - , , " -.
..:.. .'. L.,. - -. l< ~.

.•. •:•::;;r;·. .·.•.••.:; :;:f.:.•·. · . ,·,. www.puucho.com


·-.·' '
' ',, . '.' .,. ' ' :-
Anurag Mishra Mechanics 1 with www.puucho.com
"\' ..
,, I
rl•· FORCE ANAr~""'
...,-5r,~"-'' -'"r -' i"':~.:r_ , •· · - 2231
;;;L : _," • _ " <1~, A:.~·"'"~=-··-'-'-" . ,;;;.:,-.~~-'---"''-'--'----'-''-C.-·-'--~== ~ ','-· _.,_,~,"
19. In the ,system showp. i~'. figu~~ '·r;:-½·-·---·-", (al a 1 > a 3 > a2 · · ,', · (bl. a,·= a2,a2 =.a,'
m1 >m2 ·,.System is held atrest.bj..'·j ,· · . ' (c) a 1 =a2 =a 3 - , .. ', (d);:a1>a 2 ,a 2 :aa3
thread BP. Just after the thread BP.is l ._ 24. A man has falleli'into ~'ditch of i -_· ~ ·· - ·,
burnt : ' - width d and two 'of liis· trienas' ,ar~ · , · '• " - -1
(a) Magnitude of acceleration of
b th bl cks will be equal ·to . m1Lf'--1.,;1.....,
~lowly' pulling him ciut using a light
rope and. ·two fixed pulleys as .
r· : . , ·. Jj
·
~m1 :_: ; 2 ) g: shown in. figure .. Indicate ·the ' - . · ·- -
( m,.
+ 2m2 , correct statements : (assume, both the friends !ipply
· , equal forces of equal magn/l}lde) ·'
(b) Acceleration of
m 1 will be equal to zero . ,
- (a) The force exerted by both the friends·deqeases as
(c). Accel,eration of m 2 'l½U be upwa!_ds ' the man move up · ·, · _, .
(cl) Magnitudes of acceleration' of two blocks will be · , mg 2
non-zero and unequal ; . , · · (b) The force iipplied by'each friend is , h ;-~ d~ + 4h
4
20. A . particle .i,s resting over a · · - when the man is at depth of h
smooth horizontal floor.. At t = 6, ' ·,. -~c) The force exerted by both the friends inc~~ases as ··
a horizontal' force start's ·acting the man moves up ·
on it. Magnitude · of the fore~
increases with time.according to (df The force applied b~ ea~ fri~nd is 7, .Jd 2
+h
2

law '_F ~- at, 'Where · g.' ~1~ a 49-:,_.


1
:::z:::d~ .=:~:=:==::·: : ': · rd th~ figure shown m,.~ 1 'kg; m 2 = 2kg; pulley ls

·s:,
25.
constant. For figure which of the . ·· . ·. -
·following statement is/are correct? · ·. ·
(a) Curve B indicates velocity against time
(bl Curve B-indicates velocity against acceleration
(c) Curve A indicates acceleration ag~inst time
ideaE At t = 0, both masses touches the
""'"" ,.i _•ram. Afu= F - "
is applied to2pulley (t is in second) then
(g=1Dm/s ):
,
·
' ,
:
i ·• -
iI ·
6tJ I
(a) m~ is lifted off the ground at t = 20 ~ ro 2
(d) None of t:l\ese 1
sec . ,
21. TwoparticlesA&Beachofmass '.[- · ., ·,::--·,1 (b) acceleration of piillhwhen m 2 is about ,o lift off
mare in equilibrium in a vertical ·r ,: a_:r'A. '. • is 5 m/s 2 · · •
plane under action of a
horizontal force F = mg on
·: . , : 2 B ' F~.;g\
: :. . _ I
(c) m
1 is lifted off the ground at t = l_O sec-
0

particle B, as shown in figure. (d) both blocks are lifted off simultaneously
Then: 26. In the following figure all . surfaces are smooth. The
(a) Zf1 = ST2
(cl tane = 2tana
(b)- T1 ./2 =-T245
(d} None of these
22. The magnitude of difference _in ~ - -
accelerations of block of mass· { . JlJ .---~~~J' I .
m in both the cases shown . . . . I -----'-..c
~~- .,~"""''""
. .. I ' ," .
lielowis: ,·:, · · m, . m
(a) g · . . · F-Zmg •. 2m
~ •-
''. (a) acceleration ofwedgeis_greater then g sine
2
(b) .~ . . . (b) acceleration of m is.g~i + 2cos e
(c) acceleration of mis g '.
(c) Zero (d) acceleration· of wedg~ is g sine
(d) g/3i' · 27. In above question 26, the normal .fdrce acting,between:
23. In the figure the block A, B and C of mass m each, have (a) wedge and incline plan~ i~ Mg cose · . ,,
accelerations a 1 , a 2 &a 3 respectively. F,.&Fi are (b) m and wedge is mg cos~·
external. · forces of magnitude · '2 ·, mg - a_nd mg
(c) m and wedge is•zero · ·. . ·
respect~iv~: . - . . . . r ( d) m ~nd wedge is mg siri 8: : > .
·
. . ..
E·I··;l
/ : '..~-,

m : , -\ 1
/>.{ ['~ ~mg B zm_

. . "T[~ ·:. '


F J

• , m ,
~
~ 1
S

.2,~..,.~~'.°
~ I'

m , , .·.
' • •,

2~. ~~j• : . ;; .
28. In the figure shown .."'.i '." 5 f ~
kg, m 2 =10_kg & fnctt<_m f \7okii.
J µ,=0.1·
coeffi_cient between ri11 &· m_2 i'n:·,~11_~-).1 Jlll?:tan
i,s µ = 0.1 and grou°:d. · ,is. l (Smoot_h ?rou~d) - ·
frictionless then: · _, .. :. ·-::: .. . .··• · ..<' .
i . "-.-.--1
j

. J,
' , . www.puucho.com
"
. ,,..
~
Anurag Mishra Mechanics 1 with www.puucho.com

I 224 .,,;,•, ·.•


' - - ' · - - ' - = " ' - - '- - ~ - - ' - ' ·;.•'
· ;:. ·
(a) r{ a horizontal force F ,i, 20 N is applied on mi (b) If0 < tan-i µ the block cannot be pushed forward
then the friction force ·11cting' .on m2 is· 5 N in the, for any. value of F
girection of F (c) As B.decreases the magnitude. of force needed to
(b) Maximum amount of horizontal force that can be just push the block M forward increases ·
applied to m 2 s1,1ch that there is no relative motion
(d) None of these
· between blocks is 15 N
= 31. .In the arrangement
(c), If a horizontal force F 20 N is applied on rri 2
then friction force acting on rrii is 20/3 N in the shown, coefficient of ,.,.
friction for all the [:' ,, ·.
. ~ l .. . ·
.:
A m . Ti ;, ' · .
, direction of applied force
\ (d) Maximum amount of horizontal force that can be .surfaces. isµ _and blocks f.t;;;;;;;;!~-
applied fo mi such that there is no relative motion are movmgwith constant t:.'t!'-- ii",...._._~,-="'·- -J_
speeds, then : ·
, between blocks is 8 N · ·
29: Pi. block of mass 0.1 kg is kept on ·ca) Ti =µmg (b) F= 3µmg
an inclined plane whose angle of ,-----.=,•-;:,:"'·\·t·.···'./l,.,...,•·I (c) Ti= 2µmg (d) F = 5µmg
inclination can be varied from ,i<'"i ) 32. A triangular block of mass m rests on a fixed rough
B.= 30° to B = 90°. The coefficient · ·.•·;: inclined plane having friction coefficient µ with the
of friction between the block & µ =··1·J'". block. A horizontal forces F is
the inclined plane is µ = 1. A force 8·· ' } ;· applied to it as shown in figure
. + 1 .. .
of constant magnitude - mg below, then .the correct statement
' ' 2 is :.
newton always acts on the . block directed up the · (a) Friction force is zero when
.inclined plane and parallel to it. Then : F cos0 = mg sin0 .

~
1·f "
(b) The value of limiting friction
is µ (mg sin B+ F cosB)
(a) 11.f (c) Normal reaction on the block is F sinB + mg.·cosB

d: jt/6 .. lt/4 rr/2


(d) The value
µ (mg sinB-F cosB)
of limiting friction is

33. A body is moving down .a long inclined plane of


inclination 45° with horizontal.The coefficierit ,of
friction between the body and the plane varies as
µ = x/2, where x is the distance moved down the
plane. Initially x = O&v = 0.
(a) When x =· 2 the velocity,of the body is ~g./2 m/s
(b) The velocity of the body increases all the time
(c). At an instant when. v ,;, 0 the instantaneous
acceleration of the body · down the plane is
lt/2 8 g (2- x)
.. 2./2
C.
(d) The body first accelerates and then.decelerates·
,o
!ti .: 34. , Suppose F, FN & f are the magnitudes of the contact
lit:' -··-! . force, normal force and the frictional force exerted by
(d) ~·--mli---.,,f;c.tsi.-.-cy"'.1r-r-+8 one surface on the other, kept in contact, if none of
. . '- ,.- ~~;:~1::i-"'
' . these is zero :
30. · In, the .situa""t1'"·0-n-sh~o-wn__,
in the (a) F > f (b) FN > f
'.'t'rgure.'
~ '· '
the friction
-
coefficient (c) F > FN (d) (FN-f)<(FN+f)
b~tween M and the horizontal 35. Bl.ock A is placed on block B. ~ . · ~-,.·,.···.,
surface is µ. The force F is There is friction between .the . [fl- P • : , ·\
applied at an angle B with
vertical. The cortect statements
blocks, while the ground. is nnJ:n!nm mnlm»
1 1l
smooth. A horizontal force P · ':-""-"'"'...I
are: '
increasing linearly with time, begins to act on A. The
(a) If B > tan-i µ the block cannot be pushed forward accelerations ai & a 2 of A and B respectively are
for any value of F plotted against time (t ). The correct graph is :

www.puucho.com
-~-
Anurag Mishra Mechanics 1 with www.puucho.com

39. The friction . coefficient · I·.


between plank and floor is µ. ·
The man applies, the· m
maximum possible force on
the. string and the system
remains at rest. Then : ~ ------- -

(a) . frictional force between plank and surface is


2µmg
1+µ
(b) frictional force on man is zero
(c) tension in the string is Zµ mg'
. 1+µ
36. In the - shown diagram
(d) net force on man is zero
..•
",;..,.~ I, ~ 1 =.m 2 = 4_kg and m3 = 2
·40. In the shown figure, friction -- · -- -- -- • ·-·· ,
kg. 'Coefficieni: of friction
between m1 and m2 is 0.5.
The mass mt is given a
exists betw-een wedge and block
and also between wedge and
w__·- -\
m I,
. . M
velocity v and it just stops ' - - - - - - - - - ' = ' - " ' fl_oar. The system 1s m . . :
equilibrium in the ·shown ~ ------ - ··· ··· ··' __ _;
at the other end of the mass m2 in 1 sec. Let a1 , a 2 and
a3 be the acceleration m,, m2 and m3 respeGtively, position:
then: (a) frictional force between wedge and surface is
µ(M+m)g .
(a) fort < 1 sec, a1 = 5 m/ s2 , a 2 = a 3 = I. m/ s2 - (b) frictional force between wedge and surface is mg
. . 3 .
(b) fort< 1 sec, a1 = Sm/s 2 ,a 2 = a3 = O (c) frictional force between wedge and block isµ mg
(d) minimum coefficient of friction required to hold
(c) the value of vis 5 m/s
(d) fort> l sec, a1 = a 2 = a 3 = 2m/s 2 · the system in equilibrium is ~

r;-2···_·\
M+m
3 7 • . 0 is a point .at the bottom of a 41. A block is projected with velocity v 0 up the inclined
I . 'A
rough plane inclined at an angle a
to horizontal. Coefficient of
. between AB 1s
fri ctton . -tana
2
- and
r_-
i\ . •
.. - - '
B
! ---·· -. . . ., ... _
' ,, j
,.
,
plane from its bottom at t = 0. The plane makes an
angle 8 with the horizontal. If the coefficient of friction
between the block and the incline is µ: ~ tan a (a > 8)
then frictional force applied by the plane on the block
· · : B·o . . 3taiia B . h
b etween IS - - . 1s t e fort> Vo will be:
I . 2 g [sine+ tan a case] ·
middle point of AO. A block is released from rest at A,
then which of the following graphs are· correct : (a}" tanamg case (b) zero
(c) mg sine (d) tanamg sine

(a)LJ~]- il (b)
r·-.
(.,_ . ,.__ - - -
---·
-!-- _ -~t
'_'!
. 42. In the shown diagram friction •· --- -- --·
exists at each contact · surface / ~
with coefficientµ and the blocks I _ .'
are at rest. Then :
·_ m

'.
-
_'
M ·
, ' e.
---
;
l
·' :
. (a) frictional fo~ce between L_.. - ·-··:- · -· · - ·
wedge and surface is mg sin 8 case
(b) h~rmal force by the surface is (M + m)g
(c) friction force on m kg is mg sin8
(d) net force of m is zero
38. In above question 37 : 43. A sphere of weight W ~ 100 N is kept
(a) velocity of block at O will be maximum stationary on a rough inclined plane
by a ho,izontal string AB as shown in
(b) velocity of block at O will be zero
'figure. Then_: .
(c) velocity of block at B will'.be inaximum·
(a) tension in the string is 100 N
(d) -average velocity of the block is zero·
(b) normal reaction on" the sphere by the plane is
100N 1

www.puucho.com
Anurag Mishra Mechanics 1 with www.puucho.com

[}2s_;~_-.____ ....;__-'-'----"-"" ·='.·,_·,_''af:,.,';,~··C..---~-----~---ME_c_HA.;_N_l(S_,1_,q

(c) tension in the string is lO~ N (a) The car cannot make a tum without skidding.
2 + .;, 3 (b) If the car turns at a speed less than 40 km/hr, it
'
(d) ,orce offri cnon
. on.the sp.here 1s
· ~
lOO N slips down. ·
2+-v3 . (c) If the car turns at the correct speed of 40 km/hr,
44. The position vector of a particle in a circular niotioh the force by the road on the caris equal to mv 2 /r:
about the origin sweeps out equal area in equal time : (d)" If the car turns at the correct speed of 40 km/hr,
(a) Its velocity remains constant the force by the road on tlie car.is greater than ing
(b) Its-speed remains constant as well'as greater than mv 2/r.
(c) Its acceleration remains constant 49. A body moves on a horizontal circular road of radius r
(d) Its tangential acceleration remains c<_>nstant with a ta!).gential acceleration a,.
The coefficient of

t'M-~j-
. 45. ABCDE is a smooth iron track in the friction: between the body and the rpad surface is µ. It
begins to slip when its speed is v, then :
vertical plane. The section ABC and 1 . ·
CDE are quarter circles. Points B and ~· -/ . (a) v 2 '=µrg
D are very close to C. M is _a small · L.. El•..:._••_,.·_'_.:, ,V2
magnet of mass m. The force of . D·f :, ' (b) µg =-+a,
attraction between Mand the track is --·....; __ EL
r
v4
F, which is constant and always normal to the track. M (c) µ2g2 =-+a;
starts from rest at A, then : r2
(a) If M is not to leave the track at C then F ;., 2mg (d) The force of friction makes an angle tan-1 (v 2 /a,r)
(b) At B, the normal reaction of the track is F - 2mg with the direction o( motion at the point of
(c) At D, the normal reaction of the track is F + 2mg slipping
(d) The .normal reaction of the track is equal to F at 50. A particle P of mass m attached to a
some point between A and C vertical axis by two strings AP and BP of '"""---,-
46. A particle· i~ .moving alohg a circular path: The angular length L each. The separation AB = L, P
velocity, linear velocity, angular acceleration and rotates around the axis witli an angular
centripetal acceleration of the particle at any instant velocity 'ro'. The tensions in the strings .P
AP&BP areT1 and T2 respectively, then:
are ro, v, a, a
0 respectively. Which of the following (a) T1 = T2
relations are correct ? (b) T1 + T2 = mro 2L
·cal roj_ v (bl roJ_a (c} T1 -T2 =2mg
(c) ro J_ "it: Cd) vJ_a;, (d) BP will reJ?ain taut only if ro 2'~2g/L
.. 51. As shown below AB represents an infinite r---:---:::7

[r".t.
' '

47. Suppose a machine consists of a· wall tangential to a horizontal


8
~ag~ at the end of one arm._Th~ arm --:-·,,1· semi-circular track. 0 is a point source .of
IS hinged at O as shown m figure . 0,........- ...,,..;F. ·
s,uch that the cage revolves along a / ·v_- light on the ground at the center of the o¥E--~·:
circle. A block moves along the circular
vertical circle of radius rat constant cf----:·-·-G.;;;:::··------)G track with a speed V starting from the A
linear speed v = .,fir. The cage is so j \__ "[ll]~ point where the wall touches the circle. If ~ - - ~
attached !hat the man of weight W, ! B'".,_,_,_.... ·. the velocity and acceleration of shadow along the
standing on a weighing machine L_,_}._ ·
length of the wall is respectively V and a, then :
inside the cage, remains always vertical. Then :
(a) The reading of his weight on the machine is equal (a) V = v cos (;)
to W at all positions .
(b) The weight reading at A is greater than the weight (b) · V = vsec (vt)
- . R
2

reading at E by 2W.
(c) The weight reading at G is same as that at C. .'' (c) a= ( ~ }ec 2
( ; }an(;)
(d) The ratio of weight reading atE to that afA = 0.
2
48. A smooth circular road of radius r is banked for a 2
(d) a= ( ~ }ec ( ; }an(;} 't'
speed v = 401anjhi-. A car of mass ni attempts to go on
the circular road. The friction coefficient between the
tyre and the road is negligible. The correct statements
are:
www.puucho.com

.'
Anurag Mishra Mechanics 1 with www.puucho.com

=t7""~ "' ",- - -·- -~-·- #--~ -·q"f- - - ·-:--


~f;(}!C~(Aj!A~~J;:~::·,.·
,·=---_-~. ·:.'.:i:::••.i .~' -. ~- .. ' ·-1-·· -~·"""'~--··-" ·--- --
•-'--.-"--~
I - ,·
~
.; ' ,'\
.• ..-w.:•~-\--·

52. A curved section of a road is banked for a, speed. v. If (a) S (b) ',Js
there is no friction b~tween road and type. Then : (cl s312 (d) None of tliese
(a) a car moving with speed v will not slip cin road 58. A ring of radius' r' and mass per unit length' m' rotates
(b) a car is more likely to slip on the road at speed with an angular_ velocity 'ro' in free space then :
higher than v, than at speeds lower than v (a) Tension in ring is zero
-(c) a car is move likely to slip on the road at speed (b) Tension will vary at all points
lower than v, than at speeds higher than v (c) Tension is constant throughout ring
(d) a car cannot remain stationary on road ahd will- (d) Tension in stri~g is mro 2 r 2
start ~lipping ·
A body moves on a horizontal ~ircular road of radius r,
53. A tube of length 'L' is filled completely with an in with a tangential acceleration Uy, Coefficient of·_
compressible liquid of mass 'M'' and closed at both friction between the body and road surface is µ. It ·
ends. The tube. is then rotated in a horizontal plane begin to slip when it's speed·is v, then :
about one of it's ends with a uniform angular velocity
(a) · v 2 = ,trg
'ro'. Then which of following statements are true : v2
(a) The force exerted by liquid at the other end is (b) µg =-+ar
r
. 1Mro 2L , v4
2 (c) µ 2g 2 =_._+a;'
r2
(b) Ratio of force at middle and point of the tube will
be 4;1 (d) The force of friction makes an angle
(c) The force between liquid layers linearly with the tan -J ( ~ ) with directiqn of motio~ at point of·
distance along the length of tube_ a 1. X r
(d) Force is constant slipping.
54. Aparticle of mass m describe circular path of radius 'r' 60. A simple pendulum has a bob of mass m and swings
and its radial or nmmal or centripetal acceleration with an angular amplitude qi . The tension in thread is
depends on time_ 't' as aR = Kt 2. K is +ve constant. T. At a certain time the string makes an angle 0 with .
Then: the vertical (0 S: <I>) : · ' -

(a) at ~ time 't' fore~ .acting on particle is (a) T = mg case for all values ofe
m-,/kr + k 2t 4 ' , (b) T = mg c,:,s0.for only 0 = qi
1
(b) Power developed at any time t is mkrt (c) T=mg,fo~0=cos- [½c2cosqi+l)]
(c) Power developed a~ any time t is mk~'2 /r3/ 2t
(d) Twill be larger for smaller values of 8
(d) Tangential-acceleration is also val)~ng. ·
61. A particle of mass m moves along a circle of radius 'R'.
55. Aparticle of mass' m' describes circular path of)adius The modulus of the average vector of force acting on
'r' ·such that its kin~tic energy is given by [( = as_ 2 • 's' i_s the particle over the distance equal to a quarter of the
the distance travelled, 'a' is constant : , , · · , circle is : · '
(a) Power ·developed at distance; is' proportional io s2
(a) zero if the particle moves with uniform speed v
(b) Tangentiai'accelerationis proportional to 's., 0. 2 ' '
(c) Radial acceleration is proportiqnal to s3 - · (b)· ' mu if the particle moves with uniform speed v
itR
(d) None·of these . 2
56. Three particles ·describes circular path of 'radii r1 ,' 12 (c) z,./imv if the particle moves with unifonil speed v
11R :,
and r3 with constant speed such that all the particles
take same time to complete the revolution. If (d) ma ·if particle moves with constant tangential
rot,ro 2 ,ro 3 be the angular velocity, v 1 , v 2 , v 3 be linear acceleration 'a', the initial velocity being equal to
velocities and_ a1 , a 2 ,a 3 be linea'r acceleratior; tha~ : . zero
(a) ro 1 :ro 2 :ro 3 = 1:1:1 62. Aparticle'~' moves afbqg a circle of radius R = 50 cm,
so that its radius vectllr 'r' relative to the point 0
(b) vi :v 2 :v 3 ,=·r1 :r2:r3
rotates with the c'ilns&Jmt a~gular velocity ro = 0.4
(c) a 1 :a 2 :~ 3 aal,1:1 . _· ·, rad/s. Then : J · .
(d) a 1 :a 2 :a3 =r1 ·:r2 :r3 (a) lirn;ar velocity of particle is 0.2 m/s
57. A particle of mass m describes a circular path of radius (b) . linear velocity of particle is 0.4 m/s
'r' such that speed v·= a-Js ( S is distance traveled).
Then power is proportional to :
www.puucho.com
Anurag Mishra Mechanics 1 with www.puucho.com
'\

(~~ magnitude of net acceleration is 9.08 m/ s 2 67. Two blocks· of masses mi = 2 kg [


(d) acceleration of particle is :tero
63. Two bodies are moving with constant
speed v clo~ise andi,:!ll"e initially
diagonally OPP.CJSite. The },~rticle B now
achieves a tangential acceleration of a
m/s 2 • Then: : ' · ·.
,:©
.,
'~
R
A.-,v.
.
and rrii
= 4 kg hang , ,over a
. massless pulley as shown in the
figur~. A force F0 7 lOON acting at
the axis of the pulley accelerates ,;kg
the system upwards. Then : · I
~

4kg
(a) acc.eleration of 2 kg mass· is <-.....;..cc.-_..;
15~/s 2 ·: •

(a) they c~llid~ after tin}e ~ . v· B·


+-
(b) acceleration o_f 4kg mass is 2.Sm/s 2
(c) '.'cceleration of both the masses is same
(b) ~~y collide afte; time ~ 21;; (d) ·"\!cceleration of both the masses is upward
' ·,
(c)' · relative velocity just before collision -is .JrcaR 68. Which of the following is / are incorrect:
(d) -~~lative velCJcity just before collision is .J2rcaR (a) If net normal force on a surface is zero, friction.
will be z¢ro.
64. A P1\r1;ii:le P is attached by means of two equal strings
to 'two points· A and B in same vertical line and '(b) Value ofstatic fii~tion is given byµ ,N.
desct:1be~ horizontal ~ircle_ with uniform angular speed . (c) Static friction oppo~es relative motion between
~ {2i"where AB= h. ·_ . · two surfaces is contact.

· ...-ii]. .·
~,;:'' (d) Kinetic friction reduces velocity of an object.
69. A spring block system is
placed on a rough i m.:,;:,::;
?orizontdaltfloor.dTh: bhlock ( ..
.
. ·
,J::1:;,,,i,;. :
1s pu 11 e owar s ng t to ~---·-- -~--
give spring some _elongation and released. Then: .
(a) the bloc1' may s~op before the spring attains its
(a) T1 > T2 ,·, • (b) T1 : T2 = 5: 3 natural length
(c) T1 :.T2 ~ ;/5_ : -J3 '. (d) T1 = T2 (b) the block m\lst stop with spring having some
65. A particle is &cted upon by constant magnitude force compression
P~il>endiculaf to it which is alw;iys perpendicular to (c) the block may stop with spring having some
velocity ofj>4rticle. The motion is taking place in a compre_ssion
plane it follows that : (d) it is not possible that the block stops at mean
(a) vela~!~ i~ constant ·position
(b j accel~riition is constant 70. In the above situation the block will have maximum
(c) KinetiJ'~~etgy is constant velocity when:
(d) ii lllOVes in circuiar path (a) the spring force becomes zero
66._ A parti~le 9f mass m moves in a.conservative force field (b) the frictional force becomes zero
x
along' aifis where the potential energy U varies with (c) the net force becomes zero
position coordinate x as U = U0 (1- cos ax),U0 and a · (d) the acceleration of block becomes zero
being positive constants. Which of the following 71. A book leans against a crate on a
statement is true regarding its motion. Its total energy table. Neither is moving. Which r.so-cratel
is U O and ~tarts from X = 0. of Lhe following statements ! '. :1
.,_~ .j

(a) !i]e 'cceleration is constant concerning this situation is/are


(b) It's speed is maximum at the initial position. incorrect ?
(c) It's maximum x coordinate is~ (a) The force of the book on the crate is less than that
2a . of crate on the book
rd) It's maximum kinetic energy is U0 (b) Although there is no friction acting on the crate,
there must be friction acting on the book or else it
will fall

www.puucho.com
Anurag Mishra Mechanics 1 with www.puucho.com

(c) The net force acting on the book is zero 73, In the sy~tem shown in the figure m1 > m2'. System is
(d) The direction of the frictional force acting on the held at rest by thread BC. Just after the ·thread' BC is
book is in the same direction as the frictional , burnt:
acting on the crate
r, . . ' .
72. An iron sphere weighing 10 N rests in a V shaped
smooth trough whose sides form an angle of 60° as
shown in the figure. Jhen the reaction forces are:

·

14
.. li;

G ___ ____1~---'~____ 60° 60~

.c_(ii-'-i)_-.__...,
(a) initial acceleration of m2 will be upwards
(b) magnitucje of initial acceleration of both blocks
will be equal to ( mi -. m2 ) g
(a) RA= ION andR 8 = 0 in case (i) ·
m1 + m2
(b) RA = l0N andR 3 = ION in case (ii)
dR 10 N , . . ("') (c) 'initial acceleration of m1 will be equal to zero
(c) RA=. .f:3Nan
20
8 = ../3 m case m (d) magnitude of initial acceleration of two blocks
(d) RA = l0N andR8 = 10.N in all the three cases . will be non-zero and unequal.

www.puucho.com
Anurag Mishra Mechanics 1 with www.puucho.com

: 230

Compreh~nsion Based Problems

-:: -"· --, 3. The tension on side ofheavier\nass will be:


I pA s.S'A;l(E 1 ...: ~,~--- (a) m1g (b) m 2 g
Effect of friction between pulley and thread : (c) '2m 2g (d) 2m 1g
In ideal cases i.e., when pulley and strings are massless and 3 3
,no friction exists at any contact surface, then tension in the 4. The tension c;m side of lighter ll_lass will be:·
string is constant throughout its length. But consider a, (a) m1g · (b) m 2 g
massless pulley and massless string but friction exists:
(c) 217\2g (d) 4m1g
,between pulley and string With coefficientµ. Then tension ·3 3
:at the two .ends of the pulley will be different. As .shown in.
figure, consider an element of string :
, ,_ t ., '• ,- ,·r-rr'~fi;t
dN = (T + dT) sin de + T sin de
2 2 PASJJl\'.G'E ·., ._ .?.Ji~~
, Cons!<le; tl)~-situ~ti~~.sho~ in figure in which a block 'A'
,+.dN of mass 2 kg is plac~d over a.biock.\B' of inass 4 kg. The
µdN i
8 8 combination of the blocks are 'placed on a inclined plane of
Tcosd2 ~ : ' (T+dT)casd2 ,
,inclination: 37° with horizontal. The:coefficient of fyiction
; ,··· e d;e___ ~_T+dT . between block B and inclined p1ape is µ.; and in lletween
m, the twq,
b\oci\5 is µI. 'f~e system "is r.eleased from rest:
. (Take_ g· "';'lcim/ sec 2 )
--···:· de· . de. . ,/ . : ' ' :
,,' ' '
·
- - --: ~ .... ,-
· • ·

,(T + dT)cos- -Tcos- -µ dN = dr/a= 0


, · 2 2
· (massless string)
de.=µ dN
dTcos-
2 37'
.____ __ ,------ - ,-- - - ~I
,._ I

dTcos de= µ[er+ dt) sin do+ Tsin de] '


.

2 2 2 1. Ifµ. 1 = 0.8,µ 2 = 0.8then:


(a) both blocks will move ,together
de
dT -cos-=µ , de + dT ·Sm-+
[r sm-. · . de T sm~
. de]
2 2 . 2 2 (b) only block A will move and blockB remains at rest
(c) only block B will 1n~ire and block A remains at
de de]·
dT=µ [ T·-+O+T- =µTease rest.
2 2·
(d) none of the blocks will move
JliTdT = l"oµ. de ·(T.~ J= µ1t T
T2
=> In =} ~ = eµ:c 2. In the previous question the frictional force between
T, block B and plane is :
!Suppose coefficient of friction between the string· and' (a) 36 N (b) 24 N
' . 1 .
pulleyis µ. = - . (c) 12 N (d) 48 N
'-·-· · · - · - •. , lt.
3. If.µ· 1· = 0.5,µ 2 = 0.5, then :
1. What should be the ratio of heavier mass to lighter ~a) Both block will move but with different
mass for no motion ? · accelerations
(a) e (b) ~ (b) Both block will move together
e
(c) Only block A will move ·
(c) eL (d) e"
(d) Only block B will move
2. If m2 = 2em,, D.1.en- acceleration of each mass is :
4. The frictional force acting between the two blocks in
(a) g (b) g/3 the previous question is :
(c) eg/3 (d) zero (a) 8 N (b) 6 N
(c) 4 N (d) 0
www.puucho.com
Anurag Mishra Mechanics 1 with www.puucho.com

[ FORCEA~ALVSI~- ~-~-- -__: .• -_- ___ -_ -~~- 2311


5. Ifµ 1 = 0.4,µ 2 = 0-5 then: 5. The acceleration time graph for 2 kg block is:
(a) Both block will move but block A will slide over a
the blockB
(b) Both block will move together 1/31----/
(c) None of them will move ~ t
(d) Only block A will move
6. The frictional force acting between the blocks in the a
previous case will be:
(a) 8 N
(c) 4 N
7. Ifµ 1 =0.5,µ 2 =0.4,then:
(b) 6.4 N
(d) zero
(c) ,)---!
, ; ~' (d)
1/3-----~
i i
6 28/3

(a) Both blocks will move but with different 6. The acceleration time graph for 4 kg block is:
acceleration I ''"
8 8
(b) Both blocks does not move
(c) Only block A will move
(d) Both blocks move together (a) : 1 ••••
/3 , ~ : (b) 1/3~
,-··· t t
8. The frictional force acting between the blocks in the · 6 , 6
previous case :
a a
(a) 8 N (b) 6.4 N
(c) 6 N (d) zero
(c) : 1/3~-----; (d) 1/3~
0 ' t
'' t
6 28/3 6 28/3

: PASSAGE 3
7. .The frictional force acting between 3 kg block and
;In the given figure, the blocks of mass 2 kg and 3 kg are'
ground w.r.t. time will vary as:
placed one over the other as shown. The surface are tough
with coefficient of friction µ 1 = 0.2,µ 2 = 0.06. A force : -1,
F = 0_5t (where 't' in sec) is applied on upper block.in thei 41---~~.--
:direction shown. Based on above data answers the j 3 -------- .. .Jh.;
(a) I.hi \
'following questions. (g = 10 m/sec 2 ) 6 28/3
I
: F =0.5t / µ1 =0.2 'I -- -

µ2 =
-~.,
a.as-;tlXJ""\ '
•f,
-..- ~- --- J __

3
1. The motion of blocks 2 kg and 3 kg will begin at time (b)
t = -,- respectively : 6 28/3 • .... ?I'"

(a) 8, 8 sec (b) 6, 8 sec


I__
(c) 8, 6 sec (d) 6, 6 sec ·-1;
2. The relative slipping between the blocks occurs at t =
(a) 6 sec (b) 8 sec I ;3
28 (c) ....-··· I
I
(c) - sec (d) Never 6
3 ,_
3. The frictional force acting between the two blocks at
t = 8 sec. (d) None of these
(a) 4 N (b) 3 N 8. The friction force between the blocks and time graph
(c) 3.6 N (d) 3.2 N is:
4. The frictional force acting between the blocks at t = 10
sec is : ·41----=--
(a) 4 N (b) 3N
!3 ------- -- 0, i
(a) 0 ! ! t'
(c) 3.6 N (d) 3.2 N 6 28/3
www.puucho.com ---· --- ·- . I
Anurag Mishra Mechanics 1 with www.puucho.com

j232 .- ., - . '.
._. _.M~CHANICS,1 j

(al mg · (bl mr(cxtl 2


(cl m~rg~2 -+_r_2-(cxt_l_
4 (dl m~~g~2 +_(_ra_l~
2

2. Friction force acting on bead at timet (< t 0 l is given


(dl None of these
by:
r··-- - .... --,---. - ':--'!.?.
1 (al µmg (bl mr(atl 2·
! ' . . ., •.-,n --.,. ,• .,,.7 •: 4
, P}A,S_Sll,.G;_~ ~1 G: '--,. (dl µm~g2 + r2(cxtl4
Irt the adjacent figure, x-axis has been taken down the - 3. If the bead start sliding at t =t 0 then value oft O is
inclined plane. The coefficient oHrictioh varies with ·x as given by:
µ = kx, where k = tane. A block is released at O. .'-er====

\i . ·_·l~l-. I

I (al
µ~g2 + (ral2

L •-
:I ··----~-~--~
I
-~-===c.....---
1. The maximum velocity of block will be :
x! :;,'

' J µ~g2+(ral2 {mg


(al ..Ji (bl sine Jg (cl
ra 2 -v~
(cl cose ·Jg (di tane Jg
2. Maximum distance traveled by the block :
(al lm (bl 2_m
~;:~N;e;;~;s_~{- - · ~ - . -'. • , :
(cl 3 m (dl ~m
3. Frictional force acting on the block after it comes to
2
[A very sma_II c_ube of mass 2 kg is pla_ce·d·. on the surface ofa
furinel as sho\vn in figure: The funnel. is,totating,about'ifi;
rest: -.;ertical axis,of syrnmetrywith"'iingulat velocity'ro': The~all
(al mg sine (bl 2mg sine of funnel mai<es"an angle 37° with:horiiontal. The distance
. (cl mg sine of cube from the axis,_of rotation is' 20 cm and fric'°i;i,:m
2 .
(dl . 2mg cose coefficient·is µ. (Take g = lO_m/s~l .,
0
.. . .
. . ,---j).,
4, Frictional acting on the block just before it comes to ·
rest :
. I_..,..__
(al mg sine (bl 2mg sine ·, i r:::20cm
(cl mg sine ·I
(dl 2mgcose ;, I

,----, .
--······
2
--··-. :. - _~...,,--- .. ';)
'; '/" 'j
e
::'l
;',::.
-.'_ ~:~_.; ;_)·/."
! ir:il s1sti- olJ:
, ,~n.
1
-- ~~
S
If \ 5
'· , "•!

'--~~~--~-;~.:__:l.__·_,,'-.--'.~'--·___?'..-''
le·''· ,

:The figure shows a .r9d _wl\ich ~t;trts 'rotating with an~lar


Iacceleration a about verticru:axis·passing through one ,of its 1. The friction force acting between the block ancl surface
;end (Al in horizontal plane. A bead_of.mass mjust fit's· the (if µ = 0.3l of funnel at ro = 5 rad/ s is _:
'rod .and is· situated ·at a clistance '·r' from.end A: Friction (al 6.6 N (bl 4 N
;exist between rod ahd the bead with coefficientµ. As 0e (c) 2.2 N (d) zero
1angular velocity of r~~ increas~s the b~ad starts sliding ove~\
[!P-5'_.r_Qc;l _(siiy~fter.!lJn
..~.lo), ~---- ·.

www.puucho.com
Anurag Mishra Mechanics 1 with www.puucho.com

I FORCE ANALYSIS 2331


2. For what value of OJ, there would be no frictional force
acting between the surfaces : fA
" ..s··sii,G'iE'·
(a) .5 rad/sec (b) H rad/sec .
·- .__ -- {iu
i.m c;,
it,
jA body of mass m = 1.8 kg is placed on an inclined plalle,
Ithe angle of inc)ination is a = 3,7" 1 and is attached to., the
(c) ..J?, rad/sec (d) -./40 rad/sec ltop end of the slope with a. thread which is parallel to the
3. The maximum value of angular velocity for which no slop. Then the slope is - moved With a horizontal
relative slippjng occurs and also direction of frictional acceleration of a. Fraction is negligible.
force is : (takeµ = 2/3) ---7
(a) J¥ rad/sec; down the surface of funnel -4i
(b)

(c)

N
rad/sec; up the surface of funnel

rad/sec; up the surface of funnel


L------·'-----'-i__,_ :
1. The acceleration, if the body pushes the slope with 'a
3 '
force of - mg is:

(d) N rad/ sec; down the surface of funnel (a)


3

~ m/s 2 (b) 0.5 m/s 2

4, The minimum value of angular velocity for which


(c) 0.75 m/s 2 (d) ~m/s 2
relative slipping occurs and also the direction of 6
frictional force acting µ = 2/3 :
(a) N (b). J¥ 2. The tension in thread is:
(a) 12 N
(c) 8 N
(b) 10 N
(d) 4N
(c) H; (d) . ffs 3. At what acceleration will the body lose contact with
plane:

ii1i'si:(i '-. 1, ~~ ,: . ,_j)


J,.__,.._.,..,,L-;.;J.: __ _

A car is moving with speed v and is taking a tum on ~


l 40
(a) 3 m/s. 2

(c) 10 m/ s2
(b) 7.5 m/ s2

(d) 5 m/ s2

circular road of radius 10 m. The angle of banking is 37°.


T_he driver -wants that car does not sli,P on the road. The. fifsi~G}j g@ ~
[coefficient of friction is 0.4. (g = l0m/ sec 2) _·_ __
A lift can move upward or do~ward. A light inextensible
1. The speed of car for which no frictional force is string fixed from ceiling of lift with a frictionless pulley and
produced is : tensions in string T1 • 1\vo 'masses of m1 and m2 ~re
(a) 5 m/sec (b) s./3 m/sec connected with Inextensible light string and tension in this
(c) 3-/s m/sec (d) 1_0 m/ sec string T2 as shown in figure. Read the questions carefully!
and answer. · ·
2. The friction force acting when v = 10 nvsec
and mass of
car is 50, kg is :
(a) 400 N (b) 100 N
(c) · 300 N (d) 200 N
3. If the car were moving on a flat road and distance
between the front tyres is 2 m and the height of the
centre of the mass of the car is lm from the ground,
then, the minimum velocity for which car topples is :
(a) 5 m/sec (b) s./3 m/sec
1. If m1 + m2 = m and lift is moving with constant
(c) 3-/s m/sec · · (d) 10 m/sec velocity then value of T1 :
(aJ·;,,mg (b) =mg
(c) :,; mg (d) > mg

www.puucho.com
Anurag Mishra Mechanics 1 with www.puucho.com

2. If m1 is very small as compared to m2 and lift is moving


with constant velocity then value of T2 is nearly:
(a) m 2g
~;;5,5f~;~~ T~ ,c 1~·~·-·~ --~--~··;o
,....2_3_4_~~---~··£'-----..:..:....'-'-"----...:......::--:.:::::···===~:::::·.·!:'=;r~."'--~~--MEC~~N·cs:!J

I~ th:·~e~sho~:~~assqfthe trolley is 100ifand it,


(b) 2m1g
can,move'.without friction on the.horizontal floor. Itslengthl
. (c) Cm1 + m2)g is 12in. The mass of the gidis:sokg; friction exists between
.Cd) zero the· shoes of the girl and. the: trolleys upper surface, with
3. If m1 ; m2 and m1 is moving at a certain instant with µ ;1/3. '.J:'lfe girl can run·witl). a D1aximum speed ;,9m/s on
velocity v upward with respect to lift and the lift is the surface•ofthe trolley, with respecttothe surface..Att ~ o
moving in upward direction with constant acceleration the girl ~t!l.rts nmJling from left:· to thti _right. ~-e trgMy: was
(a < g) then speed of m1 with respect to lift: initially, sfationary. (g ; lQm/l) . , I
(a) increases
(b) decreases
(c) remains constant
(d) depend upon acceleration of lift
•.. ·~,.~
. i 1. The minimum time in which the girl can acquire her
.,... .e1 !"'· re, ml Ii! ' maximum speed, for no slipping, is:
~A~.tll:~:! \!JJ ~- .,·.,' . . I (a) 1.5s (b) 1.8s
A ,,;hot putter with a mass Rf 801<Kpushes the iron bill of; (c) .2s . (d) None of these
mass. of 6. kg from a stancliitg position acc!'!lerating it;
2. The total kinetic energy of system (troliey + girl) at the
uniformly.form rest at an angltof 4s with the'horizonta!I
0

instant the girl acquires her maximum relative. speed


1
during a, time. interyal of in
'isec:onds. The bail foar,es his1 with respect to troliey, is:
~~!~~e?;:;~;~,:~~ ap~ve_;~e~evel ground :md hi!s thej (a) 1350J Cb) 1250J
(c) 2475J (d) None of these
1. The accleration of the balll in shot putter's hand:
2 3. The displacement of the trolley by the time the girl
(a) 11.Jz m/s
reacltes, the right end of the trolley, i_s: ·
(b) 10W2m/s 2 (a) 6m Cb) 12m
(c) 9W2m/s 2 (c) 3ni (d) 4m
(d) 9.Jz m/s 2 4. The minimum time in which the girl can stop from 9m/s
relative speed, to zero relative speed, without causing
2. The horizontal distance between the point of release
her shoes to slip is:
and the point where the ball hits the ground:
(a) 5/3 s Cb) 4/3 s
(a) 16 m
(c) 9/Ss (d) None of these
Cb) 18 m
5. At a certain moment when the ·girl was acceierating,
(c) 20 m
the earth frame acceleration of the trolley is found to be
(d) 22 m 1 m/ s2. At this moment, the friction force between the
3. The minimum value of the static coefficient of friction if girl's shoes. and the trolley's surface is: ,
the shot putter do.es not slip during the shot is closest (a) 200N Cb) 150N
to:
(c) l00N (d) None of these
(a) 0.28
6. Suppose the girl accelerates slowly, at a constant rate,
(b) 0.38
and acquires the relative speed of 9m/s only when it
(c) OAS reacltes the right end of the trolley, then, what must be ·
(d) 0.58 the earth frame acceleration of the girl ?
(a) 2.5m/s 2 (b) 2.25m/s 2
(c) 1.125m/s 2 (d) 3.375m/s 2

www.puucho.com
Anurag Mishra Mechanics 1 with www.puucho.com

i FORCE ANALYSIS --- --·- ---- -


'--,-·--~---#~----·-· ~--4 - - · ~ ------------ __ . ____ ._, ---- ----- - -

m
(A) Minimum value of--'- so that m (P) --~
M 3
1. · A motorcycle moves around a vertical circle with a ,slides down
constants speed unaer the influence of the force of
M
(B) value of~ so that m (Q) 1
~ ~
1Minimum
gravity w, friction between wheel and track f and
. . ~ m
normal reaction between wheel and track N : :slides up
(C) :Value of!!!. so that friction force on (R) 3
M 5
(A) :Conitanrmagnitude·
,~ •'r, :.. ~:: ( ••, •
'N .
:. t
, (P)
, : I •
1m fa zero
• . I, . '
(B) iD.ir<(qted towards;(Q) IN +f (D) 'Ratio of vertical component of (S) 5
:cen\i'Ei when value ·in· I acceleration of m and acceleration
,of M:,
i~~n:~~ro < . j
(C) · fl;'otal reaction force, by' (R) ;f + ~ 4. A river is flowing with speed 3 km/hr west to east. A
!track • · ! -' man swims with speed 5 km/hr in still water. Man is at
(D)
',When. ;notion is. along, (S) · ;~
.
~ c.,f
south bank of the river. Match the column-1' with
' . .
;veiticaltheval'Ue is zero;·
·1.-,+w+
,
direction of velocities of man w.r. t. ground in
1
column-2.
2. A block is projected with an initial velocity v Block on a
long ~Qnveyor belt moving With velocity V Block (at that
. : 'instant) h~ving constant 'acceleration aB,It. Mark the (A) ,Man swims at an angle· (P)
correct option regarding friction after long time 1127' from river flow
(friction
•' ,,
coefficient
,, . betweeri
' block and belt =µ). If: i
I
(B) ,Man swims right angle (Q)
(A) 'v 81;ik = 2v'a,r, and da,r, =0 , (P) zero 'to river flow
I
' . '
'
(B) iv block = 2va,It and aae1, > µg 1 (Q) J, static friction (C) ·Man swims at an angle• (R) '
· ;co< f, < Al 143° from river flow
(C) ;varock. = 2va,r, and a 6,r, = µg (R) Ji limiting
(S)
' · · ' friction
(D).'. JvBI;,ci:' = 21/~elt an_d aaelt < µi; , (S) jK kinetic friction 5. A particle is moving on a straight line. It is initially at
rest.
3. The inclined s·urface is rough withµ = .!. For different v = instantaneous velocity
. 2
values of m and M, th~ system slides down or up the P = instantaneous power S = displacement
plane or remains stationary. Match the appropriate F = force t = time
entries of column-1 with those of column-2. Mathe the possible expression of the quantities in
column-1 with the situation in column-2

(A) ,v
' 3
=S (P) P =constant
(B) •v 2 =t '(Q) -p oc V

(C)'v 2 =S 1(R) F =constant


(D) v =t 1
(S) F=-
V

,(T) p =t

www.puucho.com
Anurag Mishra Mechanics 1 with www.puucho.com

[236-- .. --~-'- - - - - - · - - - - - - - - - - - - - ' ' - - - - - - - MECHANfcs.n


-----·-.,-~
6. Match the column:
'' - ,.. ···- - . -- -- -,
~

if sine µ=O 11

!L(. ~··F F case Mg


y

F = lOON, m = 7.5kg

.,Ic~t~fi!J.,\:
(A) .0 = 37° , (P) if is upwards I • '
i (A) !Just after string W breaks (P) aA =0
(B) 0 = 45° 1
(Q) If is downwards
.:eI = 53 • (B) Just after spring X breaks (Q) ;aB =0
(C) 6
: (R) If is static
I
..
. ( C) Just after string Y breaks !CR) :ac = 0
, (S) If is kinetic I
(D) Just alter spring Z breaks' Ii (S) :aB = Uc
J Lift can move in y-axis as well as along x-axis. A ball of
mass m is attached to ceiling of lift with inetensible 9. In the situation shown, all surfaces
light rope and box of mass mis placed against a wall as are frictionless and triangular
shown in figure. Neglect friction everywhere. wedge is ' free to move. In x
f ··-y·~-- -- --·-----; column-2, the direction of certain
vectors are. shown. Match the a
direction of quantities in '.mliirtm=nmmilim
T Column-1 with possible vector in column-2.
m

!acceleration of
;block X relative to:
1
1ground

(B) iaccel.eration ofblockX 1' (Q)


(A) ,In figu_r_e lift is moving along x-axisl (P) ;zero
!then.value ofT may be
I .
I
I
relative to wedge
i .
/
(B) :Lift moving toward right along, (Q) > mg (C) :normal force by block! (R) : · · r \ : -

!x-axis with decreasing speed, then: ;on wedge l e ·. :


;value of N may be I
I
:
'
: :
'
1
I '
(C) :Lift is moving in upward direction! (R) < mg ' I
(D) Inet force. on the wedge ' (S)
:(y-axis) then value ofT may be l I •
(D) ,Lift is ,moving in down;ward: (S) = mg 10. See the diagrams carefully in Column-1 and match
:direction with constantvelocitythenl each with the obeying relation (S) in column-2. The
value of T may be string . is massless, inextensible and pulley is
frictionless in each case. a=g/3, m= mass of block T =
8. In the diagram strings, springs and the pulley are light tension in a given string, apulley = acceleration of
and ideal. The system is in equilibrium with the strings movable pulley in each case, acceleration due to
taut (T > 0), match the column. Masses are equal. gravity is g.

www.puucho.com
Anurag Mishra Mechanics 1 with www.puucho.com

1 FORCE ANALYSIS 237'


·:. :.- =- :. . :- =- - :.:.:··=·-=·==-=--=-=-=-=:::::::::::::::::;;;;-------·
.=-·::::-·:..:-·;...::-=·-:..:.·.:..:---=---.:...:·
<1
-- -.. -------·- -- --· -- - -- -----·-- '
1----~-
Column-1 Column,i! ,j_' 12. Column-! shows certain siruations and column-2
shows information about forces.
(A)

(A) ' Situation


(P) ,Fi+ F + F
2 3
is
centripetal force.

(B) a ( Q) apulley ,; a

Front view of a car roundi\lg a


T
curve with constant speed
m
(B) (Q) Fi is sta\ic
friction
(C) (R) T>mg

Passengers in a rotor not


T ,sliding relative to rotor wall
m ,cylindrical rotor is rotating
a with constant angular velocity
about its symmetry axis.
(D) (S) Force
support
on fixed (C)
(R) F1 can be in
T1 > (3/ 2)mg direction
_, opposite to that
F, shown in figure.

a
·Particle kept on rough surface
11. A block is placed on a of a bowl, no relative motion
rough horizontal surface. A
constant force F is acting F~~0<9~!
-rm-=-2-,/i-lkg_____ µ=1
on the block as shown in ,mn1111min~
!
-c· of particle in bowl, bowl has
constant angular velocity.

the figure.
' (D)
(S) Fi_+ F2 ~ 0
Column-! gives the magnirude of force F and
column-2 gives information about friction acting on
the block. Match the entries in column-! to all possible
entries in column-2.
. ..
, , Column-1 Cohi'mnc2. ." ' _.,.
. 9 . . -
(A) lSN (P) Static friction 1Car moving on a banked road
with constant speed, no
(B) 20N (Q) Kinetic friction ·sideways skidding
(C) 25N (R) Zero friction (T)
(D) 30N (S) Limiting friction
(T) Magnirude of friction is equal to
:magnirude of normal

www.puucho.com
('
Anurag Mishra Mechanics 1 with www.puucho.com

I 238 l)IECHI\NIC~:U

AN9WER9
·--· _.,.,,,.-~,
. ~ev~:1: Qnly ()ne Al~~~n_ative is C~rrfi!ct~

1. Cd) 2. Ca) 3. 'Cc) 4. lcb) 5. Cc). 6.


I
,Ca) 7. ' (d) 8. •:c~)
I'
I I
9. (c) '
10. : Ca) 11. ,Ca) 12. !Cb) 13.
i
!Ca) 14. :(c) 15·_ ,(a) 16. (b)

24 . ttd).
1
17. Ca) 18. Ca) 19. :(b) 20. 'Ca) 21. Cb) 22. :Cb) 23 . 'Cb)

25. , (b) 26. . Ca) 27. Ca) 28. ;(]:,) 29. :Cd) 30. Cb) 31. ,(b) 32. !(b)
33. Cc) 34. :(b) 35. (c) 36. .(b) 37.
'
•(b) 38. (d) 39. Il(d) 40. i(c) \
' F"
41. (a) 42. (b) 43. ·Cd) 44. lee) 45. : (d) 46. :ca) 47. i(c) 48. :(d)
,,
' Ca) I'
49. (c) 50. Cb) 51. Ca) 52. !cb) 53. 54. (d) 55. :(d) 56. 1(a)
I ! L.. . t

57. (b) 58. (d) 59. Cc) 60. !Ca)


i
61. (c) 62. .(a) 63. CcJ I 64. ;Cb) I
I ::::' , j
1(d) . I 72.
65. Ca) 66. Ca) 67. Ca) 68. i(d) 69. ,(b) 70. ,Cb) 71. • I
!(b)
i I

.(~) i' ichJ


1Cd)
73. (c) 74. (a) 75. Cc) 76. (b) 77. 78. '(d) 79. 80.
''Cc) I. . '. I
81. (a) 82. (d) 83. :Cc) 84. ;Cc) 85. 86. (a) 87. '.Cd) : 88. (c) " i
' . '
89. (a) ' 90. (a) 91. (d) 92. ,Ca) 93, :ca) 94. '(d) 95. :ca) 96. CaJ.· I
97. ·(a) 98. CaJ 99. 'Cd) 100. Cc) 101. 'Cb) 102. Ca) 103. :CcJ

I
1. (b, d) 2. (a, c) 3. (c) 4. . Ca, b) 5. ;Cb, d) 6. Cd)
7. (a, c) 8. (a, c) 9. (a, b, c)
'
10. :(a, b) 11. ;Ca,c,d), 12. Cb)
'i
13. (a, d} 14. (a, b) 15. '(a) 16. Ca) 17. !Cc, d). 18. l (c) ·' 1'
I ' ~, '
19 . . Cb,c) 20. (a, b, c) 21. ,ca, b) 22. .(b, c)
I
23. :Ca) j 24. !Cb, c);, :< . '
' t, ' :
25. :(a, b, c) 26. (c, d) 27. :ca, b). 28. 'Cb dJ
' '
29. :Cb)
I
30. 1Ca, c)
l I_ · ._ - ... r 1·1,;.
31. ,(a,d) 32. '(a, c) 33. (a, c, d) 34. iCa, c, d) 35. ·C~) j'."36. : (~; .c,d)t '
' I ,
37. Cb, c) ' 38. (a, c) 39. :(a, b, c, d) 40. !Cc, dJ 41. :cc) l 42. i(a, c, cl) ,
' Cb, c, d)
43. , (a, c, d) 44. (b, d) ' 45. :(a, b, c, d) ! 46. •(a, c, d)
I
47. j 48. ;~, d):- ~,. ·i
50. , Cb, c, d) ;ca, d) 1' : 54. 1Ca, b} : . .. ,
49 . . Cc, d) 51. '.Cb, d) , 52. 53. Ca, b)
I
55. (a, b) 56. (a, b; d) 57. '.Cb) 58. ,Cc, dJ 59. i~,·cj . · I 60. iCb; c, dJ. )
61. .(c) ' 62. (a, c) , 63. ·Cb, d) :c
64. l _a, b)' 65. ,cc, dJ
'

; 66. !C~;c,.~l_
; ; ' ' " -.- 1' ;j

,I
67. (a, b, d) 68. , (a, b, c, d) 69. i(a, c) lee, d) 71. 'Ca, b, d)' . I 72 I(~, b; c);J ,J
70.
. 'I . . I
73. :(a, c) J

www.puucho.com
Anurag Mishra Mechanics 1 with www.puucho.com

ifORCfANAL!SIS _ _, 23~j
'"; ,---~~,-, '";h-1$&',~,--,,, ------·-~-~.t)i4•o;.¼i0.\i :.;;; _____ ~

Level-3: Comprehension Based Problems ,:.~


--- -·-· ' . -·~- - -~---
Passage-1:
1. (a) 2. (b) 3. (c) 4. (d)
Passage-2:
1. (d) 2. (a) 3. (b) 4, (a) 5. (a) 6. (b) 7, (d) 8. (d)

Passage-3:
1. (d) 2. (c) 3. (c) 4. (a) 5. (c) 6. (c) 7, (c) 8. (a)

Passage-4:
1. (b) 2. (b) 3. Ca) 4, (b)

Passage-5:
1. (d) 2. (b) 3. (a)

Passage-6:
1. (b) 2. (b) 3. (a) 4, (a)
l
Passage-7:
1. (b) 2. (b) 3. (d)

Passage-8:
1. (d) 2. (a) 3. (a)

Passage-9:
1. (c) 2. (b) 3. (c)

Passage-10:
1. (c) 2. (b) 3. (b)

Passage-11:
1. (b) 2. (a) 3. (d) 4. (c) 5. (c) 6. (b)

==;.,~~~l~110!!P,!! ;,;~-.;!~~~~.
1. A-S; B-P, S; C-Q; D-R 2. A-P; B-S; C-R; D-Q 3. A-S; B-Q; C-P; D-R
4. A-P; B-R; C-Q 5. A-P, S; B-P, S; C-R,T; D-Q, R,T
6. A-P, S; B-P, R; C-Q, R 7. A-Q, S; B-Q, R, S; (C) P, Q, R, S; D-S
8. A-Q, R, S; B-S; C-P, S; D-P, S 9, A-Q; .B-P; C-R; D-S
10. A-Q, R, S; B-P, Q, R; C-P, Q, R, S; D-P, Q 11. A-P; B-P, S, T; C-P, Q, S; T; D-Q, R, T
12. A-P, Q; B-P, Q, S; C-P, Q, R; D-P, Q,R

www.puucho.com
Anurag Mishra Mechanics 1 with www.puucho.com
. -.. ......

1· 24_0
'_.,.·..-.: _____
~·· ·_ ·-·, ~--=-' . .
',., """~,.. -, ........

-< _· .:::,:·:t:'.".':rt:t~..:;'~;:;~~~
• v,,,,,, ~,.,~ - · ~ ~ - - - " ' " -

L 11·0 IO Al . , . ·
e:ve O : . n y · ne temiilt1ve rs orrect
c··· "---._·~"'". · ·...,.?·-.,,, :·:.:,::;i
· ..,-. ·ii=
N=T'+T ·
ir·+·1< . :·.·:.~!f·:·\::·}<.: ._:, •·c·
1 •

1. [d] ;;'; '·,.,:·N',;,if; _.-:.,'·:.s·:_:,, •. ~-' ... (i)


Cycle and cyclist moves with uniform velocity this N = 600 - T , ... (ii)
means that net force on this system is 0. => T = 600 = 150 N
:. Inclined plane applied force' mg' vertically upwards 4
so that net force become 0. 5. [c]
2. [a]
Earth is applying a force of magnitude Sg downwards
, while falling.
_' :. From Newton's mrd Jaw block will apply a force
Sg N upwards,
3 .. [c]
Let AB = diameter = D
~-
~ : 2 tN

AC =Dcosa (LACE= 90°) ·. Both blocks will moves together


Time to reach C = t 21 = 3x a

~
1
-xg cosa xt
·2
= Dcosa a= 7m/s 2
2
=> T=lx7=7N
t=f! ... (i)
C B
Net external force on block A = 7 N
6. [a]. 1

[. ~: gs:J
Time to reach B = t' Tension at all points will be F
=> .!xgxt' 2 =D _j
i.(' a,J,g "." . => rope is not moving,
2 gcosa ·~ acceleration will be 0

=> t·=f! ... (ii)


F-T=0
F=T
From eqn. (i) and (ii) 8.· [c]
T~20w

[~L
t = t' Reading in spring balance
·4_ [b] =T/g
For equilibrium T -4 tension in thread
~---~---~
T' =2T; >,.
connected to spring

t~ Jl:4 tTI Here system is


equilibrium and T = 20 x g
in

,£, •t,.
.
read mg=-= 20g 20
g

www.puucho.com
V
Anurag Mishra Mechanics 1 with www.puucho.com

\;FO! RCE ANALYSIS ,~'J.,;


"'"'-'-··~-'----~=··'-----'-~'-"'..,,,~_....,,---
9. [c] 15. [a]
Tension T = !Oxg T=mg ... (i)
Also T'=T=_lOxg Mg= zr case [.
... (ii) '.·T~e. . . .
\a e.
i" . . . .
, , T...
T .~-
reading in both the spring From eqn. (i} and (ii)
T' T Mg= 2mgcose 1 · . Mg ·: . •, mg
= - = - = 10kg I~--~-· . ,•-- ,, , ~---,
g g M = 2mcose
10 .. [a] M<2m (as case< 1)
Magnitude of F1 and F2 may be equal or may not be 16. [b]
but their direction cannot be same because F1 is kx=ma
accelerating and F2 is decelerating.. k
a=-x
11. [a] m
Deceleration of body A It is a straight line. Here
~ dA=(MAg+f) X is the compression in
MA block.
In our question X = X O - x
Similarly dB =·MBg + f
MB Since X is decreasing with ·x i.e., spring is coming to
natural form and X0 is initial compression.
Now, v = 0=u 2 +2ah
2
17. [a]
u = same for both bodies
u2 u2
Let F= kv
~ hA=-= mg-kv=ma
2dA 2(g +_L__) kv
a=g--
. MA m
u2 u2 with time velocity will increase since initial
hB=-= velocity was 0.
2dB z(g+ ~B) 'a' is decreasing also after certain time
a=O
MA >MB mg
hA > hB v=-
k
12. [b]
When cable is ·cut down then chamber will fall freely when velocity = mg then a= 0 and. ball moves with
k
under gravity, wedge and block both will also fall constant velocity.
freely under gravity.
18. [a]
:. acceleration of both will be g ,J.
:. block will remain at top of wedge
13. [a]
While going upward a = F - mg·
m
~
2 T'=m 1g
h =.!..at
2
t1 = /2h
v~ T' .
... (i)
m2g--=m 2 a
2
While moving downward
T' .
, F+mg -m 3 g = m 3a ... (ii)
a=---
m
2
Form eqn. (i) and (ii)
t2 = ~ as a'> a (m 2 m 3 )g
f-;,- a=
-

m,2 +m3
~ t1 > t2 '
Putting value of ci
14. [c]
In both cases initial relative velocity of elevator = 0
and g,J.
( m m·)
T'=2m 2 l+ 3 - 2 g
. m3 +mz
· :. time will be same
www.puucho.com
(•
Anurag Mishra Mechanics 1 with www.puucho.com

2m 2 x2m 3 23. [b]


m1g =. g
m2 +m3 Mg-T=Ma
4 . 1 1 T-mg-=ma
-=-+- l' :r
~1 m2 m3 (M-m)g=(M+m)a
19. [b] (M-m)g
a=
mg-T=ma (!"f + m)
r'
for minimum value of acceleration 'T' should m(M+m)g
T =mg+----~
M+m
be max.
3
..' T= 2mMg
given Tmax =4mg . .
~ .. M+m
.n,g
a. =f4 T-2mMg =2mg
mm M
20. [a] Total downward force on pulley =2 T =4 mg.
24. [d]
Block B will come to rest when V!!locity of block A =
velocity of block B
cc} J~12t dt =J~3dt .
. '..(i)
6t 2 = 3t .
Now let m kg sand is put cc} t =0, 0.5
cc} (M+m)g-B=(M+m)xf ... (ii) t =0.5sec
. 6 25. [b]
From ~qn. (i) and (ii), m =~ M

21. [b]
System is in equilibrium
5

...Fnet = 0
(,~:~;IlJ·
t .:___ _ ~]
:::::} · T--600= 60a ... (i)
~ ~ ~
a+ a'= Sg ... (ii)
cc} T+W+N=0 4
...
also, W and N are at right
... . T-100g=100a' ... (iii)
angles Solve eqn. (i), (ii) and (iii)
T2=W2+N2 And get T = l 9 soo ~ 1218 N
16 .
also N cose =W sine
N=Wtane 26. [a]
22. [b] Tcose = mg ... (i)
... .
For. moving with constant velocity F = 0
...F+mg=0
_,
net rT--
!8 -. -r:,~
, ·- :•
. -· . -. - -.·.l
- •• .. r '' j
[____~ ·.
. A
cc}

...
A is in x-direction
,
.. . . /.
B

. ...
:. For net.force to be 0, F should be in +ve y-direction m~2

Now
......
v xA_= mg
T'-mg cose = --·= 0
r
T'= mgcose ... (ii)
vAsine = mg . . • 2
2
v=·~ ; = cos 0 = ( ~ )
A sine
For min. v, sine should be.maximum cc}
T' 3
v= mg T 4
A
www.puucho.com
Anurag Mishra Mechanics 1 with www.puucho.com

1
IS-'-'-~"------~-"""'-'-----~~''""·::1""·t1"':;c..?...
i...:..FO...:R...:C_E_AN...:A._LYS;.,.___ · _ _.....:..,_•..•i~-'~}~'-'---
I.• ....

27. [a] y--_.,l--_. dv


I - o .. a=-=0
As shown in figure dt
'j 0;58
.- =} F=0

28. [b]
vsine = Vo case
V = Vo Cote I~_ ij. , ,.
sEf
33. [c]
-+ -+
Finclin~d + Fgravity =0
-+ -+ --+
~ F inclined = - Fgravity = - Mg
34. [b]
Since the block is held held against a wall, the
coefficient of friction will be equal to the weight of the
block. Hence
µ=mg = (0.1 kg) (9.8 !IlS-2)
= 0.98N
I
'-'·--- 35. [c]
2Tcose = F N = Mg & Fp,non
For any mass flmax =µMg
Tsine=ma Clearly the magnitude of net force acting on the block .
T sine
.a=--=
F
m 2cosem
F sine

Fx
from the horizontal surface is
F = ~f2 +N2 = ~f2 +M2g2---
a= -tan8 = ----,=== But- o,;;J,;;µMg
2m 2m.J a 2 - x2 =} o,;;J2 ,;;µ2M2g2
29. [d] =} M2g2,;; f2 +M2g'2,;; M2g2 +µ 2M2g2
=} Mg,;; ~f2 +M2g2 ,;; Mg~l+µ 2
36. [b]
The force constant is inversely proportional to length.
If the length 1 of the spring is cut into x and 1- x such
that
x=2(l-x)
I then x = 21/_3
From the inverse relation, we can write:
30. [b] k1 l l 3
-=-=-=-
dv
.mv dx = (ma - Toe) k X 21/3 2
37. [b]

J;mvdv= J;cma-kx)dx ~a,


I
0 Since the blocks cannot accelerate in horizontal
direction therefore the nom1al interaction force
ber,,reen the blocks as well as between 5 kg block and
kx2 the wall is F = 1000 N. Again both the blocks
0=max--
2
2ma accelerate downward with acceleration s2 gm/ and
X=-- therefore the relative acceleration between the blocks
k .
is zero. Hence the friction force between the blocks is
31. [b]
zero.
f, = µkN 38. [d]
=µdF+Mg)cos8 If a block is released on ·an inclined plane of inclination
32. [b] 8 and having friction coefficientµ with the block then
Slope of displacement-time graph gives v~locity which the acceleration' a' of the block is (assuming tan 0 > µ)
is constant here
a = ..!. (mg sin 8 - µmg cos8) = g (sin8 - µ cos8)
v = constant m

www.puucho.com
. --
Anurag Mishra Mechanics 1 with www.puucho.com

.- d

Hence greater the value of µ lesser is the value of F1 = mg sin9 + µmg cos9 ... (i)
acceleration irrespective of mass of the block. F2 +µmg cos9 = mg sin9 . . :(µ)
39. [d] => mg sin9 +µmg cos9 = 2(mg sin9-µmg cos9)
A block begins to slide on an inclined plane ifµ = tan 9 => 3µ cos9 = sin 9
irrespective of mass of the block, where µ = coefficient => 9 = .tan-1 3µ
of friction and' 9 = angle of inclined plane with
horizo1_1tal. 46. [a]
40. [c] With respect to platform the initial velocity of the body
of mass mis 4 m/ s2 towards left and it starts retarding
fl~ax =µN = (¾)(lO~) N~Fsin30 at the rate of a= 2m/s 2
30 Using v 2 =u 2 +2as
25,.J3 Fcos 30
=--Newton 3g we get: 0 2 = 4 2 + 2(-2)(s)
2
=> s = 4meter.
Since 1_let force (excluding friction) acting on the block
· is 20 N upwards therefore f = 20 N downwards. 47. [c]
41. [af · · If F1 & F2 are not zero then friction force on m1 acts
west wards & on m2 acts east wards.
For tile insect to be at equilibrium
For m; to be in equilibrium F1 - ·f = 0
Ffr = mg sina
For m2 to be in equilibrium
or µN = mg sin a
or I+ (mg coscx) = mg sin a. F2-f=O => F1=F2=f
Hence, · · cota = 1/µ = 3. But f S 10 N. Hence F1 = F2 & F2 S 10 N
48. [d]
42. [b]
Consider A and Bas a system. There is no vertical force
in upward direction to support their weight.
Therefore, the system cannot' remain in· equilibrium.
49. [c]
Limiting force of friction between A and B is
F1 =µ1mAg=90N
Limiting force of friction between B and C is
.flmax = µN = (0.5)
(45) = 22.5 newton. F2 =µ 2(mA +mB)g = BON
Since magnitude of net external force except friction is Limiting force of friction between C and grou~ci is
25 N, therefore, . f = 22.5 N F3= µ 3 (mA + mc)g = 60 N
and . lal=.25-22.5 = 1.25 m/s2. As F is gradually increased the force of friction
2 between A and B will increase. When F = 60 N block A
43. [d] will exert a horizontal force of 60 N on C. Hence C will
Tension in the ·string, T = Mg . be on the point of motion. Hence the least value of Fis
Ther~ 'are
two forces acting on the pulley. The force T 60 N.
acting horizontally and the force (M + m) g acting 50. [b]
vertically· downward. The resultant of these force is The acceleration of blocks down the
( ~CM +m) 2 +m 2 )(gl. incline will be g sin 9. Horizontal
component of this acceleration is .·,,,,:
Nl
44. [c] aH = aco.s9 and vertical component
If T is the tension in the string, then a, ';'asin9 f.',"'' \ •...
',

T = mg (for outer masses) aH = acos9 = asin9cos9


\mg
2f cos9 = :/2 mg (for inner masses) and av =asin 2 9
2(mg)cos0=:/2~ · For body A:
or cos9 = 'lj:/2. Mg-N=ma, .
=> 9=45° or, 2
N = mg - mg sin 9 = mg cos 2 9
45. [d] and, µN 2' maH
Let m be the mas of the body. µmg cos 2 9 2' mg sin9cos9
www.puucho.com
Anurag Mishra Mechanics 1 with www.puucho.com

µ;:: tan0
or, 0=tan-1 (µ) _
51. [a]
Horizontal acceleration of the system is
F F
a=-----=-
2m+m+2m Sm
L_et N be the normal reaction of the system is Tcos45°= ma
2F or, T =.fi. ma
N=2ma=-
5 mg-Tcos45°= ma
Now B will slide downwards of mg-ma=ma
µN;:: mBg a= g/2
so, T.=mg_
µ(~);::mg . .fj_
p;:: 5mg 55. [d]
2µ Extension in the spring= AB -R = 2R cos 30°-R
52. [b] =(.J3-l)R
Friction force between A and B(=µmg) will accelerate
B and retard A till slipping is stopped between the two
and since mass of both are equal acceleration of B =
retardation of A =µg
V1=Vo-µgt
and v 2 =µgt
Hence the correct graph is B. When the slipping is
ceased the· common velocity of both blocks becomes So, spring force = kx
• v 0 /2. c-./3 + l)mg c-./3 ..:1)R = 2mg
53. [a] R
Free body diagram (RB.D.) of the block (shown by a Free body diagram of bead is :
dot) is. shown in figure. N =(F + mg)cos30°
For vertical equilibrium of the block, = (2mg + mg) .J3 = 3-./3 mg
. F
N = mg +Fsin60°= ,J3g +-./3- ... (i) 2 2
2 · 56. [a]
For no motion, force ·of friction Tangential force =F sin 30° - mg sin 30°
f+-
.

- N. -:-:~---
·
.
·1
'· •
\.fert1ca1
I =(2mg + mg)si~30'= m{
I;f" ··-· .: . : I> ••.,. ~.,.

1·, . . F cos60'_ • ; : ,' : Horizontal i Tangential acceleration = g /2


57. [b]
r: : .!1'19 + Fsin 60° ,.. : •. " I When the inclination of the slant side ·reaches the
angle of friction, sand will betin to slid do~. So, for
J;::Fcos60°
or µN;:: Fcos60° maximum heightµ= tan0 = - · .
R .
F
or g;::- or,
2
58. [d]
or F ~ 2g or 20 N
T=Mg .
Therefore, maximum value of F is 20 N N =60g-Tsin60°
54. [d] Also, T cos 60° =µN
Just after the release B moves downwards and A Solving these three equations
moves horizontally leftwards with same acceleration
say a.As shown in the free body diagram of both A and M = 32.15 kg
B.
www.puucho.com
Anurag Mishra Mechanics 1 with www.puucho.com

· MECH~~
59. [c] T should be maximum
For m2 mass 1000-SOxlO = SOa

Also,
m2 g-T1 = m2 a
T1 =T2 + f.
T2 - m1g = m1 a
. From above three equations
(m 2 -m 1 )g·-J
... (i)
. .. (ii)
... (iii)
Now

=>
a= 10m/sec2
1 . . 2
10=-xlOxt
2
t = .Jz sec.
f~
a=- ~~-- 64. [b]
m1 +m2 fmax = 0.2xN
60. [a] . 1000- 0.2x SOg = SOa
. 400
a=-=18
so
Now ' d=-xl8x(v2)
1 r;:; 2
=18
2
:. distance between man and ~lock= 20-18 = 2 m
65. [a]
fmax = lOx 0.2 = 2N
For equilibrium of
,--B~-
l
N'.
3mg sin37°= f + 2mg cos37° Initial force = SN > 2N
=> f=2m block will move with ~2t)
For man, mg sin37°+f = ma acceleration ' 10
6m+2m=ma
a=8m/s 2 a=S-21:-fmax S-2t-2
1
61. [c] S =~[2n:...l] dv
n 2 -=3-2t
dt
a
Sn+1 =-[2(n+l)-ll v=3t-t.2 (•: at t = 0, V = 0)
2
~ [2n - l] v=O
so,
Sn+l [2n + 1] ' => t_= 0,3sec
:. at t = 2 sec block is moving
:. Jmax will_ act i.e., frictional force acting = 2 N
66. [a]
Small block m will fall vertically as no external force is
acting on it.
67. [a]
=> N'=mgcose
N'= 2mg + mg cos 2 8
= 2mg+ mg= Smg
2 2
fmax =µN:= S~ xµ

= mg case X sine= mg N=.J3mg


2
1 fmax =.J3x ~mg= ¾mg
µ =-= 0.20
s
63. [c] :. block will not slide
fmax = SOg X 0.2 = 10g ·1·~·,,,N',,1
..
.· ·.· •.·, .i""'. T'-'
Since
3
f= mg _ mg =mg <f,
2 2 max
T-SOg = SOa f
I.'.__. 50g. 68. [d]
For minimum time acceleration of
man should be maximum As shown in the figure the forces F and Mg passes
through the center of mass and so they have zero
www.puucho.com
Anurag Mishra Mechanics 1 with www.puucho.com

[ FORCE ANALYSIS
torque. But friction will produce clockwise torque. So
for rotational equilibrium the normal should produce 74. [a]
an anticlockwise torque. mv 2 ·
69. [b] N =--+mg sine
3mg, . R
By conservation of energy

70. [b]
Zf sine= F
I
[2m 1:9
B
mgR sine= ~mv 2

mv 2
.
--=2mgsm0
2

Tcose = mA
1-----f F ,:
R
N =3mg sine
Tease Tcos8 So, Ratio =3:2
75. [c]
T T. mv 2 .
From Q. No. 74, - - = 2mg sme
2Tjsin9 R
8 ' .---------------7
i .-. I
I
·1

F ···-······t-,::ra--··
2tane=-·
mA

A-F( .Ja2 - x2
- 2m
X )
I i '~ 'ff'I
71. [d] mgRsine = ~mv 2 = K
2
mv 2 . e =K-
N»=mg--- So, mgsm
R» R
mv 2
mv 2 N =3mgsme=-
. 3K ·
Ne=mg+-- ND =mg--- R
Re Rv
mv 2 . mv 2
76. [b] Nsme=--
NE=mg+-- r . Ncose~
RE
Ncose =mg
(where N x stands for normal reaction .at point x of v·2
path and Rx for radius of curvature at point x.) tan0 = -·-
NE >Ne =}·NE is maximum. rg ' ~8- Ns,]a
·: RE <Re =}

3 v2 mg
72. [b] -=
·: Centrifugal force = (mass of body) x 4
l0x 10
2 3 300
(Angular velocity of frame from V = 100 X - = - = 75· v =
4 4 '
s./3 m/ sec
which body is observed) .,
x (distance of particle from axis of 77. [d] i;> ,,.,,,,.

rotation) = mro ~a When car just topples, -·


.. ~~ Ne NA
73. [c] contact at B will be no •
"----2m____.
more i.e., N 8 =0 ,
mv2
Moment about A is just
~

r
1m
zero B • A ,, .
2 mv 2 -
mg
.
=} mgX-=--Xl
T1 =. m1ro 2 r (for m1) ... (i) 2 r
2
=} g x r =v 2
and T2 -T1 =m2ro (~) (form;,) ... (ii)
=} 10 x 10 =v 2 =} v =lOm/sec
T2 2m, +m2
from eqn. (i) and (ii) we get : - = -'----"-
T, 2m,
www.puucho.com
Anurag Mishra Mechanics 1 with www.puucho.com

[248~~-· ,C,/ ·' ~. ,· ·,: •• > ,, •' ,·,:~·;s,r.,E~~~~·~s-q

Friction force = µN = µ m~ . ~l-


m . .
____.;.:_~.~----~ IT)Q

=} Retardation=~ = ( 7} 2 2
T sine= 41t P 2 mr ... (ii)
But sin8 =· r/1 and cos8 = h/l
µ and r constant. =} retardation oo, 2
. ·: Particle is to be in contact with the table only
7?. [a] when N ;:,, 0 ... (iii)
. mv 2 From eqn. (i), (ii) and (iii) and using values of sin8
Nsm8=--
r and cos8
and Ncos8=mg
Hence
v'
tan8=-
rg 83. [e]
:. For correct value of u car does not slip even if there
is no friction. But for any speed, other than v above
'-·~· ',
condition is not satisfied and the car slips. This is also
true for a stationary car.
80. [b]
·: When total acceleration vector makes 45° with
I
. · radial acceleration, then !· "
,. . ' v =30m/s' •
a, = a, =_2t ... (i) " ·--·-•-....:...
L .. '' .....--t,_..:,.;...._
Jy/' ' ' •

..~

dv ·
a, =-=2t =} v=t 2 mv 2
dt mgcos8=-- : .. (i)
r
v2 t4
and a =-=-
' R R
... (ii)
r=--
v' v' v3
gcos8
. from eqn. (i) and (ii),
. t4
2t = - =} . t 3 = 2R = 8 =} r = lOMO metre.'
R.
84. [e]
=} - t = 2sec. ~~-2 ··. l
81. [a] mgcos8-N = - -
R
Let, when particle is at angular position 8, then
distance travelled = 1. Using energy conservation
between A & B.
V = a./f_
dv a dl av a2 mgR(l - cos8) = -1 mv.2
But a--------- 2
' dt 2./f. dt a.ff. 2·
N = 3mgcos8-2mg ... (i)
2
and· , a, = vr = ( a;zJ if N > 0, then ball will be in contact to lower surface
and if N < Oit will be in contact with upper surface.
Angle between a 0 et and v is same as N >0 =} cos 8 > ~ [from eqn. (i)],
angle between a.et & a,· : .·_·,- 8~--_
e ,' . ,, .
·a·_·(2"'
,.,
r•
3
- ' · net,
a
tan ex== ___f_ . (X' •
, .• ,;:v.. a,

a,
=} 1 85. [e]
a= tan- (~)
Given -
dv v2
= ... (i)
82. [d] dt R
V dv t l
N+Tcos8= mg ... (i) =}
f2=
uoV
J-dt
oR
www.puucho.com
Anurag Mishra Mechanics 1 with www.puucho.com

[ FORq ANALYSIS

=? R (__!__· -
Uo
.!) =
V
t ... (ii) =? tanet= gx
v2
(·:v =21t11X)
Again from eqn. (i) =? x =(gcotet)
dv ds v 2 dv v 2 4it2112
-·-=- =:> v - = -
ds dt R ds R 90. [a]
2
V dV '"' ds v2
=? J - J ====> v = Uoe21t ... (iii) -=20 =? R=20cm
""---;;- o·R R
2
from (ii) and (iii), t =~ (1 - e-2• )
Rro =20
co= l rad/s =?
Uo At new position R = 10 cm
86. [a] So, v =Rro =lOcm/s
26 = <I> I ~ I And acceleration = R 2ro = 10 cm/ s2
=?

=?
2(!~) (!:)
2.roA=COc
=
r@II
II . A•'
91. [d]
At any angular position 8
mv 2
=? 1:2
COA
COc
=
T-mgcose = - -
r
mv 2
=? T =mgcos8+--
. 87. [d] r
N cos8 = mg and N sine= mro 2 r ... (i) when mass is released from displaced position, 0 starts
2
ro r
decreasing and v starts increasing. As a result mg cos0
=? tan8=- 2
g as well as mv both increases. Hence graph will be
r
=? --=- (d), (c) is not acceptable because at t = O, T ;t 0.
R-h g h
(b) is not acceptable because the variation is not
linear.
=? h=R-_L
. (02
92. [a]
88. [c] Considering an element oflength
mv 2 dx at distance x from axis of
Car slips if - - > µmg friction.
r
=? V > .JµriTrue dm=(7)dx
Hence (a) and (b) are both true.
Again at x = L, T = 0 (T for tension) (T + dT)
T X m 2
If there is tangential acceleration then for slipping : = JdT= J-xro dx
µmg=m
K 2 )1/4
2
=? T
0

= -nt2 ( x22
LL

I
=.Jµri l - _a_
=? V
( µ 2g2 =?
2
T = -mro (x2 -L2) = mro (L2 -x2)
2

·: (d} is also true. 2L 2L


=?
89. [a]
93. [a]
N sina Let 'F' be force of friction in each case for stopping car
. by applying brakes
N cos
mg i .!
2
mv 2 ~ F. r (i.e., work done by friction should be
.
greater than kinetic energy)
2 2
mv p;,, mv
N cosu = - - and N sinu = mg =? ... (i)
X 2r

www.puucho.com
Anurag Mishra Mechanics 1 with www.puucho.com

MECHANI~
2
For turning the car
2
N = mg - mv will be different
F :c, mv R
... (ii)
r 99. [d]
The required force is less in case of applying brakes. At position B acceleration is
94. [d] . only vertical.
Direction of speed is changing so velocity is changing For particle 1.
=> acceleration and force are also changing. Let velocity at B·= v
95. [a]. from energy conservation
Length of thread = I 1 2
T -M1v =MgL1 ... (i)
Mass= m 0
2
N ~ 0 in limiting case M v2
Also atB T1 -M 1g = -1- ... (ii)
Tcose = mg L,
Tsin0 = mrOJ 2 From eqn. (i) and (ii), T1 = M 1g + 2M1g
=> tan0=--
rOJ 2 rng • T1 = 3M1g
g Similarly for particle 2 :
For block to leave contact e :c, 60° T2 = 3Mg
2 T M m 1
=> .J3:,; !xsin60°xOJ => OJ :c, 10 -1 = -1= - = -
10 Tz M 2 2m 2
OJ= ext => t = 20sec 100. [c]
96. [a] As car is moving in anticlockwise
, direction and have
When particle is at point A tangerrtial acceleration .(swell as radial acceleration
acceleration g ,J, :. Friction component should be along tangential and
Point , B acceleration is radial direction
towards ot 101. [ b]
:. acceleration varies as 2 2 1tR
VB "' V0 + 2a X
i.e., clockwise 2
97. [a] vf = a!tR
Conserving energy at points
AandP ic2
2
Normal acceleration at B = .....!!.. = a 1t
mgxRcos0=~mv R
2 2
v2 Net acceleration at B = ~a + a 2 it 2 = a~l + 1t 2
- = 2gcos0 102. [a]
R
2 At the highest point, we will have
· mv
=> T-mgcos0 =- - Mg +N = mv 2 /r
R
= 2mgcos0 Hence, minimum the curvature r, the maximum is the
T = 3mg cos0 normal reaction.
when particle is only horizontally accelerated at this
103. [c]
a
moment
=> T cose = mg => 3 mg cos0 x cos0 = mg
Net acceleration of the bob in
position B has two components. ~-.,',
~ e =· cos-1( .J3
->
=> cos0 = => 1 ) (i) an = radial acceleratioh''(towards
BA)
.' '
~--~,.~(
'~
98. [d] (ii) a, = tangential acceleration (perpendicular to BA)
Since earth is also rotating
Therefore, both will have different velocity w.r.t.
Therefore, direction of ais correctly shown in option
(c). .
centre of earth as they are moving in different
directions

www.puucho.com
Anurag Mishra Mechanics 1 with www.puucho.com

FORCE ANALYSIS ·251

2. [a, c]
Particle is not accelerated as seen from both the N = 70gN
frames. 4
:cc; frames are not accelerated w.r.t. each other T = S0xlO
:cc; either both are inertial or both are non-inertial but 4
moving with same acceleration. T = 125 N
3. [c] =;If N=30g
For t < 0 system is in equilibrium =; . 30g+T-30g=30a
I -- . ·- -· · 1;- - - ··----
3T- 50g = 20a
, F ~ F, j' F2~FI
4T = 10a
50g
:cc; F = F1 = F2 for t <0 a=--
For t > 0 system accelerates 70
:cc; F-F2 =ma>0 T = 150g
F2 <F 7
F1 -F > ~ If boy applies no force on rope T =0
:cc; free fall will be there
F1 >F
4. [a, b] 8. [a, c]

=;
if a is +ve
i.e.,
N-mg=ma
N=mg+ma
N>mg

elevator speeds up while


going up or speeds down while going down.
"i
ir' a ·•.·.

!L_ ·___•· .mg


.
r
-x2+y2=h2
-2xvx + 2yvy =0
Tease . ~
~mgl
I
··--··- - .. . - __ J

5. [b, d] =; V
V
=-y-
T = w.r.t. elevator x cosB
=; a=O Vy= Vx COS0
=; It can move only when with uniform ay = ax cosB
speed ablock = aring x cosB ... (i)
6. [d] 2mg - T = 2m x ablock ... (ii)
S2 is accelerated w.r.t. S1 T cos0 = m X aring ... (iii)
=; relative acceleration of the twci frames is not zero From eqn. (i), (ii) and (iii)
:cc; minimum one of the frame is non-inertial 2g cosB and T = Zmg
aring =
at least one of F1 and F2 * 0 1+2cos2 8 1·+ 2cos 2 8
F1 = F2 = 0 9. [a, b, c]
is not possible. When block does not slip
7. [a, c] mg =N coscx
For equilibrium N = mg seccx
N+T=30g Since block m does not slip on block 2m
3T-N=20g :. both can be taken as on~ system
4T = 50g. N'=3mg
N = 30g- SOg
Normal reaction on 2m by ground = 3mg
4 Also from figure 1

www.puucho.com
Anurag Mishra Mechanics 1 with www.puucho.com

M~CHANl~~-1 .]
14, [a, b]
. , ~-N:cosa
For equilibrium on man, net force
Nsina , on him should be zero.
' .· .',, .mg· Also as shown in figure
m=f, =µN
N sin a= ma, N = mgseca
~ a=gtana
15. [a]
And from figure 2
,,
N'
'

Since small block m is not moving
N .
F=3ma '

'.F /'/'., w.r.t. wedge


~ F=3mxgtana . :. Both can be considered as a single

10. [a, b]
F= 3mgtana 3mg, system which is accelerated (M·~·;;g,
horizontally
H2+x2=y2 N=(M+m)g
Differentiating 16. [a].
2xxvx = 2yxvy ... (i) N= mg
cos0
Vear= y xvblock
X 17. [c, d]
~x2 +H2
1/x = Vear = ----Vbtock
X
X
~ V x1 --a====
2 2
--block
V '
vx +H
Differentiating eqn. (i) again,
2 2
xax+vx=Yay+Vy
given In equilibrium acceleration of each block is zero.
a;=acar=O
v2X -v2y ~ kx 2 =(m1 +m 2 -m 3 )g
~ --~=ablock Just after .the string is burnt only
y
T = 0 and no other force is changed
~ acceleration of m1 = m2 = m 3 is zero
v2H2 kx 2 -m4 g ·
~ -(H_2_+_x_2_)~31~2 = ablock and acceleration of m4 =
,m4
12. [b] = [ (m1 + m 2 ) - (m 3 _+ m4 )Jg
Let acceleration of pulley·is a m4
OON 18. [c]

R
T T· '

~
At first B will move downward and C towards tight
with a constant acceleration and·v, =at·
~":_i_
- :T }9s·
--- , ,· .- .t_.1_t
l-,,a ,- . .
. . . . . 50 100 • ,: The· moment when B touches ground A will lift up.
. • T T
Now as C is moving toward tight A will rise and string
T-50 = S(a+ a') ... (i) between BC will become loose. Therefore block C
T-100 ='l0(d-a) decelerates with a constant deceleration due to the
... (ii)
tension generated in string between A and C. At a
zr = soo ... (iii)
certain moment v c = 0 (after this A moves
From eqn. (i), (ii) and (iii) downward). C again accelerates in the opposite
d=ss direction upto the moment A reaches the ground.
2 19. [b, c]
13. [a, d] Just after BP is cut .
Clearly if'B is stationary and pulley moves then block · For block A-no force has changed
will rise. :. acceleration of m1 = 0
VB =u+vA for m 2 downward force is being reduced
aB = 0+aA :. m 2 will move upwards
www.puucho.com
Anurag Mishra Mechanics 1 with www.puucho.com

I FORCE ANALYSIS . '253]


20. [a, b, c] 23. [a]
. at In 1st case: 2mg-mg = ma1
Acce1eranon = -

through origin
m
i.e., a straight line passing 1. c=J:--+•tl In znd case :
a, =g
2mg-T = 2ma 2
dv at T-mg=ma 2
-=-
dt m a2 =!
at 2 3
v=- Parabola In 3 rd case : mg +.mg -T = mag
2m
at t
V=-X-
m 2 T-mg = mag
. t
v = acce 1eranon x-
2
21. [b, c]

+--'t;,,-,,~-+ T2 sln 8

mg
AO--o
T2 •
T2 cos a
-,=~I
mg
24. [b, c]
tane=-
d
d .
cose=zF'
2h
T

d2 +h2

R
T2 sine= mg ... (i)
4
T2 case= mg ... (ii) - - __'!2_ga__ _...1
T1 sincx = T2 sine ... (iii) sine= . ~ s i n e+Tsln8
T1 coscx = T2 case+ mg ... (iv) T T
. 4
From eqn. (i) and· (ii)
Teas 8 - TC0s,8
tane = 1 .as man moves slowly
, . mg, . .
e = 45° 2Tsine = mg
=} T2 = ..f2.mg ... (v) T = ...!1!!L_
2sine
From eqn. (iii), (iv) and (v) ·
as man moves upward e becomes small
tancx = mg sine decreases =} T increase
2mg
1 mgRd2
T=-- h +-
tan ex= - 2xh 4
2
2tancx = 1 = tane = mg.J d 2 + 4h 2 ·
Ti= ~ = mg-./s ... (vi) 4h
smcx 25. [a, b, c]
=} T, ..f2. = T2 X -.J5
22. [b, c] 2t-2T=0xa
{1-)2t . ~-
.T =_t
In 1st case:

In 2nd case :
ma=2mg-mg
a=g
[] =}

For
T=mg=lO
So
m1 to.

t=lOsec
lift
off.
T =t

10
TMT
T-ing=ma' Similarly for 2 kg block aN = 20 sec
2mg-T = 2ma' 26. [c, d]
The acceleration of mass' m' and' M' along the inclined
a'=!
' 3 plane is g sine so the contact force between them is
zero.
a-·a'= 2g So mass 'm' will fall freely with acceleration g and
3
acceleration of wedge will be g sin_9.
www.puucho.com
Anurag Mishra Mechanics 1 with www.puucho.com

I 254· ·'. '.· MECHANICS-I J

27. [a; b] Hence f = mg sin0 - mg


As discussed in question No. 26 contact force between 2
'm' and 'M' will be zero. So contact force between 1 1
30° s; 0 + 45°; sin- ( --)
wedge and inclined wedge will be Mg cos0. 2../2
28. [b, d] =µmgcos0, .
· (i).Let the force F be applied on m1 and both the blocks ·
.:: . -l (2../2
+sm ~) < __
0 < 90°
accelerate without any relative acceleration. 4
fm,';. =0.lxSxl0= SN
Hence (b) is correct curve between 0 and friction force.
F-J=Sa
30_. [a, c]
adding: F = 15a
a=F/15 N =Fcos0+Mg ... (i)
fmax =µN = µ(Fcos8+Mg) ... (ii)
Hence J=10(;s) 3f
F-=- To just push the block

~ -j-
2
3 15 Fsin0 = fmax
=> Fmax = - fmax =~newton
~
2 2 => Fsin0=µ(Fcos0+Mg)
Hence (b) is correct and (c) is wrong. => F= µMg _ _ M9,c._
sin0-µ cos0
(ii) Let the force F be applied on m2 and both the
blocks accelerate without any relative acceleration. => sin0-µ cos0 >. 0

adding:
f =Sa
F-f=l0a
F = 15 a
If~~ ] =>
=>
=>
tan0>µ
tan0 > tan(tan-1 µ)
0>tan-1 µ
F
~ a=- Hence the block can be pushed forward only if
15 0 > tan- 1-µ.·
=> J=s(:S) => F=3f Again as 0 decreases sin0 .decreases while. cos 0
increases, therefore, sin 0 - µ cos0 decreases.
=> Fmax = 15 newton
Hence .Ca) is wrong and (d) is correct. Hence µMg increases.
sin0-µcos0
29. [b]
31. [a, d]
At 0 = 30°, mg sin0 = mg/2 which is equal and
opposite to external force. Hence at this moment
The free body diagram of blocks A and B is as sho'IVll
below.
friction force is zero. As 0 starts increasing from 30°,
the mg·sin0 component starts increasing. Here
!! f2 q_N_"c, " ' • j
I
( mg sin_0 - m;). will be compensated by opposite . T,
I mg ,•
friction force until
· mg sin0 ~ mg < µmg cos0
2
sin0-.! < µ cos0
2
N1 =mg ... (i)
sin0 -µ cos0 < .! N2=2mg+N 1 =3mg ... (ii)
2
. 1 f2=µN,=µmg ... (iii)
sin0- cos0 < -
2 J/=µN2 =3µmg ... (iv)
../2 (cos.'.: sin0 - sin.'.: cosa) < .! Tr= f2 =µmg . ...(v)
. 4 4 2 F =fr+ f2 +T1 = Sµmg ... (vi)
sin(0+%) <
2 1
•lt
0 <4+sm• -1 ( 1 )
2../2

www.puucho.com
Anurag Mishra Mechanics 1 with www.puucho.com

I•-FORCE
L..
ANALYSIS· ·· •• "· {•2p5j'
- ~ - - - - - - - - - ' ~ - ' - - - - - - - - ' ; ; , , , , · , C C . . ' " " ' , e . '- - - - ~ --------'------~=~
32. [a, e]

N = mgcos0+Fsin0 f = (mn)a
Also if mg sin0 = F cos0 then the p
friction force acting on the block Adding: P = (mA + mn)a ~ a
is zero.
f,rati,lmaximum = µN =µ(mg cos0+F sin0)
(The maximum static friction that can act on a body
under a given solution is known as limiting friction
under the given conditions.)
33. [a, e, d]
Let
When v ¢ 0 the acceleration is 0 :-;; t < (mA + mn)mAµg
Hence for
ains, = _!_m (mg sin 0 - µmg cos0) kmB
J ,-------
! a,
X ) g(2- X)
=g ( sin0- cos0 ../2 : µmAg ----- 82
2 2 1~8 I
I 8~
v dv = __f__(2- x) J (mA + ms )mAJJ9
dx 2../2 km 6

=> gm f<Z- x)dx


fvdv = 2,i2 kt
0 0 a1 = a2 = - - - -
(mA +mn)

v: = 2
~[ 2x- x:J: (mA + mn)mAµg
And ,,or t > -~-~~=
kmn
v =Jg../2 m/s kt-µmAg
a,
Also it is clear that for x < 2 the body accelerates, at · mA
x = 2 the acceleration is zero and for x > 2 the body µmAg
and a2=--
retards till it comes to rest. mn
34. [a, e, d] 36. [a, e, d]
Hence F = JJ 2 ·+FJ Frictionrorce on m, = µmg = ~ ?~x
x 10 = 20
Hence f>FN &F>f.
I
rn, D .
Also F·= JFJ + f 2 + 2.FNf-2.FNf
= J(FN + fl 2 -2.FNf
For m 2
L :
o•

·---.
=--,:... T For

- - - _ _ _20
_ J'
,

As obvious from diagram that the masses m2 and m3


= (FN + f)-o will not move, and de-acceleration of
20
Again F = JFJ +/ 2 -2.FNf + 2.FNf m1 = = Sm/s 2
4
= J(FN .:_ fl 2 +2.FNf Also using v=u-at
= (FN - f) +o O=u-Sxl ~ u = Sm/s
> (FN - f) When m1 stops slipping over m2 ,
Hence FN - f < f < FN + f
35. [e] · a m3g = 2xl0 = 2 m/s2
For a certain maximum value of r--;:::==:=:;:;;, .m1 +m 2 +m 3 4+4+2
P both the blocks move without I 1 : ,.~J
any relative acceleration. In this ~;;/~m""'""'"'~ J
range

www.puucho.com
P' )\
\ . I'1:·
Anurag Mishra Mechanics 1 with www.puucho.com

1256. ME~HII~
37. [b, c] Since tan a=µ > tan0 so block will came to state of
I· . •-.~I permanent rest and then required frictional force will
be mg sin 0.
i '"-~ ~ i•. 42. [a, c, d]
t ;..~s~ .....~..J since mas 'm' is at rest so riet force on it will be zero.
For motion between AB Also friction force will balance mg sin 0, so its value
g sin0 will_ be equal to sin 0.
a=-- (downward) For M + m as a system net normal force will be
2
For motion between BO
(M + m)g.
43. [a, c, d]
gsin0 For equilibrium
a=-- (upwards)
2 T,,/3 ~ f =50
Also the velocity is increase from zero to maximum 2
value at B and then starts decreasing with same rate
and finally become zero at 0.
38. [a, c] Taking torque about centre
As discussed the above question velocity is maximum T=f
at B and zero at 0. 2
r(,,/3 + )=50; T = ~ = f
39. [a, b, c, d]
For man and plank as a system
T+N=2mg
N=2m-T
-~----~ ·-;· --~:1
2 2+,,/3

T = µN = µ(2mg -T) ,.•-·····


T=~g ~~,11.••' ,.• \ I

l+µ ,... s~,/3 I


Friction force =µN =T _._ 100
1
___________ _J
No horizontal acts on man, so no friction force will act n 50 r;:; ·
on man. Also he is in equilibrium so net force acting on· N=5w~+--
man is zero. 2+.J3
40. [c, d] 10W3 + 150 + 50
=
Tension in the tread =mg i~N '·! 2+.J3
Assuming (M + m) as a single IM m) .· T·=. ipgj = 10W3 + 200 = l00 N
mass unit, the only external 1fr l 2+,,/3
force acting is rightwards so !~.- - -(M + m)g ·. · I
- · __;,;_.;,_- '-"'j 44. [b, d]
frictional force T = mg Speed is constant and tangential acceleration is zero.
Normal force between wedge and block is zero, so no 45. [a, b, c, d]
frictional force acts between wedge and block.
For ABC part :
At limiting condition for (M + ml system.
mv 2
T=µ(M+m)g N+F+mgcos0=--q ... (i)
=>

41. [c]
mg=µ(M+m)g
µ=--
M+m
m
'•
··---- N ;~1 r

If block moves up, downward acceleration


=g sin0 +µcos 0. i . j--~J.:
.. I
So using v = u + at I ,:
0 = v 0 -(g sin0+µg cos0) ' " - - - - - - - - - •E
t= Vo
and from energy conservation :
g (sin0 + tanacos0) v q =.J2gr (1- cos0) ".(ii)
www.puucho.com
Anurag Mishra Mechanics 1 with www.puucho.com

FORCE ANALYSIS -- 257


~-------------- ---- - ·--·. --- --- ----- -------- ------------ - - - - - ---
From eqn. (i) and (ii) atG R=mg (·:8 = 90°)
N =F+ 3mg case- 2mg ate R=mg (·:8 = 270°)

~-~,
(a) at C ·: e = 90° 48. [b, d]

~~~·· , ...
=> N=F-2mg
=> F=N+2mg
=> F :i! 2mg
(·: N :i! 0 for M is not to leave the track at C ) ~t:';->_:::, m: ---~-- . ·-ii~J".'9 __ ,
(b) at B e = 90°
. . mv 2
=> N =F-2mg s1n8 = - -
1V
r
(c) For CDE part
Ncos8= mg
mv 2 v2
N -F - mg case= - - ... (iii)
r Hence cane= -
rt
From energy conservation
(a) carwillnotskidifv = 40km/hr.Hence (a) is na_se
v = ~2gr(l + case) ... (iv)
(b) if V < 40km/hr
From eqn. (iii) and (iv) v2
N -F = 2mg +3mgcos8 r1=--
g sine
=> N_= F+ 2mg + 3mg case
ForD 8=90° => r1 < r Car will slip down
=> (b) is true
=> N=2mg+F
(c) If v = 40km/hr
True, when cosB.= ~ which is possible between A and
3 mv 2 l
N=--·- (:.sin8<1)
C. · r sine
46. [a, c, d] mv 2
=> N>--
Consider point P on circle of r
motion => N = mg
--+ --+ --+ " --+ Also, Ncos8 = mg
V = V i, ac j, (1) = rok_ case
--> => N>mg
a=ak (d) is true, (c) is false.
--> --> --> -->
(a) ro .L v => true (b) w .La => false 49. [c, d]
--> --> --> -->
(c) w .La, => true (d) v .L a, => false
47. [b, c, d]
t· - .. - - - ~ - - -

v2
a, =-=g. => (d) is correct
r -->
N-mg = ma, cos8 ·: Friction force = m anet
at A N = mg+ ma, (·: 8 = 0)
=mg+mg=2mg=2W
atE N = mg - ma, (·: 8 =it)= 0
3
for G and CB = ~ and " respectively :. N = mg
2 2 => (c) is correct.

www.puucho.com
Anurag Mishra Mechanics 1 with www.puucho.com

[.2ss: MECHANI~-· I
50. [b, c, d]
at x = L, F =!Mro 2L
2
r = ~1
2
_-(½r 1; = l at x =L/2, f = 1/8Mro 2L
54. [a, b]
T1 sin 30° = T2 sin 30°+mg
v2
~ T1 =T2 +2mg ... (i) aR = -
~ (c) is true, (a) is false r
2
= rkt 2
Tcos30°+T2 cos30°= mro 2( 1; L)
v
1 · v=-JK.rt

~ T1 + r; = mro L2
... (ii) dt
=
-dv = "Kr = ar = constant
2
~ (b) is true v Krt. 2 2
aR =-=--=Kt
From eqn. (i) and (ii), T2 = m(ro~L -g) r
acceleration net = ~ Kr + K.2t 4
r

~ ~ ~
ro 2L
'-¼

~ T2 > 0, only when - - ;;, g Power = F- v = Ftangential v


2

~ co e: Jf i.e., (d) is true. 55. [a, b]


dk ds
= mkrt

51. [b, di P=-=2as- ... (i)


dt dt
X=Rtane .
~
dx 2 de
V.badow =-=Rsec e--
½mv =as ~
2 2
v=~s ...(ii)
dt dt
From eqn. (i) and (ii)
e=vt de=~
R 'dt R p = 2aS X {2a S = (2a)'\'2 S2
v-;;; ml/2
~ _Vshadow = V SeC2 ( ~ ) dv 2a
aT =-=-s
dt m
2 2 2
e,hado(~{!:)= ~ sec (~)_tan(~) 56. [a, b, d]
T is same for all three particles
52. [a, d] 21t 21t 21t
Since the road is banked for ~ -=-=-
co, 002 Ol3
speed therefore,
· mv 2 ~ ro 1 :ro 2 :ro 3 = 1:1:1
mg sine= - - cos8 V1 = r1ro1, Vz = r2C02, V3 = r30>3
r
here a car moving with ~ v1:vz:v3 =r1:r2:r3
speed v will not slip even in :::::::) a1 :a 2 :a 3 = r1cof;r2ro~:r3ro~-= r1 :r2 :r3
absence of friction. 57. [b]
It speed is less than or greater than v the above V=a..fs
condition is not satisfied and car will slip, this is even dV a VdV a 2
true for stationary car (v=0). ~ a=--=-
dS = 2../s aS 2
53. [a, b]
a2
M P=F·V=-~..fs
dM=-dx 2
L
58. [c, d]
dF = dMro 2 x
Consider a small section of ring
dF = M ro 2 xdx f" X
L
·Tcos8
. .~
. .-Tcos8i
lM
F=--Ol X
2L
2 2
+C T,-! 'W!"T _
F=Oatx=O T sin8 r·sinB
~ C=O
Zf sine= dmrro 2

www.puucho.com
Anurag Mishra Mechanics 1 with www.puucho.com

FORCE ANALYSIS 2s91


sine-ease is small Average force = m x average acceleration
zre = (2r8m)rro 2 =mX---
2v 2 -./z
~ T = mr 2ro 2 = constant ltR
59. [b, c] 62. [a, c]
at time ·of slipping f = µmg r = 0.5
f cose = mar ro = 0.4 rad/ sec
f
. mv 2 v = rro = 0.4x 0.5 = 0.2m/sec

r a= rro 2 = 0.5 x (0.4) 2 = 0.5x 0.16


fsme=--
r
= o.sm/sec 2
!2 = (mar)2 +( m~2

r
63. [b, d]
For collision
(µmg)2 = (mar)2 + ( m~2 Position of A = Position of B
ltR + distance travelled by a = distance travelled by B
v4 itR + vt = vt +.!at 2
µ2g2=a:+- 2·
r2
v2
Also, tan8=- t=~2:
arr
60. [b, c, d] .-. they collide after time t = ~ 2:R
1-·- --- ---- -----
1~ ,Potential energy= O va =v=at=v+-J21taR
I a ' · VB == V
'
I 64. [a, b]
! ______ mg____ -~- l
For angle = <jl v = 0 (extreme position)
mv 2
T-mgcose =- -
1
From energy conservation
(T1 -T2 ) sine= mg
.!mv 2 = mgl(cose- cos<jl) 4
2 (T1 +T2)cose = m x dx x 2g
~ T=mgcose+2mg(cose-cos<jl) h
2L
T = 3mg case- 2mg cos<jl ~ T1 - T2 = mg x - ... (i)
h
~ T = mg cose ~ e = <jl
2g L SmgL
~ T=mg ~ T1 +T2 =mxdx4x-x-=-- ... (ii)
h d h
~ mg = 3mg case - 2mg cos <jl
e = cos -i[(l
+ 2cos<jl)]
3
5
From eqn. (i) and (ii) T1 = mgL
h
T2 = 2mgL
~ if e is small case will be large h
~ T = 3mg cose - 2mg cos <jl will be large ~ T1 :T2 = 5:3
61. [c] Clearly

[ _, . _, ] 66. [b, c, d]
F = -U O a sin ax ~ acceleration is not constant for
Al_~--v
a = Vfina1-V· 1mn
··a1
average time elapsed K-U 0 cosax=0 ·

vi-vJ
=--=---
2v 2-./z
i\J21
(_____ J
K=U 0 cosax=0
xmax~V=O
K=v 0 cosax=0
U+K=2U 0

ltR ltR
2v
www.puucho.com
Anurag Mishra Mechanics 1 with www.puucho.com

~ .... . "

ax= 2:, at this point Fis -ve. So particle comes back. 68. [a, b, c, d]
2 (a)' A cork is fixed in a take, net normal force on
Kmax = V0 cosax = v 0 curved surface is zero but friction is not zero.
67. [a, b, d] (b) µ,N is value of maximum static friction.
(c) Static friction opposes tendency 9f _relative
motion.
• • I
. A f l n i t i a l at rest rough!
1
: Rough !
• (d) . 1 . • I
' 2 Moving ,
5 .. '
a~celeration of 2kg block (a2 ) - o- 20 = 15m/s 2 i 'I
_.:__..J
2
acceleration of 4kg block(a 4 ) 5o- 40 = 2.5m/s 2 i velocity of upper block will be increased by kinetic
4 , friction.

r.
Ib -, +.
;.--. '#
~
~- f'
,. . --,·
~S. -V/
n

.e '. .
• ' :3--. --.~~-T~-~~he~~i~n ·B;;e~:Pr~bl~m~ .
- - - · - - - - - - · - --·-..··-~ - - - - - - · - • • •, ••, ••• , ••• ' _ _ h _____ --
- -- . -- ·--·-~
. . "
.1
' ·J'
, --- '<.. ~ _:',,,

Passage~~ i.e., µ .< tan0


1. [a] both block will move

: _" ;z:·N:' f:f,~~=~:5·:;6~~8'1


2. [b]
m 2g-T2 = m 2 a ... (i)
'---7
. '
'
:
20.~ o,s
.· 2QxQ.8°.
I
•I

I
T-m 1g =m1a ... (ii) N · N' , :. ' ·, !
T2 = eT1 ... (iii) · . ' ~ \ i· 2f ,~;,=0.5•60•D.8.:. I
m2g - eT1 = m2a f .,·, =,24 · .
I
eT1 - em 1g = emaa !
1
--~ 4oxo.a·
-
'
.. ' ;
I L.4~·~--6-'.,' ..... ····---····-
(m 2 - em1 )g = (m 2 'f- em 1)a j,m,g ...... , ,
,I

em1g g Let both blocks move together with acceleration 'a'


a=--=-
3em1 ·3 12-f, =2a
3. [c]
·2m 2g
T2 = m 2g - m 2 g I 3 = - -
24+ Ji ~24= 4a
• 3 12= 6a
4. [d] T-mg = m,f =} T =·4m1g a= 2 m/sec 2.
' . 3 3 =} f 1 =12-2x2=4N
Passage-2 i.e., f, required = f1 max.
1. [d] :. both will move together
tan37°= 0.75 4. [a]
Here µ 1 =µ 2 = 0.80> tan37° i.e.,µ> tan0 See previous question solution
=} sufficient friction is there at each surface · 5. [a]
:. No block can move. µ < tane for both block =} both will move
2, [a] · F1max = 6x 0.4= 6.4.N
Friction force = (2 + 4) g x· sin 0 t,m•• = 60x 0.Sx 0.5 = 25
= 6x lOx 0.6= 36N =24
3. [b]
Here µ 1 =µ 2 = 0.5 < tan3r·~ 0.75

www.puucho.com
Anurag Mishra Mechanics 1 with www.puucho.com

i,t___FORCE ANALYSif·.- ;,
,,_.--~:_:_~,,L';_ _,;_ _ _ _ _ __

for motion to begin F = f,


2f1 <= f2max
2f, <= 2f2max
=> F <= f2max =. 3
=> t = 6sec for both blocks
2. [c]
12- f, = 2a
24+!1 -24=4a
12= 6a
a= 2m/sec 2
=> f 1 = 12-2x2 = SN
But f 1mll = 6.4N Let both blocks move together with acceleration 'a'
:. both block will not move together and frictional a.st - J, = 2a ... co
force acting = 6.4 N f1 -3=3a ... (ii)
6. [b] when f 1 equals to 4 N then relative slipping just likely
See previous solution to occur. Putting f, = 4 in eqn. (ii)
1, ••.I
7. [d]
I .)t:.
N 4-3 = 3a
Here µ 1,µ 2 < tan0 i.e., 0.4 and '1'~ I => a= 1/3
I I
0.5 < tan37°= 0.75 12 16 I
Putting f, =4, a= 1/3 in eqn. (i)_
.-. both block will move ' I · 28
We get t = - sec
N N I 3
Let they move together with
acceleration ' a' iI 3. [c]
f1 ' .
J,max =I.6xO.5=8N 24 _ - ,f Since the two blocks move together for t = 28/3'
Jtax = 6Ox 0.8 x 0.4 = 19.2 at t =8 sec no relative slipping occurs
both can be treated as single body
12-fi =2a
F-3= Sa
24-19.2+ J, = 4a SxO.5-3= Sa
12+ 4.8 = 6a
=> a= 1/5
a= 2x 0.8 = 2.8 1
Putting value of a = 2.8 => f 1 -3=3x-
5
F1 = 12-2x 2.8;,, 12- 5.6 = 6.4 < SN 3
f 1 = 3+- = 3.6N
:. both block will move together 5
8. [d] 4. [a]
See previous solution 28
At t = 10sec > -sec
Passage-3 3
relative slipping between the blocks occur

1. [d]
·I · _
1~·-,
I
·2 ' · i.1: l
' 5. [c]
friction force = 4 N
, t, .. ·I
; , 20 , .: Upto 6 sec there is no motion 6 ·:, t :, 28/3 blocks move
L ..... ---- - -··· .. together with
N 1 = 20, fimax = 4N r-- ·- - - · ' - ;:::::J
f .......N •. -~ · I ·~ , a . s t 1
l· f'Jmax =: 4N ~,' '\', J•·· -•i•/.
'· ;J '.
l_~--'_N,··'_f_),
)~ . , I
l

O.St - 3
a = ---
- - ~--- ___, -· ...j-.f~---

(from qu_estion 2)
. ; . I 5
L __ ... _. -~Q __ .c. 1 28 ,;;. t reIanve
. s1·1ppmg
.. occurs
-
N2 = 50, f2max = 4OX 0.06 = 3N 3
at first will be no relative slipping between blocks •O.St ~ 4
a=---
since f1max > f2max. 2
www.puucho.com
Anurag Mishra Mechanics 1 with www.puucho.com

f 262 l\\ECHANICS,1 · j
6. [c] 4. [b]
Just before coming to rest, maximum frictional force
will be acting fmu = 2mg sine
Passage-5
1. [d]
Upto6sec a=O N, I
N1 =mg
(No motion is there)
6:,; t < 28/3
0.St -3
a = - - - (blocks move together)
N 2 =mra
N= .J~N_f_+_N_i
N,
goo .
(3-d,~g\Jre)
I
5
28/3 :,; t ·relative slipping between blocks 2. [b]
4-3 = 3a Only frictional force
a = 1/3 = constant gives the required centripetal force as it is the only
7. [c] force acting along the surface of rod
f2 = 0.St (t :,; 6sec) f = mrro 2 = mr(at) 2
for 6:,,t f2 =3N 3. [a)
8. [a]. When f mu = mrro 2
upto 6sec f 1 = 0.St (Since a = 0) ... (i) Then at this moment sliding just occurs
6:,; t:,; 28/3 both blocks move together mr(at ) 2 = µJ(mg) 2 + (mra) 2
0
~ 0.St - f 1 = 2a
J1 -3=3a Passage-6
1. [b]
ft= 1.St + 6 ... (ii)
28 2
For :,; t f 1 = 5 N (maximum) ... (iii) • ro, •0.6' -~ N.. ' · i ·
3
roo2 • \/ •• 8 8. 8 : .
Passage-4
1. [b]
At any position x ,
.7
r<ii2 X 0,8
·"·12
° ,
,' .16
,,_c'-;,e20e,__..J

a= g sine-µg cose = g (sine-kxcose) N sine= 2 x o.2ro 2


a=gsine(l-x) Ncose= 20
uau = g sine(l- x) N -16 = 2 x rro 2 x 0.6
dx . =2x0.2x25x0.6
V X
~ f vdv = f g sine(l- x)dx N-16= 6
0 0 ~ fmax = 22x 0.3 = 6.6N
Also 12- f = 2>< rro 2 x 0.8
v2 =gsine(x-x2)
2 2 . 12- f = 2x 0.2x 25x 0.8
vwillmaxwhen a=O ~ x=l f=14N
Vmu = sine .Jg f required = 4N
2. [b] 2. [b]
For u = 0, X= OJ X=2m
3. [a]
At x=2,
N
µ = 2k =: 2tane
/
[9' .s-4),
• ,·
fmax =µN = 2tanemgcose ' . ()

mg cos 8
= 2mgsine Ncose= 20
f=mgsine N sine= 2x 0.2xro 2

www.puucho.com
Anurag Mishra Mechanics 1 with www.puucho.com

. 0.4ro2 2 3 300
tan0=-- => v =lO0x-=-=75
20 4 4
CO =~ x tan 370 = 200 x~ = 600
2 v= 5--J3 m/ sec
0.4 . 4 4 16 2. [b]
(0 = 1oJ6 = ~ .[{, = [75
4 2 f2 1· ~.Ncos.8~f_.·
- ...·•
3. [a] I · . .a,· r-l.
For maximnm co frictional force f acts downwards. :• . _e
2
f=µxN=-XN 1· mg .•.: · •:
3
2 2
N -16 = 2 x rco x 0.6 ... (i) . + f cose = -
N sme mv- = ::..:..:.c..::.::...:.
50x100) X cose
f + 12 = 2 x rco 2 x 0.8 ( r 10
(N cose - f sine= mg = 50 x 10) sine
=> f = 500cose- 500sine
,.,,_ ~ .·.
_, = 500x 0.8-500x 0.6
:~-~·:·/
•. __

f = 500x 0.2 = 100N


12 3. [d]
1_ ..
Ne ·NA

=> -N + 12 = 2x rco 2 x 0.8 ... (ii) •
3 >rriv2
r
from eqn. (i) and (ii) 1m
2 25 B A
(0 =- ,mg
3
(25 When car just topples, contact at B will be no more i.e.,
=> co= VJ rad/sec
NB =0
4. [a] M9ment about A is just zero
For minimum value of angular velocity' f should ·act 2 mv 2
·=> mgx-=--xl
upwards (i.e., up the surface) 2· r
~,.,~-
=> gxr=v 2 => 10xl0=v 2
~f T~).rro2. : 0;6 => v = lOm/sec
..
12A1s Passage-11
...,,.,, 0,8
1. [b]
2 ..... (i) If velocity of girl w.r.t. ground = v,
12- f = 2x 0.2xco x 0.8
N -16= 2x 0.2xco x 0.6 2
... (ii) trolley velocity= -v/2
vre1 = 3v/2
f=~N ... (iii)
3 Fmax =µgm => amax = µg = 10/3
(0
2
=-
25
9
.==> NS CO= -
9
and Vmax(rel) = 3v/2·=·9m/s
Vmax = 6m/s
· 6x 3
Passage-7 t = v/a.= - - = 1.8s
10
1.. [b]
2. [a]
mv 2
Nsine=-- v = 6m/s Vy = 3m/s KE= KE 8 + KE 7
8
r N cos8 .... '. •
1 · 2
Ncose = mg • KE; = - X 50 X (6)
2
v2 ~sin8 1 . 2
tane = -
rg :•. and KEr =- x lO0x (3)
. ..' 2
3 v 2 .
mg
.· ..
KE= 1350J
4
= l0xlO www.puucho.com
Anurag Mishra Mechanics 1 with www.puucho.com

3. [d] ni
-=5
2 M
3
Dr 1
·=> --
Dr+Dg 3 4, (A)
Dr . 1
=> -=- => Dr =4m
12 3
4. [c]
When vrel = 9m/s =;> vg = 6m/s
Max. retardation = µg = (10/3) m/ s 2
Minimum time= v/a = 6/(10/3) =18/10 = 9/Ss (B)
5. [c]
Force.on trolley 7 lm/s 2 x.IOOkg = IOON
This also the force on girl by newton's 3 rd law
6. [b]
D• 2 D• 2 Dg .2
-=- => => - = ~
Dr l Dr+Dg 3 , 12 3 (C)
=:, Dg = Bm (in earth frame)
= 9m/s
v,el
=> vg = 6m/s
v 2 -u 2 =2as 5. P = k = constant F = k = constant
2
=:, 6 - O= 2 x ax 8 =:, a= 36/16 Fv= k ma=k
=:- 9/4=2.25m/s 2 mav=k
dv
m(v:} =k mv-=k
~ ~. c~~ ~ - . ds
~ ~Ma~~h!!l!/;I!'!!t~le~.!~!!1~\ 11. Minimum value of F regarding to move the block is
2. Maximum acceleration due to friction is µg F. = µmg =lx2Jzx10= 2 0N
So a_> µg ~ A mm. ~l + µ2 J'j,
a<µg~f, For less than· 20 N friction will be static always. For
a=ug~f, 20N friction can be static as well as limiting since
a=0~f;,,o F, max.= µ N and µ = 1 so F = N maximum value of F
3. regarding to move the block is
µmg= 2J'i.g = ;!SN
So for F = 25N friction can be static, as well as kinetic
and limiting. ·
F = 30N friction can be . zero when force is acting
nearly at 90°. ( s_ince F > mg).
12. No slipping any where.
-mg sin37°-µmg cos37°= Mg
Net force is centripetal as v = CO!Jstant.

www.puucho.com
Anurag Mishra Mechanics 1 with www.puucho.com

:; ,K;: ~~~-
, , r ·,, •
,c,.b·.L;:i:i~t,
\

WORK AND ENERGY/

WORK DONE Examples:


(i) Work Done by a Constant Force 1. Consider a block sliding over a fixed horizontal
The work done on a body by a constant force is the surface. The work done by the force of gravity and the
product of the force in the direction of motion and the reaction of the surface will he zero, because force of
magnitude of displacement. gravity and the reaction act perpendicular to the
__, __, displacement.
W =Fscos0 = F· s N
~
F sine -2-+
direction
rmmrmmrrrlmr of motion
F cos B mg
Fig. 3.2

s
2. Consider a body moving in a
Block displaced by an circle with constant speed. At -->
external force . every point of the circular path, t
goo s

~ r=go•
5
the centripetal force and the 1 '
displacement are mutually
F W=O perpendicular (Fig. 3.3). So, the
0 0 work done by the centripetal
Fig.3.3
s F F force is zero.
3. The tension in the string of a simple
Sign of work depends an angle
between force and displacement pendulum is always perpendicular to
Fig. 3.1 displacement. Which place along arc
(Fig. 3.4). So, work done by the ,
Case I : tension is zero. . . :r
When 0 = 90°, then W = Fscos90°= 0
So, work done by a force is zero if the body is
displaced in a direction perpendicular to the
Fig. 3.4
direction of the force.

www.puucho.com
Anurag Mishra Mechanics 1 with www.puucho.com

!266 ______MECHA~t~s-17
Work done by a force is zero if the body suffers
no displacement on the application of a force.
(a) (b)
A person carrying a load on his head and standing at a
· given place does no work.
Work done by a force is said to be positive if the
applied force has a component in the direction of the
displacement.
(c) (d)
Examples of Positive Work:
1. When a horse pulls a cart, .the force applied by horse
and the displacement of cart are in the same
Fig. 3.7
direction.
2. When a body is lifted vertically, the lifting force and In (a), 0 = 0°, cos0 = 1 (maximum value). So, work
the displacement act in the same direction during done is maximum.
lifting. In (b), 0 < 90°, cos0 is positive. Therefore, W is
positive.
In (c), 0 = 90°, cos0 is zero.Wis zero.
In (d), 0 > 90°, cos8 is negative. W is negative.
1. Work is defined for an interval or displacement.
->
2. Work done by a force during a displacement is
1 Positive work
5
i
independent of type of motion i.e., whether it moves
i__ _ __ Fig. 3.5 · ____ _j with constant velocity, constant acceleration or
retardation etc.
3. When a spring is stretched, by an external force both 3. Work by a force is independent of time during a given
the stretching the external force and the displacement
displacement. Work will be same for same
act in the same direction. displacement whether the time taken is small or large.
Work done by a force is said to be negative if the 4. When several forces act on a body, work done by a
applied force has component in a direction opposite force for a particular displacement is independent of
to that of the displacement. · other forces.
Examples for Negative Work : 5. A real force is independent of reference frame.
1. When brakes are applied to a moving vehicle, the Whereas displacement.depends on reference frame so
braking force and the displacement act in opposite work done by a force is reference frame dependent.
directions. Unit of Work
2. When a body is dragged along a rough surface, the In SI i.e., International System of units, the unit of work
frictional force acts in a direction opposite to that of is joule (abbreviated as J).
the displacement. . One joule of work is said to be done when a force of one
3. When a body is lifted, gravitational force acts newton displaces a body through one metre in its own
vertically downwards while the displacement is in the direction.
r-· ·- ------ ----- --·--- --- -- ----
vertically upwards direction.
ljoule = 1 newton x 1 metre= 1 kg x 1 rn/s 2
= 1 kg ms-2
1 t Work done by a force when an object is displaced
along a general path ..
. ->
->
The differential work done dW by any force F on a
5 ->
system is scalar product of F and differential change in the
Negative work -> .
position vector dr of point of application of the force
b •.. ·-·- ·-·· _ Fig. 3.6 _____ _
Fig. 3.7 shows four situations in which a force acts on a
box while the box slides rightward a distance d across a
frictionless floor. The magnitudes of the forces are identical,
their orientations are as shown.

www.puucho.com
Anurag Mishra Mechanics 1 with www.puucho.com

IWORK AND ENERG'i'.


+ The work done on the system by the force
component Fx as the system moves from X; to x f is
the area under curve between X; and x f.
W = J:I
Fxdx+ fyYJ Fydy + f:l Fzdz
' ' '
'Fx

X
_ _ _ _ _ _ _ _ _ _J Flg,3.8
........
dW = F-dr
Each term is the area under the curve of the graph of
--+ A A ,.,_
that force versus the corresponding coordinate.
dr =dxi+dyj+dzk
W = fi
f •
(Fxi +Fyj +FzK)
• ,. ke~«~il_C~
-(dxi+dy j+dzk) !An objecUsdisplaced;:;~::~:~ vector;t1 =·~2 i+3);
=ff Fx dx + ff Fydy +ff Fzdz
I , l l
to ;t2 =(4):f 6fc)m under aforce• F= (3x 2i + 2Y.i)N. Find
If force F is constant, the. work do[lej)y_this iorce.
W = Fx J;
dx+Fy f; dy +F,f; dz
Solution : W
1--+
= f! F- dr
---+

=Fx(Xf -X;)+Fy(yf -y;)+F,(zf -Z;) r,


........ ....
= F-b.r = f_; (3x i + 2.Y.i) - (dx i + dy j + dz k)
2
....
where b. r is displacement of system. '1

+ When more than one force acts on system, . ....


---+ ---+ -+ ---+ = f :2 (3x2dx + 2y dy) = [x3 + y 2J(f ~?
F,ota1 = F, + F2 + F3 '1
f.... ....
Wtotal = Ji Ftotal • dr = 83J
or W,0 , .1 = f.1-iF,-dr + f.
l
--+
1
f--+ ---+
F2 · dr + J.1 F, 3 -dr + ...
f -+ --+

=W1 +W2 +W3 + ... An object'is displaced from point A(2m, 3m, 4m) to il point
Total work done on the system is work done by the total
force or algebraic, scalar sum of the work done by individual
forces.
+ When a particle moves along a curved path, the
IF~ (2 i.~
IB(lm, 2tii1 3m) under 'ci _constant .force
3) +4 k)N. Find.the work done by this force in
this process., . . , .
work is done by tangential forces only. ; -+ --+
W=fF,ds Solution : W=f} F-dr
Centripetal force is perpendicular to small displacement r,
.... 3
ds along curved path; therefore centripetal force does not ·= J(lm2n\ ml (2 i+ 3j +4k)-(dxi+ dy j + dzk)
(2n\3m4mJ
perform any work., - - - - - - - - - , , - - ,
= [2x + 3y + 4z]Clm2m3mJ
(2m3m4ml
F,
;(·.

I ······•·.·..
...
=-9J

~.
Illustration for Work Done ·
(i) The Fig. 3.10 ~hows a smooth circular path ofradius
.. .( R in the vertical plane which is quarter of a circle. A block of
··- ...... mass m is taken from position A to B under the action of a
constant force F that is always directed horizontally.
__ Fig. 3.9 (a)

www.puucho.com
Anurag Mishra Mechanics 1 with www.puucho.com

'•'"
i:.· ~-,·::,-,·,. tr·/: :,i
.{:.~h~-~---~-~·~·~=-~·- ~ "-:,~.;-:; •, - - -
'·1 MECHANf($-1 l
- - - - ~ - - ~ - - - - --·-.. --~;.;/4...l

,r;-----··"·7--:·7
?-------~'--,- 13 · Fl
I ::·
tRl,
" . : ''

I [ ~'-'::+F

A
Fig,.3.10 (a)i ,,
...... = fFdscos0
WR= JF-ds
or · W = J:Fdx (dscos0 = dx)
or W =FR
As the block moves from A to B, the displacement of the
block in the direction of force is equal to radius R. -
--:a·-~
,R !: , •: ;l: .. Thus
... ...
dW = F-ds = Fdscose

~
: :.:. r
dsj(! d~
dX-,.,, ,
=F(Rda)cos (~-%)

~--Fl~g.. 3.10 (E]...:_ ., . or dW = FR (~os.C: + sin.<:) da


-./2 2 2
Therefore, the work done by the constant force F is
. / . W'=FR W = ~[ J;12 cos%da+ J:12 sin%da]
(ii) If the block is pulled by a or W = FR-.J2
force F which i~ always tangential to
Conservative and Non-conservative Forces
the surface. In this case force and
displacement are always parallel to A conservative force is one whose work done is
each other. The displacement of the independent of the path taken by the system, or whose work
done along a closed path is zero.
block in the direction of force is ~ R.
2 We can write the above definition in the mathematical
Thus, the work done by the force is_ form

W=F(:)=~FR w. ·= f.c1 ose.d path i. ds = ofar a c~nservative force·

(ili) Block is pulled with a co,nstant force F which is wp~th1 = wp,th2 for a conservative force
always directed towards the point B. In this case angle where :
between force vector and displacement vector is varying
'~~-, R • <?'"fP- - -

,;
!
I
-'
·'
... ...
. In Fig. 3.12 (b) the a,ngle between F and d sis~- Block is
lnitlal,position and-final.Positidn, ·
- '

at angle a from vertical. The magnitude of ds is R da. The ·-"


.·-· ---·- -·
""'"
Fig. 3.13
- ----- - <

relation between 0 and a is If the work done by a force around a closed path is not
zero or if the work done by a force as a system moves
between two points depends on the path taken between two
www.puucho.com
points, then the force is called non-conservative. Work
Anurag Mishra Mechanics 1 with www.puucho.com

I WORK AND ENERGY p ·, 2691


1-----~ ' ----·--------------- --"-----------------·~·-~
done by frictional force as a block is dragged along the W,0 ta1 is not equal to zero; therefore frictional force is
ground depends on the path, length, therefore it is a not conservative.
non-conservative force. CONCEPT OF POTENTIAL ENERGY
r:liE -- -~ -.- . r-;i ~
[,_; C?~~~l;?z.;}~j 3 ~
When a conservative force acts on a system it changes
energy of system. Energy associated with conservative forces
r-·--·· ·. · -- ---- -·-··· --------------:--7
;A block is being pulled slowly along a frictionless incline1
is called potential energy. Only conservative forces have
potential energy functions associated with them. Since
;[Fig. 3E..3(a}J. . .. .• ..... __ . _ __ __ . j
' A conservative forces are function of position only, therefore
! : F C S2 , ,
potential energy functions are functions of position of the
' system.
Formally we can say that the work done , by a
A
S3 conservative force on the particle is the negative of the
change in -potential energy of the particle.
J:~ dU = - J:: F(r) · dr for a conservative force
-c---- b ····> c- ------ b. ------•

Fig. 3E,3 (a)


Urf -Uri = -f't F(r)·dr
r,:
+ Work done by a
( a) Show that the gravitationalforce is conservative . . conservative force
(b) Now.consider the incline tb be rough to show thatt~ej does not express
ftjc_tion,alforce is non-conservative.:: .... _ _ . _· --~--_;-.J absolute value of i
potential energy at a
Solution : (a) In Physics the phrase "slowly" implies
-+ point, . it express (
that the body moves in equilibrium, i.e., L F,oral = 0. We change in potential
1

arbitrarily choose a triangular path ABC as shown in Fig. energy. We choose a


3E.3 (a). Work done by the gravitational force can be convenient reference Initial position
calculated separately along each of the paths AB, BC and CA. point and assign it zero , Fi~~ 3.11 _
W AB = mg xLcos<1>, where <I>= 90°+a potential energy, then
we obtain
W AB = -mgLsina = -mgh
W8 c = mg x bcos90°= O
Wrn = mg x hcos0°= mgh
~U= J: dU
= U(r)-U(r0 ) = U(r)- 0
= WAB + Wsc + WCA
w,otal whereU(r0 ) is reference point energy.
= -mgh+ O+ mgh = 0 In example 3, negative of work done by conservative
which proves that'the gravitational force is conservative. force mg is change in potential energy of block as it is
Another important point to notice is that dragged from initial point to point,
WAB +Wsc =WAc u 1 ...,u, = -(-mgh) = +mgh
i.e., if the block is taken to C along ...... -; , '
u 1 =mgh+u,
path A ~ B ~ C or along path A ~ C, If we choose a reference level at the base of incline and
work done is same. Work done by assign it zero value, we obtain
gravitational force does not depend on U =mgh+O
i ' --1-- -- ·~·--·--- ----- --,
path taken, it depends only on initial
and final positions. ! •
1
•Path
.~
• Fin~I. ;
I

(b) We
path A~ B~ A.
consider the closed
.(b)j . followed
by particle
_/ pos1t10n,~
,'

WAB = (µkmgcosa)Lcos180°
= -µkmgLcosa
W 8A = (µkmg cosa)L cos180°
= -µkmgLcosa
w,otal = wAB + WBA = -2µ kmgL cosa
Frictional force is always opposite to displacement, z
therefore it is negative. F,lg. 3.15
---
www.puucho.com
Anurag Mishra Mechanics 1 with www.puucho.com

1210 MECHANICS-I I
We can assign any value to potential energy ofreference Note that potential energy is either equal to negative of
level, e.g., if we choose U; = 100 J, then work done by conservative force or it is equal to work done by
u1 = (mgh + 100) external agent.
Note that CLASSICAL WORK-ENERGY THEOREM
l!.U=Ut -U; Consider a particle moving along a general curved path
= (mgh + 100) - 100 under the influence of an external force F.
=mgh From Newton's second law,
Le., l!.U remains unchanged whatever be our reference dv dv
F,=m-=mv- ... (1)
level. dt d,
+ A particle is moved from initial position to final mv 2
position under the influence of gravitational force. Fn
= R
- ... (2)
I!.-;= (x1 -x;) i+(yj-y;)J+(z1 -z;) t< From eqn. (1), F,ds = mv dv
....
F=-mg j
. i------
1 , ;,• I-axis
.... .... I• -~··'/ Finaf
Wene,gy = F · I!. r 'position ·
= -mg(y f -y;) .k-~-F
Gravitational Potential Energy (GPE),
Ug =-Wgravity=mg(yf -y;)
n-aXis
GPE increases if elevation of body increases, i.e.,
Yt >y;, Fig.3,17
GPE decreases if elevation of body decreases, i.e.,
Yt <Y; On integrating the above eqn. from initial pos1t1on
+ Elastic potential energy of spring: Consider where velocity is v; to final position where velocity is v 1 .
the block in the figure being pulled by an external .!.J 51 F,ds = f-" 1 mvdv = .!.mv 2 -.!:mv 2 ..
agent. The block is being pulled slowly, i.e., the 2 Si Vj 2 1 2 l

block is in equilibrium. · Note that only tangential forces perform work, so that
fs,S1 F,ds = w,otal = I!. KE
+ The above equation is the classical work-energy
.... . theorem, which states that work done by all the
t::~:::::::;J,~.[1'::::F external forces acting on a particle is equal to change in
Fig. 3.16 kinetic energy of the body. If work done on the
...,------------------~
.... system is non-zero, energy is transferred ,to the
['Fspring [=[ Fexternal [ system. IfW10ta1 is positive, the kinetic energy of the
-t ,.-t " · system is increased', If w,otal is negative, the kinetic
where F,pring =-kxi, Fexternal = kxi energy of the system is decreased.
W · =Jx1 (-kxi),dxi=.!.kx 2 -.!.kxJ2
+ Note that CWE theorem is independent of nature of
~- ~ . 2 ' 2 forces acting on the system and the path followed
by the system.
W external = ff (kx i) · dx i + The work done by all the forces can be classified
' into two categories, namely, work done by the
= .!.kx,2 _.!_kx2
2 2 ' conservative forces and work done by other forces.
If we assume initial stretch in spring zero, Le., X; =0 W total = W,ons. + Wother = I!. KE
From definition of potential energy,
w,pnns = -½kx2
W cnns.= -I!. PE
wextema1 = -lkx2 W other = I!. KE - W cons. = I!. KE + tJ. PE
2 .
This is the general form of the work-energy theorem,
Spring force is conservative force, therefore negative of which states that work done by the "other forces" on the
work done by spring is change in potential energy of system system is equal to the sum of change in kinetic energy and
as the ~pring is stretched or compressed. change in po[ential energy of the system.
Elastic Potential Energy (EPE) ,U, = .!. kx 2
2
www.puucho.com
Anurag Mishra Mechanics 1 with www.puucho.com

rwoRK AND ENERGY .271 I


Conservation of Mechanical Energy Concept: 1. Net work done by static friction is always
In an isolated system of objects that interact only zero.
through conservative forces, the total mechanical energy of 2. If block is placed on a conveyor belt that. is a'
a system remains constant. The total mechanical energy of accelerating, there is no displacement of block relative to belt.
the system is defined as the sum of kinetic energy and Work done by static friction on block is positive whereas on '
potential energy. belt it is negative.
Principle of conservation of energy states that
E, =Ei
or K;+U,=K1 +u1 _,
-s
or i\.KE+i\.PE= 0
Le., change in total mechanical energy of the system is
D
~ ~ - • • fstatic

zero. Fig. 3.19

Case II : When a force F ~


Concepts: 1. The total work done by all forces is which is sufficiently large to r-;:::-i F
overcome friction, i.e., F > fmax· 1..., ,___ L:_J __....,
wtotal = wnoncon. + wcons.
Here, the work done by the F > fmax: fmax = µsN

• _energy
.
This total work equals the change in the system's kinetic!
I' friction force is negative because fk = µ,N
force of friction and
Wnoncon. + Wcons. ~ liK:, I Fig. 3.20
displacement are in opposite
The net work done by conservative forces changes the 1'
. direction. If F > µ ,N; friction is kinetic.
system's potential energy ' Case III : In Fig. 3.21 shown when the block A is
pulled with a force F. The friction force and displacement
'Wnoncon. = Af( - Wcons. = M - (-i\. U) = L\. (K + U) are oppositely directed in case of block A while in case of B
2. The net work done by the non-conservative forces in an, they are in the same direction. The friction force does
isolated system equals the change in the system's total· negative work on block A and positive work on block B.
mechanical energy. ·
Wnoncon. = i\. (K + U).
3. The net work done by non-conservative forces during
any process equals the decrease in the system's internal,
_1,
en erg)( U int. ~-s.
Fig. 3.21
Wno neon. =- li Uint·
4. The total energy of an isolated system-equal to the (B) Work Done by a Spring Force
kinetic energy 'of its particles, the potential energy' (i) The work done by the spring force for a
associated with conservative forces acting within the, displacement from X; to x I is given by
system, and internal energy-is conserved. or W = -~k(x12 - x2)
. Ll(K + U + U 10,) = 0. s 2 '
(ii) A spring stretched from its equilibrium position
Some Conceptual Points for Numerical Solving by an external force.
(A) Work Done by Friction Fspring and x are antiparallel, wsping < 0
Case I : Consider a block =
5 0
placed on a fixed surface. When F,
Fexr and-x are parallel, Wexr > 0
a block is pulled by a force F 1
which is insufficient
overcome the friction, i.e.,
to .,•,____ 1
0
1
- - • 1'
l':::!!'.:'•;;;;~,
o
x<O m
x>O F5:pring
-s
L-[J-F,,,
F < f max· Here, the work done 10 = natural length During extension
by the friction force is zero Fig._3.18
s-
because displacement of body is
zero. Fsprin~F.,,..1

. Fig. 3.22

www.puucho.com
Anurag Mishra Mechanics 1 with www.puucho.com

[272 -··-----·--·- -·
(iii) A spring is compressed from its equilibrium And, the work done with respect to ground, i.e., in the
position by an external force. zy frame is
Fspring and s are antiparallel, Wspring < 0 W =F(X1 -X,)
F,xt and s are parallel, W,x, > 0 Coordinates in the two reference frames are' related as
- -·-------· ___ . --· . -
,.,_ - . X1 =x 1 +x' 1 andX1 =x1 +x' 1
! Concept: The work done by spring force depends on the: therefore, W =F[(x1 -x1)+Cxi-x 1 )]
'initial and.final state of spring only. The net work done by the'
or W =W'+F(X1 -X1)
:spring force is zero for any path that returns to the initial
I •• where X 1 - X 1 • is the displacement of the
pos~uon. _ _ . _ _ .
trolley with respect to ground.
(C) Work Depends on the Frame of Reference (D) Work due to Internal Forces (Friction)
Displacement in a given time interval depends on the Although resultant of internal forces for a system is
velocity of the frame of reference used to measure the always zero but network due to internal forces for whole of
displacement, hence the work also depends on the frame of the system may or may not be zero.
reference.
The Fig. 3.23 shows a cart moving with a constant
,---,-•Fextemal -
velocity v O along the positive x-axis. A block is pushed with a m,
constant force Fon cart. The reference frame x y' is attached fstat!c --~~--'-,--,~fstatic [on block 2]
'ran black 1] m,
with the trolley and the frame zy is attached with the
No friction
·Y There is no slipping of block m1 on block m2

Fig. 3.24
y't x' _DF In Fig. 3.24 Fext,rna! acts an block m 1 such that m 1 does
"'-=-~~-:;::=;~;,Ji-
!®!) : (!@ V not slide on m 2 but has tendency to slide.
! i-x•.-:
0

Displacement vector of m1 and m2 will be same and let it


: I :
I ' ' bes.
a~----------.x
l Xj X1
Work due to friction on m1
W1 = -(J,)s
Initial position of block and cart
with respect to ground. Work due to J, on m2
Fig. 3.23 (a) W2 = (J,)s
If m1 and m2 are part of the same system then f, is
ground. The Fig. 3.23 (a) shows the initial position of the internal force and total work performed by this internal
block in a coordinate system attached to cart represented by force;
x, y' coordinates and a coordinate system attached to
ground x,y.
W = W 1 + W2 = -CJ, )s + f, (s)
W=O
And, the Fig. 3.23(b) shows the final positions of the
block. Concept: If relative displacement of one body of system;'
y w.r.t. other body along the direction of internal forces is zero,'
then total work due to internal forces is also zero.
' I
y' Total work· performed by static force of friction for a
tf®!JX'
~1-"o
'system is always zero.
Let us see a case where work done by internal force is
'

?- x'f- x'1 ---! not zero.


' '
I f :
I .~----,~·----cc·-+.x ~
I, 0 X1 x, r---r-_,...,. Fe,temal
Final position of block and F m1 ~
cart with respect to ground. kineUc+--,-~-~-=,,=.. Fkinetic [on block 2]
Ian black 1! m2 ~
Fig. 3.23 (b)

The work done by the constant force with respect to ... ... No friction
block m1 slips on m2, s 1 and 52 displacements
cart, i.e., in the X y' frame is of block m1 and m2 respectively
W'=FC:x:1-X;) Fig, 3.25

www.puucho.com
Anurag Mishra Mechanics 1 with www.puucho.com

If m1 slides on m2 and s1 > s2 then s1 - s 2 is the


displacement of m1 w.r.t. to m2
W1 = work due to Fk on m1 -m,·T.:
-·- · - - . ·----·- -.
pendulum bob in equilibrium position was discussed. Now
we wish to find the maximum deflection 8 of the pendulum
from the vertical.

l ·mq~i. ,./:
W1 = -(Fk)s1
W 2 = work due to Fk on m2
W2 = -(Fk)s2 ·~a
Network performed by Fk (internal forces) for system is:
. mg
W = W1 + W2 = -Fk(s 1 -s 2 )
W = -Fksrel not zero because there is some relative
lL---~ ~,--- _
(a)
Fig, 3,29 .
(b)
---··-··" J
displacement s,.1 a/?ng the direction Cl_f_ i-~~e~al ~°..r':'::. --·- The work done by gravity is Wg = -mgl(l- cos8)
Concept: Thenetwork performed by kinetic friction onj The work done by pseudo force is Fps = mal sin 8
the system is..· always negativ.e. and {t depends on relai:iv.e The work done by tension is Wr = 0, because tension is
displacement betwee1:_the contact sicrf(lces: _________ · perpendicular re-displacement.
At the extreme position the velocity of the bob is zero.
(E) A block of mass m is
projected with an initial velocity v ~ Applying work-energy Theorem, we get
towards a fixed spring of stiffness k Wg + Wps + Wr = MC, at the extreme
attached to the wall as shown in the position block is at rest
Fig. 3.26. -mgl(l- cos8) + ma1sin8+ 0 = 0
The work done by the -.-.---,-------~·cc·-:--
spring force is negative beca~se
the force exerted by the spnng [ .
!F~~,,~~ ·
r::::l_ ·;"::oooo,' ·.
mmlmclm mmt1m~
·~1! or
2
g[2sin ~]~a[2sin~cosU.
8 a
is opposite to the displacement j . regain its natural length _' or tan--= -
'2 g
of the block. [' . Fig. 3.27 . .
Let x 0 be the maximum ·-·-·-·-··-·-···--· 1
or ___ 8=2tan- (:)
compression in the spring, then work done by the spring
W = -~k(x12 - x2)
-- ·- ', '

Concept: In this case pendulum will execute oscillations!


2 '
.1 2 of angular amplitude .
W=--kx 0 ,.
2 8.= tan·' a/g
As; · you have learned· earlier equilibrium;'. is at''
..
From work energy theorem, we get
I,a= tan- a/g that· this artgl~ is double io thaC'at
1 the·
W=MC=Kf -K, ,,, !
-~kx~ = O-~mv5 J'quilibrium. --··-·· • · _ --·--'---···--·-·----···-·- .l
2 2
or x 0 .=v 0 R (G) How to Apply Conservation of Energy Equation?
A block of mass m falls r·- .. [ffi]-- - · · - ~ ·· · --- -··
When the spring gets completely : .-:-- : - :-Motio-n -~:.
compressed, then it begins to original / . •dx ~ : ·
from a height hon a massless 1 _hI . _:
spring of stiffness constant k. 1
length during this phase the spring : F~ Let the maximum 1
force F and the displacement dx of the l\\imm,miu\~ compression in the spring be · · , • l
block are in the same direction. The I • Fig. 3;2s ·· ·
work done by the Spring force is L. - - · . . . x. Weforassign
level the energy
potential reference
at ..c @;,i:l:··,:,~lt::.fi~.firence;
k'' Lev.el ;
. positive. Form work energy theorem we get .• , the position of maximum · . .'.. . · I
W =-mv
1 2
compression, reference level . (a) · ··. (b) ·
0
2 can be assigned arb itrarily, ..,.'
(F) Work Energy Theorem in a Non-inertial according to convenience. i:..-~-----··'r_.:g._~- 3~. . _______ J

Reference frame From work energy theorem, we get


A pendulum of mass m and length 1 is suspended from W,pring + w,ravity = ·O
the ceiling of a cart which has a constant acceleration a in 1 .
the horizontal direction· as shown in the Fig. 3.29. We have
--kx 2 + mg(x+ h) = 0
2
previously solved a problem in which the deflection of or alternatively
www.puucho.com
Anurag Mishra Mechanics 1 with www.puucho.com

, r,

Applying energy conservation theorem, we get = 371 cos0 + 3mg sine- 2mg
K,+U,=K 1 +'U1
O+·mg(h +x) = o+~kx 2
= mgl 3,J2sin(e + ¾)- 2]
2. T is maximum at e = 1t/ 4 ·
x
2
-2(7 )x-2(:g )h = 0 Tmax = mg[3J2-2J
illustration 1.
After solving qua:;ti['c e q ~ ] e get . A plank of mass M and r-~ . M . I
i .... ·
x=k
....· -,
l+v1+mg
. .~
-
~:~thL ~~~~~~!t~~:::c:~ ~ ["':0'."""" ,'"
.small block of mass m is _,__, F,g. 3 -31 {~)
!~~oo!h!
...
I Concept:Eqtiilibriumposition of block is at x0 , ; mog_ projected wii:h a velocityv 0 as shown in the Fig. 3.31 (a); The
' . . . k. coefficient of friction between the block and the plank is µ,
: If block is released slowly it will stay at Xjj. If th~ ·block is plank is very long so that block eventually comes to rest on
1droppedfrom h ;= .Q then the defo'rmation in the ;pring is just it.
;_d.Q'!b,le_th~,stah,;' deforrrmtiQf!._ . , ...ci.i "", ' ·' - · ' · (i) Find the work done by the ftjctioh force on the block
during the period it slides on the plank. Is the Work
krix· a- · ...--.. 'l·e_:r-:;-7,"';:-,,. positive or negative ?' _
~~--- .l'DJ:>0.v-~~~
(ii) Calculate the work done on the plank during the same
-~ pend11l~~ b~b of_;,_;;s ; ~ ;~j,~~1ed at rest. A constanj period. Is the work positive or negative?
=
horizontal force F mg starts acting on it. Find : '' . (iii) Also, determine the net work done by friction. Is it

l
positive or negative ?
! (~) the:m~~um angular deflection of the string.· ·
Solution : PrQblem solving strategy:
: (b) the. max/1:'um p~.1~J~11,jo: in the string: . -
Step 1. Apply Newton's law, determine acceleration of
; - - ' ,l \ . .. blocks.
', . /' : ·, Step 2. Determine instantaneous-velocity of blocks·.
Step 3. When slipping slops blocks have common
• i
· · velocity.
. '
I
The free body diagrams of the block and the plank are
r________ _ , ,I Fig, 3E.4 {a) shown in the Fig. 3.3l(b).
--~----~---·---------~-~--'---' Block :
f- .
a1 = - =µg
Solution, (a) Let at angular deflection e and let velocity m
be v, from work energy theorem change in kinetic energy= Instantaneous velocity, v 1 = v O - µgt
work done by all forces
Plank: . a,-= l_ = µmg
~mv 2 = -mgl(l- case) +Fl sine M M
2 N, .
a1
= mgl [-1 + cose + sine] +- 7 Motion
Maximum angular deflection v = 0 =}' e = 90° f=.µmg .
1 -- •• mg,
. !. ; '.
,,' ' '

Fig. 3.31 {b)


/sin 8
_µmgt
'' .....-·
,)case+ · .... /
'
, F=.mg Instantaneous velocity,
.-. M
V2---

:· - mg Finally, both the block and the plank start moving


Fig, 3EA {b) together, i.e., v1 =v2
(b) Tension at angular deflection e then . Vo -µgt= µmgt
· mv 2 'M
T-mg (cose+sine) = - - t= Mvo
1 or
(M+m)µg
=} T = mg cose+ mg sine+ 2mg(-1 +case+ sine)

www.puucho.com

'I
Anurag Mishra Mechanics 1 with www.puucho.com

. .·. mv 0
and, the fi na1 common ve1oc1ty 1s v = - - . = 5; 4 t(t 2 dt) = 4 J;t 3 dt
.

to its change in kinetic energy; i.e.,


M+m
(i) The work- done by friction on the block is equal = 41 ~12 ~
4
=
0
4
(24 '_ 04) = 16 J
_
W1 =Kt -K, Method II. From work-energy theorem, W = c. KE
1 1
X = t /3
2 2 3
=-mv --mv 0
2 2 dx ;

or w, = .!_
2
m( mvo
m+M
)2 - .!_Mv~
2 · At
Ve1ocity v = -
dt
2
•t = 0, Vi = 0 = 0
=t

2
:t·mM(M + 2m)v~ At ,t=2,vf =2 =4m/s
= 2 (M +m) 2 1 2 2
Work done W = m(vf -v;)
The work done by friction on the block is negative. 2
(ii) The work done by frictio_n on the plank is given =!x2(4 2 -0)=16J
by 2
r --··,..-~l:-,.,,.__
b.fi:;~a~lti~~¾ 6 ~
.. - . ··--- . . --•- - · - - - - -·· • - - - - - 'I
:A force of (3 i-1.sj)N acts on 5 kg'boczy. .The body is at al
:positionof(2 i-3])m~ndis travellingat4 ms·'.. .Th.eforce:
i . , ,. ,.... I
Jacts on the body until it is at .the position ( i + 5 j) m:
;Assuming no other force does work on' ilie bod); -the finali
!,,peed of the body.__________ . ··-·" · ·-· ____ ·---·
The work done by friction on the plank is positive. Solution: Given, mass of the body= 5 kg
(iii) The net work done by friction is Force
__,
F=3i-1.Sj
- -
W=W1 +W2 --, 3• 3•
1 mM 2 a=si-lOj
=----Vo
2M+m Now displacement
The net work done by friction is negative. --+ A A A ,._ ,. A

s = {(i + Sj) - (21- 3j)} m = (-i + Bj) m


r~~Exam;:;;11~-Q~
tfr---- ~- .,"'7'7-1!L.::J~ From work energy principle
r- -- -- -------···--:-:"--=-·. --·--- ----- '1
·- -·-· ··--.. --+--+ 1 2 2
I'
Under the action of force, 2. kg· body moyes such that it.ii
. 3 ' 'i
W=F·S=-m(v -u)
2
lposition x as a function of ti,;,e t is given by x = !..., ·xis in\ => v=Mm/s
; . - 3 I
!metre and tin•second: Calculate the \\'Ork.done by theforce in' k-==xcai~M~~1 ·· .·
•the first 2second. _______' ... · --~ ____ ·__ _- ____ . _· I r-,'"'?"""'-=c-"\£1'6~~.);.'""Sc"c/~

Solution : Method-I: Based on basic expression for ¼spryng block system is placed on a rough horizontaz" surface;
work done iliaving" coefficient of friction· µ. Spring is given initial'
W = Fdx f :elong.ation 3µm.·g. /k (where m = mass of block and k =spring!'
;constant)- and the block is. released from rest. For· the
t3 .
as x=- [subsequent motion find: '
3 I - ----- -- ,.- I

on differentiation, we get .
, 1 3µ
!; ~ m91k
i,o---oi 1
I
\ I ' . j
~ dx ' m- . i
v=-=t - :~1//ll/HIIJJJl,l
dt
dx= t 2 dt L - --- ".'9.·.~~-7 . ·- -
dv · (a) initial acceleration of block
a=-= 2t (b) maximwn compression in spring
dt
Resultant Force F ;... ma= 2(2 t) = 4 t ,___ (c). maxirl_l_'!m spee~ gf th~. ~fock_<· •.
Work done by force W = fi dx
www.puucho.com
Anurag Mishra Mechanics 1 with www.puucho.com

276 MECHANl~S-1 \
Solution : (a) From Newton's second law,. we get-· ' [where h is the distance fallen by block of mass m ]
ma= 3µmg-µmg => a= 2µg ·. ~2gh '
Solving v=--=2m/s
(b) From work-energy principle_Wspring + Wfri,tion = 0 . 7
. ·1 2 2 ' . . ' .,'
-k(x0 -x1 )-µmg(x 1 + x 0 ) = 0
2 ,. . :1. • .
or x 0 - x 1 = 2µmg/k
=> x, =µmg/k 'Ilvo blq_ck$ having md.sses 8 kg arid 16 kg are connecteq tp the
· (c) Speed will be maximum where net force is zero two erids of a light spring. The system is placed on a smooth
µmg= rox horizoht~l floor. An inextensible string aJ,o connects B with
=> x ~ µmg/k (extension) ceiling as J/iown in figure at. the initial moment. Ihitially the
Now from work-energy principle, we have spring hq§,tts natural length. A constant horizontal force Fis
' 2, . .,.
appliedtq,,(he·heaviei- block as_shown. What is the maximum
lk(µmg) (3 2 -1 2 )-2+,tmg (µmg)=1mv 2 possible. vti!ue of F so .the lighter ,block doesn't loose. contact
2 'k k · 2
withgrolf~1- ~---iA
., ,
_Solvj~~ we ·get,
...-ir', .v
.~ 2µ,g ~T
· (rn
1•
4m _A_ I
F

)In the figuf;~JiolJ'n, tM mass of th~ hanging block is m, while Fig. 3E.9 (a)
that of th~ ffi(iitk resting on the floor is 3 m The floor is
horizontal aJ~ffi¢fioriliis and all pulleys ideal, The system is Solution : Draw FBD of B to get extension in spring.
0

initially held,sfafiqnary·, with tl/.e i1tclined thread making qn Instant when block B just looses contact with ground net '
angle a= ~Q'f,·wJQN~e. h,orizont,~L The, blocks, f~e now force on it is zero. , ·
released from rest ·a1frJ, allowed to ,move. The hanging block kx-Tcosa = O·
lfalls through a h¢igljt_'(49/5) m' befor;e hitting thef/.oor. It is ·Tsin9+N-mg = 0
!found that'·the va[ue'..of a becomes'-60°, when the hanging
block hits th'e flciot. :F(nd the speed with which the hanging to riseN = 0
block hits the·jlocir. "; T
N
... ' ·'j--~·~--7 8
'

.. . • mg J-kxl
'fig.3E,9 ( ~
kx . ·a =mg
--sm
', ;. ~ cos9
,_, Flg.3E.8 mg 80 60
'----~~·-~:------.,,.-------'---------' X=--=---
ktan9 kx(4/3) k
Solution: First apply string constraint velocities are
related as v 3mcos9 = 2vm - If spring has to just extend till this value at their
At the moment of strike extension it should be at rest.
Vam = 2vm sec60°= 4Vm Now we apply work energy theorem to get
Let· Vm =v => v 3m = 4v &=1kx2
From work energy theorem Wg,avity = t.KE, we get 2
. 1 2
1 2 F= 30N
=> mgh=-mv +-(3m)(4v)
2 · 2 . .

www.puucho.com

7
Anurag Mishra Mechanics 1 with www.puucho.com

.. 277J
-----··· -- ---------·--

(a) A 2 kg block situated on a smooth fixed incline is A ring of mass m = l kg can slide over a smooth vertical rod, A'
·connected to a spring of negligible mass, with spring constant' light string attached to the ring passing over a smooth fixed
k = 100 Nm-1, via a frictionless pulley. The block is released. pulley at a distance ofL = 0.7 mfrom the rod as shown in Fig.'
from rest when _the spring is unstretched. How far does the· 3E. ll (a). At the other end of the string mass M = 5 kg is
block move down the incline before coming (momentarily) toj :attached, lying over a smooth fixed inclined plane of
rest? What is its acceleration at its lowest point? ; inclination angle 37°. The ring is held in level with the pulley
(b) The experiment is repeated on a rough incline. If the block! and released. Det_ermine the velocity of ring (in m/s) when,the
is observed to move 0.20 m down along the incline before. iti ·string makes an angle (a= 37°) with the horizontal.:
comes to instantaneous rest, calculate the coefficient of kinetic'. [sin 37° = 0.6]
friction.
k = 100 Nm·1
·· ...
37° ..

37°

Fig. 3E.11 (a)

Fig. 3E.10 Solution : Let xis the vertical distance covered by the
ring. Then x = L tan 37° = 0.7 x ~
Solution : (a) At the extreme position blocks stops. 4
Applying work-energy theorem, we get · L
L'.l = Lsec37°-L = L(sec37°-l) -=L'.l=}
4
mgsin37°=_!ks 2
2 M = distance moved by block M
2 X 10 XS X ~ = _! X 100 X s2 L
5 2
on solving s = 0.24m
Acceleration at its lowest point v,
ks- mg sin 37°
a=--~-- Fig. 3E.11 (b)
m
100x0.24-2xlOx~ Now, from constraint relation
=--------"'-5 6m/s 2 4
VM = vr cos37°= -vr ... (1)
2 5
2
a= 6m/s v r = velocity of ring,
(b) = 6.KE
Wg + W friction + Wspring v M = velocity of the block at this instant
From work energy theorem, we get
or mgssin37° + µmg cos37°xs = .!ks 2
2 W g,-a>ity = L'.KE
mg sin37° _ _!ks= µmg cos37° -mgx+Mgt'.lsin37°+.!mv; +.!Mvt = 0 ... (2)
2 2 2
3 1 4 On solving eqns. (1) and (2), we get
2x lOx--- x l00xs = µ x 2x lOx-
5 2 5 v, = 0m/s.
gives s = 0.20 m
12- 50s
µ = 16

µ=s1 '
'From what _minimum height h must the system be released
'when spring is. unstretched so that after perfectlv frtelastic:
,collision (e = 0) with ground, B may be lifted off ,he ground
1_(Spring _constant= _k) . ·
Solution : Just after collision with ground
www.puucho.com
Anurag Mishra Mechanics 1 with www.puucho.com

- - •- •w•-···--• ·--·· ··- ~- •--•• •• 1


,- 278 , MECHANICS-I I
~-------···-·-------------------
1 2
Et =-mv
m A 2
From conservation of energy,
Initial state E, =Et
1 2
mgh = -mv
2
or
~----
v = ~ 2gL(l - cos8)
Cb) Now we apply Newton's second law at the
lowermost point.
(a) (b) mv 2
LFy =T-mg=--
- L
At the moment
of lift off
mv 2
Agaln when
spring Is relaxed v>0 m fv or T=mg+--
L
· c:ptwi --- tx [extension]
X = 2mg/k =mg+
m, 2gL(l- cos8)
L
= mg(3_:_ 2cos8)
~ 2m
~~~~g[~J 14 )J>
(c)
Fig. 3E.12
(d) ; - --
•A boy throws a ball with initial velocity u at an angle of
Applying COE, 1
projectiori e from a tawer of height H_ Neglecting air
·resistance, find ·
_!-_mv 2 + mgx+_!-_kx 2 = _!-_m(2gh) + 0+ 0 (a) hoiv high .above the building the ball rises, and
2 2 2
l(b) its speed just before _it hit§_ the_ground.
1 -····-
=> -mv 2 > 0 => h > 4mg/k

.--f
2

i _-E:>f~-1'!'.Pj~=-~
A pendulum bob of mass m and length L is released from angle:
8 with the vertical_ Find ' Fig, 3E,14
(a) the speed of the bob at the bottom of the swing, and
(b) tension in th_e suing i;,t_that tj17J_e. ___ _ Solution (a) Only gravitational force acts on the
ball, which is conservative; therefore we can apply
Solution: (a) We can apply conservation of energy to
conservation of energy. We assign reference level at the top
bob-Earth system because gravitational force mg is
of the building, i_e,, Ugi = 0. At the topmost point, the ball is
conservative and tension is always perpendicular to velocity,
it does not perform work We choose reference level at the moving horizontally with velocity u cos8.
loWermost point, i.e., Ugi ::; 0. Initial total mechanical energy
-- --. - - -- 2
T
E-
I
= O+_!-_mu
2

L Total mechanical energy at the topmost point


X 1 2 2
·- ... Et = mu cos 8 + mgH
••. m
_.. ~
V
2
From conservation of energy, we have
·····-. -"· ---·R·· •f=ence
•••••••••• -.. ... ......._......... 'k'.T'
·--· level mg E, =Et
(a) (b) _!-_mu 2 = .!_mu 2 cos2 8 + mgH
Fig, 31:.13 2 2
u 2 -u 2 cos 2 8
Initial total mechanical energy, or H=-----
2g
E, = mgh = mg(L-Lcos8)
(b) If vis the speed of the ball at the ground,
Final total mechanical energy,
1 2
Et =-mv -mgH
www.puucho.com 2
Anurag Mishra Mechanics 1 with www.puucho.com

: WORK AND_ENERGY 279


From conservation of energy, we have
E, =Ef
1 2 1 2
-mu =-mv -mgH
2 2
m v=~ru~2-+-~-H-
:A block of mass m hangs on a vertical spring. Initially the
L_:g_~f!.~J:?J~ fwl> spring is unstretched, it is now allowed tc fall from rest. Find
( a) the distance the block falls if the block is released slowly;
(b) the maximum distance the block falls before it begins to
Consider an Atwood machine with both the masses at the
move up,_
,same level as shown in Fig. 3E.15. Use the principle of
lconservation of energy to find · Solution : (a) When the block falls slowly, it comes to
(a) speed of either of the masses as a function of its position rest at a distance y O, which is referred to as the equilibrium
and position. From, condition of equilibrium,
(b) the acceleration of either of the masses. LFY = ky O - mg = 0
.------
. ··..
--- .:' -------,·.. Upper extreme
• -·.
Yo ' (amplitude)
'------
,· -------.
A
Reference ____!__ _____ _:_ . Equilibrium
l~Yf' : :
'. ••••••• : A
position

___________ Reference
Xo
.L mg ...--------.
'
Equilibrium --- :------. Lower extreme
level • position ' ..--- ·---·.
'

y Fig. 3E.16
Fig. 3E.15 mg
or Yo=,:
Solution : (a) We choose reference level at the initial
position of masses, (b) When the block is released suddenly, it
E,=U,+KE,=0+0=0 oscillates about the equilibrium position. Initially the speed
1 2
of the block increases then reaches maximum value and
E1 = m1gy + m 2g(-y) +-(m1 + m 2 )v then decreases to zero at the lowest position. In this
2
situation the block oscillates about the equilibrium position.
Mass m1 moves above the reference level, so its The block is released from rest, therefore its total
potential energy is positive. Mass m2 moves below the mechanical energy initially,
reference level, so its potential energy is negative.
E,=Ug +U, +KE
From conservation of energy,
= O+ O+ 0
E, =Ef Final total mechanical energy,
= m1gy + m2g(-y) + ~(m1 + m 2)v 2 1 2
E1=-mgym+-ky +0
2
2
On solving for v, we obtain
From conservation of energy, E1 = E f
1 2
V - ( :: : :: J2gy 0 = -mgy m +-ky
2
2mg
(b) Since acceleration of Atwood machine is or Ym =-k-
constant, we can use the kinematic equation
v2 = 2ay At a general pointy, the total mechanical energy is
1 2 1 2
or v2 = 2ay = (m2 - m1 J2gy =-mv -mgy+-ky
2 2
m1 + mz

www.puucho.com
Anurag Mishra Mechanics 1 with www.puucho.com

.~ ··, · · . M~CHAm.~i!_l

~~atneJJ?.J 1a ~
--~---- -------------- ----
'.In Fig. 3E.17, the· mass m 2 te.sts on a.rough table. The mass
:ml is pushed. against the spring to 'which it is not attached.
!Force constant of the spring is k, coefficient of friction is J!k·
1
(a) Find the speed of the blocks after the spring is released
and m 2 ·has fallen a distance of,h.
'(b) If the spririg is attached to the block and it falls a distance
j h before coming to re.st, calculate• the coefficient offriction
µk.
--( · Solution: (I) For individual bodies :
! l[§J
!-·-"-- · ~~.
i WA= 5x2
;5 - '. !
.•
·,' l ~1s
-·. ----·-- .,1
,
• WB = 1Sx2+(-S)x2
, · 'i Fig. 3E.17
1:W = 30 = Af<Esys = z1 X 10v 2 1
+ x
2 Sv
2

L ---·- -..------c--------'
--- ___ _.h,,==------=--~ .....
v=2
Solution : (a) From work-energy theorem, the
(II) We know work done by static friction will be zero'
energy dissipated by friction equals the change in
because action-reaction will be in opposite direction but
mechanical energy. We consider the table, blocks and spring
displacement of contact point will be same. Thus
as a system; then w ext = 0. We assume initial potential -t -t -t -t -t -t -t -t
energy of the system to be zero. fA·dsA+fB·dsB =ObecausedsA =dsB butfA =-fB
Initial mechanic_al energy E. = !2 kx 2 Thus, 15 x 2 = (1/2) x 10v 2 + (1/2) x sv 2 .
l
V=2
When each block has moved through a distance h, the
final mechanical energy
~~a:.me.!~~:~
1
Ef =.kE1 +Ug1 = (m1 +m 2 )v -m 2 gh
2

2 iPind vefoci~·;f A and B when A---.-~--a-_b-~-u-~_to_t_o_u-ch-th--~-gr-ou_n_·d~, I


Energy dissipated by friction=µ km 1gh. IA/so verify that work done by tension 011 the whole system and ·
From work-energy theorem, l>W = M !N between A q1]d_B /§_~r_o~_..
i · . . ·
. .
, µ km1gh = 1 - [l2
2
kx2
(m1 .
+ m 2 )v 2 - m 2gh ]
/':I--;,-,
I • •

Now on solving the above equation, we obtain ,!'


·• A i
g=10ms-2

v=
kx 2
- 2m 2 gh - 2µ km 1gh
. : .! ~1-------~
2 37°
m1 ~ m2
: ·;
(b) When the blocks comes to rest, the final kinetic 1· , J . Fig. 3E.19 (a)
energy of the system is zero. L ___11J~_;"j_kg,_rijB.5._l_O/cg_,_.- - - - - ~ " -
Initial total mechanical energy, E;=O ,.., ...,
Final total mechanical energy, E1 =- 1kh2 -m gh 2
Solution: /v /=/u I
2 Net speed of block
From work-energy theorem, t, W = M
2
= o-[½kh 2 -m 2gh]
2 2
vB = ~u +u -2u cos37°
µkm 1gh

m 2g - -kh
1 = ~2u
2
- iu 2
~ = v,Jf
m µk= 2
m1g

www.puucho.com
Anurag Mishra Mechanics 1 with www.puucho.com

1
I
woiiiANii ENERGY- ·
--------- - -------
_ '_'.j1
_ _ _ _ _ _ _ _ _ _ _ _ _ _ _ _ _ _ 281

Fig. 3.32 (c)


Fig. JE.19 (b)
·= -Nsin8x + Nxsin0cos8 - Nxsin0cos8
By energy conservation, = -Nsin0x
Decreasing in P.E. of block= Increasing in K.E. of wedge ·Net work done by normal reaction
+ Block ·Nsin0x - Nsinf!x = 0 __ _.. __ _ J
1 2 1 2
mgh=-mv +-mvn
2 2
1 2 1 2 2
mgh=-mv +-mv -
2 2 5
5 x lOx 2 = .!.10v 2 +.!. X 5 x ~v 2 µ= 0.1
2 2 5
12 , A
SxlOX2=-v- •u=O B
2
µ=O
v=/¥ ,Tsin a Smooth I
I
Velocity of wedge= 5~ mis
X
Fig. JE.19 (c)
2m II
Veloci~ of block ' . .I

=vl =v¥xi=Fl =2/¾m/s


Fig. JE.20 (a)
Concept: Work done by tension: If.ind velocity of A, B_and C wl,en_ C has_d~q;_nded 2 f7!.
(1) On wedge Tsin 8
Solution: Here work is done by kinetic friction
W = (I'-TcosS)x between A and B so it will not cancel out. But by tension on

-~T:
'(2) On the block A and C will cancel out.
'rcosS(x- xcosS)- TsinS xsin0
: =TXcosS-Tx X

x sine
Net '1 WA =Tx2-lx2
3.32 (a)
W = Tx- 1xcos0 + Txcose -Tx = 0
Fig. J_!:_.20J~)
By normal reaction between
AandB N
'(1) On the wedge We =l00x2-Tx2
, NsinS-x Total work = 100 x 2 - 1 x 2
,(2) On the block N
99x 2 = .!. x 10v 2 +.!.x 1 xv 2
X N cos8 2 2 100
Fig. 3.32 (b)
- -·· ! , Fig. JE.20 _(c):
:-N~in0(x_~_co~8)_+ (-Ncos_SxsinSJ. .. I
99 2 2
v2 x x Jv=6m/sJ AandC
11
Concept: Thus except ten.sion, normal and static:
friction even if we write work because of action and reaction\
1_,m_ a _syste"! it is_ not necessary that total work will be_zero. '

www.puucho.com
Anurag Mishra Mechanics 1 with www.puucho.com

I 282 ____ _ ___ra_EC_HA_NICS·!J


Finding displacement of B 1N Solution: f = 3Mg
a8 = 0.5 ms-2 , u = 0, t 4
B
From A and C, t = 2/3 µMg 3Mg
Fig. 3E.20 (d) --=-- => µ = 3
1 1 4 1 4 4
S=-X-X-=-ffi
2 2 9 9 Work done by friction force when chain completely slip
_,_, 1 1 2
off the table.
F-s=1X-=-x2xv
9 2 df =µdMg
1 -1 114 M
or v8 =- ms
dW = dfx= µ-dxg f
3
0 l
You can see that work done by kinetic friction on A and
B is not cancelling out completely. WI =3Mg(x2)114 = 3Mg!
Concept: Work energy theorem is valid only from' l 2 0 32
inertial frame of reference and we must try to stick to inertial; Now decreasing in PE
frame while using it. = increasing in KE
:But if we observe from non-inertial frame the
write work done by pseudo forcefor dist. s (a) m
~ta', 1 2
PE,-PEJ =
We should · , 2 mv +Wf

(-9Mg1)-(-
'From ground frame
T-mg=ma Mgl) = I_Mv 2 + 3Mgl
T;
~' 32 2 2 32
T=m(g+a)
W = [m(g + a)-mg] = mas (b)af 7Mgl = I_Mv 2 + 3Mgl
' 1 2 32 2 32
mas=·- x mv
2 mgl I_Mv2 = 4Mgl
From frame oflift 2 32
· T = m(g +a)
Total work= 0
(c) j V = I.-Jg[
2
'Total change irt KE = 0 ma As we have learnt from previous problem if some forces
mg I
Fig; 3.33 J are acting on a body
W1 +W2 + .... +Wn =KE1 -KE,
If some of them are conservative and others are
non-conservative, then for conservative forces we can write
If chain starts slipping find
its KE when chain P.E.
LW, + LWn, = KE f - KE;

. -·
becomes completely straight.
L{-(Uf -U;)}+LWn, =KE1 -KE;
, Fig. 3E.21 (a)
LWn, = KE! -KE, +L(Uf -U;)
Solution: w. = (KE 1 - KE,)
';A Term on RHS is often called mechanical energy.
But w.=-(U1 -U;)
-Uf +U; = KE 1 -KE,
KE 1 +u1 = KE,+U;
Find U by using calculus emphasise that 1
if we have tried to find work due to gravity Fig. 3E.21 (bl_
directly, then it would have been very
1, [gi~m;p}grn1~
"" --,:::;~.c.~ --·~'·--,

Blocks are at rest and in equilibrium.


~

'.Find how much m will rise if 4 m falls awaY:

difficult as compared to the solution we are giving.

L~-a.9.mel~ ~;>
1Chain is on the verge of slipping, find the Solution: Applying WET on block of mass m
'velocity of the chain, when it has slipped. ~g +W,p =Ki -K,
Let finally displacement of block from equilibrium is x.
Fig. 3E.22 (a) -mg(Smg +x)+I.k(25m2g2)_I.kx2 =0
k 2 k2 2
www.puucho.com
Anurag Mishra Mechanics 1 with www.puucho.com

I-WORK AND ENERGY 2831


2 2
lk X 2 +mgx----=
-
15m g
0 in the direction of normal. For O < 0 < ~ N will never be zero
2 2k 2
2
3mg as both mg cos0 and mv are positive. Hence it will be
x=--
k R
. 1acement from IIllll
. . :al 1s 5mg 3mg = -
. --+-- 8mg contact and will have circular motion.
D!Sp - Using work energy
k k k
mv 2 mu 2
~~q~el¢-:~ -mg[R(l-cos0)] = - - -
2 2
r---- - -.---- ~-- - 2
:Find velocit;y of ri11g .wh~I)_ §PJ:i_ng becomes..J!orizonta/. v
2
= ~( m~ - mgR(l- cos0) J
v2 = u 2 - 2gR(l- cos0)
v2 = u 2 -2gR+2gRcos0
2
m(u - 2gR + 2gR cos0)
N = mg cos 0 + ~ --~R-~--
l0 =4m
mu2
- ----
Solution: m = 10 kg,
-- -----
Fig. 3E.24 (a) = mg cos0+---2mg + 2mg cos0
R
N = m[u
R
2
.
-2gR +3gRcos0]
k= 400N/m
Natural length of spring= 4 m 0<0<~
Decreasing in PE = Increasing 2
in KE Normal will not become zero.
1 1 2 If we want to find minimum C
-kxl+mgh =~mv
2 2 value to reach B there is no need to y;::;Q: .
.!x400xl 2 +10xl0x3=.!x10v 2
2 2
·
see the equation of normal all that
matters is speed. B i
200 + 300 = 5v 2
!ii) ;
At0=~
5v 2 = 500 2
2
0 = u -2gR+ 2gR(O)
A
Fig. 3.34 (~-- _!
I
V = ,/100 = 10 m/S
VERTICAL CIRCULAR MOTION u = .J2gR
Consider a block projected on Case I: u = .J2gR it will just reach B.
C
inside of a vertical circular track. Motion: A~ B~ A~D~ A ~B
What is the minimum speed to reach At B, N = 0 but it will not loose. constant.
BandC. · Case II: u < .J2gR
1 ' 2 The body will not reach B but its velocity will become
(B)-mu = mg(R)
2 . zero before B.
A
u = .J2gR L__ Fig. a.aa <at_ e.g., Letu = .,/iii
Solve for (C) like this 0 = gR-2gR+ 2gRcos0
1 1
(C)-mu 2 = mg (R) cos0 = -
2 2
u = .J4gR. This is wrong. Why? :. At 60° the body will stop. The body will not remain
At any 0 with yertical. stationary as its tangential acceleration will not be zero.
· mv 2 what if0 > ~
N = mg cos0 = - -
R 2
mu 2 · Here the normal will become zero before velocity. TWs is
N = mg cos0 = - - why .J4gR was wrong as we were considering speed and not
R
normal where as to reach C it is necessary that 'N' does not
This equation is valid through out for become zero.
0 >~as cos0will go.negative and component ofmg will act Find minimum speed to reach C.
2 .
www.puucho.com
Anurag Mishra Mechanics 1 with www.puucho.com

284 MECHANICS-I I
= It]

fj :'~""'
Q=u 2 -SgR Q v=O;T=-ve
[0 a·
u = ,JsgR
,.case III: u = ,JsgR ·
2
v ·= SgR-2gR-2gR=gR
Minimum possible val;e of 'N' and 'v' is at 'C'.
A
V = ..Jiii Fig. 3.37 (b)' ,
2
, mv . alid .
As mg=--.ISV A~ P~Q~P~ A~ P'~ Q'
R Case W: u = ,J4gR
So the body will continue moving m
v = 0, T ~ -ve .
circular motion.
The body will stop· at the top.
u = ,JSgR implies the body has just
Case V: u > ,J4gR ·Forever will do circular motion.
completed circular motion. £!~· 3;34'(bl
,..------~----'===:::; Concept~:.·.
· Note: We check for' 1t' as cone has maximum negative
value. If N is not O at this point thenfor all 0 < 1t the normal Case J: !]vb < ,J2gr, th~n the velocity vanishes before
will never be zero. tension T, then the particle will oscillate belo,w the horizontal
,___ ----------------'
diameter without 'leaving the circular path, but the particle
~ase W: u > ,JsgR will not rise upto the horizontal level of 0. '
The body will freely move in a circle and 'N' will never
Case II : If vb = ,J2gr , t~e velocity v and tension ·T
be zero.
C~se V: ,JsgR u > ,J2gR.> vanish together,'then the particle will rise upto the horizontal
level of fixed .point O and_ will os~illate along semicircle..
·Th~ norm at will become zero some
Case III ;,.[(vb > ,Jsgr I then tension as well as velocity
where between B and C. At this point [ ~ c __ . ,_,,
•.'. ·,;'.,i does not vanish ;,ven at the hig/ie$t point and" the pa,;ticle
v ¢ 0. It will leave circular motion and --······:·-.-~- <'. ,_' completes ci~cl~ successfully. · '
will become projectile because
symmetry will no more be there as in ..N=O • ,,' . ' Case W.: tfvb = ,Jsgr, then velocity will not vanish at
B' the highestpoini where as the tension will just become zero in
the next instant velocity will decrease Fig, 3.35
further for which N should be negative this case the string will not slack due to velocity particle moves
which is not possible and so it will leave orward and due to string constraint circle · just gets
circular motion and will have projectile completed. ·
o. ·6
motion. Case V : ,J 2gr < vb < ,J Sgr; then tension vanishes at
For a mass tied by a, string about 0. so7:1e point; i.. e.,. the strilJ/r b,ecom~s slackened but thf ve!oc.ity
Here instead· of normal 'Tension' is the being not zero,,, . ., .· . ., . ,_
l'--:'.-::---------;::::=::;:- . --
worrying factor.
,r·
[
.
..•
.. Where v, = ,Jrg cos~
T = 0 ~ String is slack and ' m' will Velocity of projection at the bottom, is given by·
_Fig. 3.36
leave circular motion.
vb= ,Jgr(2+ 3cos~)
All previous cases are valid similarly.
B
Consider a pendulum bob
connected with a rod. Rod (Rigid)
Motion on the Outer Surface of a Fixed Smooth
Case I: u < ,J2gR - Pendulum
Sphere
Case II: u = ,J2gR will reach B and m
come back. A small particle is released
Fig. 3.37 (a) ,
-~ an outer surface of sphere
outer surface of a smooth
Case m: ,J4gR > u > ,J2gR. The body will continue sphere, starting from rest at
moving in circular motion as tension of a rod can go the highest point.
negative which is allowed as then the rod instead of pulling
Which force makes particle
the body will push it. ·
move along circle ? What is
role of normal equation ? \____
· _!:!g. 3.3B
Resultant force towards ·
centre is
www.puucho.com
Anurag Mishra Mechanics 1 with www.puucho.com

------ -------------
2
mgcosP-N
mv
=- - Case III: If $g < vb < -Jsri, the particle wil_l leavej
r ,contact with the outer side of tube at P but, it is constrained to -
2
mv lmove within the tube, hence will change side of the tube ani
N = mg cosP---
r 'will start mo~ing on the inner side within the tube as shown,
To avoid loss of contact; N 2' 0 :in the above figure. For constrained motion inside tube thei
mv 2 'minimum value of v_b for_ complete circular motion is fiir- i
mgcosP--- 2' 0 A:
r
v 5a .,f'rg_c_o_s~p
or v critical = v max :::; ,/rrg-c-os-p~
If the velocity of the body becomes critical at an angle p,
then
from work energy theorem, we get
= dKE
Wgravity
1 2 1 2
mgh = -mv --mu
2 2 I Fig. 3.40
I -
2 2
or v = u• + 2gh
where u = 0, v =v, = .,frgcosP [!g~~~01?~~:[251>
h = r(l - cos Pl :A block of mass _m starts from rest-with spring urtstretched on,
v; = rg cosp = 2gr (1- cosp) :a rough incline_ Force constant of spring, k = 8 N /m,'
2 !coefficient of kinetic friction,µ k -= ½- What is the speed pf the,
cosp = -
3
'[blo,/< when i~h_a§_s_lid a tijstsmce x_ = 0.5 m down the incline? '
R
t-' = cos -1 -2
3 Solution : From CWE theorem, dW = !ill
Vertical distance of this point where the particle leaves Work done by gravity, Wg = mgxsine
contact with the circle; Work done by frictional force,
h = r(l - cos Pl w1 = -µkmg cosex
(cosp = i)
Concepts:
Motion of a particle inside a circular Tube: 1
In this case body will s_tart moving from the lowest point'
A on the outer side within the tube with velocity vb- ' [

i
B

Fig. 3E.25
D c- Work done by spring force,
W
s
= -~kx
2
2

dW = mgxsin9- µkmg cos9x-~kx 2


2
Fig. 3_39 1
/ill= -mv 2 -0
Case I : If vb < J2ri,
the particle will oscillate about-A)
1 2 . '
2
1 2
within the tube on the outer side. [ or -mv =mgxsme-µkmgcosex--kx -_,(l)
2 2
Case II : If vb = ,/2ii, the particle will oscillate in the, Note that we have not counted elastic potential energy
:
,semicircle O\Ll on the outer_s(de_11lithin t/te_.tul,_e,_ _ iof spring in /ill. Instead, we have counted work done by
spring in dW. We can write the above equation in another
www.puucho.com
way.
Anurag Mishra Mechanics 1 with www.puucho.com

MECHANIC£!]
AE = liUg + iiU, + t.KE
IiW = W friction
or Total frictional work done as the chain completely slips
µmg cosex = (-mgxsin0- 0) +(½kx O) +(½ mv O)
2
-
2
-
off the table
m Jl-nl dx
=-µTg O X
... (2)

We have assigned initial position of block as reference =--(l-n)nmgl
level. 2
Mathematically eqns. (1) and (2) are same. Note that different elemei,ts on chain move different
distances on travel, that is why we have calculated work
On inserting numerical values in eqn. (1) or (2), we
done on a small element and then integrated it for the entire
obtain v = 2m/s.
chain.
l,:axam,,..-f~
~=--~S~~~
.........,~-
!A~hain~j;;;;;;··;,,-;;-~;il;ngt_h_l_l_ies-on~·; rough table. -The] IA unifo~ chai~-~flengthl and;;~-;,, ;kep-;o~-~-s;,,ooth;
;chain just starts to slip when the overhanging part equals n th
:table. It is released from rest when the_overhanging part was/
:fraciioi, of the chain length. If the chain is slightly distributed
jn th fraction of total length. Find the kinetic energy of the 1
iso, that it completely' slips off the table, what is t_he work
'.pe,formed by the friction forces. · ' chain as it complete /y_~li12.s..off_tliLtabk ....

17'"'.'.~--
1
I , ---· ---- -·------------- -·-;:-··1 · , Reference '.

! \ I'
!'
1, i
(1-nl)
' ' I

X
lI !I !
dx
L _________ 1 _:=:: ~;:'. J_E:2~ (~}_,_.:::~
--- ------- -I
m Solution: We assign reference level in the table; thus
T(l-nl)g
the potential energy of part of the chain on the table is zero.
As the chain slips more and more, the length of the chain
.' - Fig. JE.26
·-----,,-.--·--~-------------~-
.
goes below reference level, thereby decreasing potential
Solution: We will calculate coefficient of friction first. energy of the chain. This loss in potential energy is
Initially the chain is in impending state of motion. From converted to gain of kinetic energy of chain. . .
conditions of equilibrium: Method 1: Consider a small differential element dx at
Equation for part on table : a distance x from the table.
:r.Fx =T-µN = 0 Potential energy of this differential element
m m
:r.Fy = N - (1- nl)g =-Tdxxg
1
m Total potential energy of the hanging part of the chain
or T = µ 1 (l - nl)g ... (1) nlm ., .

Equation for hanging part : J


=- o Tgxdx

m = -~mgn 2 l
:r.Fy = T nlg = 0-'z ... (2) .
2
· m m When the chain has completely slipped off the table, its
From eqns. (1) and (2), µ - (I - nl)g = - nlg
l l potential energy ·
n I m
or µ=-
1-n J
=- ozgxdx

Now we consider a differential element dx at a distance =-~mg!


x from 0. Frictional force on this .differential element 2 .

7
= µ dxg. Work done by frictional force as it slips distance a ·Loss in the potential energy= -~mgn 2 1-(-~mg1)
2 2 -
X

www.puucho.com
Anurag Mishra Mechanics 1 with www.puucho.com

rwoaiiAND ENERGY 287

= .!.mgl[l- n 2 ]
2
= gain in kinetic energy
Method 2: Consider a small differential element on A
the edge of the table. When it falls through a distance x,
work done by gravity while the chain slips completely
- rl-nl m dx
- Jo Tgx Fig. 3E.2B (a)

=_!_mg (l- nl)2 Solution: Since friction is absent, we can apply the
2 l
law of conservation of energy. Centre of gravity of a
=.!.mgl(l-n 2 ) semicircular arc is at a distance (21t/r) from centre.
2
According to CWE, Initial potential energy = (11.irr) g ( ~)

AW= AKE=_!_ mgl(l- n 2 ) Final potential energy = (11.1tr) g (-;r)


2
Method 3: Potential energy of a body of finite size is When the chain is completely slipped off the tube, all
calculated from the height of centre of gravity of the body. tlie links of the chain have the same velocity v.

r
I
I
I
rrr/2
I
I
I .G.
I
I
Fig. 3E.27 (b)

For the sake of convenience, we assign reference level


i
~nlg
on table, therefore potential energy of this part is zero. Fig. 3E.28 {b)
Centre of gravity of hanging part is at a distance nl from the
2 Kinetic energy of chain = -1 (11.irr) v 2
2
table. Centre of gravity of uniform body is at its centre.
From COE,
When the chain completely slips off the table the centre of
gravity is at 1/2. 11.irrg (~it) = (11.irr) g (-;r) + ½(11.irr) v 2

Initial potential energy= ( 7 )g (-;l)


nl
._____,__, '---,,-'
From which we find
m h

Final potential energy= mg(-½)

Loss in potential energy= ( 7 )gnl (-;I )-(-mg½)


L-J;:°~A~PJ~ : _2~L-->
A chain of length l < rtR/2 is placed on a smooth
= .!. mgl(l- n 2
) hemispherical surface of radius R with one of its ends fixed at
2 the top of the sphere.
= gain in kinetic energy ( a) Find the gravitational potential energy of the chain.
. - .. -·. . r;;i Consider reference level at the base of hemisphere .
L..:~-'59,'}}P_~(?u 2s ;> (b) If the chain slides down the sphere,find the kinetic energy
of the chain when it has slipped through an angle e.
A heavy, flexible, unifonn chain of length irr and mass 11.1tr lies (c) What will be the tangential acceleration of the chain
in a smooth semicircular tube AB of radius r. Assuming a when it starts sliding down.
slight disturbance to start the chain in motion, find the
Solution: (a) We consider a differential element dl of
velocity v with which it will emerge from the end B of the tube.
a chain at an angle 0 with the vertical, that subtends
differential angle d0 at the centre and its mass is
www.puucho.com
Anurag Mishra Mechanics 1 with www.puucho.com

dm = m(Rd0)
l
Potential energy of differential element is
PE= ( 7 R de)gR cos0
\,,i pendul~iii bob is. suspend~d~n;~ flat. car that ~~~ei: i!;ith
Potent.ial energy of chain = J 1/R ( m R
' ' 0 l
da) gR cos0 /velocity v0.'111e'jlat car-is stopp?d.by a bump~r:. · ·~ ·: •
;ca) What is the angle through,\Yfi~h the pendulum swings.
(Note that the chain subtends an angle Z/R at the centre. (b) If the. sw/ng angle is 0 =·60° /llld l = 5 m, what \Yas the
of the chain.)
r.,
·!. ·. ·. "·.
'·--·;'
<
· \ initial speed of the flat'car?",.'' : .
. , ~ _ ; _ , ·- . a
' · 's,f ' · .
!~41
'
,::·r:·.·.:: .i
d ..... ._ I
f ti "' 1;~
t, ~
+-~
Vo .
t~
Ji,& ·A>%!-,,.·,',.. -.·-·---_··
.

level
•• •••• .-··••• ·
R~f~;~~~~~~~~ 1
A ------

L_-~:__,_,
2•

Fig. ~ ~ ~ - - - ( - b ) - - - ~

Solution : When the flat car collides with the bumper,


due to inertia of motion. the bob swings forward. No work is
. done by tension of string on the bob, therefore energy is
(b) Final potential energy when the chain has conserved. ·
slipped throujlh angle 0 is
KEA+ PEA= KEB + PEB
r1 -~i;c::::;:~7
.!.mv~ + 0 = O+ mg (l-lcos0)
I 2
v~
l\1..---~~-""'-~--l
or = 2gl(l -
= 4gl sin 0/2 2
cos0)
... (1)

I Fig: 3E:2f(b)

(m)
or 0 = 2sin-'( z#) ... (2)

J
Uf = s+1/R
0 1 Rd0gRcos0 On substituting numerical values
2
e = 30°' ! = 10 m, g = 10 m/s 2,
= m~R [sin(a+¾)-sine]

From conversation of energy, Ui = U f + KE we obtain


'
KE=U,-Uf V = z.Jg[ sin!!.
2 2
= ~R [sinCD+sin0-sin(0+¾)]

·, (c) Tangential force on differential element dm,


dF, = dmg sin 0
Resultant tangential force on chain IA 'p~ndul~inb~b-can swing alJnga circular'p~th i~1ismopth
inclined plane, as shown . in Fig. · ;3E.3il, 'iwhere
= JdF, =Jt~(7Rd0)gsin0 m = t.2/<g; l = 0.75m, 0 = 37°: At the lowest poifwo,t. the
circle the,t'e'nsion in the sting• is.T .= 11 ON.· Determine: ·
= -mgR [cos0] gR '(a) the speed af:the bob dt the)i:Mestpoint, 1 >· . · ·
l Vi) thesp~ed of the. bob at theflighes,t point on the.'cifcle'. and
= 'f(:-cos (¾)]
1cc) Jhe tension in th?: strimcqt1thd{ighest positiim,.:.'~2 .J

www.puucho.com
Anurag Mishra Mechanics 1 with www.puucho.com

(!ORK AND ENERGY 2897


1- - - - - -- ---- - - - -- --- I

---- -- - - -- ------------
1:
Ii
·- ·- - - ------·,

,--- --- ---- ---------_--- -·- --- -- - -------/


'I a, ·A small toy car of mass m slides with negligible friction· on a1
ii (. :bi-:\
,' I
;"loop" the loop track as shown in Fig. 3E.32. The toy car starts 1

Ii ·· .. .,; +o Ip-om restat.apoint H:above the level of the lowest point of the,
'I
l:'---=-
1 !
8

(a) (bl
lI
,
I
It.rack :
(a) If H = 2R, what normal force is exerted by the track on:
i the toy car at point q?. What are the speed and normal;
''
1·- Flg.3E.31. __ --- -- - - - - i[ i force at point r ?
-- ---------- ---·-- - -- -------· '(b) At what hright will the ball leave the track and to what·
Solution : (a) From Newton's second law, at the I maximum height will it rise afterwards?
lowest point, we obtain (c) If H = 4R, what is the speed and normal reaction at point;
2
i s? -
TA - mg sine= m~o ... (1)

2 TAI .
Iii --------- --- -- - --- -- ---- -
p
or v0 =--glsme I'
m '
j j
' _......,,
N

= (ll0)(0. 75 ) -(9.8)(0.75)(sin37°) i I H , ,

or
= 64.34
v 0 = 8.02 m/s
(1.2)
:ii j '
.L ........... ;~;· Reference level(b)
' "" :,
mg : '.
(b) At the highest point,
mv 2
1
TB +mgsme=--... (2)
.
u__________ ~__!i~~!:~~ - . ______;....,...,...,_...__,,,.___.}.__:
From energy conservation between position A and Solution : (a) From Newton's second law, at point q,
position B,
---·--- ,7 mv 2
N-mg=-
R
... (1)

i'1 I Note that velocity at point q is not known, therefore we


C--• apply conservation of energy.
KE,+U,=KE 1 +Ut
O+ mg(2R) = I.mv 2 + 0
2
' 'H
. "!9.' or v=..}4gR ... (2)
0
_ Fig. 3E'.3_1 (c)
. --.-- -- -- - ---- On substituting expression for velocity v in eqn. (1), we
obtain
KEA +UA = KEB +UB
1 2 1 2 • N = mg+4mg = 5mg
-mv 0 + 0 = -mv + 2mglsme
2 2 Similarly; we can obtain velocity and normal reaction at
or 2
v = v~ - 4gl sine point r.
From Newton's second law at point r,
= (64.34)- 4x (9.8)(0.75)(sin37°) 2
=46.7 N = mu ... (3)
or v = 6.83 m/s
·R
(c) From eqn. (2), From conservation of energy;
mv 2 KE,+U,=KEt +Ut
TB= ---mg sine
l 0 + mg(2R) = I_ mv 2 + Pg(R)
2
(1.2)(6.83) 2
= 0.75 (l.2)(9.8)(sin 37°) = ..j2gR
V ... ( 4)
From eqns. (3) and (4), we obtain
= 67.56N
N = mv2 m(2gR) = 2mg.
R R
www.puucho.com
Anurag Mishra Mechanics 1 with www.puucho.com
'(,
..•,-,..

(b) The toy car will lose coiltact with the track at
the point where normal reaction vanishes. Let contqct
breaks at an angle.a. with the vertical.
·r-- . ----- ----··-

. -_.. .. ·..f;,·· ..
... ,: ....... t, ... Jt ._ Reference:
· ieve1
I

\·:r_: __ .. ,._, _:_:-... _


! O 'Re"ferenceO level ''.,'.:\~,! ·r· '• M
J
··:-~ ::l
:e / 1,,. ~lg. 3E,32 1(J)',:,. -.. ·;, , ·
,.
r;.· L,-----L--- ····.1.~,·-•._: .!_,.NJ
From conservation of energy,
,'
Fig. 3E.~2 (c) KE;+U;=KE1 +Ut
i_ ___ . - ,. _ _ _ - - • -M es
1 2
0+4mgR-= mv 1 +2mgR
From Newton's second law, 2
1- • •
mgcos0-N = - -
mv 2 or vJ = 4gR , .. (9)
R
When contact breaks, N 0 = On substituting expression for vJ in eqn: (8), we o~tain
Thus, v 2 = gR cose, ... (5) mv 2
1 -mg= 3mg.
N =--
From conservation of energy, R
'KE; +U; = KEr+Ut
1
O+ 2mgR = -mv 2 + mg[R+Rcos0]
20
2 iA srrtaU~°.~! 1is·J-~lle<i'wit~; s~ee~'!:u'fro,m; point,\J:,~1~~-~)
or. v = 2gR(l - cos0). ... (6) ' fsmooth ·e,rcu/ar track as slzo_Wl!•clt): Fig, 3E.33.:Jf·;i,;;;!;}!<,I
On equ\lting expressions for v 2
, we obtain determ_ine.: tlfe. r~qufed speed -~. ~o'J:lzat_ t~e ball i-et:itnJ Iii A,f
. -. 2 2 2 the poznt-bfp_r0Ject1.0n. ,W7\at ~;t1!e 1f/lnl(11U':1 Vallfe,oli.:rto:1
cos0=- v =-gR ~h,ch the·?fl/Jcan reach t~J_~t7!t,~pro1ect1on ?, :. ; '':; I
0

3' 3
• J C · :C-( :~ j
After breaking contact with track the toy car moves ou a I ---'-----:-:"Iii..: "I ,, :."

parabolic trajectory as·a projectile. · 'j ·~j'l,:_,,.'



·Now we apply conservation ·of energy between highest '
point of trajectory and point where contact breaks.
· KE; +U; = KE 1 +U1
1 2 =-mv
~mv 1 '2 +mg ' h- .... (7) A\111=""'-'-'='-="~~
2 2 1 Reference
,I
vJ
=v 2 cos 2 0
level I

· where ',Vi
2
= -gR
2
and cose = -
- ' 2 Fig. 3E.33
,..,__,_ _ .· ( • "'-' ';; 1
~_,,,,~,----------~,AA'>M-,~ l
3 3 Solution: After reaching point C the. ball becomes a
On substitµtirig these values in eqn. (7), we obtain projectile with vertical displacement 2R and horizontal
·. h=2R , . , -
displacement 3R. Let the velocity at'c be Ve.
27 Motion from C to A :
· Therefore maximum height from base of track'
=R+RcosfJ+h
=R+~R+2R= SOR
3 27 27
(c) From Newton'~ second law,
mv 12
N+mg = - - .. ,(8)
R From conservation of energy between points A and C,
KEA+UA=KEc+Uc
1 :i · .1 2 .
-mu + 0 = -mvc + 2mgR
2 2 ·

www.puucho.com
or, 2
u =-V~ + 4gR
--
Anurag Mishra Mechanics 1 with www.puucho.com

~- - - ---- --·7
LWORK AND ENERGY . . - -- - -- - 291
- - • - --------- J
9
=-gR+4gR
4
or U=~Jii
2
A particle attached to a vertical strin;r of length l m is:
projected horizontally with a velocity 5,,/ 2 m/ s. 1

(b) Minimum velocity with which the ball can reach ( a) What is maximum height reached by the particle from the:
point C is Jgi{, for which u must be ~SgR. lower most point of its trajectory.
Motion from C to A for Ve = fiii : (b) If the string breaks when it makes an angle of 60° with
downward vertical, find maximum height reached by the·
particle from the lower most point of its trajectory._ _
X=Vc Xt 2R =_l_gt2 Solution : (a) Minimum velocity at lowest point for
2 completing circle is u min = Jsii
=ffexrf or t =ff As
U
u=S-J2m/s, l=lm
= Umin = -!sii
or =2R Therefore Xmin = 2R To complete the whole circle is satisfied
Hmax = 21 = 2m

-· I

!A block of mass m is pressed again.st a spring offorce con.stant;


,k. The block after leaving contact with the spring moves along;
'a_ "loop" the loop track. The sliding surface is smooth except,
lfor rough portion of length s equal to R as shown in Fig.; •
Fig. 3E.35
-~-- -w
- ___ j

,3E.34, where the coefficient of friction is µk. Detennine the' (b) By work energy theorem from A to B, we get
minimum spring compression xfor which the particle will not
lose contact with the track? ' 1 2 1 2 1 0
-mu =-mv 1 +mg (1-cos60)
2 2
v 1 = .,J4Dm/s
Height from the lowest point
2 . 2 600
H = 1(1- cos60°) + v, sm
- 2g

,_ Fig. 3E.34

Solution : We know that minimum velocity required 'A particle is .suspended by a light vertical inelastic string of,
at B so that the block can complete the loop is v B = ~ SgR. length l from a fixed support. At its ';9.0-librium position it is_
Work done by friction when the block moves along the projected horizontally with a speed -.J 6gl. Find the ratio of the.
rough portion = -µmgs. ten.sion in the string in its ho1izontal position to that in the
From work-energy theorem, •string when the particle is vertically above the point of
support.
LlWnon-conservative = AKE+ ll.U g + 6.Us
2 Solution By work-energy theorem,
- -µmgs=(½mv~-o)+(o-½kx )
.!.m[vf -u 2 ] = -mg(l) . v2
1 2 1 2 2 +-i:rl---
or -kx =-mvB+µkmgs
v, =-J4gi. T2 mg
2 2
Thus, when at horizontal position,
or tension is T1
mv 2
or r, =--
1
T1 = 4mg
or At the topmost point, velocity is v 2 ___ Fl~- 3E.3_6

www.puucho.com
Anurag Mishra Mechanics 1 with www.puucho.com

- - MECHA.Nics;~
--- - - ------ --------- ----- ----------~
1 2 2 Solution:
-m[v 1 -u ] =-mg (21)
2 At extreme v =0 At vertical position
V2 =-fiii.2
mv 2
Thus, T2 +mg = - - =2mg
l

[£?iti~i~l'?-_ ,Gl>
T2 =mg
T1
T2
=4:1
·~ (a)
mg

. Fig. 3_1,_.3~
Vo
mg
(b)
and T2-mg=mv~//
~ mv~ = mgl(1-cos0)

T1 = mgcos0
A small ball is hung as shown on a string of length L
T2 = mg+ 2mg(l- cos0) ·
(a) If v O > .j2gL, find the angle 0 ( < 90° ) [ in terms of,.
v 0, g, L] ;With the upward vertical at which the string'
Given T2 = 2T,
becomes slack. 1 mg(3 - 2 cos0) = 2mg cos0
(b) Find the value of v O [in terms of g, L] if the particle passes' 3-2cos0 = 2cos0 =:> cose = 3/4
through point of suspension.

' '
A heavy particle hanging from a string of length l is projected!
horizontally with speed ..Jii.
Find the speed of the particle at:
Fig. 3E-37 (a) ,the point where the tension in the string equals weight of the•
lpa,pcle. _ ____ . ____ ·-·- ______________ . _ _,
Solution: (a) At the angle 0, when the
string becomes slack
Solution : Speed at bottom = < ..Jii -fiii.
1 1 2
mv 2 mgl(l-cos0)=-mgl--mv ... (1)
- - =mg cos0 ... (1) 2 2
L
mv 2
Also, T-mgcose = - -
.!cmv~ =.!:_mv 2 +mgL(l+cos0) ... (2) 1
2 2
Solving eqns. (1) and (2) gives
V
. ~h=/(1-cos0) !
v 0 = .jgL(2+ 3cos0) 9 T :
Fl~_- 3E._37 (b):
v 2 2gL
=:> cos0 = 0 -
3gL
• Af, /v I'
1---+ I

(b) After the string become slack, the ball follows ·vo=-,/(gl) mg i
the path of projectile. F!l!- 3E.3~_ J
For it to pass through point of suspension But T= mg
L sin0 =v cos0t (x-direction) ... (3) mv 2
- - = mg - mg cose
-Lcos0=vsin0t-.!:_gt 2 Cy-direction) ... (4) 1
2
i.e., .!:_ mv 2 = mgl (l - cos0)
Solving eqns. (3) and (4) gives, tan 0 =./2. 2 2
v 0 = .jgL(2+ 3cos0) eqn. (1),
=:> v 0 = ~gL(2+-,J3) 1 1
=:> mgl(l - cos0) = - mgl - - mgl(l - cos0)
2 . 2
2
1- cose =.!:_ =:. cose =-
3 3
:A simple pendulum swings with angular amplitude 0. The: V =.,fgl/3
Itension in the string when it is vertical is twice the tension in·
_it,; extre1J!<lpositi1m. Then find the value of cos0 :

www.puucho.com
Anurag Mishra Mechanics 1 with www.puucho.com

··- . . ... --- . - · 2~f3l


IWORK AND ENERGY - - - - · - ---~- - ---"---"--··--~-~---·-----··-------- ·,,,,.,_, - · - - - - __ ,; ·..J
POWER We can see that at the bottom of the curve the slope is
Work done per unit time is called power. zero and so the force component is zero. When x > 0, the
slope is positive, so the force component Fx is negative
Instantaneous Power
indicating that force is directed toward - i When x < 0 the
is defined as, P = lim t-.W slope is negative and the force component becomes positive
.M....+O Llt
or directed toward + i
or P=dW
Fig. 3.42 shows a
dt
..., ..., one-dimensional ,JFt.
~ ,.t. X •
The work done by a constant force F is W = F· s potential energy curve.
U(x) i
..., ..., Think of a potential
Position of :
Equilibrium;
Thus P=d(F-s) energy curve as a roller
dt coaster ride; you are the
~ i~ object riding without
=F·- friction over the track ax 1 x2 b X,
dt (you must remember that : Region : Regioh
..., ..., iwhere :where(
or P = F-v = Fvcos0 the actual particle motion ' slope is 'slope '
The SI unit of power is J/s. 1 J/s = 1 W is along a straight line). negative is positive
In the region where the Fig. 3.42
Power Delivered by Pump slope is positive, there is a
Consider a pump that lifts water from h meter deep well negative force. The force is directed toward left on the
and deliver at the rate of (dm/dt) with a velocity of v. particle. In regions where the slope is negative, the positive
Suppose dm amount of water is delivered in time dt. conservative force accelerates the particle to the right. So
The .work done .the range of values of x for which the potential energy curve
appears "uphill" to the particle, it slows down and the region
dW = (dm)gh+.!(dm)v 2 where U(x) appears "downhill" the magnitude of the
2
particle's velocity increases.
Power delivered,
The total energy is constant and can be represented as a
p = dW =
dt
(dm)[gh
dt
+ ~]
2
horizontal line on the graph. Because E = U(x) + K, U(x)
must be less than or equal to E for all situations: U(x) s; E.
(G) Potential Energy Diagrams: Stable and Unstable Thus, the minimum value which the total energy can take
for the potential energy is E O (see figure). At this position x 0
Equilibrium
·the mass can only be at rest, it has potential energy but no
For a conservative force in one dimension, kinetic energy.
..., ...,
dU=-F·ds =,-Fxdx :· Li<x>
I
F =-dU '
X dx EsH.------------
The force is negative derivative of the potential energy
function. Graphically the force is negative of the slope of the
line tangent to potential energy curve. For example, the
potential energy function of a spring-block system is E1 H---'--,..._---br-1(
kx
U = (1/2) 2 • By differentiating U, we get
Eo1..--,_ ___;___-:...:::,,_.-,:-1 :
..>fa
Fx =-: =-!(½kx )=-kx 2

U(x)
X/,i X4 X3

____ f.lg. 3.4_3


,Xo X1

As K = E - U(x), the kinetic energy at any value of xis


represented by the distance between the E line and the U(x)
Total curve at that value of x.
energy E
Consider an object with total energy E 1 • At position x 3
Slope
tangent
01/ '-stopeof
tangent
and x 2 the total energy will be the potential energy; the
velocity is zero. If x > x 2 or x < x 3 , the potential energy
negative positive
I would be greater than E, meaning
,
K = .!2 mv 2 < 0 and v
II Fig. 3.41
_, -----M~o~""-,• - - - - •• would be imaginary which is /4ossible. The points x 2 and
x 3 are called turning pc:µnts of the motion. Similarly,
www.puucho.com /
Anurag Mishra Mechanics 1 with www.puucho.com

j294 MECHANICS-I j
a and bare turning points in Fig. 3.43. A particle with energy Solution : Setting U(x) = 0, we get
E tot;;
is confined to the region a ,;; x ,;; b. a
X=-
A particle with energy E 2 has four turning points but the z1/6
particle can. move in only one of the two "potential energy
The force is negative derivative of potential energy
wells" depending on where it is -initially. For example,at a function.
position x 4 , U > E 2 which means v would be imaginary; a
particle cannot reach it.
For energy E 3 , there is only, one turning point since·
UcxJ < E 3 for all x > x 5 • A particle initially moving to· the left The potential energy has its minimµm value when its
will have variable speed as it passes through the potential slope is zero.
wells but eventually stops and turns around at x = x 5 • If
then proceeds to ·the right indefinitely without return. On setting Fx = 0, we get x = a; The·minimum occurs at
x = a, which is the average spacing between atoms in such a
At x = x 0 , "the slope of potential energy curve is zero; the molecule. The minimum energy of a molecule is slightly
force Fx = :...dU/dx is zero and the particle is in equilibrium. greater than the minimum -U0 , so the energy needed to
A particle is in equilibrium if the net force acting on it is, zero. separate atoms is slightly less than U0 •
If the particl~ is displaced from x = x 0 , the force is directed
back toward ·x = x 0 • The equilibrium at x = x 0 is. stable
e~uilibrium. If a particle returns toward its equilibrium
~~am.~·le~f"4il;;>
position whe~ 'displaced slightly, is said to be in stable IA particle of mass 2 kg is moving under the influence d] aforce
equilibrium.
llwhich. a. lw.·<zy.s acts towards 3·the. c.·en.·tr·e·· and whose po..t.ential
-dU energy is given by U(r) = 2r joule. If the body is moving in a
For particle at x = x 4,Fx = ~ = 0. When x > x 4 the
,circula~ofqitof;radius Sm, then.find its energy.·· '. ~ .
slope is negative and the force Fx is positive and when du -d(2r 3 ).
x < x 4 the slope is positive and the force Fx is negative. The Solution: F=- dx' F dr
force is in the direction that will accelerate the particle
toward jower potential energy, but the force is away from mv 2
F'=-6r 2 , F = - -
the equilibrium position. The maximum at x = x 4 is a point r
of unstable equilibrium. The object will accelerate away Required ce_ntripetal force,
from tpe equilibrium position if displaced slightly. ' 2
mv = 6r2
· For a· particle at x = x 6 the force is zero for some r
distance, the object is in equilibrium. A small displacemen,t mv 2 = 6r 3
results in zero force and the particle remains in equilibrium,
called neutral equilibrium. KE =.!mv 2 = 3r 3 PE= 2r 3
2 '

40 ~
Total energy = PE + KE
k~~~'~"J
·--4ft:~:::;- -r-::c"" .
Total energy= Sr 3
r· -.. Total energy = 5 x (5) 3
jThe forse b,el:}1/een two awms_,in a diatomic molecule cqn be
represented approximately by the potential energy function . TE= 625J
]':, .·,,., : · _·U. = U '[(a)12-_-2'(a_).6]·
1
· · •,
0 ~--
'·1 .. '
[E_~~tn-~~
. - X X
. '

~her~ U0 ciri1
a are constants. (d) .: At what, value of x; Is the IA single ~onversationforce Ei~)acts on a i.o k'gpµrticJe that

n=----!
potential energy zero? (b) Fiitd the force Fx. (c) At what value moves.along 'the x,axis: The potential energy U(x) is g/ven'by: .
:of xis the potential energy a minimum? · · U(x)=20+(x..:2) 2 . . •
where x. is. in meters. At x = 5.,0.m-the particle has a)iinetic
i' ' energyof20J. · : . ,· ·.. ,

x~~
(a)· What is the mechanical energy of the system?
'(b) Make a plot of U(x) as" a function of 'x for

L
l·._. -U,

Fig. 3E.40
x' I
'

'w
...:1om.:;; x s 10m.
'(c) The'least value of x and

move
~e) The maximum kinetic energy ~fthe particle'and''
The value ofiat which it occurs.
,
'(d) The greatest value of x between which the particle can

www.puucho.com
Anurag Mishra Mechanics 1 with www.puucho.com

295\
'(g) Determine the equation for P(x) as a function of x. U(x)-- -
(h) _Fo,· wh,rt(finite) value _of x_ilpes f_CxL":' 01 - l U3 ----------------------
U2 ------------,·----
Solution: (a) At x = s, PE= 20 + (5 - 2
2) = 29 J
, u, ------------
ME = KE + PE = 20 + 29 = 49 J ! Uol------1
U(J) E
:---· --·---·· 1a4 -
ol--------'.-,"---c~-d~-x'.
(b)

U(x)

10 xi

3,38
I Uo
E
-
_______ Fig, ,3E,42 _ __
X
(b) U(x = 0) = 24J a b

Umin(X= 2) = 20J -u,


(c)
U(x = 10) = 84J
U(x = -10) = 164J
U(x)
(c) and (d) When PE=ME=:>KE=0,v=0
49 = 20+ (x-2) 2 -~9 ----
(x- 2) 2 = 29 E

x-2 = ±5.38
-b/2-a/
=> Xmin = -3,38m, Xmax = +7.38m
(e) KE is max when PE is min ( = 20) ''
=> KEmax = 49 - 20 = 29 J
(f) KE max when PE min at x = 2 m ''
!
L ··- ,_. _____ !i

Solution Total mechanical energy of the particle is


(g)
dU
F=--=4-2x given by
dx
E=K+U
K=E-U
As kinetic energy K is always positive, particle can exist
only in that region where U < E,
' (a) For x > a, U > E
:diven below (figure) are examples of some potential energy,
:junctions in one dimension, The total energy ofthe,particle is: K becomes negative. Thus, particle cannot exist in the
'indicated by a cross on the ordinate axis. In each case, specify, region x > a.
'the regions, if any, in which the particle cannot be found for' (b) For any value of x, U > E, therefore, the particle
;the given energy. Also, indicate the minimum total energy the; cannot exist in any region
particle must have in each case, Think of simple physica!I
(c) In region x < a and x > b, the value U > E ,
'contexts for whic~_th~~ l?.'!!entia[~"::rgy_~hapes are relevant.
K is negative, The particle cannot be exist in these
U(x)
region~-- _ ____ _ __ _ ____ _ ___ ____ ___
7
' Concepts: (a) Kinetic energy of particle can never be:
- '
I, Uo inegative. -
i
, E
-------·---------- i (b) Total energy of particle can be negative _ I
(c) Potential energy can be negative
I -~---'-------x _(d)_IUl<IE!for K > O _____________ _
! 0 a
Fig, 3E.43 (a) ___ _J

www.puucho.com
Anurag Mishra Mechanics 1 with www.puucho.com

_. MECHANICS-I I
-- ---=-'-·~-,--··----------- -------~-- -.. . -_ . ·--.:-- w,

(H) Internal Energy Sources and Work · Concept: In an accelerating car the types on the drive
I
,wheels push back on the ground; the ground pushes forward
Concept: We will consider self-propelled objects, e.g., a_ :on the tyres and the car accelerates forward. But the region
car~ frog, helicopter, people, etc., that have their own internal' '.between the tyre and the road is motionless, and no work is
energy sources. Each can be accelerated by a net external force ·done on the_ car by the ground. The car does not derive its,
(F = ma) arising from its interaction with surroundings. As a: energy froni' the ground, it just pushes off it; the energy·
rule, such a force does no net positive work on the active': !equivalent to L\.KE comes from the fuel via the engine. ·
non-rigid body, and W "' t;KE. No energy is transferred to the - - - - -. - . . -- - ·-
body from the environment via the reactionforce even though'
I
that force accelerates the body. '
The energy required to walk, climb, skate or jump comes/
·11vo particles of mass m and 2m, connected by a massless rod,·
from the internal energy stored in the person. When we jump,·
slide on the inside of a smooth circular ring of radius r, as•
the upward reaction force that accelerates us acts at the shown in Fig. 3E.44(a). If the assemb(y is released from rest:
stationary foot-floor interface. If the floor is rigid, there is no when 8 = 0, determine 1

motion of the point of application of the force, and no work is: (a) the velocity of the particles when the rod passes the'
:done by the floor on you, Wnoor = 0, even though t;KE > 0 In', : horizon ta/ position,
reality the floor sags slight(y while. it exerts a normal force on' _(b) the maximum velocity Vm,. of the particles.
us and W floor is positive though very small, since displacement
'is very small.

2m

Fig. 3E.44 (a)

Solution: (a) In the absence of friction the energy of


the system is conserved.
Fig. 3.43 KE,+U,=KE1+U1
A swimmer's hand pushes back on the water and the: O+ 2mgr = .!mv 2 +.!(2m)v 2 + mgr(l- cos45°)
water pushes forward on the hand, accelerating the person.: 2 2
The hand does positive work on the water; the force it exerts is! +2mgr(l - cos45°) ... (1)
in the same direction as the displacement. On the contrary,. ~v 2 = 3grcos45°-gr ... (2)
or
the water pushes in thefonvard direction on the hand. It does I 2
negative work on the swimmer. W w, < 0 even though or v = 0.865.,/ir
t;KE > O The water gets energy from the swimmer, it gets' (b) At any general position 8 of the rod, the
. . I
·warmer. i conservation of energy between initial position and final
position gives
---:..--~----:~ -~ ~ -~::-~-,;:-===-~=-=:=t==~==-:::- -~-~~ \ '
2mgr = mgr(l - cos8) + 2mgr(l - sin8)

~%:i,~o;;~~~~. or
+.!mv 2 +.!(2m)v 2

~mv 2 = mgrcos8+ 2mgrsin8-mgr


2
2 2
... (3)

... (4)

Fig. 3.44 or v 2 =~gr[cos8+2sin8-l] ,,_(5)


3
The swimmer is a self-propelled source and uses water For v to be maximum, the expression in bracket must be
to generate a reaction force so that she can swim. If you maximum,
suspend a motor boat in air with its engine running at full
speed, will it move? The fuel provides the required energy i.e., ~ (cos8 + 2sin8 -1) = 0
d8
but without water to push on, the boat cannot accelerate.
www.puucho.com
Anurag Mishra Mechanics 1 with www.puucho.com

!' WORK AND ENERGY_________ _______________ _ 297'


- I

V = ..jzii. = ~Sg(I- x)
3
or x=-1
5
co' (b) From conservation of energy between position B
and C,
-~""'
•O
I
~

KEB +UB = KEc +Uc


.!. l __ ·________,_., _-'-R-c:eference level· 0+ mg(l -1 cose) = .!:_ mv5 + mg(l- x)(l- cos<j,)
2
Fig. 3E_,~4j_b)
or v 0 = [2gl (1- cose) - 2g(I- x) (1- cos <j,)]1/ 2
or - sine+ 2cose = 0
(c) From conservation of energy between position B
or tane= 2
from which we obtain sine= Js and cose = .Js
andE,
KEB +UB = KEE +UE
0+ mgl(l- cos60°) = 2-mv~ + mg(l- x)
Substituting these values in eqn. (5), we obtain 2

v
2
= 1gr[.Js + Js-1] or v~ = 2gx- 2glcos60°
2

or vmax = 0.90BJji
Or -VE=[ 2g(x-½)f
i·e.-xam~·!e :I~·-----
L~':- "f::> ii.---
- ·-· - c..::·:- 0
:,

Tize figure shows a pendulum of length l suspended at a' ,Tlvo blocks are connected by a massless string that passes over
,distance x vertically above a peg. , :a frictionless peg as shown in Fig. _3E.46 One end of the string
( a) The. pendulum bob is deflected through an angle e and! .is attached to a. mass m1 = 3 kg, i.~., a distance R = 1.20 m·
then released. Find the speed of the bob at the instant' 'from the peg. The other end of the string is connected to a.
shown in Fig. 3E.45. block of mass m2 = 6 kg resting on a table. From what angle
- .
A ~ '.e, measured from the vertical, must the 3 kg block be released
O···-····F,
.·:-<...;
/:•• 0: X
t iin order to just lift: the 6 kg block off the table?

.. ::::·· iI \E
B (j° ·:;_·········;····· Smooth peg

.. ~·.. /
..... i ·......
. .... ... :::,-.._;__....-:: ... ~.- ...
C Reference level
Fig. 3E.45 I
(b) The pendulum is released when e = 90°. For what x
(position of peg) will the pendulum complete the circle?
Fig. 3E.46
(c) The pendulum is released when e = 60°. What is the'
velocity of the bolJ as it p_as~".5 pp.,_iti,on E, ___ _ Solution: This problem involves several concepts.
Solution: (a) As we have learned earlier, the First we will apply conservation of energy to find the speed
minimum velocity required at the lowermost point so as to of the block m 1 at the bottom of the circular path as a
complete the circle is given by the expression function of e and the radius of the path, R. From Newton's
second law we will determine the tension at the bottom of
v=.Jsii its path as function of given parameters. Finally, the block
From conservation of energy between position of m 2 will lift off the ground when the upward force(tension)
release and position C, exerted by the cord just exceeds the weight of the block. We
KEA+UA=KEc+Uc take bottom of ·me circle as reference level. From
conservation of energy, we have
0+ mgl = 2-mv 2 + 0
2 KE-+U- = KE! +Ut
' ' 1
or v = ..Jzif. 0+ m1g(R -R cose) = - m1 v 2 + 0
2
For just completing the circle, or v 2 = 2gR(l - cose) ... (1)
www.puucho.com
Anurag Mishra Mechanics 1 with www.puucho.com

;;:, ~1_.,;~-' -..


:.~- -
Applying Newton's second law on block of mass m 2 , we For angle' e to be real,
have .. . . 2 ,(4m) 2 -4x6mxM>'0
V 3M
LF. = T - m1g = m 1 R or m>-
2 2
m 1v Fromm= 2M, eqn. (4) reduces to
or T=m 1g+-- ... (2)
R 12cos2 e '-Sease+ 1 = 0
As the string is massless, tension T is constant 1 1
throughout. When m 2 just lifts off, the normal reaction cose = -,
2 6
becomes zero. For block ·m 2 , we have
T= m2g · ... (3) As cos e = ½ occurs first, so the required angle is 0 = 60° .
From eqns. (1), (2) ·and (3), we get
ZgR(l - case)
m 2 g = m 1g + m 1 ~ - - - - l""@xcimj1.lg,£j 48 I~
~-=-=:=.~ .;;.!!~.:;p-..=;r,~~.
. R
or case·= 3m 1 - m 2· 3 x 3- 6 = .!. :A force acting on a certain particle\-7
r · • •. ', 1'""
y(m)
2m1 2x 3 2 Idepends on the particle's' position B(0, 1)>----~ 'C(1, 1)
or e = 60° in the xy=plane. This force F is .
given'by the_idcpression ·· · ·•
t:i§·~~J 47-~ -t<-.-.-~ ..... x(m)
F=,(xyf +xy j)(lN/m~) 0(0,0) . A(1,0)
i.Fig. 3E.47 (a) shows a circula" ring of mass M that hangs in a where x and
. y are· expressed, in Fig. 3E.4B
lvertica'1\7,1~ne.. .' Two beads pf mass m are·., released . ~
metre,.If'.F is a conservative force?
' '' ' .. .,·,
ll'#multa;eouslyfrom th(top ofthe ring in opposit~ ~irectiohs: • ' e '' ,, • '' ~,,_ ,~::--·1
There il.no frictional f<!rce between the bead and the ring. Exp]ai~'jour' answer. · ' ' .: __J
3
Show ..that. the ring
. will start i~· rise,' if m M. If m = 2M, ~i
. > 2
Solution: For each of the paths from o to c, work
done is given by ·
ivhat,(lriilttqftom the verticCl/J/ifsJ,_gppens?__._.:_ w;,,; Ji-d-;
·1~ .T --+ ,. " --+ "· .....

~ '.
._ ~ fl
s. N
!), i
where F = xy i + xy j and ds = dx i + dy j, so the·dot
I .· I
m
~~/f\
:.• mg
l
--+ --+
product F· ds = xydx + xy dy. The path OAC consists of OA
and AC. Along OA,y = 0 and dy = 0, and along AC, x = 1 m
. -

l
'I

r · ·l
~/ m and dx = 0.
•• So, WoAc=W0;._+wAc=O+f~ydy
• M

~
2

=ly2 [ =½J
l_·______<•_>_ _ _. ·_F,_lg_._3~-~!_;_ _ _ _ __ i_.'.:, The path OBC consists of OB and BC. Along OB, x = 0
Solution: Figs. 3E.47 (a) and (b) show force diagrams and dx = 0. Along BC,y'= 1 m and dy = 0.
of ring and bead .respectively. Let v be the velocity of the So WoBc=WoB+WBc=O+ 0 xdx ' f'
bead at'this position.
mv 2
LF = N + mg case = - -
n .
... (1)
R
. =lx2I' =.!.J
2 ' 2
0
From energy conservation, equation for bead, ·Along the straight line OC, y ~ x, so dy = dx and
--+ --+ . 2
2
.!. mv = mgR(l- cos0) F-ds·= xy.dx+ xy dy = 2x dx.
2
v 2 = ZgR (1- case) ... (2) Hence, f
. W~c = F · ds = f~ 2x dx 2

From eqns. (1) and (2), we.get • 1


2
· .iv= (2-cos0)mg
From the· force diagram of the ring, we see that, at the =l tl 0
=¾J
instant the ring begins to rise, tension in the string reduces 1
Although W oAc = WoBc = - J, the work done Woe along
to zero. 2 .
2Ncose=Mg ... (3)
From eqns. (2) and (3), we have OC is not equal to WoAc or WoBc· The force is p
2(2- 3cose)cos0mg = Mg non-conservative, because work done between two points
or 6mcos 2 e-4mcos0+M=0 ... (4) depends upon the particular path.

www.puucho.com
Anurag Mishra Mechanics 1 with www.puucho.com

•. ' IWORK AND ENERGY


\ > • '

r . ---~ . --- ---------. --- - -- ------- -- ---


'
IL .1- ·
~-- ------------
·only One Alternativ~ is Correct
--~~--------- - - -···--- ------------~- .. - --
l

1. A small block of mass m is kept on a rough inclined r·~·-----,


,li, : -·· • ' . ) ~ ~
.surface of inclination 8 fixed in a lift. The lift moves up i~ .· . ' '' !
(b) '"' --- . . _,, ..
with a uniform velocity v ,ind the block does not slide : I
. ._ ' ______ t-+,
on the incline. The work done by the force of frictio!l ,
on the block in time t will be:
(a) Zero
(c) mgvt sin 2 8
(b) mgvt cos 2 8
(d) mgvt sin:?B
r-L·-----·--7i
jKE ,~·~I
IKE ' . . .i
J.I
Cc) I . i (d) I
.. '
.
2. Consider two observers moving with respect to each
other at a speed v along·a straight line. They observe a
l," 0 IJ1sp(ace~ent j '
!. . '•
O... ~sp!~men~l
·I

block of mass m moving a distance 1 on a rough 6. If the block in the shown


surface. The following quantities will be same as arrangement is acted upon by
observed by the observers: a· constant force F for t ~ 0, its
(a) . Kinetic energy of the block at time t maximum speed will be: .
(b) Work done by friction (a) Fl Jmk (b) 2FI ..J,;ii
(c) Total work done on the block (c) Fl .J2mk (d) ..fiiiI ..Jmk
(d) Acceleration of the block 7. A block hangs freely from the end of a spring. A boy
3. The force acting on a body then slowly pushes the block upwards so that the
moving along.x-axis varies with string becomes strain free. The gain in gravitational
the position of the particle as potential energy of the block during the process is
shown in figure. The body is in equal to:
stable equilibrium at: · (a) The work done by · the boy , against the
(a) X =X 1 gravitational force acting on the block.
(b) = X2
X (b) The loss of energy stored in the spring minus the
(c) Both x =x1 and x =x 2 work done by the tension in the spring.
(d) Neither at x = x1 nor at x = x 2 (c) The work done on tlie block by the boy plus the
4. A uniform chain has mass Mand length L. It is lying on loss of energy stored in the spring.
a smooth horizontal table with half of its length (d) The work done on the block by the boy minus the
hanging vertically downward. The work done in work done by the tension in the spring plus the
pulling the chain up the table is: loss of energy stored in the spring.
(a) MgL/2 (b) MgL/4 (e) The work done on the block by the boy minus the
(c) MgL/8 (d) MgL/16 work done by the tension in th!! spring.
5. A blodc is resting over a smooth horizontal plane. A 8. A particle of mass m is moving in 'a circular path of
constant horizontal force starts acting on it at t = 0. constant radius r such that its centripetal acceleration
Which of the following graph is correct: is varying with time t as a, = k2rt 2 , where k is the
constant. The power delivered to the particle by the
forces acting on it is:

www.puucho.com
Anurag Mishra Mechanics 1 with www.puucho.com

(b) mk 2 r 2t body is permitted to fall instead, through what


distance does it stretch the string?
(d) zero
(a) d (b) 1.5 d
(c) 2 d (d) 3 d
9. A self-propelled vehicle of mass m, whose engine
delivers a constant power P, has an acceleration a = 15. A running man has half the .kinetic energy of '! boy of
(P/mv). (Assume that there is no friction). In order to half his mass. The man speeds up by 1 m/sec and then
increase its velocity from v 1 to v 2 , the distan~e it has has the same kinetic energy as the boy. The odginal
to travel will be: speed of the boy was:
3 ' m 3 3 (a) 2 m/S (b) 9.6 m/S
(a) P (v~ -vf) (b) -(v -v )
2 1
m ' 3P (c) 4.8 m/s (d) 7.2 m/S
m .3 16. An elastic string of unstretched length L and force
(c) ~(v~ -v{) (d) -(V2 -Vi)
3P 3P constant k is stretched by a small length x. It is further
10. A stone tied to string oflength I is whirled in a vertical stretch by small length y. The work done in the second
circle with the other end of the string at the centre. At stretching is:
a certain instant of time the stone is at its lowest
position and 'has a speed u. The magnitude of the
(a) (½)ky 2
(b) (½)k(x2+y2)
change in velocity as it reaches a position, where the
. (c) (½)k(x+y)
2 (d) (½)ky(2x+y)
string is horizontal is:
(a) ~u 2 -2gl Cb) fiii 17. A ball P is projected vertically up. Another similar ball
(c) ~u 2 -gl (d) ~~2(-u2---g-p Q is projected at ari angle 45°. Both reach the same
height during their motion. Then, at the starting point,
11. A ball of mass 5.0 gm and relative density 0.5 strikes ratio of kinetic energy of P and Q is?
the surface of the water with a velocity of 20 m/sec. It (a) 0.50 (b) 0.25
comes to rest at a depth of 2m. Find the work done by (c) 2 (d) 4
the resisting force in water: (take g = 10 m / s 2) 18. A particle of mass m is moving in a horizontal circle of
(a) - 6 J (b) + 7.5 J
· (c) - 9 J (d) - 10 J radius r unde: a centripetal force equal to (- r~ }
"12. · A particle of mass 1 gm executes an oscillatory motion where k is a positive constant. Then if kinetic energy,
· on the concave surface of a spherical dish of radius 2m potential energy and mechanical energy of the particle
placed on a' horizontal plane. If the motion of the are KE, PE ME
and respectively. Which one is correct?
(a) KE=(~), PE= -(!5.), ME=-(~)
. particle starts from a point on .the dish at a height of 1
cm from the horizontal plane and the coefficient of 2r r 2r .. ·
friction is 0.01, how much total distance will be moved
by the particle before it comes t,o rest: (b) KE=(~) PE=-(~) ME= zero
2r'· 2r' -·
(take g =" 10 m / s2)
(c) KE =zero , PE =zero, ME =zero
(a) 100 m (b) 1 m
(c) 10 m (d) 0.1 m (d) KE =(~), PE=-(!), ~E =(;r)
13. A spring is compressed between two toy-carts of
19. A 10 kg block is pulled along a
!r~--.
masses m1 and m 2 (spring is not atta.ched to the toy
carts). When the toy-carts are released, the spring
exerts on each equal and opposite average forces for
frictionless surface in the form of ,an
arc of a circle of radius 10 m. The ~·

6.O~··:O. F
..··
,

the same time t(t ~ o). If the coefficient of friction isµ applied force F is 200 N as shown. If
between the ground and the carts are equal, then the the block started from rest at point P,
displacements' of the two toy-carts are in the ratio: then its velocity at Q will be? (take g = 10 m/ s2 )
(a) ~ = - m2 (b) ~ = - ~ (a) 14.7 m/s (b) 15.7 m/s
s2 m1 s2 m2 (c) 16.7 m/s (d) 17.3 m/s

(c) s;
Sz
=
.
-(m2)2
m1
(d) ~ = ~(m1 )2
s2 m2
20. 1 kg block collides with a
horizontal massless spring of .J
(-=-·.~:-,._;0~
force constant 2 N/m. The
14. A light spring is hung vertically from a fixed support block compresses the spring by 4m. If the coefficient of
and a heavy mass is attached to its. lower end. The kinetic friction between the block and the· surface is
mass is then slowly lowered to its equilibrium position: 0.25, what was the speed of the block at the instant of
This stretches the spring by an amount d. If the same collision? (take g = 10 m/ s2 ) .

www.puucho.com
Anurag Mishra Mechanics 1 with www.puucho.com

IWORK AND ENERGY


· (b) ifz m/s
2
(a} 7.2 m/s mv ·
(c) 4.5 m/s (d) 10 m/s
(b) W =--.2r, W' =zero
r
21. A stone with weight W is thrown vertically upward , mv 2
mv 2
(c) W = - - , W = --.2,rr
into th_eair with initial velocityv 0 • If a constant forcef · r r
due to air drag acts on the stone throughout the flight (d) W = zero, W' = zero,
& if the maximum height attain by stone is h and
velocity when it strikes to the ground is v. Which one is
correct?
26.
~::::e !:sv; ;gu:t:~
sliding down a smooth
fl, . . ~
. - - --~m~it··~·.::.:-:-M. - :.'
and stationary circular
(a) h =vg(i+ !)/2g,v=v 0 ii/Ht' ,mm

track. Which of the · -------- -·· . - - - -


(b) h = v~ I 2g(1 +;} v = zero following graph best represents the variation of
magnitude of the force applied by the track on the
mass and the angle 8?
(c) h =v 0 !)
2 /2g ( l + - ,v=v 0~-J
--
W W+f
1 .F. ---·-1 !~F
(d) h=v 0
.
2 /2g ( l + -
W
f} v=v 0~+J
--
W-f !smg~--- :
i
··.1:
,3mg ... I
22. A stone of mass m, tied to the end of a string, is . (a) i : al (b) ·i : 8:
whirled around in a horizontal circle (neglect gravity).
I O ,c
l_ .._. _L _ _...1
" I
. l -- ½- _!
r-~F.
. . -·:
7C _
The length of the string is reduced gradually such that
mvr = constant. Then, the tension in the string is given ·
by T =Ar", where A is a constant and r is the
instantaneous radius of the circle. Then, n is equal to:
(a) + 2 (b) - 2
(c) l, LT
pig.
·O~e.
i (d) '
',2mg---.·
: 8
I
(c) + 3. (d) - 3 L__. ____ 2 _ _ ..J .
1
l:_ _____2
1t 1t
•..•...
'
!
23. A block of mass m released from rest from point O as 27. In the Q. No. 26, if M = 2 m and friction exists
shown below. The velocity of the block at the lowest between the circular track and the horizontal surface
points are v O , v E, v F respectively. Assume coefficient then, which of the following lot best represents the
of kinetic friction between surface and the block is variation of frictional force versus the angle 8:
same in all cases. Then,
r-·~F--
.. --- ·:
j
-·--- - - - . -----7 :·F~--···· .- I

~ ( B ) y ~ (C)y~! j3mg____ I !~--.


(a) 1 0 '21. " e
1
(b) ·; 0 . 'E. " ·a
__D_______'. __ I= ______x_ ..~I IL_,. ___2. _______ ,
!
, 2
- - - - _, ___ --«

1. 'F~---··
.... ·1
(a) Vv > VE > Vp
(c)vv=VE=Vp
(b) Vp > VE > Vo
(d)vv=VE=Vp=O
24. The mass m slides down the ; - - ?a] :F~- '
I mg
iy-
I
3mg
' 2
1t
- ' '

:

! _0 (d) \ · .2
track and completes the , ! "------~ (c) a : 8:

1
1 :E. " ! 0 : ' 1t :

:~~~ s~~f~~e. rii~~i~7~~:


0
~
'. h ... - ~--
I
L--------- -
. 2
- _ _ _j
I
·------- -·---- - ·- --'
i

value of h will be: 28. A particle of mass m is whirled in a vertical circle with
(a) R (b) 2R the help of a thread. If"the maximum tension in the
(c) 2.5 R (d) 3 R thread is double its minimum value then the value of
25. A body of mass m is moving in a circle of radius 'r' with minimum tension in the thread will be:
2 (a) 6 mg (b) zero
a constant speed v. The force on the body is mv and is (c) 3 mg (d) can't be found
r
directed towards the center. If work done by this force 29. A particle of mass m- is located in a one dimensional
in moving the body over half the circumference and potential field where potential energy of the particle
complete circumference is W and W', then: has the form U(x) =~ -~ where a and b are positive
mv 2 ,
X2 X
(a) W =--. itr, W =zero constants. The positi_on of equilibrium is:
r
www.puucho.com
Anurag Mishra Mechanics 1 with www.puucho.com

-, __ , ____
(a) _!!c (b) 2b (a) total energy (b) kinetic energy
.-2d~ . a, ,:· (c) potential energy (d) none of these ,
(c), ~ ' ' (d), 2a 37. The work done by ,in the forces (external and internal)
": b , ,' b ' on: a,·s)istem equals the change in:
30. Two, cy)im/_ri'cal vess~ls of equal cn;,ss-sectioµal area, A (a) total energy (bl kinetic energy
contairi water upto height ,h1 and h 2 • The vessels are (c) potential energy (d) none of _these
interconnected so that the levels in ,them become 38. .................... of a two particle system depends only 'on
equal. The work ,done by the force of gravity during the separation between the two' particles. The most
the process is: appropriate choice for the blank space in the· above
(a) zero (b) pA( h 1 ~h 2
)\
sent~nce. is:
(a) kinetic energy
,(b) total mechanical energy
(c), pA ( h I; 2h ) g
2
(d) pAh;h 2 g (c) potential energy
(d) total energy
31. A block of mass 100 g moved with,a speed of 5 m/s at 39. A block of mass m slides down a smooth vertical
the highest point in a closed circular tube of radius 10 circular track. During the motion, the block is in:
,cm kept in a vertical plane. The cross-section of the (a) vertical equilibrium
tube is such that the block just fits in it., The block (b) horizontal equilibrium
makes several oscillations inside the tube and finally · (c) radial equilibrium
steps at the lowest point. The work done by t\Ie tube (d) none of the above
on the block during the process is: 40. A particle is rotated in a vertical circle by connecting it
(a) 1.45 J (b) - 1.45 J to a string of length l and keeping the other end of the
(c) 0.2 J (d) zero string fixed. The mininlum speed of the particle when
32. A heavy stone is thrown from a cliff of height h with a the string is horizontal for which the particle will
speed .v. .The_ stone will hit ground with maximum complete the circle is:
speed if it is thrown: (a) :.JiL (b) .Jiil
(a) vertically downward (t) .J3gl (d) .JSgl
(b) verticaliy.upward 41. In the shown diagram mass of ~ - - - - -
(c) horizontally A is m and that of B is 2 m. All k
(d) the speed does not depend on the initial direction the surfaces are smooth.
33. Two springs A and B(kA = 2kB) are stretched by System is released from rest
applying forces of equal magnitudes at the four ends. with spring unstretched.
If the energy stored in A is E, that in B is: , Then, the maximum extension
(a) E, (b) 2E (x~) in spring will be:
2 (a) :mg (b) 2mg
k . k
(c) E (d) 1!_
(c) -3mg (d) 4mg
', , ' ,. . ;. 4,
34 .. , Two equal masses are attached to the two ends of a k k
spring of spring constant k. The masses are pulled out 42. In above question, speed of block A, when the
;YI)l~etri~ally to ~tretch the spring by a length· over x ., , ,' :,.,, ,
extension in spring is
.. ,, X .·
___!!!_, is:
, '

its natural length. The work done by the spring on . 2


each mass is:
(a) ~kx 2 (b) -~kx2
(a) 2gt (b) 2gP2
2
(c) ~kx 2 (d) -~kx 2
2
(c) 2g'\/3lZ
{2m (d) ~4m
g 3k
4 4
35. The negative of the work done 'by the conservative 43. A chain of length L and mass Mis arranged as shown
internal forces on a system equals the change in: in following four cases. The correct decreasing order
(a) total energy (b) kinetic energy of potential energy (assumed zero at horizontal
(c) potential energy (d) none of these ... surface) is:
36:'· The work done ·by the external forces on a system
equals the change in:

www.puucho.com
Anurag Mishra Mechanics 1 with www.puucho.com

\\VOIIKAND ENERGY ' :: ., :~


l . • • - - ____,.~--~-----'-',_;___._._.--~--'----='. ~~!J
I,
~ -
': r_' _. ' '. ~:i:,~-~-I:i.~: :,·.•,~
~-:. :_ c.,·_:·=j ·, '~i--/:~,·_:'.
~
49, ~la~~~U~:~~~; 0~ 'am:i~z!t!~
·, --. surface as shown iri:figure. Two
_1,:,·.((Dm
: rTl •• , c·_ 1_1'

(i) · · (ii).·' ' . ' ~ ' · ·, (iv) . '':J)"./ _,smallsphereseachofmassm;just ;·- M, ..


_. · -'----' fit ih the tube one released from 1 •
1
(a) i > ii > iii > iv > v the. top. If tlie tube' looses contact : "'"'""'"'"'"'" ,
· (b) i .= ii·;. iii > iv>. v· ' · with the ground ate·= 60° .then
(c) i = ii > iv > iii > v the value of m/M: ,' . ,
(d) i = ii > iv·> V > iii (a) 2
44. A block cif mass m is pulled· by a· constant p9w~r P
placed on a rough horizontal plane. The _friction (c) 3 ·
coefficient· between the block and surface is µ:· The

..
maximum velocity of the block is: ·
p .
(a)-.
mg
(c) ...!_
µmg·
' µP
' '.' ·
(b) -
mg
(d) _P_
µ2mg
',·
50. A spring mass system 'is held at rest
with the spring relaxed at height h
above the ground. The· ;minimum
value bf h for which the system has a
a

tendency to rebound ·after hitting the


·ground is (assume zero coefficient of
, 11
l;;L.~.'.. . .m]
45. Forces acting on a- particle moving in a straight line restitution for lower block and
~aries with the velocity of the particle ;s·F =~;where ground):
V, .
(a) 2mg (b) 3mg
u is constant. The work done by ,this force 'in time , ' k ' k
interval M is: · ··
(c) 3mg ' (d) 6mg
(a) ooit (b) ]:_Mt 2k• ' . k' ''
2 51. In shown figure, the trolley ~ - - - - - ,..-,--i
(c) 2a.M . (d) u 2t.t . .
46. A pendulum of mass 1 kg and length 1 =l ·mis released starts accelerating with .
acceleration a. The ·· , .
~Lm
, ·
al
.J
I "·t-·_
from rest at angle 60°. The power delivered by all the .maximum angle deflected by ,,...,,,Q,;,,4J,p
forces acting on the bob at an angle 0 = 30° is:
, thread from vertical will be:
(g=10m/s 2 ) ·
(a) l.'34 OJ (b) 13.4 OJ
(c) 0.670J (d) 5 OJ
47. A system consists of two identical· \
cubes, each of mass m, linked.
together by a compressed
'
m . - ·• I
-7 (b), tan-I(;)

(d) tan-I(~) .

weightless spring of force constant I I .... . .


52. A force F = -K(y i + x j) (where K is a positive
k. The cubes are also connected by .! , i
a· thread which is burnt at a certain I m I •. constant) acts on a particle 'moving in the X -'Y plane.
moment. The minimum value of
hlitial compression x 0 ; of the spring for which lower
~--·= · · Starting from the origin, the particle is taken along the
' ' positive x-~s to the point (a, 0) and then parallel to
cube bounce up after the thread is burnt-is:> the y-axis to the point (ri, ci). ·The 'total_ work.done by
(a) 2mg (b) 3mg ,- the force F on the particle is:
2
k k ... (a) -2Ka 2 Cb) 2Ka
2
(c) 3mg (d) 6mg ·, (c) -Ka 2 ,C\li Ka
2k k' 53. A particle free to move _along x-axis has potential
48. In the shown figure, the mass in energy given by U(x) = K[l- exp(-x) 2 ] for
sticks to the string just after it strikes ~= $ x $ ..,.,, where· K is . a positive c.o_µstant of ,
it. Then the-minimum value of h, so appropriate dimensions. Then:
that the lower mass bounce off the (a) At point away from the origin, the particle is in
ground during 'its rebound is: · · ·· unstable equilibrium, ·
(a) 2mg , · , (b) ,3mg (b) For any finite non-zero ,value of x, there is force
·k . ·,"k directed away from the origin ·
(c) 3mg.,' . . (d) 67/l&· (c) If its total mechanical energy if K/2, it. has its
2k ' ' k' www.puucho.com minimum ri at the origin · ·
Anurag Mishra Mechanics 1 with www.puucho.com

·-- - - -MECHANlcs:_!J
____-----
r-308
, .
..
------ --- -- . ------ ··-- ------ -----------·------~--· ------·-- -- ------- ---- --------·--·-----
-·----- --·· -1
(a) Work done by F is 12olz J
1
(al Point D is position of
neutral equilibrium
u I
I
_, '
(b) Work done by F2 is 180 J (bl Point B is position of
_, unstable equilibrium :-
(c) Work done by F3 is 45,c J (cl Point C is position of C I
_, A B D,
X'
stable equilibrium '
(d), F1 is conservative in nature i
(dl Point A is position of l . - - - - _,_ - - -·--MO-·----'
19. The potential energy U in joule of a particle of mass 1 neutral equilibrium
kg moving in x-y plane obeys the law U = 3x + 4y, 24. A smooth narrow tube is in
where (x, y) are the co-ordinates of the particle in form of an arcAB of a circle of
meter. If the particle is at rest at (6, 4) at time t = 0 '/~- !
center at O and radius R. is
then:
(a) the particle has constant acceleration
fixed so that A is vertically R: ' .
.
above O and OB is horizontal: '
(b) the particle has zero acceleration O
c............ B
Particles P and Q of mass m . Q
(c) the speed of the particle when it crosses y-axis is and 2m respectively' with an
10 m/s
(d) co:ordinate of particle at t ~ l sec is (4.5, 2) ideal string of length":,, co,nnecting them is pla_ced as
20. A simple pe'ndulum ,., .
,. ,. ______ .. ,
shown in the figure. ·The speed of the particles as P
consisting ..of a mass M reaches B will be:
l ' :a.
attached in a string of 1. I £ :

length L is released from ·


:
(al ~2gR
. 3
(bl ~2gR, ,,
rest a~ an angle a. A pin is • 2(1 + 1t)gR ,
3,c
located at a distance '1' i . e !. (c) (dl ~21t;R ,.
below. the pivot point. - .. ______ -- ..... ..:.•. - ~ 3
When the pendulum swings down, the string hits the 25. In a children's park, there is a slide which has a total
pin as shown in the figure. The maximum angle 0 length of 10 m and a height of 8 m. A vertical ladder is
which string makes with the ·vertical after hitting the provided to reach the top. A boy weighing 200 N
pin is: climbs up the ladder to the top of the slide and slides
(a.) cos--i(Lcosa+l)
L+l
(bl cos-' cosa +
L-l
(L l) down to the ground. The average friction offere.l by
the slide is three tenth of his weight. Then:
(c) cos-' cos a
. L-·l
(L -1) (dl cos-' cos a
, L+l
(L -1) (al The work done by ladder on the boy as he goes up
is zero
(b) The work done by ladder on boy as he goes up is -
_,
lm, 1ml under a force F ~ (y i + x jlN. The work
.
21. An object is displaced from a pointA (0, 0, 0) toB (lm,
. 1600 J
(cl The work done by slide on boy as he comes down
done by this force and the nattire of the force is: is.:. 600 J' · f
(al 1 J, non-conservative (dl The work done by slide on boy as he comes down
(bl 1 J, conservative is 1600 J
(cl zero, conservative 26. A particle of' mass m is kept at the top' of a smooth
(d) zero, non-conservative fixed sphere. It is given a horizontal velocity v then:
22. A particle iµass is tied to an ideal string and whirled in (al it will start moving along a circular path if v < .jiR
a vertical circle of radius L, where L is off-course the (b) it will start moving along a circular path ifv > .jiR
length of the string. If the ratio of the maximum to (cl it will start moving along a parabolic path if
minimum tension in the string throughout the motion V < .jiR
is 2 : 1, then the maximum possible speed of the (d) it will start moving along a parabolic path if
particl~ will be: V > .jiR
1
(al .,jl lgL (bl ..Jsif, 27. The total work done on a particle is equal to the
(cl .,jlOgL (dl .j3if. change in. its kinetic' energy: '
23. The following plot shows the variation of potential (al. always .
energy (U) of a system versus position (xl. From the (b) only if the forces acting on it are conservative
graph we_ can interpret th.at: (cl only if gravitational force along acts on it '
(d) only if elastic force along acts on it

www.puucho.com
Anurag Mishra Mechanics 1 with www.puucho.com

IWORKANDENERGY ________________ - - - - - - - - - . : "309]


28. A particle is acted upon by a force of constant coordinates of the point being x and y, measured in
magnitude which is always perpendicular to the metres. If the particle is initially at rest at (6, 4), then:
velocity of the particle. The motion of the particle (a) its acceleration is of magnitude 5 m/ s2
takes place in a plane. If follows that: (b) its speed when it crosses they-axis is 10 m/s
(a) its velocity is constant (c) it crosses the y-axis (x = 0) at y = - 4
(b) its. acceleration is constant (d) it moves in a straight line passing through the
(c) its kinetic energy is constant origin (0, O)
(d) it moves in a circular path 34. A ball is projected vertically upwards. Air resistance
29. You lift a suitcase from the floor and keep it on a table. and variation in g may be neglected. The ball rises to
The work done by you on the suitcase does not depend its maximum height H in a time T, the height being h
on: after a time t :
(a) the path taken by the suitcase (1) The graph of kinetic energy Ek of the ball against
(b) the time taken by you in doing so height h is shown in figure 1 •
(c) the weight of the suitcase (2) The graph of height h against time t is shown in
(d) your weight figure 2
30. A particle of mass m is attached to a light string of (3) The graph of gravitational energy Ek of the ball
length 1, the other end of which is fixed. Initially the against height h is shown in figure 3
string is kept horizontal and the particle is given an
upward velocity v. The particle is just able to complete h E0
a circle.
(a) The -string becomes slack when the particle
reached its highest point
(b) The velocity of the particle becomes zero at the
highest point
I
(c) The kinetic energy of the ball in initial position IL..---'-------+1
0 T 0 H h_
1
was -mv 2
= mgI (1) (2( (3)
2
(d) The particle again passes through the initial
l,. __ - - - - - - - _ _ _. , _ -- - -------- - - -
(i) - Which the figure shows the correct answers ?
position (a) 3 only (b) 1, 2
31. The string of a simple pendulum can with stand a (c) 2, 3 (d) 1 only
maximum tension equal to 4 times the weight of bob
(ii) In the above situation the block wili have maximum
suspended to it. The string is made horizontal and bob
velocity when:
is released from rest then:
(a) the spring force becomes zero
(a) String will break somewhere during the motion (b) the frictional force becomes zero
and will then follow straight line path
(c) the net force becomes zero
(b) String will break somewhere during the motion
(d) the acceleration of block becomes zero
and then follow parabolic path
(c) It will complete the vertical circle (iii) Two particles move on a circular path (one just inside
(d) lv!otion will be oscillatory and string will not break and the other just outside) with angular velocities ro
and 5ro starting from the same point. Then :
32. A particle of mass m is at rest in a train moving with
(a) they cross each other at regular intervals of time
constant velocity with respect to ground. Now the
parti~le is accelerated by a constant force F0 acting -27thw en th"eir angul ar ve1ocines
·· are opposite ·1y
4ro
along the direction of motion of train for time t O• A girl
directed
in the train and a boy on the ground measure tl).e work (b) they cross each other at points on the path
done by this force. Which of the following are subtending an angle of 60° at the centre if their
incorrect? angular velocities are oppositely directed
(a)' Both will measure the same work
(b) Boy will measure higher value than the girl (c) they cross at intervals of time...::.. if their angular
3ro
(c) Girl will measure higher value than the boy velocities are oppositely directed _
(d) Data are insufficient for the measurement of work (d) they cross each other .at points on the path
' done by the force F0 subtending 90° at the centre if their angular
33. The potential energy in joules of a particle of mass 1 kg velocities are in the same sense
moving in a plane is given by U = 3x + 4y, the position

www.puucho.com
Anurag Mishra Mechanics 1 with www.puucho.com

__. ,,
.,
-~..-....'....,.;..'.;, . '
.
--~----__.,,;_:,.,L.L.
. . ·;: ,ME~HAN@:(1
~

Comprehensio~ ~~e-d Problems-.--~


" - ---,<~·-··--·. -----· -----·-·-----·----·-----· ---···=-··--·-··-·-·--- ~_J
....
,, .' ' . --- ---------·--- 3. The maximum speed of the particle is:
(a) 1 mis · · (b) 29 mis
·- "' ·r,, ·-P ,;1,
p;as-S.A161E n ;,; ~l'' j'•
~, (c) ../29 mis (d) -Jss mis
..; ...,,.. ~,,, _ i'..nJ l.4 Lb:
4. The minimum speed of the particle is: ·
~ fu the iho~figure, the sprfug and string is id;al. Th~ (a) 1 mis (b) ./40 mis -
spring the stiffness of 100 N/m;and m = 1 kg friction (c) -Jss mis (d) zero
exists between mass 2 m and surface with coefficient 5. The maximum value of potential energy is:
W =s O,s;' Tlie sy~tem is released with spring from its (a) zero (bl 20 J
reiax~d. position. Based on aJ;,ove data, answer ):he (cl 29 J (d) 49 J
following question: (take g .= 10 m/ s2 ) 6. The least value of x (position of particle is) will be:
--~, (a) zero (b) - 2
(cl -../29+2 (d) ../29+2
.:;] 1-"0m""-*; 7. The largest value of x will be:
1/ I· .· (a) zero (b) - 2

L_:__,_. :,~I=-_:____ ;-__:~_____ j 8.


(cl -../29+2 (d) ../29+2
The position of equilibrium and its nature is:
(a) x ·= 2, unstable
1. Maximum extension in spring is: (b) x = 2, stable
(a) 8 cm (b) 4 cm (c) 36 cm (d) 20 cm (c) x = 2, neutral
2. Magnitude'. of work done by gravity during the motion
(d) no equilibrium position exists
of system is:
(a) 0.8 J(b) 1.6 J (c) 0.4 J (d) 4 J
3. Magnitude of net work done by spring after the system
is released for motion is:
! fil s's:h'GTE
·- ,.. -~-· -
,i~~;;' ~
l..{.....

.
' '
i, -,_ i»Tii~-- ~
' -

.
:I
(a) o:s J - (b) 1.6 J (c) 0.32 J (d) 2.40 J · A block of mass m moving with a velocity v 0 ·on a/
4. Frictional force acting on the mass 2 m when it finally smooth horizontal surface strikes and compresses a
comes to rest is: spring ofstiffuess k till mass coines to rest as shown in
(a) 16 N (b) 8 N the .. figure. This phenomenon is observed by •two
(c) 12 N (d) zero observers :
5. After what displacement of mass 2 m, its velocity A : .~tanding on the horizontal surface , .
becomes maximum? B : standing on the bloc!,
(a) 4 cm
""v,--~~
(c) 2 cm
I oJ;j;JyJJ~J"'
1. To an observer A, the work done by spring force is :
(a) negative but nothing can be said about its
A s4!gle;. conservative
.
fr.xJ. acts ~n a. m = 1 kg particle magnitude
. moving ~long-the x-axis. The potential energy,UcxJ i~ 1 2
(b) --mv 0
given bf - · · · · 2
(c) positive but nothing can be said about its
Ucxl = 20.+(x- 2) 2 magnitude
. '~ '
where xis in·mettes. Atx = 5 m, a particle has kineti 1
(d) +-mv 2
0
~~energy of.;!O J._____ ---- . ---.---~ 2
1. The total mechanical energy of the system 'is: 2. To an observer A, the work done by the normal '
(a) zero (b) 20 J reaction N between the block and the spring on the
(c) 29 J (d) 49 J block is:
(a) zero 1
(b) --mv 2
2. The minimum.potential energy of the particle is: 0
2
(a) zero (b) 20 J 1 2
(c) +-mv 0 (d) none of these
C~) ;29 J Cd) 49 J 2
www.puucho.com
Anurag Mishra Mechanics 1 with www.puucho.com

,W~RK AND ENERGY.


3. To an observer A, the net work done on the block is: 1 2 1
(d) --mgat 2
(c) - mgat 0 0
(a) -mv~ (b) +mv~ 2 2
1 2 4. According to the observer A :
(c) --mv 0 (d) zero
2 (a) the work done by gravity is zero
(b) the work done by normal reaction is zero
4. According to the observer A :
(a) the kinetic energy of the block is converted into (c) the work done by pseudo-force is zero
the potential energy of the spring (d) all of the above
(b) the mechanical energy of the spring-mass system ~-':?T' --·-:
is conserved
(c) the block loses its kinetic energy because of the
0

:
PASSAGE ···
' • - -· • - ' .I
-~~Jj f

negative work done by the conservative force of A sprii;ig block system js placed ·on a rough horizontal
spring floor. The block is pulled towards right to give spring!
(d) all of the above an elongation less than Zµmg but more than µmg
K K
i
5. To an observer B, when the block is compressing the
spring : and released.
(a) velocity of the block is decreasing
(b) retardation of the block is increasing
(c) kinetic energy of the block is zero
(d) all of the above 1. Which of the following Jaws/principles of physics can
6. According to observer B, the potential energy of the be applied on the spring block system ?
spring increases : (a) Conservation of mechanical energy
(a) due to the positive work done by pseudo-force (b) Conservation of momentum
(b): due to the positive work done by normal reaction (c) Work energy principle
between spring and wall (d) None
(c) due to the decrease in the kinetic energy of the 2. The correct statement is :
block (a) The block will cross the mean position
(d) all of the above (b) The block will come to rest when the forces acting
..• i · on it are exactly balanced
(c) The block will come to rest when the work done
PIISSAG~ by friction becomes equal to the change in energy
' stored in spring
A block of mass mis kept in an elevation which starts 1 (d) None
moving downward with an acceleration aas shown in·
figure. The block is observed by two observers A and'
B for a time interval_ t O• • • PAS S,A Gl

u]I·B
This diagram depicts a block sliding along a,
frictionless ramp in vertical plane. The eightl
numbered arrows in the diagram represent directions,
I to be referred_ to \Vhen answerin~ the questions. :

1. The observer B finds that the work done by gravity on


the block is :
' *12
··: ~"' 7
'. I 6 4
3
Ill
2 1 2 2
(b) --mg t 0
•.·.. I5I . /··-
(a) .!:.mg t5 ··---··
2 2
1
(c) -mgat 0
2
(d) _.!:.mgat5
______ "_J
2 2 ! ----··-··-
2. The observer B finds that the work done by 1. The direction of the acceleration of the block, when in
pseudo-force on the block is : position I, is best represented by which of the arrows
(a) zero (b) -ma 2 t 0 (c) +ma 2 t 0 (d) -mgat 0 in the diagram ?
3. According to observer B, the net work done on the (a) 2
(b) 4
block is:
1 2 2
(c) 5
(a) - - ma t 0 (d) None of the arrows, the acceleration is zero
2
www.puucho.com
Anurag Mishra Mechanics 1 with www.puucho.com

r312- MECHANICS-I j
2. The direction of the acceleration of the block when in 3. Choose correct statement :
position II is best represented by which of the arrows (a) In ground frame, work done by friction on ground
in the diagram ? is positive
(a) 1 (b) 3 (c) 5 (d) 8 (b) In ground frame, work done by friction on ground
3. The direction of the acceleration of the block (after is negative
leaving the ramp) at position III is best represented by (c) II: frame 2, work done by friction on ground is
which of the arrows in the diagram ? negative
(a) 2 (d) In frame 2, work done by friction on ground is
(b) 5 positive
(c) 6 .-- --- ......
(d) None of the arrows, the acceleration is zero i
'PASSAGE
The potential energy at a point, relative to the
PASSAGE reference point is always defined as the negative of
The kinetic energy of any body depends on the frame work done by the force as the object moves from the
of reference of the observer. The kinetic energy is reference point to the point considered. The value of
given by l/2mv~1• Similarly the displacement of the potential energy at the reference point itself can be
set equal to zero because we are always concerned
object from different frames of reference will be:
only with differences of potential energy between two
different. But the forces acting on the body remain
unchanged. So work done by the forces as seen from: points and the associated change of kinetic energy. A
different frames will be different. But work energy! particles A is fixed at origin of a fixed coordinate
theorem will still be hold in every inertial reference' system. A particle B which is free to move experiences
frame. · an force F= (- 2a +
r3
1-) t due to particle
r2
A where t
For example, if a block of mass 2kg is moving with;
velocity of 1 m/s towards east on a rough surface its is the position vector of particle B relative to A.
1 ' I It is given that the force is conservative in nature and
KE=-x2xl 2 =1J
2 _potential energy at infinity is zero. If B has to be
If it comes to rest, its KE = 0. removed from the influence of A, energy has to be
supplied for such a process. The ionization energy E 0
Work done by friction= K 1 -K, = -lJ
is work that has to be done by an external agent to
If we observe it from a frame 2 moving with 1,m/s move the particle from a distance r0 to infinity slowly.
toward east, its initial velocity will appear to be, H_ere r0 is the equilibrium position of the particle.
1-1=0.
1. What is potential energy function of particle as
Initial KE = _! x 2 x 0 2 = 0 function of r:
2
Final velocity = O- 1 = -1 (a) ~-f
2
(b) -~+f
2
2
r r r r
Final KE=_! x 2x (-1) = lJ
2 (c) -~-f
2
(d) ~+f
.? vl/or_k don~ by friction= 1- 0 =_lJ .. _ .. _ rr r2 r
2. Find the ionization energy E O of the particle B :
1. According to passage: p2 2p2 p2 p2
(a) In 2 nd frame, force of friction was opposite to (a) - (b) ~ (c) - (d) -
displacement 20: o: 40: o:
(b) In 2 nd frame, force of friction was in same 3, If particle B is transferred slowly from point
direction as displacement
(c) In ground frame, force of friction is in same
P1 (-J2 r0 , -J2 r0 ) to point P;(!Q..., !2...) in the .zy-plane
-J2 -J2
direction as the displacement by an external agent, calculate work required to be
(d) None of the above done by it in the process:
2. What should be the velocity of an observer so that he
will report the work done by friction on the block to be
ca) 9p2 Cb) L
640: 160:
0:
(a) I.m/s W
1
(b) -m/s E (c) L
640:
(d) None of these
2 2
(c) lm/s W (d) lm/s E
www.puucho.com
Anurag Mishra Mechanics 1 with www.puucho.com

WORK AND ENERGY 313

MATCHING TYPE PROBLEMS

1. Match the following:


Column-1
.__ ___ - ----·- - ------ __ , ____
Column-2
---- ---- - -··--·
(A) Work done by all the forces (P) Change in potential
B : QR
F~U~
A;C X p: X

energy
(B) Work done by conservative (Q) Change in kinetic
forces energy ___Column-1
_______
.,:..:.,..,,_. ------- Column-2
---- - --
···-

(A) A (P) p
(C) Work done by external (R) Change in
forces mechanical energy (B) B (Q) Q
(S) None (C) C (R) R

(S) None
2. A particle is suspended from a 5. A body is moved along a straight line by a machine
string of length R. It is given a delivering a power proportional to time (P = t ). Then
velocity u = 3.jgii at the bottom. match the following:
Match the following: . .· ..
Column-1 Column-2
(A) Velocity is proportional (P) t
Column-1 '. Column-2
to
----- ---- ---- (B) Displacement is (Q) t2
(A) Velocity at B (P) 7mg
proportional to
(B) Velocity at C (Q) ~5gR t3
(C) Work done is (R)
(C) Tension in string at B (R) ~7gR proportional to
(D) Tension in string at C (S) 5 mg 6. A pendulum is released from
point A as shown in figure. At .
(T) None
some instant net force on the
3. A force F = kx (where k is a positive constant) is acting bob is making an angle e with C 0_·:/ : .. A
_on a particle. Work done: the string. Then match the ··- .. __:___ ... ·
following: B
Column-1 Column-2
···--·.
(A) In displacing the body (P) Negative Column-1 Column-2
fromx = 2 tox = 4 ------------------------ -------·--
(A) Fore = 30° (P) Particle may be moving
(B) In displacing the body (Q) Positive betweenB &A
fromx=--4tox=-2
(B) Fore = 120° (Q) Particle may be moving
(C) In displacing the body (R) Zero between C & B Particle
fromx = -2 tax= +2 is at A
4. F-x and corresponding U-x graph are as shown in (C) Fore= 90° (R) Particle is at A
figure. Three points A, B and C in F-x graphs may be
(D) Fore= 0° (S) Particle is at B
corresponding to P, Q and R in the U-x graph. Match
the following: (T) None

www.puucho.com
Anurag Mishra Mechanics 1 with www.puucho.com

. MEotANiCS-1 :
-7. Match the following: 10. Initially spring are in natural length. An application of
external varying force F causes the block to move
' c"otiir'i\~~1 .. · ____
1--,---'---···~---~~.:::_· Column:2 slowly distance x towards wall on smooth floor :
1
(A) Electrostatic potential' (P) Positive
'energy
' '
(B) Gravitational potential (Q) 1Negative
energy
(C) Elastic potential energy (R) Zero
. . \ Column-1 ;:;,. . , . . Colutjlri~,~--
1----·~· -~-'-----·.
(D) Magnetic potential (S) Not defined
energy (A) Work done by S2 on block (P) ' Zero

8. A particle of mass m kg is ,displaced from one given


(B) Work done by.S 2 on S1 (Q) _.!_(
k1k2 )x2·
point to another given point under the action of : ' 2 k1 +k2
several conservative and non-conservative forces
(C) ,Work done by Fon block ,(R) •.!_( k1k2 .Jx2
(Neglect relativistic considerations). Now match the
following. 2 k1 + k2

:·~::>~-~~nz~:~9~l~flin~f·· , Coluni~--2 __ _ (D) Work done by S1 on wall '(S) ·1 k I k 22 x 2


(A) '.Displacement of particle (P) Path dependent

(B) ,work done by, (Q) :Path independent 11. Column-1 represents potential energy graph for
:conservative force certain system. Column-2 gives statements related to
(C) ·Work done by (R) 'Frame dependent graphs.
]non-conservative force Column-1 . ,.. · Column-;z,.,
~,,iJi,.._-~~-
"i, '-~ ~--"'---- _ . _ . . _ ; . , _ . _ _ _

(D) Angular displacement (S) Frame independent (A) U(B) (P) If total energy
, (T) Dependent on location is E 3 , it.is not
........................... 2.5 mg!=E3 possible for
of observer in a given
frame .......................... 2mgl=E2
;the body to
have any
9. In the figure shown, upper block is given a velocity turning. point
6m/s and very long plank, velocity 3m/s. The in its motion
following quantities are to be matched when both • e
attain same velocity.
j. - rciUgh- ·- - . U vs 0 graph for a bob hanging
· ~kg-6m/s
vertically from a string with its
lowest position as reference level
;____ ~:,~~mm,Ji;;;,3
smooth --~-~
mis, and 0 is angle of string from'
vertical line
(B) U(x) (Q) For a small
1 :displacement
(A) work done by friction on 1 kg (P) Positive
'block in joule ..............................•... E, about point 0
'Work done by friction on 2 kg potential
(B) (Q) Negative .• .•••..•..•.. •.. ..•..••..•..• E2
energy
:Plank in joule function is
------ -------- --------- E1
(C) !Magnitude of change in' (R) 3 'quadratic in
momentum in N-s of 2 kg plank 'variable
plotted on
(D) ,Change in KE of system (S) 7 A pmticle moving along x-axis x-axis
consisting of block and plank in
1
with potential energy function as
2
joule , i) U(x) = [1- e-x ]
(T) 2
www.puucho.com
Anurag Mishra Mechanics 1 with www.puucho.com

r-----.
, WORK AND ENERGY
r.-----'---._-_··.c_·-c..·-'----'--"-'-~-=;_:__:__;_:_:_=__;_--::;-·c.___:::;-- -- ·- - -- - - --· --- - ---- - -- - - - -- - -·- - - --
U(x) . . - 13. In column·l, a situation is depicted each of which is in
(C) (R) For a small
displacement vertical plane. The surfaces are frictionless. Match
. . . . . . . . . . . . .. . . . .............. E, with appropriate entries in column·2.
about position
, ••••• Ez 0 motion is 0;•C<iluinn~1
--"::_-':-1-=_"a-+--'+"'a+-'--=•x
simple
harmonic
------- ;!,•
·--'--·
'.' -~- -~--
Column~2' ,
-·-·-------~·--'- -
(A) .Bead is threaded oh a (P) ,Normal force is
............... ······· ....... E, circular fixed wire and is zero at topmost
,projected from the lowest point of its
point trajectory
,Potential energy function of a
panicle in an arbitrary force field
(D) U(r) (S) If total energy
is Etoral < E2
panicle
. ·····························E, 5m/s
executes
a Ro b periodic and
······E2,
(B) _Block loosely fits inside the (Q) Velocity of the body
oscillatory
-----· '
..............
' .' ·········E1
motion for all
fixed small tube and is_· is zero at topmost
projected from lowest point' 'point of its
energy values
trajectory
greater than
Graph represents potential energy atO
energy for a particle .. •
.-··
(T) Point Q is
position of -JZomls
stable
equilibrium (C) Block is projected (R) Acceleration of the
horizontally from lowest body is zero at the
12. A bob tied to an ideal string oflength 1is released from point of a smooth fixed' topmost point of its
the horizontal position shown. A peg P whose height is cylinder trajectory
adjustable, can arrest the free swing of the pendulum,
as shown in figure.
~=1m


I 11 I \Ill\\" I I\ II I\\\

Peg°"'\ ! ''
' ~

y:f:
I, .•
....,' 6m/s '.

···'-O-······ (D) Block is projected on a fixed: (S) Normal force is


,hemisphere from angular radially outward at
position 0 -topmost point of
21 21 ·trajectory
(A) For what range ofywill the (P) - <y<-
string wind up on the peg,: 15 3
remaining taut throughout'
~'
the swing
~
_ _cos 8 = 2/3
(B) Forwhatrangeofywillthei(Q) •0<y <~
'pendulum become: : 5
'projectile
(C) For what value of y will (R) ~ < y < l
mechanical energy always 5
remain conserved
l l 21
'(S)
-<y<-
!3 3
www.puucho.com
Anurag Mishra Mechanics 1 with www.puucho.com

[316 _____ -------- __ ------ __ -- --~------- ~---


"' . . ' <· . ''--" ' .·,
14. A block of mass m is tied with an inextensible light Column-1 Column-2 ·
string of length!. One end of the string is fixed at point <-----·-------- ---I
(A) Highest point · (P) Acceleration is horizontal
0. Block is released (from rest) at A. Find acceleration
of particle during its motion in vertical plane at (B) At lowest point ; (Q) :Acceleration is vertically
positions specified in column-1 and match them with 'upwards
column-2. Given that A and O are at same horizontal
(C) At 0; tan- 1 (-.J2) (R) 'Acceleration is vertically
level.
A---i._-_----,0' ,with vertical ,downwards
I
, (S) ·Acceleration has both
: ! horizontal and vertical
components

- --
. -· ,. - . - -
AN8WER8
- - -- -

=-Lf:vel~~:- O~iy ~~e ~A!te~na.tii~-is :c~~r~~ ~"3.

1. (d) 3. (b) 4. (c) 5. (c) i 6. (a) 7. (c) 8. (b)


(c) 2.
11. (c) 12. (b) 13. (c) 14. (c) 15. (c) 16. (d)
9. (b) ' 10. {d)
17. (a) 18. (a) 19. (d) 20. (a) 21. (c) 22. (d) 23. (c) 24 (c)

(d) 26. (b) 27. (b) 28. (a) 29. (d) 30. (c) 31. (b) 32. (d)
25.
36. (a) 37. (b) 38. (c) 39. (d) 40. (c)
33. (b) 34. (d) 35. (c) '
(d) 42. (d) 43. (c) 44. (c) 45. (a) 46. (d) 47. (b) 48. (c)
41.
(a) 50. (c) 51. (c) 52. (c) 53. (d) 54. (d) 55. (b) 56. (b)
49.
57. (a) 58. (b) 59. (b) 60. (c) 61. (d) 62. (b) 63. (c) 64. (a)

65. (c) 66. (b) 67. (d) 68. (a) 69. (bl 70. (a) 71. (b) 72. (c)

73. {al 74. {dl 75. {cl 76. ' {al

= 1:_~v~1-2: f.!or.e-i:11a~ o_n~ Alt~r-n_~tiv~ i~t~r~ c;;rr~~i' ·

1. (a, b) 2. (a, b, d) 3. (a, b) 4. ,(a, c) 5. (a) 6. (b, C, d)

7. :(a, d) 8. :(a, c) 9. {b, c) 10. (b, d) 11. (b, d) 12. ·(a, c)

13. ,(b,c, d) 14. (a) 15. (b, c) 16. (a, d) 17. (a, d) 18. (a, b, c)

19. (a, C, d) 20. '(c) 21. (b) 22. i(a) 23. (b, C, d) 24. :<c)

25. (a, c) 26. (a, d) 27. ·(a) 28. (c, d) 29. ·(a, b, d) 30. (a, d)
31. ;(d) 32. '(a, c) 33. ,(a, b, c) 34. (i) (a) (ii) (c, d) (iii) ,(b, c,d)
I

www.puucho.com
Anurag Mishra Mechanics 1 with www.puucho.com

=-
r- -~-- ----. -----· --- ---- - -- - -317 -
; WORK AND ENERGY ------- ----------------------------- _________ J
,.__ - - -·- -- -·- ..-- ------·--·- ---- ·---- ---- - - - - -
L~vel-3:C~~~r!he~~io~ B~s~d ~~o~l~~s ,_:>

Passage-1:
1, (a) 2. (b) 3. (cl 4. (c) 5. (a)

Passage-2:
1, (d) 2. (b) 3. (d) 4, (d) 5. (d) 6. (c, d) 7, (d) 8. (b)

Passage-3:
1, (b) 2. (b) 3. (c) 4. (d) 5. (c) 6. (b)

Passage-4:
1. (c) 2. (a) 3. (b) 4. (d)

Passage-5:
1. (c) 2. (c)

Passage-6:
1. (b) 2. (a) 3. (b)

Passage-7:
1. (b) 2. (b) 3. (c)

Passage-a:
1. (b) 2. (c) 3. (b)

=a:Match_i·!'~-~pe_ P_~bi_e_rns~
1. A - Q, B - S, C - R 2. A - R, B - Q, C - P, D - T

3. A - Q, B - P, C - R 4. A - R, B - S, C - P
5. A - P, B - Q, C - Q 6. A - Q, P; B - T; C - R; D - S
7. A - P, Q, R; B - Q, R; C - P, R; D - P, Q, R 8. A - Q, R; B - Q, R; C - P, R; D - P, R, T
9, A - Q; B - P, S; C - P, T; D - R, Q 10, A - Q; B - S; C - R; D - P

11. A-P, Q, R; B-P, Q, S; C-P, Q, S; D-Q, S 12. A - Q; B - P, R; C - Q


13. A - Q, S; B - P, Q; C - P; D - Q, R, S 14. A - R; B - Q; C - S

www.puucho.com
Anurag Mishra Mechanics 1 with www.puucho.com

r3Ts ·. · ·
L_:__.._:_..__:___ _;_:;="'-'-·,_·-·----~·-:_,.~,'":.:.:...~---~-'·-,!

=!!~:-1i9~~
0
7. (c)
0n~. Alter!!~~fs.c:;~ ti.KE = 0 = Work done by boy + Work done by gravity
1. (c) + Work done by spring
As shown in figure F = mg sin 0, Work done by boy = - (Work done by gravity
vertical displacement in time + Work done by spring)
t = vt 8. (b)
Work done = Fvt sin 0 a = k 2 rt 2
= mgvt sin 2 0 2 '
2. (d)
or ~ = k 2 rt 2.
r
Kinetic ~nergy of a body depends upon the reference or ·V=krt
frame and so does the work done. Since two observers
are not accelerated w.r. t. each other so they will Therefore, tangential acceleration, a, = dv = kr
dt
observe same force acting on mass and so they will
observe same acceleration of block. or Tangential force, F, = ma, = mkr
3. (b) Only tangential force does work.
At x 2 , if we displace the body in +vex force acts in -ve Power= F,v = (mkr)(krt)
direction and if we displace the body in - ve x, force or Power = mk 2 ; 2t
acts in +ve direction. So it is a position of stable vdv P
9. (b) a=-=-
equilibrium. dx mv
4. (c) f~ v dv = f: Pdx
2 2

W =U1 -U1 =0-(-~g %)=Mt m( 3 3 m 3 s


Px = - V2 -Vi) ~X = -(V2 -Vi)
3 3P
5. (c)
10. (d) From energy conservation
(a) KE= .!.mv2 = .!.m(!:.c)2 - _F_2t_2 v 2 =u 2 -2gl .... (i)
2 . 2 m 2m
Now since the two velocity vectors
F F2
(b)' P=Fv=F-t=-t· shown in figure are mutually
m m perpendicular, hence the
I ' ,
(c) KE= .!.m(2as) = mas magnitude of change of velocity r :

2 will be given by '. ,-'.


.: 1·

6. (a) It,. ;I = ,l-u-2_+_v_2 -·- .,, .. -~.... ••. !


Speed will be maximum where a = 0
Substituting value of v 2 from
kx=F~x=F/k equal (i)
1 2 1
kx - - k:x: = - mv 2 (by work energy theorem) 2 2
2 2 iti. ;I= ~u +u - 2gl
F2 1 2 1 F2 F
-=-mv +-k-~v=--
k 2 2
2 k .Jmk

www.puucho.com
Anurag Mishra Mechanics 1 with www.puucho.com

11. (c) Because to reach same maximum height their vertical


By work energy theorem, velocity should be same,
AKE = Work done by (gravity + buoyant force + 1 2
- mv 1
resistance force) -2- - = _! = 0.5
2
1
-mv 2 2
o-.!mv = mg x 2- 2mg x2+W 2
2 2
1 18. (a)
W = --mv 2 +mg x2 dU -k k
2 --=-=}U=--
1 dr r2 r
= --x 0.05x 20 2 + 0.05x lOx 2 = -9J
2 mv 2 k 1 2 k
--=-=>-mv2
--
12. (b) r r 2 2r
µmgs= mgh k k -k
Mechanical energy = KE + PE = - - - = -·-.
Since radius of sphete much larger than the 2r r 2r
displacement of particle' so it can be assumed to 19. (d)
perform linear motion. By work energy theorem,
1 1 2
0.0lxmg xs = mg x (1cm) =}S= --cm= lm = w1 + Wgravity
0.01 • 2 mv - 0

13. (c) r
= 200xPQ-mg-
2
r
= 200x r- lOx lOx-
2
=} ½10v 2 ;=150Xl0
=} v = .J300 = 17.3m/s
14. (c) 20. (a)
In first case, mg = kd 1 2 1 kx2
-mv -0=- +µmgx
d= mg 2 2
.
=} k
2 2
.!x 1 xv = Lx 2x 4 + 0.25xlx l0x 4
· 1 2 2mg 2 2
In second case, mgx = - kx =} x = - - = 2d
2 . k · u = ..J52 = 7.2m/s
15. (c) 21. (c)
If original speed of boy is v O then
For vertical motion
2
.!Mv = .![.! M v~] ..... (i)
2 2 2 2
1 2 1M 2
=} h = Wv~ v~
Also -M(v+l) =--v 0 ..... (ii) 2g(W + f) 2g(l+ f/W)
2 2 2
Also for whole motion; work done by gravity is zero.
From eqns. (i) and (ii),
2 1 2 1 2
2fh = -mv 0 --mv
v+1) "l 2 2
( - V - =2=}V= ..fz-1
2 2 ·4fh 4ghf 2vU
Vo-V = - - = - - = - -
u=..f2+1=2.41m/s m W W+f
v 0 = 2u = 4.Sm/s
16. (d) ·
From equation (i),
v = v~(1-.....3L) = v~(W- f)
2
· W+f W+f
1 2 1 2
W =-k(x+y) --kx
2 · 2 v=vo~W-f
1 W+f
=-ry(2x-l:y)
2 . 22. (d)
2 2
17. (a) · mv m ( - c ) (asmvr=c)
We know T=-. -=-
. 450 V2 r r mr
v 1 =V2SIIl = ..f2
=} T oc r- => n = -3 ·
3

www.puucho.com
Anurag Mishra Mechanics 1 with www.puucho.com

~[3_2_0_ _ _ _~--~~-----~~~-~---:~·''"'--'--'--~""---~M_·~_HA_N_l~~S_,l,j
23. (c)
For any incline plane . if the block
slides down the incline plane, work
d ~ friction ~ µmg cos0
..Jx· + y• = µmgs which is
independent of y and 0. So in all the
- case, since µ and 's' are same so loss in PE will be same
· so final KE and so the speed will be same.
24: (c) 31. (b)

mgh = mg(2R)+.!mv 2 Work done = Loss in energy


2 1 2
=mg2R+-mv
_ 2mgR mgR _ Smgk 2
mgh - +-----
2 2 = 0.1 X 10 X 2 X 0.1 + _! X 0.1 X 5 2
. SR 2
h=-=2.SR
2 = 0.2 x 0.1 x 125 = 1.45 (negative)
25. (d) 32. (d)
Since force is always perpendicular to velocity, it will Because kinetic energy does not depends direction of
always do zero work. projection.
26. (b) 33. (b) For spring A:
By energy conservation, we have 1- 2
F = kAx =>E = -kAx
2 .

=>
1 2 =mg
-mv
2
mv
--
. R . 0
sm
2
,
= 2mg sm0
.
~t'!Jv·
I/f-
L_
.... N·
mg.
For spring B:
F,=kBx'=>EB =½kBx'
2
R .
· mv 2 1 p2 l p2
N=--+mgsin0 EB =-kB-=--
R 2 ki 2 kB
N = 3mg sin0 lk 2 x 2 1
= __A_= -(2kAX 2 ) = 2£
27. (b) 2 k~ 2
Hz force on track due to mass m 34. (d)
= N cos0mg sin0cos0 = ~mg sin20 Work done by spring = __! kx 2
2 2 ,
= frictional force Work done on each mass = - .! kx 2
28. (a) 4
We know, Tmax_-Tmin = 6mg_ 35. (c) Potential energy
Since Tmax = Zfmin 36. (a) Total energy
=> Tmin =6mg 3 7. (b) Kinetic energy
29. (d) 38. (c) Potential energy
F=_dU =-2a+_I!_ 39. (d)
dx x3 x2 Since the biock has acceleration both · in vertical,
For equilibrium, F=O horizontal and radial direction. So it is not in
equilibrium.
__!_(b-
x 2
2a) = 0=>x = 2a
X b 40. (c)
30. (c) By conservation of energy,
1 2 1 3
h 1 +h 2 -mv = mgl+-mgl = -mgl
When level become equal then h = 2 2 2
2
I => V = .j3ii.
Work done by gravity change in potential energy of the
water ·column

= 2(Ahp)g!:-[Ah,pg!!J..+Ah2Pgh2]
2 _2 2 .
www.puucho.com
Anurag Mishra Mechanics 1 with www.puucho.com

321'
,.
41. (d) 46. (d)
By conservation of energy, J • ' \
I • ,
-2mgx+.!.kx 2 = 0 => X = 4mg 'i ~
0 ••• i
' '
2 k ; PE~ O····· ...,. . : ,
42. (d) I
). --· ---··:.- ·- ' '
';
By conservation of energy, L____ ·--· _.-·· -- - I

2
lk(Xm)
- - .12m2
+- v +-mv1 2 - 2 mg-=
Xm O By conservation of energy:
2 2 2 2 2 2
O= ½mv -mg(Rcos9- ~)
_!.kx! +~mv 2 -mgxm =0
8 2 KE= mgR( cosa-½)
_!.kl6m2g2 +~mv2-mg.4mg =0
8 k2 2 k p = dKE = mgR(-sin9 dB)
dt dt
~mv 2 = mgRsin9co
2
. 1
43. (c) =lxlOxlx-xco=Sco
2
L
(i) = Mg - = 0.SMgL 47. (b)
2
(ii) =Mg!:_ = O.SMgL Extension in spring to lift the lower block x = mg
2 . k
r-·-- - - - -~-... -- " "'1
(iii) = Mg 2R = 2Mg !:_ = 2MgL = 0.2MgL j +mg!
1t 1t 1t 7t2
' k:
(iv) = _!. MgL = 0.4MgL
lt2

(v) =Mg( R-2:) I


! -·-- -·- -·---- _J
= (1t-Z)MgR = 1t-Z MgL
7t lt2
By conservation of energy,
2
1 2 = mg (mg)
-mgxo +2kxo k +2k k
1 (mg)
1 14
= · MgL = 0.114MgL
2
7t 1 2 3m2g2
(v) < (iii) < (iv) < (i)= (ii) 2kxo -mgxo = 2k
44. (c) 3m2g2
Fv=P
kx5 - 2mgx0 - --"-- =O
k
p
v=- 2mg ±~4m2g2 + 12m2g2
F Xo = 2k
for maximum v, F should be minimum which is
2mg ± 4mg 3mg .
equal µmg just to drag the block. Xo = =--
p 2k k
vmax = µmg 48. (c)
Errata: The mass of the lower block should be m.
45. (a) F=~ By conservation of energy:
V - - - --~- --

dv =-o: => Jmvdv =I o:dt , D


=> m-
2
dt V
!ht __ ····-··· .. tmgi
·_
,,,, '
--~--''

( m~ ) = o:t

Af<E = Mt = Work done

www.puucho.com
Anurag Mishra Mechanics 1 with www.puucho.com

0 a
tan-=-
2 g .

e =2tan-1 (i) ma
.
'---==·· -~
49. (a) .52. (cl
Conserving-energy for any sphere_
' 1 . dW =F. ds where ds =dxi + dyj + dzK
.... .... A A ,._•

0=-mg(R-Rcos60°)+-mv 2 · .... A A

. . 2 ' and F = K(yi + rj)


. '2 • .
. '!!_=gR==>v=..{gR dW = K(ydx+ xdy) = -Kd(:xy)
-. 2 2
As shown in figure, ·for tube. 2N cos 60° = Mg (when' w·= r" dW, = -Kr"
Jqo
0 0

Jqo d(:xy) = -K[:xy]~g


it just lifts off) · '
· W=~Ka 2
~ - µ ~,-~·
-
... 53. (d) U(x) = k(l- e-x )
2

,·'r.::: ·: ~. mt .·_ :~-


- • J • • • ' ,,

It is an exponentially increasing graph of potential


energy (U) with x 2 • Therefore, U versus x graph will be

.I("., as shown.
From the graph it is clear that at
origin.
Potential ·energy U is minimum
(therefore, kinetic energy will
, N=Mg . ... (i)
be maximum) and force acting
· · mv 2
. For any sphere N + mg cos 60° = - - on the particle is also zero because F = - : = (slope
. R
N +mg·,; mg~ N = mg ... (ii) of U - x graph) = 0
2 2 Therefore, origin is the stable equilibrium position.
From eqris. (i) and (ii), Hence, particle will oscillate simple harmonically
mg. . m about x = 0 for small displacements.· Therefore,
-·=Mg==>-=2
2 M correct option is (d).
(a), (b) and (c) options are wrong due to following
reasons:
a At equilib'
() ..
num pos!Uon dx =o·1.e., slope
F =-dU

U - x graph should be zero and from the graph we can


see that slope is zero at x = 0 and x = ± =.
Now among these equilibriums stable equilibrium
_position is that where U is minimum (Here x =0).
Unstable equilibrium position is that where U is
maximum (Here none).
Neutral equilibrium position is that where U is
constant (Here x =± =).
Therefore, option (a) _is wrong
(b)For any infinite non - zero value of x, force is
directed towards the origin because origin is in stable
equilibrium position. Therefore, option (b) ·is
51. Cc) incorrect.
By work · energy theorem from trolley (c) At origin, potential energy is minimum, hence .
maLsin0 = mgL(l- cos0) asin0 = g2sin 2 0/2 kinetic energy will be maximum. Therefore, option
(c). is also wrong.

www.puucho.com
Anurag Mishra Mechanics 1 with www.puucho.com

~KAND ENERGY 3231


-------~--~-- -~--------~----...~
54. (d) F=-dU ds = vdt
dx f
s ~ rv 2 dt =- s ~ r312
dU=-F.dx
or U(x) = -J; (-kx + ax 3)dx 3. (a, b)
Whenever the displacement is towards the relaxed
kx2 ax4 position, the spring does positive work.
U(x:r=--- 4. (a, c)
2 4

U(x) = 0 at x = 0 and x =
'fik
f-;i" a= L;v = Lt
m m
2
J t
>~
P = fv = - [Option a]
U(x) = negative for x m

From the given function we can see that F = 0 at x = 0 __


F- lmv2 __ m __
f2t2 [O. ]
2 2 m2 . ptton c
i.e., slope ofU - x graph is zero at x = 0. Therefore, the
most appropriate option is (d). 5. (a)
55. (b)
Let x be the maximum extension of
the spring. From conservation of
k For mass m2 to· move, Kx = µm 2g
mechanical energy:
Decrease in gravitational potential By work energy theorem on m1, Px - µm 1gx - 2Kx = 0
energy = Increase in elastic potential 1
P =µm,g+-Kx= 0
energy 2
Mg x = ~kx 2
2
p = µm,g + ~zg = µg( m, + J
~2

2Mg
or x=-- 6. (b, c, d)
k
Work done by gravity and tension force is equal to
56. (b) 20J.
Gravitational field is a conservative force field. In a 7. (a, d)
conservative force work done is path independent. v2
- = kt 2 =; v 2 = kRt
2
W 1 =W2 _=W3
R
57. (a)
1 2
KE=-mkRt
From F = _du 2
dx dKE
f~CxJ dU = -J; Fdx =J; (kx)dx P=-=mkRt
dt

U(x)=--
kx2 Paverage = -If,O mkRtdt = -
mkRt
-
t 2
2
8. (a, c)
As U(O) =0 -+ -+
Therefore, the correct option is (a). dw=f.'ds
Since body is hauled slowly, so
=7:;el_::;~,~!~~:n,,?~ A~ma!i".!'Fj;~~-;;~~ f = mg sine+µmg case
1. (a, b)
f
W = (mg sine+ µmg case) ds

Final KE will be larger than initial KE. Larger the initial


KE larger will be the final KE. = f mgds sine+ f µmgds case ds
2. (a, b, d)
mdv
= f mgdy +f µmgdx
Fv=P=;--v=P
dt = mgh+µmgL
mvdv =Pdt
v2 = 2Pt =;v~J2Pt
m m
www.puucho.com
Anurag Mishra Mechanics 1 with www.puucho.com

9. (b, c) 13. (b, c, d)


For no sliding rimg =.µ(ri - l)mg ,---'-·.,.----,
'. .........x_ .,.._ '!
Resultant force on particle = F - ,,:ng
1~··,
=> µ =.c3_
ri-1
If x length is remaining on table,
work done by friction force for
; , •,' 01
j· . ,", 'qi
1.· · · · oJ
I.· 0-1
= (2-az)mg-mg = (1-az)mg
mvdv
- - = (1-az)mg
dz
.
,·f~-·'
'
l _._'_
;;; .
' F
':'

displacement qf ,ch: will be:.


-dW=µ-xgdx
. l
m => v: =(z-~ J 2 h- •"' fug

W = rc~-l) µmg xdx = µmg (fl -1) 2 12


(a) For maximum height v =0.
2
Jo 1 21 => Z=H=-
a
µmg(ri -1) 2 1
= Velocity at
H 1
2 -=-
2 a
10. (b, d)
(b) If two blocks are sliding over eacl, other, kinetic (b)
friction does +ve work on one end. negative are
otheL ·
(c)
(d) Work energy theorem is valid from non-inertial
frame and we have to consider the work
done
- by the pseudofo!ce
\ . . . also.
11. (b, 4),
For KE to increase power should be positive , 14. (a)
_, _, !fit slides down with constant velocity,fcirce of friction
·f.v>O=>e < 90° = mg sine
P = .J2Km => p ~ -.JK For motion up the plane a= -2g sine
u2
F = -dU = (-'-2a
dr r3
+~) r2
02 =u 2 - 2(2g sin0)S => S
'Jg sine.
16. (a, d)
2a b 2a
For l= 0, · , ,. 2 =; :-:g =>.r = -b To. complete the circle tension at
. r r . top point should just become zero
2
d U_ +6a _·2b _ 2 (3a
2 4 3 3
-o) mv 2
for this - - = mg
dr r r r r l
For
. ' ' 2a.
r=-, => v=..Jii
' . b
2· .
d U =2(~axb _b)=2.!?.=~(+ve) 17. (a, d)
2 3 3 2
· dr r 2a r 2 r kA > kB
So U is minimum. lk 2 • 1 2
.wA =- Ax ;WB =-kBx
So it is a positjon of stable (steady) equilibriu!". 2 · 2
· '. dF -d 2u => WA >WB
For maxnnum - = - - = 0
For same force;
dr dr 2 ·
. ·F
=> '2(3a_i,)=o.
3
=>r=3a F=kAXA =>XA = -
kA
r r . b

r.
3 3
· 2a b 2ab b
F=---+--=---+- 3 2
(3bar (3ba 27a . 9a

b3 2b' b3
= 9a 2 - 27a = 27a 2
2

www.puucho.com
Anurag Mishra Mechanics 1 with www.puucho.com

IWORK AND ENERGY, _·


18. (a, b, c) 23. (b, c, d)
Work done by F1 = 20 x 6-J2 = 12()-J2J At stable equilibrium position U is minimum, at
Work done by F2 = 30 x 6 = 180J unstable It is maximum and at neutral position it is
21tR constant.
Work done by F3 = 15 x - = 45itJ

:-----·-·1
4 24. (c)
19. (a, c, d)
-du
By conservation of energy,
1tR 1
mgR = 0-2mg-+-(3m)v.
2
. .,
(a) F =-=-3 2 2 ·
X cJx
-du _, • • 3;,,v 2 ml
- - = mgR(l + it)
Fy = - = -4=> F = -(3 i+4j)N
dy 2 '
2(1+it)gR ____-_2'!'J
I
(c) For particle to cross y-axis x = 0 v=
1 2 3
X =vxt +-axt
2 ' 25. (a,c)
-,6=0-.!.x3t =>t=2sec 2 Frictional force = ~ x 200 = 60 N
2 10
For resultant velocity Work done by ladder on boy is zero
-t ,.. ... -+ because while ladder applies force
v = 0-(3 i+4j) x 2 =>\vi= lOm/s on boy, his point of application does
(d) 6x = 0 - .!_ X 3 X 12 = -1.5 not move.
2 Work done by slide = Work done by friction
1 2 = -60 X 10 = - 600 J
f.y = -0- - X 4 X1 = -2
2 27. (a)
Co-ordinate = (6 -LS, 4 - 2) = (4.5, 2) The statement of work energy theorem.
20. (c) ~8_. (c, d)
Since speed of mass is zero at C i O ---1 Kinetic energy will be constant because, the f~rce w/11
and B, so they must lie on same
horizontal surface.
AP AO-PO
cos0=-=---
PB PB
Lcosa-l
=-~--
i- ~ ~LJ-'
L~--~-- C
do zero work. This is the case of uniform circular
motion.
29. (a, b, d)
Since work is done against gravity which is a
conservative force, do work done is independent of
L-l path followed.
21. (b) 30. (a, d)
J
W = F.ds = J(y l+ xj)-(dx l+dyj) For mass m to complete the vertical circle, the string
becomes slack at highest point.
= J(ydx+ xdy) = Jd(xy) 31. (d) r -· - - - - -

f(l,I) d(xy) = (xy )at = 1 J


= J((\O)
W
T - mg sin0 = mv2 :·rs::-/,° PE=O
r I
1 •
T · .,.
-
-.
22. (a)
=> .!.mv 2 =mgrsin0 l ''. · · .mgsin8 !
We know Tmin = 6mg
Tmax - 2 . [8 mg I
~ -·~-__:
and Tmax = 2Tmin (given)
=> T = 3mg sine
Tmin = 6mg, Tmax = 12mg maximum value of T = 3mg and given that string can
Since tension is max at lowest points with stand a maximum load of 4mg. ·
2
mv :. it will not break.
T=mg+--
L
2
mv
12mg =mg+L

=> V = ~llgL

www.puucho.com
-
'
Anurag Mishra Mechanics 1 with www.puucho.com


326
'--~;;:;;:;;:::::::;;::;:::::;::;;;:;:::...---~----~---,--,
Leve~~2 sompreh~~n Based Prob~em~

Passage-1
MECHANICS-I j
~·------"-'-----' •..
2. (b) , U will be minimum at x
3. (d)
Kmax =E-U = 49-20= 29J
= 2m so, Umin = 20J

1. (a) .!mv 2 =29=>v=.J5Bm/s


2
By work energy theorem
4, (d) zero
mg(2x)- 2µmgx-.! kx 2 =O 6. (c, d)
2
20x-16x- 50x 2 = 0 Particle will move between the points where KE
becomes zero or its PE is equal to total energy.
x= 0.08 = 8cm
Thus, 49 = 20 + (x- 2) 2
or, (x-2) 2 = 29
X=2±,,/29
2. (b) 8. (b)
Gravity does work only on the hanging mass and it is au ·
F = - - = -2(2-x)
equal to = mg(2x) = 10 x 0.16 = 1.6 J dx
3. (c) For F=0,x=2m
Work done by spring force = energy stored in it Since at x = 2m, PE is minimum so it is a position of
1 2 1 stable equilibrium.
= -kx = - X lOOx (0.08) 2
2 2
= 0.0064 X 100 = 0_32 J
2
=!!a!ih}~~}vpe ~~~lem~
4. (c) 1. Work done by conservative forces is negative of
-------~7 change in potential energy.
120~81 2. v~ = u1-2ghAB = (9gR)-(2gR) = 7gR
I ~~
As shown in figure f, + 8 = 20 => f, = 12 N
Vn = ~7gR
mv 2
fmax = 0.8 X 20 = 16N Further, TB = __ B = 7mg
R
s. (a)
At any instant where 2m has been displaced by x.
Again, v2 = u1 - 2ghAc
So mg - T = 2ma .... (i) = (9gR)- 2g(2R) = SgR
zr - kx - 2µmg = 2ma .... (ii) Ve =~5gR
From eqns. (i) and (ii), we have mv 2
Further, Tc+mg =--c
R

¢T

mg
2
T

• EJ.:: icx
.....,.
a
2µmg
Tc =4mg
3. From x = 2 to x = 4, force is positive and displacement
is also positive. Hence, the work done is positive .
Similar logic can be applied to other parts also.
4. A is the point of stable equilibrium, so potential energy
2mg - kx- 2µmg = 2ma
For max velocity is minimum. Similarly, point C is the unstable
a=O equilibrium position, where potential energy should
==> 2mg(l-µ)
X;:; _ _ __ 20 X 0.2 = 0.0 4 = 4cm be maximum.
k 100 5. p ~t
Alternate method:
W =f Pdt =f atdt
Velocity blocks will be maximum when their
acceleration become zero. From mass 2m or W oct 2
kx+l6= 20 => X = 4cm Since, work done is equal to change is KE
Passage-2 Hence, v 2 oc t 2 or v oc: t
1. (d) Further, v = -ds ds
-~t
at x = Sm= 20 + (5 - 2) = 29J2 dt dt
Total Energy = 20 + 29 = 49 J or ds~tdt
or soc t 2 (by integration)
www.puucho.com
Anurag Mishra Mechanics 1 with www.puucho.com

,_w_o_RK'-.
r A_N_li_EN_ER_G'""~--.C---''----------~------'",.-."-·'-"---·-·:_·',-'.\_,,~;;~.,...:;::_0. r,~;'..3,~I;]
6. Angle between net force and the string can never be 14.
obtuse. It is 90° at A, 0° at B and acute in between. A -;-'~'..O ;
13. (A) u = Sm/s N• _.-......,:
-· 0 •.
2
v =u -2gR(l- cos0)
2 • ,._.T :
···s;i / ... ~-·-~-;~i- ;.~.j ··--~\
U < Umm, ...•,.
...... ,,, ..
V = 0 at 0 = COS-l(¼) ..---a ~-------~r:~~A~~
.. YB'·
mg ,,. IT!lil l.~-
(BJ u=-J20m/s<umin,
1-·- At 'If: v = 0,. No centripetal ~cceleration
So acceleration is downward (Due to mg)
I At 'B' : T and mg both are vertical so acceleration is
Ii vertically upward (centripetal acceleration)
mv 2
L.---
At 'C' : T - mg cos8 =- - ... (1)
(C) u =6m/s < umin l

f7 1::-·--1 mg Zcos8 =.!mv 2


2
••• (2)

!( : mg,; From eqns. (1) and (2),


Ir : s··,
. ms1
1 T - mg cos8 = 2mg cos8
'- T =3mg cos8
After leaving the cylinder it will follow projectile path. IfT cos8 =mg then vertical component of acceleration
(D) · will become zero.

(3mg cos8) cos8 =mg


cos8=J_
./3 ~tan8=-J2
. .
So at 8 = tan-1 ( .,/2) acceleration has only horizontal
.!mu 2 =.!mv 2 + mg R (1- cos8)
2 2 component.
v=O

www.puucho.com
Anurag Mishra Mechanics 1 with www.puucho.com

' ·-
I
\
-
I
I

IMPULSE AND MOMENTUM,'


.- ,-'

IMPULSE i.e., Initial momentum of system = Final momentum of


When a force Facts on a body for a very short interval, it system
is called an impulsive force. The effect of impulse is which is the principle of conservation of
characterized by a vector quantity represented by the momentum. We may state it as:
-->
symbol J. If the net external force exerted on a system is zero
-->
-->
Impulse of force F exerted on an object is (:E Fext = O), then the momentum of the system is conserved.
When no force acts on a system from outside the system, we
-->
J =f,1' -->
Fdt say that such a system is isolated. The internal forces
' exerted by one part of the system on another part have no
Principle of Impulse and Momentum
effect on the linear momentum of the system.
Newton's second law may be expressed in the form ----- --- --------- --7
d fF
I.
--> -->
Fext =-(mv)
dt - •••••••••••••,,.-bsystedm
--> --> : : oun ary,
or Fdt =d(mv) .: :'
i
t2 --+ --+ --+ -: .·-- - _____ j
or f<1 Fext dt = mv2- mv1 T,
"i
On rearranging the above equation, we obtain
... (1)
I::.............
: '

--> ll (a)
, '
,
"
(b)
Initial momentum + impulse of force (Fexr) ,
L ______ --------- -
Fig. 4:1
_____ . ----·----------·
= final momentum The classification of _a force as internal or external
For a system of particles we may add vectorially the depends on the choice of system. For example, consider a
momenta of all the particles and impulse of forces acting on system of Fig. 4.1 (a), consisting of masses m1 and m 2 and
the particles. We may write impulse momentum equation as -->
the string that connects them. The force T1 exerted by the
--> -->
:Em v 1+ :E Imp 1__, 2 = :Em v 2 ... (2) -->
string on m1 is an internal force; so is the force T2 exerted by
Conservation of Momentum --> --> -->
--> the string on m2 . The force F, m1 g and m2 g are external
If F.xr =0 for a system, then we may write impulse forces. These forces are exerted by agents that are outside
momentum equation as • -->
the system; e.g., force m1 g is exerted on m1 • and thus on the
--> -->
:Emv 1 =:Emv 2

www.puucho.com
Anurag Mishra Mechanics 1 with www.puucho.com

IMPULSE AND MOMENTUM 3297



-,
system as a whole- by the earth, which lies outside the dP- ·
1
system boundary. -- = sum of forces acting on particles j
dt
Now consider the system consisting ofm1 only [Fig. 4.1 -, -,
-, -,
(b)]. The forces F and m1 g are external forces as they are Fj = F j,int + F j,ext
-,
shown in Fig. 4.1 (a), but Tis an external force as well The forces exerted by particles within the system may be
because it is exerted on the system by the string, which lies written as:
-, N-,
outside the system boundary. Fj. int = L, Fj,,
Conservation of Momentum for a two Particle fal
i#j
System -,
Consider two particles where Fj, . stands for the force exerted on particle
isolated from surroundings, number _j by particle number i. The term with i = j is not
that interact with each other. included in the sum, because a particle cannot exert a force
From Newton"s third law, both on itself.
the particles must exert equal The rate at which forces change the· system's total
and opposite forces on each momentum:
other, i.e.,
-,
L F internal == 0
-, -,
dP
--,= - I,·Pi
dt
d
dt
( ) N-,

j=l
F21 + F12 = 0
-, -,
dP1 dP2 d -, -,
or -+ - = - ( P1 +P2)=0
dt dt dt
-, -,
or P1 + P2 = constant.
-,
F21 is force exerted on particle 2 by particle 1, it will
-, = sum of external forces acting on particles
change momentum of particle 2. Similarly F,_ 2 is force + sum of internal forces acting on particles
exerted on particle 1 by particle 2, it will change momentum The first term is the total external force :
of particle 1. But momentum of system, that includes -, N -,
particle 1 and particle 2 will be conserved if the basic
-,
Fen = L,Fj,en
j=l
condition L F ext = 0 is met. -,
dP -, -
Formal Proof of Momentum Conservation Then, - = Fext + sum of internal forces
We model a system as a dt acting on particles
large number of particles [Fig.
4.2 (b)], labeled with
numbers, 1,2,3, ... ,N, whereN
is the total number of particles
in the system. The particles o •3
[){~
l
·j·.
'",
The sum of internal forces is the vector sum of every
force acting on every particle. In accordance with Newton's
third law these forces occur in pairs, and since we are
summing over all the particles in the system, we are adding
and the forces they exert obey
Newton's laws. The particle
l" '.
Flg.4.2 (b) ' . ;
both forces in each pair. Thus the sum of internal forces,_
equals zero.
-,
with number j has mass mj, velocity 'vj, and momentum dP --+ --+ --+
-=F.;.,+Fint =Fext+O.
-, -, dt
'f'j = m j ,; j. The total force acting on particle j is Fj, which is We have shown that the rate of change of a system's
the sum of forces exerted by objects outside the system, total momentum equals the total external force acting on the
-, .
Fj, ext , and forces exerted by particles within the system, system:
-,
-, dP -,
Fj, int. Then Newton's second law, applied to an arbitrary -=Fext.
particle number j, has the form : dt

www.puucho.com
Anurag Mishra Mechanics 1 with www.puucho.com

!330,

I· Concept: For an isolated system, there is no extern~!, .


!force.and.the$y~tem's tota/.11wmenwm cannot change, When
jextemal forces do act on a system, ·any change of its
momentum resultsfrom_an. impulse.delivered by the extern~!)
!
!forces. · ,
ruustration 1.
Consider two toy
caru equipped with
spring bumpers. ,The
· ..,Befpre .. I '•, •. ,
.. =·.
•: '' . • • ..I•
'"' ,.,. _After~· ,..
caru are tied together
with a string, while tlie
' '" :-: '·,.I 1, •

'-------'----·
Flg1'4:4 >
springs are compressed, -+ -+
When the string is cut, ma Vat+mp Vpi = ma -+Va/+ mp -+Vpf
the carts are pushed _, _,
apart so that they move 0+0= ma VaJ+mp Vpf
apart -m opposite . _,
_, mpVpf
directions. Friction in the or Va/=
wheels and between ma
wheel and ground is The negative sign indicates that the astronaut moves in
negligible. the direction opposite. to the direction of motion of the
Each cart is acted on pencil.
I . •
by gravitational force Relative Velocity and the Conservation of Momentum
and normal reaction in
I
vertical direction; these - - - - Flg.4.3
-~>
Consider a rifle that fires a bullet with the speed of 500
m/ s when the rifle is fixed rigidly. If the rifle is free to recoil,
external forces add to
the bullet leaves the barrel of the rifle with a speed of
.ln
zero . horizontal direction the spring exerts force on each
500 m/s relative to the backward recoiling rifl~- In this case
cart. That is why we cannot choose cart A or cart B as our
$Y5tem · for application of conservation of momentum the velocifY. of the bullet relative to. ground is less than 500
principle. · If the $Y5tem includes both the caru, then m/ s. While applying conservation of momentum equation,
_, . we must specify all velocities relative to a single,
I: Fext = 0, because the spring force is internal to the system. inertial coordinate system.
PAf = 0, PBi = 0, When assign subscript R for rifle, B for 'bullet and E for
From principle of conservation of momentum, Earth, we have-
_, _, _, _, _,
VBR =VaE-VRE ·' ": .. (1)
P1 = P;
Rearranging eqn. (1), we get
or. or -+ '-+ -+
= VBR+VRE VBE ... (2)
If the bullet is fired with velocity v relative ,to the rifle
which shows that the final velocity of cart B is opposite
and the rifle recoils with velocity V relative to Earth,_ then the
to the velocity of cart A. ·
absolute velocity of the bullet (relative to Earth) is the vector
ruustration 2. Consider ·an astronaut trapped in
space, isolated from surrounding. He can reach his sum of the vectors ~BR and ,IRE as shown by eqn. (2).
,
spacecraft if some velocity is gained by him somehow. . . . M'' s.
tm · .
Suddenly he realises that he has got a small pencil in his
pocket. How can he acquire velocity (momentum) ?
~u alt H I~___.
Bullet
Rifle
We take the system to consist of the astronaut and the
pencil as shown in Fig. 4.4. We assign the positive direction •· Fig:"4;5
of the x-axis to be the direction of throw. The gravitational
force acts on the $Y5tem, which indeed is external force. Thus, l,; 8 £1= v-V
However, this force is directed along the y-axis, it will not From the Jaw of conservation of mo~entum, we have
change momentum along x-axis. we· can thus apply the
0= m(v-V)-MV
conservation of momentum to this system. What happens to
the force exerted by the astronaut on the pencil while or V=~
throwing it? M+m
www.puucho.com
Anurag Mishra Mechanics 1 with www.puucho.com

:::IM::_:PU~LS:.:E:.:::A:::.:ND:..:M:::.:O:::M:;:EN::.:T.:::UM:::......--.:.-'-.L........:.___ _ _ _ _'----_ _ _._L:__._ _ _-'-'_....:...._ _ _


t..1 -------'"~1j
illustration 3. A man Thus SCM = 0, since Fext =0 & uCM = 0
of mass m standing at the .. t:-"ral ·: m(-xA) + 2mxB =0
a
end of trolley of mass M
jumps off with a velocity ure1
nL.i XA =2
(a) ' XB
relative to final state· of
trolley. The recoil velocity of .. MI~ _1-"1 XB = Xo/6
the trolley may be obtained
by applying momentum
conservation.
If velocities of the man
I =· ,., •
Flg.4.6
XA = Xo/3
(b)Conserving momentum -mv A + 2mv B = 0
Writing work energy eq. for the system
kA (1/2) mv~ _ 2
and trolley are assumed with
respect to ground as v 1 and v 2 , respectively.
kB (1/2) 2mv~ .- 1
1 2 1 2 1 2
P, =0 (c) -kx0 =-mAvA +-mBvB
P1 = mv 1 -Mv 2 2 2 2
Form concept of relative velocity, we get VA =2VB
1 2 1 2 1 2
V1 +V2 = urel -kx0 =-mA(2vB) +-mBvB
• 2 2 2
Applying momentum conservati~n, we get
121 2l2m2
P1 =P, -kx =-mx4vB +-x xvB
2 0 2 2
· mv 1 -Mv 2 = 0 work done on B by spring is charge in K.E. of B
m(u,e1 -v 2 )-Mv 2 = 0
murel
1 2
-x2mvB = -
kx5
or V2=-- 2 6
m+M
b:Ecxp,in:el!,~.~
~$;(~~~~ == ~·=L=)~
lJn the Fig. 4E.l (a) shown thesp_ri_n_g_is_c-om-pressed by' ; 0' a~d] i:..·small cube ~fmas; m slides down ;~{rcular path of radius RI
1 cut into a large block of mass M. M rests on a table and both 1
released. '.lwo blocks 'A' and 'Bl.. of masses 'm' and '2m'i blocks move without friction. The blocks initially are at rest

kept on a smooth hot~iind


respectively are attached at the en~ of the spring. Blocks are
released.
and m starts from the,top of the path [see Fig. 4E.2(a)], Find
the velocity v of the cu~g.s_it leQY.<!§._lhe block. ·

c.~~:_4E.1~ ; l~I
I
( a) Find displacement of block A by the time compression of.
the spring /s reduced to x0 / 2.
L_______ll_F_ig_.4_E·~------- ___ j
(b) Find the ratio of KE 4 blocks A & B by the time Solution: Let velocity of
I1 compression of the spring reduced to x 0 /2. the block is u when the cube
(c) Find work done by force of spring on blockB when spring reaches its bottom to leave with
~aches natural length, _ _~ - - - - - - - - - ' velocity v. By conservation of
Solution: (a) Let xA and xB denote displacement of momentum in horizontal
block A and B respectively. . directions.
XA +xB = Xo/2 0= mv+Mu
·mv
U=--
M
By conservation of energy, mgR = -1 mv 2 + -Mu
1 2
2 2
= .!. mv 2 + .!.M (~~v) 2

HJ
mgR
2 2 M

on solving, we get v=

In the absence of Fextemal CM remains fixed.


www.puucho.com
Anurag Mishra Mechanics 1 with www.puucho.com

1332 MECHANl~S;I j

r-----·-·- ------- ~--··----·----: ,- ---- . ------' ...... --··-- ----- ------,


i71vo blocks of masses m and 2m are connected by a relaxedi ;Fig 4E.5.shows a small block. of m.as.s m. placed o_ver. a.wedge of)
/spring with·a;Jorce constant' k. The blocks rest on a smooth' 1
mass M. _The block is pushed ·on the wedge at, a -speed v.
ihorizontaltable. Att = 0, the block on the left is given a sharp '.Assume that all. the. swfaces are fri
. . 'ction. less, inin.·al ve/ocity ,of.I'.
!irhpulse "]"_towards the right, a~d_the blocks begin to slide
lcilong the. table_ (see Fig. 4E.3). ,Find the maximum ilv.=t, - - - -~
jblo_ck is horizontal. Wedge has gradual curvature ,, . ,' ·
~-·- !
/compression in the spring.
i. . -' r·~1 . .

!i l II
j Fig. 4E.3 i ,. • 11
---------_..,__..._..____...____ ==----,"""-",;;....,"'"''°"'-' - - - - - - - ~ i!'
h i'
J :l:vx '
Solution: -=Vo I .
m
II'/ M ----, M
At maximum compression both the blocks will move
with same velocity v,
ii -., .... ·--·-' ____ ,__ .. '
· mv 0 = (3m)v, lL
i . -----:---~·"--;- . -
Fig.4E.5
. ' ---- ' --·---,-_--'
.!mv 02 =.!kx 2 +.!3mv 2 (a) Find the speed of the Sl!,l~ll~r, mass when it brea]<s, off the
2 2 2 C
I : larger mass at height h. ".,_' , . · :. I>;' ·: :, "
'(b) Find'the maximum height'(from:the ground) that smalleq
X=Vo~¾7 . /__ .mass -ascends. ____" -____ ,___ y: __ · + · ' ,·. ,c __ ·.' ·~--. :$ ·-J
.. . =x Solution: (a) In the absence of any external force
Maximum compress10n =-J~m
-. acting on the system in x-direction, its momentum remain
m 3k
constant and velocity of C.M. remain constant. The velocity
fi.E2<@m~J.~If~;;> of C.M. is
mv+M,xO mv

~o:o~Lr~~in?.-~i;u~-- ~:~tsv~~~~rz:,~oj~::s:.~.~~n~j
1

height h that car, alsp move along the plane. The pui:k 'begins
VcM·=
m+M
=--=vx
m+M
At the instant block is on the vertical part of wedge, both
the wedge and block have common velocity in x:direction.
to slide up the'"hill" [see Fig. 4E.4(a)]. lf the hill is initially at
From work-energy theorem, we get Wgravity = AKE
rest, iyhat value of v provides for the maximum subsequent
;velocity u of the "hill"_? Assume that all swfaces' are -mgh= _!(v 2 +v 2 ) + _!Mv 2 _.!,;,v 2
2 X y 2 2

·r-~, -~---
--- -i
X
kinetic energy of block kinetic energy of wedge
'.lftionless._ ·. ' ' ' '.': ·j- .!mv 2 =.!(v 2 +v 2 )+.!Mv 2 +mgh
; I h , - · 2 2 X y 2 X
..;._) [ffi1 ~ l '.,·

_i__ ~ I Fig. 4E.4 (a) :_'__.__ :._ Where v Y' vertical component of velocity of smaller
block at height h.
Solution: For the maximum subsequentvelocityu, the
puck comes to state of relative rest at the top of hill. We have vx =(~)
m+M
~ - m 'i
II JJ,.·.,•·.· i 121 221 mv
2
--mv =-m(v +v )+-M - - +mgh
II fiii1 ~
. . :
I
2 2 x Y 2 ( m+M ) .

L..~--- ____'.~''!: ~:-~~~ : Vx2 +vy2 = [(M


---
2
+Mm+m
--~v
2
) 2- 2gh]
· (M+m) 2 ·
V
mv= 4mu => U=- 1/2
I
1 2 1 2
4
and v'=,;v 2 +v 2
x Y
= [(M2·+Mm+m2)i,2
(M +m)2
2gh
]
-mv =-4mu +mgh
2 2
2 2
(b) After breaking off we can apply work-energy
mv 2mv ·
--=--+mgh or v -= ~Bgh theorem.
2 16 3 Let hmax is the maximum height, v Y = 0 ,
[Revisit the problem after studying C.M. and solve the 1 2 1 2 1 2
problem in C.M. frame.] --mv = --mv x +-Mv x + mgh max
2 2 2
www.puucho.com
Anurag Mishra Mechanics 1 with www.puucho.com

-- - - -- - ---
i.IMPULSE AND MOMENTUM _______ _ 333:
mv
where vx = - - -
m+M

A smooth wedge of mass M rests on a smooth horizontal: \,li, - -·· ~---·-·~---·~

surface. A block of mass m is projected from its lowermost Fig. 4E,7 (a)
point with velocity v 0 [see Fig. 4E.6 (a)]. What is the'
maximum height reached by_the block? Solution: As long as the block moves from A to B, the
reaction on the wedge presses it to the wall. When the block
reaches the lowermost position, its velocity from energy
conservation is
V = ~2gr
When the block moves along the right half of the wedge,
.M
during its upward journey as well as downward journey the
Fig. 4E,6 (a)
reaction of the block on
the wedge is towards
Solution: At the right as shown in Fig.
instant the block breaks 4E.7(b). Therefore N'
N""
contact with the wedge, they during the entire motion
of the block from B to C

i
have common x-component B
of velocity. In addition, the and C to B, the wedge is
block has a vertical accelerated towards Fig. 4E.7 (b)
h
component of velocity. Due right. Thus to find the
Initial position maximum velocity attained by the wedge at the instant
to this vertical component,
the block rises upwards till when the block passes separated from the wall,
the vertical component of P, =Pt
velocity vanishes. Fig. 4E,6 (b) mz~2gr = m1v1 + ffi2V2 ... (1)
From momentum From energy conservation, E, = E1
conservation along x-axis, 2 2
m1V1 m2V2
mv 0 =(m+M)v ... (1) mzgr=--+-- ... (2)
mv 0
2 2
or v= ... (2) On solving eqns. (1) and (2) simultaneously, we obtain
(m+M) two solutions
From energy conservation between initial and final
V1 = 0, Vz = ~2gr
positions of block,
1 2 1 2 and V1 -
_ 2m 2 ~2gr
-mv 0 =-(m+M)v +mgh ... (3)
2 2 m1 +m2

or 1 2 1( m ) 2
2
_ m2 -m 1 l2gi
-mv 0 =- - - - v 0 +mgh Vz - "y4:,t
2 2 m+M m1 + mz
The first Solution corresponds to the instant when the
or
h=~[m:M] block reaches for the first time at point B. At this instant the
block moves with velocity v 2 and the wedge is at rest. The
l_E~g;~:Bl~~J7;,> second Solution corresponds to the instant when the block
has the maximum velocity
A wedge of mass m 1 with its upper surface hemispherical in 2m 2 ~2gr
shape, as shown in Fig. 4E. 7 (a), rests on a smooth horizontal. (v1lmax
m1 + m2
surface near thewall. A small block of mass m2 slides without
friction on the hemispherical surface of the wedge. What is the
'maxi.mum velq_city gttg/_ned.by the w~dge?

www.puucho.com
Anurag Mishra Mechanics 1 with www.puucho.com

!334 ' , ·~

IA ball B is suspended from a string oflength l attached. to- a


cart A, which rn'ay roll on 'a frictionless surface. Initiall,y .tlie
cart is at restandthe ball is given a horizontal velocityv 0 [see
Fig. 4E.B(a)]. Determine:
I
Cart A mA
~ K

I
·concept: Solve problem in CM frame
B•
me
v, mAmB ) Vo,i -
-1 ( ---"-~- ~ mAg
' h
2 mA +m8
Fig. 4E.8 (a)

(b)Jhe mcajmum hejghtreached bythLbJlli


' '
(a) the velocity of B as it reaches the maxim=um~_h_e_ig_h_t,_·_· _I Wgravity
.Af(II
.'.,_

=0- 1
-pi Agh
mAms v~
2mA +ms '
· Solution: We choose ball and cart as our system. No
external force acts on the system in x-direction; therefore
momentum along x-axis is conserved. + Note that for mA >> ms,
The· ball will continue to move upwards until its velocity (vB)f =(vA)f =0
2. '
relative to the' cart is zero. · ' and h= Vo
2g
i.e.,
_, _, Ball B oscillates as a pendulum with A fixed for
or VB= VA, mA << mn,
When the ball reaches maximum· height, the cart and (vslt =(vA)f =v 0
ball move horizontally with same velocity at the extreme and h=0
position. A and B move with the s,ame constant velocity v 0.-
~----•~-,,-----~r,----.-,-~·
~--'-,''(v,Ji =O (v,J, = ~
A --- --··· A 1 - -.. @iExam:~~~
., _,': --- ·_ . _._., ' " ---·: ::,
'.lwo identical wedges of massM are smoothly conjugate<i. The
wedges are free to move· on a smooth ·horizontal surfaqe, A
Reference level
, , :t+ ( ),-( J-
............ B - Ve - v~ t.·-
V block of mass ni is released from a' ·height h on one of the
B . ~egg<!§_(see Fig; 4K~J~-----·-~----~~....,
:. Initial position .:J:ifl~I position
Fig. 4E.8 (b)

pf= mBvB = mavo


P/= mB(vB)j +mA(vA)f
= (m 8
+mA)v
From conservation of momentum, (a) (b)
P, =Pt Fig.4E.9
m8 v 0 = (mA + m8 )v (a) Show that the height h to which ,the mass m ascends :the
or v=-~~- mavo I .. . . . M 2, ' · '
l nghtwedge,,s,hmax. = · . 2 h. ·,
mA +mB I ,· . (M + m) • , • , .
In order to find maximum height reached by the ball we (!,) Wh_gt_17.!f!,iU.'f!.!io..:.(M/.!!!l result§._i!l)lmax.--=..h/3:,~
will apply law of conservation of energy.
Solution: (al .When the block reaches the bottom of
1 2
E, = mAgl+-m8 v 0 the left wedge, we can 'apply energy and momentum
' 2
conservation.
P, = 0
www.puucho.com
Anurag Mishra Mechanics 1 with www.puucho.com

- - - · - - - - - - _ _ _ _ _ 33[)
P1 =Mv+mu
-> -> M2
hmax = ----.
2
h
(M +m)
=Mv-mu
P; =Pt (b) h = h
max (1 + m/M)2
Mv=mu ... (1)
Similarly from energy conservatfon, h
As hmax =-,
E; =Et 4
1 2·1 2
mgh =-Mv +-mu ... (2) (1+:r=4 or m=M
2 2
When the block reaches hmax on the right wedge, the RECOIL, DISINTEGRATIONS, EXPLOSIONS
block and the wedge will move with common velocity. The In these cases the internal forces are exened by or on
vertical component of velocity of block reduces to zero at the particles that compose the system during very shon

Sy·s·~r~-- ·:a:
this moment.
---- intervals. External forces, such as gravitational force, are
negligible in comparison to the large internal forces.
Y ,.•• .;• v.\ Generally the problem stans with a system of two or more
: : : - : particles with no relative motion. Then some stored energy
is released causing the parts of the system to fly apan. The

t<;;~;~ :::.~;,: '"~/ total momentum of the system at the instant before
disintegration or explosion is equal to the total momentum
of all the particles immediately after the event. ,
The total kinetic energy of the system is not conserved
P;=mu,P1 =(M+m)v (similar to inelastic collisions). The source of released
energy may be chemical, mechanical or nuclear sources.
P; =Pt
mu= (M +m)v ... (3) . li:E~q~e;le}wl~
==- - · - i ~
1 2
E; =-mu
2 IA rocket-·; p~~je~ted- vertical~-;;;,~ards. It explodes at the/
E1 .=-(M+m)v
1 2
+mghmax,E;=Et topmost point of its trajectory into three identical fragments .
2 One of the fragments comes straight down in time t 1 while the 1
other two land at a time t 2 after explosion. Find the height at
1
-mu 2 1
=-(M+m)v 2
+mgh max ... (4)
2 2 which the explosion o.,:curred in terms of t 1 and t 2 ?
) V3 V2 I
From eqns. (1). and (2), we obtain v = mu
M !'
2
1, (mu) 2
and -M - 1
+-mu = mgh ... (5)
2 M 2
From eqns. (3) and (4), we obtain

~ = (Mn~tm) c___ ------ -- - -- -------·=------~


Fig. 4E.10

2 Solution : At the topmost point of the trajectory, the


and .!.mu 2 =.!,(M+m)(~) +mghmax momentum of the system is zero. From conservation of
2 2 M+m momentum,
2
or mgh 1
=-mu 2 1
--(M +m) ( -mu
-) .. ,(6)
m1V1 + m2V2 + m3V3 = 0
max 2 2 , M+m as m1 = m 2 = m3
Now we divide eqn. (6) by (5) to obtain v 1 +v 2 +v 3 =0 ... (1)
mu2 (mu)2 1 m The second and third fragments reach the ground
hmax _ · (M + m) _ (M+m) simultaneously, therefore vertical components of v 2 and v 3
-h-- (mu) 2 - m
must be same; secondly, v 1 is downwards, the vertical
mu 2 + - - 1+- components of v 2 and v 3 are-~ (i.e., directed upwards).
M M .2
hmax M2 • 2

h (M+m) 2, For first fragment, h = v 1t 1 + ~ 1 ... (2)

www.puucho.com
Anurag Mishra Mechanics 1 with www.puucho.com

2
For second fragment, h =- v,t2 + gt2 ... (3).
2 2
g(t?-tf)
From eqns. (2) and (3), V1::::
2t, + t2

and h = gt,t2 (.t, + 2t2)


2 2t, +t2

IMPULSIVE FORCE
When a force, of relative higher magnitude acts for
relatively shorter time, it is referred as an impulsive force. Consider a large ball. colliding with small ball
An impulsive force can change the momentum of a body by N1 ,N 3 = Impulsive; N 2 = non-impulsive
a finite magnitl.!de in a very short time interval. Impulsive JI
force is a relative term. There is no clear differentiation ,,; '

between an impulsive and non-impulsive force.


1. Gravitational force and spring force are always
It ''

non-impulsive.
2. Normal, tension and friction are case dependent.
3. An impulsive force_ can only be balanced by another
impulsive force. Fig. 4.8 (b)
1. Impulsive Normal : In case of collision, normal
~--- ----------.
2. Impulsive Friction-: If the normal between the
forces at the surface of collision are always impulsive: two objects is impulsive, then the -friction between the two
N 1 =Impulsive; will also be impulsive.
Normal reaction due to ground is N 2 = Non-impulsive Both normal force N 1 and N 2 are impulsive

• ----+ ,, ~

,,, ,'.

N,.: '
(a)

Fig: 4:9 (a) ·


' ,'111l1m;±m,m±mol11i ,,, '.,, Friction at both surfaces is impulsive if it exists.
Collision Of blocks

,,,··(bl:',, ;''

,'h__- ~~ . ~N1_.:_
~2 ...
·LL{-}--N, •-
:·: . ~19 "'''," m2g.
} ,·

,. . (c) . .
_,..,:.,.:._~_ ·1 Fig. 4.7· · J',· Consider collision of large ball with small ball
Consider a ball dropped on a large ball. Friction due to N 1 is non-impulsive and due to N 2 is
impulsive
Both normal forces N 1 and N 2 are impulsive

www.puucho.com
Anurag Mishra Mechanics 1 with www.puucho.com

IMPULSE AND MOMENTUM ~- -:: _:--~~---- 337]


Concept: For block J 2 =mV ... (iii)
Impulsive Tensions In a string: When a string is Solving, these three equation, we get
jerked out equal and opposite tension act suddenly at each: V=.1::'.
end and impulses act on the bodies attached with the string in 3
the direction of the string. · Illustration 5. Two identical block A and B, connected
by a massless string are placed on a frictionless horizontal
plane. A bullet having same mass, moving with speed u
strikes block B from behind as shown. If the bullet gets
embedded into the block B then find:
I - -
T is non-impulsive , ~ c~-!~
lmhndn111i1111111111~111;
Fig. 4.12
All normal are impulsive but tension i
T is impulsive only for the ball A The velocity of A, B, C after collision.
(a)
Fig. 4.10 Impulse on A due
(b) w
tension in the string
One end of the string is fixed: The impulsive which . Impulse on C due to normal force of collision.
(c)
acts at the fixed end of the string cannot change th< Impulse on B due to normal force of collision.
(d)
momentum of the fixed o/;ject attached at the other end. The Solution : (a) By conservation of linear momentum
object attached to the free end however will undergo a change; u
v=-
in momentum in the direction ofthe string. The momentum 3
remains unchanged in a direction perpendicular to the string., · mu
In this direction string cannot exert impulsive forces.
(b)
fTdt=-3
JNdt=m (3u -u J=-2mu
Bath ends of the string attached to movable, (c) --
abjects: In this case equal and opposite impulses act on the, 3
two objects, producing equal and opposite changes in'
(d) JCN-T)dt = fNdt- fTdt = mu
momentum. The total momentum of the system therefore 3
remains constant, although the momentum of each individual
object is changed in the direction of the string. Perpendicular, => fNdt=2mu
3
to the string however, no impulse acts and the momentum of1 !>Jg!e: - - - - - - - - - - - - - - - - - - - -
each particle in this direction is unchanged. Impulsive forces are those forces which can have very large
In case of rod: Tension is always impulsive. value in very small time, e.g., Tension, Normal and friction.
When impulsive forces act then momentum along the
In case of spring: Tension is always _no_n:impulsive. __ _ direction of force cannot be conserved.
illustration 4. A block of mass m
and a pan of equal mass are connected by illustration 6.
a string going over a smooth light pulley.
Initially the system is at rest when a
particle of mass m falls on the pan and
sticks to it. If the particle strikes the pan
with a speed v, find the speed with which m
the system moves just after the collision. J (a)
Fig. 4.13
(b)
· Solution : Let the required speed m
,m
is V. In. (a) momentum cannot be conserved in vertical
Fig. 4.11
Further, let direction just after collision while in (b) it can be conserved
J 1 = impulse between particles and pan just after collision.
J 2 = impulse imparted to the block and the pan In (a) Tension will reach a large value in small time and
M
by the string
Using impulse = change in momentum
fTdt * 0.
0
For particle J 1 = mv - mV ... (i)
For pan J 1 - J 2 = mV ... (ii)

www.puucho.com
Anurag Mishra Mechanics 1 with www.puucho.com

,; . "' 'i ,--, ' ·J~ " ' "':"..t __ ,.~.,.

:Li3=.·:::3=.8_~---:-':.a·:_..··---------~--:----··-·-'~'-'_·.~-' ..:'...:..L...2~, :··,·.:'<~· " " •;,,.:-;~,- .


. k~~r;.m:~,;IJ?J 1i" ~ V1 =
Mu-mv 0
M

.. j'Iwo, ,blo~'A and B qre jofned'by means of a ·sl'.1~ked';~ U= Sm/s


lpassmg, over .c;z massless pulfoy as shown m dtagrprrz .. :The · 90x 5-l0x 30 450~ 300
=-~~-
'isyste111 is feleruedfrom rest and_itHecomes tautwherfB falls a . . V1 -- 9Q '90
Idistance' 0.5; m; ·then ";.' · ;• •, ,,
:, ·, ·,.··1
. , .
·2kg
A
,'
.
,,

Energy of explosion;, K. E. 1 ~K. E.;


= ·(1
-Mv 12 +-mv
·
. 2 .. 2 2 . 2
2) '
_ -~(m+M 1
)u2 1
2
1 kg . 1 (m+ M)[Mu + niv~ · _2 ] .'
Flg.4E.,11, . \•.\ - ' -
=- -~--"- U·
2 . ·M · . · ·
J1;·a.:? Fi.nd t/t~ ~-()mmon. velocit;x. of
become taut. ·· :
o blocks just aft~r. } ~
™'_:_
·, 10
= .!_(m+.M)[mv~:·] =· .!_(100)[ X 900] = 5000 J
rt,) ·Find the n"tagnitude of impulse on the pu_lley by the c!ari'tp ·2 · M 2 90
Lluri/Jg'J!J.~.. small interval w.l!il~ strj!)gJ?.ecom~ t@t.,, · ,J
'' CENTRIPETAL ACCELERATION REVISiTED
. Solution: Veloc(ty of a just-before the string is tl!ut. . ' .
Consider a particle
. VB ';' .J 2g,1h /= .fw 1}1/s . of mass .m moving at
(aScominon velocity= V ·_ speed v on a frictionless
' ;,,-
.. · Ko surface inside a fixed
v = v.8 /3 =-·-m/s horizontal circular loop
. . .3 .
of radius ras shown in
: · (b) Magnitude·of impulse ori A·
Fig. 4.14. . Assuming

l
=·Magnitude of.impulse on B collision -of particle
:= 1(.Jro- ..Jw). ~Fla
. · .. 3
.
=
3
N-m/~
.
with , the . hoop as
elastic, we can see that
the · magnituae of
. :; .Impulse'on
. pulley~ iz-1mpulse
: on A= ~.Js
3 N-m/s momentum· of · the -
·particle is constant but · : ·;.;
:ijexdt1m:m,~ · ·· its direction is changed
with each collision.
·' r l I ~

r::. . . - .
P,ig. 4E.12 shows a; ·
. ,-
' r . . . .
·--------c----· j , -_,:
Thus momentum of the '---'-----=-,-~---~
Flg:4:14 ,, ·:. i
i. ·. · particle ·1s not conserved. When viewed from centre the
+
IMan Rock~t:Lau'ncher of totl:!l mass =¥ =90 kg
. · , _, ."~... -~·, . · we
' ,.· ', successive collisions are separated by angle 0. From Fig. 4.14

"" -;.;~r-·~r~lj .
-l~·=Sm(s;'.massofrocket"=.TTf,=lOkg' . ·. see that · · · ·
'
!Muzzle velocity

.I
. ,, of Rocket =Vq_'.= 30 rn/'s

·.. '
'

"' "" ~,", , , . _...


':~\•>j. ,f· , PJ'
a

=mvcos ~ i_-mv.sin _2 j '


• • Fig. 4E.12
(a) What wi.ll'be m~n_'sand rocket's v,e_locity'after firin,g_.· . .
'{/,J__fill~_of._f2CP.losio"' . · ·:i.,-1..J ..
• . ·• ··.
I The change in momentum .1 P of the partjcle is .
· -'>
.:lP = ~1 -P,_-· .= -2mvsin - j
_, -> · (9),
r 2
Solution: Man fire rock~t with muzzle velocity= v 0 •• ·4 · (0)
.. Let·velocity o(rocket
· is~~· and.man is v momentum in
1
IAPI= , 2mvsin
. . . -2
.horizontal direction remains. ccihstant. The hoop exerts an im~_ulse on.the p~c)e that changes
· Initial mo111eritum = final lllomentum its momentum. .
(m.+M)u=Mv 1 ~mv 2 F _·Llt=Af'=2mvsin(!!.)
v 2. = v 0 +· u~. avg._ , 2

(m+M)u =Mv1 +fmCvo +u) · Thedistan~ betw~~ns~cc!!~siveco~sio.nsis2rsin,(!} .-


i 7
www.puucho.com
l
Anurag Mishra Mechanics 1 with www.puucho.com

_l. IMPULSE AND MOA1ENTUM 3391


Hence the time interval between successive collisions is From conservation of momentum, the new speed is
A ', 2r ,
ut =-;;-sm (8)2 given by
. 3
3mv+0=4mv' or v'=-v=gm/s
As the panicle continuously remains in contact with the 4
hoop, the time interval M' approaches M, the time Note that the firial momentum of the system is 4mv'
associated with the contact of panicle with the hoop. because the only effect of the pulley is· to change the
Substituting M' for M, we get direction of tension in the cord, the sense of motion of both

Favg.· -;;-
2r sm. (8)2 =2 . (8)2 mv sm
the bodies is same.
(b) The loss in kinetic energy of A is

.F = mv
2 Ic2mi~2 _Ic2mlv'2=Z.mg2 3
or avg. r 2 2 9

From Newton's second law the force of the hoop on the ~,!;f~am'RJ~! 14 ~
panicle is mass times acceleration. Thus we can say that r . . . -- ---"·- .,
v 2/r is the magnitude of the centripetal direction. · 1After falling from rest through a height h a body of mass m'
!begins to raise a body of mass M (M > m) connected to it;
~~~i>~k~:e~ Jthrough a pulley. . j
( a) Determine the time it will take for the boey of mass M to j
[nv~-bodies.of.mctsses. m and 2m are conne~t~d by·~ light i return to its original positio~. · , .:·
\inextensible string passing over a smooth pulley and released. (b) Find-the fraction of kinetic energy lost when the body of
,Afterfour second a mass m is suddenlyjoined to the ascending I mass'M is jerked into motion (see Fig.4E.l4,). :
iboey. Determine J 1-·- ··----- ~----,1
(a) the resulting speed, and . i
;i:
0

(b) how much kinetic energy is lost by the descending body .


I when the boey of mass m is added. ·
i'
I . 'I
'
~·~-
,: +v' mg
mg -I
2m
· a= O (a) (b) I

i· 2mg
2m I Before the After the i
I string tautens string tautens' :
i i
!<4s
(a)
·t>4s
(b) (c) . i Fig. 4E.14 , - i . I
L' i
- - - ~ - - - - - ~ - - - - - - ~ - ~ - -- -~-~---- .. J

. • .....k.a'--""'=-··--- FlgAE.13
Solution : (a) The speed of the body B just before .the
string becomes taut is v= .J2gh.When the string is jerked,
Solution: (a) For. the sake of convenience the
proble!JlS involving pulleys can be solved by including'blocks large impulsive reactions are generated in the string. At this
and pulley in the system. This single body has mass moment effect of gravity is negligible. So momentum of the
mA + m 8 and is_.acted upon by a single force (mAg - m 8 g). system is conserved ·at this instant. Let v' be the common
Fcir t < 4 s, the system is accelerated speed of the two bodies· after they are jerked into motion.
I From conservation of momentum, we have .
As mA = 2mn" = 2m, so._the·equationof motion is
mv = (M + m)v'. or v' = -
in- v
M+m·
. The acceleration ,of the system is
Alternatively, . . M-m
'J:F =mg-Mg= (M + m)a or a=--.--g
M+m
·The speed at t = 4 s is The acceler~tion is negative, opposite to v•:
· v=D+at=4g m/s· Let the system return to original position at time t;
. 3
, 1 2
The addition ofmassatt = 4s_is equivalent to a collision O=v't+-at ·
. 2 ! •
between the system and a body of mass m which is at rest. @I
2v'· 2m
or t=--;;-= M-mfg
www.puucho.com
Anurag Mishra Mechanics 1 with www.puucho.com

·Li3:....4:..:0_c.::·~·, i..:'f2~··_,_'tc_·....,''-··='.;::;'·:.:J!c~"-~-~;·_-'-'-_·~-""-'-'---'---'-----'--"-'-----~-·--'----M-"E~..ci<-ij~_N~
(b) The fractional loss of kinetic energy is I,m;X;i+ I,m;YJ+ I,m,z,k
. .!.mv 2 _ _!CM +m)v' 2 = i i i
2 2 · M M
1
-mv 2 M+m
2
. CENTRE OF MASS
Consider. twci particles unequal mass connected by a where t; is the position vector of the ith paritcle.
massless rod ·(Fig. 4.15). If a force is applied between the
lighter particle and the centre of mass, the system rotates r:=x;i+y 1J+z 1k
clockwise [Fig. 4.15 (a)]. When a force is applied between
heavier particle and centre of mass, the system rotates Centre of Mass of a System of 'n' Discrete Particles
anticlockwise [Fig. 4.15 (b)J. When a force is applied at the Consider a system of n ------··--·-----
1 v
centre of !llass, tlie system moves in the direction of force point ~asse~ m1 , m2 , m3 , ... m~ i
· without rotation [Fig. 4.15 (c)]. whose position vectors from I
origin O are given by I
f·- · .
t,J-J-
The overall motion of Lighter Rarticle .... ~.- - - • I
a system can be described r-+1 ,r-+2 ;r-+3 , ... rn
--+ ,
respecnve y.
in terms of lr point called Then the position vector of the
centre of mass. The : (a) .
1· _ ··:
centre of mass C of the system
centre of mass of a ~ : r • '' • • ,_

is given by
system represents

[,.;ff~b-
Fig. 4.16
translational . motion of . '--
the system. It moves as if
all the mass of the system
is concen~ated at this
point. If tli.e resultant

· · _ ·_ .~
t· .i. ··b···-~·
external force on- the (
. . -, ,", . '

system is E Fext and the [.icJ' • ·. n


total mass of the system I,m,
is M, then _, /: . :. : .:Fig, 4,1s ··--· ___ I >al
. '-> • LF
.'. a CM-,-~
-· en
-+
rCM = -1~
£.
-+
mi ri
'M >al .
The system behaves as if the resultant external force is
·applied to a single particle of mass JI/located at the centre of
where M(= i 111;) is, the. total m~ss of_ the system.
mass. The total momentum of the system is the same as the
product of the mass of the system and the velocity of.the Position of COM of-Two Particles
centre of mass point. . Centre of · mass of two r
-, -, particles of mass m1 and m2 COM
Psystem = Mv CM separated by a distance r lies ~
·+ For a syst\'m of many· particles in three dimensions ·
between the two.parnc ·1 Th ,m, ,~2
es. e I• - •I• •I
distance of mass from any of the ' 1 2
the position of centre of mass w.r.t. any fixed axis is '
determined from particle (r) is inversely Fig. 4 -1:L, _
. Lmixi proportional to the mass of the particle_ (ml ,
XCM =.m1X1 + m2_X2-+m3X3 +...+mnxn = _,_;-- i.e., r~-
1
·mi +m2 +m3 +...+mn Lmi m

Similarly for y- and z-coordiilates of the centre of mass, or


we may use the equations
I,m;Y;
Y. . _,_i_ _
or .l
-· CM .M and
M
+ In vector notation, the position vector of centre of and
'mass. is _, · · · :

. . r'CM';-=XCMi+YcMj+ZcMk Here,
www.puucho.com
Anurag Mishra Mechanics 1 with www.puucho.com

r - --- .
L IMPULSE AND MOMENTUM - - - · - -- _,_ , __ - - -
341 i
- - - - · - -.. ---<

and r2 =distance of COM from m2 MOTION OF THE CENTRE OF MASS


From the above discussion, we see that When the centre of mass of a system of particles is
moving its velocity can be obtained as
r1 = r2 = .! if m1 = m 2 , i.e., COM lies midway between
2
the two particles of equal masses.
V
- -,CM = dtdrcM
- -, = dtd[Lm
f ·r'. ]
Similarly, r1 > r2 ifm1 < m 2 and r1 < r2 ifm 2 < m1 , i.e.,
COM is nearer to the particle having larger mass.
+ To find the centre of mass of an object with = Em/d~/dt) = "f.mjvi
continuous mass distribution, we replace the
M M
summation with integral
-t f1 dm Since mi vi is the momentum Pi of the jth particle :
rCM = - - - ---> --->
M ---> (LPj) p
f xdm fydm fzdm VcM =--=-
or
M M
XcM =~, YCM = ~ , ZCM = -
M
-
... (1)
where dm is a differential element on the body whose ....
position vector is Since P changes only if external forces act, the same
r=xi+.Y.i+zk ....
must be true of v CM.
Concept: For any symmetric object, the centre of mass Concept: If the net externalforce·acting on a system is
lies on an axis of symmetry and on any plane of symmetry, zero, its center of mass moves at constant velocity.
this is valid of mass distribution is uniform.
The gravitational potential energy of a system of particles The net force acting on a system equals the rate of
in a uniform gravitational is the same as if the entire mass is change of its total momentum. Thus if the system's mass is
assumed to be concentrated at the centre of mass. con~tant, we have:
U= Im,gh, =gim,h,
.... . ....
--> dP d ---> dvCM
i i Fnet,ext = - = -(MvCM) = M - -
From the definition of centre of mass, dt dt dt
Im/1, --->
Fnet,ext =M
....
acM. . .. (2)
hcM =~'~-
M Newton's first and second laws apply to a system as if it
or U=MghcM were a single particle located at the CM. So far we have
treated complex objects-rockets, automobiles-as particles
CENTRE OF GRAVITY but have not given logic behind it. These theorems show
Any object can be assumed to be composed of a very why the particle model is correct.
large number of point masses. Gravitational forces on all the For example, when the external forces acting on a
particles can be considered to be parallel, all of which system are entirely due to gravity, the total external force is:
combine to produce a resultant force, the weight of the body. .... ---> •
Centre of gravity of an object is that point where the total F,xt = "E Fg,j = "f. (mi g)
weight can be imagined to act, i.e., a single force (weight) .... --->
acting at the centre of gravity produces exactly the same =(Lmi)g=Mg.
result as· having gravity act on all the point masses Comparing this with eqn. (2), the acceleration of the
constituting the body.
system is a'cM = "g. In the absence of air resistance, a
Position of centre of gravity can be calculated from the
system's CM falls just like a particle, regardless of what the
expression
L,migxi system's individual pieces are doing.
Xca = ~'=--- The · center of mass reference frame as zero
Ii m,g momentum reference frame
Concept: A reference frame with its origin at the center
, Concept: If gravitational field is uniform over the body,,
·of mass of a system is called the center of mass reference
·g cancels out in the above expression, and the expression·
frame. If the motion ofa system is describedfromthe center
·reduced to that for centre of mass of the body. :
of mass reference frame, we find that the total momentum of
If g is constant, the centre of gravity coincides with the; the system is zero.
centre of mass. '
www.puucho.com
Anurag Mishra Mechanics 1 with www.puucho.com

!342
The position vector of the center of mass is We. already µsed · the correspon.ding relation .among
--+ ·1 , --+ . position vectors (Fig. 4.17) 'in th~ derivation of eqn. 1.
rCM =--Lmiri
mtotal i . We may use eqn. (1) to calculate the linear momentum
· where m,0 ,.1 is the mass of the entire system of particles. of a system in its CM frame. The linear..momentum of
Ifwe choose the origin for the coordinate system to be at the particle j in this frame is: ·
center of mass itself, then the position vector of the center of --+ --+ --+~
p.CM
J, =m·V·CM
] }o =m•(V·-Vci,i)
mass point is. zero; that is, I } ) ' •

_, _, _,
• 1
rCM = Om, ' or .. 0=-~m-?-
,4,,1 l l =Pi-mivCM.
·· mtotal i
The total linear momentum of the system in the CM
where the 1•, locate each particle with respect to the frame is: · ·
center of mass as the origin. Since m,0 ta1 0 kg, , the * --+
PCM =:!:P-J, CM
--+
=:!:(P.J -
--+
m-Vci.i)
--+

summation must be zero: J

--+. --+ ' --+


Differentiating this equation with. respect to time, we But P1ab =Mv °'1, so PcM = 0.
find . .
Be~ause the total momentum vanishes in the CM frame,
0 = L,mi~'t it is .sometimes called the center of momentum reference
i· ' frame.
where v!,
is the veiocity of the ith particle relative to:the. Kinetic l;:nergy of a System of _Particles
center· of mass frame. Each term in the sum is the The· center of mass also provides a useful simplification
. _, ' :when finding· the total kinetic energy of a: system of many
moinentu~ P; of the respectjve particle in the center of moving particles. We clioose a coordinate.system.with origin ·
mass frame, and so the summation. is the total momentum 0 at some· convenient point in space and locate a typical
_, .
P total wn CM of the system when measured with respect to the · particle in the system by means of a position veciqt
. . 1,
ceriter of m~ss; that is,
--+ --+ . -
originating at O and another position veci:or1 i originating ~t
0 = L,Pi = PtotalwnCM the center of mass; as shown in Fig 4.17. The two vectors· are
related by ·
Concept: Thus the total. 17!e>mentum of the systel7! is 0

... (1)
zero when measured with -respect to the center of mass. For
this reason, the center of mass reference frame, with its origin · where r'cM is position vector of the center of mass with.·
Jat
the center of mass, also is,known:as a zero momen~m · respect to the origin 0.
· jre.ference.frame. ·
Differentiate the position vectors with respect to time to
Physicists often refer to such a .natural frame as the obtain the relationship between the velocities:
laboratory frame (even if there is no laboratory to be seen!). --+ --+· --+
The motion of particles within ·a system are often best Vi =VCM+V'i ... (2)
. described with respect to the ·system's center of mass-that : The kinetic energy of the ith partjcle as measured in the
is, in a reference frame with its· origin stationary at the reference frame with its origin at O is ·
system's CM. We shall need to describe _systems in this center 1
-m-v ..2
of mass reference frame as well as in a .laboratory frame and 2 ' l

to transform physical quantities between the . two The total kinetic energy of the whole system is the scalar
descriptions. . sum ·of the kinetic energies of each of the particles:

I.
I, . '
.Concept: Th_evelocityvjofa>particle in the labor'!tory
• '
KE,0 tal
1
= '1"2
~ -m-v 2
ll
-

·!frame equals its velocity ':J},CM in,t11,e center of m_ass frame ~!m.v t. v.
; ;: t2. t 1
tplus thewelocity of the center of mass with respect ·to 'the
·!laboratory frame : · ·
Substituting for ll, using equation (2), we o~tain
--+ --+ ~ --+
I ·vj=vj,CM+V·tM , .. (1)-" ~1 (_, _,')(_, -+,)
KE total== ~-mi vCM+vi · vCM+vi
•·. . ; 2
·'· '!
www.puucho.com
,\:
.-
Anurag Mishra Mechanics 1 with www.puucho.com

[ IMPU!,SE Aijo fl!J>MENTUM . .343]


But --, --,,) (-+.
( VCM+vi -+,) 2
· vCM+vi =,VCM
,2
+vi +2Vci.i·Vi
-+ -+, _MOST IMPORTANT CONCEPT
1. Kin~tic energy of system in centre of mass reference
The total kinetic energy thus is , frame is given by
~ 1 2' ,2 -+ -+, '
KE,otal = ""-mlvCM +vi +2VCM·V;) · 1
= 2µv 2rel
, 2 J(/,system/CM

1 "" 2 1 . '2 +vCM- (3) m1m2


=-."'1lmivCM + ""
..£..i-mivi --, ""
,£.mi --, '
vi··· where .µ=
2 i i 2 i + m2 m1
1. 2 3
µ is referred as reduced mass. Reduced mass
...::'y•;stem does not have any physical significance, its just a
combination of terms. · · ·
y, .../··· ••••••••••• 2. v rel is the relative velocity of blocks. Always
. 0 \-
remember that vre1 • is independent of reference
frame.

CM
. 3. Consider a two particle system
-+ -+ ..
JP1/CMJ= µJvreil
0
also
........··· Momentum of both the particles has same magnitude in
X CM frame.
z Locate a give particle with , Since CM frame is zero momentum frame
two positicin vectors. _, --,
Fig. 4~ 18
JP1/CMJ= -JP2/CMJ
Both the particles must have momentum in opposite
Now we interpr~t the three differeµt terms in expression direction in· CM frame. · .,
for KE, 0 ta1. 4. Consider-· a gas
Iri the first terin, the sum of all the masses is the total filled .cylinder kept in a

~
mass m,0 ta1 of the system of particles. v CM is fixed for system moving train. The gas random
motion
and there is only one v CM . The second is th,e kinetic energy molecules move randomly of gas
cif the system of particles with respect to the center of mass - in container.. If the trains molecular
or KE of system in CM frame. For the third sum, consider the --,
following, from definition of center of mass we can see that
Im/;=D
velocity is v o and a gas
molecule 'has random
. --,
I
,I
rmrrmn7TTT1TTTTT17TTTTfTlltlmrrmr 1
i • Flg.4.19
velocity v. The molecules I
[If we assign origin of a coordinate ·system at center of . ' . -+ -+
resultant veloci_ty is v O+ v.
mass, what is position vector of center of mass in this
coordinate system?] The total kinetic energy of gas is
differenti!3-te this with respect to time . K,ota1 = Kin, +KcM
Kin, is kinetic energy of gas molecules in refe~ence
we get -
frame of container and KCM is kinetic energy-of the_ centre of
and so the third term in equation (3) is zero. The total mass (bulk motion). The internal energy is independent of
motion of the train.
kinetic energy of the system of particle is then
.1 2 ""1 ,2 5. Consider a wheel rolling on a surface. An observer in
'
.
,KEtotal = -mtotalvCM + ."-1-mrvi CM frame will see that
2 -- , 2
each particle of wheel in
Concept: Therefore ,the tot£1!' ki_netic . energy of the pure rotation about CM.
system is'the'srim of:_
(a) the kinetic energy of the center of mass, as if the total
mass of the $JStem,were concentrate~.all at .that point; dnd
(b) the kinetic energy of the particl~ in reference-frame
with its Origin at the center of mass.
.___Th_at_is.a.,_ _ _ _KE_~to.!;l,l = KEof<;M + KEwrtCM
Iii this case two different
motion are involved one
that of particles w.r. t. CM
and other that of CM w.r. t
ground_
!
Fig.4.20

Therefore;· KE,ota! = KE system/CM + KECM


: • X1-
,

www.puucho.com
' -
Anurag Mishra Mechanics 1 with www.puucho.com

13~··· ++··· ·.' ~ ". f ME(HAtflCS-1 l


.In rotational mechanics you will learn that KEsystem/CM is (e) At the instant of maximum compression velocity of
r~tational kinetic energy.
6. Consider a two blocks system connected with a
spring. An impulse acts on
. frame block so thai: it acquires e~· ' f;I"' r Tl
;,,~Vo~CM
both blocks i.s same in ground frame.
. ·~
·
-
a velocity v 0 • A ground m2'°*"""!1 m, .
. ;lnjtlaJ state Maximum extensicin
observer see the sequence_ of _11_11 i~i,. ' :,
,,, (i) (ii)
events as follows. · m, :- m2, ,· · ,
(a) Spring beings to :..: ·. · Fig. 4:_21 ~ - ' -
stretch, now spring force
~etards m1 but accelerates m2 • Stretch in spring continues
fill velocity v 1 and v 2 are equal i.e., v,,1 is .zero. At the state
. of maximum extension both blocks have same velocity.
l-.:
I Max,trilum,corilpression
(Ill),·. Fig.4.21 (ej
Relaxe_d length
(iv)

~v:rce Sp(i~g.forrv, . In ground reference frame system moves forward with

. ,mu:=:::mm;Jmm,b;, spring . getting compressed and rel.peed, then extended


alternatively. . ,,
Now lets see what· happens in CM frame .
.'"' · . Fig._ 4:21 (b) _j
(a) Initial state
(b) In ground frame system translates forward .while

r-~: mwl=t~
,,
n
blocks also oscillate with respect to CM.
v~,
.<
--:---~n-"•.~.
.
- ........ ____J
.-=.
r1,=~=l"if=2.=~.'""""=.· ~~"if=.~-=-1 ~--._,!
I ~ . 'J.,,;
l~itlal state (b) At maximum extension
.. --- -··· ------ --- . v;~o ·L , . I
v2 a0, .

:ix:1:::;::;:~~v
_· r .:
q; -~.... ___
----- -- --------- -- --- .
... -·· . "
'I D-0 L_:M"~i2!
(c) At relaxed length of spring

.
veIocrty v CM
"' - "": f
• L

m2vo
= --"--"--
'Spring i~ fully ¢tr~~Cti~d1and
system haS translated:forward,

Vo
--
Fig. 4.21 ('.")

(c) Forward translation of system ~ontinues with


. .

to½9B
(d) At maximum compression

v~=O
CM frame
m1 +m 2
At the instant of maximum extension
of spring blocks are stationary in
2
I cl .

.
-··t~~~~-j~~",·r· - vi~1
- • •:"
-·. . '.' ' - ½
-- . I
·. 1'P2Vo '_ ~ In CM frame the observer sees only oscillatory motion of
vCM= m1 +mi: blocks
· At the maximum extensiori of spring blocks are at rest in
CM frame
,._ CM frame.
!' · Fig. 4:21 .(d) ·1 or
In the absence of any'external force CM ·= canst. At the v
instant of maximum stretch of spring both the blocks have or
same velocity which is equal to velocity of centre of inass.
In CM frame .blocks will be stationary at this moment.
· (d) While translating forwarcj spring begins to return to
or Xm~ =/gf vo
tts natural length. When spring regains its natural length, CONCEPTUAL EXAMPLE: Initially blocks A and Bare
blocks still have velocity and spring begins to compress. · given impulse in 'opposite directions as shown in Fig. 4.22 (a).
Now we have to calculate the
www.puucho.com
Anurag Mishra Mechanics 1 with www.puucho.com

·-- . -- --- -
I IMPULSE AND MOMENTUM 345
Most Important Concept:
..., ..., ...,
V ,.,/ground ::: V ,.,/CM + V CM"/ground
~7 frame / frame Yframe
,. Fig. 4.22 (a) ..., ..., ...,
similarly V 8 /ground = V 8 /CM + V CM/ground
(i) Maximum stretch in spring / frame / frame Yframe
(ii) Maximum velocity of block A and B in ground frame. Note that velocity of any block in ground frame is
(iii) Minimum velocity of block A and B in ground frame. :superposition of two velocity vectors, velocity of block in CM
Solution: vCM = (2m)(2vol- mvo = Vo frame and velocity of CM with respect to ground.
3m
; A/ground is maximum when ; ,.,/CM has maximum
At initial instant · 7 frame 7 frame
..., ..., ...,
V NCM = V Ajg-VCM/g = Voi ,magnitude and is in same direction as vector; cM/f::~d.
-,,

V 8/CM
-,, -,,

= V B/g - V CM/g = -2v oi


A
..., ...,
Similarly v ,/gmund is minimum . when v ,/CM and
•7 frame / frame
Initial state ...,
2vo v v CM/ground vectors are in opposite direction.
., Yframe
~~Vo
l -,, -t -t

CM frame
Jv Afmaxl=Jv /,/CM frame
J+JvCM/gmundJ=
Y·frame
2vo
..., ..., ...,
At maxirrium stretch Similarly Jv B/maxl = Jv 8/CM J+ Jv c,,i/grnund J= 3v 0
Ve/CM= a VA/CM= a
r frame '/ frame
Minimum velocity of A is attained when block is at
~ 1--vo ..., ....
4--- lo + Xm __. equilibrium position (spring is relaxed) and v A/CM and v CM/g
CM frame
are opposite to each other.
Fig. 4.22 (b) ...,
Thus Jv Amin I= 0
From work energy equation in CM frame Minimum. velocity of B is attained at the instant B is
We get W spring ::: AKE system/CM
...,
moving toward left (opposite v cMJ and velocity magnitude is
_ _!,_kX2 = o-.!:. (2m)(m) (3v )2 v O (see Fig. 4.23)
0
2 max 2 (2m + m)

or
1
2 max 2 3
6
Xmax = ~ : Vo
1(2 )
-kX ~ =- -m (3v 0 ) 2
-
:-'-7
lva,cMI = vo
~

CM frame
-
'.""7
lvAJcMI =-
2
Vo

f-- Vo

Fig. 4.23
Blocks return to I
relaxed state
Thus
Vs/CM =2vo vNCM =_vo
~
Block
What appears in
_.Vo
lj'~~iimiRJi . Ji57>
diagrams I
CM frame
representing I\vo block A and B of the same mass are connected to a light
situaUon when spring and placed on a smooth horizontal swface. B is given
block returns
to relaxed
state
- -
vAJg = 3v 0

~
What appears in
v819 = 0
velocityv 0 [as shown in the Fig. 4E.15 (a)] when the spring is"
-in natural. length. In the subsequent motion:

ground frame Fig. 4E.15 (a)

Fig. 4.22 (c)

When spring again regains its natural length in CM


frame.

www.puucho.com
Anurag Mishra Mechanics 1 with www.puucho.com

I346 ,,, --,· - ..-.~--: '. ' . -· . "<·"'"~7: '.·· . '


: = :_
: ::
_ :: =---:::--=--·=--=---=--===..:====:::;::::::;-----'--
.:.
:

'(A} the' max/1n.;ir, velocity of B will be v 0, Initial velocity of B = v - .':'. = ~ v = ~ v to right ·,


'(BJ as seen from ground, A can m~ve towards right or{ly__ . 5 5 5 · · ·
'.CC) the spring will have maximum extension, wlien A and B Blocks are executing SHM in CM frame with initial
i both stop , position as equilibrium-position ,
· '(DJ the spring will be at natural length again when B' {s at Step III: Velocity variation of B in ground frame,
L_.n,st,.____ ____ ·. ______:;__~ considering right as +ve( ). , _
4v v · 4v · v 3v
Solution: In CM frame the blocks will perform SHM as from + = v to + = - ~ '·
V = Vo
5 5 5 5 '
CM 2 so lvamaxl=v&lva · l=·o '

Jv~24£J::·;·~11
1
· - Velocity variation A in ground frame mm · • -
7
[ from (~•~)-- ;to--~+~~-~-:
1 Thus minimum velocity of A is -v when. spring is at
I -~~~ . 5 .·._
maximum extension.
Both blocks can have maximum velocity v 0 /2 towards,
right in CM frame.
Thus, Cvalmax = v 0 in ground frame
' ",:, :-::;r;: ' _· : ' -_< .•.. ~ ..; ' '" . ~:;:;:-::::,-'."'.f":4
(v A ) min = 0 in ground frame when A has. Th'.9 mas~es,A and B Connec_ted with' a.n ih.e.Jf!~n:ible ~iring; bfJ
velocity v 0 /2 towards left in CM frame length) Ire qn a smooth honzontrtl11lane,A zs mven a.yelofit.Yi
Also, in CM frame Va = v 0 /2lefyin the case when spring
is at natural length
Thus v a = 0 in ground frame at that instant.
of V m,/s alorzg the gr~un;:t perpendicular to line AJ{ds
in Fig, 4E.17 -(a). Find th~, t~.nsipit in
subsequent_mo_tio~.•·; _. ,
during- thefirl
~Lm
... .,"·':..'·L:'.:'J ,·. ·. _ -~tring
·i ·: '
Jhownl

L~a:r,,if~t~J 16 I~
!Tu,~- bl~cks Aand B.of masses 2m and 3m placed on smoothl
·t·<;.·

A2m
e 7 .·:. -
"j :..
lhorizontalswface are connected with a light spring: T/le twoj
blocks are given velocities as shown in Fig. 4E. l 6 when spring , Fig. 4E.17 (a) [' . •; , -i:t::_~
is at natural len"'h. _____ .
l Solution:
-
( '"---·!(
t~oooooooOoooooooo.F}--+]v I c9;~;,I-so-1""".~--.-th_e_p_·~-o,-bl_e_m-_i_11_C_,'M_._fr_a_m_e_:_In_C_M_1'-r9-rn~
:J
-------- -~...!~~~~__:__ _ __:_ _______
.~,-,-.; '~1·~· ~ -~ ·.' ....,.., "'"'"w!l,!"~" w
lpilrtides'A. and B mo~e .Cllorig circular-path with same· ci/

VCM
. (mx O)+ (2m xv) 2v
= ---~--~ = -
, 3m 3
_ro7:r,-'"t~~Q~Y.!!I~tl.r:;'.f_.:.--·-~t ~!~):~:f'~~~U,f~~~umn ..11
In CM frame
(A) mhtlm~m magnitude of v~locity of A I (P) v
m-7
(vAmin )_during motion [
(B) maximum magnitude of velocity of A 1 (Q)
(vAmax /during motion · ·
v
5
. c:M.1r·
'7
., ,
. . e,3: ., . '. ;
. : 2m · ·
(C) maximum magnitude of yelo_city of B (R) O.
·Fig. 4E.17'(b),
(v8 ~ )'during motion · ·'
,; . of zm mass =v--
ve1oc1ty 2v = -V
7v
~
(D) 1velocity .of centre of mass· (vCM) of the (S)
3 3
isystem comprised of blocks A, B and 5 m •
Angular velocity of particle A
1spnng , . ,
.
Ol=--=-
V/ 3 V I ~1M, 03 :o;~
Solution: Step I: vcM = C3mv)- Zmv = .':'.

Step II: In COM frame


Sm 5
1I 3
·
T=Mm r=2m.-.-
2
l
v2 1
I
,
,
2m
• •

3
V
c, ,·;
'i! •

3

J CM frame . ;. .
12 3
Initial velocity of A = (-v -~) = - ~ to left 2mv2
I' ,, Flg.4~.17(cj ; j
or T=--
31

www.puucho.com
Anurag Mishra Mechanics 1 with www.puucho.com

.JMPUISE AND MOMENTUM


,!4?]
kexarp:el~J~D> When string is jerked along length of string velocity
component along string is same for both particle
\nvo small balls .A arid B are-i-nt-e-rc_o_n_n-ec-t-ed_b_!)'_a_n_in'--ext,-e_ns_ib_'~le illustration 7. In the Fig. 4.24, a block of mass m
· !string of length L. Mass of ball .A = m, mass of ball B = m. The moves with velocity v O toward a stationary block of mass M
~alls are resti11g·on a frictionless horizontal swface, with the on a· smooth horizontal surface. Find the maximum
,
I
distance between them = 3[.15. In this position, ball .A is
suddenly given a horizontal velocicy- v o,perpendicular to the c·;;,--~,
compression in the spring of stiffness constant k.
k,, -M --- -!
llinejoining the two bal¼,_[see Fig,3E.18 (a)]
B I
. . . D~ :2:mO. 1

Velocity of C.M .. is unaffected /'


j by . the compression in the j
spnng.
Il__________ ' - _,_ _____.......J'
Fig. 4.24'

I ·, A
Solution : We apply work energy theorem in CM
I i Fig. 4E.18 {a)
frame. At maximum compression blocks are at rest in CM
frame.
(a) Find the speed of ball B just after the string becomes taut.
:(b) Find the impulse of the tension:in ~tring when _the string wspring = o-½kXiax ·
I becomes taut. ·
(c) Find the steady tens.ion, in string /7luch after tf,e string h~I Llk = 1µv 2 1 -0
2 "
L_becpm_g: tC!.!!t ' · · .• · • ' ....J
Velocity of centre of mass,
Solution: When string is jerked the motion of two
particle system is super position of translation ahd rotation Iriitial velocity of the block m w.r. t. centre of mass,
of two particles about CM. In CM frame two particle system
, Mv 0
will rotate about CM. Fig. 4E.18 (b) and (c) show lab frame Ul=Vo-Vc=--
and CM frame. m+M
Initial velocity of the blockM w.r.t. centre of mass
B
, -mv 0
:b.. U2=-Vc = - -
..
··,;L m+M
···-•• ~ ·
v0 cos'e KE system/CM = mu, + lM,2
1 ,2
u2
•• 0 Vo 2 2
· v0 sin0
.Thus.!( 2
mM )v~=.!kx orx=v 0~.!(·mM)
,, · situation before String gets jerked
2 m+M 2 k m+M
=.!( mM )vo2 =.!µv21
~ -"';"
2 m+M 2 "

Ja<: •· ••r-l'v 0oos e


"';'. illustration 8. In the Fig. 4.25 shown, if all the
surfaces are smooth, then determine the maximum height h
~~sme
-2-
attained by the block on the wedge, assuming it to be very
"--\v 0sin8 large ..
ln CM frame velocity of B.
Lab frame
(b) (c)
; Fig. 4E.18
2
·111 COS0) (Vo ' '
.., . . . 2
.1.ens1on 1n string :::; - - - - -
mvi cos2 8 Afthe highest position both the block
and _the wedge move together with the
I'

L/2 2L velocity of CM I1
When string is jerked ~-----F~lg~._4._25
_____ __,
Tangential velocity remains
unchanged whereas velocity change ~
v,c~se_u
Solution:
In CM frame block and wedge are at rest when block is
. , B . v sine
along stnng ,or 1s -.- - at maximum height.
2
. · mvsin8 F!g, 4E.18 {d) Ws,avity = LlKE
Impulse of tens10n = - - -
2
www.puucho.com
Anurag Mishra Mechanics 1 with www.puucho.com

348 MECHll~ICS-1 I
-mgh ~ o-1( .mM )v5
2 m+M

or h_ v5 (M )
· 2g m+M
If wedge is infinitely massive M ~ = In CM frame, frqm work energy theorem, we get
2
h = Vo F'i x1 + F' 2 x 2 = lk(x1 + x 2 ) 2
2g 2
or F'1 (x1 + x2 ) = l k(x1 + x 2) 2
2
" 2F' .
=> (X1 + X2) = __l
1\vo"blocks of mass m 1 and m2 a" connected by a spring of, k
!force constant. k. Block of mass m1• ·is pulled by a constant
1
lforce F1 and other bloc~ is.pulled by a constant force F2 [see or x = ~(F1m2 +F2m1)
max k.
m1 + m2
IF,ig. 4E.19 (a)]. Find the maximum elongation in !he spring.

I
i
8::1::::cr
I~_ . i
I Fig.4E.19(a)
1~6~~~J20~
lnvo
·------
blocks m1 and m2 ·are connected by a spring o/f6rce
'constant k and are placed on a frictionless horizontal sutfate,
Initially the spring is given extension x 0, _when the system is

im,.,B
Solution: We will solve this problem in CM frame released from rest. Find the· distance moved ·by two-blocks
which is accelerated. before they again comes to rest. · ·
(F1 -Fz) ,---·-
aCM =

:f:1 .
L
m1 + m2 klo ·
Assuming that F1 > F2 , CM frame is non-inertial, we 0000~0~~~~~!~,~o~:oo
have to apply pseudo force on the blocks. Fig. 4E.20 (a) .

Therefore net external force ~n m1 ( Fi _ Fz )


.
Solution:
F 1 =F1 -m1a, =F1 -m1
m1 +m2

= (Fi mz + Fzmi ) towards right


m1 + m2 [:b~~:~2~ :~:~fa~=~~-~:~::aJ
- ·"~""""--«•-
0
--·• • "'••,·• ••,·•-· ••••-·••-~···-•·-·-·:,··"--

~33··==·~~
lo'-Xo, --- ~ •

[flooooo~~o~~o~GJ ....1
I .
L--------------
and on m2 ,
Fig. 4E.19 (b)
---..i r--
:: .'
Ax2
·"-
7
·/-" ·:-/'

"Fig, 4E.20 (b)


. . . ( F1 -F2 )
F 2 = F2 - m 2a, = F2 + m 2 In absence of external force blocks again come to rest
. m1 + m2 when spring is compressed by x O• There is no change in the
= (Fi mz + Fzmi ) towards left position of C.M. of the system, Le., Axcm = 0 of block m1 ·
m1 + m2 displaces by Ax1 and m 2 _displaces by Ax 2, then
. .
In CM frame, the blocks move. in opposite directions We have
thereby stretching the spring. The spring will have ... (i)
mazjmum extension when blocks 1re instantaneously at rest and
in CM frame of right block displaces distance x 1 and left '·-= =· m1 Ax1 + m2Ax2 = 0
"" ... (ii)
displaces a distance x 2 from their initial positions. m1 +m2
After release spring block system will execute
oscillation~. •Figure shows five ~tages

www.puucho.com
Anurag Mishra Mechanics 1 with www.puucho.com

IMPULSE AND MOMENTUM 349


After solving equation (i) and (ii), we get Illustration 9. Consider a system
... _ 2m2Xo , .. _ 2m1Xo. of two particles of masses m1 and m 2 -···--
UA.J - L.l.A2 -
m1 + m2 m1 + m2 separated by a distance r. Suppose they : ·~1 F CM F-~2'.
: ......... 4--9:
start to move towards each other due to
~.~r1-r2-.:'
[;5_x_q.~g,1~~~.i211> their mutual attraction (attractive force
may be electrical, gravitational, etc.). Since
.
··7····
..
_, system
nvo blocks of equal mass m are connected by an unstretched· particles start from rest, and F,n = 0, Fig. 4.26 (c)
spring and the system is kept at rest on a frictionless
horizontal surface. A constant force Fis applied on one of the
blocks pulling it away from the other as shown in Fig. _, _,
m1 V1+m2V2
,4E.21(a). It follows that 0 ... (1)
'(a) Find the position of CM at time t, m1 + m2
(b) If the extension of the spring is x 0 at time t, find the
... (2)
displacement of the blocks at that instant.

... (3)

Fig. 4E.21 (a) ... (4)

Solution: The centre of mass of such a system remains at rest


_,
y unless acted upon by an external force. In the eqn. (4), A r1
C.M.

1. r
and A 2 are absolute displacements of particles m1 and m2.
'
: ill(:
'
:
.
: ill(
'
:
If r1 and r2 are displacements of m1 and m 2, then
~ 1~ : :c2M : r1 +r2 = r and . m1 r1 -m 2r2 = 0

~SJ .! [:}o~~~~~::e~~~~o~~fr
Ol4 - - - - ill<, :
If follows that
2
r1 = - -
m1
mr
- and.
+ m2
Fig. 4E.21 (b) which shows that the particles collide at the centre of
mass.
(a) The acceleration of centre of mass Illustration 10. A projectile of mass mis fired with
-> F F an initial velocity v O at an angle 0 to the horizontal. At its
\aCM\=--=-
m+m 2m highest point, it explodes into two fragments of equal mass.
The position of C.M. at time t One of the fragments falls vertically with zero initial speed.
1 2 1 F 2 Ft 2 Since the only external force acting on the system is
t.xCM = -aCMt = - - t - gravitational, the motion of centre of mass of the_ system
2 22m 4m
(the fragments) follows the same parabolic path as the
(b) Displacement of C.M. is given by projectile would have followed if there had been no
t.x - m1f.X1 + m2M2 explosion. Force of explosion is internal, it cannot change
CM - ,
m1 + m2 the trajectory of the system.
where t.x1 and t.x 2 are displacements of m1 and m 2
respectively.
Ft 2 mt.x1 + mt.x 2
or
4m m+m
2
Ft
or M1 +M2 = - ... (i)
2m
The extension of spring is,
---R-
O+--R/2-+- m CM X

t.x2 -t.x1 = x 0 ... (ii)


----X2---•
After solving equations (i) and (ii), we get Fig. 4.27
2 2
t.x1 =I_[Ft -x0 ] and t.x 2 =I_[Ft +x 0 ] No external force acts on the sytem in x-direction.
22m 22m Therefore

www.puucho.com
Anurag Mishra Mechanics 1 with www.puucho.com

f~~o_-_--_-- -_--_·________________________ MEftiiti1cs!f!


-, -, (xCM)f = (4M)_(x)+M(x-5R) =:fx--R)
m 1 X1 + m 2 X2
XCM = 4M+M · · - ··
m1 +m2
x-coordinate of centre of mass.is fixed.
R = mxR/2+mxx2 ' '
or Therefore CxcM J, = CxcM) t
2m (L+R) = (x-R) or x = L+ 2R
or 'x 2 = 3R/2
If we choose origin, at position of centre of mass, then [=.. e-,.xam"''iJ
• ---·~- 22 ~
I~

-, -, -,
XcM = 0= m1 X1 + m2 X2 l4.fro~ s,;;~n the end of a lo;;-;;:iard of l~~~r'h i: Th~-boa~a.:
m m~N~+mx½=O Irests on a frictionless horizontal table. The frog wants to jump I
or x 2 = +R/2 !to the opposite end of the board. What is the minimum;
+ If one of the fragments lands back at the initial jtake·off speed i.e., relative to ground v that allows the frog .to[
position of the projectile, \do the_gj_ck? The. board and the frog)1ave 1cqual masses. ____ !
;..,- .
-R- mx0xmxx2 Solution: Taking v for the plank in ground frame and
XcM - -
m+m conserving linear momentum in horizontal, direction
or x 2 = 2R
+ If the vertical component of velocity of both the
fragments, after explosion, is zero, they-land at the
m:: :~~s;s0)2usin0
: ri~s······-.. _, ._. :,
v~
same time. If one of the fragments is moving t =- - - lifftil/lH/1UUl1Jillf1M,i
1

downward after the explosion, the other fragment g L._ - fig. ~!:,?? ._J
2
( , will have upward component of velocity. In this .. L = 2u(u cose + u cos0) sine = 2u sin 20
V case, the downward moving fragment strikes the g. g
ground- first. The ground exerts a force on it before
~
the second fragment reaches the ground, that is an
external force on the system, therefore our analysis
U=
v~
does not apply after this· instant.
Minimumu=N
mustration 11. A small r::-·--· . .. ·_7
sphere of radius R is released from j Y .. ·······-,;7Ysteml
I bExaJm,~p
rest on the inner surface of a large ,
sphere as shown in Fig. 4.28.
There is no friction between any
surfaces of contact. When the
small sphere reaches the other
:
/ 6R

\, (L, O)
~
·· ........ ·
~-. · .
\
x=/] Find total W.D. by friction assuming plank is sufficiently long. I
, . p7.,.." ·----_ ·1
_2m I ,
. : I
:
extreme position, there is internal [___· -~~~-28 . I '--.smooth I
transfer of mass without any I..__________ -------==--=-=~..,.,,._._
· !
. Fig. 4E.23 (a)
·- - -····~ ,_:J
external force in the x-dire.ction. Therefore the position of
the centre of mass of the system remains fixed in the Solution: Where slipping stops both moves with same
horizontal direction. When the small sphere moves to left, speed by momentum conservation
the larger sphere moves to right so that the centre of mass mu= 3mv
does not move in the x-direction. u
v=-
3
Now, Work done by friction = LI.KE
-, -,
=Kt -K,
or m1 LI. x1 + m 2 LI. x2 = 0
Let centre of large sphere move through x towards right.
=!.2m(~) +!.m(~)
2 3
2

2 3
2
_!_mu 2
2
Then M(-10R+x)+4M>:=0 2
3mu 1
=----mu 2
or X= 2R 18 2 .
Now coordinates of centre of mass are (L + 2R, OJ = -!.mu 2 joule
Alternatively, 3
(x ). = (4M)(L) +M(L + SR) (L +R)
CM' 4M+M

www.puucho.com
.,
Anurag Mishra Mechanics 1 with www.puucho.com

I,. IM~UISE AND MOMENrilM , 351 I

,··---
,Find maximum height reached by smaU'.mass m iii Fig. 4E.24[
---, Solution: Ci) [Y => ~ :

'(a} and Fig. 4E.24 (b). . · ., I . (__ Fig. 4E.2~~-----.J


Initially no momentum along x-axis. So, final
momentum will be zero also and relative velocity' is also
zero. So, no velocity of any object.
By energy conservation,
V
initial potential energy =final potential energy
I, i1.1777mmmn=_(•=l
I ; m
'7777m7m7777
Fig.4E.24
m

I'
(b)
Hence, 8 =90°
AXCM = 0
i
m(2R-x)=Mx
Solution: Mass of both the blocks =m m(2R) =(M + m)x
. bigger block remains at rest till smaller reaches at 2mR 2(M/2)R
.bottom of circular part. , · x=--=
. M + m M + (M/2)
· Velocity of smaller bl6tj{ at-lowest point u = ~2gR.
Now bigger block also start moving let smaller block
Ir--:-- - -- ---- :
reaches up to height h.
By momentum conservation
mu =2mv
1C:±J ~I
I Fig. 4E.25 (c) _ _~

u (ii) Maximum velocity of wedge will be when the ball is


v=- at the lowest point in the wedge as till this point the
2
By energy conservation horizontal component of normal on the wedge will be
speeding the wedge.
increase in PE of smaller block= dee. in KE smaller block
Pi= 0
+ KE of bigger block
Pt =-Mv+mu
mgh +.!mv 2 = .!m(u 2 -v 2 ) ·
, 2 · 2 Pi =pt
Mv
·mgh =.!mu 2 _,!2mv 2 U=-=2V
2 2 m
1 u2·mu 2
2 1 Ui +Ki= Ut +Kt
=-mu --2mx-=---(4-2)
2 2 . 4 8 mgR+ 0 = o+.!mu 2 +.!Mv 2
2mu 2
2gR 2 2
mgh = - - =*· mgh = m - 2mgR = m(2v) +Mv 2 2
8 4
R
h=- 2xM xgR=4mv 2 +Mv 2
2 2

~~;~_1€~ MgR=4xM xv 2 +Mv 2


2
MgR = 2Mv 2 +Mv 2
In the Fig. 4E.25 (a) shown the ~A RB
we_dge of mass. M has a semicircular_· . smooth MgR = 3Mv 2
··z
groove. 'A -parttc .~ M. M. /
',. e o, mass-m
· 2 u; ,lm\=s===~
=~ w,~fu v=~
released "from Adt slides on -the Flg .. 4 E.25 (a)
smooth circular track and starts- Concept Review: ( Revisit Concept Review after
climbing on
th~ rightface. . studying Collision Theory)
(i) Find the maximum value of 8 which it can subtend with 1. Law of conservation of momentum states that in an
veftical and also find the distance displaced by we~e at inertial reference frame the momentum of system
I this position. ,
(ii) Find the m__aximum velocity of wedge during process of,I
remains constant if net external force acting on system
is zero.
I motion"---"'"'-'---------------~

www.puucho.com
Anurag Mishra Mechanics 1 with www.puucho.com

An isolated system implies a system in which there 14. In elastic collision KE before and after collision is
is no interaction between system particles and bodies conserved. During collision KE is not conserved.
external to system. · During c~llision deformation of bodies takes place,
2. Law of conservation of momentum is valid in all fraction of KE converts to deformation energy.
inertial referenee frame although momentum of a 15. In elastic collision total KE of the particles before and
particle depends on reference frame. after the collision is same. Momentum of particles in
3. Law of conservation of momentum is a fundamental CM frame changes in direction only and magnitude

.R
law of nature, its not consequence of Newton's laws. remains same.
. 16.
4. Centre of mass of a system is not a physical point these
may or may not be any mass present physically at the
centre of mass.
Vo
.. ·•
. all I Ball JI'.
The location of centre of mass depends on choice of ' statlonary
· reference frame. · ·

.
oblique and elastic collision .
5. Momentum of system is given by between two particles-in ground frame
-, .
= M,otal VCM ·I
J:.p,
system •
vrf2
)I: .
, vrf2
II
vrf2
6. Equation of motion for centre of mass is
CM frame
--+ d --+ . -+
Fextemal = M total - (v CM) = m aCM Fig. 4.29 (a)
dx .
7. The reference frame in which CM is at rest is referred After collision let the deviation of ball I from ~CM is 0.
.as CM frame. Total momentum of system is zero in CM
frame. then .other particle rrioves at 1t - 0 with ; CM in the
8. ·Momentum of particle 1 in CM frame is given by opposite sense.
--+ _;--+ --+--+'
P1/CM
-+
=m1(v 1-vCM)=µ(v 1-v 2)
--+--+
P:\'CM = m 2(v 2 -vcMl =µ(v 2-v 1)
---t--+
A.
I......
.11/4"••• vrf2

-, -,
. P1/CM = P:\'CM = Jlv rel
9. KE of particles in CM frame is Ball I
1 ·2 · Fig, 4.29;(b) .
KE,ystem/CM =
2µvrei v1 is velocity of ball I in ground frame.
10. During collision. the force of interaction between From figure a. = 0/2 After oblique collision ball's line of
colliding bodies is large as compared to gravitational
0 motion make right angle with each other.
frictional, (i.e., non-impulsive forces) when smooth
bodies collide, force of .interaction acts along line of a.+~= 1t/2; a.= 0/2; ~ = ~-~
r-------- . 2

;,u~1
impact. 2
11. Coefficient of restitution is independent of reference
frame. Experimentally it has been found that e
depends on impact velocity material of colliding·
bodies, shape and size of colliding objects.
12. During perfectly in elastic· collision bodies stick vof2 l
together and move with common velocity·
KEbefore collision+ KE after collision
KE always decreases during in elastic collision.
<~>
,Ballll
Fig. 429 (c) ·
,.,J.I I

13. II) CM frame total momentum of system is zero so


before collision the two particles have equal and From vector diagrams of ball I and II
opposite momentum. After collision the combined
we get,
mass moves with ; CM • Thus in_ CM frame particles are ' .
V2 = Vo SIIlCX
at rest. Therefore total KE of the particle is converted
into internal energy of system;
'

www.puucho.com
Anurag Mishra Mechanics 1 with www.puucho.com

- ------- - ___________ _ ,

I/ -IMPULSE AND MOMENTUM -


-- --- .. __________ ------- -~---·
, ---------· 353
17. Figure shows oblique elastic
collision with a stationa,y ball ·~ 0 '"? J"rdm y
rcM = - - -
of mass 2m. If ball of man m .m 2m M
turns by an angle of 30° in CM Ba/11 Ball2' The component of this
frame, we wish to determine the Fig. 4.30 (a) equation are
angle of divergence between 1
XCM = Mf xdm;
balls after collision.
Step 1: YcM = ~f ydm;
z
Step 2: ZCM = _!_ f Z dm. Fig. 4.31
M

·T[]
..
Consider a thin rod of
mass M and length L as shown in Fig. 4.32.
- . .

Ball I
·---'a -,-------- Ball II y
d:,
......,----,,.~ ...... ..... "
,, --.,,
r "::
P1tcM •-0 P21CM h ::
,, ..
I \.,=,,,jj
z i .·------- ·-·
(a) (b)
Fig. 4.32

The infinitesimal element in this case is a slice of length


dx. The rod has to be thin enough to ensure that all the
particles of the element are at the same distance from the
origin. If the volume-mass density (mass per unit
-> I ;::,;_
IV2fcM Vo volume) of the rod is p (kg/ m 3 ), the mass of the element dV
3
is dm = p dV = pA dx. If we define 11. = pA, we have
·dm = 11. dx. The quantity A.= M/L is called the linear mass
---> Vo
lvcMI = 3 density (mass per unit length) and is measured in kg/m.
Fig. 4.30 (b) For a disc or a cylinder, the appropriate element is a ring
of width dr and area dA = 21tr dr, which extends through the
body of the solid as shown in Fig. 4.32 ·(b). Its mass is
1v'~CMI=~ dm = p dV = ph dA. If we define cr = ph, we have dm = cr dA.
3
· Now apply trigonometry on vector triangles The quantity cr = M/A is called the areal mass density
(mass per unit area) and is measured in kg/m 2 • Note that A
sin a= 2sin(0 - it)
is the cross-~ectional area in a plane of symmetry.
0 = 30°; a= tan-
1
[
1
1
+ .J3] ~,~xgm_~J_'T----~>
So angle of divergence is ,-
.( a) Show that the CM of a uniform thin rod of length L and,
. a+P=75°+tan- 1
(
1
1
+ .J3) mass M is at its centre. (b) Determine the CM of the rod
assuming its linear mass density A. (its mass per unit length)
FINDING THE CENTRE OF :varies linearly from A. = A. 0 at the left end to double th<).t'.
\value, 11. = 211. 0, at th_e right end. :.
MASS BY INTEGRATION
A continuous body may be regarded as a collection of
! y
dm=Adx
point particles. The typical ith element has mas l'.m, and the
0 x----.i~ X
position of the centre of mass is given by · dx
L,r/1mi
rcM =~'~--
M
Fig. 4E.26

If we take limit l'.m ~ 0, each element shrinks down to Solution: (a) Let the rod be placed such that origin of
an infinitesimal element of mass dm. In the limit, the centre coordinate system lies at the left end. The rod is assumed to
of mass of the extended body is expressed as the integral be thin, soy CM = 0 and zcM = 0. The linear mass density of
www.puucho.com
Anurag Mishra Mechanics 1 with www.puucho.com

-
1
,3_5_4 :-+·: :_,_1!,_':c_.·'\:; .' MECHANJC~~I -j
the rod is;\,= M/L. We now imagine the rod as divided into They-coordinate of elements js y = R sin 0. The angle 0
-infinitesimal elements of length dx, each of mass elm = ;\, dx. varies from O to 1t. So
So. 1 ' 1
, L YCM =-Jydm=-JyA.Rd0
X
CM
= _!_rx=L xdm = _!_JL 'Axdx =
AfX=O MO
!::.I
M 2
x21 M
=_!_J• (Rsin0)A.Rd0
M .

0 Mo ·.
;\,L2 L
=~=-
2M 2 =..!..~~;\,]"
M . o
sin0d0
Thus CM of the uniform rod is at its centre. 1
(b) Now we have ;\, =;\, 0 at. the left e11d, Le., x =.o
=MR 2 ;\, 1·(-cos0) 1·
0
and ;\, =2;\, 0 at x =L. Ac~ording to tp.e g,.ven condition, ;\,_ =_!_2R2;\,
· varies linearly.,So we.write M
;\, =A 0 (1+roc) Using ;\, = 7tR
M, we have
Atx = L, ;\, = 2A. 0 , so.a= l/L. Now· .
XCM = _!_ r=L ;\,'xcfx : 2R
YCM = -...
M x=O . 7t

=I_A.oJL(l+,~)xdx In this case centre of mass of th~ .body is not within the
.' M o ·. ·L · body of the object. · ·

,;~ (x: +~)~·=i~L2 . ijf#a~J5iltjjil 2a ~


. . ' 0
Determine ;the position• of centre'. offn.ass of an object ofmassJ ·
Now w~ will determine M in terms of ;\, 0 and ;\, :a
M in 't~~ ~hape of right. triangle wh?se dimensions_ a~el .
. ,', M~JL .dm=JL;\,dx shown in Frg, 4E.2& The ob;ect has-a .uniform mass.per umt, ·
· · x=O . , - 0 • · . '., '
area_
=1- 0 1i{1 + f.) dx dril'
.,

=A.ol(x:~)( =~A.al
. 5 A- 0 2 5 ,.
Then . · . xCM = - - L = -L Fig. 4E.28 .
. . , ': 6M . 9_
Fe-·x~~: l Solution:, We divide the triangular lamina intci narrow
of width dx and height y as sho_wn in the Fig. 4E.28.
· f '.rips
11le mass dm of each, strip_ is · . ·
1r·etermine tit,~ ce~tre of. mass of~ loop of radius' R, ~US$)M. Total mass of the object f th .
dm= · . . xareao estnp
• •• • - ~. " ; , , _ Y dm::::Ad's=I\.Rd8 ," Total area of the object
'- . . 1
. ' "' ~ .

_2M_y
I : ,. ~ ...,.·-:-M-:-cc· (y dx) =
(1/2) ab ,_.
dx

· Now x-coordinate of the ~entre of mass _is


ab · .' ... '

J J" X(2M)'
~ ·-.- y dx
X 'XCM . 1 ·x dm=-
=- .1
Flg.4E.27 ·M.- '· .
M O ab

· Solution: We choose origiri of .coordinate system at =·-2


ab O •
xydx f"
the centre of curvature. and the y-axis on the loop's line of
. To evaluate this integral we must express y in terms !)f
symmetry. Theri x CM =0 because of symmetry. We ch<;>ose a .
x. From similar triangles in the figure we see that
· mass element of length ds =R d0. Since the· total length of . y b ' b
· the loop is 7tR, the mass per.unit length is;\,= M/7tR, where -=- or y=-x
M is 'the ,totalinas~ .. The ·mass of .the element is thus x a' a
"· •· elm= ;\,ds = A.Rde .Hence,. ·xCM =-f"x - xdx
. .
2· (b)a
ab 0

www.puucho.com
,.
Anurag Mishra Mechanics 1 with www.puucho.com

c..l_IM_P_ULS-'-E-'-'A_;_:;Nec:l}cc.M:.:cOMccE;::.N:.:.TU:.:.M::.__~-· ---·

= __!_fnl•
~2Jax2dx=il(x3 Jla
a2 o - a2
3
=~a
3
and-·.·. YCM
M o .
dmRsin0
.
0 1 fn/2 M · · ·
On similar lines we can calculate the y-coordinate to be
= -· 0
· - -2 (21tRfos0Rd0)Rsin0
. 2 . M 2itR ,. ·
YCM=-b RJ = . sm ·a cos e·de =·-·
.n/2 : R
3 . 0 . 2 .
k-lsXaroe;~~ · ~~amJ,£Je:j~
0
!Determine the coo,rdinates of centre of mass of a half disc ofi . Determine thii'CM of a unifo.111}:solid cone of height h' cind1
! , . " ' • ' ", • ; •,.,'

!mass M and radius. R, assuming uniform mass distribution . .: · . semiangle a as shown in Fig. 4E,31, · · :
' 1--.- - -
1 ...-t--y-
I .
~ i

f
--d,- h -.,J
i _F,..;ig;.._4E,_'.;,29;___x._lc.._'----~__J _! --1
---s~i~u~~; w~:se a semicircular strip of radius " · 1· - x _. · /::

.1
and thickness dr as differential element .. From, symmetry we ,· L.--~-=""'-_,___F_le_._4_e._3_~,--·_ ..J.1--------~>: ...:
can see that xCM = 0. · Solution: We place the apex of the-cone at the origin. .
Mass of the element
.
is thus '
It is.clear that the CM-will lie along they-axis.:we divide the :
dm = mass per unit area x area of strip cone into disc of radius x and thickness dy .' The volume.of'.
. M. . .
such a disc is-dV = nx 2 dj, = ic(y tana) 2 dy_. The mass of the
dm=--1trdr
rtR.2/2 . disc is dm = dV. First we 'win determine the total mass of. p
the cone. · · ·
The centre of mass of this element is at 2r/1t, from " . h
2 2
previous problem. So M= fdm"'.1tptan af~y d.Y, _
y CM = __!_fR 2r dm ~ __!_fR 2r( 2M nr_ dr)", _ 2 h3
. M o. 7t M 9 1t 1tR2 .· . -1tptan a.-
. 3

=__±_fRr2 dr =__±_I (cJI~= 4R


ltR2 0 ltR2 3 31t
. The position of the CM-is given- by
;YCM. =:__!_fydm
' 0 . .. . M- .
kli--e-x--gm~- . lt--~-='--i~:-3-o~
: _"-':·-,l ... '= __!_ np'tan 2 af h.j ady .
M . _o.
!Determine the position of centre of mass. of . a:--;-~: =·.1 2 li ·
-1tptan a-,-
4
... (2)
}:~if=e;:; :~;;:b~~ 0
~ass M _a~d ·ra~ius_ R, ~u~ingl
M
From eqns. (1) and (2), we have
. 4 ;

3h
Solution : In .this case Yo.i=+
element is a circuhµ· strip of ·
thickness ds. The thickness- of ring ·mustratioil 12. - If some mass of area is r~moved
subtends angle d0 at the centre of from a rigid body, then the position of centre of mass of th~
the hemisphere _as shown in Fig. remaining portion i~ obtained from the" following ~ormuiae: '
• ,...+ ·-+·
.. 4E.30. Radius of ring element is Ai:"r1 -A 2r2 ·
R cos 0. Mass of the. element is
,- A1 -A2'
·. A1 X1 - A2X2
dm -= mass per unit area x area of circular strip
M . , ", . .A1·-;-A2
= -·- x (21tRc6s0)Rd0 ~rY1 -A2Y2
21tR2
A1 -A2.
Then xCM =· 0- from symmetry
A1i1 -A2Z2
or ZcoM
. A1 .:..A2
' '
www.puucho.com
Anurag Mishra Mechanics 1 with www.puucho.com

'
.l ' ME~HANICS-1 J
. 2
Here m1,A1,r1,X1,Y1 and Z1 are the values for the
. ' .-
Area o· ·1
f crrcu . = -ita
ar ponon
. -t . . . 4
· whole mass w~ile m2, A 2 , r2 , x 2 , y 2 and z 2 are the values · 2 2
for the mass which has been removed. Let us see two an~ ·a~ea of square poriton = [ ar;;] = _a_
examples in support of tl):e above. theory. : · 2v2 8
illustration 13. Find If G1 and G be the positions of C,G. of the cut squar~
the position' of centre of mass of · portion and remaining portion.
· the uniform lamina shown in
Fig. '.(33.
. 1ta2 (0)- ~2 a (E.)
ThenOG= 4 · 84 32 -a
Solution: Here, 1tU2 . a2 .
( 21t8-1)" 4(21t -1)
A 1 = area of complete circle
. = 1ta2 -
4 8

A 2 = area or small circle,


·. The· C.G. · of the remaining portion is .at a
distance of a from the centre.
=1t(%r =!t:2 ·; Fig. 4.33' .. ·-~
4(21t- l)

.(x1 , y 1 ) '." coordinates of centre of mass of large circle


= (0, 0)
_and, _ _ !A uniform solid right circular cone has its base cut out in
!conical shape shown in Fig. 4E.32 (a) such that the' holfow is
(x2 ,y 2) =coordinates of centre of mass of small circle_. la right circular cone on the sam~ base. Find whauhould be,
. =(%,o) , ,the height of the hollow so that the centre of mass of the'

Using fnm,_;,rtwo•'Ylth-'~""?""""
we get ·
. -¥(%) . -(½) a
..xco"!= .2.ita"=(~)a=-6 .l Fig. 4E.32 (a) ••
ira -4 · 4 ,, '
Solution: .
andy~dM = 0asyl andy~ both are zero. 1 2h' y=- ·h
Therefore, coordinates of COM of the lamina shown in . M=p-1tr . (Before removal)
3 . ' 3
Flg. 4.33.are
. .
(:-E., a) .. :.
6- . M1 = p-1tr l 2h , h1
i, Y1 = -. (For removed part)
3 3
illustration 14. ,·A square hole i~ punched out of a ·. h' My-M1Y1
YCM = 1 = - ~ - -
circular lamina, the .diagonal of the square being the radius . M-M1 .
· of the circle. If.' a' •be the diameter of the ciicle, find the .
distance ofC.G. of the remainder from the centre of the
circle.
p.! 1tr2(!!:., _
3 4 4
hf)
· Solution : Consider r,:.......,~:::::::·::::::::::::~---,, =-..,,...~---~
1 2 .
the Fig. 4.34 shown p-1tr (h - h1 )
b'elow. · \et 'AB be the .
diameter passing through r~
l.
.., ,
3

the diagonal OB of the I . h1 = h1 + h ~ h1 = !!. '


square portion where O.is 1·/\,r-:-,:--"<Gl>---<00(.;r--tG>---<9!B 4. 3
the centre of the circle.
Portion where O is the
1
~~.;;_1ce,,.j 33 ~ : .
centre of the circle.
ipetermine th~ centre of gravity of a thin ho_mogeneous plate
Mass of the portions having theform of a rectangle wit!t sides r and ·41' from :which
can be replaced by. their. a semicircle with a radius r is cutout ofYi&: 4E.33 (a).~-_ _
respective areas .at their.
C.G..

www.puucho.com
...
,
Anurag Mishra Mechanics 1 with www.puucho.com

IMPUISE ANI> MOMENTUM 357·


-- --- ----·1
! Solution : We assume the system to consist of three
uniform square plates. All the plates have the same area,
C therefore the mass of eaclt plate is m/3. From symmetry, the
centre of mass of elicit plate is at the geometric centre of
eaclt plate. We cltoose origin as shown in the Fig. 4E.34(a).
\_ r B
The position vectors of centre of mass of eaclt of the plates
Fig. 4E,33 (a) are
-> •
Solution: xCM = m,x, - m 2 x 2 r1 = aj
-m2 m1 -> •
r2 = (0m)i
m1 ~ mass of the complete plate -> ••
m2 ~ mass of the semicircular r3 = ai
plate
The position vector of the centre of mass of the
Let density of plate is cr three-partjcle system, is
-
m1 - cr2r 2 '
->
7lT2 -+ . I.mir1
m2 =cr-.- rCM =-·--
2 . ix M

. ~ .1
2
2crr2 (~)- 0"7lT . 4r = ~[m aj + m(Oni) + m ai] = ~(i+ j)
2 2 31t m 3 · 3· · · 3 3
XCM ;:::;_--~--c.-cc-2--
2crr2 _ 0"1tr ,
2
L_ .!'.L(~-.-4E:::,.3_3--l(bi _ j ·Alternatively, we may use
• - I.mixi
XCM--- and YCM =-·-·-·
I:m.y-
·2r m ·m·
=---
3(4-1t) I m m m
-x0+-'-xa+-x0
illustration · 15. The position' vector of three _ 3 3 ·3 a
or X CM -
particles_ of l!'ass m1 = 1 kg, m2 = 2 kg and m 3 = 3 kg are m 3
m m m
r, =Cl+ 4j+fc)m,r2 ':' ci+ j + k)mandr3 = (2i-j-2k)m -x0+-x.a+-x0
a
respectively. Find the position vector of their centre of mass. and YCM = _3 3 3 =-
m 3,
Solution: The position vector of COM of the three
particles will be given by whiclt is same as obtained above. ·
"-+ -+ -+ . Method 2. The original system can be considered· to
-+ - -m1 r1 + m2 r2 + m3 r3
.
be the remaining portion if a square of side a is removed
rcoM =~~-~~-~~
m1 +m 2 +m3 froin a full plate of side 2a. The mass bf the large square is
Substituting the values, ·we get · 4m/3, while that of the removed portion is m/3. The position ·
-> (l)(l + 4j + k) + (2)(1 + j + k) + (3)(2i-]-.2k) vector of the tnetre of mass of the large plate is ·
rcoM = --~·~-----~-----~-- -> • •
1+ 2+ 3 r1 = ai+aj
91 + 3j - 3ic
=-~-- 1 • • ,.
-(3i+j-K)ni
-
while that of the·smaller plate is
6 2 . ,-, 3· <" 3 s
r2 =-at+-aJ
.~Xa!m.\BJjj 34 ~ 2 · 2
The system· can be consi.dered
. fLocate the· ;osition of ce~e of ,;ass of a unifonn.plate of, to be the .superposition. of a large ·
mass m, as shown. in.Fig. 4E.34 (a). plate and a small plate· ofnegative ·
Y a .. inass (- m/3). · Here· we have a
two-particle' system in which the '
contribution of the smaller square is to be subtracted.
1~1 ·. a -+ 'l -+ • . -+
· rcM =. -m (m1 r 1 - m 2 r2 )
2a -----1-----1----,
2 ·3 1 [4 · :·
-=(4m_m) 3m(a1+,aJ)-3lzai+2aj
s ·m (3 • 3·)]
0
2a
. 3 3 · ·
Fig. 4E.34 (a) 5 : 5 s
=-a1+-aJ
www.puucho.com 6 6
Anurag Mishra Mechanics 1 with www.puucho.com

MECHANICS-I )
..., .
.·li@·¥f~'~!I~~ . r1 = Ii
..., .,
. 1c~:is~e;. ~ ·disc of radius a wim'· u~~ m~· distrib~i/cm, . r2 = lj
!from whicha,arcular sectionofradiusb has been removed,·as
·. fshown in Fig.' 4E.35., The centre of the removed disc is at.a ·
. fdistan~e c.trom th,e ~ntr~-of thtf Ia,:ge disc. Find the centr;e of, 'The position vector of the centre of mas~ of the systen:i is

t:ass~ftheremammgd,sc.; ·· ½;·.: _: .-., ."·, given by


-+ 1 -+
.
-+:--+
-rCM = -(m1 r1+ m2 r2+ m3 r3)
M
.

.
\-'" '\- ·,·. -L ' "' 1 • • •
= -[2m1i + mlj + m(O)j] ='-(21 1 • + j)

4m 4
§E$i<a~~,Le~ ~-
Flg.4E.35_ !A rocbt e:g,l~des att:h~topmost point ~fits trajectory, 550 m

Sol~tion:· te'tcr
be the sud'ac~ d~nsity ~f ~ass; then
!from the point of projection. One of t~e fragments is found at
· la locatioJt 550 m east and 120 m _north of the launch point.
·. ·.· .. .<,·_-· ·ma~s'.ofentire,;lisc=ia(:1ta 2) · /SeFond fragment is found at a location .550 m east anq 65 m
,· '. •. ·' i . , iilass of remov~d d~c, = O'(~b 2 ) south.· of.th·e·. l.aunch point Fi~_,t two. fragments are ofeq_ual
· mass m ~nd•third fragment has mass 2 m. If all: tlte ·three
.The ;ystein ~ be considered: to be the super position
agrnents_ struck the ground ·simultaneously, what is the
of a full disc and a small disc of negative mass. · ~ocation~the thirdfr._agment? , : · · · '- ' · · ·,
• •., t :~·,'_ •' •. •I .', •-+' ---...... A' • •

. , _of ftill disc,' .: r1 =. Oi :I' OJ ·


Position' .vectoi: ~ Solution: Force~ generated by explosion are internal
;. _'c .~·, ',.,-',, ),-.· • '' -+•,A .. A•
· · forces. Motion takes place under the gravitational force even
· •PositJon vector of reiµoved disc, r.1 = ~ + Oj
. after explosion..Therefore the centre of mass of the system
':-:;o~ition v~ctor of sys;e~•~ l..cm1 r1:.. m,
'. . .' M
r~)- continues its parabolic trajectory, i.e., centre of .mass strikes
the. gr_ound a_t the-same place where the entire rocket would
'/- . . .. 'cr(ita~) x 0-c;mb 2(cl + OJ) have done, at a distance R = 1100 m east of the -launch
· >·", - ·a(ita 2 J'-a(itb 2 ) . Po.int. '' •i
' ; .' ·.-. . 2 . . . - -, -~.
.,. . = ,.;b C f z(m) Fragment 1
. .)!:I;:!
, a ~.:: b2
._, l'c
...---,-......... '
· _The.ef&re centre of mass of system is· to. the left ~f b. · ',
~
~e@/AA,ije~ . (North)

,
Y(m). Fragment 2 \ .
•·
".

' Frag111ent 3
550 x(m)
<;o~ider ~herr;artides connected 6y m~less. rods.. Find the (East)
location .of:thfcentre of mass of the,system of three po.rticl~ . Fig. 4E.:i7;
· . as shown_in F.:ig, 4E.36. · · ·· ·
~ ~~-~~-~~
m1X1, + ni2X2·+ m3X3
y . XCM ·-
. i -~- ··:,, ), ,•
. _m1 +m2 ·+m3
m .:-- ..;_ ,. c:itoom) = m(SSO) + m(SSO) + 2mx3
m
. 4m ·
or x 3 = 1650m
I s_linil;p-!y y CM =· m1Yi + m2Y 2 +_ m3y 3
·:1 :. .m, .·"
m1-+ m2 + m3
I Because y CM = 0, so . o·= m(120) + m(-65)+.2my3
· 2m
.. ·. ;(a) "' :• (b) 4
.. '·, Flg.AE.36. or y 3 = -27.Sm
,., ,,. . . . ·..
. '· . ·.. The~efore location_ of third fragment is 1100 m east and
-.~ Sbl!Jtlon _.: We__introduce ·a coordinate . system as. 27.5 m south of the launch point.
'· -shown in· !'ig. 4E.3~. ·Th~ position vectors of the three
. masses are' ..
www.puucho.com

'.
Anurag Mishra Mechanics 1 with www.puucho.com

IM~ULSE ANI> MOMENTUM c:,/i·, , •-' .. 351]

-----··-- --~ 38Q;>


L:ce~amru:el
-
IA wedge of mass ~ is kept on a sprilJf;- balance. A sniall. block
2
I~-- -r
eS,·'
!of mass ni1 can move. along the ftictionless incline of the '(.
"v"'•.- "
1wedge. What is the reading of the.lialanc~' while the block "
lslides? lg!IQ[!Lthe rec~i1 of the w~c/gt!.
Fig. 4E.39 (a)
Solution: We co nsider wedge and block as our sys tem.
Action-reaction force between block and wedge is'internal Solution: We assign ·the initial position of bead as
force. N x and NY represent reactions on the wedge by the origin. There is no external force along the x-axis; hence
balance. · position of CM will remain unchanged.
~ (XCM)initial = 2mx·O+ml =i ... (1)
. . 3m 3
(X l _ 2mx+ m(x + 1cos8)
: . ?.~·-0 CM final. - 3m · _ ·
'.
. g,sln B 3x+ !cos8 ... (2) .
=
t
g sin2 ,B
3
On equating eqns. (1) and (2}, we get
(bl l 3x+lcos8
I -=
3 3
I_,___ _ Fig. 4E,3B
1(1- cos8)
or x=
From Newton's second law, 3
Ny - (m 1g + m2g) = (m1 + m2l(aCMly .. :(1) Method 2. CM is initially at rest, in the absence of
. . m1 a1y + m2a2y external fates it will continue to ·be at rest .
where (aCM l y = - ~ - - - ~ ... (2)
· /111 +m2
.,
,,
(-g
= m1 ~in 2 8)+ 0 ... (3)
m1 +m2 ..., ....
Note that the block has acceleration of magnitude g sin 8 or m1AX 1 +m 2AX 2 = 0
along an incline y-coinponent of this acceleration is g sin 2 8. .... ....
where AX 1 and AX 2 denote absolute displacement of
From eqns_. (1) and (3), 1
masses m1 and m 2. Thus we have
Ny = (m 1g + m2g) + (m 1 + m2l(aCM ly
2 m(l - l cos8 + x) + 2mx = 0
·
=(m +m2lg+(m +m2l (-m
- =
1g sin .8)
· - - 1(1- cos8)
1 1 or x=
. . m1 +m2 .
.3
= (m1 + m2)g - m1g sin 2 8 . Conceptual discussion: A block is released on the
Similarly, Nx = (m1 + m2)(£icM lx convex-surface of a hemispherical
_ m a1x + m2cz2x
(QCM ) x - 1
wedge as shown in Fig. 4E.39(b). .
where
m1 +m2
We wish to determine the .Smooth!
displacement of wedge, when the urfaC8l
= m 1 (g siri.8cos8) + 0 · block reaches the angular
m1 + m2 position 8. There is no _external . . Fi~::E.39·(b;~
force in the x-direction, so
.... ....
m1AX 1 + m2AX 2 = 0
G"be~d of mass 2 m can slide on a smooth rod. A ·particle ofi m(Rsin8-x)-Mx = 0
mass m is attached to "the bead by a light string of length l. x=
mRsin8
or
Initially the particle is held horizontally in level with th_e bead m+M
and the string held just taut. Find the distance through:Which
the bead will move when the string has tumed through an
angle 8 witli the horizontal ·

www.puucho.com
Anurag Mishra Mechanics 1 with www.puucho.com

r.
:,=::,=====:,,e,;,===========~============
j 360 ~ M_ECHANICS-1] ,.i •..•• ':: ; "

This is equal to the energy expended by the man in


jumping,
¼.man of mass BO kg is ridi,ii o;
d trolley"of mass--,ip,,;; Conceptual discussion: If the man jumps with
velocity v 1 relative to the initial ,state of trolley the absolute
. '!which is rolling along a level surface a,t a-speed of 2 m/s. 'He
1wnps off the bat:k of the trolley so that his speed relative to velocity of man is
l_th!'. ground.is mfs ui tne directiqn,opposite to the motiq,;t pf,j
1_
-->
VmG =VmT+VTG
--> ...,
jthe trolley.• . . . : . . ·: e

(a} What is the speed of the centre of mass of the man-trql/ey IvmGI= -Vi+ V
1 : _,system bef9re and after his jwitj,s? · '- · '· :· '
(p) What.is the speed of the trolley after.the manjwnps? The equation of conservation of momentum is
(c) What is the speed·of the centre .<if mass of.the system.after
1
-m 1 (-v 1 + v) + m 2v = (m1 + m 2)v
( · the man hits the ground and comes to rest? - Similarly if the man jumps with velocity v 1 relati_ve to
(d} Whatforce is responsiblefor the change in vCM? · · final state of trolley.
(g) I-Iowmucliprgyfid the man%j1end injumping?
lvmGI= .'.vi +v'2
Solution:
The equation of conservation of momentum reduces to

Ii' j2'.~n
~ Initial state ,'Final state ·
-m1C-v1 +v'2)+m2v'2=·(m1 +m2)v
~~-e,.-.~-~----~-~-~L-!,e-.@-r·-4-,1] ~
.. • Fig. 41!._40 i IA,; exp.losive.ofm.'i.zss 6 kg is pTOJ.·e·.~t. il at 35 m/Sat an ang.' ofil
e_ le.

60° with th~ horizontal _At the.top of its flight it explodes,


(a) Velocity of CM of man-trolle_Y· system before the man breaking into two parts, one 'ofwhit:h has twice the mass o.fi
jumps, the other. The two fragments land simultaneously. The ·lighter
.m1V1 +m2v2
vCM =~~-~==v [asv1 =v 2 =v]
!fragment. larids back. at the laui1c1i point. Where' dpes the
m 1 +m? _other fragment land. What is the energy_~! the explosion?
~ ~ 1 ,, '"•
2 ._, ij 2
Before the man touches the ground, there is no external H=v0 s1rr =~
force, hence v' CM = v CM = v 292g· .,
R::; v~sin 28
- (b) Determine. ,the final vel~cities of' man · and g
trolley by._ v''1 and v'. 2 , we have, m m Path ofm 2
. from
.
cci!lservation
. of ~
Vo ....)........,.; after 13xplos!on
momentum,
-m1v(+m 2v 2 = (m1 ._+m 2 )v'
/·····r··,=:=:.:-·-...·
oi:,_R/2-R/2__;;, .. X
or v 2 =v+m1 (v 1 +t! 1 )=2~ 80 (2+1);=8m/s
m2 ·.,. _ 40 . Fig. 4E.41 (a)
(c) Tiie man conies to rest after hitting the. ground,
so the speed of the centre of mass of the system is · Solution: Just J:,efore thJ' ~plosion the projectile has·
velocity components vx = v cos0, Vy = 0 and is at the
-~'CM·= m1 xO+m 2v\ = 40x8 2 :67 m/s· =- topmost point of its trajectory. .
· · m1 +m2 . 80+40
From conservation of momentum along x-axis, we have
. (d) ·Due to the force of friction exerted by the·
(as ,J ]y = v' 2y c" 0)
/
Mv X- = m1v' Ix +m2v' 2x
ground, on the man; the velocity of CM is changed. ·
Fragment m1 will land back at the initial launch point if
' (e) While jumping, the .force'. between man and v'1x = -Vx·
· trolley is internal to ·the system. It has ·no influence on the
Therefore
. motion of CM. However, it changes. the total energy of the
, _ Mv~ -m1v'1x M +m1
system by the amount· . v 2x - . = --~vx =• M +m1 Vax
JIB = KE 1 - KE; . . . m2 m2 m2·

,2)
= ( 1 m 1v ,21 +·1 m 2v 2 - ·(1cm 1 +- m 2 Jv 2) Time ,taken by.m 2 to reach the ground
2 2 2 . t=i &= Voy

=l_·x 80 X1 2
~! X 40X 8 2
- !(80+ 40) X 2 2
·
'I~ g
· 2 · 2 2 · Distance covered during this time
=; 1080J _ ,
d. -v~t- -(M+m;) _M+m '.R
--~v~-----~
Voy 1
m2 g_ m2 2
,, www.puucho.com
Anurag Mishra Mechanics 1 with www.puucho.com

IMPULSE AND MOMENTUM 361 I


Coordinate of second fragment, + A system is defined
to include all the , · · _... --------.. ,,.SYste;;;-J
x= R +d=R +(M +m 1
2
=~R
2 · m2 . 2 m 2
)R particles taking part 1 I F
. ~1 ·.-._. /F'. 12 • •
. 2
= ~ X ( 3 S) ~in 60° +
in the collision.
Conservation of
'

I
',
·-··-.
m,. ..·.·
------"½
: '1

I
4 9.81 momentum principle
is applicable if the
l
FHa.p ;
= 162m; ·-. ·" i
where m1 = 2 kg, m2 = 4kg, v 0 = 35m/s, 9 = 30° total external force •••>=•.._system j
on the system is
The energy of tlie explosion is
either zero or can be . --·-\. --~~\.;~:__
____ :
AD 1 ,2 1 ,2 1 2
ur, = -m1vx +-m 2 v2.x --Mvx neglected compared • ,.. :-:.. ._He •: F
2 2 2 to forces present 1 - Total ···--· P- He.
2m1M 2 during collision. I,
=--Var Fig. 4.36 ,
m2 + If the total kinetic ------ ----------~
2x2x6 . ' energy of the particles is not conserved, the collision
= · (35cos30°) 2 "'5512J
4 is said to be an inelastic collision.
Method 2: The . internal + If the particles stick together after the collision, the
force of explosion does not. . collision is called . a completely inelastic
change of trajectory of CM. collision.
m1 X1 = m2X2. CM lies here·
b._~:xa~~~

X2 =-XR =-
4
R
2 .I
.
i.----X1

Fig. 4E.41 (b)


X2.,...

IA 'car of mass M = 25001<g ri,~-in_t_o_a-sm-aller car of -;,,~J


m = 1500 kg parked in traffic lane..The sliding wreckage!
leaves marks on the pavement for a distance of d = 5.2 m. If,
.
Coord mate of m 2 ·= R +-
R
= -3R ~ 162 m
2 2 the.coefficient of friction between wreckage and pavement isl
,µ k = _0.34, what was the limousine's initial sp~ed (Fig.:
COLLISIONS
Consider the collision of a bat
l1!l.,,1..?J1 _ _ _ _ _ ...... __ -·--· _ ---··· ---· I
with a ball and take the ball to be a \F;..,~.
n· Solution : The incident occurs. in two steps: first the
system. The force exerted by the bat cats collide and then the sliding wreckage is brought to rest
on the ball is considerably greater by friction.
than the weight of the ball, ·so that Sehlp: We choose the x-axis to be parallel to the
direction of the first cars incoming velocity. Figure .is the
the total force on the ball just' equals
the force of the bat during collision,
\ free-body diagram for the wreckage. We apply Newton's

L
1,·
approximately. The variation of this ~·- second law
force is s!Iown in Fig. 4.35. Forces
Flg.4,35
--=------.J LFY = (m+M)ay, Lf'x = (M + m)ax.
that act for short intervals are called impulsive forces. N-(m+M)g=o.· -A=(m+M)ax.
Examples are: a cube ball hitting an object, a pendulum liob .- The magnitude· of the friction force is fk = µ kN.
a
. released when the string is slack, comet deflected by the Combining these relations gives us
sun. -µk(m+.M)g = (m+Mlax.

• ·Toe impul~ive forces are assumed to be greater than


any external forces present. .
After the collision, the wreckage decelerates uniformly
with ax =-µkg.We use eqn. relating the acceleration a,,
+ In a collision, the individual momenta of the distance Ax = d, and change in speed squared:
particles do change, but the total momentum of the . vj -vr= 2axAx = '-2µkgd,
system of colliding particles does not. During where v f is zero and vi = u1 is the wreckage speed after
collision the forces of interaction are an the collision, Thus: · · · · ·
action-reaction pair, internal forces, for a system of U1 = ..j2µ kgd. .. . (i)
colliding particles. Momentum is conserved in: the- perfectly inelastic
+ Momentum of an isolated system just before collision, s6:
collision equals the total momentum of the system Px (after)= (M + m)ui
just after the collision. · = Mv1 + 0 = Px (before) ... (ii)
+ If the total kinetic energy- of the particles is
conserved, then the collision js called an elastic
collision.
www.puucho.com
Anurag Mishra Mechanics 1 with www.puucho.com

I~:=========--=--=::.::.::.::.::.::.~=-~=,
362 (,IEC!fiyi!ffi
-----------~--:-~---'-----'--'
~ · ~ : ., rn:_ The spring force is a conservative force so total
~~- •1• . , ?...Ji mechanical energy is conserved. Thus this is a model of an
· "'' •:- •· _ , " ' "" '. ' ' • elastic encounter. Llnear momentum is also conserved.
(a)
• : ~ Let's see what happens in steps?
~ . wfbru Step 1. The incoming block approaches the stationary
~ -----
' :W WM NJ2# Mif 1 1 11 -I · fsl1
object (state A).
,(b) . Step 2.- When block conies into contact with spring it is
compressed. Compressed spring exerts force··on both blocks
! ~~ ~ and it slows the incomlng block and accelerates the
"'""'!.'li·,lA5'2 'i\,~ :Z. block-plus-spring object to the right. When the spring has its
maximum compression· and both objects have the same
(c),
;; · +Y velocity. Spring continues to be compressed till both blocks
(d) ' L. ·attain common velocity i.e., v rel becomes zero. · ·
. (m + M)g x ·Step 3. The spring begins to regain its natural state it
L.._ _ _ _ _ _ _ _ _..,_,Fig, 4E. 42 _ __ exerting a leftward force on one block and a rightward force
on the other. Once the block on the left leaves the sppng,
. Substituting relation (i) for u 1 into (ii), the initial speed
forces stops actings and the collision is over (state C).
of the limousine is: · ·
We choose the x-axis to be along the direction of motion.
v,-_ (m + M)u1 ---v-'!'kgu
_ m + M ,,,;;-::;;d
Momentum is conserved throughout the encounter, and the
M M
--------- v CM of system remains constant.At, the end of step 2, state
' (4.0 x 103 kg)~2(Q.34)(9.8 m/s 2 )(5.2m) (B), both objects move at the·cM·velocity; ·
=
2.Sx 103 kg State A State B
=9.4m/s. = (m+M)vCM
MODELS FOR ELASTIC AND =
INELASTIC COLt.lSIONS
The kinetic energy of the two bl\)cks moving together is-
The terms ela.stic and inela.stic describe the' result, of a . ·
less than· the original energy of the· smaller block.· Some
collisiJl>n without giying any details of the interaction
energy has been converted to elastic .energy stored in the
between the particles. When objects collide, they exert spring:
forceJ on each other depending on the objects' geometrical
structure. The nature of those forces determine the changes State (A) State (B)
in energy that occur. Kinetic energy 1
-mv 0 2. 1 2
-(m+M)vcM
. A block of mass m and initial speed v O collides with a 2 .2
spring attacheµ to a stationary block of mass M. (Fig. 4.37). Elastic energy 0 _!ks2
Describe the state of the system when the spring reaches J\ 2
maximum compression and when it has .re-expanded.
___,.______ --------···1 1 2 1 1
-mv 0 =-(m+M)vCM
2 2,
2 1 2
+-ks
2

X
e:. .,,. +Y_
L.
.x
or -
1
2 -
ks2 1 2 1
= -mv 0 --(m+M)vCM.
2 2
2 ·

r-e-...; . ~1 2[
or -1 ks2 --mv 0 1
m(m + M)]-
~
---=,---+
2 2 (m+M) 2
VCM
or . 1 mM 2
=---Vo,
(a) (b) 2m+M

After the spring regains its natural state (C), the two
objects have velocities umi'and uMi We compare the linear
momentum and energy in states (A) and (C).
The spring is uncompressed in each state and there is no
elastic potential energy.
www.puucho.com
Anurag Mishra Mechanics 1 with www.puucho.com
)

IMPULSE ANI> MOMENTUM

State ("-) State (C) x-component of momentum:


Mass Before Px = mv 0 + 0
m M m M
After Px = mu 1 cose + mu 2 cos45°
x-component.ofmomentum mv 0 , none
y-component of momentum:
Kinetic energy_ 1 2 none
Before Py=O+O.
2mvo
After PY = -mu1 sine+ mu 2 sin45°
Set quantities equal: From conservation of energy, we get
Momentum
Energy
1 2 1
mv 0 = mum +MuM.
2
-mv 0 =-mu +7MuM
1 2
v~ = uf l!i + ... (i)
2 · 2 m 2 From conservation of moinentum, we get ·
These equations are the same as· those for the elastic x-component of momentum:
collision and have the same solution. With m2 = M. and v O = u 1 cose + u 2 / ./2 ... (ii)
~ 1 = m in eqn., we have : y-.tomponent of momentum:
m-M
um =---Vo 0 = -u1 sin0+u 2 /J2 ... (iii)
· M+m. From eqn. (iii):
2m
and UM= ---Vo- u 2 = C.J2)u1 sine ... (iv)
m+M Then, substituting eqn. (iv), into eqn. (i),
Analyze If M. > m,- then um-< 0 and the incoming
v~ = uf (l + 2sin 2 9) ... (v)
block returns to the left.
For ·an elastic collision to occut, the particles have to and eqn·. (iv) into eqn. (ii),
exert conservative forces on each other that is, have the v 0 =. u1(cose + sin9) ... (vi)
ability to transfonn and store potential energy. . Now, equating v~ in eqns. (v) and (vi),
(1+2'sin 2 9) = (cos0 + sin9) 2
k,~~!elm~~~
= 1+2sin9cos9 ·
Fig. 4'/:.43 shows two discs kept 07! a smooth horizontal (Remember: cos 9+sin 2 9=1.)' Thus sin8= cose, so
2

9 = 45°. It follows from eqns. (iv) and (v) that:


surface. Disc ~ ,is projected towards aisc 2 with velocity ~ 0•
. u1 =u2·=vo/J2: .
· After collisiqn disc,2 moves .at an angle e~ 45° with x<cixis.
Cal~ulate the magnitude of outgoing velocities of disc 1 and Two-Dimensional Collisions ·
· disc 2 . . ___ · The momentum of a

I
system of two particles during
" collisions is constant for· an
isolated system. This result is
Y 2 .-\~~:······ 1 ~fore2 ~50 ,
applicable in each of the
(!lll "x C!m-.- • • •---.=;-: direction x, y and z, as
momentum is · a vector.
m m 1 ;;'~ ,
Consider a collision between

~
two particles with particle 2
'"= ., at rest initially. After the
collision, particle 1 and
particle 2 move at angle e and
Solution: The incoming particles are free to slide on <j> w.r:t, horizontal respeGctvely.
the frictionless surface. Disc 2 is knocked at an angle to the Conservation of
direction of disc 1's incident velocity so the problem is two momentum can: be applied. in (b) After colliSion
dimensional. Choose the x-axis to be along the direction of 4.38
,-_ _ _..c.Fc.l,g._ _·,$_·.:
the component form.
disc is initial velocity and the y-axis as shown in Fig. 4E.4;!.
x-momentum:
We are give disc l's initial and the direction of disc 2's
·outgoing velocity. ni1Vux + m2v2ix = m1v1fx + m2v.2fx ·... (1)
Kinetic energy: y-momentum: '
Before 1 2
-E=-mv 0 +0 m_1V19' + m2V2iy = m1V1.6' + m2V2ty ... (2)
2 In our case, the above equations reduce to
After 1 2
E = -mu1 .+-mu 2
1 2
m1vu = m1v11 cos9 + m 2v21 cos<j> ... (3)
2 2
O= m1ti 11 sin.9-m 2v 21 sin<j> ' ... (4)
www.puucho.com
Anurag Mishra Mechanics 1 with www.puucho.com

, 364
where n_egative sign in eqn.''(4) appears due to the fact . two parts: At 1, period of deformation; At 2 , petiocj of
that after the collision particle 2 has a ,y-component of recovery.
velocity pointed downwards. ' . The impulsive force
If the collision is elastic, we can apply conservation of increases to a maximum value
kinetic energy. ' at the end of the deformation ·
1 · 2 -1 2 1 · 2 period and then decreases to
2m1vi; = 2m1V11 +2m2v2J
zero during the recovery period:
Oblique Collision : Common normal to the colliding In Fig. 4.40, P and R represent ,.,_t.t,----Aj,,~ /
· I·· , · ·c1 ·' , Period of, Periodo',
surfaces is called the line of . !_; un.pu
, s1ve. ,orces
d unng ·r '•deIormation recoveli)'•1. •
impact. If centre of mass of , de,ormatmn an recovery . , . _,
colliding bodies :lies oµ line of I periods respectively. 1 -Fig. 4.40- L.~
impact, the impact is central The coefficient 'of restitution (e) is defined as
_impact, otherwise. the impact is Impulse ofrecovery
e=
called eccentric impact. In a Impulse of deformation
head on collision (direct , • = (v's ).- (v'A )~
impact) the · velocities of the , Direct centralimpacf , J
(v A)n -(vs)~
colliding bodies are along the ' f Fig. ~.39 (a) j
line of impact. If the velocities = Velocity of separation along the line of impact
of one or both particles are at an angle with the line of Velocity of approach along the line of imp~ct
impact the impact is said to be an, oblique impact or glancing
collision. ~-
. .... - ·0'·m.)t1'~
Let .~s .consider an oblique collisi!]n betwe~n two •+, ~ : - - - ·• ,1 :·.'.···:.t~)~
particles. We assign different axis for oblique collision, · ..: Peiiodo(deform
..ation. /·. __,,]_ ·
, normal axis (n-axis) along line of impact and tangential axis
(t-axis) along tangent to surfaces in contact. We assume the
particles to' be _smooth and frictionless, so that the impulsive
(p"'+ -~·::~····
'· ·"- Periodbf/e·covEtry ." .d,
:\-
force exerted by particles on.each other is internal force fpr ; .
the system. These impulsive forces act along line of impact .° Fig. 4,41_'." ' .·
(n-axis). · -----;..,, · :< ..-------=---'-'-'-"·-------'·
+
Note that in fonnula· for' e, velocity components.
, .. ~.v'e n•axis \.
· along n°axis are substituted. •
t~i.ix1S 1
.:--·;·. ,'Line of
>,~ . s: , ' impact illustration 16. Consider collision between a block
~. ,' Vg A and ball B; the ,block is a
constrained to move along ..,.. "'-·--,,,----,-1,---~
'" , ,_.
,,
a horizontal surface. Ignore 1· ·.'.; :-. ·

, v· friction- at· any of the · • ,, v'A


• . . ,. A
Obli_Que cel'ltra't ·1_tnpact ·
surfaces. Impulsive. force .... VA
between block and ball is ··'4•. a 1
j"
'', • " '.,I:'
...:...,+-~+
~:-.w...-''---F!!![4,3~]'...._J_ _ _ __, along the · line of impact j .....'.:.:J:TITI:IIl:tl:JI__'. ·~
. We can form 'the following equations. and the impulse ofreacti<;m · 1· , • • . , • ':' :·•
exerted · by ground is 'in I: '· ·· Fig; 4.42 . ~ ;
1. Since no force acts along t-axis on each particle'. vertical direction, which is an external force on the system.
considered separately, the component or momentum along
t-axis is· conserved; hence t component of the velocity of· 'ball The
. following equations can be formed for the system of .
arid block:, , · · · ·.
each particle remains unchanged. Therefore, we obtain .
1. (v~), = (v's ),
{v A), = (v'A ),; {vs),= (v's ), ... (1).
_2. 1'otal. momentum of the two particles is conserved 2: mAVA_+ms(Vs)x = mAv·'A+(msV's·ix,
along n-axis. Note that ·we -have. applied l~w of conservation of
. mA(vA)~ +ms(Vs)n = m,i(v'A )n +ms(V's )n , .. (2) momentum on ·our systeiµ along x-axis only. Momentiun of
· system is not coµserved in the clirection in which a ·body is
3. From definition of. coefficient of restitution,
(v's ). -(v'A ln = e[(v A) •. -.:" (us).]
.... (3)
prevented from motion. . · · · ·
Coefficient, of restitution : In a typical collision 3. (v'B );- (v'A ln .=,e_[(v Aln -(vs)~]'
between .two bodies the force versus time graph is shown in·
the Fig. 4.40. We can divide the,total period of impact into . .'
I , www.puucho.com
;
)
Anurag Mishra Mechanics 1 with www.puucho.com

[i~PULSE A~ll_l\\O~ENTU~ - ---- - 365[


OBLIQUE IMPACT --- r--i
Oblique impact is the impact in which relative velocity
i,,}E~~~E?J~J 44 1,__>
of approach of the colliding bodies is not along the line of An elastic collision takes place betwee~ two masses m1 and
impact. m,., moving on a frictionless surface, as shown in Fig. 4E.44.
The-spring constant is k = 600 N / m

:~'--0-- .cC)axisG--C?S~-:
Before collision ' Ncrmal axis After collision l
- -
v 1 i = 4 mis

' 6 :::B
v2i = 2.5 mis

m 1 =1.60kg m2 =2.10kg
Fig. 4.43 Fig. 4E.44

Procedure for Solving The Problems (a) What is the velocity of the block 2 at the instant.block 1 is'
Step 1. Drawn-axis and t-axis at the point of impact. moving to the right with a velocity 3.00m/s?
·(b) What is_ compression of the spring at that instant? .
Step 2. Conserve the momentum of the system, along
and perpendicular to the line of impact, i.e., along n-axis and Solution: (a) From momentum conservation, we
t-axis. obtain
We obtain m1vli + m2 v 21 :;:: m1vv + m2 v 21
m1u1 cos0 1 + m2u·2 cos0 2 = m1v 1 cosp 1 + m 2 v 2 cosp 2 (1.60)(4.00) + (2.10)(-250) = (1.60)(3.00) + (2.10)v 2f
and · v 21 = -1.74 m/s
m1u1 sin0 1 + m2u 2 sin0 2 = m1v 1 sinp 1 + m 2v 2 sinp 2 ... (1) Negative sign implies that block 2 is still continuing in
If the colliding bodies are smooth, no force is acting on the same direction.
m1 and m 2 along the tangent; the momentum of m1 and m2 (b) Because no friction or non-conservative force acts
remains conserved along t-axis; on the system, we can Use conservation of energy equation.
i.e., m,u, sine, = m1V1 sinP1 ... (2) We obtain
1 21 21 2·1 212
and m 2u 2 sin0 2 = m 2v 2 sinP 2 ... (3) 2m1V1; +2m2V2; = 2m1V1/ +2m2V2J :"zkx
Coefficient of restitution is defined along line of impact
only. On substituting numerical values, we obtain
e = v 1 cosp, -v 2 cosP 2 ... (4) X= 0.173m.
u 1 cos0 1 - u 2 cos0 2
Now we have four equations and four unknowns
[J:~~Q._t;QjB~c:~ ,i45li>
v,., v 2,P 1 and p 2. Solving the four equations for the four Consider two particles that undergo an elastic collision on a
unknowns; we obtain frictionless surface as shown in Fig. 4E.45. One particle of
A (m1 + m2)u 1 cos0 1 + m 2(1 + e)u 2 cos0 2
V1 COSp1 = -~-~~-~-~--~-~ ... (5) mass m2 is at rest initially.
m1 + mz
A m1(1 + e) cos0 1 + (m 2 - em1 )u 2 cos0 2 ~I~
Vz COSpz = Before collision
m1 +m 2

~ 0----;,
After collision
tan Pi = ~ = Final tangential component of velocity Fig. 4E.45
v 1n • final normal component of velocity
(a) Find the velocity components v 11 , v 21 of the particles
Similarly we can find v 2 and p 2. after the collision. Discuss the results.
,Remark: . .. -- - - - -- .
(b) if m2 » m1, (c) if m1 » m,., (d) if m1 = m2.
Impulse= m{rl 2 (1+e)(u1 cos8 1 -u2cos8 2) Solution: (a) In this case, both the momentum and
m1+m2
kinetic energy are conserved; therefore we have
Energy loss= m{rl 2 (1-e 2)(u1 cos8 1 -u2 cos8 2)2 m 1vi, + m 2 v 2, = m 1v 11 + m 2v 21 ... (1)
_2(m,-i:m 2 ) _ _ ,
1 21 2l-2l·2
m1 vu + m2 v 2, = m1 vv + m 2v 21 •.. (2)
2 2 2 2

www.puucho.com
Anurag Mishra Mechanics 1 with www.puucho.com

.MECHANICS"! j
From eqn. (2),
m,(Vii-Vi1)=m2(V~f -vt)
or m1(vli -vlf )(vli + vlf) = m 2(v 21 -:v2i)(v 21 +v2i) ·
... (3)
From ecjn. (1),
. m1 (vli:-vlf)=m 2(v 21 -v2i) ... (4)
We divide eqh. (3) by eqn. (4) and obtain
. vli + vlf = V21 + v2i
(vii -v2i) = -(v,1 ·-v 21 ) ... (5) 1 2 4m m
Eqn. (5) shows that relative speed of the two particles ----~c--=~--
2
(m, -m 2) +4m1 m2 .
before collision (vli -v2i) equals the negative of their 1
relative speed after the collision, -(vl/ -v 21 ). Now we can The transfer of KE will be maximum when denominator
solve eqns. (1) and (5) to obtain · · · is minimum,
. i.e., m1 = m2
v,1 =·(m, - m2 )vli + ( . 2m2 )v2i : .. (6) 1
. m1 +m 2 m1 +m 2 If m1 = m 2 ; ~ = 1; tra~sfer of en~rgy is hundred :per .
v 2~ =(_ .2m,. )vli+(m 2 _-m1 )v2i · ... (7) cent.·
· m1 +m2 m1 +m2
The ·above · results are very important in solving
problems of one-dimensional collision.
Consider bvo particles m 1 and m2 · that und;rg~ p"e,t;~tly
If particle 2 is initially at rest, _then v 2i = O in eqri. (6) inelastic collision.
and eqn.. (7). ·
. (a)' What is. loss in kinetic energy during" collision? .
v,1 =(m' -1'.'2)vli :(b) Whatis:frictionless changetn,.kjrtetic energy? ·
. . m, + !112 (c)0)iscuss _the resultform 2»:mJ.Jmd v2i - O<--·----'
· (. lm ) ' · · .·Sohiti~n: After collision two particle's stick together
V2f =
. m1.+m2
V'l
. and move with common velocity v I after collision,
(b) If m2 '>> m1, v 11 = -vli · From conservation of momentum,
and v 2/ = v2i. ".' 0
m1 v 11 +m 2v2i = (m1' +m 2 )v1
When fl very light particle collides head on with a very . - ..:_ !111:VH + m2V2i.
or v1 - . .
heavy particle that is initially at rest, the light particle has m1 -:t-m2··
velocity reversed and heavy particle stays .at rest. . Initial kinetic energy of the two-particle system is .
(c) if m1 >> m2 , vlf ": ·v,i . . ., 1 2 1 ·2
~i = m1 vli + 2m2v2i
. V2j = V2i 2
When a very heavy particle collides head on-with a very Final kinetic energy of system is' .
. . 1. .
· light particle that" is initially at rest, tlie heavy particle
1., E1 _~ (m; +m 2 )vJ
cqntinues its motion unaffected after the collision and the 2
.light particle rebounds with a speed equal to about twice .. Loss in kinetic energy is ·
. the initial speed of the heavy particle. This would .happen
when a moving heavy. atom, such as uranium, with -a light
... .KEi-KE/ =
[(l m1vli+
2 1· 2)
m2v_2i -
1 • ;]
(m1 +m 2 )v1
-ato!]l, such as hydrogen. 2 2 2
(d) If m 1 "'. m 2 , . vlf =_v 2i _ 1_ [ . 2 · 2 ' (m1v 1;.+ m2v 2;)2_].
- - m1vli + m2V21
V2J = Vti . 2 :. (m1 + m2 ) •
· · When particles of equal mass collide, they exchange
velocities. ·· · · c' _![m1m2(vii +vt-2vliv2i)]
2 m1 +m 2
(e) Kinetic energy transferred from projectile· tb
target: 1· m11(1 2 2
= -~~-(vli-V2i)
2 (m1 +m 2)- .
Ifv2i=O,

www.puucho.com
Anurag Mishra Mechanics 1 with www.puucho.com

_1_MP_u_1S_E_AN~il_.m~o~.fn_E_N_T_U_M_ _~-·~· -----"-=------------~-~---··_·•;::as1 I


and
"-EE-: .......,.,_;-:--ra;i4s
i.;.i!-:~--~~!B!#'iiih">~\, ~
·:--,.,..
~-- -- ----"AC,-------~~~---~ ___ ...._ -----~-----,~· '"
IA .bullet of ,mass m1 is fired into .a .large block o/mass m 2
isuspended fro.m .wires. 'The coUision. is perfectly inelas((c, so
which implies that if a light projectile strikes a heavy !that the, combined system swings through a height h, · ··• . · ·
target, the entire kinetic energy is lost in the collision. ra) What is .the initial speed of th.e bullet?

'T~,... ·;:;:~1
'(b) If the ballet emerged from the.block with half of its initial
~~~~~~
. ~.a..
iCo,;;;de~. a.
~ne.C.dirnensional elastic; C.ollision b~tween ; ;;-.e~
incoming bodjJ dnd body 2, initially at rest.Bow would you
choose the mass ofB 'in comparison to the mass of A in order
that B · shoµld '.re.co. il w.ith ~. a) gr.·lat~'st speed, (b) grearestj'
momentum, (cJgreiltest kinetic energy?
-~· '
____ .
""""' -----
I m
1
-,=.=71 ...............
I ~~m_,~· ·:;;·_-····-+
~

. ·JL
Solution: Since the collision is elasti~, energy as well
as momentum are conserved. ---~--·~l,~~~~==-- -~-·4···~·
· Solution: (a) There are two parts in the problem:.
m1vli = m1V11 + m2V21
. riI2 (i) Collision between bullet ·and
V1; = V1J +-V2J ... (1) ' block: Impulsive force exerted by bullet and block is very
1111 . large, so that we can neglect all the external forces on ·the
For the sake of simplicity we take a.parameter system of block and bullet. · ,.
.·k= m2 · ·Froin conservation of.~omenti.tm, ·, ·
! • • '
. m1 ··' ,m1v;, =.(m 1 + m 2 )vf
Eqn. (1) reduces to · vli = vlf + kv 21 , " (m1 + m2)
Vli =~---VJ ... (1)
1 2 1 2' 1 2 m1 . , .
and m1vli = m 1vv + m 2v 21
2 2 2 (ii) The combined · system (bullet block) +
2
V1; = 2
V1J + kv22/ , .. (2) rises to a height h: We will apply·conservation of ei1ergy
Now we ~ubstitute.the expression for v11 in eq~. (2) to after collision. ·
obtain · · 1 Cm1 +.m_ )vf2 = (m1 +m 2 )gh
. ·. 2V1·
2
2
V21=--' Vf = .J2gh , :.(2)
l+k
(a) Particle .2 will recoil with maxi1,11um speed·when k. From eqns. (1) and (2), ·
_is minimum, i.e., , . ,· . vi,= (m1 + m2 ). .J2gh .
k """?. 0 or m 2 <<m1 m1 . .
(b) Momentum of particle 2' is· (b) On the pattern ~f pari: (a) .;ye may , write
2km1Vli conservation of momentum equation (for block and .bullet
P2 = m2v21 =-~~
. l+k. system) and conservation of energy (for; block) ,after
Particle 2 will recoil with maximum momentum when collision. , ·
,denominator is· minimum, whici1 is possible if k-, oo or
.m2 >> m1.
m1~ 1 /= mv mi.(½"vli) ..
2 2;_; ,.(3)

(c) Kinetic energy of particle 2 is . . 1 . 2 . ~ ,·


and m 2gh= m 2v 2t· •, ..-(4)
1 , 2 , .f · . ( 2vli ) 2 2
K2 =2m2v21 =_2km1 l+k from eqns. (3) and (4),
. ~ 2 ~
_ 2m1v~,k 4{KE 1 )k · Vzt m1 Vu
h=-=,--
- Cl+k) . Cl-k) 2:+4k
2 2g Bmfa
Particle 2 will recoil with maximum kin~tic energy when
denominator is minimum. ·
i.e., k = 1 or · mB = in A
, .
www.puucho.com
Anurag Mishra Mechanics 1 with www.puucho.com

.,,
Solution : , ,; , '
,•1 ~· rI, , illustration 19. A ball of mass ,m moving at a speed
(a) By momentum conseiyation, v makes a head on collision with an identical ball at rest. The
2(4)-4{2)"' 2c.::.2) + 4{v2) khte'tic' energy
'
of the balls after the collision is 3/4th of the
=*· v 2 ,= lm/s origin~l. Find tlie coefficient of restitution.
'' velocity of separation 1- (-2) Solution ·, As we have seen in the above discussion,
(b) - e = - - -- thaturider the given conditions: ;·,_,, .ri1
velocity of approach 4-(-2)
(c) At maximum deformed state, by conservati_on of
momentum, COffimOn Velocity Js V = p, , •;:; '
· · J 0 -=m1 (v-u 1 )=m 2 (v-u 2 )
1- _=-2(0-'--'4),=-8-N-'s '' - , ,·_;,, I~,
= 4(0-2) = -·8N-s
or =; 4{0-2) = ~8 N-~-
' (d) Potential energy_1 maidrriufu deformed state;
-u ,; lo;;~ iii kinetic -eiiei-gy during deformation
or U;, (:! rri1u,2 't- .!__m 2u~ ).,-- _! (in~ +m 2 )v 2 · - and
l: ~- .. ·_. 2 ' ;:?-

.,,,.
' .'ll- i'
,:~';;- ·'2'2_(4) +24{2)(-, -2(f+4)(0)
,1 '. 2 )' 1 2
,,,:· Given that
. , 1',d it ,.,; r 11 ' ,- , ,, . ' ,, ,_,.) ·r1 .."1"::• ,,
or __ _[f = 24j6tile ". ·: ·, or
(e}, J~_;,,_,ri}1('!_; ~m2(V '.'.:,~2) iv) ,'·l·
, ._ ~
1
2'
1
.' ' 2
=2(-2-0)=-4N-s.
) ) _.).ll.: , ' - :, • '
-
•••.• - '.. l . Substituting the v~.1!~, w~ g_et ( : e)- +( ; e) = ¾
or =, 4{0-1)
•• : ,
= -4 N-s - J
,_ I ' ~
1
or (1 + e) 2 + (1- e) 2 = 3
,; 1...or1I _e1= ~R ', .......=> JR_.~-eJ~-.-~- ,•,' :,
. \ .,· ....{D' .- 2+2e 2 =3

.,
·,' or
' 1
., , = (0.5)(-8) =-_11'1-s, ·.: __ or 2
e., -= -1 · · or e=-
illustration.'18: A ball,is --- . JJ -,.,---n,.:,,?'71"~=,,,,1 2 F2

8
•••• ~t,,.¥-,-'t
moving , with velocity 2 m/s . '· ,. '. \ · ,, ·,c :
. Th~ Velocity of the Center of . ' Mass for . Collisions
. . - ·
'towards\ 'a 'lieayy wall moving. . ,, ,-:·~-.::,:-:;;;:} '.J When various masses collide in-a complicated manner,
.. .

fowards the_ball with speed 1 mh, t ~5 , ::••1'ri\1s t_ the velocity of the center of mass' unchanged, if the external
as, shown in Fig. 4.45 (a): F ~ ·: _:, " ~
Assuming collision to be elastic, ,.-"· ;. -.:·,: ;, , forces on ,the _system. of colliding particles during the
I'' ·1 • I 11 ' ,,-J 'i::
' • 1
find·. the velocity of the ,o(llr· h. ·
1
',,,:';.,,±,.<el
l
collisic5n are negligible: compared with the internal colli;ion
immediateiy'after the collision.
, ··, ' -
t,'·:- .,Flg, 4:4~1'.'L'i:l.'[J , ; . - 1· -, '
forces._ .
• . . . ,·,
·
I
Solution : The speed of wall will not change after. the Consider a system of two,particles of masses m1 and m2
- . ' .
collision, So; let:UJ,e the velocity.~fthe ball after collision'in moving.· with· velocities '.111 and v~/ , ·respectively, and
the direction shown in Fig;'4.45 (b), Since collision is elas):ic
(e = l)/,1- ·, ,_ ,1 ~ , :,:,~c , -1·, subjected -to zero total external force. Some times the
'. ,r;-~~,il~l;'r.:-::::,,,c:-:';=c;::;;;~r~1:-:,:,:--bt"c<;o~~lfJ~.:J;Cl71 external forces are very small, they can be neglected·,
·,.., The · position vector of. the. center of . mass of the
two-particle system is, · ·
~ . ..·,1 . - --+ ~
rcM
.
= m1 +m2
-- - (m,
,.
r1 + m2 r2)
-
The velocity of the center .of inass is

, ' )- .' /'.'. -


separation
.
speed
'
= appro~tjl
,. - -~ - - . speed ,
or /" : o/ ~, .
·
I"
v-1=2+1
or ,V_ ''.':'T m/c5: C . - . ,_.,

www.puucho.com
---· __________ .,.., ---- ., ...:,........,...... __
_,... . ____ - . ------·-
Anurag Mishra Mechanics 1 with www.puucho.com

IMPULSE AND MOMENTUM 371


y
m,

m,

Fig. 4.48

m, , For two-particle collisions (Fig. 4.48), the particles' total


momentum in the CM frame is zero, so their incoming
momentum vectors are equal and opposjte. The particles'
Fig. 4.47
outgoing momentum vectors are also equal and opposite.
Rearrange this slightly to find
Concept: Conservation of kinetic energy in an elastic
-> -> ->
( m1 +m 2 )vcM =m 1 vli+m 2 v:,; ... (2) ,collision ensures that the magnitude of each particles'
The terms on the right-hand side of Equation (2) are the momentum of the CM frame is unchanged by the collision.
momenta of the individual particles; the term on the In case of two-particle elastic collision in the GM reference
left-hand side 9f the equation is the momentum of the center frame of the particles total momentum of a system in its CM
of mass, if we imagine the entire mass of the system to be frame is always zero and kinetic energy is conserwd, each
concentrated at that po'nt. particle has the same speed after the collision as it did before:
-+· -+ -, -+ -
If the two particles undergo a collision and have velocity Iu 2 1=Iv 2 ~ IU 1 I=Iv 1 [ In a two-dimensional elastic collision,
vectors v,f and Vzf, the total momentum after the collision. ;only the angle 0. - is unchanged by the collision between
is ;incoming and outgoing velocities is not fixed by the incoming
. __, ->
-> __, velocities. In Fig. v 1 and v 2 are approach velocities of
m1 vlf+ m 2 v 2f

But we know that momentum is conserved in all i_ncoming particles and u and. u are outgoing particles.
1 2
collisions where external forces are negligible compared
with the internal fores arising from the collision.,Therefore CONCEPTUAL EXAMPLE 2. Show that the
--) -+ -+ ---+
Vu+V2i = ml V11+ m2 V21 magnitude of each particle's momentum in the CM reference
frame is unchanged when the two particles collide elastically.
Using this equality in Equation (2), we find
Solution: Can•be expresses in terms of its momentum:
1 l
K = -mv 2 = -(mv) 2 /m = p 2 /(2m). The total momentum
-> -> 2 2
= m1 vlf+m 2 v 2f in the CM frame is zero, so the particles have equal and
This means that the velocity of the center of mass is the opposite momenta and both before and after the collision;
same before and after the collision. -+ -+ -+ -+
IP1_,l=IP,,;I= Pi and IP1,f l=IP:1,fl= Pf ... (i)
Although we derived this result for two-particle
collision, the result is true for any number of colliding The total kinetic energy K of the system remains
particles because conservation of momentum, is true for all constant.
collisions (with negligible external forces). Before: K P1,;2 Pzi2 P, ( 1 1 ) C')
11
Elastic Collisions ·in The CM Reference Frame
= 2m + 2m
1 2
= ; 2m1 + 2m 2 ...

The incoming velocities completely determine the


outcome of a one-dimensional elastic collision. We obtained After: K = P;'j +. P{j = Pf ( -1- + - 1-) ... (iii)
· 2m 1 2m 2 2m1 2m 2
an interesting result for two-dimensional collisions, the
angle between the outgoing velocities is fixed, while the Equation (i) was used to similarly eqns. (ii) and (iii).
absolute direction of either velocity is not. Eqnating K in eqns. (ii) and (iii) gives P; = Pf, as required.
Since the magnitude of each particle's momentum does
not charge, the only unknown is its direction. Which can not
be fo,md using the conservation laws alone.

www.puucho.com
Anurag Mishra Mechanics 1 with www.puucho.com

[_@ ____
Inelastic Collisions in CM Reference Frame (a) Find velocity of centre of mass
An elastic collision is an ideal model. Even when things (b) Maximum extension in the spring.
like billiard balls interact, a small amount of kinetic energy
is converted to other forms. The opposite ideal case is a ·v+--[ni} 000000~000000 i2m]--+2v
perfectly inelastic collision, in which the colliding objects A B
Fig. 4E.53
stick together. The CM reference frame allows us to give a
~~~:_pr~?se.~e~i.tion: . 4mv-mv
' Solution: Velocity of CM vcM = - - - - = v
. Concept: 1. In a perfectly inelastic collision, all the 3m
1
kinetic energy of the incoming objects in their CM reference In COM frame. Initial momentum =0
frame is transformed to internal energy within the outgoing at the time of maximum elongation both the masses will
:objects. be moving in same direction with same speed.
I
; 2. After · a perfectly inelastic collision, the outgoing Initial relative velocity v ,.1 =3v
:objects, viewed as particles, have no kinetic energy in the CM, Decreasing in KE = Increase in PE of spring
'frame. They are all stationary in the CM frame-stuck 1 2 1 2
:together. In the lab frame the system still has its original, -mvrel = -kx
2 2
:nonzero linear momentum and thus has some kinetic energy.
1
'f11~.J>g_tgqjr1g_o]Ji,ects_ move together with their CM velocity.
1 mx2m( 3v) 2 =_!kx 2
2 3m 2
[;Ex;ca,t:ri,~le I- 521 }I-~
§.- ---,.:::.=~~l!f~:--,...,-~.:.:.-.,
">, 3mv 2 =_!:_ kx 2
r - - - . ··--
2
:nvo particles _of mass mi, m 2 moving with initial velocity u1
!and u 2 colli~d,-head-on. Find minimum kinetic energy during,
:collision. ·Thus prove that maximum kinetic energy is lost in
iP!!.rfe~tly_i11elas_tis; ,o_llision. Concept: 7\vo identical blocks of mass m, each are
~ u 1· · ~ u2
connected by a spring as shown in the Fig. 4.49 At any instant
Solution: cv·········~ . of time t = 0, one block is given a velocity v 1 and other is given
Fig: 4E.52 a velocity v 2 ( v 1 > v 2 ) in the same direction simultaneously
as shown in the Fig. 4.49. The maximum energy stored in the
In C·frame initial kinetic energy of system.! µ(v 2 - v 1 )' . rs
spnrzg . given
. b!}' -1 m( v -v )2 • '
. 2 4
1 2

where µ = mi m, . During collision at the instant of v,


m1 +m2
maximum deformation we get minimum kinetic energy in
C-frame as they attain same velocity thus no relative
·,mJ::l::=::~v2
-- k

smooth
velocity. When system have minimum kinetic energy in Fig. 4.49
c.frame it also has minimum kinetic energy in ground frame
·as velocity of CM is constant. ·
K0 =.!:.µv~.1 + :!.m,v~
at maximum deformation. Thus
2 2 A smooth ball is dropped from a height hon a smooth incline,
minimum kinetic energy during collision is .!cm,+ m 2 )v:, ,as shown in the Fig. 4E.54 (a). After collision the velocity of
2 the ball is_ directed horizontally.
(m 1u 1 + m 2u 2 )
w h erevc =
m1 + m2
In inelastic collision final kinetic energy is .!:.2 m,. v ,.2 of CM is ~h
constant.
~--- -
~~~~jt~,:i 53
·r--.., __
p ,··..-ii
ucose u
0

/;1fo blo;ks-A a~d B. ~f musses ,11 and 2m placed on smooth Fig. 4E.54 (a)
Ihorizontal surface are .:onnec'.ed with a light spring. The two
iblock§ are given velocities as .;!wwn when spring is ~t natural· (a) Find the coefficient of restitution.
l~11gth~ -- (b) Ifthe collision is elastic, what is the impulse on the ball?

www.puucho.com
Anurag Mishra Mechanics 1 with www.puucho.com

-~-----, - -· ~-
t tMPUISE AND MO~lN_T_UM_~_
Solution: (a) Normal axis and tangential axis are Time taken to return to the point of projection after
shown in the figure. Reaction of the incline is along n-axis d
impact =----
and in the absence of friction there is no force along t-axis; ev 0 cosa
therefore velocity along t-axis remains unchanged, Note that x-component of velocity after impact is
i.e., V COS8 = U sin0 ... (1) ev 0 cosa.
From the definition of coefficient of restitution, d d
Total time of flight = - - - + - - -
vsinB
e= ... (2)
-ucose = ;~::a[~v:;ra
Or V = eucot0 ... (3)
From eqns. (1) and (3), There is no change in the vertical component of the
(eu cotB) cosB = u sine velocity after impact, therefore total time of flight remains
or e = tan 2 0 unchanged. ·
(b) When the collision is elastic, the component of 2v 0 sina sina
T = -~-- = ~---
2./ifi
velocity along n-axis is reversed in direction. Therefore the g g
change in velocity
t.v = 2v cose d
.JiFi
cosa
(1 +e e) = 2./ifi sin a
g
or (1:e)= hsi~2a
d
or

u 0
Fl~. 4E.54 (b)
A pa~cle ~ throw~fr~m-~h;~,;~-h-horizon~cill~-;,~-;,~ al
There is no change in velocity along the t-axis, therefore
no impulse along t-axis.
·vertical wall movingmvay with a speed.v shown in the Fig.' as
4F..56 (a) ..If the particle returrrs:to· ihe point' of projection
. Change in momentum of the ball after suffering _two ·elastic collisions, one with, the waWand
= mt.v ,another with the ground, find. the total time of fi.ight· ahd/
= 2mvcos0 initial :
!
- --- - ·'· --· - ---· __ ::_
~ . ' •
I
Velocity of the ball when it strik~s the plane= ~2gh. • V _ --.v/4 .
Thus impulse = 2m~2gh cose .
• __ _ --;· -- • u • f ----1.._

k~~~'Ji'J:>.~~:J _~~~;.> h X

- • --- • - , I
,,
-An inelastic ball is projected with velocity v 0 =..fiFi, at an • ______ Fig. 4E. 56 (a)_____ / ·,
'angle a to the horiz@tal, towards a wall distant d from the,
SeparqtiQ1J _xJ,_e~ee11. tl!e_partic(~ C!tlrl th.e_ wall~~-,_;1J_.::._
:point of projection. After collision the ball retums to the point'
!of projection. \'\!hat is th_e ~qefficie1JUJf_restiiution? - Solution: While colling with wall x component of
velocity gets charged while component remains same. We
Solution: Time taken to reach the wall
have
d
=---
v 0 cosa

v0 cosa=vx
Velocity before impact
.... .. :.·b

'..:J"y
ev0 cos-a= Vx
:o..,_d ______.. Velocity after impact h ,14··_ _.,,x'---I,
-1
Fig. 4E.55
www.puucho.com
F;g,
"
4E: 5~.(~) , ' ·
- •••. ~- - ,, __ ,,._ ·-
I
j
Anurag Mishra Mechanics 1 with www.puucho.com

rai4 - .----- -- - --- - - -- - - -MECHANICS-I :


--- -------- - - - - - - - - - - - - - - - - - - - --------------·--· - ---- - -~
It should be noted that time of flight will be 'it', where t
is time of ball to the ground and first collision must be ~~=~~!ieI~ .(5al>
occurred with wall and second with grouod. Whereas if. wall
was moving towards the ball then, first collision must occur :A ball is projected with velocity v 0, at an angle a to the
at ground and second with wall. horizontal, towards a smooth wall which approaches the ball·

Time of flight= 2 x.ff . :with velocity u. After collision the ball retraces its path to the'
:point of projection. What is the time t taken by the ball from'
!the instant of projection_ to ]?Dint of impact?
Let separate between point A and wall is x 1 when ball
u
hits the wall
X1
V
+~=T=2fg{2h
V/2
time taken by ball to cover this distance

t1 =; =%Hi v 0 cos a

Fig. 4E.58
Initial separate in x = x1 - ~ t1
4 Solution: Since the wall is smooth, the vertical
component of velocity will not change, as explained earlier,
=(%-~x¾}~=vHi· the tangential component, remains unchanged. Collision is
elastic, therefore coefficient of restitution is 1.
Relative velocity of separation
e=
Relative velocity of approach
V
l=
(v 0 cosa+u)

~:~=:~;J':
• --- 1

,A ball is shot in a long hall having a roof at a height of 15 mi Thus relative velocity of separation v = -(v 0 cosa+u)
Velocity relative to ground = (-v 0 cos a+ u) + u
Time of flight depends on the vertical component of
M<h ,p,ro <heflOOS ~"
velocity which is unchanged.
T = 2v 0 sina
g
X
Let time taken before impact be t, the distance covered
. . -~-~~~ - ! before and after impact are same,
ball lands on the floor at a distan'f'shown x =__ mfr<;>m! v 0 cosat = (T-t)(v 0 cosa + 2u)
,the point of projection. (A§sume_cq(lisions as elastic ifpri:y)_ ,J
t(v 0 cos a+ v 0 cosa + 2u) = T(v 0 cosa + 2u)
2
Solution: y = xtan0- gx or t = (v 0 cos a+ 2u)v 0 sin a
2u 2 cos 2 0 g(v 0 cosa+u)
r-· -- -- -- - --· - -,--i-
ts'o~R=~''.it\M3t!!irJ 59 1i>
I - - --- - -- - -- -- . . -
,A smooth ring is kept on a smooth horizontal surface. From a
:point P of the ring a particle is projected at an angle a to the
x=15
Fig. 4E.57 (b) :radius vector at P. If e is the coefficient of restitution between'
',the ring and the particle, show that the particle will return to
1Oxx 2 x25
4 ;the point of projection after two reflections if
=> 15=x·-
3 2X (25) 2 X 9 : 2 1 1 1
cot a=-+-+-
2 . e e2 e3
=> x - 6Ox-t-15 x 45 = 0 L - - - - .

=> x = 15, 45 (45 is rejected) Solution : Let u be the velocity of projection at P. We


Tora!= 30 m can find the velocity of rebound at point Q from Fig. 4E.59
(b).
usina. tana
tanp =--
www.puucho.com eu cosa e
Anurag Mishra Mechanics 1 with www.puucho.com

! IMPULSE-AND MOMENTUM
L... ____ - - · - - - - - ·-- -- --
-
-- - --

and v = ~u 2 sin 2 a+ eu 2 cos 2 a. Note: that friction between A and B will be


vsinp tanp tano: non-impulsive therefore we ignore it.
Similarly at R tany = - ~ - = - - = - -
ev cosp e e2 Also e =1 => v=v 2 -v 1 "' (2)
2v
and w = ~v 2 sin 2 a.+ev 2 sin 2 a (1) + (2) Vz =-
3
R Using COM for (A + B)
2
2m( ; ) = 3mv 1
4v
VJ=-
9

( 3 )2--(3m) (-4v )2 =-mv


1 2v 1 42
(b) Af.=-(2m) -
2 2 9 27

(a)
_ _____- - ~P•c¥;
(-----
-.Exam·
······-- 'le..'J, 611
61 }:.->' 1·---:-,_.

A small ball i5 projected from point P on floor towards a wall'


u*s~a 5
u sin a)/(v u eu cos a
~
,as shOVfn in Fig. 4E.61 (a). It hits the wall when its velocity i5
u
'

··.. ,~/
I
;horizontaL Ball reaches point P after on bounce on the floor. If
','Q Ithe coefficient, of restitution i5 the same for the two colli.ions, !
,:····· ·····._t-axis ,/a, '(i-nd its value_
uCos a ,'
Velocity components Velocity components
before impact after impact

Fig, 4E,59 (b)

Since the particle returns to the point of projection, Fig. 4E.61 (a)
o: + p+ y = it/2
or tan(o: + P+ y) = = Solution: We have
or 2vxvy 2vy
R=---, T=--
1-[tano: tanp + tanp tan y + tan ytano:] = 0 g g
or .1 = tan 2 o: - + - +-
e e 2 e3
[1 1 1] After first collision
v'x = evx
2 1 1 1 v'y=Vy
or cot o:=-+-+- Distance covered before 2 nd
e e2 e3
collision
. T T evxvy
d1 = v~.- = evx.- = ---
- --~ 2 2 g
A block of mass m i5 projected with velocity v as shown in Fig. i After second collision
'4E.60_ The ground i5 smooth but there i5 friction between A; v; = v~ dz d1
and B. If colli.ion i5 elastic vn = ev' Fig. 4E.61 (b)
A _ 2v" X v" yY Zeb X
ev y
[mlµ d2 -
V
g g
[ml--- I 2m Is evxvy . ,
µ=O -1 d1 +d2 =--(1+2e)
g
Fig, 4E.60.
I R 2vxvy
(a) Find the final common velocity of A and B. but d1 +dz=-=--
(b) Find total energy di.sipated in friction. 2 2g .
Assume that A does not fall off 13_._ e (1 + 2e) = 1; 2e 2 + e -1 = 0
Solution: (a) Using conservation of momentum -1±./I+s -i ± 3
e=----=--
between B and C 4 4
mv = mv_ 1 + 2mv 2 ... (1) Rejecting -ve value
Collision between B and C is elastic e = 1/2
www.puucho.com u .... til'
Anurag Mishra Mechanics 1 with www.puucho.com

fTh'.;f~~uctz:·au~ §1lna1~ at r~:~~~'/nt.~ontg_ct;<m ~ _table. ;t]


((~. ~ef d¼c/o.,j,s..9me·m.CIJS b.ut, o.,f'".ifou,b.•.le !ad1us ,st;nk.es ,t~e.1!1'
lsjrrlrriemcally, iind '.'itself comes' to . rest .after impact. The Alternative
' •' 2./2
cos0=--.
3
----- -·-·~-·-- ---.
·v;
!
F" I~ - , ..
. i ~ : :,'.;_J~!~t~l,___ j_ _ .,___'., .. ·-··
u

Solution:

Fig. 4E.62 (d)

Assume. initial velocity of big block =u


and final velocity of small ball is. v
Conserving momentum
=> mil = 2mv cos8 ... (1)
V2 -V1
=> -e=~-~
U2 - U1
.v-0
... (1) => -e=----
O-1Lcos8
V
=> e=-- ... (2) .
U"COS8
From eqns. (1) and (2)
1 9
e=---~-
2cos2 8 16
Passage: (Example 63-65)
; . . -- - - - . ' ··~-·- ---·- . - ~-:-'""·:~7
,Two smooth balis A and B, each of mass m and radius R,'.havei
!their centres at (0, 0,R)and at.(511.,-R,R) respectively) in,~
;coordinate 'system as shown'. Ball A,- moving .along· po_sitivel
'.x:axis, collides with ball B. Just before .the collision,_speed o/i
.ball A is 4 m/s and ball B is stationary. The collision betll/een
!the balis is elastic. _ ·_ __ · · ·· \ _ .
! y l ' ,,,· 'l

+--+-"+-....,..-sc---x(m) '.
• I
i
B
I Fig, 4E.63 (a)
J..,.,,....-~----- , ~~-·-----""· · - - -
__l,'

I .
!Velocity of'the ball A just after the' collision is:
(a) (t+.flj)m/s · (b) (i---.13i)m/s /.
(c) (2I+.--.13])m/s
. , .
(d) (2i+2])m/s
- . '
• I
. ---·- - ---,·-- ._,_ --~J

www.puucho.com

n
Anurag Mishra Mechanics 1 with www.puucho.com

IMPULSE AND MOMENTUM


Solution: (a) Solution: (b)
_,,. ... 4 sin 30"
A
A
__.,. 0
..

Before Collision
Before Collision
(a)
(a)
4 sin 30'
4 sin 30°

.,. 0
.. ···

After Collision After Collision


(b)
(b)
Fig. 4E.63
Fig. 4E.65
- -- - '
vA = 4sin 30° [cos 60i + sin 60JJ (1)
1 -(v 2 -v 1 )
---~ -~
vA=i+.f:3]m/s 2 (0-4cos30°)
- ----- \
;Jt = ./3
: _E;x9_rQple :_ ~~- v (2)
Vz

m--
2
= mv 1 + mv 2
Impulse of the force exerted by A on B during the collision, is Vz + V1 = 2./3
equal to: 3./3
v 2 = --m/s
(a) (./3mi+3mJ)kg-m/s . 2

(b) ( 1; mi- 3m]) kg-m/s


3
v 2 = ./3 m/s[cos30° i+sin30° (-J)J
2 .

=( 9, 3./3,)
-1---J
(c) (3mi- .f:3mJ) kg-m/s
4 4
(d) (2v'3mi + 3mJ) kg-m/s - - - - I- . 1--.
-, -, -, ;,J~~9:~ij)fr:..~
Solution: (c) JAon·B;:::; mVBf-VBr

= m[ 4cos30° (cos 30° i - sin 30° JJ - OJ Two spheres. are moving towards each other. Both have same'
radius but their masses are 2 kg and 4 kg. If the velocities are
= (3mi-~3mJ) kg-m/s 4 m/s and. 2 m/s respectively and coefficient of restitution is
e = 1/ 3, find,
(_,F 2 Atl}J>le . 65 I,-> (a) The common velocity along the line of impact.
(b) Final velocities along line of.impact.
Coefficient of restitution during the collision is changed to (c) Impulse of deformation.
1/2, keeping all other parameters unchanged. What is the (d) Impulse of reformation.
velocity of the ball B after the collision ? (e) Maximum potential energy of deformation.
1 r;c: A 1 A ,-;'

(a)-(3-v3i+9j)m/s (b)-(9i-3-v3j)m/s (f) Loss in kinetic energy due to collision.


2 . 4
(c) (6i+3.f:3J)m/s (d) (6i-3./3J)m/s

www.puucho.com
Anurag Mishra Mechanics 1 with www.puucho.com

378
-,---~-- - _ - : ..•:=.-::....-=-:.~-::.---:--_=.--:-.....=::..·_---- J
From the above two equations, . ,
2
v 1 =--m/s
-J3
1
and v 2 =-m/s.
-J3
. ./"~, 4kg (c) J0 = m1 (v-u1 )
· = 2(0-4cos30°) = -4-,J3 N-s

·--~~~-Jf~ ... ~ --------~~~:.?! motion (d) JR= e.fv = .!(-4-./3) = _ _±__ N-s
3 -./3
R... .. R: (e) Maximum potential energy of deformation is
R·· •• :f!kg equal to loss in kinetic energy during deformation upto
Line of motion 2m1s a ··-... I 1 maximum deformed state.
···-.Line of impact I j
U= 1 11!1 (u 1 cos0) 2 + 1 m2 (u 2 cos0) 2 - 1 (m1 +m 2 }v 2
2 2 2
__ L~~---~g:4E.6~------lJ· - 1 1 1
= -2(4cos30° ) 2 +-4(-2cos30° ) 2 --(2+ 4)(0) 2
S o I u t.10n.. I nu
•AB C sm0
. = -BC = -R = -1 2 2 2 ·
AB 2R 2 or U = 18 joule.
M 0=3~ (f) Loss in kinetic energy,
(a) By conservation of momentum along line of
impact. uAK£1(
= - m1 u 1 cos0) 2+1
- m2 (u 2 cos0) 2
• LOI
·-- -------1 2 2
··...

2~!j~4m/s
4 sin 30°
-(½m,v: +½m v~) 2

1 - 2 1 2
4 cos 30 •, = -2(4cos30°) +-4(-2cos30°)
~cos30° '.·-xn30° 2 ·2
-784kg
2 sin.30° - 2 sin 30° ••• -(½2(lJ2 +½4(lJ2)
Just Before ColHsion Along LOI -Maximum Deformed State AKE = 16 joule
~ - - - - - - · · F i g . 4E.66 (b) _, _ _ ___ __ ]
6-exam,~,;~
2(4cos30°)- 4(2cos30°) = (2+ 4)v . '
or v = 0 (common velocity along LOI) '.lwo bl~~ks of mass 2 kg and Mare at rest on an incli~~dpla~e]
(b) Let v 1 and v 2 be the final velocity of A and B . and are separat~d by a d~t~nce 6.0 as shown in Fig. 4E._671
respectively then, by conservation of momentum along line ~a) .. The coeffi~1ent of friction ·between each block mi.d• the1
of impact. inclined plane IS 0.25. The 2 kg block is given a velocify ofr
10.011!/s up the inclined plane. It collides with M, comes batk
'ii; /4sin30° and has a velocity of 1.011!/s when it reaches its initial
position . .The other block M after the collision moves 0.5 in up
21<~~'.
4~2
land tomes :10 rest. Calculate the coefficient of restitution
between the blocks and the mass of the block M. · I
2 sin 30° · [ Take sin0 = tan0 = 0.()5 and g = 10 m/s 2 .] ,

Just After Collision Along LOI


~--i !
L__ Fig. 4E.66 (c) ,i
·------- , "'E
' C U')
I

2(4cos:30° )- 4(2cos30°) = 2(v 1 ) + 4(v 2 ) : iii C\!


! LO 0
, ••••••••••• II II I,
or 0=v 1 +2v 2 ... (1)
,E'
By coefficient of restitution,
e = velocity of separation along LOI :h 1 ;; 6 sin 8
: =0.3m
velocity of approach along LOI
1 V2 -v 1 Re°ferepce level
or -=--~~~--
3 4cos30°+ 2COS30° Fig. 4E.67 (a)
or V2-V1 =-./3 ... (2)
www.puucho.com
Anurag Mishra Mechanics 1 with www.puucho.com

~I_M_PU_~_E_AN_D~,M_O_ME_N_TU_M_ _ _~----~----~-'-----s=-------'-679j
Solution: This problem can be divided into the From the conservation of momentum for collision,
following steps: (2)v 1 = Mv 3 - (2)v 2
1. Block of mass 2 kg moves up the incline collides with M = 2(v1 +v2) = 2[8+5] 15.12kg
block M with velocity v 1 • · ·
V3 1.72
2. After collision the velocity of the 2 kg block is v 2 • The
2 kg block returns to the original position with velocity 1.0
m/s.
k~~m.i,;1 68 ~
----···-·-- - - - - - - ,
3. B.lock M moves up with velocity v 3 and comes to rest Fig. 4K68 (a) shows a smooth spherical ball of mass m
after moving a distance 0.5 m up the incline. striking two .identical· equilateral triangular wedges of mass
First step: From the work~energy theorem we may M; At the ill$tant of impact velocity ofthe ball is v 0. TakiJJg
calculate v 1 as follows : coefficient of restitution e, determine the velocities of the
W friction = ,iKE + ,iUg sphere and the wedges iUSLaftf!r collision....
1
-6µmgcos8 = -m[v; -(10)2] + mgh1
2
v;
= -2[f.j.lg cos8 + ghi] +(100)
where cos8 = .J1- sin 2 8 = ~1- (0.05) 2 = 0.99
v;
= (100)- 2 [(6)(0.25)(10)(0.99) + (10)(0.3)]
v 1 ~ 8m/s
Fig. 4E.68 (a)
Second step: We may apply work-energy ·theorem ·---'------"----'
on return journey of the 2 kg block. Solution: Let J be intpulse between ball and wedges
W friction = ,il{E + llUg and v 1 and v 2 be the velocities of the ball and the wedge.
-6µmgcos8=-m[(l) 1 2 2
-(v 2 ) ]-mgh 1
From intpulse-momentum equation on the ball,
2 2J sin 30° = mv 1 - Emvo)
-12(0.25)(10)(0.99) = [(1) 2 -(v 2 ) 2]-2x (10)(0.3) 2 J = mv 1 + mv 0 ... (1)
or v 2 ~ 5m/s From the wedge, J cos30°= Mv 2 ... (2)

Third step: We may apply work-energy theorem on On eliminating J from eqns. (1) and (2), we have
the upward journey of M. 2
../3Mv 2 = mv 1 +mv 0 ...(3)

Before colllslon After collision


Fig. 4E.67(b)
·- ... ·--· --·. ---''-------'-'--------"--'
W friction = ,iKE + ,iUg
. 1
-(0.5)µMg cos8 = -M[O- (v 3 ) 2] + Mgh 2
2
-v~ = -µg cos8- 2gh 2 From the definition of coefficient of restitution,
v~ = (0.25)(10)(0.99) + 2(10)(0.025) . V1COS60°+V2COS30° V1+../JV2
e=~-------=----
v0cos600 v0
v 3 ~ 1.72 m/s
From the definition of coefficient of restitution, or ev 0 =v 1 +../3v 2 ... (4)
On solving eqns. (3) and (4) for v 1 and v 2 , we get
e= Relative velocity of separation (2eM -3m)v 0
Relative velocity of approach V ------
l - 2M+3m
V2 +V3
= ../3(1 + e)mv 0
VI V
2- ----~
- 2M+3m
5+1.72
e= 0.84
8

www.puucho.com
Anurag Mishra Mechanics 1 with www.puucho.com

~-ai ...-c- /; . ,r
MECHA~~f-fj
>.e~:.;;.:.......-'-"-·-"'"-'---~-'.--'-C..-----'"--"·-'--;====~·-":::;' . _·_-_-_-_-_-,...:.-·· -----·- . , ·-- I
.. i
'k qXc(~i~J;~~~
.~--~!~
~------,···-·-· ...,_,, __ ,= ---··-~,.-------· ·-- - - --~ ::--·---:; i
=
IA 'ting.of mass .M. 0.25 kg free to slide on a fixed smooth/
· horizontal wire is attached to a particle of mass m = 0:75 kg1 .i _____ ,,,...._._..____Fig."'"""'""-~·"'""""''""·~·--
,.
4E.70
--
· by astring of l~ngth 1 m which pas§es over a fixed smooth,peg Solution:, Velocity of ball relative to _wall before
Pat a depth-d-= OA m belo~ the wire anq in the same,vertical collision is (V'+ u). After elastic collision the velocity of ball
plane. The system is released from rest when the ring is 0.8 m relative to wall will be -(v + u). The velocity of the ball
\from the peg. }'ind th~ maximum angle' the ring will make relative to ground will be
lwith the verticaLafter it loses contact with th peg. ·

I
I_ , . . , , , I M = 0.25 kg
.
S/ooth wire 1I
1
·I'
-(v+u)-u = -(v + 2u)
Kinetic energy before impact= tmv 2
·1,'.
.
0.8m
d= 0.4m I
'
I

Kinetic energy after impact =Im


2
(v + 2u)~
. 2
''
'I

The change in kinetic energy i~ equal to 2mu(u + v ).


' Now we calculate the work of reaction forces actir g on
m = 0.75 kg; j
''. the ball during the _impact. Let the collision continue for t
'
! i_ -------·
Fig. 4E.69 ·'
seconds. Assume the reaction force to be constant· (the result
Soh,1tion : The velocity of the ring when the string is does not depend on this assumption). Since the impact
vertical can be determined from the energy conservation changes the momentum by 2m(v + u), the force of reaction is
equation . · · 2m(v +u) · ·
F
t
mg(0.8- 0.4) = IMv~
. 2_ _ _ _ _ . The work of this force is
or ~ ..
v0 =~2mg, 1w
--(0.8-0.4) =--m/s
W =rs= F(ut) = 2m(v-t,u)ut
t
M 5
Note that the velocity of mass mis zero and that of ring = 2m(v +u)u
is v O in the horizontal direction. So we can see that this work is equal to change in kinetic
When the string makes the maximum angle 0 with the energy.
v~rtical subsequently, we have SYSTEM OF VARIABLE MASS;
(M+m)v=Mv 0 ROCKET PROPULSION
(from conservation cif momentum) Fig. 4.50 (a) shows a system of mass M and momen'.tum
And .from energy conservation, we have
1 2 1 2
Mv at same time t. A tiny infinitesimal mass dM travelling
mgl(l-cos0)+-.(M+m)v =-Mv 0
with velocity u combines with the system in an infinitesimal
· -2 . 2 time dt; so the mass is M + dM and velocity-is v + dv.
. t; Mv+
Tota1 momentum at time .., u
..,dM
or mg1(1-cos0)=I[M-~]v~
. 2 M+m
Total momentum at time t x dt: (M + dM) (v+ dv)
1-cos0= 1 Mv~ Ixix 196x3 ..,
2 (M + m)gl 2 4 25 X 9.8 So the change in moment)lm dP is
=0.3 --+
dP = (M + dM)(v+ dv)-(Mv+_udM)
--+ --+ --+ --+
1
or cose = 0. 7, e = cos- (0.7)
=Mdv+vdM +dM dv-udM
~~m~~> From Ne'\\'.ton's second law, we have
....
r-· ---·--~·.-.. -:. -.----- -~-_- -_--_--..--------· ..... ~ -._, --··1
·A bdll moving with a velocity v strikes a wall moving towa~d· r.i =dP =Mdv+vdM-udM
!the wall with a velocity u. An elastic-impact occurs. Determine' ____ ':",t__
d!_ .... ·- __ dt
ithe velocity of the bdll after the impact. What is the cause bf; dM ,-~v1
lthe change in the kinetic energy of;.t~e. ball? Consider the mass:
)of the wall td be infinitely great. , !
-~
. ~;
l.:-------··- -·---- ... - - _,_ - ---... ---· .. ···~""
(a) (b)
Fig. 4.50
www.puucho.com
Anurag Mishra Mechanics 1 with www.puucho.com

- - - -----·------·-- 381/
dM
We ,bave neglected the term dM dv/ dt , in the limit of =v- Llx,~-07
infinitesimals it is zero. Thus we get
--i
LFexr =M--(u-v)-·
dV
dt
-+ """? dM
dt
... (1)
dt
= (2.20) (75)
= 165N
->
I
r
Uy

Note that the direction of force Fext is same ,/-"/


Note that the quantity (ti- v) is the relative velocity, as the velocity of the belt. Fig. 4E.71 _(b)_
->
Vrel, of dM w.r.t. M. So we can rearrange eqn. (1) as (b) Rate of work done by Fext,
dv .., .., dM dw ->->
-
2dM = 363 W
= Fext' V = V -
M-=LFex,+V«J - ... (2)
dt dt dt dt
-> • which is the power output required of the motor.
where L Fext denotes the external force on the mass M
The rate at which gravel is gaining energy is
(for a rocket it would include the force of gravity and air
resistance). dK =j_(.:1cMv2)=_:lcdMv2
dt dt2 2dt
The force exerted by dM on M is dM, which v'rel _,
dt which is only half the work done by Fex,. The other half
represents the rate at which momentum is being transferred of the external work done goes into thermal energy
to (or from) the mass M (for a rocket this term is called the produced by friction between the gravel and the belt (the
thrust). same friction force that accelerates the gravel).
The equation (2) has application in rocket propulsion. It
propels itself forward by the ejection of burnt gases. The L:cE~i,ii;ppJ,~,j-72[>
mass M of the rocket decreases during the process, so - - '
dM/dt < 0. Another application is the dropping of material A rocket has a mass of 21000 kg of which 15000 kg is fuel.!
(gravel) onto a conveyer belt. In this case the mass of the The rocket engine can exhaust fuel at the rate ofl90kg/swith,
loaded conveyor belt increases, so dM/dt > 0. an exhaust velocit;y of 2800 m/ s relative to the rocket. If the
- .., - - -:- !-- --, • rocket is fired vertical(,- upward, calculate:
L-~~~~pJ E;:,.l!~J/ ( a) the thrust of the rocket;
(b) the net force on the rocket at blast-off and just when all
A hopper drops gravel at a rate of 75.0kg/s onto a conveyor the fuel has been used up;
,belt moving at a constant speed v = 2.20 m/s. (c) the rocket's velocity as a function of time, and ,
(a) Detennine the force needed to keep the conveyor belt. (d) its final velocity at the bum-out. Assume that·
moving. , acceleration due to gravity is constant at g = 9.8m/s2 i
(b) What power output must the motor have that drives the: and there.4. no. air r~istan~ .. .. _
conveyor belt?
Solution: (a) The thrust of the rocket is
_, dM Vrocket
Fthrust = V rel -
dt

••
= (-2800)(-190)
= 5.3xl0 5 N
We have taken upward positive, so v rel
,~Mff(I is negative because exhaust velocity. is
Fig. 4E.71 (a) downward, and dM/dt is negative because

Solution : We assume that horizontal component of


velocity of gravel at the moment it lands on the conveyor
belt is zero.
the rocket's mass is decreasing.
(b) F,x, is blast-off
=Mg= (2.1 X 1Q4 )(9.80)
;, 2.lxl0 5 N
,~M
!
-> -> ->
(a) V rel = V gra\"el - V belt
Fext at bum-out = M rocke,g
= 0-v ~gasa,
dv -, dM = (6xl0 3 )(9.80)
As Fext =M--vrel - Flg.4E.72
dt dt = 5.9xl0 4 N
=0-(-v)(:)
www.puucho.com
Anurag Mishra Mechanics 1 with www.puucho.com

3::::8~2---'~-----";.;__----'-"-..:.;._--_;;_---·....._.----'-----M-EC..;...HANIC,S~IJ
!...'.I

so, F••, = (5,3 x 105 - 2.1 x 10 5 ) (blast- off) So


dv
Fext =M-+v-
dM
= 3.2x 105 dt dt
4 . dv
5
F0 . , = (5.3 x 10 - 5.9 x 10 ) (burn- out) (P-µkpgx) = (px)-+pv 2
dt
After bum-out only the force of gravity remains, i.e., On rearranging the equation, we have .
-5.9x10 4 N. '
2 . ! 2
dv P-µkpgx-pv P v
dv dM - -~~---=--µkg--
(c) As M-=Fen +v 1 - dt px px x
dt re dt
dv = Fen
M
dt+v 1 dM
,. M
iciE~x@.ml~Lt,'~J74l;>
where Fext ; ~Mg and M is the mass of the rocket as a IA chain oflength L and ·;ass per ~-n-it-1-en_g_t_h_p._is_p_il_le_d_o_n ~J
function of time. horizontal swface. Orie end of the chain is lifted vertically!
So,. Jvdv=-J'gdt+v,. 1 JM dM with a .constant velocity v by a variable force P. Detennine: I
v0 o · Mo m ( a)' P as a/unction of the height xof the end above the"surface.
Since v rel is constant we have taken it out of the (b) the energy. lost during tne lifting of the chain. ·
integral.
' ,----p-·-
Thus v(t) = v 0 -gt+ vre1 In-
M
l f
Mo
where v(t) is the rocket's velocity and M its mass at any
l
I
X

time t.
Note that vre1 is negative (-2800m/s) because it is
opposite to the motion and that ln(M/M 0 ) is also negative
because M O > M. Therefore the thrust on the rocket is
Flg.4E.74 __
...,_
positive and acts to increase the velocity. · Solution: (a) Let x be the displacement of the end of
the chain above the surface.
(d) Time taken to use up all the fuel (15000 kg) at a
Fext =·P-pgx
rate of 190 kg/s, so, at bum-out
Vre1 = 0-V
t = l.50x104 = 79s dM
190 -=pv
dt
If we take v 0 = 0, dv = 0
6000 dt
v=-(9.8)(79)+(-2800)(ln )
. 21000 dv dM
From the equation, M - = Fext + v rel -
= 2730m/s dt dt
or O = (P-pgx) + (0-v)pv
~~~•:~tei@);;> p =p(gx+v2)
(b) From work-energy theorem;
IA pile of loose-link chain, mass per ·unit length A lies on a
1rough surface with coefficient of kinetic friction µ k· Of!e end fPdx-M:=AK+t.U .
where JP dx is work done by external force P, M: is loss
of the chain is 'being pulled horizontally along the surface bya in energy.
constant force P. Detennine the acceleration of the chain in
' dx
tenns of x and- = v.
. Pdx =f Ji
(pgx+pv 2)dx
'dt
=--:-=-p-
1- . _, 1
=-pgL
2
2 2
+pvL
;us ;;e~ • On substituting in the work-energy equation, we get
Fig.4E.73 12 2 1212
-pgL +pv L-M: = -pLv +-pgL
2 2 2
· dv · dM 1
Solution: ·As M-=F +v 1 - 2
M: =-pLv
dt ext re dt 2
here vrel ::::;Q-v The link at rest on the platform acquires its velocity
dM dM dx abruptly through an impact with the link above it. Work
and ·-=--=pv
dt dx dt done by internal non-elastic forces during impact is
Also Fen =P-.µkpgx converted into heat and acoustic energy.

www.puucho.com
Anurag Mishra Mechanics 1 with www.puucho.com

IMPULSE AND MOMENTUM 3531


. I

---- -,
Only One Alternative is Correct

1. A set ofn identical cubical blocks lies at rest parallel to


each other along a line on a smooth horizontal surface. (a) 1200 J (b)° 300 J
The separation between the near surface of any two (c) 600 J (d) 900 J
adjacent blocks in L. The block at one end is given a 5. A body falling vertically downwards under gravity
speed v towards the next one at time t = 0, all breaks in two parts of unequal masses. The centre of
collisions are elastic then: mass of the two parts taken together shifts
. (n + l)L
(a) The last block starts movmg at t = -'---~- horizontally towards:
v (a) heavier piece
n(n- l)L
(b) The last block starts moving at t (b) lighter piece
2v (c) does not shift horizontally
(c) The center of mass of the system will have the (d) depends on the vertical velocity at the time of
final speed v breaking
(d) The center of mass of the system will have the 6. A block of mass Mis placed on the top of
::~·;I
V
final speed - a bigger block of mass 10 Mas shown in
n figure. All the surfaces are frictionless. 10M '
2. A boy of mass m is standing on a block of mass M kept The system is released from rest, then ~.2.2~::d
on a rough surface. When the boy walks from left to the distance moved by the bigger block
right on the block, the centre of mass (boy + block) of at the instant the smaller block reaches the ground:
the system: (a) 0.22 m (b) 0.20 m
(a) remains stationary (c) zero (d) 0.24 m
(b) shifts towards left 7. In the figure shown, the two
(c) shifts towards right
identical balls of mass M and •,

-! :
(d) shifts toward right if M > m and toward left if radius R each, are placed in .,
M<m contact with each other on the
3. A uniform sphere is placed on a smooth hori~ontal frictionless horizontal surface. 1 t 1
surface and a horizontal force F is applied on it at a The third ball of mass M and ,
distance h above the surface. The acceleration of the R ;<f- _.\
radius -, is coming down
centre: 2 \\\ \\\\!
(a) is maximum when h = 0 vertically and has a velocity · -··· __ J
(b) is maximum when h = R = v O when it simultaneously hits the two balls and
(c) is maximum when h = 2R itself comes to rest. Then, each of the two bigger balls
(d) is independent of h will move after collision with a speed equal to :
4. An open water tight railway wagon of mass 5 x 103 kg (a) 4v 0 (b) 2v 0
coasts at initial velocity of 1.2 m/s without friction on ./s ./s
a railway track. Rain falls vertically downwards into (c) ~ (d) none .
the wagon. What change then occurred in the kinetic ./s
energy of the wagon, when it has collected 103 kg of
water:
www.puucho.com
Anurag Mishra Mechanics 1 with www.puucho.com

384· - - - ---------·-------· _,._ ·- ·- · MECHAN1cs,TJ


~~----'--------'-------------------~·~·---- . -- .
8. A ball kept in a closed box moves in the box making (a) (l+e)u2sin28
collisions with the walls. The box is kept on a smooth g
surface. The velocity of the centre of mass: 2
(a) of the box remains constant (c) (l-e)u sin28
g
{b) of the box plus the ball system remains constant
15. A ball is dropped from a height h. As it bounces off the
(c) of the ball remains constant
floor, its speed is 80 per cent of what it was just before
(d) of the ball relative to the box remains constant it hit the floor. The ball will then rise to a height of
9. Two identical billiard balls are in contact on a table. A most nearly:
third identical ball strikes them symmetrically and (a) 0.80 h (b) 0.75 h
comes to rest after impact. The coefficient of (c) 0.64 h (d) 0.50 h
restitution is:
16. Internal forces can change:
(a)'~ (b) .!. (a) the linear momentum but not the kinetic energy
3 3
(b) the kinetic energy but not the linear, momentum
(c) ~ (d) :./3' (c) linear momentum as well as kinetic energy
6 2
10. A ball is projected from ground 'with a velocity v at an (d) neither the linear momentum nor the kinetic
angle 8 to the vertical: On its path it makes an elastic energy
collision with a vertical wall and returns to ground. 17. In an elastic collision of two billiard balls which' of the
The total time of flight of the ball is: following quantities is not conserved during the short
(a) 2vsin8 (b) 2vcos8 time of collision:
g g (a) Momentum
(c) v sin 28 (d) v case - (b) Total mechanical energy
g g (c) Kinetic energy
(d) None
11. A sphere moving with velocityv strikes elastically with
a wall moving towards the sphere with a velocity u. If 18. A block of mass M lying on a smooth· horizontal
the mass of the wall is infinitely large, the work done surface is rigidly attached ·to a light horizontal ·spring
by the wall during collision wiHbe: of spring constant k. The other end of the spring is
(a) mu(u + v) (b) 2mu(u + v) rigidly connected to a fixed wall. A stationary gun fires
(c) 2mv(u+v) (d) 2m(u+v) ·bullets of mass m each in horizontal direction with
speed v 0 one after other. The bullets hit the block and
12. On a horizontal smooth surface a disc is placed'at rest.
get embedded to it. The first bullet hits the block at
Another disc of same mass is coming with impact
parameter equal to its own radius. First disc is of t = 0, the second bullet hits at t = ;7t~M: m, the third
radius r. What'should be the radius of coming disc so
that after collision first disc moves at an angle 45° to . att
bu IIet hits - k - + 21t~M+2m
=21t~M+m k and so on.
the direction of motion of incoming disc ?
(a) 2r (b) r(-.J2-1) The maximum compression in· the spring after the n th
bullet hits is:
(c) r (d) r../2 nmv 0 .Jk ·
(-.J2-1) (a) ----'C.:.:C,.,,- (b) _(M_+_n_m_)~31_2
312
13. A ball is thrown vertically downwards with velocity (M +nm) nm11 0 .Jk
.J
2gh from a h_eight h. After colliding with the ground
(c) ~ (d) nmv 0
it just teaches the starting point. Coefficient of 312
(M + nm) .jk(M +nm)
restitution is:
1 19. A boy hits a baseball with a bat and imparts an impulse
(a) -
../2 J to the ball. The boy hits the ball again with the same
force, except that the ball and the bat are in contact for
(c) 1
r---- -- - --- twice the amount of time as in the first hit. The new
14. A ball is projected with initial impulse equals :
velocity u at an angle 8 to the i ~ \ (a) half the original impulse
horizontal. Then horizontal . ~ x ~ _ J
displacement covered by ball -as . .,, (b) the original impulse
(c) twice. the original impulse
it collides third time to the ground would be, if
coefficient of restitution is e: · (d) four times the original impulse

www.puucho.com
Anurag Mishra Mechanics 1 with www.puucho.com

[-,M~UlSE AND M~~~N--'T-=-UM:.c:____ _-.c_ _ _ _ _ _ _ _ _ _ _ _·_-_ - _ · · - - - - - ' - - - - - - _____·_-·3asj


20. A shell is fired from a cannon with a velocity v at an 25. Three blocks are
angle 8 with the horizontal direction. At the highest initially placed as
point in its path, it explodes into two pieces of equal shown in the
masses. One of the pieces retraces to the cannon. The figure. Block A
speed of the other piece immediately after the has mass m and
explosion is: initial velocity v to the right. Block B with mass m and
(a) 3v cos8 (b) 2v cos8 block C with mass 4m are both initially at rest. Neglect
3 friction. All collisions are elastic. The final velocity of
(c) -vcos8 (d) vcos8
2 block A is:
21. A disc of radius R i;cut out from a larger disc ofradius (a) 0.6v to the left (b) 1.4v to the left
2R in such a way that the edge of·the hole touches .the (c) v to the left (d) 0.4v to the right
edge of the disc. Then center of mass for the residual 26. A square plate of edged and a I w - . --·- . --
disc is: circular disc of diameter dare : _1
(a) at 2R from center of the original disc away from Placed touching each other at the l&\\\\M~\\\\M
t,.-d ...... d-1 :
3 midpoint of an edge of plate as ··-- ---· -·- -- --·-·
the center of the hole shown. Then center of mass of the combination will be
(assume same mass per unit area for the two plates):
(b) at R from center of the original disc away from the "2d
3 (a) - - left to the center of the disc
center of the hole 2+ 1t . ·
(c) at the center of the original disc (b) ~ right to the center of the disc
(d) at the center of the hole 2+7t
22. There are some passengers inside a stationary railway (c) _.±!_ right to the center of the disc
compartment. The track is frictionless. The centre of 4+1t
mass of the compartment itself (without the (d) _.±!_ left to the center of the disc
passengers) is C1 , while the centre of mass of the 4+ 1t
'compartment plus passengers' system is C 2 • If the 27. A rocket of mass 4000 kg is setfor vertical firing. How
passengers move about inside the compartment along . much gas must·b~ ejected per second so that the rocket
the track. may have initial upwards acceleration of magnitude
(a) Both C1 and C 2 will move with respect to the 19.6 m/s 2 ? [Exhaust speed of fuel = 980 m/s]
ground (a) 240 kg ss1 (b) 60 kg s- 1
(b) Neither C1 nor C 2 will move with respect to the (c) 120 kg s- 1
(d) None
ground

y~;;: ffi-~·~-
28. Select the graph(s) which best represent the graph of
(c) C1 will move but C 2 will be stationary with respect
to the ground bouncing ball. Assume ball dropped from height and it
(d) C2 will move butC 1 will be stationary with respect
to the ground l=f , : 'E
~ac~ im_p_acr
'
23. All the particles of a body are situated at a distance R (1) ' ~ : : • ' (2) ' ~ •.••......... I
1 .2 • • time , - time__,. f
from the origin. The distance of the center of mass I ~ ' : fr '
I _ ~-·-" __ _ __ , 'i5 ______ ----· _1
from the origin is: ·
(a) = R
(c) > R
(b),,; R
(d) e: R (3)
' . ~ ~ -:
:i
;t time~
!,gF--
(4) _; ............ II
'8 ~time !

24. Two trains A and B are running in the same direction •u '
-·- - - ----l ._a:i,. ~ ----- -
on the parallel rails such that A is faster than B. Packets
of equal weight are transferred from A to B. What will (a) l, 2, 3, 4 (b) 1, 2, 4
happen due to this:

·r --- - .
(c) 3, 2. (d) 3, 4
(a) A will be accelerated and B will be retarded 29. A ball of mass m falls
(b) B will be accelerated and A will be retarded vertically from a height h and •, ~ j
(c) There will be no change in A but B will be collides with a block of equal : - µk= 0,2 /
accelerated
(d) There will be no change in B but A will be
mass m moving horizontally , .....:--~ v / '
with a velocity v on a surface. •illi/77777771!~7
I m
: '7n
The coefficient of kinetic ,. - - .--- -- I
accelerated
friction between the block and the surface is 0.2, while
the coefficient of restitution e between the ball and the
www.puucho.com
Anurag Mishra Mechanics 1 with www.puucho.com

3
block is 0.5. There is no friction acting between the 34. An open water tight railway wagon of mass 5 x. 10 kg
ball and the block. The velocity of the block decreases coasts at an ihitial velocity 1.2 m/s without friction on
by: a railway track. .Rain drops fall vertically downwards
(a) 0 · (b) 0.1~2.gh into the wagon. The velocity of the wagon after it has
. (c) o.3-fiih (d) can't be said collected 10 3 kg of water will be:
30. A particle of mass m; ·ihitially (a) 0.5 m/S (b) 2 m/S
(c) 1 m/s (d) 1.5 m/s
at rest, is acted upon by a
variable force F for a brief
interval of. time T. It begins
35. Two identical balls A and B lie
on a smooth horizontal
-,-.. -.~. .}.=-i
to move with a velocity u surface, whjch gradually b_0:+B ·/23:3
after theforce stops acting. F o<....~"'um_e_.~Ti-... merges into a curve to a height
is· shown in the graph as ·a 3.. 2 m. Ball A is given a velocity 10 m/sec to collide
function of time. The curve is a semicircle, then head on with ball B, which then takes up the curved
. itR2 ~T2 path. The minimum coefficient of Restitution 'C for
0
(a) u = - (b)u=- the collision between A and B, in order that B reaches
2m Sm
the highest point C of curve. (g = 10 m/ sec 2 ) ·
(~) u = ~oT (d) u = FoT
4m . 2m (a) .!. (b) ~
31. A particle strikes a horizontal
2 5
frictionless floor with a speed u, at (c) .!. (d) ~
an angle 0 with the vertical, and 4 4
rebounds .with a speed v, at· an 36. if collision takes place between 2 particles then which
angle cj> with the vertical. The of the following statement is/are true:
coefficient of restitution between the particle· and the {a) kinetic energy is conser:ved during collision
· floor is e. The angle cj> is equal to: (b) momentum is conserved during collision .
(a) 0 (b) tan-1 [e tan0] (c) momentum is conserved only before and after
collision
(c) tan-1[~tan0] (d)(l+'e)0
(d) conservation of momentum during cqllisio,n
depends on the type of collision · · ··
32. Two masses A and B
of mass M and 2M 3 7. On a smooth horizontal plane, a uniform string of
respectively
connected by
are
a -j .'
·L~·· .
- '
mass M and length Lis lying in the state of rest. A man
of the same mass M is standing next to one end, of the
compressed ideal ~ I'\" , ~ string. Now, the man starts collecting the ; string.
' '~- ' Finally the man collects all the string and puts it in his
·spring .. The system is
placed on a horizontal pocket. What is the displacement of the man with
respect to earth in the process of collection?·

.· ·. · ·. .·. ·.·. . . t I
frictionless table and
given a velocity uti in
the z-directicin as shown in the figure. The spring is
then released. In the subsequent motion the line from
B to A always points along the i unit vector. At some ii/: I I I 1. I I I I I I I I I I I I: I I I I I_, I I ii
instant of time mass B has a X'component of velocity as
v xi The velocity v A of as A at that instant is:
(a) -
L (b) ~
(a) v,i+uti · (b) .:.v,f+uii 2. 4
L
(c) -2vxi+uk (d)2v,I+uk (c) - (d) none
8
33. A particle A of mass 100 g moving along +ve x-axis
with 10 m/sec, collides at origin, with particle B ·of
38. In the figure shown surface
is frictionless and spring is _ ~ ~ .
I
r;;;,?.· = =_=. '.: m.,
mass 200 gm moving along +ve y-axis with 10 m/sec. in natural condition. If
After collision the particle B moves along line x 1, x 2 and x 3 are the maximum compression in spring
4x.- 3y = 0 with speed 5.m/sec. The equation of line for elastic, completely inelastic and inelastic (e = 0.5)
along which A moves _after collision. respectively then:
(a) y-3x=O (b) 3y-x=O (a) x 1 > X2 > X3 (b) X2 > X3 > X1
(c) 4y ~· 3x :i O (d) None (d) X2 > X1 > X3
(c) XI > X3 > X2
www.puucho.com
Anurag Mishra Mechanics 1 with www.puucho.com

,,
IMPUISE AND MPMENTUM
39. In a smooth 43. In the · anangements shown in -.----1
stationary cart. of figure masses of each ball is 1 kg V £ J
length d, a small - 1kg ·,
and mass of trolley is 4 kg, In the kg '
block is projected
along it's length with
velocity V towards
figure shell of mass 1 kg moving
horizontally with velocity v = 6 ~-J
m/sec collides with the ball and get stuck to it then its
front, Coefficient of · maximum deflection of the thread (length 1.5 m) with
V
restitution for each vertical is:
collision is e, The cart (a) 53° (b) 37°
rests on a smooth (c) 30° (d) 60°
ground and can move ____,, __ ..
44. Centre of mass of two thin uniform rods of same
.freely. The time taken by block to come to rest w,r, t,
length but made up of different materials and kept as
cart is :
shown, can be, if the meeting point is the origin of
(a) ed (b) ed coordinates:
(l- e)v (1 + e)v ~1--y----·---
d
(c) - (d) infinite . --
e 1 :•

[ll~-
40. In the shown figure both blocks are
in equilibrium m = 1 kg, a bullet of
L
mass m moves with velocity 10 m/s
and get embedded into the block A,
then just after collision: i • t•o;,:10
L__rt)_,...;c,_J
1
I. ..
I
I
Vo
( a) vA =-,Va= 0
2
(b)vA=va=~
2
I '+==========~I X
.__,_ _ _ _ _L_--': :.__ ___J
= Vo
(d) VA =v.
3
41. In the previous question the string will be tight again
(a) (!:2'2!:)
after 't' seconds from collision then t =
.! sec
(c) (!:3'3!:)
I (. ·..
(a)
2 45. A is a fixed point at a height h
(b) 1 sec
(c) 2 sec
above a perfectly inelastic
smooth surface. A light
I Ah. . •

(d) string .will never be tight again extensible string of length , "" n)I,·
42. The inclined surfaces of two movable wedges of same >
l (l h) has one end connected
mass M are smoothly conjugated with the horizontal to A and other to a heavy particle as shown in figure.
plane as shown in figure, A washer of mass m slides The particle is held at the level of A with the string
down the left wedge from· a height h. To what tight and released from rest, The height above the
maximum height will the washer rise along the right plane, where particle is again instantaneously at rest is
wedge? Neglect friction. , S, then which is incorrect?
r· (a) velocity of particle on the surface is = ~2,gh cos8
"' (b) vel~ of particle when it leaves surface
= -J2,gh cos 2 8
hs
(c) S = -
z4
(d) None of these
46. A hemisphere of mass 3m and radius R is free to slide
(a) h (b) hM with its base on a smooth horizontal table. A particle
(M + m) 2 (M+m) 2 of mass m is placed on the top of the hemisphere. If

(c) h(~)
M+m
2

(d)h(~)
.M+m
particle is displaced with a negligible velocity, then
find the angular velocity of the particle relative to the

www.puucho.com
Anurag Mishra Mechanics 1 with www.puucho.com

centre of the hemisphere at an anguT.ar displacement 0, (ii) The kinetic energy of a particle, is independent of
when velocity of hemisphere is v. · the frame of reference ·, '
(a) ~ 1
--~,-r-:;,r
~ -. (a) Both (i) and (ii) are true·
Rcose (b) (i) is true but (ii) -is false
(b)~ (cj (i) is false but (ii) is true
Rcose (d) both (i) and (ii) are false
Cc)
5v ·
Rcose
53. ABC is a part of ring having radius - . · B ,;:]~
R 2 and ADC is a part of disc having : ~ !, ~.;
-1
.(d) ~ 2
inner radius R1 and outer R2. Part A ....... :, ..... _.• cj
Rcose

r~: __~ J
ABC and ADC have same mass. R
47. 1\vo balls with masses in the ratio of 1 : 2 moving in
opposite direction have a head-on elastic collision, .If :;::n':!tass will be located, .· . , •• ?.
their velocities before impact were in the ratio of 3 : 1,
,__ ,,,_,,""'-""'""':""

then velocities after impact will have the ratio: (a) (R2 -R1)C2R1 +Rz) (above).
(a) 5 : 3 . (b) 7 : 5 31t(R1 +R2) ·
(c) 4 : 5 (d) 2 : 3 (b) (R2 -R1lC2R1 +R2) (beiow)
3it(R1 +R2 )
48. A projectile is fired at a speed of 100 m/s at an apgle of
37° ·above the horizontal. At the highest point, the 2R1 +R 2
(c) · (above)
projectile breaks into two parts of mass ratio 1 : 3, the 3it
smaller coming to rest. Then· the distance between (d) 2Ri + Rz (below)
launching point and the point where the heavier piece '3it
lands:
(a) 480 ni · (b) 960 m
54.
~dtv~l~ci:~ of:~ti:~ P}vr:,;~mis. :. ~,~!9~~1
(c) 1120 m (d) 640 m particles are as shown in l,1kgn · ~ ·~ A1kg:
49. 1\vo identical spheres A and B lie on a smooth
horizontal circular groove at opposite ends of a
the figure. They are kept
on a smooth surface and tc...::,.· • ·
t \~ ~!\: :1 ' · . ':::,_,;j
.diameter: A is. projected along the groove and at the being mutually attracted by gravitational force. Then
end of .tinie t, impinges on B. If e is the coefficient of position of .the center of mass at t = 2 sec:
·' restitution, the second impact will occurs after a time: (a) X;, Sm (b) X = 7m · ·
' . (a) 2t (b) !. (c) X = 3m (d) X = 2tn .
, . . e;. .e . 55. 1\vo identical ba1ls are dropped from the same height
(c) 7tt . (d) 21tt. onto a hard surface, the second ball being released
e e exactly when the first ball collides with the surf~ce. If
50. 1\vo blocks of masse·s 10 kg and 4 kg are connected by the first ball.has made two more collisions by the time
a string of negligible mass and placed at a frictionless the second one collides; Then the- coefficient of
h~rizontal surface. An impulse gives a velocity of 14 restitution· between the ball and the surface satisfies:
m/s to the heavier block in the direction of t lighter (a) e > 0.5 (b) e = 0.5 .
block. The velocity of the center of mass is: ·
(c) e = -.J3 - l · (d) e ;< -.J3 - l
·ca) 30 m/s (b) 20 m/s 2 2_
(c) 10 m/s ·. · (d) 5 m/s
51. A stationary pulley carries a ropes one end of which
56. A bullet of mass 20 g traveling
horizontally with a speed of 500
E:&n,Ji~. c':l:Jll
,--,;,"'" ·
'.lli
supports a ladder with a man and the other a counter m/s passes' through a wooden· .i\\W\\\\\\\\\\\\\\\\1.\1.\.,_\}~~l
weight of mass M.The man of mass m· climbs 'up a block of mass 10 kg initially at
distance 1c w.r.t. the ladder ·and then stops. The rest on a level -surface. The bullet emerges with a
displacement of the centre of mass of this system is: speed of ,100 m/s and the block slides 20 cm on the
(a) ~ (b) ml surface before coming to rest. Then coefficient of
M+m 2M friction between the block and the surface is:
(c) ml (d) ml . (a) 0.8 (b) 0.16
M+2m 2M+m (c) 0.32 (d) 0.24
52. Consider the following two ~tatements:
(i) The linear momentu)Il of a particle is independent
of the frame of reference

www.puucho.com
Anurag Mishra Mechanics 1 with www.puucho.com

_·_IM_PU_LS_E_AN_._D_M_OM_E_N_TU_M_ _ _ _ _ _ _ _ _ _ _ _ _ _ _ _ ~-~-~,~= .,
5 7. A gun is mounted on a railroad car. The mass of the 63. Initially spring is at it's ![iii}+•o~I
car, the gun, the shells and the operator is SO·m where natural length and collision • ~ ~ - ·"'· ·- . -~
m is the mass of the one shell. If the muzzle velocity of is elastic. Then find maximum compression of spring
shell is 200 m/s, what is recoil speed of car after during motion:
second shot?
(a)
200
49
m/s (b) 200 +(J_ J_)
48 48
m/s
(a) ~ v 0

/mv
(c) 200 (J_48 + J_)
49
m/s
(c)
V2k 0

64. In the shown figure, if all the


(d) 200 (J_ + l m/S
) surfaces are smooth and the two
48 48x49 masses are allowed to move then
58. A man of mass 60 kg can throw a stone of mass 1 kg up centre of mass of the system will
to a height 5 m. If he is standing on ice skates of move:
negligible mass, the maximum velocity that he can (a) upwards (b) downwards ·
generate in same stone if he throws it with same force (c) leftwards (d) rightwards
in the horizontal direction: 65. Four identical rods of mass M 1---· .. "' ·---·1
(a) Vmax = 9.9 m/s (b) Vmax = 12 m/s and length L are placed on one , !
(c) Vmax =7 mis (d) Vmax =10 m/S another on the table so as to i· j'
59; The density of a linear rod of length L varies are produce the maximum r · .
overhang as shown in figure. '---~ --- ~
,

p = A + Bx where x is the distance from the left end ..


The maximum ·total overhang will be:
Then, the position of the center of mass from the left
end is: 1 (a) 3L (b) 24L
2AL·+ 3BL2 2 4 25
(b) AL+ 3BL
(c) 25L (d) 4L

.=-~-· .m·~
(a) 3(2A + BL) (2A +BL)
2 2
24 3
(c) 3AL + 2BL (d) 3AL + 2BL 66. A 20 g 'bullet passes through a
(2A +BL) 3(2A +BL) plate of-mass 1 kg and finally ~
60. Three identical balls each of j
-· ----·1 comes to rest inside another : 20 r;' "" ·-··· · ·
mass 5 kg are connected with
I
+vo=9 I
o--------o----o i plate of mass 2980 g. It makes I"
1 kg
the plates move from rest to '--- - - ----- -
2980 g
each other as shown in figure,
'~--~ cI same velocity. The percentage loss in velocity of bullet
and rests over a smooth
horizontal table at moment t = 0 ball B is given between the plates is:
velocity 9 m/sec then velocity of A in direction of (a) 0 (b) 50%
velocity of B just before collision is: (c) 75% (d) 25%
(a) 9m/sec (b) zero 67. 1\vo particles of mass m1 and m 2 in projectile motion
(c) 3m/sec (d) 6m/sec
61. In above question 60 the velocity with which A collides
have velocities v 1 < v
2 respectively at time t = 0.
with C is: ' They collide at time t 0 • Their velocities become v1 and
(a) 6m/sec (b) 9m/sec v at time 2t
2 0 w~still moving in air. _The value of
(c) 3m/sec (d) 2-m/sec --+\.IS:.
2 \(m, V1+m2V2)-(m,
--+--+ --+
V1+m2V2)
62. A box is put on a scale which is adjusted to read zero , (a) zero (b) (rri, +.m 2 )gt 0
when box is empty. A stream of pebbles is poured into (c) 2(m1 + m 2 )gt 0 (d) :! (m 1 + m 2 )gt 0
the box from a height h above it's bottom at a rate µ 2
pebbles/sec. each pebble has a mass m. Consider the 68. A small disc of mass m slides
collision between pebble and box to be completely
inelastic. Find the re'ading of scale after t seconds of
down a smooth hill of height h i
from rest and gets onto a plank ih
:~: • i
I

falling of pebbles.
(a) µtmg
·
(b) µmt-J2gh
of mass M lying on the I)
horizontal plane at the hill. Due
__________,
M I

(c) µm(gt--J2gh) (d) µm(gt +-J2gh) to friction between the disc and the plank the disc
slows down and after a certain moment, moves in one

www.puucho.com
Anurag Mishra Mechanics 1 with www.puucho.com

[390 ,.· . MECHANICS,! I


. pi,;ce with the plank. Then the work performed by the (a) 65 ms-1 towards east
friction force in this process is: (b) 25 ms-1 towards west
mM (c) 65 ms-1 towards west
(a) zero (b) - - g h
m+M (d) 20 ins-1 towards east
M mM
(c) -gh (d)--gh 75. From a circle of radius a, an isosceles right angled
m M-m triangle with the hypote.nuse as the diameter of the
69. In the figure one fourth part of a circle is .removed. The distance of the centre of gravity
uniform disc 9f radius R is shown. . of the remaining position from the centre of the circle
The distance of the centre of mass is :
of this object from centre 'O' is : (b) (it - l)a
(a) 3(it - l)a
(a) 4R (b) 2R 6
3it 3it a
(c) -./2. 4R (d) -./2. 2R (c) 3(it -1) (d) 3(/+ 1)
3it 3it
76. A particle of mass 3 m is projected from the ground at
70. In the figure shown a hole of some ang\e with horizontal. The horizontal range is R .
radius 2 cm is made in a At the highest point of its path it breaks into two pieces
semicircular disc of radius 6it m and 2m. The smaller mass comes to rest and larger
~
at a distance 8 cm from the ~ .
mass finally falls at a distance x from the ,point of
centre C of the disc. The · projection where x is equal to :
distance of the centre of mass
(a) 3R (b) 3R
of this system from point C is :
4 2
(a). 4 cm (b) 8 cm
(c) SR · (d) 3R
(c) 6 cm (d) 12 cm 4
71. When.the momentum of a body increases by 100%, its 77. 1\vo balls A and B having masses 1 kg and 2 kg,
K.E. increases by : moving with spe~ds 21 m/s and 4 mis respec~vely in
(a) 400% (b) 100% opposite direction, collide head on. After colli/,ion A
(c) 300% (d) none moves with a speed of 1 m/s in· the same direction,
72. A ·small' sphere is moving at a constant speed in a then the coefficient of restitution is :
vertical circle. Below is a list of quantities that could be (a) 0.1 (b} 0.2
used to describe some aspect of the motion of the (c) 0.4 (d) none ,.
sphere. 78. 1\vo massless string of
I - kinetic energy length 5 m hang from
II - gravitational potential energy the ceiling very near to
III - momentum each other as shown in
Which of these quantities will change as this sphere . the figure. 1\vo balls A
moves around the circle? · and B of masses 0.25 kg
(a) I and II only (b) I and III only and 0.5 kg are attabched :.,o. AS .;;___f'
(c) Ill only (d) II and III only to the string. The a11 A •.'-~-~'-. , 8
is released from rest at '
73. From a uniform disc of radius R,
a height 0.45 m as shown in the figure. The collision
an equilateral triangle of side between two balls is completely elastic. Immediately
../3 R is cut, as shown in the after the collision, the kinetic energy of ball B is 1 J.
figure. The new position of The velocity of ball A just after the collision is:
centre of mass is :. ·
(a) 5 ms-1 to the right
(a) (0, O)
(b) 5 ms-1 to the left
(b) (0, R)
(c) 1 ·ms-1 to the right
(c) (o,~Rr. (d) ·1 ms-1 to the left ·

(d) none of these


79. An ice block is melting at; constant rate
.
ldml
dt
= µ. Its

7 4. A truck moving on horizontal road towards east with initial mass is m0 and it is moving with velocity on a
velocity 20 ms-1 collides elastically with a light ball frictionless hori.zontal surface. The distance travelled
moving with velocity 25 ms- 1 along west. The velocity by it till it melts completely is :
of the ball just after collision: •
www.puucho.com
Anurag Mishra Mechanics 1 with www.puucho.com

~
.,, :.,~ . ' '

(a) 2m 0v (b) mov sphere and tbe wall is e =.!. The velocity oftbe sphere
µ µ 2
(c) mov after it hits tbe wall is :
(d) can't be said
2µ (a) i- j (b) -i + 2j
80. A ball strikes a smootb horizontal ground at an angle (c) -i-J (d) 21-J
of 45° with tbe vertical. What cannot be tbe possible 85. A man of mass M stands at one end of a plank of
angle of its velocity witb tbe vertical after tbe lengtb L which lies at rest on a frictionless surface. The ·
collision? (Assume e S: 1) man walks to otber end of the plank. If tbe mass of tbe
(a) 45° (b} 30° plank is M, tben tbe distance tbat tbe man moves
(c) 53° (d) 60° 3
81. Two identical balls A relative to ground is :
and B are released from (a) 3L (b) ~
tbe positions shown in 4 4
figure. They collide (c) 4L (d) ~
elastically on horizontal 5 3
portion MN. All surfaces 86. Two particles of equal ma~s haye velocities 2ims-1
are· smootb. The ratio of and 2j ms-1 • First particle has an acceleration·
heights attained by A and B after collision will be:
(neglect energy loss at M and N) J)
ci + ms-2 while tbe acceleration of tbe second
(a) 1 : 4 (b) 2 : 1 . particle is zero. The centre of mass of tbe two particles
(c) 4 : 13 (d) 2 : 5 moves in?
(a) circle (b) parabola
82. As shown in tbe figure a
body of mass m moving (c) ellipse (d) straight line
vertically witb speed 3 87. A man weighing 80 kg is standing at tbe centre of a flat
m/s hits a smootb fixed boat and he is 20 m from tbe shore. He walks 8 m on
inclined plane and tbe boat towards tbe shore and tben halts. The boat
rebounds witb a weight 200 kg. How far is he from tbe shore at tbe end
velocity v f in the of tbis time?
horizontal direction. If (a) 11.2 m (b) 13.8 m
L. of inclined is 30°, tbe velocity v f will be: (c) 14.3 _m ... (d).1-'i~'!:_m
(a) 3 m/s (b) -.J3 m/s 88. · A sphere strikes a wall and rebounds witb coefficient
' 1 of restitution !. If it rebounds witb a velocity of 0.1
,(C) -.J3 m/S (d) this is not possible
3
83. The system of tbe wedge m/sec at an angle of 60° to the normal to tbe wall, tbe
and tbe block connected by loss of kinetic energy ls:
a massless spring as shown (a) 50% (b) 33,! %
in tbe figure is released 3.
:with tbe spring in its M (c) 40% (d) 66~ %
natural lengtb. Friction is f_1:__ _....:..c~L:,,~I 3
absent -maximum 89. A spaceship of speed. vi, tra~elling ,aloni;:)t,y axis
'elongation in tbe spring will be: suddenly shots out one fourtb of its !ffii't ~ speed
(a) 3Mg . (b) 6Mg . 2v 0 along +x-axis. ;,;y .axes are fixed witb respect to
5k 5k ground. The velocity of tbe remaining part is:
(c) 4Mg (d) 8Mg 2
(a) ,-v 0 ,
./2o·
(b) - v 0
5k 5k 3 3
(c)
.rsv
- 0
m
(d) - v 0
84. A smooth sphere is moving on a horizontal surface 3 3
witb a velocity vector (21 + 2J) m/s immediately 90. If tbe linear density of a rod of lengtb 3 m varies as
before it hit a vertical wall. The wall is parallel to A = 2 + x, tben tbe position of centre of gravity of tbe
vector J and coefficient of restitution between tbe rod is :

www.puucho.com
Anurag Mishra Mechanics 1 with www.puucho.com

[ 39~-2'-·~·------0-·~----~~-'--'--'----- ·'------ __----'_MECHI\NICS,.!J


(a) Zm (b) 12m
3 7
10 . 9
(c) -m (d)-m
7 . 7
91. Which of the following graphs represents the graphical b
relation between momentum (p) and kinetic energy

rl2
(K) for a body in motion?
. ...
.'
Ca)
l
l," p~
I
· -., . lI
. !
I
·I,
(b)!"P, .··,,'·.
'
}' ;. 1- • ' , - 'I 2a-c (b) 0 = tan-1 2a- c·_
(a) 0= cot -I -
I
, ~
.
~--lnK ___ :
. ;
L__ __l_n.JL.~ -
2b 2b
(cl 0 = coC1 c - a (d) 0 = coC1 c - a

1·~···.····i
'in p . . . r .
2b b
96. In the above, suppose that the smaller ball does not
stop after collision, but continues to move downwards
J

(c) '.: .. ·. . . . :: (d) None


with a speed = Vo, after the collision. Then, the speed
· f - , Ink ;. 2 .
92. A s~,;11 b~cicet ~f ~~ss .M kg is attached to a long of each bigger ball after collision is:
inextensible cord of length L m. The bucket is released (a) 4v 0 (b) 2v 0
from rest when the cord is in a horizontal position. At Js Js
its lowest position, the bucket scoops up m kg of water (c) ~ (d) none
and swings up to a height h. The height h in meters is: 2'-/5
(~)L
2
97. A body-of mass 'm' is dropped from a height, of 'h'.
(a) ( ~ .) L (b)
M+m M+m Simultaneously another body of mass 2m is thro,wn up

(c)(M;m)\ (d)(M;m)L verticall[ with such a velocity v that they collide at the
height -. If the collision is perfectly inelastic, the
2
93. A buggy of mass 100 kg is free to move on a velocity at the time of collision with the ground will
frictionless horizontal track. Two men, each of mass 50 be:
kg, are standing on the buggy, which is initially
stationary. The men jump off the buggy with velocity (a) ~S!h (b) ./ih
= 10 m/s relative to the buggy. In one situation, the
men jump one after the other.In another situation, the (c) ~ (d) -J10gh
3
men jump simultaneou_sly. What is the ratio of the
velocities of the buggy in two cases? _ · 98. In the figure (i), (ii) and (iii) shown the objects, A, B
(a) 5 : 4 (b) 5 : 3 and C are of same mass. String, spring and pulley are
'(c) 7 : 6 (d) 7 : 5 massless. C strikes B with velocity 'u' in each case and
94. Two men, of masses 60 kg and 80 kg are sitting at the sticks to it. The ratio of velocity of Bin case (i) to (ii) to
ends of a boat of mass 60 kg and length 4 m. The boat (iii) is: ·

.!~~
is stationary. If the men now exchange their positions, ·
then:
(a) the centre of mass of the two men shifts by 2 m
(b) the boat moves by 0.4 m
(c) the centre of mass of the two men shifts by~ m , (i) tC (ii) ______ (11l)__j
. 7
(d) the boat moves by 0.6 m (a) 1 : 1 : 1 Cb). 3: 3: 2
95. A billiard table whose length and width are as shown (c) 3 : 2 : 2 (d) none of these
in the figure. A ball is placed at point A. At what angle 99. A force exerts an impulse I on a particle changing its
'0' the ball be projected so that after colliding with two speed from u to 2 u. The applied force and' the initial
walls, the ball will fall in the pocketB? Assume that all velocity are oppositely directed along the same line.
collisions are perfectly elastic: (neglect friction) The work done by the force is:

www.puucho.com
Anurag Mishra Mechanics 1 with www.puucho.com

[J~P_!I_LSE ~~D 11\0MENTUM

(a) ~Ju (b) .!Ju 104. A ball of mass m is released from A inside a smooth
2 2 wedge of mass m as shown in ~e figure. What is the
(c) Ju (d) 2Iu speed of the wedge when the ball reaches point B ?
100. In the diagram shown, no friction at any contact . ·-------"$? .
surface. Initially, the spring has no deformation. What
will be the maximum deformation in the spring?
Consider all the strings to be sufficiency large.
Consider the spring constant to be K.

11· .· ll' 112


Smooth j

: °BoooooooooooooFl a:::;jll Ca) LR )


( 3-/2. (b) ~'lgR

'-·-····· ·-····- ----· - ····-·-·-·-····--·-_I


4F
(c) 5R)
(2./3
_!_
112

(d) ~~gR
(b) BF
Ca) 3K
3K 105. Two identical spheres move in opposite directions
(c) _f_ (d) None with speeds v1 and v 2 and pass behind an opaque
3K screen, where they may either cross without touching
101. A flexible chain of length 2 m and mass 1 kg initially (Event 1) or make an elastic head-on collision (Event
held in vertical position such that its lower end just 2)
touches a horizontal surface, is released from rest at (a) We can never make out which event has occurred
time t = 0. Assuming that any part of chain which (b) We cannot make out which event has occurred
1
strikes the plane immediately comes to rest and that only ifv 1 = v 2
the portion of chain lying on horizontal surface does (c) We can always make out which event has occurred
not from any heap, the height of its centre of mass (d) We can make out which event has occurred only if
above surface at any instant t = .Js (before it Vt= V2
106. A block of mass m starts
completely comes to rest) is: from rest and slides
(b) 0.5 m down a frictionless
(a) 1 m
(c) 1.5 m (d) 0.25 m semi-circular track from !·-:-~-----u.
'
!I ' h
' \ .
~--
. . '1
'
l
a height h as shown. . :'
102. A parallel beam of particles of mass m moving with
velocity v impinges on a wall at an angle 8 to its
normal. The number of particles per unit volume in
When it reaches the
lowest point of the
· · - -l-
••• •
. --- _____ __:
i • I

the beam is n. If the collision of particles with the wall track, it collides with a stationary piece of putty also
is elastic, then the pressure exerted by this beam on having mass m. If the block and the putty stick
the wall is : together and continue to slide, the maximum height
that the block-putty system could reach is :
'(a) 2mnv 2 cos8 Cb) 2mnv 2 cos 2 8 (a) ~ (b) ~ .
2 4 2
(c) 2mnvcos8 (d) 2mnvcos 8 .
1 (c) h (d) independent of h
103. A small ball falling vertically downward with constant. i07. Two billiard balls undergo a head-on collision. Ball 1
velocity 4 m/s strikes elastically a massive inclined is twice as heavy as ball 2. Initially, ball 1 moves with a
cart moving with velocity 4 m/s horizontally as speed v towards ball 2 which is at rest. Immediately

i -------
shown. The velocity of the rebound of the ball is:
?4m/s : after the collision, ball 1 travels at a speed of !:. in the
' '
same direction. What type of collisjon has occurred?
3

(a) Inelastic
(b) Elastic
(c) Completely inelastic
45• 4m/s 1
:L--,..._--~..._....L.> ! (d) Cannot be determined from the information given
I 108. A 4-kg disk slides over level ice toward the east at a
·----- --------' velocity of 1 meter per second, as shown. The disk
(a) 4,J2 m/s (b) 4..J3 m/s strikes a post and rebounds toward the north at the
(c) 4 m/s (d) 4/sm/s
www.puucho.com
Anurag Mishra Mechanics 1 with www.puucho.com

f~94,. .;. ,,,

same speed. The change in the · magnitude of the (A) Is the momentum of the rail car + bowling balls
eastward component of the momentum of the disk: is: system conserved in this collision? · ·
(a) Yes, the momentum is completely conserved
-~, . , ·. ·. .;, . ·+·. N·.. ' (b) Only the momentum component in the vertical
. ~t
- .. · Post -...
.,iW S
,
E
direction in conserved
. ' (c) Only the momentum component parallel to the
. 1 mis , track is conserved
Disk' Top View (d) NcHomponents are conserved: ·
(B) What is the average speed of the rail car + bowling
(a) - .4 kg mis (b) -1 kg mis balls system some time after the collision?
(c) 0 kg m/s (d) 4 kg mis (a) (M +Nm)v 0 •
1 Q9. Ari isolated rail car of mass M 'is moving along a , M
straight, frictionless track at an initial speed v 0 • The . Mv 0
car is passing under a bridge when a crate filled with (b) (Nm+M)
N bowling balls, each of mass m, is dropped from the
(c) Nmv 0
bridge into the bed of the rail car. The crate splits open
and the bowling balls bounce around inside the rail M . . .
(d) The .speed cannot be determined because there is
car, but none of them fall out. not enough information

,. ... ~

www.puucho.com
Anurag Mishra Mechanics 1 with www.puucho.com -·;'
'\'; .•
·-.::., .

t~PULSE A~_Mc.cO:.c. Mc::EccNT:.:.U::.:M_ _ _~ - - - - - - - _ _ _ _ _ _ __:_:_ _ _ _ _ j~j

~~-wJ~r-2 _ ·-· -~~~~-t-ha_-n_--i~--e_A~e:~~i_i·ve~~V_a_,.,_e_Correct


1. A uniform chain of mass M and length L is held (c) the total momentum of the ball and the earth is
vertically in such a way that its lower end just touches conserved
the horizontal floor. The chain is released from rest in (d) the total energy of the ball and the earth remains
this position .. Any portion of chain that strikes the floor the same
comes to rest. Assuming that the chain does not form a 5. Two identical buggies move one after other due to
heap on the floor. Then: inertia (without friction) with the same velocity v 0 • A
(a) Force exerted by chain on the floor when a length man of mass m rides the rear buggy. At a. certain
3 moment the man jumps into the front bnggy with a
x has reached the floor is Mgx
L velocityu relative to his buggy. If mass of each buggy is
(b) Force exerted by chain on the floor when a length equal to M and velocity of buggies after jumping of
x has reached the floor is 2Mgx man are v rear and v front. Then: ·
L m
(a) Vrear =v 0 + - - u
(c) Force exerted on the floor by the falling part of m+M
chain at any moment is twice as great as force m
(b)v =v 0 - - - u
rear m+M
exerted by the part already resting on the floor.
mM
(d) Force exerted on the floor by the falling part of (c) Vfront =Vo+ 2u
chain at any moment is same as force exerted by (m+M)
the part already resting on the floor. mM
(d) Vfront = Vo - - - - U
2. A ball strikes a wall with a velocity.lat an angle e with (m+M) 2
the normal to the wall surface and rebounds from it an 6. Suppose a cannon of mass M fires a shell of mass m
angle ~ with the surface. Then: with initial velocity u (relative to cannon) at an angle 9
(a) (0 +~)is never less than 90° if wall is smooth to the ground. In the process cannon recoils freely
(friction neglected). Then:
(b) If wall is smooth coefficient of restitution= tan~ 2 •
cote . . . u sin29
(a ) Range of the proJect11e 1s - - -
g
(c) If wall is smooth coefficient ofrestitution < tan~
tan9 . ·1 . M u2 sin29
(b) .Range of"th e proiecn

-------]
e 1s
(d) None of these m g
3. A ball of 1 kg strikes a wedge
of mass 4 kg horizontally
with a velocity of 10 m/s.
Just after collision velocity
of wedge becomes 4 m/s.
[ ----
~- ~
-- ---- ---
...

Friction is absent everywhere and collision is elastic.


'
()
.. .
c Ra nge ofth e proJect11e 1s

. ti! .
Cd) Range ofthe proJec e 1s
M u 2 sin29
M+m

M-m
g
M u 2 sin29
g
7. A nonzero external force acts on a system of particles.
Select the correct' altemative(s): The velocity and the acceleration of the center of mass
(a) speed of ball after collision is 6 m/s are found to be v O and a0 at an instant t. It is possible
(b) speed of ball after collision is 8 m/s that:
· (c) impulse between ball and wedge during collision (a) v 0 = 0, a 0 = 0 (b) v 0 = 0, a0 ¢ 0
is 16 N-S (c) v 0 ¢ 0, a0 = 0 (d) v 0 ¢ 0, a0 ¢ 0
(d) impulse between ball and wedge during collision 8. A small particle traveling ~,- - - - - - - ~
is 32 N-S with· a velocity v collides v
p
4. A ball hits the floor and rebounds after an elastic elastically with a spherical
collision. In this case: body of equal mass and
(a) the momentum of the ball just after the collision is radius r initially kept at rest.
same as that just before the collision The center of this spherical body is located a distance
(b) the kinetic energy of the ball remains same during p( < r) away from the direction of motion of the
the collision particle. After collision:
(a) particle stops and spherical body move in the
direction of particle
www.puucho.com
Anurag Mishra Mechanics 1 with www.puucho.com

'' · MECHA,Nl~S,1,j
(b) both particle and spherical body move 13. A cannon shell is fired to hit a target at a horizontal
perpendicular to each other distance R. However, it breaks into two equal parts at
(cl Velocity of particle after collision is vp its highest point. One part (Al returns to the cannon.
r The other part is:
(d) Velocity of the spherical body after collisioµ is (a) will fall at a distance R beyond the target
~~r2 -p2 (b) will fall at a distance 3 R beyond the target
r (c) will hit the target ·
9. A stone of mass mis dropped from a height of 200 mat (dl have riine times the kinetic energy of A
t = 0, At t = ls, a second stone of mass 3 m is dropped 14. Consider a hollow sphere
from the same height. If velocity and position of center
of mass are respectivelyy (from top) and v att = 5sec.
Then: (g = 10 m / s 2) . ,
(al y = 91.25m
(c) v = 52.Sm/s
(b) y = 81.25m
(d) v = 42.Sm/s
of mass M and radius R
resting on a smooth
sui:face. A smaller ·sphere
of mass m and radius r is
initially held at position P ·
:G~.Initial · Q final

10. Two particles A and B of equal size but of masses m kg within the bigger sphere. If the ~mall sphere is now
and 2 m kg are moving directly towards each other released, it rolls down the .inner surface of hollow
with speeds 21 m/s and 4 m/s respectively. After sphere and finally stops at its bottom point Q. Then:
collision, A moves with a speed of 1 m/s in the original (al Horizontal displacement of the smaller sphere on
direction. Then: smooth surface is M(R - r)
(a) After collisionB moves with.a speed of 6 m/s in a M+m
direction opposite to its motion before collision (bl ,Horizontal displace1Jlent 'of the bigger sphere on
(b) After collisionB moves with a speed of 6 m/s in the smooth surface is m(R + r l
original direction M+m·.
(c). The coefficient of restitution is 0.2 (cl Horizontal displacement of the bigger sphere on
(d) The impulse of the collision is 20 m N-S smooth surface is m(R - r l · '
11. Two friends A and B (each weighing 40 kg) are sitting M+m
on a frictionless platform some distance d apart. A rolls (d) None of these
a ball of mass 4 kg on the platform towards B which B , 15. As shown in figure, the block B of
catches. Then B rolls the ball towards A and A catches mass m starts from the rest at the
it. The ball keeps on moving back and forth between A top of a wedge W of mass M. All
and B. The ball has a ,fixed speed of 5 m/s on the surfaces are without friction: W
platform. . can slide on the ground. B slides
(a) speed of A after he catches the ball for the first down onto the ground, moves ,
' . -10mls
t1.me.1s
along it with a speed v, has an elastic collision with
11 wall, and climbs back onto W: Then:
(bl speed of A after he catches the ball for the first (a) B will reach the top of W again
. 1s
time . -10mls (b) From the beginning, till the collision with the wall,
' 9 the center of mass of 'B plus W is stationary
(cl The center of mass of the system will remain (c) After the collision, the center of mass of 'B plus W
stationary irrespective of the direction of motion ,
moves ',th el ·
wt av oc1ty - -
2mv
of the ball m +M
( d) A can rolls the ball for 6 times (d) When B reaches its highest position cin W, the
12. A ball of mass 1 kg is thrown up with an initial speed . 2mv
spee d ofw 1 s - -
of 4 m/s. A second ball of mass 2 kg is released from m+M
rest from the same height. Select the correct options: 16. A strip of wood of mass M and length I is ·placed on a
(a) The center of mass of the two balls comes down smooth horizontal surface. An insect of mass m starts
with· an acceleration off at one end on the strip and walks to the other end in
3 time t, moving with a constant speed.
(b) The center of mass first moves up and then comes (a) the speed of the insect as seen from the ground is
down l
<-
(c) The acceleration of the ,center of mass is g t
downwards
(d) The center of mass of the two balls remains
stationary
www.puucho.com
Anurag Mishra Mechanics 1 with www.puucho.com

j IMPULSE AND MOMENTUM 397


(b) the speed of the strip as seen from the ground is bounces back in vertically upwards direction relative
to wedge?
f(M~m) (a) -.fj (b) _2_
-.fj
(c) the speed of the strip as seen from the ground is

f(M:m) . (c) 2 (d) .!.


2.
(d) the total kinetic energy of the system is 21. The particle A and B each of
2(m+M)
1
(1)t 2
mass 'm' is attached by a light
inextensible string of length 21.
The whole system lies on. a
17. A sphere A moving with a speed u and rotating with an smooth horizontal table with B
angular velocity ro, makes a head on elastic collision
initially at a distance I from A. B
with an identical stationary sphere B. There ·is no
is given velocity' u' as shown in figure. Then just after
friction between the surfaces of A and B. Disregard
the jerk.
gravity. Then:
(a) A will stop moving but continue to rotate with an _ (a) VA 0
u-J3
=--
angular speed ro 4
(b) A will come to rest and stop rotating u-J3
(b)VB=-
(c) B will move with a speed u without rotating 4
(di B will move with a speed u and rotate with. an
(c) component of vB along AB is u-J3
angular velocfty ro. · 4
18. In an elastic collision between spheres A and B of ' u
(d) component of vB perpendicular to AB= -
equal mass but unequal radii, A moves along the x-axis 2
and B is stationary before impact. Which of the 22. A ball of mass m is released from rest
following is possible after impact:
(a) A comes to rest
at position 'A'. The mass of the wedge
is 2- m. When ball reaches point B
l~A
,
R I '
(b) The velocity of B relative to A remains the same in :, _ 2m_ _________
B . . !J
then:
magnitude but reverse in direction
(c) A and B move with equal speeds, making an angle
of 45° each with the x-axis (a) velocity of ball w.r.t. ground = ~ {gR
(d) A and B move with unequal speeds, making angle
. 7/3
of 30° and 60° with the x-axis respectively (b) velocity wedge=~
19. The ring R in the arrangement
shown can slide along a smooth
fixed, horizontal rod XY. It is
lRR-
I -~_-] s!l1I
B
3
(c) velocity of ball w.r.t. wedge = ~ ;R
attached to the block B by a light II
string. The block is released from (d) velocity of ball w.r.t. wedge = 3~
rest, with the string h9rizontal. Then:
(a) One point in the string will have only vertical 23. A ball is projected with velocity v 0 at angle 0 at t = 0
motion · with horizontal. It strikes a wall at a distance L from it.
(b) R and B will always have momenta of the same Coefficient of restitution is 'e' then: .
. 2v 0 sine
magnitude (a) Ball will return to ground at t =--'"---
(c) When the string becomes vertical, the speeds of R g
and B will be inversely proportional to their
masses
(b) d=e(R-L)(~is v
2
;'.;2())
(d) R will lose contact with the rod at some point 2 2
20. A ball A is falling vertically
------ ·. . , .
(c) maximum height dunng monon = ~v 0 -
sin 0
-
downwards with velocity v 1 • It 2g
2
strikes elastically with a wedge . . 2v 0 sin 0
(d) 1t will return to ground at t < - ~ --
moving horizontally with velocity g
v 2 as shown in figure. What must
be the ratio ~ so that the ball
V2

www.puucho.com
Anurag Mishra Mechanics 1 with www.puucho.com

3---".9~8_ _._,_..._...c...._ _ _ _ _~ - - ' - - - - - - - ~ - - - - ~ - - · · ~ ~ ~ - M _ E ~ = H ~


LI

24. The two balls A and B as shown in 2 7. A charged particle X moves directly towards another
figure are of masses m and ·2 m, ~harged particle Y. For the X plus Y system, the ,total
respectively. The A moves with momentum is p and the· total energy is E. , .
velocity u toward right while B is at (a) p and E are conserved if both X and Y are free to
rest. The wall is at extreme right is fixed. Coefficient of move
restitution for· collision between two balls is .! and (b) (a) is true only if X and Y have similar charges
2 (c) If Y is fixed, E is conserved but not p
between ball and wall is 1. Then speed ofA and B after (d) If Y is fixed, neither E nor p is conserved
all possible collisions are: 28. In a one-dimensional collision between two particles,
(a) VA =~ l (b) 1,IA =~ their relative velocity is~, before th~ collision and ~ 2
u u after the collision.
(c)vn=s (d)vn=4
(a) ~, = ~2 if the collision is elastic
25. A ball of mass m = 100 gm is
suspended from a point A by an (b) ~1 = - ~2 if the collision is elastic
inextensible thread oflength !. Ball.is
drawn ·a side and held at same level
(c) l~2l=l~,I in all cases
as A but at a dis~ance ../3 I from A as (d) ~, = -k~2 in all cases,_ where k <! 1
2 B
29. In a one-dim~nsional collision between two identical
shown in figure. Now ball is released.
particles A and B, B is stationary and A has momentum
Then:
p before impact. During impact, B gives impulse J to A.
(assume string applies only that much jerk which is (a) The total momentum of the ½ plus B' system is p
required so that velocity along string becomes 0) before and after the impact, and (p -J) during the
(a) velocity of ball just before experiencing jerk is ../ii impact .
(b) During the impact, A giyes impulse J to B ,
f3gf
-
(b) velocity of ball just after experiencing jerk is
vz . ,1·. . 2J .
(c) The coefficient of restitution 1s - ~ 1
'

(c) impulse.ap,plied by string Jfo (d) The coefficient of-~estitution is!_+ l


p

26. A block of mass M is placed


' · on a smooth horizontal f l o o r . . ~ .m
I
(d) ball will experience jerk after reaching to point B
I p
30. A block of mass m is connected to another .block of
mass M by a massless spring of spring constant k. A
The block has massless rod of . . constant force f starts I k . , _
length L. Pivoted on it at O. acting as shown in figure, - ~f
,}//4.r. "J

The rod has a point mass m attached to it's end. The then: - _, 1
whole system is released from the position shown in (a) as observed from ground both blocks ,will come to
figure. Then at the constant when rod becomes momentarily rest simultaneously
vertical.. (b) as observed from their centre of mass blocks will
come to momentarily rest simultaneously
(a) ratio of the velocities of M and. m =m '
. c--=--c- M (c) maximum extension in spring will-. · Zmf
(b) . vel~city of M is. m / 2gL k(m+M)
. VM(m+M) (d) maximum extension in spring will mf -'
, k(m+M)
(c) velocity of m is M ~
v~
(d) both m and M will move. left

www.puucho.com
Anurag Mishra Mechanics 1 with www.puucho.com

,.i
i:-·3·
,;· ,·· '
Comprehension Based Problems
- ,;,,

iis's 11:o)i :.:1 rr~


, I ,. " '": .',,.,}!A.'k
'·'~1 l
r: :,j · 5. If collision were elastic in above question then velocity
of B after collision in CM reference frame will be:

,While, dk;i!1l'lg ;with:collisiohs,berween particles, ;;;)l,


(a) zero (b) ~ mis leftwards
3
must•liaVe deal from an: ln:ertial. reference fratnel
Ofteh we ' ctibose that, fnttn~ fo
be fixed in the! (c) ! mis rightwards (d) 8 mis leftwards
Lab~fa,toix,;in which the, cQillsff>h js .observed. soit isj
called· the Labofatory reference' frame or Lab fralne
· Frdm.'fab,T~aihe';w~
,,_,'.i':'·,<<.·.·~--·
define,•!ltl elastic collisiorvas
,'' ,, i - ' , , ,. '
a..
collisiofr: hi,cwhich 'KE before .and after collision is ' ' ' ' :(' ' ,. --I ' \'•,

cpnseryed, .and a ,J?,erfectly' i!)(,!fast;ic: collision a,s \l Two blocks each of, mass· '.@' = ·1 kg ar\! con:n~l:ted
. collisio!'lJn ..whicli after ~o]li,~joh the two cojlidJng, with each other by a flexible light imd fri<;ti~\iless
· bodies' J:mve .same velocity fector along the· Jin~ 'df1 pulley. At t = 0 an
inelastic ball of, mass 'm'',;, ,1
actiori•of;inipulse during ~ol)i$ioh. • : .: ' coHides frpm bottom witli.the block A with yeJgdtyl
If we discuss the head-on, collision between two v 0 = li?,m1sec vertically up;wards, On the basis ·oJi
' particles ·from ~enter di:mass ~eference frame,, then abovfinformatiim. Answer',the following "qmistiops.

,ll,
' w•i>~•-•-i* *


the ve,loc,Hy of' center ·ofm~ss:{c;;M)'will be ta1'en'to be
, . , zero it).'~riy:trne of coJlision)if, velocity of CM befbre! ,"
and a:ffe!' collision wilJ botiVbe zero.' Since be{nrel
. collislo1i''velocity of CM w::r( zero (as our frame 'is
fixed 'id' CM) and no external impulse acts, it will •m t"o = 16mts1
temalh·t~er(f foreve~;.>: · "
·,._
1. From CM frame, after a head-on elastic collision:
(a) velocity of particles change in direction but not in 1. The velocity of blocks just afrer collision in ml sec is:
magnitude (a) VA= 16,vB = 0 (b) VA= B,vB = 0
(b) velocity of particles gets. interchanged (c) VA= 16,vB = 16 (d) VA= 0,vB = 0.
(c) velocity remains unchanged 2. The acceleration of both the blocks just after collision
(d) mom~ntum of each particle remains conserved in m / sec 2 is:
2. From CM frame, after a perfectly inelastic head-on (a) aA = 2g a8 = f
collision: 3 ' 3
(a) velocity of particles change in direction but not in = 2g
Cc) aA =!r_
3' a 8 3 (d) aA = aB =g
magnitude
(b) velocity of particles gets interchanged 3. The time after which string become tight is at t =
(c) velocity remains unchanged ......... seconds:
(d) momentum ofeach.particle"becomes zero
~
3. Two particles of mass 2 kg l-7,.-· -Q •
8-.-.,-~
(a)
5
(b) 1

and 1 kg as shown in the erm/s (c) j (d) ~


figure, make a perfectly Zk k ·
19 5 4
9
inelastic collision. Then if we ·- ·· 4. Velocity of block A just before string becomes tight
are dealing with center of mass reference frame, the will:
velocity of B before collision is: (a) always be zero
(a). zero (b) 8 m/s leftwards (b) may be greater than zero depending on v 0 (initial
(c) 4 m/s
.
rightwards (d) ~3 mis leftwards velocity of ball)
(C) may be greater than zero depending on mass of
4. After collision, the velocity of B in CM reference frame ball
will be: . (d) may be greater than zero depending on masses of
(a) zero · (b) 4 mis leftwards blocks
(c) ~ mis rightwards (d) -8 mis leftwards
3 3. www.puucho.com
Anurag Mishra Mechanics 1 with www.puucho.com

I 400 , r

5. The maximum height up to which blockA reaches is:


16 64 . 3
(a) -m (b)-m
5 15
16 A'bloc,k of mass 2 m collid~s,elastically with a m11hs m
(c) ~m (d) m
16 15 kept atl;rest. Friction eltist~',between the blockB ,tmd
6. The velocity of both the blocks just after string become surfac¢<With coefficientµ;,: 0.. 3, whereas no.ft;iction
exiStS' between !1lo_ck.A.?!id'.th!Lllurf(lce.

-
tight in m/sec.
8 9 m/s
(b)vA=vn=-
3
(c) VA= 4,Vn = 6 (d) VA= 6,vn ": 4 ,s\\\\\\\\\~\'\\
7. Acceleration of the two blocks just after string become A B
tight in m/ sec 2 :
1. The velocity of the blocks just after the first collision
(a)aA=a8 =!
3 will be:
(a) 12 mis, 3 mis (b) 0, 9 mis
(c)aA=~,a8 =½ (d) aA = ! an = 2g
(c) 3 mis, 12 mis (d) 0, 18 mis
3' 3
8. Energy loss due to the impulse applied by string in 2. The blocks will again collide after time:
joule is: (a) 6 sec (b) 4 sec
(a) 64 (b) 64 (c} 8 sec (d) =
9 6 3. How many collision are possible between the blocks?
(c) 64 (d) zero (a) 2 (b) 4
3 (c) 27 (d) infinite
9. The graphical representation of modulus of velocity of 4, After how much time collision between the blocks will

t7~L
.'. ·'
bloc~. B is.8corre.ctl.y sh~own
'I: +8/3 '•.··· ·•.
(a) : ;,sec .
.• ,.·n:····

5.
not take place practically:
(a) 8 sec
(c) 12 sec
(b) 16 sec
(d) infinite
Maximum distance between the blocks during the time
I . 4/5 8/5· • interval between first and second collision is:
. l"-· ·y~ ·- - -t(sec)-
----
· ···
(a) 12 m (b) 13.5 m
(c) 20 m (d) 9 m
i.B~·-.
I v•: :
• 6. Which of the following graphs best represent the
I T .. •
(b) , m/se,c . : 0 e · distance between the blocks between first and second
I 4/5 8/5' , collision?
IL_ .....
· _ ......
t (sec)-
---·-·--·-" I ·D. .. __,

(c)

!
~/t~~.c~1
., .• • ·415
' t(~,e,g)-,
. : :
(a) , · · 3·4~8 '
----~
t
r
(b)

,8t-----....
(d) t.r.-,,.. .
(d)
vt
m/sec.
8 0
"'----"-'-"-~
4/5
-HA-1:
~ - - - - - - - - ~ ,.i

,t(sec)-
~------,--~-.

www.puucho.com
Anurag Mishra Mechanics 1 with www.puucho.com

~~LSE AND ~M~MENTUM -401]


.- s·,:;-:~,j ~
PAS SAG'~ , ,.
One particle .of mass 1 kg is moving along positive
x-axis wi_th velocity 3.m/s. Another particle ofmass2
pA SAS:~: ~::l B :f
ma:·~~,, is suspended with IighJ
inelastic string of length t from a block A of samej
kg is moving along y-axis with 6 .m/s. At time t = 0, 1 mass m Which can move on smooth horizontal surface
kg mass is at (3m, 0) and 2 kg at (0, 9m). x- y plane· as shown in the figure. The ball is displaced by angle e
is the horizontal plane. (Surface is smooth for from equilibrium position and then released.
,_ _q~§tion 1 and roughfaLquestion 2 and 3),_ ____, A1m t7
1. The centre of mass of the two particles is moving in a
straight line: L-8~)µ=0
(a) y = x+ 2 (b)y=4x+2
(c) y = 2x-4 (d)y=2x+4 ~ ~B
2. If both the particles have the same value of coefficient
of frictionµ = 0.2 The centre of mass will stop at time 1. The displacement of block when ball reaches the
t =..... sec: equilibrium position is:
(a) 1.5 (b) 4.5 (a) L sine (b) L sine
(c) 3.0 (d) 2.0 2
3, Co-ordinates of centre of mass where it will stop (c) L (d) none of these
finally are: 2. Tension in srring when it is \'ertical, is:
(a) (20m, 14.25m) (b) (225m, 10m) (a) mg (b) mg(2- cose)
(c) (3.75m, 9m) (d) (1.75m, 12m) ·cc) mg(3-2cose) (d) none of these
3. Maximum velocity of block during subsequent motion
~--' • j ' '
of. the system after release of ball is:
PASSAGE·
'.
, (a) [gl(l - cose)J 112
A uniform chain of length 2L is hanging · in (b) [2g1(1- cose)J 112
equilibrium position, if end J3 is given a slightly (c) [gl cose] 112
downward displacement the jmbalance causes an (d) informations are insufficient to decide
acceleration. Here pulley is small and smooth and 4. The displacement of centre of mass of A + B system till
string is inextensible__. - - - , the string becomes vertical is:
(a) zero (b) :!: (1- cose)
2 ' .

(c) :!:
. 2
(1 - sine) (d) none of these

~:A$.(AGE !.•r ,, -I
1. The acceleration of end B when it has been displaced A ball of mass m = 60 gm is shot with speed v O ;= 22
by distance x, is: m/s into the barrel of spring gun of mass M = 240g
initially at rest on a. frictionless surface. The ball sticks
(a) ~g (b) 2x g
in the barrel at the point of maximum compression of
L L.
(c) -g
2
X
(d) g I th
e SP,_~~~ - --- -- M·;;24og ~- m =·60gm I

;;;;;;,,l:m,m~
2. The velocity v of the srring when it slips out of the 1
pulley (height of pulley from floor > 2L):
(a) ff (b) .f2gi 1.
(c) ..Jgi, (d) none of these

www.puucho.com
Anurag Mishra Mechanics 1 with www.puucho.com

•._·-------~_,__~-~'____________ ____M_EcHAtifcf'.!J
_4_02_"".""::_··~·· ;]~-~·-'_·_""·:,~
LI

1. The speed of the spring gun after the ball stops relative 6. The distance moved by the man with respect to ground
to the barrel, is: is
(a) 2.2 m/s (b) 4.4 m/s (a)__!!!!:__ (b) ~
(c) 4.4 cm/s (d) none m+M m+M
2. What fraction of initial kinetic energy of the ball is (c) m L (d) ML
now stored in the spring? M m
(a) 0.2 (b) 0.8 7. Choose the correct statement:
(c) 0.4. (d) 0.6 (a) As the man starts moving the trolley must move
backward
- ··~, "'" ·;~· (' 1W
, P,;A StSA:Gil "'! t~,1 ~~
r;·· .i~S'-·
- ·

(b) The distance moved by the trolley is independent
of the speed of the man
-_, '1' _., ···"T•" _,, • ' •·::-·, ;=,-· •s ~ - • ·,
(c) The distance moved by the trolley can never
The figure shows a man of mass m standing at the end exceed L
Aof a [trolley of_m~s-M,.~i,'}~ed at.rest on a smoi~ (d) All of the above
horizontal surface.- The ·man. starts' movinf't:owards
the end B with a velocity ,u,e1 with respect to ili_e
trolley. The length of the trolley is L. - .

.... ..
' i, .
,_ 'Jwo persons of mass m1 ;:trid m 2 are standing at the
twb·en_ds A and B respectively, of a trolley ofmass·M
At~\\\\~\\\\\\8\\\~ B ,as shown.
L
' ,-.-~--,-----'--~~-----,
m1 1-- -. m2 I .
1. When the man starts moving, then the velocity of the
trolley v 2 with respect to ground will be: A¼%(\\\\\~\\\\\\<t,~. B
(a) Murel (b) mu,el L
m+M m+M .

m M
(c) -Urel (d) -Ure! 1. When the person standing at A jumps from the. trolley
M m towards left with ure1 with respect to the trolley, then:
2. The velocity of the man with respect to ground v 1 will (a) the trolley moves towards right
be:
(b) the trolley rebounds with velocity miurel
(a)· Mu,.1 (b) mure1 . m+m
1 2 +M
m+M m+M. (c) the centre of mass of the system remains
m M stationary
(c) -u,el (d) -u,el
M m (d) all of the above
3. The time taken by the man to reach the other end is: 2. When only the person standing at B jumps· from the
(a) (m+M)....!:_. (b) (m+M)....!:_ trolley towards right while the person at A keeps
M Urel m Ure! standing, then:
(a) the trolley moves towards left
(c) ....!:_ (d) none of these
Urel (b) the trolley moves with velocity m2ureI
m1 + m2 + M
4. As the man walks on the trolley, the centre of mass of
the system (man + trolley) : (c) the centre. of mass of the system remains
stationary
(a) accelerates towards left
(d) all of the above
(b) accelerates towards right
(c) moves with ure1 3. When both the persons jump simultaneously with
.. same-speed then:
(d) remains stationary . . ... ...
(a) the centre of mass of the system remains
5. When the man reaches the end B, the distance moves stationary
. by the trolley with respect to ground is: (b) the trolley remains stationary
(a) __!!!!:__ (b) ~ (c) the trolley moves toward the end where the person
m+M m+M with heavier mass is standing ·
(c) m L (d) ML (d) none of these
.M m
.,...,,,'\..
www.puucho.com
-·.
Anurag Mishra Mechanics 1 with www.puucho.com

IMPULSE AND MOMENTUM 403


4. When both the persons jump simultaneously with u,el
with respect to the trolley, then the velocity of the
trolley is xr /A
Im, - m2lzire1 (b Im, - m2lz,re1
(a) ~ - - ~ - ) M ~ l

(I) (II) (Ill)


mzurel (d) none of these
m1 +M
X X X
A
5. Choose the incorrect statement, if m1 = m2 = m and
both the persons jump one by one, then: A A
(a) the centre of mass of the system remains B l
stationary
(bl the final velocity of the trolley is in the direction of B
the person who jumps first
(c) the final velocity of the trolley is (IV) (VI)

mu,el - mu,el )
( M+m M+2m 1. Which graphs pertain to physically possible
(d) none of these explosions?
(a) ii IV and V (b) ini VI

!PASSAGE I
10. ..::...:..;..;_____:
~" (cl ini v (d) Ii ui VI
2, Based on the above question. Match column A with
' the column B.
A projectile of mass "m" is projected from ground 1
with a speed of 50 m/s at an angle of 53° with the[ coi~mn:£i
:«i/,'),·,
', ',
horizontal. It breaks up into two equal parts at the', · ;(Graph :nu_i:rib11_r
highest point of the trajectory. One particle coming to! ----"--
(A) mA = mB (P) I
__ I_esJ illlmedia\ely_ a~er _th!! e_xpl_osion._ ___ j
1. The ratio of the radii of curvatures of the moving (B) mA > ma (Q) II
particle just before and just after the explosion are : (R) III
(C) mA <mB
(al 1 : 4 (b) 1 : 3
(c) 2 : 3 (d) 4 : 9 (S) IV
2. The distance between the pieces of the projectile when (T) V
they reach the ground are :
(U) VI
(a) 240 (b) 360
(c) 120 (d) none (a) A-U, B-R, C-P (b) A-Q, B-T, C-S
(cl A-Q, B-S, C-T (d) A-U, B-T, C-S
3. If following graphs are possible then, in which of the
PASSAGE following cases external impulse must be acting on the
An initially stationary box on a frictionless floor box?
explodes into two pieces, piece A with mass m A and (al II (bl IV
piece B with mass m 8 • Two pieces then move across (c) V (dl VI
the floor along x-axis. Graph of position versus time .,,--~ -----.
for the two pieces are given ?t - · 1_2 _' __]
PASSAGE
A rope ladder with a length l carrying a woman with a
mass m at its end is attached to balloon with a mass
M. The entire system is at rest in equilibrium in, the,
air. As the woman climbs up the ladder to reacb,
balloon,_ the ba_llool}_ descends by a height h. ~ ·

www.puucho.com
Anurag Mishra Mechanics 1 with www.puucho.com

- . . ·--. ,- r
'404 - - MECHANICS-I !
,_ .· ------- ---· ------- ·-·---- -- - - --- - ____1

-:---_- _ MATCHING TYPE PROBLEMS

1. If net force on a system of particles is zero, then match


the following:
--,
m
Column-1 _ _ _ _ _ _C_o__l_um_n-2. J
(A) Acceleration of centre of. (P) Constant
mass
1. The potential energy of the woman : (B) Momentum of centre of, (Q) Zero
(a) increases by mgl (b) increases by mg (l- h) mass
(c) increases by mgh (d) increases by mg (21- h) (C) Velocity of centre of mass (R) May be zero
2. The potential energy of the balloon :
(a) decrease by mg (l - h) (D) _velocity of an individual' (S) May be constant
(b) increase by mg (l - h) particle of the system
(c) increase by Mgl
(d) increase by mgl 2. In the arrangement shown in figure match the
3. The ratio of masses of woman and balloon (m/M)is: following:
(a) 1-h

(c) _l_
h

- l-- ---····-------·--
h
(b) _h_
1-h
(d) l - h
l
- 2m/s

r s)K~
All-smooth

. . Column-1
I'--'"------·~----~._. ____ . - - - ~ ~ - -
: ·· - C~l~;;,4:1:QJ
===..::_,._._ ',}_
(A) Velocity of centre of mass (P) 2 SI unit
A system ,onsists of block A and B each· of mass m (B) Velocity of combined mass (Q) 1 SI unit
connected by a light spring 'as shown with block B in when compression in the
contact with a wall. The block A compresses the spring is maximum
spring by 3 mg /k from natural length of spring and

;b '
then released Jrom rest. Neglect friction anywhere .. (C) Maximum compression in the (R) ,4 SI unit
spring
!
' k
/img/k (D) Maximum potential energy (S) 0.5 SI unit
' i~ 00000000 ':G]
.I __________ --·
stored in the spring

3. A particle of mass m, kinetic energy Kand momentum


·1. Acceleration of centre of mass of system comprising A P collides head on elastically with another particle of
and,£.just after A is released, is :
-~),:,g- (b) 3g
2
'
I
mass 2 m at rest. Match the following (after collision):
-·----·-·--
Column-1
. . .~,.-~
Co~n;2_'.~
(c) 3g (d) None of these ·cA) Momentum of first particle (P) ~p
2. Velocity of centre of mass of system comprising A and 4
B when block B just looses contact with the wall :
(B) Momentum of second particle K
(a) 3g {m (b) 3g {m · (Q) --
1/IZ 21/T 9

(c) 2g t (d) None of these (C) Kinetic energy of first particle


(R)
_J!_
3
3. Maximum extension in the spring after system looses
(D) ··Kinetic energy of second particle BK
contact with wall : (S) ·9
(a) 3mg (b) .J3mg
./2.k 2k (T) None
(c) .J3mg (d) None of these
./2.k
www.puucho.com
Anurag Mishra Mechanics 1 with www.puucho.com

IMPULSE AND MOMENTUM 4051


4. Ma_tch the following: 7. In the system shown in figure, m
(P = momentum of particle, K = kinetic energy of mass m is released from rest
particle) from posmon A. Suppose
potential energy of m at point A
with respect to point B is E. Dimensions of m are
(A) P is increased by 200%, corre- (P) 800% negligible and all surfaces are smooth. When mass m
sponding change in K reaches at point B:
,-- ---- .. -- .· --·
(B) K is increased by 300%, corre- (Q) 200%
sponding change in P
\-------
Column-1
------ - - ;;;·,::'- ·scolu;;;:;;.Jr~-:-7
.
E
----.,;
(A) Kinetic energy of m
(P) -
(C) Pis increased by 1%, correspond- (R) 0.5% 3
ing change in K (B) Kinetic energy of 2 m 2E
(Q) -
(D) K is increased by 1%, correspond- (S) 2% 3
ing change in P (C) Momentum of m .{4=:;
(T) None
. (R) V3mE
5. In a two block system shown in figure match the
following:
(D) Momentum of 2 m
(S) '~¾mE
Rough
(T) None

.5 m1..--j
t »ti10mls
2k 8. In column-I given are some quantities match these
Smooth with the situation given in column-2. Take the
situation just before and just after collision.
f~">'-·,,t;::'' r. -r:: .. ,.,"" __ r- ;:::-c??:1::-·----:--·
F~f~f,.,:}.iColumn-1_ . ~----- ~olumn-2: · L_ ·· :~{~oll:l.!'1':1".1_
(A) Velocity of centre of (P) Keep on changing all (A) Some mechanical en- (P)
mass the time ergy of m1 and m2 as a
system is lost during
(B) Momentum of centre of (Q) First decreases then be-
the collisions.
mass come zero
(C) Momentum of 1 kg (R) Zero
block Perfectly elastic
collision
(D) ·Kinetic energy of 2 kg (S) Constant
'block (B) Taking m1 and _m, 2 as' (Q)
system external 1mpul-,
6. A particle of mass 1 kg has velocity ,/1 = (2t) i and sive force are involved.
another particle of mass 2 kg has velocicyv1 = (t 2 ) j.
Match the following:
~-,l:;~r'.column-1 7_
--- ____ Column_;2______
Perfectly elastic
collision
(A) Net force on centre of (P) ZO unit
mass at 2 s 9 (C) ·Equation for coefficient. (R)
of restitution can be':
(B) Velocity of centre of (Q) .J68 unit used (along the line of
mass at 2 s impact)
( C) Displacement of centre ..Jso .
(R) - -1111lt
ofm.ass in 2 s 3

(S) None

www.puucho.com
Anurag Mishra Mechanics 1 with www.puucho.com

f -406' MECHANICS-I I.
r '
I L...:;....~ __ _.::-:.,___ ---1 :I
;:·,:;~~;-! iCp~f!1!1~1 ;~0;:t ~ "!:.,~C~i~~1f;~
i (S) 'v;
;i Eil (;SJ
4 4;,~ 1 ---,
1-~--'"-"';;;1.•.. . . .. '
rough I
m~m~"surfacej (A) Ram alone jumps to the _left !(P) I_ 17 m/ s
I , Perfectly elastic , I 40·
I _collisio~ 1 I (B) Shyam .alone jumps to ·the j (Q) --m/s
1
' .... ~ )I

right 2
..
9. Two men of mass 60 kg and 80 kg stand on a plank of
mass 20 kg. Both of them can jump with a velocity of (C) Ram jumps to left and Shyam (R) I~ m/s
lm/s relative to the plank. In each event shown in
column-I, find the velocity of plank after the event.
bumps to right simultaneously, 1
' I
ls
'
I
60kg +.
ao·kg __ _
i
(D) jRam jumps _to left an_d after/CS) )-~m/s
1that Shyam Jumps to nght I . 8

I ,/,!,:~.:,,,.~::.~,:~ !
[ Smooth levAground ~~--J
v

[ AN8WER8

1.
9.
(d)
(a)
I i I 2.
10.
(c)
(b) .
3.
11,
. (d)

(b)
I
I
4.
12.
I
I (b)
(c) I1 s.
) 13.
I
ii
(c)

(a)
/

'
6.
14.
(b)

(d)
7.
15.
!
,
(c)

: · (c)
i
I
,
8.
16.
.Cb)
(b)
\ '
17. (c), 18. (d)
'
I 19,
!
(c)
I

\ 20. i
'

(a) \ 21. ' (b) I 22. (c) ! 23. r (b) i '


24 , (b)
25. (a)· 26. (c) , 27. (c)
I
28. I (b) 29. i (d) 1· 30. ! I cc) : 32. (c)

33. (c) 34. (a) I 35. (b) I 36. (b), 37. (b) 38. (c) 39. I CdJ 40. . ca) .
41. (a) 42. {c) I 43.
(b) l 44. (d) 45. (d) .. '
I
46. (a) 47. (b) 48. (c)

49. (a) so. cc) I 51. (b) 52. (d) 53. (a) 54.
I (b) I 55. (d) 56. (b)
57. (c) 58. , (a). I 59. (d) 60.
I
(c) 61. ·ca) 62. I
i
(d) 63. (a) 64 (b)' '
65. (c) I 66. (d) 67. (c) I 68. \ (b) 69. (c) 70. (b) 71. (c) 72. .(d)
73. (b) 74. (a). 75. .. (c) , 76. _(c), , 77, (b) l 78. (d) 79. ' (b) 80. (b)• i'

81. (c) 82. (b) 83. i (b) I 84. (b) 85. (b)
l
I
86. (d) 87. ,'(c) 88 (d)
\
89. (b) 90. I CbJ 91. (d) 92. (a) 93. (c) 94. (b) 95. , (a) 96. ·;ccJ°,;'
' , ·-. : .
97. (d) 98. (b) 99, (b) 100. (b) I 101. . (d) 102. Cb) 103. (d) 104. (a)
105. (a) 106. , (a) 107. . (b) 108. cal I 109. I' A-(c) 109. B-(b) I
. . • • . h-
Level~2: More than One Alternatives are Correct· • :--..,
,; ~

1. (a, c:J 2. (a,~) 3. (a, d) 4. cc. d) I s. (b, c) 6. (c)


7. Cb, aJ 8. (b, c, d) 9. (a, d) 10. (a, e, d) i 11. ' . ca; c, dJ ,' 12 . . (b, c)
13. · i ' (a, ii) · 14. (a, c) I 15.
I •
(b, c, d) • 16. (a,'c)~ I 17.
I, (a, c) i, 18.
i
! (a,b,c,d)
19. !
\'
(a, c)
" '
20. (b) - I 21. Ca, c, dJ I 22.
I

i
(a, b, d) i 23. 1.
1 (a, b, c) ; 24. '
j
'
(a, d)
25. (a, b, c) 26. (a, b, c) i 27. (a, c) ' 28. (b, c, d) : 29. (b, c) · I 30. I (b, c)

www.puucho.com
Anurag Mishra Mechanics 1 with www.puucho.com

=, ·. ---~,-~ ~,_.~-- __ , __,

1i~1PU1SE AND rnoMEilruri


t.__ - - - - ~ ~ - - - - - - ~-- "' --·----~ ''" "''
' " 1@
c:;:•;tr:<~/: ' - , 0, • :.,:,j?i!11f,,Wg,i,,,YM,.,, ·~-
0
Leve1~~j;•CJ rnprehension"1ia~:lli:oblem~J4i'.'ij.;,;;~
: . . . . . o , ~ ,·;.·.,·{;.,,
" ,,,_, ·''.i;i•'·"i' ':/' ,' ··,, '·~'<;;_+-~!i-.K/2::\rw
e •

Passage-1:
1. (a) 2. (d) 3. (d) 4, (a) 5. (c)

Passage-2:
1. (b) 2. (d) 3. (c) 4. (a) 5. (b) 6. (b)
7, (a) s. (c) 9. (a)
Passage-3:
1. (c) 2. (c) 3, (d) 4, (c) 5. (b) 6. (a)

Passage-4:
1. (b) 2. (c) 3. (d)

Passage-5:
1. (a) 2. (c)

Passage-6:
1. (a) 2. (d) 3. (a) 4. (b)

Passage-7:
1. (b) 2. (b)

Passage-a:
1. (b) 2. (a) 3. (c) 4. (d) 5. (a) 6. (b) 7. (a)

Passage-9:
1. (a) 2. (d) 3. (a) 4, (a)

Passage-10:
1. (a) 2. (a)

Passage-11 :
1. (a) 2. (b) 3. (d)

Passage-12:
1. (b) 2. (a) 3. (b)

Passage-13:
1. (b) 2. (b) 3. (a)

1. A - Q; B - P, R; C - P, R; D - R, S 2, A - Q; B - Q; C - Q; D - P
3. A - R; B - T; C - T; D - S 4, A - P; B - T; C - S; D - R
5. A - R, S; B - R, S; C - Q; D - Q 6. A-Q;B-R;C-P
7, A- Q; B - P; C - R; D - R 8. A-P,R;B-P;C-P,Q,R,S ,
9, A - R; B - Q; C - S; D - P

www.puucho.com
Anurag Mishra Mechanics 1 with www.puucho.com

11. (b) r .. v
1. (d)
tl-u .
-1 = - - =}V'= 2u+v
'i .,_..,. ·l-1·
O,
1

-v-u !' I
'" u.,
After each collision each block will come to rest and i, ·~1
I" ·.·
the other will move with v.
mv+O+.. :... v 1 (
W=il.KE=-mv+2u )2 1
--mv 2 i ~ :_ .__· ~ l
VcM = . . l :~ !
2 2
2. (c)
m+m+......... nnmes n
.L._·.· .:1.td
When boy will walk, friction force will act forward, ·= .! m(4u 2 + 4uv) = 2mu(u +v)
2
boys shifts· right and the block does not move (since
14. (d)
lower surface is rough). Therefore, centre of. mass
shifts. After each collision vertical velocity becomes e times
4. (c) whereas horizontal velocity remain same. So after
each collision time of flight becomes e times of
Conserving the linear.momentum of (water + wagon)
previous one. So that horizontal d_isplacement =
system
R+eR+e 2R
5xl0 3 xl.2+10 3 xO= 6xl0 3 v 2 • 28
= (1 +e+e 2 )u sm
.> V = l m/S
g
Final KE = _! X 6 X,10 3 X 12 = 3 X 103 J 16. (b)
2
To change the linear momentum external force is
Initial KE = _! X 5 X 10 3 X (1.2) 2 required.
2
18. (d)
= 3600J
Change in. KE = 600 J Since each bullet hits the combined mass when springs
6. (b)
is at relaxed position so, conserving linear momentum
of n bullets and ma~s M.
No horizontal force acts so &(Hz) = 0
nmv 0 = (M + nm)v
10Mx+M(x+2.2) = 0
Also if A is amplitude of SHM the
9. (a)
mv 0 -2mv cos30" v=Aro=A~
V=~
../3 nmv 0 =Ar------;:-- =}A;, -nmv 0 _
v-0 M+nm - ' i j ~ .Jk(M+nin)
-e=-----
O-v0 cos30° 20. (a)
--J3 Vo At top point net linear momentum = mv cos8'
evo2= ../3 After collision assume velocity of second piece be v 0
2 then ·
e=·- . m m
3 mvcos 6 =-'--Xvcos6+-xv 0
2 2
v 0 = 3vcos~

www.puucho.com
Anurag Mishra Mechanics 1 with www.puucho.com

f~I_MP_U_LS~EA_N....
D_M~O_M~E_NT_U_M_·- - - - - - - - - - - - - ~ - - - - - - - - - ~ · ~
21. (b) 35. (b)

. XcM
·M
Mx0--R
4 R
=---~-=--
M-M
4
3
I@
L-········--·
Let Vi and v 2 be velocity of A and B after collision.
Then mxlO= mvi +mv 2
V2 -Vl
~-~=-e
-10
.... (i)
..... (ii)

23. (b) By conservation of energy


-R $ XCM SR
mxl0x 3.2= -1 x mxv 22
24. (b) 2
Because momentum of B will increase while that of A v~ = 64. [from (i)]
will decrease. V 2 = 8·m/sec
26. (c)
vi = 2 [from (i)]
V2 -Vi$ lQe
0

6 $ lOe
3
e~-
5
0+M 2 xd 38. (c)
XCM = .
M2 +:\.n
(d)2
2
For elastic collision
velocity is interchanged
(·: mass is same)
d 4d
=--=-- 1 1 kx2 . fn%
1+.:: 4+it 2 mvo =
2

2 i =} X1 =v,f
4
For completely inelastic collision
i.e., right to centre of disc.
Vo
28. (b) mv 0 = 2mv =}V = -
2
First graph - straight line with constant slope, 2nd
1 1 = ~mv
2k
2 2
graph - parabolic graph, 4th graph - constant 0

acceleration.
Now
2 kx 2 = 2 x2mv ='>X2

30. (c) For e = 0.5


1 T itFoTo -0.5 = V2 -V1
Fdt = mu :e,-7tF0 . -0= mu :e,u = - -
f 2 2 4m 0-Vo
31. (c) Vo ..... (i)
V2 -V1 =-
Vertical component after collision = eu cos0 2
Horizontal component = u sin8 Also mv 2 + mv 1 = mv O .... (ii)
='> tancj> = u sin8 = tan0 From (i) and (ii),
eu cos0 e
v 2 = 0.75v 0 :e,x3 = ~
34. (a)
Let equation of the line is y = mx, m = tan8 2 slope of 40. (a)
line 4x- 3y = 0 is tan 01 = 4/3 Collision is with A only so vn = 0 Gust after collision)
. 0 4 8 3
S!Il 1 =5' COS 1 = 5 For collision with A
Vo
Conserving momentum about:x and y axis mv 0 = (m + m)v .=:. v =2' _,,__ "' ..
100 x 10 = 100 (v cos8 2 ) + 200 (5 cos8 1 ) ,- ... ,

3
10=vcos82 +lOx- =}Vcos8 2 = 4 ~ fr; fSm/sec)
5 41. (a)
Similarly; v sin 8 2 = 12 ~xl0xt 2 =5xt-~xl0xt 2
12 2 2
y = -X=}y-3x= 0
4 10 t 2
= St :e,t =~sec
2

www.puucho.com
Anurag Mishra Mechanics 1 with www.puucho.com

[41ff' ,.·' _,. , • :l.: ,.' , - ' -~ ''


43. (b) => V2-V1=4V ;,,: (ii) '
Conserving the momentum of two balls before and From (i) and (ii)
after collisirm in horizontal direction · 5v . -7v

,-~~------~l
V2 =3,Vi = 3
lx6= (l+l)xv
v = 3m/sec V1 7
-=·-
Now at the instant of Vz 5

~~;~ r ~ :.~~·
48. (c)
Suppose the mass is 4 m. Horizontal momentunl'will
remain conserved. i.e.,
ball w.r.t. trolley will be O) 4mvcos37°= mx0+ 3mv
320
Let it be v 0 => v=-m/s
- 3
2x3=(4+2)xv 0
Time taken by mass to reach the ground·
Now conserving energy
vsin0
,! X 2X 3 2 - 2X lOX 1.5(1- cos0) =--=6sec
2 - ,g
Disp,lacement (horizontal) of ~ m mass
=.!x2xv 2 +.!x4xv 2 . 320
2 2 =-x6= 640m
' 3
=> cos0 = 0.8 => 0 = 37°
45. (d) Distance fro~ projection ·point = 640 + Range
- 2
The mass m strikes the surface with u = ~2gh and
moves along the surface with u cos0 cos0 = hll as , =1120m
collision is perfectly inelastic and finally leaves 'the 49. Ca)
plane with velocity (u cos0) x cos0 in direction J_ to First collision occurs at t = 1tR
AC

r·-
If~
-~- _- -•-~
c,os 8) x cos 8 ~
Ae'
10
..
After collision their relative velocity =eu
. will occur after' = --
So second coJlis 10n 21tR = -2t
eu e
u. .

I , -' C • .u cos 8 B
.' . .':
'
'
Also after that energy is conserved '

2 2
=> mgS = .! m(u cos 0)
z .
'
' '
'

=> S= h/ (Putcos0 = T)
47.. (b)
Rest masses be m and 2m

,_e -s
A velocity before collision be 3v and v respectively
3v' V

-
"2

(§)
50. (c)
'',, ..~ . ·> J '1
-•,=14kg '-"+•2=0'
[},00_9ooooovfl . l
( m1 =,10kg, :_· :m2 ~4,kg
before collision :after collision

. Conserving momentum m1 v1 +m2 v 2


VcoM =
·3mv - 2mv =; mv 1 + 2mv 2 m1 +_m 2
=> v 1 +2v 2 =v .. : (i) _ 10x14+ 4x O = 140 = lOm/s
Also e=l 10+4 14
-l=V2-V1
, ·-v-3v

www.puucho.com
Anurag Mishra Mechanics 1 with www.puucho.com

j JMPULSEJ\ND MOMENTUM 411j'


st:- Cb) 0
µ = ·64 = 0.16
Mass of man = m 4
Mass of ladder = M - m 57. (c)
A- _ Mx-(M - m)x+ m(l- x) After first shot
=cM - 2m _

52. (d) ,
=
mx+ml-mx ml
2M. 2M .
X

=:,
·- 200
0 = 200m-49mv =:, v = - mis ,
Since velocity and its magnitude depends upon · 49
reference frame so P and KE should depend upon After second shot
reference frame. ,
53. (a)
G I
. .,.-....20011
We known location of centre
of mass of the shown system is
v' .. - ~ a·
at ---49mv = ---48mv'+m(200- v)
· 4 (R 3 -R 3 ) 2
X-= 2 I -200 = ---48v'+200- 00
. 31t (Ri -Rf) 49
So locating of combined 48
48v' = 200 + 200 X
system 49
- 3 3
2R2 -M 4 (R2 -R1)
M• 2 2
v' = 200 (..!.. + ....!..) m/s
48 49 '
1t 31t CR2 -R1)
= 2M 58. (a)
CR2 -R1)C2R1 +R2) Above '
=~~~-~-~ ,.E._ -c1o~v)
31t(R1 +R2) [E] OJ '
54. (b)
..al position
Imt:J. . . ofC-M = -
lx0+lxlO
- - - - = 5m Velocity of ball= figii =10 m/s
2 He throws horizontal then suppose its velocity is v.
vCM=lx5-lx3=lmls Force is same =:, velocity of ball = 10 - v
. 2
So 60v = 1(10 .:.v)
Displacement of CM in 2 sec = 2 m 10
=:> V =-mis
So location of-CM= 5 + 2 = 7 m 61
55. (d) 10
V = lO-v=l0- 61
Velocity of ball before first collision = ~ 2gh max.

Time taken between _first and second collision = 600 = 9.9 m/s
- ·= 2efigii 61
g 59. (d)

So 2e..figh + 2e~ figii < ~ XCM =


t
0
x(A + Bx)dx x A
3AL + 2BL2
=----
g g fg- , f ~(A + Bx)dx x A 3 (2A + BL)

2e 2 +2e 2 -1<0
·:J3-l
=:,e<-- 60. (c)
2
Momentum in direction of velocity of B is conserved.
56. (b) Also velocity of A = velocity of C due to symmetry
Conserving linear momentum Just before collision velocity of A = velocity of (C
0:020 X 500,;, 10 XV+ 0.02 X 100 upward) = velocity of B
v = 0.8rn/s Let this velocity is v
So using . v 2 = u 2 + 2as for block.
mv 0 = mv+mv+mv =:>v = ~ = 3 mlsec.
0 = 0.8 2 - 2µ X 10 X 0.2 . 3

www.puucho.com
Anurag Mishra Mechanics 1 with www.puucho.com

,. --

,.·
412· :: • MECHANICS:fl
61. (a) The resultant CM of rods must
Let velocity .of.A be vx in lie on the edge for maximum
. horizontal direction just overhang.
before collision. ~ ' if n rods are placed on the
:. velocity of C will be vx table the CM oftop (n -1) rods
in just opposite direction must lie on edge of the
Energy is conserved . n th rod.
· (L ) , =L-
(2 ) (v )2
1 2 . 1 v 1 M --x =M(n-l)x =}X
. +v 2 +-m - 0
-xmxv 0 =2x-mx - 0 2· n n n 2n
2 .2·9X2 3
Here n=4
V
X
2 = 27 =} maximum overhang= Xi + x 2 +x3 + x 4
LLLL25L
=-+-+-+-=-·-
2 4 6 8 24
66. (d)
Also
From conservation.of momentum 0.02u + U02v + lvi
= 27 +, 9= 36. -(where vi be the velocity of plate of 1 kg) and
vA = 6m/sec 0.02v = (2 98 + 0.02)Vi
62. (d) (·: plate of 3 kg has also same velocity i.e., vi)
After t second total mass of pebbles inside box = or v, = 0.02-
V

µtxmg 3
th V
Also at 't ' s~cond rate of change of momentum Substituting 0.02u = 0.02v + 0.02-
= µ X m XV = µ X m.J2gh . 3
This will also
exert a force or
Av 3u'
u =-orv·=-
3 4
F-= m µ.J2gh (F = : )
% age loss in velocity = '.'!...::!: x 100
So total reading of scale = µtmg + µm.J2gh
u
3
= µm(tg + 2,{gii) v--v
= --4- X 100 = 25%
63. (a)
V
At the instant of ·maximum compression both the 67. (c)
blocks connected to spring'must have same speed
. Now conserving momentum
mv 0 = mv+2mv =}V = -Vo = change in momentum of the two particle\
3
Now collision is elastic , i
=_Iexternal orce on .the system\ ~ time interval
121 21212 = (m1 + m 2 )g(2t 0 )
=} -mv +-2mv +-kx =-mv 0
2 · 2 , 2 2 = 2(m1 + m 2 )gt 0
68. Cb)
X=~Vo
Speed of ball at bottom of hill = .J2gh
64. (b) Conserving linear momentum of (m + M)
No horizontal force acts "!1 the -system, so centre of m.J2gh = (M + m)v
·--~-. mass will not move horizontal. It will move ·· work done by friction = ~KE ·
downwards due to force of gravity.
65: (c)
= mgh. _ .!_2 (M + m)v 2
The maximum overhang of top most rod is L/2 and for = mgh _ (M + m) (m,.fiifi) 2
second top most will be .
2 (M +m) 2
L ). L
Mx2 =M ( 2 - X2 =} X2 = Mmgh
4 =--
M+m

www.puucho.com
Anurag Mishra Mechanics 1 with www.puucho.com

IMPULSE AND MOMENTUM~ 413 1


mu
=~;v;.-'~~;e_t;a: ~;~~te;a~~;fs;:re ~o~'c;;: ·, -, => v=v 0
M+m
----

Conserving linear momentum of man and front buggy


1. (a, c)
m(u+v)+Mv 0 =(M+m)v'
If x length has arrived on floor then weight arrived
M
=rxg m(u+v 0 -~)+Mv 0 -(M+m)v'
M+m

Momentum change= M dx,./2gx (M + m)v


0
+ Mmu. = (M +m)v'
M+m
Mdx
Rate of change of momentum= --v2gx
L dt
M
L
= -')gx
L
= ,
V =Vo+
Mmu
(M+m)2
2Mgx Mgx 3Mgx 6. (c)_
Total force = - - + - - = - -
L L L Suppose common recoils with a speed v, then:
2, (a, b)
If coefficient of restitution is e
r··- -~~u I
tan8 . tanP
tan(90°-P) = - - ~ e = - -
e cos8 W.r.t. ground l
U'
Since e < 1 so, tanP < cot8
~
~
90°-P-< a
a+P>90°
-;-CJ --~_rf_e _/
3. (a, d) w.r.t. ground
Collision is elastic. So, KE is Conserving horizontal momentum
conserved. _Let v be the speed of
ball after collision then,
~sin3~

0

l
I
m(ucos8-v) = Mv
mucosa
J ' V=---
~(1)10 2 = ~4{42 )+~lv 2 M+m
2 2 2 , ' '' Range of projectile = Horizontal velocity
J CC?~ ~9_;
or, v = 6m/s x Time of flight
Let J be the impulse between the two during collision. · 2usin8
= (ucos8-v---
Apply impulse = change in momentum g
Jsin30°= 4x4 M u 2 sin28
=---
J = 32N-S M+m g
4. (c, d) 7. (b; d)
Momentum changes its direction after collision. Also Fner ,;, 0, vcM may be zero but it will not remain so.
during collision some KE changes into deformation
8. (b, c, d)
elastic potential energy:
It is an elastic collision

:~c-·. -~
No force acts on (Earth + Ball) system ·so momentum
between two bodies of
wm remain conserved equal mass. So velocity
· Total energy of two colliding body is always along common normal
conserved .. will be interchanged and I !
5. (b, c) along tangent will j' ___1/ cos 8 -----...!I
;----, --------------7 remain same:

I~vo lnf:"o ·I Velocity ofparticle will be only along common tangent

,:=~~2,r-l
=vsin8=_vf>.
r

L!"S·
Conserving linear momentum of man and rear buggy
Velocity of spherical body _will be along common
. tangent and that also equal.to particles initial velocity.

= vcos8= ~1-P
2

r2
(M + m)v 0 = iw:v + m(v +u)
So, the two velocities are also perpendicular
www.puucho.com
Anurag Mishra Mechanics 1 with www.puucho.com

_.,t414 ,. :r, , ,, .·-~ ;- ,-":~ECHAN~~


9. (a, d) velocity of·A will•become greater.than·S-m/sec and so
he can't catch the ball after- that. · ·
After 1 sec Ym =0+.!x10x1 2
' ' ·2 12. (b, c).
· v,,;=O+loxl'=lOm/s lx (-g)+2(-,g)
a - -g
_ mxs+o _ CM - 1+2 ·
So. YCM - - 125
. m
. 4m i.e., g downwards.
_mx10+0_ - 1X4+2x0 4
vCM - - 250
. m vCM = · - - upwards
4m . 3 3
aCM =
mx10+.3mx10
4m
10m/s 2
3 2
4· 1
SCM =-xt--x10xt 2 =:,SCM·=t --St
3
(4 )
In next 4 sec, 4
=:. SCM = 0 at t = 0, t = -
' '' '. 1 ' 2 5
liYcM =2.5x4+-x10x4
2, centre of mass first moves up then downward.
= 10-fBO = 90 cm 13. (a, d)
Position of CM = 90 + 1.25 =. 9}.25 .----------~~
u cos e u·cos8
VCM = 2.5+10X4= 42.Sm/s
.;.,.._.,.
0 - o 0:~v
•, '·10. · (a, c, d) 2m m m
Before collision Initial Final
~ - ,..,_
21 mis ·4mls
After collision
0m 02m Conserving momentum in x-direction

-
By conservation ·of linear 2mu cos0 = mv - mu cos0 ,:
momentum: 1m/s
~v v = 3ucos0 .
2lm-8m= m+2mv 0m
'

2 Since velocity of 2nd part is 3 times.

1.2.:· .•
12
v=-= 601/s
'•' 2
6-1,
--e =
-4-21 ,
" , 5 · 1
. ,.. . " e = 25 "' 5 = 0.2
it will cover a distance 3 x R/ 2 .
. ·Impulse = 2m(6+ 4) · =:, it fall at a distance R beyond the ·wg~t and KE
='20mN-S _increases by 3 2 times. · · ··
'·i'i. 'ca: c; d)
1
14. (a, c) · · .,
After first throw velocity of A: Let hollow sphere moves a distan~' :it leftwards as no
external force is applied in horiz_ontal direction.
1·v ~;';_© 5
1
=}
,ixCM = 0
Mx+m(x-(R-r)) =·o
' 1 ' M+n
40v = 20 =:, v = -2 mis
" ,. =:, Mx.+mx=m(R-r)
After A catches the ball first time x = ~ (R - r) = displac_ement of bigger sphere
M+m · · ·
~, §7 Displacement of smaller sphere= x- (R - r)
M(R-r)
5 ' = M+m
--- V
..
= 40 = 10 m/S
V 15. (b, c, d)
44 11 From beginning, to the collision no net external force
No external force act in horizontal direction. The is applied so CM; of (W + B) remains s~ponary
certtr~ of mass will rem'M..n·sta~(!l).an'-: Afte~J!achthtow· ' :conserving
.... '. momentum
-~ . on....x-direction. · ··
'
·
~

the momentum of ball increases !Jy 2mv = 40 kg m/s. :~..,... ·o=mxp.+Mv 1


So after each throw the momentum of ball increases -s~=- _ -mv .. -- 111v toward left
by 2mv = 40 m/s. So after each throw B gain a v 1 = - - Le., -
M ' · M
(lnoinentum = 40 kg m/s. Also after sixthwww.puucho.com
throw the .. ..-~ ';'""'
Anurag Mishra Mechanics 1 with www.puucho.com

fl~i>UlSE AND MO~TUM


B has an elastic collision with wall
r-·--·· . --- - ------· - , ··1
1

='} it's velocity direction will be reversed


, ~: : 8 9(rasQ~dunng comsl,on

lv.:.....O -· -~ I
! m~
t-----------------~
Then finally,
In case of 8 = 45°,
mv+Mv 1 2mv
= ---~
vCM = - - toward left vcos8 = v 0 sin8
M+m M+m i.e., velocity ofA and Bare same
When B reaches the highest point on W
Initially velocity of B w.r.t.
Velocity of B w.r. t. W = 0
A= O-v 0 cos8
conserving momentum
Final velocity ofBw.r.t. A = v 0 cos8 - 0i.e., direction is
2mv .
(M +ml---= M x v'+mv' reversed.
M+m
19. (a, e)
'
V·=--- 2mv
M+m Centre of mass of the system will move only in vertical
direction due to effect of gravity force. When string
16. (a, e) becomes vertical, by conservation of horizontal
VII . of'msect w.r.t. stnp
e oc1ty .. = -I momentum
t
Let strip moves with speed v
Initial momentum was 0
20. (b)
~ 0= mU+v )+Mv In the figure v 12 =
-,

v = - ml/t i.e., ml/t toward left velocity of ball w.r. t.


. M+m M+m wedge before collision
-,
velocity of insect w.r. t. ground and v 12 = velocity-of
l lml/t l ball w.r.t wedge after I
=-+v=-----<-
t t M+m t collision, which must L .'.
17. (a, e) be in vertically upward
The impulsive force N passes through the centre of A direction as shown. In elastic collision ii;2 and ;;\ 2 will
as it is a head on elastic collision so it's torque ~bout make equal angle (say a) with the normal to the plane.
centre O = 0. We can show that a = 30°·
~,~--~-.,- v=·o-.--
i ~u 0· LMON=30°
1
I.
V1
Now -=tan30°=-
...J3
e-N!J
A . _B _· I V2 .

lN-W
, ___ A__ B ____
21. (a, e, d)
Conserving momentum in direction of AB

~ angular velocity of A will not change so as in case ::Ne - ,. 'j


of B also
Now velocity of A and B will be interchanged (head) i~~
.
,_ -----".....·----
on elastic collision berween same masses) ..'!:.u_cos300== mvA +mvn(alongAB)
18. (a, b, e, d) · \\ v 8 c~ong'ABJ = v A since string is tight
~ If collision-is head on and elastic then velocity will
be interchanged i.e, 4,,wIJ come ti> rest. ~
' VA
u../3
=4 J
_i.-··
~ In case of oblique collision then A and B will
u../3
separate at right angles. , Also v B(a!ong AB) = 4

www.puucho.com
Anurag Mishra Mechanics 1 with www.puucho.com

·' ,. -•,, .:..'_., _ _


lto.t':..:1~~~-:-'--~--~--'---~"-'-----'--'--____.,__;-.:..· ' _ _ _...,:...;___ MEC~ICS-Jl
_.__:._..,:..___,__.
Conserving momentum perpendicular to AB after collision of ball'B with wall
u u direction of velocity is
mvB(lABl = m
2 ~ vB(lABl = 2 interchanged only finally after
22. (a, b, d) all collision
Momentum of the r-.: . mv '1 +2m'
v2 = 2mx-u
whole system is
,
1 ~ - ·..
~.E.=~
-~
. 2
conserved in , ' v]. +2v2 = u ... (iii)
horizontal direction. ' ', .. - v]. ~v2 l
. . - ' Also ~~~=--
Let ball moves With u
0--
2
velocity' u' w.r.t. wedge from momentum conservation 2
O'= -2mv + m(u - v)
~ :.. (iv).
~ m(u-v)=+2mv
u = 3v ..... (i) From (iii) and (iv),
From energy conservation ' u
Vz =4,Vl
I
=2'U.
.! x 2mv + .! x m(u -v) 2 .= mgR
2
2 · - 2 · · i.e., .
~ mgR = mv + 2mv' (as u
2
= 3v)
25. (a, b, e)
V = {gR
V3 . ' t),
~-- _·-. ,. L·-
. "(gR
~ . u=3v~ ' ', • !'

' .

Veloci~ of ball w.i::t ground= u _:_ v = ~ (gR l. ,. ·.c, -


. "1J3- l_V _v . YJ!h:a·
23. (a; b, e) ,
v .L = v O sin e does not suffer case = .J3 ~ e = 30° by conserving energy
change due to collision. 2 .
So time . of flight and .! x mv 2 =mg x l Gust after before experiencing jerk)
maximum height which 2 2 .
2
· depend .. on vertical velocity · v = gl
does not change. .d
V = ._Jgf.
-Vu = v 0 case (before collision)
Given after jerk is being applied v sin0 becomes 0
v[l(aftercollision) = evo case ~ inlpulse applied by string = mv sine
time taken to reach ground after collision
, 2v sine L
= 0.1 X ./if. X .!
t= 0 . - - - - 2
g Vo Cose
= 0.1 ./if. = .Ji[
d = ev 0 case xt';,, e(R-L) 2 20
24. (a, d) Velocity of ball after experiencing jerk
For 1 ;, collision between 2 balls. . . .J3 .J3i[
case= .Jg[ X -

~~jv;
V = --
-+V1 2 2
r:::-... 26. (a, b, e)
'.I_'.'.'./

after collision

v 2 -v 1
0-U
mv, + 2mv 2 = mu
1
=-- ~ Vz-Vi=-
2
u
2
... (i)
... (ii)
Let velocity of M be v 1
Velocity of m be v 2
Conserving momentum
Mv 1 = mv 2
~ = m ratio of velocities
l
J.m···,·
~-' , ,.::__,' ,,,

.... (i)

From (i) and (ii'), v 2 = ~. v1 = 0 ·v 2 M


2

www.puucho.com
Anurag Mishra Mechanics 1 with www.puucho.com

Also conserving energy 2. (d)


1 2 1 2
-mv 1 +-mv 2 = mgL .... (ii) After collision v A = 8, vB = 0
2 2
~ string will be loose
Solving (i) and (ii),
So
v-ml 2gL ·and 3. (c)
, - VM(m+M) v~
v2 =M~
String will be tight when
27. (a, c) distance traveled by both
If the system is isolated .(no external forces), p and E the blocks are equal.
are conserved. Electrostatic forces are internal forces. ~
- - - - -~---- -'

To fix Y, external forces must act on the system, viz., 1 2 1 2


-gt =Bxt--gt
on Y. In the case, p is not conserved. However, these 2 2
external forces do no work, as there is no displacement gt
2
= St
of Y. E is conserved. 8 4 '
29. (b, c) t = - = - seconds.
g 5
Let u = speed of A before impact. Thus, p = mu
4. (a)
Let Vi, v 2 = speeds of A and B after impact. For any general velocity v O of block A. String will be
u = v 1 + v 2 and v 1 -v 2 = -eu tight when
1 1 1 2 1 2 Vo
v 1 = -u(l- e) and v 2 = -u(l + e) -gt =v 0t--gt ~ t = -
2 2 2 2 g

J=mv 2 =m[½u(l+e)]=½p(l+e) This is the time at which


5. (b)
Ill,, Level;~:
II ;: ~o.;p;Jjie~~ton
m'±tbt
::r
Basl!d.~fuhl;~~~>::s: ·
mtt?t1tWtttd1:tt,'m:ir:t:icm1:•r:~~
The height reached by the blockA before string is tight
= 8 X_i -
_!. 10 X X(.i)2
= 32 - 16 = 16 m
Passage-1 5 2 5 5 5 5
1. (a) . . . h t vA =-,a=--
aft erstrmg1sug 8 g
Momentum of each particle does not chan_ge in 3 3
magnitude. Its direction gets reversed. _!!_ft = o ~ t
3 3
=~g = .i5 sec
2. (d)
Both particle move with same velocity after perfectly distance covered by block A after string is tight is
inelastic collision, which is equal to velotity of CM. So 2
with respect to CM its. velocity is zero. h2 = _!! X_i_,!_ XlQ X(.i)
3· 5 2 3 5
3. (d)
'32 16 '16
Momentum ofeach particle before collision should be =---=-
same. Intial 15 15 15
Total height reached by block
v CM - 2x4+0
--
8 / s. "I
-m . ofB wit
ve oc1ty "h respect
·_
3 3 A = 16 + 16 = 64 m
. 8 5 15 15
to vCM = -3 mis
.
leftwards.
6. (b)
4. (a) - Just before string become tight
---
@J·iu,~gx4 -~~~0
5. (c)
The momentum will be reversed in direction ·but no
change in magnitude.
Passage-2 4
1. (b) u 1 = gt- =·10 x- = 8 m/sec
5
v 8 = 0 since it does not suffer any change in Uz = 8-gt = 0
momentum.
let after string become tight B moves _down with
For A and ball momentum is conserved velocity v and therefore A will move upward with
lx16'= (l+l)v same velocity 'v' .(as string is tight)
·v A = 8 m/sec upwards Just after collision
www.puucho.com
Anurag Mishra Mechanics 1 with www.puucho.com

·MECHANlC~~!j
- ., after sometime v 8 = 0 and then it increases vertically
'..I il.Pi,U·ul·s~: ·. 1·filpulse;
Bt.

... A
' .
·t,; 0

1
'
upwards with acceleration g/3
Passage-3
• ·m _____ ·_2m·,- · -~
1. {c)
Impulse applied on B Fdt = m(v - 8) -J By conservation of momentum:
Impulse applied on A Fdt 2m.(v - OJ +f = 2m x 9 = 2mvA + mv 8 => 2vA +vii= 18 .... (i)
- =>· -m(v - 8) = +2mv => 3mv = m.x 8
8 ' Also· ·-1~Vs-VA=>Vs-VA=,9
·v = - m/sec 0-9 .
3 2u 8 - 2v A = 18 , .... (ii)
7. (a) From (i) and (ii)
. After string become tight 3v 8 =:2x18=>v 8 =12m/sandvA =v 8 -9=3m/s
both block will have same
2. {c)
acceleration.
2mg-·T= 2ma For blockB: 0 =: 12 - 3 x t => t = 4sec
1 2 '' ..
T-mg=ma Distance traveled = 12 x 4- - x 3 x 4 = 48- 24
' 2
mg= 3ma
= 24m
a = g /3 of both the blocks
24
8. (c) So => t=-=8sec
3
Change in energy = AKE 3. {d)
' ' 2 0
4. (c)
= .!_ X (2~ + m) X (_i!) - .!_ X (2m) X 0 2 - .!_ X m X 8 2 As discussed in part (1), velocity of blocks after
2 3 2 2 collision will v A and 4v A where v A is the velocity of A
. 1 64 1 after collision. ·
= -x3mx---x 64m
2 9 2 So time after which block B will comes to rest
_ 64 . 64 _ 32 _ = _ X 32 __ 64 J 4VA
32 2 0=4VA-µgt=>t=-
6 2 3 3 3 , µg
. energy = -64J
1ass m 2
' 3 Distance traveled by 4v A 1 · - (4v A )
B=4vAx µg - µg µg
9. {a) 2
From O- ~ sec. block B is freely falling unde; gravity 16vi
------=--=--
16vi l6vi 8vi
5 µg µg
=> v=gt .... (i) . Time taken by A to collide with B
at t = ~ sec velocity of block· B is cl1anging from 8vi
5 ' mg 8~A 8vA
8 =--=--=--
8 m/sec to -m/sec (VA) mg 3
3
Also, after each collision velocity of A will reduced to
after v 8 = _i! m/sec downward it ,decreases with one-third of its velocity before collision

:~.:iE~i.71
3
So total time
8 v0 8 v0 8 v0 8 v0
(· /' 'i-~~3 :·:·.: ,; ,';\
I' ' .. · -4/5 · · ·,
a'. ·:
, "
=--+-.-+--+-.-+
3 3 3 9 3 27 3 81
.......

· (so infinite collisions are possible)


',.
- ,
t·(sec)
... -- -~-+ ·
'··----·'
2
1
a= g/3 m/s = 10/3 Total time = _i! _l:Q_ (1 + .!. + .!. +... ) = 8 [ -.]
2 3 3 9 1-.!.
l's =_i!_f(t-~)=_i!_lO(t-~) 3
3 3 5 3 3 5 3
= 8 x ~ = 12sec.
8 2
v 8 =O=>t=-
5 5. (b)

www.puucho.com
r
Anurag Mishra Mechanics 1 with www.puucho.com

[iM_l'ULSE~§D MOMENTUM i, J,
,. . ·,;,"'··;.,_•~-_,___...,..._ _c_c·;~::.Li::.~~-~-'-~-·-·.: -· . ~ __ ,
6. (a) Passage-11
As we have discussed, the block B will come to rest 1. (a)
after t = 4 sec so for t < 4 sec. Physically possible explosions are those in which both
Distance= S8 -SA = 12t-½ x3t 2 -3t = 9t-l.5t 2 particles move in opposite directions. i.e. signs of
velocities are opposite.
Distance between will be max. where v A = v 8 ~ ri IV andV
=> 3=12-3t=>t=·3sec 2. (b)
For t > 4sec If mA = m 8 , magnitude of velocities are same
Distance = Distance at t = 4sec- 3t = 12- 3t => II
(D 4 = 9x4-l.Sx 16 = 36-24= 12m) If ~A > mn => VA < Vs => V
After next 4 sec i.e., t = B sec _they will ~ollide again. IfmA_<m8 =>vA >v 8 =>IV
Passage4 3. (d)
1. (b) As in VI momentum is riot conserved.
At time t: x 1 = (3+3t) and x 2 = 0 :· _ => Force must be acting on it.
y 2 = (9+6t) and y 1 = 0 Passage-12
This position of their centre of mass will be Mx1 = mx2
m1X1 + m2X2 Mh
Xe= . = (1 +t ) ..... (i) - = X 2 = (1-h)
m
.m1 +m2 - .
1. (b)
y, = m1Y1 + m2Y2 ( 6 + 4t) ..... (ii)
· mi "i;"m2 Woman assends by (1- h) => His PE riser.
by mg (I- It)
Eliminating t from eqs. (i) and (ii); we have
2. (a)
Yc, =4x+2-·
C
Balloon descends by h => Its PE decreases by
So, the centre of mass is moving along the .line:
Mgh = mg(l - h)
y = 4x+2
2, (e) 3. (b) Mh = m(I - h) => ~ =~-
M l-h
First particle will stop after time Passage-13
V1 3
t 1 = - = - - - = 1.5s 1. (b)
µg 0.2X 10 m1a1 + m2a2 mxO+mx3g 3g
acm=~~-~~
Second particle will stop after time m1 +m2 m1 +m 2 2
V2 6 -
t 2 =-=---=3.0s 2. (b)
µg 0.2x 10 -""'!
By COE, I '
Hence, the centre of mass will stop ·when both the ----. lo,..__ i

particles will stop i.e., after 3 s. ]:_k(3mg)2 = .!:.mv2 Oms ~v 1

2· k 2
3. (d)
Mass 1 kg will stop at: .
v? ·. 9 '
v = ~9m/2 = 3gR
X1 = 3+ 2µg = 3+, = 5.25m
4 v=mxO+mv=.':'_=3g [m
X2 =0 m+m
cm 2 2 Vk
Mass 2 kg will stop at: 3. (a) By COE in C-frame,
2
v 36
y 2 = 9 +2- = 9+·- = 18m 1 2 1 2
-µVref = -kx
2µg 4 2 2
Y1 = 0 .
Therefore, co-ordinates of centre of mas~ will be: - 2
- ½;(3gtr =½kx
x, -- ~~-~-,-
m1 X1 + m2X2 - 175
. m
m1 +m2
and y' = m1Yi + m2Y 2 = 12 m
m1 + m2 .
Co-ordinates of centre of mass will be (1. 75 m, 12 m)
www.puucho.com
Anurag Mishra Mechanics 1 with www.puucho.com

··r
r420
[_
.. _ ___c-c___ _ _ _ _ - - - ' - - -

+ m2V2 (1) (10) + (2) (-5) = 0


~-,~~tchi~~:ry.,e Prob'i;~ 5. VcM
- m1v1
-~~-~~
m1 +m2 3

2. Similarly, PcM ,,; 0


- m1V1 + m2V2 Net force on the systemjs iero hence, VcM and PcM
vCM - ~~-~= l m/s,
m1 + m2 will remain constant. .
During maximum compression also, velocity of Velocity of 1 kg and 2 kg blocks keep no changing.
combined mass is 1 rtl/s initially and finally both of them stop as vcM was zero.,
--+ --+--+--+--+ ,.,.
Now, _- Umax =K;-KJ 6. FcM = F1 + F2 = m1 a 1 + m 2 -a 2 = (2i + Bj)
· = .!. x 2x (2) 2 -.!.x 4x (1) 2 = 2J _,
2 , ,. 2 , · IFcMI= ./4+ 64 = ../68 unit
From

_We have, m1 + m2
3. = (ll c4 iJ + (2J c4]J = _4_i+_s_,,_J
------ --- -- - - 3 3
r;,.
Ii, r;,.1 • _'_P, K__. ,,;:;,.2m •
\.~~
i.,.__..;;,.., •
',:"J =>
-
\1r::':P . (?n}--+P,:
1•
·--- ! Iv'cM I= .!. ./~1~6-+~6-4 = _.Jso_o unit
· 3 3
_P1 +P2 =P ... (i) _, r2=> •
S1 = Jovl dt =(4i)
Further, K1+K2=K
.or
p2 p2
_1_+i=-
2m 4m 2m
p2 _, r2..., dt =
S2 = Jo V2
(s')
3 J

or 2P,2 +Pi= 2P2


Solving.these two equations we get,
... (ii) . (83j.·)
(1)(4i)+(2)

4
.P2 =-P
3 .
p K
and P1=3' K1=9
BK
and K2·=-
9
7. Apply conservation oflinear momentum in horizontal
p2
4. (A) K=- direction and conservation of mechanical energy.
:- ~- "'_. - ---- ··-: - : ----· 1
2m
K'= (3P)2 =
2m
9(~)2m
= 9K
9. (A) ,60 (1-v)i i--v 1
_,..60 (1-v)I

P; =PP=> 0=60 (l+v)+(80+20)v


.
=>v= -~8
% increase in K = 800%
(BJ p = .Jzfm is opposite to 2m/svelocity of Ram i.e. ~m/s towards
P= .j2(4K)m = 2.J2km = 2P 8
rigid.
:. % increase in P = 100%
p2 1
(B) 0=80(1 +v)+(60+20)v =>v=--m/s
(CJ K=- 2
2m
(C) 80(1 +v) +60(-1 +v) +20v=0
For small percentage changes,
80--60+800+60v+20v=0
% increase in K = 2 (% increase in P) = 2% 1
I -20=160v; v = --m/s
1)) P = .J2km = (2k~) 2 . ' 8
· 80
(D) After jump of Ram Dli3-- 3/8
o/o increase in P= .!. (% increase in K) 0.5%
2 now (80+ 20)~ = 80 (l+v)+20 v.
8
17
v=--m/s
40
www.puucho.com
Anurag Mishra Mechanics 1 with www.puucho.com

'
'
\

RIGID BODY MOTION,

What is a Rigid Body?


A rigid body is a body within which the distance
between any two points does not change irrespective of the
motion. of the body. We assume that a real object behaves as

fi!J
a rigid body even though no perfectly rigid body can exist. In
..
this chapter we will encounter (i) fixed axis rotation, for ..
example, hands of a clock, blades of a fan, hinged door, (ii)
A
combination of translation and rotation, for example, a
rolling wheel, a spinning top. Rotation Rectilinear translation curvilinear translation
Many times we will draw a parallelism between (a) (b) (c)
mathematical description of fixed axis motion and Fig. 5.1
one-dimensional motion. Sometimes we will draw analogies
for better description of rotational motion. During the 2. Rotation
course of this chapter you will learn that physics of A rigid body is said to be in pure rotation if every
rotational motion is analogous to physics of translation, for particle of the body moves in a circle and the centres of all
example, new concepts of moment of inertia and torque are the circles lie on a straight lines called the axis of rotation.
analogous to mass and forces. An equivalent Newton's All points in the body which are perpendicular to the axis of
second law of motion and energy and conservation laws will rotation n1rn through the same angle in the same time
be encountered. interval.

GENERAL RIGID BODY MOTION A ••


..
1. Translation ······...
If a body is moving such that a line drawn between any ···...
two of its internal points remains parallel to itself, the ~ !
motion is said to be translation. In translation all the B' : i
particles of the body move along parallel paths. If these _;•
/
parallel paths are straight lines, the motion is said to be a B ··--··
rectilinear translation; if the paths are curved lines, the
motion is a curvilinear translation. For a rigid body in
Fig. 5.2
translation, all the points of the body have the same velocity
and the same acceleration at any given instant.

www.puucho.com
Anurag Mishra Mechanics 1 with www.puucho.com

422 MECHANICS-f I
------ ---------------- ---- - . -- '
l-

' m .-. - - - -
: __ _: The system
.. .. ·s2
-. t,;,."-::·' ---------------~~ --·· c)0sun
:A1
..: D2
811
.:'' •
'
: Cz .~ : + ••
--~!.:::·........... -~----·· Elliptical orbital motion
c, o, of m about 0, such as a
planet orbiting the Sun.
(a) (b)
Fig. 5,5
Curvilinear translation Rotation
Fig. 5.3

If the line joining any two internal points does not *"'o
_.-----.~m
The system"... __ ••
remain parallel to itself, the body is in rotation.
3. Plane motion
A general plane motion is a combination of translation
Sun*
and rotation. Pure rolling is one of the example of plane
motion. Consider a disc rolling on horizontal surface, with a
(a) Straight~line,orbital motion
painted radial line. The final position may be obtained by of m abOUt 0, ·sllch as a ,
(b) Parabolic and hyperbolic
orbital motion of m about 0,
translati_ng the disc and then rotating it about A._ _ , soccer baH.passirig a goalpost. such as a non-periodic
' .
comet about the Sun.

.,..-··1B1 ___0 · ' '


Fig. 5.~
( \ A2
' '
\ A1 :'
. ' KINEMATICS OF FIXED AXIS ROTATION
·· •... ___ .•. • B2
rewr 'IM'l7iltWi...aE,tns: raiwil Angular Variables
Plane motior.

Translation with A + Rotation aboi.Jt A


Fig. 5.4
(a) (b)

Rotation about Centre of Mass ,_ Flg.,5.7

Imagine a rod kept on a smooth horizontal table. A Consider a bicycle wheel as shown in Fig. 5.7. When it
sudden impulse is applied at its end by striking it. The object rotates from some initial position, specified by 01 to some
will rotate. About what point does it rotate? final position 0 2 ; its angular displacement is A0=0 2 -0 1 • The
angular velocity is analogous to linear velocity; instead of
Concept: An unconstrained object, i.e., not pivoted or, linear displacement, we use the term angular displacement.
hinged, rotates about its centre of mass.
The magnitude of the average angular velocity is defined as
The word spin is used to describe rotational motion of a ro = A0 ... (1)
system about an axis through its centre of mass. M
If the centre of mass of the system is moving in space in The magnitude of the instantaneous angular
a partkular reference frame,we say that the centre of mass is velocity is the limit of this ratio as At approaches zero.
executing orbital motion. . A0 de
ro= lim-=- ... (2)
,lt->DM dt
www.puucho.com
Anurag Mishra Mechanics 1 with www.puucho.com

'
. ''

speeding:up , , $lpwing,down
i ~ · SenSe of angula;· acceleraticin
1

I arc
.,_~.ro<O·:':_ ~-
I
arc
length
length
I ;:~_-, . - ~
I • S2;::·r2 A82 _
I S1=r1 A8 1 I
i ;·': ..·~pe.eding up.. ·• , . Slowing down ,
1- I ,· ,
1,,.. ·-· ~- ... " -· - -
~
. , Fig. ~.9
,_ --- -·-- "

l axle- ,,,, Consider two points A and Bon a rotating body as in Fig.
5.10. In a given time interval,, relative to A, the line AB
I
I
,.
Fig. 5;8 •
rotates counterclockwise through the angle A0 to AB'.
Relative to B' the line BA turns counterclockwise through A0
L ·-· .. -· to BA'. The angular displacement, and hence also the
Angular acceleration (denoted by a., alpha) is analogous angular velocity, is the same about A and B or about any
to linear acceleration and is defined as the change in-angular other point on the body.
velocity divided by the time required during this change.
As a rigid body rotates about the fixed axis through 0,
The average angular acceleration is defined as
the point P moves in a circle. The linear velocity vector v is
- "'2 -ro1
a=~~~=
Aro ... (3) always tangent to the circular path, hence it is referred to as
M M tangential velocity. When the body turns through an angle 0,
Instantaneous angular acceleration is defined as the the distance travelled by this point measured along the
limit of this ratio as At approaches zero. circular path is s = re, and noting that r is constant, we have
. oro dro

·1;,•·
a.= lun - = - ... (4) r·. - ..-......... . -~·· ,--- 1 ·- ·-
<iHO At dt l
r ·- ·-··---· -·-- - ...... ·.... _.. --..... ·-.,·-~ . I

f
B .j:
Concept: Since angular velocity ro is the' same 1,or aU

f
:
'poin·ts. ofa_ ·.r.otating body,. eqn. (4.) t.ell us tha.t ~ a,!,so w.il·l be
1 /"~ ·:.:
[the same for the points. Thus ro and a are properties of'the Ad I•
'I

Jrotating body as_a.whole. ___ · ·__ ,·. __ · .. ··-···- ·_·.•~ '·· . A' ,,,; 1:
< i ,· -~--1<:::....--.t....:.......
Angular velocity ro is measured in radians per second; a. l-l, . .(a) • ~ ·.·.• '. , ~. (b)', , ' .: '
I I X
will be expressed as radians per second squared (rad/s 2 ). ·m,e:angula_[ Veltj_city of a body tot_atlng : i :
Radians .can be related to degrees in the following way. -~bou(an:'axisJixe·d in dir13;c_tioll 'iS th6 ;! !
',same.:about:'a_ny·point-'in.the'body. , i .}
In a complete circle there are 360°, which correspond to an . . fiFlg. 5.10 ', ' 'i i. ~ig. 5.11'.
r •""' --- ~~
arc length equal to the circumference of the circle, 1=21tr. ~--ft~~'-"'-•·+• + -- • , ... ' -~M --·-

Thus 0 = 1/R = 21tR/R = 21t radians in a complete circle; so ds de


v =- =r - = rro
360° = 21t radians dt dt
One radian is therefore The equation shows that the linear speed of a point on
the rotating object increases as one moves outward from tJ-,e
360°/27t ~ 360°/6,28 ~ 57.3°
I .• , -. . ., • centre of rotation.
I ,Concept: 1. The angular velpcity,of a rotating body, is Fig. 5.12 shows a wheel . rotating uniformly
ithe same relative to any-point on it. . ' , .. counterclockwise. Two representative points on the wheel,
1 ' , " -
; 2. If sense bf angular velocit_x.and angular acc.eleratic1n i.s at distances R 1 and R 2 from the centre, have different linear
!same, the rigid body is speeding up .and if they have opposite velocities because they trace. different arc lengths in the
lsense, the body is slowing.down (see Fig. 5.9). same time interval. Since R 2 > R1 then v 2 > v 1 as .v = Rro.
'-::--~------- ---- --- .- - -- --··--· . ..,._,__ ----~ But the two points have the same angular velocity ro because

www.puucho.com
Anurag Mishra Mechanics 1 with www.puucho.com

they travel through the same angle 0 in the same time I


interval.

r~f ,' -.
I •
I -.
(t·
t~·1 ~
As a is a- vector having a radial and a tangential ,
-

component, the magnitude of a for the_ point p on the


f rotating object is ~~- -

~-
,: ,<.. : Fig:5112'
II::'--~---~:, __
\:.;.·:--'----"
.

Fig. 5.13 shows the· distinction between r (the position


vector) and R ( the distance from the rotation axis) for a
point P on the edge of a cylinder rotating about the z-axis.

·, z : l
.;~." -~------r-~_ . . ::::::::
''):+ ________ _
. '
I
y ·--
.. .. ·· .:
,,., ::::'--:···--·+···----......
X

On a rotating wheel whose rotation speed is increasing,


~, :.. i ·: ... F/g,d:13 .,. ·. a point P has both tangential and radial (centripetal)
components of accel~ration.
Acceleration o.f a Point on a Rotating Object
:~~~:~~~: ~otion with Constant Angular
The · tangential acceleration can be obtained by 0
differentiating the expression of v w.r.t; time .. From the definition of angular acceleration, we have
dv dro dro
a, = dt = rdt = ra a=-
dt
· The tangential .acceleration. arises when rotation rate or dro = adt
increases or decreases. If angular-velocity is increasing, a, is
parallel to v, and antiparallel if v·is decreasing.
or 1
'J'"
dro = a_dt
Cl>j
J''
t;

A point moving along_ a circuJar path.has a centripetal


also referred radial, or normal acceleration of magnitude
v2/r ·dire~ted. towards _centre of rotation. Centripetal
a. or
as
ro 1 =ro;+at
<!0 =rodt
-... (1)

d0 = J'1 (ro; +at)dt


81
acceleration apses due to change ii:, direction of velocity. The
or JOi ti
magnitude of centripetal acceleration is expres~ed as ' 1 -2
. v2 • . or 81 =8-+ro-t+-at
I I .:.(2)
an=-.=. • (02
r 2
r · If we eliminate t from eqns. (1) and (2), we have
- 2 2 '
The total linear acceleration vector or'the point P is ro1 = ro, + 2.a(8 1 ~ a,)
as Note chat the kinematic expressions for rotational
motion under constant angular acceleration have the same
form as chose for linear motion - under constant linear
acceleration.
www.puucho.com
Anurag Mishra Mechanics 1 with www.puucho.com

Make the substitution


x----40, v----4ro and a----4a
1n(~)
ro
= ln(e-t/Tl = _!_
0 T
TABLE 5.1: Comparison of Linear and Rotational
Equations for Constant Acceleration Model Solving for t, we find = 1n(: t -T
0
)

ro1 ~roi+at 0
or t=+Tln(: )
1 2
0/ =0-+cM+l_at~' xr=.x(+vit +·2at
2
Because ro = ro 0 /2, the value oft is
.' - I

2
"'1
2
=ro, + 2acer- 8,J . .: 2 ' 2
vf =VT+ "2n(x1-x;)
. . t = T ln(2)
(b) dro d (
a( t)= - = - r o 0e -,;r) =--e ro 0 -t/T
TABLE 5.2 : Analogy between Rotational Kinematics dt dt T
and Straight Line Kinematics The original acceleration a 0 at t = 0 is
t ·._·. \ µiiear \ (Unit)[;,Jc{#gular \ ,(Yriit)~frj OOo -o;r
a 0 =--e =--
C.Oo

(rad)
! T T
Position X ' (m) 0
' T\te negative value of angular acceleration
' (m) L'.0 (rad) I
i Displace-
ill( indicates that the angular acceleration is in the
I
ment clockwise sense. In this example the flywheel is
! dx de rotating anticlockwise (ro is positive), the flywheel
Velocity v:;- (mis) roes- (rad/s)
: dt dt '
'I ~s slowing down. _______________________ _
' Acceleration dv (m/s2) , dro (rad/s2), ; <;oncept: A negative angular acceleration does not\
a:- a "'Tt
dt ;always mean the object is slowing down. When Oland a have,
' ,op_p()s~sl~1 ~he__"o~ation rate_iscI,e_creasi~: _ _ _ _ _j
v=fadt+c ro= fadt+c
Liro rof -Olo
x= fvdt+c' 0=frodt+c' (c) aav = - = ~ - -
M tf - t 0
~&~~ir:riei~~Gl> ;;;;
(ro 0 /2)- ro 0 0
=--
ro
tf -0 .2tf
·A gyroscope is spinning at angular velocity ro 0• The power is i
;shut off, and the gyroscope begins to slow down with its i J
(d) 0(t) = Jro(t)dt + C = ro 0 e-<IT dt + C
·angular velocity described by i = -----O)oTe-,fr + C
\ co = ro oe_-r;r ;
with Ta constant. (a) How long does it take for the gyroscope: We let 0 = 0 at t = 0, so that
0

to reach half its initial angular velocity? (b) What is the: 0 = -----O)oT+C
'gyroscope's initial angular acceleration? (c) What is thel leading to C = +ro 0T. Consequently; the angular position
:gyroscope's average acceleration between the time the power! function is
;is cut off and the instant it reaches half speed? (d) Throilghf 0(t) = ro 0T(l- e-t/T)
how many revolutions does the gyroscope tum during this!
1
time? We next evaluate O(t) at the time the gyroscope's angular
velocity reaches half its initial value.
Solution: Problem Solving Strategy: (Tln2) ·
Step 1: The angular velocity of the gyroscope is given 0 = ro 0T(l- e- T ) = ro 0T(1-½).
as function of time.
Rearrange the equation and take natural logarithms of Vector Representation of Rotational
both sides. Quantities,
Step 2: Angular acceleration function is time derivative The right hand rule: Fig. 5.16 (a) depicts a set of
of angular velocity function. coordinate axes fixed in a spinning top. The z-axis coincides
,. LlOl with the rotation axis, and x and y-axes turn with the top.
Step 3: Average angu1ar acce1erat10n 1s aav = - .
M The three axes depicted constitute a right handed
Step 4: The angular displacement is integral of angular coordinate system. Fig. 5.16(b) shows a right hand; the
velocity coordinate system is attached with the hand in such a way
0(t)= fro(t)dt+C that the sense of rotation from i to j is the sense in which the
(a) Take the natural logarithm of both fingers curl, and the thumb points along k.
sides,leading to

www.puucho.com
Anurag Mishra Mechanics 1 with www.puucho.com

'426 ---------------------~·-.•,:'.,.,.----
I '
When the circular orbit does not contain the origin of
r the coordinate system as in Fig. 5.18 Cb), the magnitude ofoo
z z
is given by v,/R, where R is the radius of the circle as
measured in the plane of this circle. From the figure it. i~
clear that R = r sin q,, so that when we look at the magnitude
of Vt implied by our cross product definition Vt = 00 X We r,
obtain v, = roo sin q, = Roo, the correct relationship between
omega and tangential velocity.
(a) (b) (c)
1 Consider a particle moving in a circle of constant radius •
, •• • Fig. 5.16
i_ _ _ _ _ _ _ _ _ - - · - - - - -- ._ - - - - ·--------·-------~J but not necessarily changing speed. By differentiating eqn.
An alternative way of visualizing a right-handed (1) with respect to time we can find an expression for the
coordinate system is represented in Fig. 5.16(c). A acceleration. When you take the derivative of a vector cross
coordinate system is embedded in a right-handed screw. The product, use the product rule from calculus and be careful to
x- and y-axes lie in the head, and -the z-axis lies along the preserve the order of the terms.
shank with k directed from the head tow~d the point. d ---+ d __, ---+
-(v )=-(ooxrJ
Turning the screw so that i rotates toward j results in the dt ' dt
advance of the screw into the wood, in the positive -+ dffi
a = - x r+ oo x -
-+ -+ di
... (2)
z-direction. If you grab the screw and curl your fingers and dt dt
tum the screw in the sense of -the curl, then the screw will In the case we are considering, the magnitude of r is
advance in the direction of z-axis. constant so that drd_ = v,, the tangential velocity. Therefore,
When you wrap your right hand -t
around a rotating object in the direction we may :write
of rotation, your extended thumb -+-+-+-+"-+
a= ax r+oo xv, ... (3)
points in the direction of angular
where we have defined the angular acceleration vector a
· velocity.
as doo. For the case of a particle travelling in ~ circle of
The relation between the tangential
linear velocity v·, and oo is
'
) *
constant radius, oo may change · in magnitude but not
.
vt = ror
where r is the radius of the circular
_Fig. 5.17, d direction; consequently, a is either parallel to oo when the
particle in increasing speed o, antiparallel to oo when the
motion. If we choose the direction of oo perpendicular to particle is slowing down. The first term in eqn. (2) is tangent
both'r and v, in the sense shown in Fig. 5.17, we can write --- -- -------
it !
... (1) rt
---->
i
•I
0).
,I

Ol

(a) . Jb),
Fig. 5;19
X X - -----·--·----- ----- -----~...-.1
(b) "' I
(a) to the circular motion, and thus, we identify it with the
. __ ,_ "--------- __ Flg .. 5.18 _ ---- -·- -- . __J tangential acceleration a,. Similarly, the second term in eqn.
Note carefully .that the origin of vector must be along r (2) is directed radially inward and is centripetal
acceleration.
same axis that passes through the centre of the particle's
circular motion. Furthermore, that axis must be normal to
the plane of the circle. In Fig. 5.18(a), we see that oo is a
_vector perpendicular to the plane of_ the circular motion.
That is, eqn: (1) provides the correct direction for v when oo ·
is along the indicated direction.

www.puucho.com
Anurag Mishra Mechanics 1 with www.puucho.com

RIGID BODY MOTION 427,


rF sin 0 is magnitude of torque. Torque is a vector and
expressed as
..., ..., ...,
't;rXF
To find the direction qf the torque, curl the fingers of the
..., ...,
right hand from r through the smallest angle to F. The
upwardly extended thumb then points in the direction of
(c) {d) ..., ..., ...,
't; r x F.
Fig. 5,19

A careful analysis of Fig. 5.19 (a), (b), (c) and (d)


combined with the right hand thumb rule will convince us Fig SE.2 shows three forces F1, F2 and F3 acting on a rod
the direction of various linear and angular quantities. pivoted at its end. Find the torque of each force about pivot.
Torque F31.'- F3
Fig. S.20(a) shows a bolt being opened with the help of ·,t
F2 ~F21. ,
a wrench. The amount of turn the bolt gets depends on the
magnitude of force, direction of force and point of
application of force. No turning effect is produced when
forceFpasses through centre of bolt [Fig. 5.20 (b)]. Turning
effect is increased by increasing e and r and decreased by
decreasing e and r.
: F1. =Fcos0 0
: r1.=rsin8 Fig. 5E.2
:
:e 0
Solution: In order to determine the sign of each
F.l =Fsin0 torque, imagine which way the rod would rotate if the given
Rotation force were the only force acting. Generally anticlockwise
axis direction is considered to be positive, but this choice is
F arbitrary.
(a) (b) 't;rFsine;Fxr.L ;rxF.L
---; ---; ---; In order to determine magnitude of torque we may
-r= r xF
choose the expression F x r.L or r x F.L according to
convenience, where r1. is component of moment arm
perpendicular to force and F.L is component of force
perpendicular to moment arm.
't 1 ; -r1 F1 sin(90°+e) ; -r1F1 cos0
't 2 ; +r2F2 sin(l80°-a); +r2F2 sinct
't 3 ; +r3 F3 sin(90°-q,); +r3 F3 cosq,
(c)
Fig. 5.20 Concept 1. Figs, 5.21 (a), (b) and (c) show a pivoted
rigid body. Tangential component F, produces torque, normal
Turning effect of force is described by a vector
component Fn produces zero torque as it passes through pivot. ,
torque. Position vector from point O on the axis of rotation
to the point of application of force is called moment arm.
The moment of the force or torque about O is defined as the
product of F and the moment arm.
't; Fr.L ; Fr sine
Alternatively resolve F into its components (Fj 1, parallel
to r and F.L, perpendicular to r). Since Fj1 passes through 0,
its moment arm is zero, hence its torque is zero.
't; rF.L ; rF sine
Fig. 5.21 (a)

www.puucho.com
Anurag Mishra Mechanics 1 with www.puucho.com

,----
1428
-
'----------- - ---------- ---- ----------
----- -----· - --· · - - - - - - - -·
y ~ Always measure angle ! --> --> -->
-}, ___ ---·> F .· cj) from radial line in ; 3. If force F is parallel to axis of rotation, then r x F is'
F exert a torque '--~~-c---.--.. ' anticlockwise direction 1 --> --> '
and causes .• · I perpendicular to axis. Component of r x F along the axis is
· rotation . •··· · ·Point where ) :zero.
.. / force is applied
axis of rotation axis of rotation
........··F,

Pivot point
(b)

0
-->
~ F2 .
F5 exerts zero torque
exerts maximum --.,,,---~,, -->
torque In )E--j____.F3 exerts a clockwise
anticlockwise torque
sense --> B B
F4
exerts maximum torque (b) (c)
in clockwise sense Fig. 5.22

--> -->
F, 4. If line of action of F intersects the axis, the point of,
A force applied at pivot . --> -->
exerts zero torque ' intersection is our origin. Torque about O is r x F =q
(c) ltherefore its component alo_ng axis will be zero.
axis of rotation
Fig. 5.21

'
I --+--+ --+
i 2_ The quantity r x F is torque of F about 0. When we:
·say torque about axis of rotation, it means axial component;
1 ~ --+ I
;of vector r x F along OA Torque of a force about axis OA is
!independent of the choice of origin 0, but it must lie on the
jaxis. ----- - ..
axis of rotation :

B
(d)
Fig. 5.22
-->
5. If force F is skew with the axis (line of action of F is'.
!perpendicular but non-intersecting with the axis), then torque,
-->
of F has magnitude
--> -->
l~I = IF!.(r.c)
B ~- _ ~~_ere _~.1, __ i:}_ ~om~T}t ann:
Fig. 5.22 (a)
Couple When a line of -4 · :r1
--> . pair of equal but -->
Torque of F. about 0 1 is action of}/ .F, - '
opposite forces is -4 ""7 ./
~ --+ --+ --+ --+ F1 and F2 ;
O 1 P xF=(r2 +r)xF applied at two different
a coupre :
--+ --+ --+ --+ points of a rigid body, --> -->
=r2 xF+rxF such a body rotates F1 =-F2 ;

--> --> --> without translating. This


Vector r2 x F.L r,; therefore it will not have any1 pair of forces is called a
------> --> couple. Although the Fig. 5.23
;component along AR Thus component of O 1P x F and I -'
'-> --> forces act in opposite
~-l!'- ?5__~!!_l9!1?:_ ~ is sam_~- _
www.puucho.com
Anurag Mishra Mechanics 1 with www.puucho.com

- - -- --- - --1
: RIGID BODY MOTION 429 ]
______

directions, but the torques about these forces act in the same where :Etaxi, is about the origin of an inertial reference
direction.
frame or CM of the system.
'net = r1F + r 2F = (r1 + r 2 )F = lF
. - 5. For any body rotating about an axis of symmetry
Concept: 1. If an object is unconstrained (i.e., not
: •:Et= Ia can be applied. Both a and.tare vector components:
on pivot), a couple causes the object to rotate about its centre: ·along th_e rotation @is.
of mass.
Torque and Moment of Inertia
2. A couple exerts same net torque about any point on the
Consider a force F exerted on a point mass that is at a
object. distance r from a pivot point, as shown in Fig. 5.25.
Newton's Second Law for Rotation From Newton's second law,
Fig. 5.24 shows a rigid body undergoing fixed axis F =ma= mra.
rotation. The axis of rotation may pass through CM or any
point on the other body. In all such cases it is assumed that Frictionless tabletop Hand pulling
the pivot is frictionless. with force _F,
producing
angular

rotation axis
C3
Circular path of radius r
acceleration a

normal to plane
outward

Fig. 5.25 Fig. 5.26


Fig. 5.24
__, __, __, Since F is perpendicular to r, torque is simply t = rF. So
The forces F1 , F2 , F 3 , ..... act on particles of masses if we multiply both sides of the above equation by r, we
obtain
m1 , m2 , m 3 ... and exert torques 1 1 , 't 2 , 1 3 ... about the axis.
rF = mr 2 a.
The resultant of all the torques acting on the body causes an 2
t = (mr )a
angular acceleration in the body. The net torque on. the body
is the sum of all the torques on all the mass elements of the This equation is the rotational analogue of Newton's
body. second law, where
Torque Analogous ) Force
'ti = (mirµ)ri
Angular acceleration ----> Linear acceleration
'tnet = 7'ti = 7mir?a = (r.TT!Il)a Moment of inertia - - - - > Mass
Recall the fact that every particle on the body has the The quantity mr 2 is called the rotational inertia or
same angular acceleration a. The quantity Lm;r,2 is called moment of inertia of a point mass min a distance r from
the centre of rotation.
object moment of inertia, denoted by I.
'tnet Concept: The moment of inertia depends not only on
Cl.= - -
I the mass of an object, bui also on its distribution relative to
the axis about which it rotates.
Concept: 1. Torque is the cause of angular acceleration.
For example, the harder we push on a merry-go-round,
2. Moment of inertia is rotational equivalent of mass.
the faster it accelerates, but more massive a merry-go-round,
3. Moment of inertia depends on the mass and how the; the slower it rotates. This is the effect of moment of inertia;
mass is distributed about the axis of rotation. larger the moment of inertia, smaller the angular
4. For a rigid body the component of torque along, acceleration. Similarly it will be much easier to accelerate a
rotaiion axis is merry-go-round full of children if they stand close to its axis
Ltaxis = Ia, than if they all stand at the outer edge. The mass is same but
its distribution about axis of rotation has changed. The
moment of inertia is much larger when the children are at
the edge of merry-go-round.

www.puucho.com
Anurag Mishra Mechanics 1 with www.puucho.com

'
.MECH~Nl~i1J
Net Torque
Fig. 5.27 shows a rigid
...,
li
body on which forces F1 , ~ ~, ~\'
..., ..., ..., v=coxr9 1
,.
F2 , F3 and F4 have been
i.
applied. Torque exerted by
reaction forces arising at
..;
pivot, F pivot is ·zero because
its moment arm is zero·.
The net torque exerted L ··; .. ·• .Fig. 5.21·
by all the forces is sum of
the torques due to applied forces Fig. 5.29 shows an object rotating about the z-axis of the
body coordinate system. Each particle in the body (with
coordinates XBYB,zB) undergoes circular motion with
. Gravitational Torque angular speed ~ and radius r.L = rB sin <P = ~ x~ + y ~~ .the
Fig. 5.28 shows an
arbitrary shaped rigid body
l,. .-, ;, ;:··.rpoment am,S·
'J:t-f:1-*•-'1;''¥--·--
distance or'the particle from the rotation axis. The speed of
pivoted at an axle. '"-~~~ the particle depends on its position, I~ I= ror.L, so a particle
Gravitational force acts on 1· of mass dm has kinetic. energy
every particle of the body;
force on the particles is II •:,~--i--~--A
... : ,., .
1
dK=-dm\v
2
...,
2
1 2 2
\=-dmro r.L
2 .
represented by m;g. Torque , .- . ~\Q · pivot m2g• • The total rotational kinetic energy ·is the sum of over .all
exerted by gravitational 1- .(tomue . (torqu~
l antjclo_~lse)' . clockwi~e)! such particles.
force on ith particle is
r1,\= m;g~ •..where d;. is . ·., ·•. • . flg.5.28 _
'L-,_,., ~-1
·,,_ -- '' K= L -ro 122
all particles 2
12 (I:r.L2
r.L dm = -ro
2
dm)

moment arm. Taking Although v differs for each particle, the angular speed ro
clockwise torque negative and anticlockwise torque positive, is the same for all the particles, and it can thus be factored
. we can see that torque on the particles to the right of pivot is from the sum. The sum over differential quantities for air the
'negative and on the particles to the left positive. elements is
...,
'ti =_-ximig K = .!ro 2 JrJdm
~ ~ .· 2
t net
=Lt.
i I
=L-m-x-g
i 1_ 1
=-(Lm-x-)g
t 1 The resulting integral Jr 2
dm depends only on the mass
But according to -definition of centre of mass, distribution of the rigid body around the rotation axis and is
·· .Lm;x; = MxCM · called the rotational inertia I. Note the analogy that the
t=-MgxCM rotational kinetic energy of a body is related to its angular ·
where x CM is ppsition of centre of mass relative to pivot. velocity in the same way as the mass. of a particle relates its
I Concept~ 0Th; grailitatl~n~( t6fque _on aJ:iy extended
kinetic energy to its linear velocity.
1 2
K = -Iro
lr,bject is equivaliiit' to the torqa~ of d .single force, ... (1)
2
?gravitationd.lforce, acting °"t the·.dbje'ct's centre· o] mass. We
!can treat the;oii_jecf_as if all it:5 "!ass l\f~re concentrated ati~ Rotation.al inertia's role · in rotational dynamics is
analogous to _that of mass in particle dynamics. Now we
lcc:.e:.cn:.ctr:.ce:.co,:.if_m:.cass=: ·, · •.. . , · • · ·
... -.~..::.. _,_- ...... - · "
· • •·;. , ··
_,_, =,_d_
.,. • ,.i
• - - - · - - - - - - ; ·,'
formally state a result:
Rotational Kinetic Energy and Moment of The rotational inertia of a rigid body about an axis A is ..
Inertia . · . the sum over all the particles in the body of the particle's
Different parts of a rotating body have different linear mass dm multiplied by the square of its perpendicular ·
velocities; we -imagine each body as a sum of differential distance r.L from the axis.
elements and sum the contribution from each element. J
IA= rJdni
Note that r.L will be different for different axis for the
same particle (see Fig. ·S.29). ·

www.puucho.com
Anurag Mishra Mechanics 1 with www.puucho.com
,\I

[iiGmeoovMotioti·--·· _· __ ~-------
A rigid body does not have·a single rotational inertia. In
-----------=--~~~ _
rotation.
- --- ·-. .
-- ··-------·--------------- I
- ..7
431

general, the body ha_§,a-different value or' inertia for each


possible rotation axis. ·
m1 = m2 = m3 = m4 = m
Rotational Kinetic Energy of a Collection of
r1 = r2 = r3 = r4 = a
Particles
I= ma 2 +ma 2 +ma 2 +ma 2 = 4ma 2
Consider a massless thin rod on which N point masses
are arranged in a straight line. The rod is fixed at one end
0

and free to rotate about


other end so that the . - - . ·-;; ··· -----· -7 '---- - - - - - - --- -- --·--- 1
IA weightlifter's barbell consists of two heavy discs of mass m!
masses rotate in the ~-~,,~--i I land radius R connected by a light rod of length l >> R (Fig. I
zy-plane. We denote each , i----,,~ I
' 'jSE.4). Estimate its moment of inertia about an axis At'
mass as m; and its '4--',-.j
col through its CM and perpendicular to the rod. Determine itsi
distance from the pivot )
point r;. When the rod
rotates with angular
D ffi1 ;;,2 ~3 ~4 lI
Fjg. 5.30 ______...J
ikinetic energy.
I :
_ _ __ :

velocity OJ, each mass A 'rt


moves with different linear velocity v; = ror;. The total
kinetic energy of all N masses can then be written as
NI 2 N1 22 I
:E -m-v-
K = i=I2 = :E -mm-r- I : - \17. __,
LI i=12 i l l
i :~fl--

Each mass turns through the same angle, hence the i


I----- - - - -
Fig. SE.4
-- ' - -
angular velocity is the same for each mass. Thus we have
. Solution: Since l >> R, we can model the heavy discs
K =2.!(f i,.,}
m-r-')00
I I
2 as particles for our estimate, and we ignore the mass of the
light rod. With these approximations, an error of only a few-
Moment of inertia for a collection of N mass points is per cent in the value of I is incorporated.
N 2 , There are only two particles in this system, each of mass
:E m-r-
I= i=I l I m and at a perpendicular distance 1/2 from the axis; the
l 2 rotational inertia is
So kinetic energy of rotation is K = -Im .
2
IA =l:mr12 =2m ( 1)2 = m/2
2 2
- --... - . - - .. --··· --- --- .----·--1
'Four particles of mass m are connected by massless rods to
The rotational kinetic energy is
1 2 1 2 2
K =-!Aro =-ml OJ
'form a rectangle-of sides 2a and_2b.as shown. The assembly, 2 4
'rotates about an axis in the plane of.the figure through thel Exercise. Esti:rhate the rotational inertia of the barbell about
;centre (Fig. SE.3). Fing the m9ment 9f:irye1:tia aboutthe axis., (a) an axis B through one of the discs and perpendicular to the rod
1
Axis of rotation · and (b) an axis C through the CM and at 30° to the rod.

0
T·O Moment of Inertia of a Continuous Body
A rigid body can be considered as a collection of
2b continuous mass-points. We mentally divide ·the body into
small mass elements t.m and perform the summation.

.l. o»==l==¾io5) , I 0 "':E


N
i=I
t.m-r-
l l
2

In the limit as the number of mass points increases, t.m;


approaches zero, the summation becomes integral.
,_ Fig. SE.3
2
I= r dm f
Solution: Since we are given that the objects are
particles, we apply the definition of moment of inertia for where r is the shortest straight line distance between
discrete particles. In that equation r, is the perpendicular each dm mass element and the axis of rotation.
distance from the particle of mass m; to the axis (line) of To perform _the integration the element of mass dm is
expressed in terms of the spatial coordinates x,y, z. This is
www.puucho.com
Anurag Mishra Mechanics 1 with www.puucho.com

'!,

achieved through density function. The mass distribution ·


inay be linear, areal (or superficial) or volumetric.
Linear Mass Distribution
If abody is long, thin and uniform, like a slender rod, we
treat it as if it were one-dimensional and use the mass per
'unit length. ;,, M
L
J+L/2x2dx = M
-L/2 L
[x 3
3
]+L/Z

. ' -L/2
2
=..!._ML
12
Note: The moment of inertia of a rod about x-axis. is
zero as the rod is assumed to have negligible thickness.
Moment of inertia about z-axis is ..!._ML2 because mass
12
distribution is same about Y: and z-axes.
M.I. of Uniform Hoop about a Perpendicular Axis
Areal Mass Distribution through its Centre
When the body is two-dimensional, like a rectangular k ::· :'..._··-----~-------~
--,,
': .. ~-~Y
~~-

plate,. we use the mass per unit area. · i,· '


1· . :-
----1---I
·;
.I
~y_.":_ i: --~-~=;dxdy, ·
1 :,,

[d{f~ -····, •..... X

t.
I , --.::.-
dx
I. ,
L
~-\
!
L. ~. ____.,. ,;; . . :
~1_'~~-·~;;g_~:;2_~~~-~l [~f ::~ i: :.Fi~- sia : -~--: .:' .!
· .. .
Area mass density = cr =· -
Mass
- L_;__~~:i..:.-·~-~·.:.t:,,,::,,~_i.J .
. Area Assume that a hocip has mass M and radius R. The axis
dm = crdA = (: )dA of rotation is perpendicular to the plane of the hoop and
passes through centre. All the mass elements are at a
Volu111etric Mass Distribution distance r = R from the axis. Thus the moment of inertia is·
· · I·= Jr 2 dm = R 2fdm-= MR 2
1. · When -the body is three-dimensional, we use the mass
per unit volume.
M.I. of Uniform Disc· about a Perpendicular Axis
-Mass
Volume density = p = - - - through its Centre ·
Volume. For the same mass M
:' .. dm= pdV =(;}v and radius R, moment of
inertia of disc is expected to
be smaller than MR 2
':: c~~cep.ti .'['h.~ in.t~gr~ls of~- a_~e collectively krio'w~ as/
0

because mass is distributed


mqme_nt,t~f the'.mass distnbutwn .. :. · , . · . . , ':l from r =·o to r = R rather
.:. J~-
f'ldm, the zeroth moment '(the to_tal system mass) ' than being concentrated at
·.: . ,I :~'f~qm; thefirstmoment (~ed to calcul~te tfi.fce~tr~
1
r = R as in the .case of a L·-·~--·-;. , . fig. 5.34:. ·'..~, ...., ,
-·-----~
hoop. We consider element
dfmass) ' ' . ' ' ' ' . .'
- - • I dm in the shape of a circular strip of radius r and thickness
I . ..r2. ~J r2dm, tJ;ze second m,oment crotationa1 1nerna{_j dr. Area of shaded strip is 21tr dr.

Moment of Inertia of the Thin Rod So dm = -.dA


M
A
M
= - - 21tr dr
1tR2
.. Let the length and mass of the rod be L · and M
respectively. Let the rod lie along. the x-axis with its ends at Thus we have,
the coordinates - Lj2 and + L/2 The linear mass density of
rod is;\. =M/L. We choose a mass element dm ata distance x
from the axis.
www.puucho.com
Anurag Mishra Mechanics 1 with www.puucho.com

I ·RIGID BODY MOTION 433'


#_:__......:_i

= 2M R4 = .!MR2 M.l.of Rectangular Lamina about an Axis Parallel to


R2 4 2 one Edge
M.I. of Uniform Solid Cylinder We consider a rectangular lamina of height a and width
b and uniform area density cr. We choose a rectangular mass
about its Axis -~
!·-· element dm = crdx dy. The shortest distance between dm and
We can imagine a cylinder to R y-axis is r = x. Integration over the
consist of a set of discs each of mass fy - -- ---:--· ·:
entire lamina requires two integrals.
dm and moment of inertia .! dmR 2 • I= Jr dm
2
jC...~dm=adx·dy:
2
The moment of inertia of the complete
cylinder is M
= J!J:x 2crdy dx dy.1..

' ,,
We first perform the y-integration. "!:11-
dx

.._·_F.lg. 5.35 - - l I " • X


1.. --~lg.:5:37_
where M is the total mass of the cylinder. We can choose a strip of thickness
dx at a distance r from y-axis, as shown in Fig. 5.37. So
Similarly we can imagine a thin cylindrical shell to
consist of a set of rings each of mass dm and moment of dm = cr(a dx).
inertia JR2
dm. The moment of inertia of the complete Now the x-integration ry-- -----
can be performed. t
cylinder is MR 2 • 2 '. a f-~--.,.,.~
·1=J:x cradx i
M.I. of Hollow Cylinder about its Axis.
We consider a uniform hollow cylinder of inner radius
R1 , outer radius R2 and mass M. We divide the cylinder into· !
= craf x2·dx = cr_Jx31b I ,,_,;x~ dm;;::oadx
thin concentric cylindrical hoops of thickness dr and radius r. . l3 0
L
If the density (mass per unit volume) is p, then ·= .!crab 3
dm=pdV 3
z
where dV . is the volume of a As total mass
dr
cylindrical shell of radius r, thickness dr M = crA = crab, we have
and height h. I= .!Mb 2
dV = (21tr)(dr)h 3
dm = 21tphrdr Note that this result is same as that for a thin rod of
So
length b pivoted about one end.
I= r 2 dm = JR 21tphr 3dr
f 2
Thus The rotational inertia is not affected by the distribution
R1 . .
of the mass parallel to the axis of rotation; only the distance
= Z1tph[ Ri ~Rt] of mass from the axis is important.
M.I. of Rectangular Lamina about Perpendicular Axis
We are given that the cylinder has uniform density through its Centre
p=constant, otherwise we would have to substitute p as a In Fig. 5.39, the rotation axis is z-axis and the edges of
function of r before the integration could be carried out. The the lamina are aligned parallel to the x- and y-axes. The
volume~~~ hollow cylinder is V = (ltRi - !tRf
)h; so its
mass ·element dm =crdx dy is at a distance r = ~ x 2 + y 2
inass is :
2 . 2 from the axis of rotation. The moment of inertia is
M = pV = 1t(R2 - R1 )hp r---z· 7 · •·

(Ri -Rt)= (Ri -Rt)(Ri +Rf) I , . <.


· 1tph 2, 2 2 2' I
we haye- I= -(R2 -R1 lCR2 +R1}

1 ( 2
2
2
I.
=-MR2+R1)
2
As a check we can see that for a solid cylinder R1 = 0 X
and R2 = R, we get i · r;:/x2 +y2
I d~~adxdy

L____Fi_g_. s_.3:.._9_ _ ____;

www.puucho.com
Anurag Mishra Mechanics 1 with www.puucho.com

. ,MECHANIC~,lJ
I= r dm J 2
I= J+L/2(ML x2dx)
= fb/2 Ju/2 (x2 + Y ')crdY dx -L/2

= M [x 3 ]+uz =_M_L_2
-b/2 -a/2
We perform the y-integration first, treating x as a
constant. . L 3 -L/2 12
2
12 ' 3 )a/2 ML
I = fb-b/2 x 2y + L crdx 12
l=-
( 3
-a/2
(b) Axis in the plane of the lamina passing
3
=cr b12 ( xa+--
2 a ) dx through the end and parallel to the breadth:
f
-b/2 12
Integrate the expression dJ = M x 2 dx, between the
Now we perform x-integration. L

Jb/2 =cr (b3


Iimitsx=Otcix=L.
x3 a3 a~ ) 2
I =CJ
( -a+-.
3 12 _;/2 -a,+-b
12 12
I= JLM
x2dx = ML
0£ 3
As the total mass is M = crab, we have ML2
J=-
I= _!_M(a 2 + b2 ) 3
12 · (c) Axis xx' in the plane of the lamina, passing
M.I. of a Uniform Rectangular Lamina through its centre and parallel to the length:
Length, breadth and mass of the lamina are L, B and M Consider a rectangular differential strip parallel to the
respectively. , :.
(a) Axis in the plane of the lamina passing
C··.-· ;v . -·- - _T___
length, of width dy and a- distance of' y' from the axis.

through its centre and parallel to its breadth:


. M
Mass per unit area = - .
LB
it:9'
,..
-~: -----------------:--11i"·.~\i
!"'.' ..
'' .

',. .·r···.
f,/./k---'-~
:- - -~ --- ·--~ ----~
fig. 5.41 ·
-- , , ·•
.. -
M.I. of the strip about the required axis is given by
',, ',iJ

il:: .
'
:-,. :
c/Jxx' .= (~ dx )x
Total M.I. of the entire lamina is obtained by integrating
2

the above expression between the 1.iinitsy =-~toy=+~.


2 2
I ' -_ +ii12M
xx -B/2 -B·x.
f
'dx
Consider a rectangular differential mass element
parallel to the breadth at a distance 'x from the axis of = MB [x3]+B12 MB2
rotation.
3
-B/2 12
Mass of the strip = M (Bdx) = M dx MB2
LB L I,=--
Since each particle lying on the differential strip is at the xx 12
same perpendicular distance 'x from the axis of rotation, (d) Axis xx' in the plane of the lamina, passing
hence the M.I. of the strip about the given axis is through one. of the ends and parallel to the length:
. . dJ=(~c1xJx2. It is easy .to· see that the required M.I. is given by
MB 2
l=--
Total M.I. of the entire rectangular lamina can be 3
obtained by integrating the above expression between the M.I. of a Triangular Lamina about its Bas·e
. . L L
11m1ts -- to + -. Co\)Sider a triangular lamina of base b, altitude h and
2 2· . massM.

www.puucho.com
Anurag Mishra Mechanics 1 with www.puucho.com

RIGID BODY ~OTION " -' -- .,, ,_,, .' --- _,..._, ... ~ ·------ .

Mass per unit area of the lamina ~ ( M ) = 2M =.!. (diagonal) D C


.!.bh bh 2
2 = _!_ (a.J2) = .!:..
Consider a 2 ./2 0
A
rectangular differential M.I. of_ the lamina BCD about BD
strip parallel to the base
of width dy, at a distance
Y from the axis of
T.
h
=(~)(:zr xi= ~:2 A
_Fig. 5.43
B

rotation (base). M.l. of the square lamina about the


From similar diagonal BD algebraic sum of M.I.
triangles
MBC, we have
MDE and l b-~'-*lc I of laminas ABD and BCD each
about their respective bases BD
I_ _ ,______· Fig. 5.42
DE
-=--
h-y
___________ J
'
= 2(~:2) =~~2
BC 'h · The M.l. of a square lamina of
DE=,(h-y)¾ mass M and side 'd, about the
axes shown in Fig. 5.44 are all Fig. 5,44_ __ __ _ .J
' b equal.
Area of strip DE= (h-y)-dy
h
Thus,
M.I. of the strip DE about the axis of rotation BC is given
by M.I. of a Rectangular Lamina about its Diagonal
di BC =(Mass cf strip DE) (distance from axis) 2 Let the rectangular
lamina PQRS have a mass M
= (2M
bh
(h-y)1!.dy)y
h
2
and dimensions L X B.
··- ... h/
By symmetry, the M.l. of
= 2M y 2 (h- y)dy the rectangular lamina ·-. :
'
h2 ~i·· ... Q:
about the diagonal
Integrating the above expression between the limits L
QS = 2(M.l. of the triangular
y = Oto y = h. lamina about base QS).
~ig. 5.45
- ---- j

h2M 2 3
I Be= f0
h 2 (hy -y )dy Let' h' be the altitude of the triangle QRS. Then area of
rectangle PQRS =2 (area of t.QRS).

= ~[(h~3 J-(\4 JI LxB=2[½x.JL 2


+B
2

LB
xh]
or h=--,===
= 2M2 [~ch3 -03) _.!.ch4 -o4)] .JL2 + B2
h 3 4 2
_2M[h 4 h4]_2Mh 4 _Mh 2 · Requited MJ., IQs=2[Mx( LB -) x.!]
h2 3 - 4 h 2 12 , 6 '.. 2, .JL2 + B2, 6

Mh 2 M L2B 2
IBc = - -
6 IQs =6 (L2 +B2)
The M.I. of a Square Lamina about its Diagonal
ivU. of a Solid Sphere
Let the mass of the lamina be M and each of its edges by .The ~ntlre sphere can also be imagined to be made up of
'd. The diagonal divides the square lamina into two infinite number of differential shells with radii varying
triangular laminas each of equal mass M. , continuously from r = 0 to r = R.
2
'consider a spherical shell of one such hollow sphere of
Consider the triangular lamina BCD, with base BD and , radius rand thickness dr. Then mass of the spherical shell is,
altitude OC. Then the distance r from the axis of rotation,

www.puucho.com
Anurag Mishra Mechanics 1 with www.puucho.com

-·~-~M_E-CH_.AJi!~
·= 3M (4itr 2 dr) = 3M r 2 dr Consider a· slice of thickness y
47tR3 R3 . 'dx' cut normallto the cone's axis, at
M.I. of . the · shell about the a distance ' j from the vertex
required axis is measured along the altitude.
Mass of the slice ·
dI = ~(3M r 2 dr)r 2 = 2M r 4 dr
· 3 R3 Rs,
={n(~RJdx]
"":':.~;;::,:~r,:r
Integrating the above expression
M.I. of the disc about the· axis
will be i:iven by · ·
dI;, :![P!tR. x ~](xR·)
I' .' .
7 2
2
2
y'
2 · 2 '::h h . '
..,:1:.
2 l~ . • ' . "
. ' I'
Iyy'':'sMR • 4 Ffg.5.48
. . pitR 4cJx ,_
0
- -4 x
M.I. of
,,
aR ight, Circular
••·, .,, , ,
Hollow Cone / 2h
1
We taJs.e'~ cone of mass M, · . ..c,.-"";;.-_-y_:_ _ _...,, . 1 hp7tR 4 4 ·
radius R and "altitude H about its
. .._._,:;,_ .,
" ,~ <J!:, .', . ·
. ., " .. r~·- .
Iyy' =-·J 2 o--h4
- x...
dx
1
aps_. •' > .' ·,. //J' .' ·. ·· ',.,,
The mags .. per- unit area of· · . i · : ·· · - pitR-4 [xs]h- P~4h
.the cone, p :;/~~i.' w!iereLis the ~~ ....:h.....l ...:. :t:? - 2h 4
.
5 -~ 0
=---~ --;~.----.· ....
slant height. : <., . .,
;,",'t' . . .• .
, : ., =~MR2
,· 10 .
Consider a 'Slice of the Cone . ,,H,,r:z:~r=:::u, ....
with its plane iiormal to the_ axis M.I. of a Hollow Sphere about its Diameter
i--- R------:
of the cone, of .width dx, at a :~-' y',; Consider an elemental mass dm with co-ordinate~ ·(x, y,
,:listance X front t:41! vertex, ,fig. 5.47 , . z) with centre of sphere as the origin. Since the sphere's
. measured along the'slant height. mass distribution is ~etrical about any diameter, .
The r~dius of.th~, ring · · · 'r' = ~ R.
therefore,, we have • · ·
,. . . .. L Ixr' = I yy' =I,... ...(1)
Its area = 2n(~)c1x. M.I. about x-axis, dI = dm(y 2 + z 2 )
M.I. of the ',Vhole sphere about x-axis will be given by
Mass 'of the jing, ·
.2 2
l,a• = J(y + z )dm
.
... (2)
: ·:· M2rrxR 2M m
"' • dm=-.---dx=-xdx
..._:·, ~ L L2 Similarly, . Iyy' = Im(x 2 +z 2 )dm ' ... (3)
M.I. of the e!eiriental ring, · and I,.,= j(x 2 +y 2 )dm ... (4)
_.. ·.·,.". ,:. dI =(2M
' . . lL 2
xch:)(~) L
2

Adc:ling eqns. (2), (3) and (4), we get


m

. 'L2MR2 .'
J.
I·yy' .= o.-L4
- x3 dx
'!x'+y2+z2
;:,;, ~

MR2
=.-2- :·-
. o:/ ,
Note: M.I. of a hollow cone is independent of the· -·--. -----------. -- ------ ---•x
, ..··:'.'"x":'
altitude or slant height or vertex angle.
;· ..···· j
M:I. of a Solid Right Circular Cone about its _Axis
. ~-/ :
ronsider a solid cone of mass· M and radius R. If' h' be "' z·. - , ,
the c, ,ne's altitude, then mass per unit volume,
--,., ."y;,·.,-.
· 3M ,:_;__'_c-:..:.'·_·....c.i;i:::.g_.S_-4_9_'·:..•:..·~·___
p = rrR.2h
I-xr, +lyy• +r••. = J2cx 2
+y 2
+z 2 )dm
m
.~,--
www.puucho.com
Anurag Mishra Mechanics 1 with www.puucho.com

...
.IIGID BODY MOTION
. .
.. ·-. 437l
[·.--x2 +y2 +z2 = R2] _ From perpendicular axis z
m theorem, I
-= 2MR2
I,,:'• =Iyy' =I.,, =3_MR 2

or
f, =I, +Iy =2I,
1
I, =Iy =-I,
2
.J
I ~3_MR 2 ·= .!_MR2
3 4 -:....Fc..:l;,.:9·c:c5c::.5c..:1~J
:....:x_·

PERPENDICULAR AXIS THEOREM The Parallel Axis Theorem


Perpendicular axis theorem is used ·for calculating M.L With the help of parallel axis theorem we can find an
of a laminar body about an axis perpendicular to the plane object's M.L about any axis if we know its M.L about a
·of the body. By a laminar body we mean an object that can parallel axis through centre of mass.
be treated as two-dimensional, the thickness t ~ 0. Any two Let the moment of inertia of an object· through its centre
perpendicular axes or rotation x and y lie in the plane of the of mas_s·be I CM. The moment of inertia of the mass through
. body and z-axis is the rotational aids that passes through the. any axis parallel to first is given by · '
intersection of the x- and y-axes and is perpendicular to 2
I =ICM +mr
them. The intersection of the axes lies within the body, _but
this is not necessary. Define the moment of inertia about the
- - - - -~-- . z -7 where m is the total mass of the object and r is the
perpendicular distance between-the two parallel axes.
Proof of Parallel Axis Theorem
,_:,

. ~·

I. . '

1
L ____________
(a)
. ""------~
Flg:5.50 ~'.,, , (b) - }' I i
.'

x- and y-axes to be I, and I y· The perpendicular axis


L___ F_is_.s_.s_2_ ____.__, ,,
theorem states that Fig. 5.52 shows an object that rotates in the zy-plane
I,=I,+Iy about the z-axis. The coordinates of the centre of mass are
The proof is as follows. Consider an element t.mi at a (xcM, y CM). Let the coordinates_ of a differential mass
distance ri from the z-axis (Fig. 5.50). Total moment of element dm be' x, y. Distance of element dm from z-axis is
inertia about z-axis, r = ~ x 2 + y 2 ; the moment· of Inertia about z-axis is
I, =,_r?t.mi f 2
I= r dm = (x + y )dm L 2 2

' In another coordinate system with origin at centre of


The distance of same ma~s element from the x-axis is y ,.
Moment of inertia about the x-axis, I x "= Ey 2
11m-l mass, the coordinates of element dm are
j I
X = x'+XCM
2
Similarly, we have I Y =Ex
f · 1 11m-1 · Y =y'+YcM

Adding I, and I y gives, I, +I y


.
= ,,(yf + xf)t.mi
' 2
=Er-
i /im- =I z
l l
,~s •.
~tallon
2
' ---

Hence, the perpendicular axis theorem is proved.


mustradon. The moment of Inertia of disc about·
z-axis, -
I =.!_MR 2
. .,._ ·,,.,
• 2 .
From symmetry, we.see that I, =I Y
www.puucho.com
Anurag Mishra Mechanics 1 with www.puucho.com

i43a
·----- ·- ··----·-- -----~
Therefore Solution: (a) We use parallel axis theorem with
f
I = [(x'+xcM )2 + (y'+y CM )2]dm
2 2
= J[Cx') +(y') ]dm+2xCMJ x'dm Axis
0
+2yCMfy'dm+(x~ +y~)f dm
The first tem, is by definition, the moment of inertia
about an axis that is parallel to z-axis and passes through the
centre of mass. From the definition of centre of mass (origin
of second coordinate system), J
x' dm = fy' dm = 0 ;
therefore nexttwo integrals are also zero. The fast integral is
~---
: ~'

simply mr 2 ,because F_ig. 5E.5


f dm= m and r 2 =XCM 2 2
+YcM 1 2
2 ICM=-MR andh=R
So we conclude that I= IcM + mr 2
2 3 2
I=IcM+Mh =-MR
2
·~ ··- . - .. 1 2
(a) Determine the moment of .i~eriia of a solid cylinder· of: (b) In this case, IcM=-ML,h=L/2
2
radius R and mass M about an axis tangent to its edge'
From parallel axis theorem, we have
and parallel to symmetry axis. · · ,' i
(b) Determine the moment ofinert:ia of a unifor~ ~od of mass;
· · · I =IcM +Mh 2
M and length L, about an iJxis perpendicular to the· rod! 2

through qi,e_end. . ..
, -.-v
. _ __ _ _ _ . __ , =..!_ML2 +M(£) =!ML2
12 2 3

Table: Moments of Inertia of Uniform Bodies of Various Shapes

' 1ml2 ' 1.. mL2


3 , 12
Slender rcid Slender~rod , ,Disc Cylinder
(~) (b) (c) ' ' (d)

£mR2 1mR2
5 2

mR2
Thin-walled cylinder
about its axis Solid sphere Hoop
. (~) (I) (g)

www.puucho.com
Anurag Mishra Mechanics 1 with www.puucho.com

1mR2 + .1. mL2


,
4 12 · ·
. A solid cylinder about
a perpendictilar
Spherical shell axis through its cnetre Coiie Hoop
(h) (i) 0) • (k)

--a.
-r;:..-~--- .....
1..mL2 £M[Rl• -Rf],·,
I,
3,
A re~tanQular abolit-a
5·. . ,Rf -_Rf
,
..
Rectangular A rectangular '1 _Parallelepiped _axist,.~. '"Athick.:.wair8d hollaw·sphere·,
parallelepiped about one edge · throµgh its.ce_ntre about a diameter • ·
(I) <n.i>__ . . . ... - ~>:....... , ----'> (<>)_ , , .

Problem solving Tactics (Moment of inertia of·


a composite body): By composite body we mean a
combination of rigid bodies, rigidly attached to each other.'
1. Break up the body into components, CM
2. Determine the moment of inertia of each L_112
component about the given axis of rotation,
3. If the rigid body can be segmented into several R
symmetrically shaped components, the moment of L---~~~-.....1----;~x.
inertia of the object as a whole is the sum of the Rotation axis
i
moments of inertia of all the segmented L_ __F.!!l:.~:55. .
components provided the moments are about the Moment of inertia of the system about the CM of the
same axis. system in the first case is
I= I 1 +I 2 +I 3 +.....
illustration 1. In the Fig. 5.54, the uniform cylinder
has mass M, and radius R; the wooden planks have mass
M w and dimensions 1x w.
"' - .-.. -- - -- --·-·7
In the second case,
2

lid~lllll
•. =..·--~,=--=-~=-~
!!J!.~.~:-:~:--:.-:,-:-:/ '=": I 2 =_!M R 2 +2[M,) +R 2 ]
2 C 12
illustration 2. A rigid body consists of a hoop and a
thin rod of same mass. Its moment of inertia about the
' Fig_. 5.~~--. horizontal axis is
I System = I Hoop + I Rod
=_!mR2+mL2 +m(R+!:.)2
2 12 2

www.puucho.com
Anurag Mishra Mechanics 1 with www.puucho.com

' . ,, ' .' , ~ ~ ' . . ~' . . . ' ' :

1: . Conc!lp~=:·subtract th~ contributio~·oJM:I. ofa qisc ~j,1·


ti:e Jha~ ojJ~'-'ho_/e·from.tJ,te M-f. :of~omplete'(lisc!;:, ,;' L· '; ·

Let ' m' be the mass of remaining disc.


Area of original disc = itr 2
. Area of removed part= it(r / 2) 2
Therefore area of the remaining disc. i
=itr - it(r/2) = (3/4)itr 2
2 2
· . m 4 m
Mass per urut area= cr = - - -2- - . -
. (3/4)itr 3 ·itr 2
Mass of the original disc= ii~ 2 cr = .'.! m
3
.,, Mass of the removed part ~ .!. itr 2cr = .!. m
4 3

I Moment of inertia of the complete disc (Icom)

E6_______.,_.i,.:._',.~
1 4
=-x-mr 2 2
=.;....mr 2
,
~. ! _______
i~:- _,,_, Fl s._s_. 2 3 . 3
Moment of iµertia of the removed part (I removed)
. Solution: The rigid body consists of a cylinder and a
piate. M.I. of the~e symmetric shapes are known.
. 1 2 =½ ;(~Y.+ ;(~r
I cylinder = 2 mR l· 2
=-mr
m'a 2 8
Iplate = -- Therefore the moment of inertia of the remaining disc
6
2 2
The tcital moment of inertia is the sum I cylinder + I plate _ · · =•(~)mr
I remauung 3 ' -(.!.)mr
8 .
To find the moment of inertia of the remaining
portion of a rigid body after a part of it is ·1 remammg
.. = (13)mr2
removed. 24
The concept to be used is: Rotation about a Fixed Axis
!VI.I. of the remaining portion = .M.I.of the (complete) In this section we will consider examples of rotational
body before removal - M.l. of the removed part. dynamics for constrained rigid bodies. The main aim.' of the

-'"~£.
problems is to determine angular acceleration of the object
~:ire:
~---=--~---,~-;-,...,-._ .
7 .
'- '~,. ~, '_ ' ,: ' J~P '. _,)
· and acceleration of CM if axis does not pass through CM.
Problem Solving Strategy , . (Rotational
A circular hol~ of,radius 'r/2; :is_ cr,,t;ft:om a· circ~lar 'disc o.f,j dynamics problems):
:rad!us-'.( Tlie; d~c;lies_'in jhe :,p:'I?Ja~e, ?~te""l!n_e'tlje Miilie(!~ 1. Always try to draw a pictorial representation of the
,of_ mertta p_bout ;q.n ·axzs,,passmg. th[~ug~ ·the centr~ ,qnd1 · situation, define a coordinate system. Draw free
perp_endiaiLai:Jii.th(p_la,J~c:oLfue.,;(lJ,sc;.<,- . . "· .-· •· ..· · ~~~.:.:.! body diagram.
.Solution:
2. Identify the axis ·about which the body
rotates.

www.puucho.com
Anurag Mishra Mechanics 1 with www.puucho.com

["RIGID BODX MQTirirL:' .: - ?


. ·.+E)
3. Identify the forces acting on the body and DYNAMICS OF A RIGID BODY
determine their moment arm about axis of rotation. Rotation about axis through centre of .mass
4. Identify torque exerted by the body and sign, of (Centroidal rotation)
torque acting on the body:
In this case r = 0 and torque is deterntined about the
5. If the axis of rotation passes through CM,
centre of mass. The equations of motion are
LFX = 0, Lf'y = 0
Where Lf'x = algebraic summation of Lf'x = o, Lf'y = o, :E't = Ia
component of the forces along where,
x-axis Lf'x = algebraic summation of the components of the
Lf'y = algebraic summation of external forces along any arbitrarily chosen x-axis,
compon_ent of the forces along Lf'y = algebraic summation of the components e>f the
y-axis external forces along the y-axis,
Constraints Due to Ropes and Pulleys L't = algebraic summation ofthe torques about the axis
During the course of r · - - - - - - - - - - · , of rotation through the centre of mass.
rotational dynamics we
encounter situations where I bexam,gle-~
pulleys are connected via
strings of belts to other /An object of mass mis tied to a-light string'wound around.a
objects. We assume that the . ~ulley that h<If fl moment of inertial and radius R, -They.,h4el
string. turns on the pulley [bearing is frictionless, and the string does 'not slip o.n the run.
without slipping. Due to !Find the tension in the string and the acceleration of the
this constraint the string's 0ij~ec~t~-----------------"""'
speed v ,rnng must be equal Solution: Problem Solving Strategy: The obj~ct
to the speed of the rim of descends with a constant downwards acceleration a, while
the pulley which has the wheel turns with a constant angular accelerati9n a ~Fig
. ..., ..., SE.B(A)]. Because the string unwinds from the pulley
magnitude J v rim I = Jro IR. If
without slipping, a = Ra. We apply Newton's law for rotation
the pulley has angular acceleration, the strings acceleration
to the pulley to determine a, and apply Newton's second law
a,ning must be equal to the tangential acceleration of the rim
..., ..., to the object to obtain a.
of the pulley la, I= JajR. · Since the object moves downward and the pulley rotates

l-c~. ~.t. =____ --,-=·-b,_oc1<_l_=_l:-JR-~


clockwise, we take these directions to be positive.
The only force that exerts a torque on the pulley is the

L~ _. . ----"·Jablockl =. J_«:lR_~
tension T, which has moment arm R.
L't1 ext = Ia
Motion Constraint for a Nonslipping Rope 1R = Ia ,------, ... (1)
,-·---;;---~-
14:~
We always assume that
pulleys move with the string. A I
pulley has significant mass, ! '/ .---t---•X
therefore a torque is required to J'
turn the pulley. Torque on the I ,, I
pulley acts only if tensions on '
two sides of the string are !. T, y I T T
different. The bearing of the T2
ra:j .
~~J
'
pulley is always considered m . • ->
mg
frictionless. The groove in
which the string slides is not
frictionless, otherwise the
string will slip. Static friction . .___F_1g_._s~ .._F_,9_.s_E_.s_c_e>_,
between the pulley groove and the string makes tension Draw a free-body diagram for the suspended object [Fig.
different. · ....
SE.B(B)] and apply :E F = ma
...
mg-T=ma ... (2)
We have two equations for three unknowns, T, a and a.
www.puucho.com
Anurag Mishra Mechanics 1 with www.puucho.com

r.w:i-- ,;MECl!i\~lf~:f:.J.
a= Ra. ... (3) string and the pulley. (If groove of pulley is frictionless, the
.The third-equation is the constraint equation relating a pulley would not turn.) Note that T2 exerts a clockwise
and a.. torque and T1 exerts a counterclockwise torque on the pulley.
Now we have three equations .to determine T, a and a.. Use Newton's second law for each block and ~ = Ia. for the
On pulley, then relate a. and a and by a = Ra..

!- ·;. :· -~~ . --·· -~--:~~-1 r-T,-1·-.·.··,·


solving for T, a, a. we get
I mR2
T=--~mg; a= g
I+mR 2 I+mR 2
Analytical Skill Problems '
j m, T1
.-.~.,,.....J Fn
/
i
I
;

!.:~c··--- .. -.~!
1. Which of the following options are correct: ' !I • m2

~- :· ~:,~.6--: _· II _m,g
,. • • ' I

]
1"
; VmR2
''·

,
' •
·
l/mR2
I
r
'
I
1.---~ - ·- · - - -
Fig: SE.9(8) ' ·
~ .~-- · - . - - - · - · · •• l
If
I i Fig. 5E.9(C)
"- .,

: - I Draw a free-body diagram for each block and for the


(a) · .(b)
pulley [Fig. SE.9(B)]. Note that the pulley does not

:~L~t=
·t
I ·
(c)
iimR2 '
(d)
' l/mR2
accelerate, so the support must exert a force on the axle Fs
that balances the forces exerted by the string.
...,

'm 2g -T2
...,
Apply L F = ma to each block.
Ti = mi a
= m2a
... (1)
... (2)
Apply Newton's second law for rotational motion,
T,_ --·- ·----, -----
Fig. SE.8 (Ci'
-· -.. , ' - -- ;
---·
.
-- L"t, ext = Ia., to the pulley and obtain another equation for
. (a) If the pulley is massless the object falls. freely and T2 - T1 • Use the non-slip condition to eliminate a..
the string be~omes slack. ·
(T2 -Ti)R =Ia,T2 -Ti= ( :2 } .... (3)
(b) If th~ p·ullcy has I ~ = then tension T ~ mg. .
(c) I/I» mR 2,a=U On solving eqns. (1), (2) and (3), we get
( d) A massless pulley does not have angular acceleration. a= m2 g
2
2. Which ofthe following graphs represents acceleration in mi+m 2 +I/R
units of Q/g vs I/mR 2. ' mi
Ti= 2m2g
mi +m 2 +I/R
!±~~~mP.±_~[91> . T (m1 +I/R )
2
--- - ----------- ------------- --- ----· ·-·-----,
;nvo blocks are connected. by a.string that passes over a pulleyi 2·= mi+ m2 +I/R2 m2g
;of radius R and moment· of inertia I. ·The blbck of mass. mi\
· Analytical Skill Problems
lslides on a frictionless,horizontal sljrface; the block of,rnassJ
:m 2 is suspended from the string [Fig, 5E.9(A)]. Find the! 1. Which ,of the following'options are correct ?
·acceleration a of the blocks and · the tensions Ti and T21 ( a) If pulley is massless, tension on either side of string will
'.assuming
\
thgt;_Jh~"s_tring
·;
'dQ'I§_n_pt~Jip 011.!~?. p1jlley. .!
,.,,
be same.
I --~ a· , (b) In case of massless pulley, accelerations ofmi and m 2
m, T1 are equal to g.
0~ (c) If pulley is massive, then T1 =. mig.
1 (d) If pulley is massive, then T2 = m2g.
T, la (e) When I is small, a approaches g for large mi.
(f) When I is large, a is reduced.
Fig. 5E.9(A) 2. Which of the following graphs represents acceleration
;
_______ ! - - - - ~ ...... - - - . . - -...... ~ ... -- ·- --·--- , ______ _ in units (a/g) vs. m2 /mi?
Solution: Problem Solving Strategy: The tensions
T1 and T2 are not equal because there is friction between the
www.puucho.com
Anurag Mishra Mechanics 1 with www.puucho.com

r--. .. -. ···, -- -· --- ·- ·-- -··a_ -·


1 RIGl~~B~_DYMOTIOII ----·-'----·.'_-_ __ 443 I
- --·-· --- - . - . - -- - -

!--~--~
~
~
'
MgL/2 3g
a=-=--=--
2
I ML /3 2L
--+ --+ ',
l: F = ma for the rod.
Mg-F0 =MacM'
' m2/m 1 _m 2/rrt 1 Acceleration of centre of mass is
L
(a) (b) acM =ra= a

;1-·lL___
~1 / ··h__
-- . ,.
~ Substitute aCM
=~~f=~g
22L 4
2

into Newton's -second law and solve for


; , m2/m 1 mz!m1
'
F0 = Mg-MacM
(c) (d)
Fig: 5E.9(D) =Mg-M(¾g )=iMg
Analytical Skill Problem
Which of the following statements are correct ?
(a)lmmediately after release of the rod its angular
iA uniform slender rod of length L and mass M is pivoted qt acceleration is maximum.
:one end. It is held horizontal. and, released {Fig. SE 10). (b)When_ the rod reaches vertical position its angular
!Assume the pivot is frictionless, find ·, acceleration is maximum.
'(a) the angular acceleration of the rod immediately afte.r iUs (c)A small particle is placed (m « M)at the centre of the
released,. and · rod; the force it exerts on the stick immediately after release is
:(b) theforc~.F0 exerted on the. rod by the pivot immediately
after-release. _____ ~--:_•_· __ .·_, . . mg upward.
4

:i.17! '
( d) When the rod reaches vertical position, the force
exerted by the rod on the pivot is less than its weight.
r -- -- - . 1 -- r.:-l
_1•.e ,
e>.G"'•i;m\-n
1·_
~~~!
lki\~
11 ;7
' Mg I
I
r~ .....
Fig.5E.10
.
i
...,,_ .. _ ..... .,,,.,"'.-.... ~-
.
-·----~-·-------.J
I
,; IF~ SE.11 (;,) shows two bloc/; mi and mz, connected b; lightj
lstrin~s, _supporteil_ on two _ide~tical pulleys of _moment of,J·
Solution: Problem Solving Strategy: The angular 1inertta I and rad,us R Deternzme the acceleration of each
acceleration is found from ~ = Ia, where ~ is the torque block and the tensions Ti, T2 and1'3 ln the string. There is nol
exerted by gravity relative to point. The centre of mass of the slipping between string and pulleys. __ _ ___ • ____ __ i
rod can have tangential as well as centripetal acceleration.
Since the rod has an angular acceleration, its centre of mass
i T2 Ij i
has a tangential acceleration aCM = .! La. The initial
i
2 l
centripetal acceleration of the rod is zero because its velocity
is zero just after release. The force exerted by the pivot is
found by applying Newton's second law to the rod. Since the
acceleration is downward and the weight is downward, F0
must be vertical. Assume that it is. upward and take the
positive direction to be downward.
Newton's second law for rotation gives
Solution: Force diagrams for blocks and pulleys are
l:~i ext = Ia shown in Fig. SE.11 (a); we consider downward direction
Compute the torque about the end of the rod. for m 2 as positive and clockwise angular acceleration of
L
~=Mg- pnlleys as positive. Equations for blocks are:
2
m 2 g-T3 = m 2 a ... (1)
The moment of inertia about the pivot,
Ti -m1g = mia ... (2)
I= _!ML 2
3 Equations for pulleys are:_
Substitute these values to compute a. (T2 -T1 )R = la ... (3)
www.puucho.com
Anurag Mishra Mechanics 1 with www.puucho.com

(T3 -T2 )R ='ta. ... (4)


Adding eqns. (3) and (4) gives
(T3 -T1 )R = 2Ia. •.. (5)
Adding eqns. (1) and (2) gives
,(T3 -T1 ) = (m 2 - m1 )g-(m1 +m 2 )a ·:·(6)
From eqns. (6) and (5), we obtain
[(m 2 ~ m1 )g -(m1 + m 2 )a]R = 2Ia. where a.= a/ R. Torque due to m 2g, ~2

= -m 2g-cos0
2
Thus a= (m,·- m,lir '
' , I Net torque, I:1ext = ~1 + ~2
m1 +m 2 +2-2 . 1
R . =. -(m
2 1.
- m 2 )gL cos0 .

r~-- r;_~a-n~
:n
l• • Tf ·_
.m
e
T

mg _
P
2

:.

,
, "'<

O (
g : T3 •
P •
I:~ ext = Ia. ...
· I:1""
CJ.=--= . .
I _ L(M/3 + m1, + m2 )
I
2(m/- m 2)g case ·

If the masses of particles are equal, m1 = m 2 , a = 0, the


·L:.!: · ,Fig.•se:u.(!1J~ ·· l system has no angular acceleration.
\ Note that we may obtain T1 and T3 from eqns. (-1) and Angular acceleration a. is zero when-0"' 1t I~ or ~,c / 2,
(2) and T2 from eqn. (3) by substituting for a and a.. If i.e.,vertical position:
ni~ = m2 , a = 0 and If m1 > m2 , a is negative; then motion is Angular acceleration a is maximum when 0 = 0 or ,c, i.e,
reversed in direction. · horizontal position.-
m,[~-""*
___®_m_1}·~ Cc) Angular acceleration continues to be positive unit
0 = - ,c/2; the system gains angular velocity at this angle.
1 Therefore angular velocity is maximum when 0 = - ,c/2
~- u~fli-orm
__. _ro_offfi<¥_ sM ~-n_ ·d__1e_·_~_t11_·_1_\v1th two_· r~i-.tic_·ies_ · ~_:_·_~--
d_-.

1and rit 2 attac1i.eg. to its eni:ls, is."piiioted·dt its centre,··.T/ie Problem Solving strategy (Rotation about a
[systeni r~t~t~, fc!'~,~~rtical plmfe"wifl];_i:ilJgg_la/velociiy.'~ 'j'.il n_on-centroidal axis): In this case axis is perpendicular to
.the plane of the body and shifted _from centre of mass of the
t·" ··,,. ~.- J '. ' . "J:•

,,,
c, ·,.~~

,_.. . .. • l'] bo(iy.


~,' ., ( <' • • ,',@ . .
1-c.·; .,·
,.·. ! ·" ·,:,
1j
A--+---':;..,=-1!--l--X ' :,
-~· 1~· }jl
·-.
,'
.·_; ' ;· -. ,·,1
,_ .' ~ :.'---:~J £Ft "l

·I, .. '. '.'i.f •,l ·,·i , .


Path of CM ,. J
. ,· -~· , , ~ ··.<:} , m1g . . ·,,11
t,·. , ·, . ~i _ _ Fig.5E.12 .· .. _ .. _, . . · "".+)
1(a):

·vetei-mut~ithe elcpression
. ,;:_ ',. ~., . ' ,· ;:_
jor'iirfii'ii.t irtomentum.:'!
,.-_·,_-,,:-,::_,.,·., .. '"
.; '<<!
• ,1
'(b). Whc;zt•is Jlig)/ll_guJar acseleraqofl;<Jf.the sys~em 1Yhen,¢h'e
'j,._·. ~-·'."' _,_,.
-. I
I· rod ljtak/!$ an·cu\g1e e wrtht~rhl-Meontai? · . ·. . 1~ ·, ··-··1
CcJ lf ~-2 >·nti, ,,w1terf d~ff- ",ap;~li1r vel~city ~~~I~l '-~.L....:,_:....:~:.c...:.c..._;F_:;ige:...:5=.5~~;~~-~~---'-'', _:·:·ifil
[_...maxtmum?-: -.- - . . , ""'~- . .- " "'"';;.!
Soluti(?n: (~) Moment of inertia of system is sum of The equations of motion of the body are:
moment of inertia of rod and two particles. · · . I:Fn = mrro 2 ••• (1)

:1 =if +mit%r +m 2 (%r


El', =mm
Tu=fu
... (2)
·--~
= L: (~ + m1 + m2 ) where .
I:F. = algebraic summation ot"the components of all' the
Therefore the angular momentum is exten1al forces along the -n-axis. It represents net
centripetal force on the body.
www.puucho.com
Anurag Mishra Mechanics 1 with www.puucho.com
--

I· RIGID BODY MOTION


l:F, = algebraic summation of the components of all the On substituting the expression for ro in eqn. (1), we
external forces along the t-axis. It represents net obtain
tangential force on the body.
1:,: = algebraic summation of the torques of the external
R = 3Mg
n 2
forces about the axis -of the rotation through 0.
When the pivot is remov'~d, the only force acting on it is
m= mass of the body.
gravitational force mg. This force does not have any torque;
r= distance of the centre of rotation O from the mass
therefore the angular acceleration of the rod vanishes. The
of the centre G,
rod will continue to rotate with angular velocity ro = .J3g I 2,
I = moment of inertia about axis of rotation,
as well as it will fall with acceleration due to gravity, g.
ro = angular speed,
Velocity of centre of mass after_ time t ls given by
a= angular acceleration.
V= v 0 +gt
i"E.va:m_nle
= -""-~-- --,r,e
J
- ,,~
13 l,-.......... L
=-ro+gt
2 .
C1 ;-;nif;rm ~l;nder ;;,·d of mass M a;dlength Lis pivoted at its L .
= -.J3g/2 + gt
lend. The tod is released from its nearly vertical positio'!- Wliat 2
1is the reaction at the pivot when the rod reaches the
rhorizontal position? If the support is withdrawn at this 1 Lexamr-.le.J
-- -~re,, --- -·
14 t:-----...
~
li!!sta~t, describe the subsequent motion of. the ·rod. · I r------·,• ---- - - - - - - - . - - - ,--~- - -·-- ,. -

Solution: The rod undergoes non-centroidal rotation Three fi,gtticles A,, B and C each of mass m are connected t~
about the pivot. As shown in Fig. SE.13, we have considered each otl/er by tliree massless rigid rods to form a rigid,
Rn and R, as the normal and tangential comportents of total equilat~ral triangul~'r b,o_dy of side I. This body is placed on a .
!horizontal frictionless table (xy-plai,e) and is -hinged to it at
reaction at the support.
l--·-'..--:--------1 the point A so that' it can move without friction about the
vertical axis through A (see Fig.. SE.14). The body is set.into
rotational motion on the table · about A with a constant
angular:velocity ro:
: : I
: .: I i

''i
:: ·-. . R, . \
i (a) Find the magnitude.of the horfy;ontalforce exerted by the1
I. hinge on the body. · ·
(b) At time·'I; when the side BC is parallelto x-axis, afim;e Fl
i
I •
,n-ax1s is applied on B along BC (as in Fig. SE.14). Obtain the,
\,.._;:....
x-component and they-component of the force exerted by[
../ Jhe hinge on the body, immediately after time T · I
R, Mg
-------------··--· y' . Ij
, ..
Fog. SE.13-
:~-axis , !
1
- - - - - - .....,... ___ ___ .J I X 1-I
The equations of motion are
L 2
LFn = Rn = M -ro ... (1)
I ·I
2 I I :

LF, =Mg -R, =M-a
L I a,=ra1 I '

2
... (2)
I 1~
' F F
-.!:,-d-------<D i "I
l

C' lJ
. L ML2
1:-r=Mg-=--a ... (3)
2 3
3g - c~~~·-·----
From eqn. (3), a=-
2L Solution: (a) The centre of mass of the system is at the
L Mg centroid of a triangular assembly. The CM moves along a
From eqn. (2), R, =Mg-M-a=-
2 4 circular path with constant angular velocity. Therefore there
The angular velocity ro of the rod can be obtained from must be a horizontal centripetal force directed toward axis
the energy conservation principle. at the hinge. The magnitude of horizontal centripetal force,
(KE + PE) initial = (KE + PE) final 2 2
F= 3m(..!:..)ro
.J:i, = .J3mLro
.
O+Mg!:=~(ML2_)ro2+~M(roL)2 +o (b) Let Fx and FY be the force applied by the hinge along
2 2 12 2 2
x-axis and y-axis respectively. The system is· in
or ro =.J3g/2 non-centroicial rotation. The three equations of motion are
www.puucho.com
Anurag Mishra Mechanics 1 with www.puucho.com

~~? ' '.-


--'---------'---~--------
:r.Fx = Fx + F = (3m)a, = 3m( 1Cl) ...
Equations of motion along t-axis and n-axis are
(1) a
:r.F, = Mg cos8-N = M -a
:r.Fy = Fy =,3m( irn 2
)
... (2)
= Ma (ro dro) = ·Mg cosB
3 dB 4
3
... (3)

l:1 =
' F x (-J3J
L =,2mL2a ... (3) 3Mg
2 Therefore, N = - - cosB
4
From eqn. (3) ," Cl=--
-J3F
4mL 3Fn = µN -Mg sinB = M(i)ro
2

3
From eqn. (1), F +F = 3m(_!:__)x -J3F = F Ma 3g sinB Mg sinB
X ·-,/3 4m£ 4 =-x-=---
3 · 2a 2
F = _!:_
µN = ~ Mg sin 8
or 4
X Therefore,
2 2
From eqn. (2), Fy = -J3mLro
µ(. Mg4
3
cosB)=~MgsinB
~~~~~~ls> 2
µ = 2tan8
.

¼unifo;~ ;od ;fleng~h 2a ~ ~i;ced-h-;,.;;;n;;,_i~ on the edgeJ Analytical Skill Problem


!of a table, Jnitially th~ centre of mass of .the rod is .at a\ Mark the following statements as true or false:
;distance a/3 from the edge. The rod· is released ·from test. If,· 1. Friction force on the rod immediately after release of the
!the rod slips after it has tumed'thtough an angle 0, find theJ -rod is zero. ·
;coefficient-of
- - - ---
friction-- between
--->••h' · - · - - - ..
the -rod
,-· -~ -
and the table .. _-----~-.ll
----·-··--·-
·· ·\ 2. Initial acceleration of CM is in vertical direction.
Solution: Initially the rod rotates about the edge. · 3. Angular acceleration of the rod is increasing as the rod
From principle of conservation of energy, rotates.
KE; +GPEi = KE 1 +GPE1 4. Normal reaction of the edge is increasing as the rod
rotates.
O+O=~Iro 2 +(-Mg~sina)
2
2
3.
2
... (1)
5. Velocity of the right tip of the rod is {fga sine.
Where I = M(2a) + M(~) from parallel axis
12 3
~~q:m:e4~j 16 ~
- ---· - ,.

!A .parked truck has its rear door wide open as shown in. J:ig. i
... (2) ISE.16. .At 'time .t = 0 the truck starts to accelerate with I
,constant acceleration.a. The door Will begin to close, andat a,;
ilater time t the door makes. an angle 8 with its original·
!orientation. Assume that the door has mass m uniformly!
distributed along its length L · ,
r·---·-- ----- ---- --- - - ---- · -~- ,j
i ..t-ax1s I i
, ·\ t Door F;° :;
L I'
'I J
11
,··l
II Hinge a a i

ta) . •• · (bl ·I
I
I
:I,,
'------ ; ___ Fig.SE.15', ... · _.____ __.:J '.. _ _ _ _ Fig. SE.16 . _ _. __ Ii
Differentiating the above expression for ro w.r.t. 8, we '(a) Iffn a.. n·dF, <Ire. the two COl!lpone.nt. s oftheforce exerte·dl'
obtain on the door at the hinge, determine their expressions in
dro 3g
ro-=-cose
de 4a · I terms .off!, m, Land a.
(b) Write.· down, without integrating, an expression for ilte
.. 1

Since,
dro dro dB
a=-=-x-=ro-
dro i total time el(ipsed from the start of acceleration to ·,the ·
dt de dt dB [__ · clo)Lingpfthe door. -----'- _:. .. ~

www.puucho.com
Anurag Mishra Mechanics 1 with www.puucho.com

-- ··- - 447·1
·-·-· - .J
Solution: We choose the 'reference frame of the
..,
JLl=rpsin<p =r.L xmv
accelerating truck. The centre of mass of the door has
r sin <I> is known as the moment arm, or lever arm,
components of acceleration .! La perpendicular to the door
2 designated as r.L as shown in Fig. 5.60 (a).
and - .! Loo 2
parallel to the door. In this reference frame a The orbital angular momentum of a particle in circular
2 motion is expressed as
pseudoforce -ma acting at the centre of mass is included in .., 2->
L = mr OJ
the equations of motion. ..,
1 Note that the direction of angular momentum vector Lis
I:F, =F,-macos0=--mLa ... (1)
2 . ..,
parallel to angular velocity OJ for a symmetric body. Fig. 5.60
I:Fn = Fn - masin0 = _! mLoo 2 .,.(2) (b) shows the right hand thumb rule for determining the
2
direction of angular momentum. Curl your finger in
L 1 2
:Et=F,-=-mLa ... (3) .., ..,
2 12 rotational sense from r vector to p vector [Fig. 5.60 (b)];
2
then the thumb points in the direction of angular
where mL is the moment of inertia of the door about an
12 momentum.
axis perpendicular to the top edge of the door through the
centre of mass.
mLa
From eqn. (3), F, = - - ... (4)
6
Substitution F, in eqn. (1), we obtain
3acos0
a=--- ... (5)
2L
1
(a) From eqn. (4), F, = -macos8
4
doo
3acos8
a=oo- 0
d02L
(a) (b)
or "'oodoo = Ja3acos8 de
J0 0 2£ .,.l= ........
rx p
OJ
2 3asin8
=--- .,....
L
2
(b) On substituting expression for 00 in eqn. (2), we
obtain
Fn = masine+2masin0
2 Circular path
of particle The system
5 . e
=-masm
2 (c)
=de= ~3asin0 Fig. 5,60
As 00
dt L
Concept: The total angular momentum of a rotating
'
the total time elapsed from start of acceleration to the
closing of the door is !rigid body is found by i•ectorially adding the angular moment'
jof its constituent particles.
t = Jn ,---i:-d8.
o'\j~ ,'.we choose the body's rotation axis to be the z-axis (Fig 5.61). i
:The velocity of a particle of mass dm lies in the plane,
ANGULAR MOMENTUM iperpendicula.r to the rotation axis and has magnitude,
The orbital angular momentum: Irrespective of ,oor.L.The angular momentum of the particle is
the path of trajectory of the particle, be it a straight line, a .--+ --+--+ --+ --+
curved path or a closed orbital path, the orbital angular 'dL = dm(r8 xv) and is perpendicular to r8 and v, as shown,
.., I -,
moment L of the particle at any position w.r.t. a reference /in Fig. 5.61. Jhe different contributions d L from different'
point is · iparticles of the boqy are in different directions. The direction
.., .., .., 'of the sum cannot be predicted for an irregular object, but for
..,
L= rxp
,g sym!]le_tric _bpdy !,_ is P'!r'!lkl. t_o ~ . of ,ptgtjo_n. .
www.puucho.com
Anurag Mishra Mechanics 1 with www.puucho.com

axis of rotation

inward velocity
differential element
on body- --> ->
LA =.IAro
Note that angular momentum depends on the axis of
rotation, ·therefore it is advised · to specify the axis of
rotation.
--> ,,. ..., -->
Condition for Static Equilibrium
'2 dt:, dL1 r1 An object in static equilibrium should neither accelerate

____J!
nor begin to rotate because of forces acting on them. There
OU~ ~ r d velocity. _ate two conditions for static equilibrium that form the basis
velocity for the analysis of all static objects.
vector vector
Fig. 5.61 The two fundamental equations that govern the
!-------------~----- dynamics are:
' · . Angular mo111e,ttum vector is perpendicular t~ the plan~
Newton's second law of motion
of podtio~ ~ector1l ·and velocity vector 1. . -. v -> ->
Ftotal =ma
.fe6'x~rnm\i~ and its rotational counterpart
->
..., d L tow ->
!Fig. SE.17 .sh~~s a barbeil rotating -=~h angulcd velo~ty ©I ~total
dt
Ia

·1~bout its centre of mass, ~~ere,t~e,:vector ro_ liJs along axisA (for systems with constant moment of inertia I)
Find-its
. '.angular momentum by.appljring. vector equation. , ....
. The system is in static equilibrium if the accel~ration
of the centre of mass of the system is zero;and the angular
acceleration of the system about an axis through its centre of
. mass also is zero.
Hence the two conditions for static
,. equilibrium of a system ate:

\,; .
f·,<' /
(

[:
. ',

...·
--¾
V2
. _;,.- I--··
--~
•..

Fig. SE.17
m···-...

I.
[ , '
j - "
j
1. The total force on the· S¥Stem must be zero,
->
F,otai =0.
2. The , total torque on the system must be zero,
->
'ttotal = 0.
L.....:I--CC-'-,,s---------·-----'-··'::,_~,..J._..,_"
3. Therefore the centre of mass of a system in static
Solution: The barbell can be modelled as two particles
equilibrium has a constant velocity: v'CM which is
on the ends of a massless rod. We define the z-coordimite
axis to be the axis of rotation and the x-axis to lie along the constant. The angular velocity about a symmetry
barbell axis at the instant shown (Fig. SE.17). · axis through the centre of mass point also . is
->.
constant: ro 1s constant.
½from axis A. The
0

. Each particle is at a distance Ir. I=


4. The acceleration i and angular acceleration ii both
· velocities of each particle ate perpendicular to r and have 8
must be zero for a system to be in static equilibrium.
5. To calculate the torque on a system, we introduce a
magnitude rol. Velocities at the instant ate ~ positive and coordinate system and calculate the torque of the
2
negative y-axis. various forces with respect to the origin of that
coordinate system. We can choose any point as the
The angular momentum of the barbell is origin about which to calculate the torques of the
-> -> ->
LA= m 1 r1 xV 1 + m 2 r2 xv 2
·various forces. For static equilibrium, the vector

www.puucho.com
Anurag Mishra Mechanics 1 with www.puucho.com

, RIGID BODY MOTION


sum of the torques of the forces must be zero
/
regardless of where we choose the origin for
calculating the torques.
6. If the lines of action of several forces pass through a A uniform board of mass M and length L rests on two spring '
parJ:icular point, i.e., the point about which we /scales. A person of mass m stands at a distance l from one end'
should calculate torques for static equilibrium 'as shown in Fig. SE.18 (a). The board and person are at rest.·
problems, these torques of the forces whose lines of What is_ the r~ading of.the tw9 scales? .
action pass through this point are zero, because Solution: Problem Solving Strategy: The forces
their moment arms are zero. acting on the board are:
1. The force of the person acting down on the board is
Concept: Regardless of the number of forces that are!
equal in magnitude and direction to the weight of
acting, if an object is in translational equilibrium and if the 1
the person.

_,_
net torque is zero about one axis, then the net torque must;
2. Weight of the board.
'also be zero about any other axis. The axis can pass through a
3. The force of the scales on the board, we call them
point that is inside or outside the boundaries of the object.
N 1 and N 2 (normal.reactions).
Consider an object being acted on by several forces such that[
the resultant. force LF = F1 + F2 + F3 + ... = 0. Figure!
describes this situation (for clarity, only four forces are
.,.,
shown). The point of application of F1 relative to O is specified,
--> .
·by the position vector r1 . Similarly, the points of application,
--> -->

-L·-
of F2 ,F3 , ... are specified by r2 , r3 , •.• (not shown). The neti
torque about an axis through O is:
--+ --+ --+ '
:E~ 0 = r1 xF1 +r2 xF2 +r3 xF3 + .... (a)
F2 y

: N,

O'----~---,~ X
-u2
mg
Mg
>+--- L-1--_,
F3 Fig. 5.62 (b)
Construction showing that if the net torque is zero qbow! Fig. 5E.18
origin· 0, it is also zero about any other origin, such as 0.
Now consider another arbitrary point 0' having a position: The first condition of equilibrium states that the ,vector
--> --> ' sum of the forces must be zero. '
vector r' relative to 0. The point of application of F1 relative• I.Fy =N 1 +N 2 -mg-Mg = 0
--> --> .
to O' is identified by the vector r 1 - r'. Like wise, the point of. or N 1 +N 2 = (m+M)g ... (1)
--> --> --> The sum of the torques about any point must be zero.
application of F2 relative to O' r 2 - r', and so forth. Therefore,, We consider anticlockwise torques to be positive. Sum of
the torque about an axis through 0' is: torques about left end of the board.
--+ --+ --+ --+ --+ --+ --+ --+ L
:E~ 0 = (r1 -r')xF1 +(r2 -r')xF2 +(r3 -r')x(F3 + ... N 2 XL- mg(L-1)--Mg = 0
--+ --+ --+ --+ --+ --+ --+ --+ --+ --+ ' 2
= r 1 xF1 + r 2 x F2 + r 3 xF3 +... -r'x ( F1 +F2 +F3 + ... )' or N 2 = [ 1 -±]mg + \:; ... (2)
.Because the net force is assumed to be zero (given that the(
object is in translational equilibrium), the last term vanishes,' Substituting the expression for N 2 in eqn. (1), we get.
and we see that the torque about an axis through O' is equal' I · Mg
N 1 =-mg+-
to the torque about an axis through 0. Hence, if an object is in L 2
:translational equilibrium and the net torque is zero about one Thus we get the forces exerted on the scales (N 1 a_nd
'axis, then the net torque must be zero_about any other axis.:
www.puucho.com
N2).
Anurag Mishra Mechanics 1 with www.puucho.com

··[!5!)~~~,it~; ;,: ,.
Dividing eqb.. (3) by eqn. (2), we get
g~xatm~~ w
tan8=-
r,;' cylinder. of weight w aitd radius ~ is to lie r,;i~~d '6n~to a F

r;
is
orizo,ri.lal step .o.if height. h..~ _shown in, Fig. SE.19 (a?; A ,rope
wrapped. around the cylinder and pulled hor!Fontally.
¾silming the ,cylinder. iioes not, slip on the,, siep;Ji.nd .(he
0~ eliminating 0 from eqns. (2) and (3), we get

The.Ladder
' N=.Jw 2 +F 2
· minimum force F necessary to raise the cylinder•, and the ·
reaction force at p exerted by the step on the cylinder.:' ' A ladder leaning against a. wall is an ideal illustration of
·a rigid body to which the principles of force and torque
C F
balance ·apply.
·~---------------,
F
. '
i
2R_:h ·
·, ' -t•
f2

L
0
1, ,,

1. t
w
(b)

...
-W=Mg

s·olution:' .. Problem Solving Strategy: At the. . We assume that the ladder. is made of a light and str,;mg
instant 'the· cylinder. is about to be raised, the reaction at Q · material so that we may neglect its weight. A carpenter
vanishes.: l;lence there are only three forces acting on the · (mass M) places the ladder on a floor with a coefficient of ·
cylinder, ~s':shown in Fig. 5E.19 (b). We.choose the pointP friction ·µi arid against a wall with a coefficient of friction
· as the point. abo\lt which to take. the torque. The torque of . µ 2 ; then climQs partway up [Fig. 5.63 (a)]. We use .the
the normal reaction, N is zero, since its line of action passes coordinate system as shown, with the origin at the bottom of
through-the.point abouqvhlch we take the torque. 'the ladder.and apply the cqnditiops for equilibrium:'
· The inomerit are d of the weight relative to tl)e point P is · Force bal8I)ce,
. d=:JR 2 '-(R:-h) 2 . . . l:F,:,;, J/'--N 2 = 0 . _.... (l)
· _iFy ·;,:Ni+ f2 -Mg= 0 •.• (2)
=.J2Rh-: h2
Torque balance ab()Ut O ,
:The monient arm of F relative to: P is 2R - h.
.The' net torque about P. Is · ''l:1o =-_Mgxk+ fi1c6s8k+Nilsin8k,= 0
. . wd-'F(2R-h)=0 .:-z.-.
.(Mgx -f2C?S8-N2sm8
. . )(-K),. ... (3)
2
~.J2Rh - h -.F(2R - h) = 0
, ·· w.J-·2Rh---~h-
2 We have used·all possible principles of dynamics and we
. -' ~-
.1
F=
2R'-h .
' .
.'..'(1) .
have only .three equations for four unknowns
(f1,f2,N1,N 2),°- The carpenter's weight .8Jld the contact
The · components of normal reaction N, can.·. be ' forces (normal ·reaction and friction) at the ends of the
determined from first condition of equilibrium. Fig. _SE.20 · ladder must have intersecting lines of action- and· must sum
(c) shows that,,the-vector sum of the three external forces is . to zero.' Modelling the ladder as· rigid, we carmqt give actual
'

zero. · solµtion. As we have learned ·that if the vector diagram


'<' F = F ..:N cos8 = 0 .: or
· "-x Ncos8=F ... (2) representing the resultant of several· vector~ is a closed
l:F~ =Nsin8-w = 0 or Nsin8=w ... (3) polygon then resultant is zero. Fig. 5.63 (b) represents two
. . . r . ·, possible solutions out of many.
www.puucho.com
Anurag Mishra Mechanics 1 with www.puucho.com

RIGID BODY MOTION

¥
'1ncdonal force 1 of the floor· oil
ihe ladder to balan~e~~;
/and thefrii;ti~nalforce ~ limited byµN. Fo~ safety, at the. top,
of the ladder( x•= lcose), the carpenterrequiresµj 1:ane e:: 1 ·
- -- ------ ' .

r
· I sin 8

- - - - ---·--· ----
!. -- --
Fig. 5:63 (bJ . J I
~_,Ex.gmeJ,~J 20 [;> I
I
r-- - --- -· --· -~ ·-----· - - - - - - - - -
1Fig. 5.63 (a) shows a ladder with a carpenter on it. In whatj
:ran.ge of angle e mey the carpenter safi.elJ,. use the l.adder if]
,t~ere is @ ft:i.s;tiQn between the ladtler q_nd_the_!Yall (µ.2 = .Q)?c.J l__. --·----------....J,,,______ Fig. 5.64

Solution: The ladder should not slide along the floor


and the limit of safety implies that the friction force· on the
floor reaches its maximum value. IA ladder rests on' the fl.oar of iz room, leaning agains·t a walL If ·
Withµ 2 = 0, f, = 0, substituting into eqns. (1) through the coefficient of.static friction between the ladder a11;dfl.oor is/
(2), we may solve for the other forces. With no friction on lµ 1 and that between ladder and wall is µw; what is the!
the wall, there is only one solution, no matter how the minimum angle e that the ladder can make with the fl.oar if1
ladder bends. · the ladder is not to slip? Assume that the mass of the ladder isi
From eqn. (1)
From eqn. (2)
Then from eqn. (3)
N 1 = Mg
N 2 = f1
Mgx N2 =
... (i)
... (ii)

··: (iii)
~.
distributed unifprmly,
~~,i
I,

I
'• 1 I
!sine N i;
w l !.
For static friction: f1 :,; µ 1N 1 .•• (iv) '
Combining these relations,
I 'I
M_gx = N 2 = f1 :,; µ 1N 1 = µ 1Mg ''
i
lsme
So at the top of the ladder, where . I' ''
I
X
= cose, Lsin a· .,'''I' '
1
tane;:: __!_
µ,
In this case contact force exerted by the ground is i
_, _, _, I
F, = f 1+ N 1 .
The force triangle OAB is similar to the Mg- I
I
f,_oc_ x II
geometrical triangle OQJ, so N, - CD - (l sine)° , ,-- ··:c L
. ~.bcos 8 4---2LC0s 8---..t I;
. 2 : :
Concept: As the carpenter climbs up the ladder, (a) (b)
the torque due to weight about the base increases. To balance __ J_-
Fig. 5E.21
\ ' -+ "'
:this increased torque, the normal force N 2 of the wall on the · Solution: Fig. SE.21 shows the force diagram of _the
,ladder has to increase. But force. balance requires
L._ --··- -- - - www.puucho.com
·the ladder. The weight of the ladder is Mg, it acts at the centre of
Anurag Mishra Mechanics 1 with www.puucho.com

j 452 ,----::-----,,&._"·',
L._ _ _ _ __....--..__~--------~----~---~ MECHA~l[!_J
. gravity. Anhe ladder is uniform, the centre of gravity is at its
-~id point, _a· distance ½' from either end. N f and' N;., are
also
2
·(L )
h = -cos0 tru\.<i>w = -µwLcos0,
2 .
1
_Substituting these in eqn. (6), we get
nqrmal reactions at floor and wall respectively. We wish to
know when the ladder will just not slip so that the friction - .!L cos0
force ai:·the floor is.f1, = µ 1 N 1 (miµdmum_value), directed µ / = -~2,,__ __
horizoritally towards the wall. Similarly, the maximum static t L sin0 + .!µwL cos0
2
frictional force at the wall is directed vertically upward and
-1 1-µwµ'f
· has magnitude f w = µ ,.N w. · or e = tan
For equilibrium, the· horizontal and vertical forces are 2µ!
balanced .
.:,i :EFx=µ1N1-Nw·=O ... (1) kEE-~am~~
r--·-,--·--·---------------- -·- -·----- - - - - - ~
LFy ""N1 -Mg +µ'WNW;" ,q ... (2) ·A block of ~eight h is projected .along a rough su_,face of,
Taking torque about pomtB, we get the'torque equation lcoeffident offriction µ. Find the point of application of the
_-Mg(½~os0)+Nw(Lsin0)+µwNwCL_cos_~)=O ... (3) I!normal force on .the block for. µ k T'- 0.5.
. - , ,

These three equations have three unknowns (N1 , N w, 0). . N ®3 l·


. solving
On . for 0, we get 0 = tan-1 (1-µ w µ f 'J @ 't'N

.
\ . . ' ' . 2µ1 . '
, Method 2: For calculating . torque, the point of
application should be such that it leads to a zero value for as
-many 'individual torques as possible. A force always
· produces_ a zero torque along its line of action-the f
w
straight .line on which the force vector lies, because the
moment arm is zero for such a choice. Fig.-5E.22
------------'----- -· __,...~-------'--~__,;J~.
Tiie torque produced by the gravitational force Mg on
the ladder is zero about an axis chosen anywhere alon:g the Solution: Problem Solving Strategy: if Wt' t¥e
vertical line passing through the centre. reference point at CM, at any instant torque of weight is
zero. We need to consider torques d1,1e to frictjonal and
1
R1 is the resultant of N 1 and µ N 1 \ it acts at angle q,1
with the floor. Extend the line of action of R 1 until it crosses
normal forces only. ·
Let the line of action of.normal reaction be at a distance
the line 'pf action of Mg as shown in Fig. SE.21. The
x from CM. Then ·
intersection A of the two lines of_ action is a point about .... .
which neither of the forces produces a torque. The line of lCM =Nx-fh=Nx-µkNh or x=µkh
action of R 2_; the resultant of N w andµ wN w', must a!so•pass Toppling of a Box on an Inclined Plane
through A, otherwise R 2 would produce a non,zero torque
and the static. condition will not be satisfied.
From the triangle· formed by .force vector R 1 and its
components, we have
µJNJ
cotq,1 = - - = µ 1 ... (4)
NJ
· Si~larly, from the triangle fi;>rmed by R2 and its
. components, we have
N -
tan<!>w = µ;:;. w =_µw ... (5)
w
The· first .of these triangle~ is sinlilar to triangle BCA,
8 = 30°
shown in figure; thus ·
L t Fig:' 5.65 (a)
. . -cose
flt= cot<!>~= 2 CA. - ... (6) .. '-------------
A force's line of action is not always· predictable! For
example, the forces acting on the box in Fig:. 5.65_ (a) are its.-
As . CA_=Lsin0+h,
www.puucho.com
Anurag Mishra Mechanics 1 with www.puucho.com

1 RIGID BODY MOTION 453


..., ...,
weight W, friction f and the normal force exerted by the so W sin(~)= W cos[(½)- x ]-
...,
ramp, N. Weight acts at the body's centre of mass. Friction
The above equation may be solved for x.
acts along the surface where the crate and the plane are in
contact. Normal reaction is a contact force. It does not act at Concept: What is the critical condition for
a single point, but it is distributed on the surface. Actually, it toppling, i.e., the effect of increasing inclined angle?
is the resultant of a large number of small' normal forces ...,
acting all over the base of the crate. Their net torque about The point of application of normal reaction N is
the CM balances the torque due to friction. The normal important for the equilibrium of the crate. We discass torques
forces are not uniformly distributed across the base, but like about point O on the lower edge of the crate. Because the line ·
..., . .... ....
weight. N has an effective point of application that may be ·of action of the frictional force f passes through 0, therefore f
.... ...,
determined by taken Crnet = 0. exerts ne torque about O and the torques ofW and n balance.
. 4
To keep the net torque zero, the torque produced by n also
(E~9m.H,1'7 ,12-;-i;> decreases; consequently ri's line of action also comes nearer to
6. (The crate presses against the.plane with a greater force on
The crate in Fig. SE.23 has dimensions m by hand the ramp is
inclined at e to the horizontaL Find the effective point of
....
the lower half than on the upper half) In the critical
..., .
equilibrium position, -both W and rt pass through 1J and the
'application.of the normal force 1L
crate is balanced on its edge and the box is jast about to
(//2)--x
'Line of ..., topple.
action of n T,
®

Line of
action off

Fig. 5E.23 0 ...,


w
Solution: Problem Solving Strategy: The ·crate is e = so 0

in equilibrium. By applying force and torque balance, we Fig. 5.65 (b)


may find where the normal force acts.
- ---,
We choose the x-axis to be along the slope and the y-axis
to be perpendicular to it. The sum of force components in :--§~2n=x~t~. 1
.~~ _,;>
each direction must be zero. A car of mass m travelling at speed v moves on a horizontal
0=:EFx =f-Wsine
track. The centre of mass of the car describes a circle of radius
and O=:EFy =n - W case r. If 2a is the separation of the inner and outer wheels and h is
We consider the net torque about the CM of the crate.
the height of the centre of mass above the ground, show that
Three forces exert torque: · the limiting speed beyond which the car will overturn in given
(1) The weight acts at the CM and so exerts no torque
by
about it. 2 gra
(2) The friction force acts along the surface between V _;: - . -
_h
crate and ramp and exerts· a torque fh, out of the N, N2
2
paper (Fig. SE.23). G X

(3) The normal force acts at an unknown distance x '


from the downhill edge of the crate and exerts a ~
torque n[ ( ½")- x] into the page. 1, 1,
I i

mg

:~;~o:1~(%): ~{(½)°~ }]k x (out of the Rage) ..


....-2a_.,
Fig. 5E.24 (a)
From force balance, we have f =W sine and
N =Wease;

www.puucho.com
'
Anurag Mishra Mechanics 1 with www.puucho.com

1454
Solution: Fig. 5E.24(a) shows the rear view of the car a
µ<-
that ·is turning towards left. Frictional force on the tyre h
provides a 'centripetal force while turning.
From Newton's second law,
· mv 2
.LFx =fi +f2 = ~
r
... (1) IA h;;,o~~al force P applied to a h~mogeneous rect~11gh1~~:
l6lock of mass m, Width b !Ind hei,;htH. The block moves withi
. · LFy·=Ni +N 2 -mg'=0 ... (2)
!constant velocity; the coefficient offriction is µ k·, · i
. From ·condition of rotational equilibrium, sum ' of '(a) What is the greatest heighth ac'which the force P can bei
moments about centre of mass G must be zero. j . applied s<H~ilt the block will slide without tipping over? 1
L~ '= C!i + f 2 )h +Nia-N 2a = 0 ... (3) (b) Through which point on the bottom face of the block will .
. a the ·resu(tant of the friction and normal forces act if
fi + fz =,CN2 -Ni),; ... (4) H· · . :
h=-? . '
On combining eqns. (1) ·and (1) we can elhninate 2 ' '
CJi + f 2), 'to obtain (c) If the.block is at r~t an~ co~fficient of sta~c[riction, is~,, J
· ·mv 2h
Ni-~=-.-
· · · ar
.,.W· L;~~s'i!e v~no:'~.t~n~-~o~-w~~ch ~:z~~ng. o_r_~z'.'pzng!
Solution: (a) In the absence of any externai' force, the
*~Nciw we can ;olve eqns. (2) and.(5).simultaneously..·to · . normal reaction N passes through the centre of gravity of the
block; when force P is applied, normal reaction shifts in the
· ·· · hm~ 2
2N, =mg---· direction of force P. Since the right part of the body is trying
ar to lift from the surface, at the instant of tipping over about
. hmv 2 the edge the normal reaction passes through the edge. From
and·· 2N 2 =mg+--.· ·•··.i
· ar the conditions -of equilibrium,
·The inner wheels·. will. leave the ground· when/Ni LFx_=O, f-P=O or f=P=µkmg
beconies zero, i.e., . LFy = _o, N - mg = o or N = mg'
. · hmv 2 ·.b · mgb mgb b
mg=-- L~=O; Ph-mg-=Oorh=--
r 2 · 2P 2µkmg .= 2µk
2 gra . Al~ernatively; the resultant of friction forceµ kN. and the
or V =-
.. h .. norma,I'reaction ni.ust pass through the same point through
We can solve this problem from the reference frame .of whic;h p passes, since three coplanar forces keeping a body
car. At the · instant the car
begins to overturn,' the forces · in equilibrium pass through a common point, i.e., they are

)~b~
Ni andfi are zero. Taking moments about 0,d'onrotational concµrrent. ' '
equilibrium,·
----'------,. . -, '
, .

'!
.

· , mv 2 , ... - -·
L·~=-·-xh·~mg ~a= 0
·- •. ·_r. B . I
....... P ·I
.· z gra Ii m P. . ~C.G. ;
V
..='h- 8

\ -~:, ! .1.
.. '
'" / : H/2 /
Note, that i{ frictional: force.
is not sufficient .. to provide . mgJ 'i.

I• • -
centripetal', , acceleration,,- ;'9 -
;' b/2
sideways skidding. :wi!J'.occur ,
instead of overturning. .L. .. (•)
. '" ,
. ,:·,
'Fig. Sf:,~5 .~
·,b) _·_ .J,
.
Ji+ f~ ~ (Ni +Nz) · . -·--- ~:;ii:;..- - -
Equality sign · holds when , From Fig. SE.25 (a), (b), - • .. 'Wlj>
skidding starts.· mg tane = µkN .; o/2
'I . . 2
. mv · Fig. 5E.24 (b) N h
·and fi+f2 =-,- ·
, r or· h=....k_
2µk
, Therefore the limiting speed at which skidding ensues is
given by v 2 +µgr. · . (b) In .this situation, impending state for tipping
~ •I -
cannot be .assumed. Hence .normal ,:eaction doe~ not pass
· , If 'fue ca, 'is to skid rather. than ~verturn,

www.puucho.com
Anurag Mishra Mechanics 1 with www.puucho.com

Li/§ BODY MOTION. .' 45~1


through the edge; we assume it to act at a point Cwhich is at b
h max = -
a distance x to the left of C.G. 2µ
Torque of all the forces about C.G. must be zero. which is independent of weight mg, height H of the body
(µ~Nx~)-N·x=O . and applied force· R
The cases that result from eqns. (1) and (2) are:
µkH Case 1. If h < hmax, P < µmg, neither tipping nor
or x=--
2 sliding occurs.
Alternately we can proceed as in part (a). Resultant of - Case 2. If h < hmax,P = µmg, no tipping, sliding niotion
µ kN, N passes through C.G. along with mg and R impending.
Therefore from geometry of_ Fig. SE.25 (c), Case 3. If h < hmax,P > µmg, no tipping, sliding occurs ..
H~ =tan0 = µk; . so. x = µ~H
Case 4. If h = hmax,P = µmg, both· sliding and tippilig' i ..
2 are impepding. ·
Case 5. If h > hmax, the body will tip over for any valu~
ofM. !
' •• ' '
If µ ·;,: !:.__, _onset of tipping occurs and tipping edge
C.G .• 'p mg
. remains stationary.
/8
·./ . · H/2 .. If µ < .!....., onset of tipping o~curs with sliding. ~f edge

-~µ,.N~',f--..J...:m~g..J·_
x~
:J ..
<J,
mg .
. on surface.
/8·
•. (c) -

:1,_,_ b --,--.1
y

.Lx· L;
y

GcY1inder. rests o~ ~ horizontal ra~ating _dis~ as shown iur1te!


Vlgure. 'Find atwhaumgular velocity, Ol, t!J~_cylinder fa~ Pffi

H
C :h/~
.
......:... p

-
.
·------
..
.
p

E
!Iisc, _if the distq~ce between_ · ~he ._ax~ of the .dis.c a,nf1··
miler is R a~d the \effici~nt ~ft~on µ > ~' whe~~ D: is ·
diametet;.J)f.t!JLcy_luyl~d h•is its height. ·, ~: ·.
.. h ' .
p~
,,

1 lm•~: .Solution: The centripetal foice·that keeps the cylinder


-~-'mg
N f (d) -~- ,_mg at rest on the disc is the frictional force f. According to .a
hon-inertial observer on the disc, a pseudoforce on· the
cylinder reacts with an equal and. opposite force F, which .
Flg._SE.25
·sometimes is referred to as the centrifugal force, · · · ·
F=Mro 2R- ..
(c) As the point of application of. force is raised..
· higher, ·the location of- the ]Jne of action· of the normal ·-, D12
reaction N moves to the left. In the limiting case (when the . ___,;··:.-1h
bloclc is about to tip over), c =!?..The normal _reaction passes -~--F•·
•. . 2 . '' p _'.:,(~~:::!::~·
through the edge as shown in Fig. SE.25 (d). R
Equation for impending tipping:
. ' b
L't=O=Ph max -mg-=0 .
2 '-'"-·-'a_>.:..w_-_.c...c:·.:...:-';,.'""~·_ _<_bl_w___..--':j,
-· Fig, 5E;26
h = mgb ... Ci)
or .max 2P where Mis the mass of the cylinder. The cylinder can fall
Equation for sliding: off either by slipping away _[Fig. SE.26 (a)] cirby tilting
about point P [Fig. SE.26 (b)],. depending on_ whicliever
LF,' = 0=:-F°+µN = 0 takes place first. The critjcal angular speed, ro 1 , for slipping .
LFy=O=-mg+N=O : occurs when F equals f: ·
or P = µmg ... (2) F= f . or MrofR =.µgM
If sliding and tipping are equally likely to occur, we can
eliminate P from eqns. (1) and (2) to get where g is the gravitational_ acceleration.

www.puucho.com
• ·A
Anurag Mishra Mechanics 1 with www.puucho.com

Hence
I
F tries to rotat~ the cylinder about P, but the weight W
. opposes it. The rntaticin becomes possible when the torque· .:1
created by F is _large enough to take over the opposing ' '
'
· · torque caused by W.
h . D . . . D
F-=W- or 'Mro 22R = 'Mg~
2 .:, 2 ' 'h:
.' '
giving Cil2 = ~ . ..
"'', N,•
' ' . . . , . , D . . .• f.
Since we are given that µ > h' we see that ro 1 :O: ro 2 a~d L~~:..:~";_-~:-·-~-~"__Fl_!J_i&::f!::___ ·"-(c'-)-~'i!£,
(b) Total ·reaction (R) at the edge is vertical and is equal
· ' . the ~linder falls offby rolling over at ro = '!l:i- to Ry·· yve .resolve·. it into components parallel and .
!i~:.Xsd~~!lf~
- - •77;;;,,'

27' ~ perpendicular to the cart. ·R11 = Ry cos8 represents the

_Solution: (a) Fig. 5E.27 (b) shows the forces acting on


the cart'. ·The reaction CR) at the edge of the. wall has been '·'
iesolyed-into components along and y-~s. x: . r·~ ,":.,· "·

From ·condition 9f equilibrium, .. ------ :


0+i!a/3~ '
' ''
:EF,; =Rx,= 0 :... (1)

or
:EFy = R_,; + N - w = 0
, Ry ~w-N . _ · · ... (2)
. - )] .

. . Taking'torque aboutedge of the wall, moment arm for N . /.-- : ./{i ' . · ·Fig. s:.2a f.
ish tan8 and for wish tan0- lsin8; we l:iave . '-~--,,__,i,~-------L--'--'-4-->J
·L~ ·= N(h tan0) __; w,(h tan8 -1 sin0) =; 0 ... (3) · Solution: The weight of the removed cylinder is 15 N.
N'= .If a symmetrical hole was drilled on the· other side, the
or .. . (1...:
' lcos8·)w-
h. '; uniform cylinder would have zero torque about P.' This
• -, • 0 \ ' r !

implies that the torque due to excess weight (15 N) on the


.. :: . and frol)l_ eqn._ (2), . .,

.-
. ·, R
. y
=w-N = (I .
cos_ e)w
h .
other side has caiISed instability. Hence· for equilibrium the
torque of T must balance the torque due to excess weight.
T(2aJ=(ls{¾a} ·
www.puucho.com
Anurag Mishra Mechanics 1 with www.puucho.com

I.RIGID BODY MOTION 4571


or T = SN Taking torque about the bottom of the ladder yields
We have considered torque about P so that torques of l
hN 2 --w-dw = 0
unknown forces N and fare zero. 2
l
k,~xom~l,e.,Jzgl,~
-,·.v-~~ N2 2
-+d
,.;;;;iii, .. - - -
or -=--
w h
IA light kidder rests on .the rough floor of a room,· leaning l
against a smooth walL The ladder touches the wall at height h -+d l
From eqn. (3) 2-
µ=- or d=2µh--
above the floor. A man climbs up the .ladder until the base ofi
1 2h 2
!the ladder is <,m. the verge of slipping. T{le coefficient of static
'.friction between the foot of th~ ladder cind. the floor is µ, . Work Done Due to Torque
'(a) What is the. horizontal distance moved by the man? ...,
Suppose _a force F acts _at a point whose distance from
(b) Solve part (a) if the ladder is uniform and has the same, the axis of rotation is R, as in Fig. 5.66. The work done by
!___d_isJgn,f)frq_m.the wall. ·
e
! w.eight as th. m.an ru:,d the base of the ladder is at
. ________
aj' this force is

Solution: (al When the ladder is on the verge of


slipp~g [Fig. SE.29 (a)],
fmax =µN1
. ...,
where d I is an infinitesimal
distance perpendicular to R
,- --- - -
_ , -

For e9uilibrium, LFx =fmax-N,"=0 -with magnitude di = Rd0 and F.1...,


or fmax=N2 ... (1) is a component of F
LFy=Ni-W""O perpendicular to R and parallel
...,
or N 1 =w ... (2) to d I (Fig. 5.66).
· fmax N2 But F.1 R is the torque about I
From eqns. (1) and (2), µ=-=- ... (3)
N1 . w the axis, so i

Taking torque about point B, N 2 xh-dxw=0 ... (4)

or N2
w
= !! .
h
w".'J-rda
If torque is applied to a
body over a finite angular
L.
Fig. 5.66

displacement from 0; to _0 1 , the work done by the torque is


d · . expressed by integral.
Hence· µ=,;:
·-------1 .
W = J?-rd0. ... (1)

Ii ' -
A A i · The work-energy theorem applies to rotational as well
N2-,--,,.jT N2 ---,.,1 as transl~tional motion. Suppose a torque T acts on a rigid

I N, l N, h. I
body whose moment of inertia is I. We use the equation
-r = Ia. to write for work done in the form
J.,
i ~1~__;._____Jl
01
W =
·
.
, f (friction) .f max
!j
Ia.d0. ·
We now manipulate the factor a.d0 in the integrand. We
have
lot- d..,..;j ~d------1 . · dro ·d0
a.d0 = - d0 = - dro = rodro.
(a) _·_<b_,,__· __· _j.t dt . dt
We can thus write the equation in an integrable form.
Fig. 5E.29
- 1
. (b) For equilibrium [Fig. SE.29 (b)], w = J"
ro1
1
Irodro
· ;
= ~r[ro
2
2
]" ·,
LFx = fmax --N2 = 0 . Olj

N which yields W = ~I(0>J -rof) = Kt -K; = MC.


µN1=N2 or µ=-2 ... (1)
. N1 2 .
This ·is the work-energy theorem in rotational
IFy =Ni-2W=O mechanics .
or N 1 = 2w .. : (2)

From eqn. (1) µ -- -N2


2w
... (3)

www.puucho.com
Anurag Mishra Mechanics 1 with www.puucho.com

l}ss
Power
We begin with the equation dW = ,;de. Using the
·---r·~=-··~ .. _:-,:: __· _ -"-~;"":r~""i
definition of power given by the equation P = dW, we have
dt
The drum of;a .\ilinch has mqss /IA a~d radiusR. A CO?'il!'.oi)ndj
around the d,-um suspends alocidpf.mass m. The entire:C11lile;
de
p = ' t - = 't(O has a limgthfrand niass per unit letigth 'J..., with a t6tctl'm.assl
dt m, = L'J.... _7'!re foa.d begins to jci.U'all toward t~e. gi:9\11id,J
Compare with the translational form given by the
--+-+ --+ .--+
funwinding_ the'cable as. it goes. J!~w
fes~ /s th~ loctd!17i~ing
equation P = F· v. In the special case where F and v are kzfter it has f.aJJen· a dist@.£e d?. .,. · . ' , ~,,.,.L~'.c'..-.J
parallel,. this expression becomes P =Fv.
Solution: Problem Solving j Mass.'=,M;·;,:-1
Proof of the Parallel-axis Theorem Strategy: Assuming the pivot to be :· ·· 1
The kinetic energy of a system of particles is the sum of
the kinetic energy of the motion of the centre of mass plus
the kinetic energy of the motion relative to the centre of
frictionless, mechanical energy is
conserved. Choose the initial potential
energy to be zero. Then' the total
,l
!

l
mass. mechanical. .energy is. zero. When the ,t
. 1 ·2 load has fallen a distance d, its potential 1
K= MvcM+K,.1 ... (1)
2 energy is ,-mgd (Fig. SE.30). The centre '' ,~_·.l
of mass of the hanging cable (mass A.d)
Consider a rigid ];:,ody rotating with an angular velocity ' d. . . ·,
ro about an axis a distance r from a parallel axis through the has dropped a distance -, so the I
centre of mass as shown in Fig. 5.67. When t)ie body rotates . 2 .::~-;;' \:/ i· __.,, '. [

through an angle de measured about the axis of rotation, it potential· energy of the cable is Fig:_ 5~~L:..:J
rotates through the same angle de measured about any other -(A.d)g(1} When the load is moving at
parallel axis. The motion of the object relative to the centre
of mass is thus a rotation about. the centre of mass axis with speed v, the drum is rotating at angular spe~d (~} Since the
the same .angular velocity' oi. The kinetic energy of this
relative motion is hanging part of the cable moves with speed v and the cable
r_,,~-.--
1 ~ r' ....---....::
..
does not stretch or become slack, the entire cable must move
.. J. at·speed v,
We find v from the conservation .cif mechanical energy.
pr· Assume that the drum is a uniform cylinder of moment of"
··---
' . . . 1MR2 . ··
'······· '--rr..,.;;;··.··~,;'·d mert1a-
2

Fig. 5.67 ___


3 ..,
From, conservation of mechanical energy,
E1=E;=0
·
K,.1 =-ICMro
1 2 K1 +u1 = 0
.. 2 The -total potential energy of load and cable when._the
The velocity of the centre of mass relative to arty point load has fallen· a distance d, ·
, _on the axis of rotation is v CM = rro. The kinetic energy of the . · d ,.,., d' m,gd 2 .
· motion of. the centre of mass is thus · · · U=-mg -(-,g-=-mgd--.--·
2 . . . 2L
· .!Mv~ = .!M(roi) 2 = .!Mro 2r 2 The kinetic energy of the wi_nch in terms of its moment
· 2. 2 . 2 of inertia I and angular speed ro,
2
The reiative energy is .!rro . eqn. (1) then becomes
. - . . 2· ,· \ .. : . , K = .!1ro 2 = .!Mv 2
w 2 4
1 2· 2 . 1 . 2
K=-Mror +-JCMro From the non-slip condition,
2 2
1 2
='-(Mr +ICM)ro =-[(O.
'2 ·1 2 we have v =Rro and I =,!_MR 2
2
·2 '. 2 ·
The kinetic energy of the cable and of the load,
where _·I=Mr 2 +IcM . . . 1 i l 2
K,.+K 10,d = m,v + mv
which is the parallei-axis theorem: · 2 2
Total final kinetic energy plus potential energy,
. . . . • . 2
·1 2 ' 1 2· 1 2 m,gd ·
.c.Mv +-m v +.-mv · -mgd---·= O
4, 2' 2 · 2L

www.puucho.com
Anurag Mishra Mechanics 1 with www.puucho.com

[ RIGiij BODY-MOTION
which on solving for v yields,
.-----~~
2
4mgd + 2m,g d /L
V = . I-~'-----"''----'----
M + 2m + 2m, IA· wooden frame consists of seve~· dzin rods as shown in Fig.
ISE.32. ' -
~E~qt;m,!i?,M~:::fTil> (a)· Calculate the moment of inerna of the frame for rotation
- -- - - -- ·- - .__ -- --- ---- . -- ---------- j about its lower edge. -~ake the mass and length,of eachr~d/
'In Fig. SE.31 as shown, a certain mass M is free to slide! ; to be m- and I respectively. - - I

'without friction on a horizontal table. This mass is connected :(b) If the frame starts from vertical with zero speed andfallsl
by a light thread to a mass m that hangs over the edge of thej i over, how fast.is the top edge travelling just before it hits]
Itable. The connecting thread passes over a frictionless pulley/ .___,J!J_e_gr_ound? :' · _____ __J
•in the shape of a disk with radius Rand mass "!p· CalculC1te1 Solution: Probl!!m Solving Strategy: When the
Ithe velodty of the string after the hanging mass has fallen a frame is vertical, all of its energy is gravitational potential
dist~nce h sta~ng from rest ___ - - - - - - , \ energy. The gravitational potential energy· of an· extended
.object can be calculated from Mghc,., where htM is the
i 1---~ I height of gie centre of mass. The 'centre of mass of the frame
i r;s::;:;::~~~ is located at a hei~ht rif .!:: above the rotation axis. ·
, 2 - ,

.\ .. =,
--- - -- ---~-=-=-=-=.-==-==-
mLJi
Solutio11: Problem Solving Strategy: We employ
If the, frame simply rotates
about its base, 'the initial
- potential energy turns entirely '
into .kinetic energy just before
the-frame slams on the ground.
the principle of conservation of mechanical energy. The Five of the , rods -rotate
potential energy 'Of mass m is converted into kinetic' energy about their ends. Therefore
due to the translation of both masses plus the kinetic energy each rod has a moment of · - Fig. 51::32
-due to the rotation of the pulley. · inertia ,!_ ml 2 • The top
From the conservation of mechanical energy t)ieorem, 3
we may write hori20ntal rod bas all of its mass .located at distance I from
the axis ofrotation and hence has a moment of inertia ml 2 • ,
U,+K,=U1 +K1
The· rod at the bottom has all its mass concentrated at a vei-y
mgh = .!.mv 2 + .!.Mv 2 +.!.Iro 2 small' distance from the rotation axis so that its. momeht of
2 2 2 inertia about this axis is I\egligible. Thus, the total moment
where v is the tr,anslational velocity of the !Jlocks and ro of inertia- of the wooden frame,-
is the rotational velo'city of the pulley.
~-- ---,-·\ ----'-------,
,
•. I=Sx(.!.mz 2 )+m1 2 =J:!m!
2
j· Concept: Note that, mass M moves horizontally along , 3 , 3 ',
,the smooth table; its potential energy remains constant, orily Thus,'the conservation of energy equation
'm appears in the potential energy ·term. At any instant the CJ,+'K,
=,Uj +K1 .
string and the masses connected 111ith it move with the saine
velocity v. Since the string does not slip on the pulley rim, the becomes MghCM+,0=0+iiro 2
tangential velocity of the rim of the pulley is also v. Thus;· iye
, jean write v = v 1 _=_R_ro_._ _ _ _ _ _ _ _ _ _ _ ___, (7m)g.!. = !(!!m1 2 )ro 2
. ,- 2 2 3 ,
M.I. for· disc shaped pulley is I = ,!_ mPR 2 ro = ~21 g _ .
2
2 · s- 12
mgh = l.mv 2 + .!.Mv 2 + .!.(.!.mPR 2)(~) The top rod is travelling in a,circular arc.of radius r-= !;-
2 2 2 2 R
its tangential Velocity, · ·
-In this expression v is the only unknowni on solving we , ,' fng- In
get
2mgh
v, = rro = 1 'fsr = ·rsg
v= ·- m
m+M +...E.
- 2
l

www.puucho.com
Anurag Mishra Mechanics 1 with www.puucho.com

1460 :MECHAM~

r- ~oncep;: Note -~e ha~ to substitute M"'


7m. Since thel
ifooter does not change height and had zero . moment of: 1 2
-Iwr -Mg-= o
Ei. = E;
L
Iinertia, so we mey be tempted to neglect its mass. flowever, we; 2 2
!must include the mass of the footer, otherwise the centre _of] Wt =~MgL = ~ MgL = {3g
ltnass ~eight of' the wooden frame W~uld ,not be i. J ]:ML2 1/T
[_ ____ '_:__ ' _:.._ ___________2_ _ ,~_ 3 -
__, .
(b) 1. Let F p be the force ~xerted bY: the pivot when the
l\$¥G-!}JJ)j~l~g~-
~ =~~~
__, __,
,- ----------- stick is vertical. Apply l: F ext = M aCM taking the upward
½ thin rod is released from rest wheh it is horizontal. direction to be positive,__, __,
!Assuming the pivot ;to be fiir;tionless, find; l: Fext =.M acM
( a) the - angular· velocity of the stick when it ';.~aches · its FP -Mg =MaCM
vertical position, and - 2. ·aCM is the centripetal acceleration,
'(b) the force,exerted by the pivot at this time.
fr =
. ·
'(c). What initial angular velocit;y is·neededfor th~ stic:/<'to v2 (½Lro L 2 L 3g . 3
I . re~h a verticai position ai: the·top of i_ts swing? aCM = __fM_ -'-~~- -Wf = - - = -g
---~---, r I_L 2 2 L 2
,~ L
ecm I ,, 2
= Mg,i,MacM '= M(g +1g) = ~Mg
. . .
-'
'
I Thus, · FP

(c) 1. Initial kinetic energy


1 = -1(1-ML 2 2)ro-i = -ML
1· 2ro;2
I K· = -Jw.
' 2 ' 2 3 ' 6
II 2. Apply the. rule of conservation of mechanical energy
Fig. SE.33 with Kf =. O and U; = 0 to relate the initial kinetic energy to
the final position
Solution: Problem.Solving strategy: . K1 +Ur =K;+U;
(a) When · the · rod .swip.gs · i:l~wn, its potential O+Ur =K;tO
energy decreases and its kinetic energy of rotation about the Mg!:= I_ML2w ,.2
pivot point increases. Similarly when it swings up, its kinetic 2 6
energy. decreases ;md its potenti.al energy increases. Since 3. Solve for the initial angular velocity
the pivot is frictionless, we use the rule of conservation of
mechanical energy. The angular velocity of the st;ick is then
found from its rotational kinetic energy. Choose U .= 0 ·
CiJ·
, = 1/T
f3g

initially. Then.. use ~. -


MgL
- w h en the stJ.'ck h angs .vertJ.c
. ally.
. . 2 , I .

(b) When the stick is vertical, there is no torque on


______ ,._______.. ..

it, so the stick has no angular acceleration and the centre of


IA rod of niass m and.length l Cis,,kept on a roug'hjJ.90rd1Z
verticalposition. The coefficient offiiction is sufficient.sh . that
mass has no tangential acceleration. -But the centre of mass the lower end of the rod does not slip over the floor. If it is
has a tangential velocity, so it has a centripetal acceleration ·
disturbed from the vertical position, itwil! rotate abqut.point
towards the pivot. We apply to the stick to find the force 0.- .. ; . . : ; . - ' - ·_'··: .
exerted by the pivot. (a) What is theforce offr'i.ction between the foot of the rod ·and
the [lo_o_ r,. when the rod makes· an angle 8 ¾it_h_. t~~
(c) We find the initial angular velocity from
conservation of mechanical energy.
(a) Final·kinetic energy of rotation
I ver.ttcal? . -. . ,
'(b) For what particular value of 8 0 does the fiii:tion·force
.
1 2 L become zero ? ·
Kr=
2 Iwr, Solution: Fig. SE.34 shows the force diagram of the
We apply conservation of e·nergy. Conservation of
rod. From Newton's law, we have '
·mechanical energy with E f = K 1 - MgL and E; ,,; 0.
2

. '

. I' ' www.puucho.com


Anurag Mishra Mechanics 1 with www.puucho.com

r----- ------.--
' RIGID BODY MOTION
L_ ----------- - --- -
461
(i) Axis of rotation passes through centre of mass of the
body, referred to as centroidal rotation.
(ii) Axis of rotation is shifted from centre of mass, but
passes through the body, referred to as
non-centroidal rotation.
For centroidal rotation,

For non-centroidal rotation,


0 where I O is moment of inertia about centre of mass and J
(a) is moment of inertia about rotational axis, to be calculated
N with the help of parallel axis theorem.
_, _,
Concept : 1, The angular momentum L parallel to oo
'' ... and proportional to the moment of inertia I A. This pleasing
result is applicable only for a highly symmetric object in
' 2 _, _,
i!Llcos 8 which the contribution to Lfrom each particle ls parallel to oo.
2
+fa sin 8
2. Many rotating objects that we encounter (like wheels,
(b)
pulleys) are symmetric and their angular momenta are
Fig. SE.34
parallel to their angular velocities. We shall restrict ourselves
" 1 . 1 ,, to two classes of objects:
"'~=mg-sm 0 =-m, a
2 3
or 3g
a= -sin . 0 as
21 '
Hence °' oo doo = 3g sin 0 d0
0 J 21 0
8
J
3
00 2 = g (1- cos0)
l
x-component of the acceleration of centre of mass, Symmeuy axis
2
l 00 1 . (a) Rotationally symmetric
aCM, =a-cos0--sm0
2 2
(a) So the static friction force is
f, = macM X )lelocity inward
3
= m3g sine ..-1.cose-m g (1-cos0).isin0
,::==J,=--',--,;!-~
~ 2 ~ _ 2
1----~~---~ Plane of
= m [ 9g sm
. 20 - 3g sm
. e] ··············--· symmefry ·
8 2
f, = 3:g [3si:20 sine]
Velocity outward

(b) On substituting for f, = 0, we get cos0 0 =.?:.


3 (b) Mirror symmetric
Spin Angular Momentum of a Rigid Body
Fig. 5.69
We consider two cases:
Axis of rotation ( a) Rigid bodies that have rotational symmetry about an
OJ OJ axis and rotate about that symmetry axis. Wheels, pulleys,
,symmetrical rolling objects are in this class of objects.
CM• (b) Rigid bodies that are mirror-symmetric about a plane
and rotate about an axis perpendicular to that plane. Tennis
rackets and cricket bats are i'! this category.

Fig. 5.68
www.puucho.com
Anurag Mishra Mechanics 1 with www.puucho.com

1462
·A rigid body that is rotationally symmetric about the axis i'. =·mvyxk- mvxyk
of rotation or mirror-symmetric about the plane Angular momentum of projectile, about point of
perpendicular
. . ".-.
to rotation
.
ax{s has angular
. , ..
momentum
-
about
. i'c·,· projection, .when it strikes 1;he ground .
the rotational' axis equal to its rotational inertia about that
axis'multiplied by ·its angular velocity · ,; = u cos9i-usin9J
... ... ,-,,------------..----
Y,
---,
L = IA o\ i
illustration: Angular momentum· of a partic).e I
in translation: Consider a particle ·of mass m moving I
along x-axis. We wish to determine its' angular momentum .!
about B, a point on y-axis.
First we determine the magnitude of angular
. .-+ . R-Range
. mom~ntum ILB I = moment arm x momentum. The direction
of. angular momentum can be determined with the help of - - - - - -Fig.
-- 5.71
- ¥<-------
right hand thumb rule. ...r =Ri.
Taking origin B, we can resolve the position vector rB ...L=rxmv
... ...
into a component rii parallel to v and a component r.L
perpendicular to v. Since rii ~ v = 0, only r.L gives . a =Rix m[u cos9i- u sin9J]
contribution to LB. We have = -muR sin9(i x J)
lr.L x·vl= Iv ...L = -muRsin0k
and LB = mrB xv = mlvk.
LB lies in the positive z-direction because the sense of Angular Momentum of an Inverted Conical
Pendulum
rotation is counter clockwise about the z-axis. .
,.. ,.. ...... ,., Angular momentum about 0,
Method 2: We can write rB = xi-lj and v =vi,
r = 1sin0l+ !cos0k
i j k
·-z · ·,; = (I sin B)roj
LB=mrB·xv=mx O =mlvk
... .
V O 0 L= rx m,; = ml 2
sin 2 0rok- mz2rosin0cos0i
as before. r-.--·c_o_n_c_e-~t:An1:1/armomentum. .~:ctori-~-;~;e;::i~~
AngularJlllomentum of a Projectile
For a body of mass m,
to. position vectqr r as well as momentum vectorp . The!
y magnitude of L. is constant but ~ direction is co~tinuouslyli
...
varying. As the particle swings, L vector sweeps out a cone.
.
The z-component of L is constant .but the horizontali
. ___ .• :.'. _·_'{J•...•... -······ component travels, around the circle with the particle. , : !
. - ·---- -- --- - - - , I
O="----~~~_;;__,__ _......,._ X zt I
I i
'

l<-----x:---;...,...i -->
J;,ro ./ L I
Fig. 5.70 ' .· Particle ·
;, I
-+ -+ . -+. ,. I
Lorbital ~ rCM.xmvCM
Position vector of l'rojectile, relative to point of
projection ·
... . .
· r"'xi+Y.l
y
-->
r

Velocity vector of projectile,


-+ ,.. A A ,_
0
v = vxi+vyj = (vcos0)i+(vsin0-gt)j Fig. 5.72 (a)
' - - - - - - - ' - - - - - - - - - - ~ ........................ - ---~J -· __ 1
Angular momentum relative to 0,
-+ -+ -+ •A A A A

L = rx mv = (xi+ Y.J)x m[vxi+vyj]


www.puucho.com
Anurag Mishra Mechanics 1 with www.puucho.com

t RIGID BODY M!)TIDN . .463!


I; · I' y
j -->
L
Particia •
-->
X V In accordance with Newton's second_Iaw of motion, the
Velocity time rate of change of the momentum of the ith particle is
vector
inward
....
--> equal to the total force Fitot~, on the ith particle.
r
....
dL -> ->
Hence - = ri x Fitotal
dt i
L
0
l Fig. 5.72 (b) Each term in this sum is the total torque on each particle
of the system.
The Time Rate ·of Change of the Spin Angular The total force acting on any particle of the system can
Momentum be written as the sum of external forces and internal forces.
. The total spin angular Now we show that net torque due to internal forces is zero:
momentum of a system is ·l Conc~pt; ~orque due to internal force between any two
the yector sum of the ! oint particles. · . · ·
angular momentum of all
Consider two particles as shown in Fig. 5.74. From

~-'"'~-·~1v
the . individual particles
Newton's third law, t/.le forces exerted by these particles ·are
' that constitute the system.
Take the origin · of the
coordinate · system, as
shown in Fig. 5.73.
The . spin angular
momentum is Direction o_f. spin
_, . _, 0
L= I,L;
i Fig. 5.73 Fig. 5.74
_, ....
F:,, 1 = F~ 2
_, The sum of torques of these forces about origin b is
Where r; is the position vector of the particle with --+ --+ --+ --+ ---+ --+
~, + ~, = r1 x F:,, 1 + r 2 x F,, 2
momentum P;. S~bstituting m;~; for the momentum, we ---+ ---+ ...:.. --+
·have = r 1 x F:1, 1 + r 2 x(- F:1,,)
.... .... _,
= (t 1 - r 2 ) x Fz 1
_, _,
The time rate of change of the.spin angular momentum The vector r 1 - r 2 is alonf the line joining the two
is _, . . ....
.... !Particles, so Fz 1 is either parallel or al)tiparallel to F41 thus
dL d ->. -> ,-4 ---+ --+
-.-= "'-(m-r-xv-) (r 1 - r 2 )x F:,, 1 =0
dt ~dt l. I l

Differentiate the vector product; be cautions to preserve So the internal forces {torques) cancel in pairs.
Therefore the internal torques between every pair ofi
the order ofvect:; = ·. _._m-(dr; x-;+ r-x d~;) particles in the system vanish. .

~
The absence of internal torques means that the total
dt '7' I dt I l . dt torque ~n a system is caused only by a force external to, the1··
D'_stem itself,____ · . .
....
dr, --+
. --+ --+ Thus the time rate of change of the spin angular
Since - ' = v ;; the first term in the bracket is vi x vi ·momentum of the system is equal to sum of the torques of
dt
which is zero. the external forces of the system (the total torque on the
system due to external forces):

www.puucho.com
Anurag Mishra Mechanics 1 with www.puucho.com

[464 •MECHA~
..... ...,
c-+ dro Here Lp is total angular 11]-0mentum of the system about
't total ext =I CM - dt ..., ...,
· point P, 'tP is torque about P and ap represents acceleration
If the moment of inertia I CM is constant, from equation
_, ..., about point P. The most general. equation· relates rate of
L=ICMro change of angular momentum about any ·point E Torque
..., about P is equal to rate of change of angular momentum if ·
..., dL .
we have
'
't total en = -dt second term.in the right hand side.of the above equation is
zero.
or in terms of the angular acc_eleration,
...,
't total ext
...,
= I CM ex ,·
...,... ,,,. ;.,,,,, ""' .. ,,. .;,,,,,;,,.,,.,...I
if~ny.. o~e of_ ~he following three.~?.ndi.tions on the refe~e_n_.Fe
point P.is met. · . .• · •. •
Total Angular Momentum and Torque (about . a . ' ' .- ..
general point) ~ -· ·,
1. The acceleration ap of the. reference ·point P is zerco,
Analogous to Newton's second _law of motion, there is an Le., the.Tefer~nce point Pis fn.aniizertial refe;~ncefram,e;,
analogous relation between the total torque and- the time 2. The reference point P is the cen(Te of mass of the system
rate of change of the spin angular momentum. Now we will ofpdrticles, .In.this case; the ~ummation ,vanishes befaUS~ of,
generalise the same relationship for the case when both spin the definition'of th_e location oftne centre of mass. '' .
and orbital motion exist, provided certain conditions are
met. ·
' ·•• '"""7.
.. . I
'
· Le.,.ifFis.the'centre ofmass,_then rCNi = Om, and,s,twe
have

. ...,
3.The :acceleration ap of :the point P is in the same
direction as tile v~ctor
...,
", -'rmiri =.O
i
Flg.·5.75
· i; e., if a,cceleration of point P '!s parallel or antipara!leUo
A system of particle is shown in Fig. 5.75. The origin at the vecior (/roin PJ locating the ~entre of mass of the system.
0 is located in an inertial reference frame. Another reference ThenJhe nine rate of change,.of.the angular mom;ntl!m
point-is at the pointP, which mayor may not be the centre of of the system"(about P) is equal to•the 'total torque (abofltP): '
mass of the system of particles.
...,
,. dLP '"""7 ., -~
Position vectors are related. to ·each other by the vector -.·.-='tp =Ia
dt .
sum
....,. ~ -) Angular Impulse-Angular Momentum
ri = rp.+r i 1
Theorem
..., ..., ..., By integrating 'tnet = Ir:t with respect to time and using
Rewrite .this as r'i = ri- rp •.• (1)
the expression L =Iro for angular momentum, we can arrive
1P =position vector of P at an angular analog of the impulse-momentum theorem. In
..., . analogy with the linear impulse ·J =JF dt, we define tl)e
r; =position vector of ith particle with respect to origin rotational impulse of a torque as : •
0 <i>=J?-rdt
..., '
r 1= position vector of ith particle with respect to Following the procedure similar to that used for the
reference· point linear case, we calculate the rotational impulse caused by
Without going into any complicated derivation, here we the net torque. In the resulting integral, we substitute Ia for
state a general result. the net torque and perform a change cif variable.
' ..., "''l'n,et = f'1 -r dt = f'1 Ir:tdt = f'1r dro
dLp -) ....,. "' to net to dt dt
to
- - = 'tp-(" m-r-) x ap
dt '7 I l .
=J"'1 Idro = I(ro 1 -ro 0 )
www.puucho.com "'•
Anurag Mishra Mechanics 1 with www.puucho.com

1iiuiiD BODY MOTioii- - - -- - 465,


~--------- -----------·- ----- ---------
Equating the first and last expression, we have ~net = M The weight of m1 exerts a torque m1gR out of the page
(scalar version of rotational impulse-angular momentum and the weight of m2 exerts a
theorem). torque m 2 gR into the page. Since
m1 moves down and m2 moves up,
Conceptual Example: A box is projected horizontally they both have angular momentum
along a rough horizontal surface. Discuss why the leading edge about the centre of the pulley
of the base is damaged more than that at the trailing edge. directed outward. Since the torque,
Solution: The box's angular momentum is zero about angular-velocity and angular- z
any reference point on a line passing through the CM and momentum vectors are all along
parallel to the velocity vector. The crate is not in the z-axis, we can forget about
translational equilibrium, but it is not rotating. Hence the their vector nature and treat the
torque about such a reference point must be zero. problem like a one-dimensional
(.) problem with positive assigned to
counterclockwise motion and Fig. SE.35
(!) ~ (outward)
negative to clockwise motion. The
@ it (inward) speed v of the blocks is related to the angular speed of the
rt pulley ro by the non-slip condition v = Rw.
CM _,
-> dL
We apply L 't1ext = -.
dt

·] ® The total angular momentum about the centre of the


pulley equals the angular momentum of the pulley plus the
angular momentum of the blocks, each of which is in the
r positive z-direction.
Lz =Lp +£ 1 +£ 2 =Iw+m 1vR+m 2 vR
w _,
Fig. 5.76 The weight m1 g exerts a torque in the positive
__,
If we choose a reference point coinciding with the CM, z-direction, whereas m2 g exerts a torque in the negative
torque due to .friction and normal reaction must balance; z-direction. The moment arm for each force is R. The net
gravitational force does not exert any torque about this torque is :
point. Torque of friction about the CM is clockwise. Torque
'tz,ner = m1gR - m2gR
of normal reaction must be anticlockwise to get zero net
torque. Therefore line of action of normal reaction must From Newton's second law of rotation,we have
dL d
shift and should act on the front half of the block. As kinetic m 1gR - m 2 gR = _z_ = -(Iw + m1vR + m 2vR)
friction force is proportional to normal reaction, its effect on dt dt
front edge must be greater. =Ia+ (m 1 + m 2 )Ra
1 2 a
,--- - ---- --- ~
= -MR -+ (m 1 + m 2 )Ra
2 R
l ~~~P.'TI_~~"j 35 :_>- 1
where I =-MR 2
and a=Ra.
:An atwood's machine has two blocks of masses m1 and m 2 i 2
(m1 > m2 ), connected by a string of negligible mass that
passes over a pulley with frictionless bearings (Fig. SE.35).
The pulley is a uniform disc of mass M and radius R. The.
string does not slip on the pulley. Find the angular
_acceleration_of th? pu(ley and the acceleration of the_blocks.
Solution: Problem Solving Strategy: We choose Suppose a person of mass m stands at the edge of a circular
plane of pulley as ;ry-plane and z-axis outward normal to the platform of radius R and moment of inertia I. The platform is
pulley. Now we will determine the directions of torque and ,at rest initially, but the platform begins to rotate when the
angular momentum vectors. We compute the torques and person begins to move with velocity v. Determine the angular
angular momenta about the cnetre of the pulley. Since m1 is _vel_ocity of the platform.
greater than m2 , the disk will rotate counterclockwise
__,
corresponding to ro out of the page in the positive
z-directions.
www.puucho.com
Anurag Mishra Mechanics 1 with www.puucho.com

Solution: Problem Solving Strategy: If the person Solution: Problem Solving Sfrategy: This
starts to walk along the edge of the platform, the platform situation is equivalent to an inelastic collision (person sticks
starts rotating in the opposite direction. The person exerts a to the platform). In such cases energy is not conserved.
force (torque) on the platform, the platform CJ,erts an equal Linear momentum is not conserved because the system is
and opposite torque on the person. Therefore the net torque not isolated; large horizontal forces from the earth are
on the system of person plus platform is zero and the total transmitted to the platform at the support. However, we ma
angular momentum is conserved. apply conservation of angular momentum because the

Lperson = (mR 2 i(*} Lp!atfonn = -Im horizontal forces at the posts supporting the platform do riot
exert any torque, as they act at the rotation axis. Here we
ignore the small frictional torque on bearings.
When the person starts walking anticlockwise, the From co_nservation of angular momentum,
person's angular momentum points upwards (from right --+ --+ --+ --+
hand thumb rule). Initial angular momentum is · zero Lperson + Lplatform = Lperson + Lptatfonn
initially, so platform must rotate clockwise so that final mvR+ 0 =(I+ mR 2 )ro
angular momentum is zero.
--, --, mvR
W=-==-
L; =Lt + mR2) (I
2 Two Bodies Rotatory System
0=mR (*)-Iro
Fig. 5.77 shows an insect.at'
mRv the rim of a ,disc that can tum
W=-- about a frictionless axle. Initially
I
,.---·-·-- ~,-·.----,-i-,,,,,_ the system is at rest. The insect
f:2.Ec~§o.~·iBl~A! 37 v crawls along the edge of the
disc; due to the friction force
,--- - . . .,_. - --- ------ -·--·-- ·- -----
(consider. the· situation of &ample 3(j. If the piat]~tm ';.,] between disc and insect, the
:rotating initially wi_th angular vel6city ro 0 and _the' person! torque of friction rotates the O'
!
system (disc + insect).
:starts tq move towards the centre.• v\f~at is the angular ~peedj
:wheu hg,_ reaches a_J!oirit·at a.distance t'from ihe centre ?:-" Since the net torque of the
L___
F_.i_g._s_.1_1_~_,

Solution: Moment of inertia of system has decreased . paired internal forces about the axis of rotation 00' -is· zero,
as there is redistribution of mass without external torque. therefore applyiog conservatio!l of angular momentum
Therefore angular momentum of the system is conserved. about 00'
--, --,
--, --,
Linsect + Ldisc =0
L, =Lt --, ..,
(I+ mR 2 )ro 0 =(I+ mr 2 )w or ILinsectl =I Ldiscl
(I+mR2) I1W1 = I2W2 ... (A)
ro = ro 0 Differentiating the above. with respect to time, we get
I +mr 2
I1a.1 = I2a.2
I =x ....,.,,;;.,, ; ~
/i-"'='-c- g,_~~·,$· ~"'~~ Integrating equation (A), we .get
--·--· ----"' .. --..-------,.-....'.,,-,
JI1 d0dt = JI 2 d0dt·
--- ..... ~ .· ~,~, .·- 1 2
:11 a person' runs at.a speed·~ along d line tangent ''th~ to
!c,.·rcumfe. re.nee of a stationary.. p. latfo17T!.1 _;.,ha.tis the r~tati~·'!?l I 101 =I 20 2
\speed of the .system ·of platform plus .person'when·the person
!lungs on to the platform ?· . __ · . · · ________ · ·

\ www.puucho.com
Anurag Mishra Mechanics 1 with www.puucho.com

L~1GID BODY MOTION - ~-· . --- ---- '

---------- ·-----·-----. -· -·- ----~-----~=__ ,~-~-.: -·- --


' angular
where 81 ,0J 1 ,a1 and 8 2 ,0J 2 ,a 2 are the
displacement, angular velocities and angular acceleration of
the insect and the disc respectively.
'A clutch assembly consists of two discs A a~d B a/moment of
(,t~<ii:1i~J~:~ ;inertia 21 and I respectively, one being the' engine flywheel, the·
,other one is the clutch plate. The discs are initially rotating
:The device shown in Fig. 5£.39-;,~-t~;;; on the vertical axle-;;;: with angular velocities ro A = ro and ro 8 ;,, 2m as shown in Fig.
·shown. The frame has negligible mass-as compared to the four· 5E.41. When the two discs are brought into contact ,the discs
masses each of mass m. Initial angular velocity of the systeni, ,rub against each other and eventuatl,ycreach·· a common
is OJ 0 • Due to an internal mechanism the spokes in the frame 'angular velocity ro.
lengthen so that the radii of the masses become 2a. What will, ( a) Derive an expression for OJ.
be the new angular velocity of the system ? ,(b) What is the angular impulse of friction ·on any one of the
. - . discs ?
~
-· , ··--- m

. i

Fig. SE.39 Fig. SE.41

Solution: There will be no torque on the system if we Solution: There is no external torque on the system.
ignore frictional torque at the axle. Therefore the angular The frictional forces do exen torque on each disc but they
momentum of the system is conserved. are internal forces for the system of two discs. Therefore
-> -> angular momentum of the system is conserved.
L initial = L final (a) From conservation of angular momentum,
I initial OJ o = lrma1 OJ -> ->
L; = Lt
4ma 20J 0 = 4m(2a) 2 0J
!ACOA +[B(J)B = (IA'+JB)OJ
from which we have
OJ= JAOJA +[B(J)B = 2Jro+J(2ilJ) 4
-co
IA +1B 31 3
Note that we have taken angular velocity of the discs in
the same direction; if they have opposite rotational sense,
the expression for ro will be
A rotating star has a period of 30 days about an axis passing (J)
IAroA -!Bros
= -'"----"---"-"- =' 0
through its centre. The star undergoes an internal explosion/ IA +Iu.
and converts to a neutron star. Initial radius of the core was·
(b) Angular impulse is changed in angular momentum.
1.0 x 10 4 km, whereas final radius is 3.0 km. Determine the'
period of ~otatiolJ. of the n~utroJ! star._' __ .. . ; -, ->
l,AMj=jliL,,i= (21) (43 )
ro -2Iro
Solution: During collapse of star no external torque
acts on it; it undergoes redistribution of mass, resulting in 2
= -Iro
change of moment of inertia. From conservation of angular 3
------- ·---- .. 1.-::7
momentum,
[
- ~~*
•.. -- - @.P'.\J?..
• •. .?.ii' - }~
.. _.1! 42 I_.'::>-
___)t;;..:.-
A man of mass 100 kg stands at the rim of a tum-tal,le of
:radius 2m, moment of inertia 4000l<g-m 2 mounted on a
,vertical frictionless shaft at its ctmtrc. 1'/;e who(e system is
initially at rest. The man now walks along the outer edge of
or tl)e tum-table )Vith a veloqity of 1!)1/~ reJf!tiye to size earth.

www.puucho.com
c,
Anurag Mishra Mechanics 1 with www.puucho.com

(a) With what angular velocity and in what direction does (a) Describe quantitatively the motion of the skat~;s after
! the tum-table rotate ? I they are· corinected by the pole.
\(b) By pulling on die pole.. the skaters reduce their distance

':-l'~t::=s~,
(b) Through iyhat angle will it ·hqxe rotated when the man
j' reach~ hIS i(litial p,psition ori.s'th~ tum-table ~ . · . ·. i
(c) Throug~wµat-arllfl~wiU it hciye rotated whenthe man
I. reaches his initial po~_itiQ[U:!l1fltive to .earth ?__ --~~
Solution: Let the man be moving anticlockwise.
I
(a) By conservation angular °:omentum · on the of 1-. .u•,- (a) · (b)
man-table system,. . . ....
.... .... ·· 1--
L; =Lt or f Fig. SE.43 , · '
,··1co vl
co,=-~ ., where com= - = -rad/s (c) Com~~fr the KE{of the system in part (a). Where does
.. _ · _I, . r 2 Ll_he change come from?. · . __________ ·
, = -100(2) 2 X l/
2
Solution: (a) As the initial linear momentum of the
. · 4oop system (skater + pole) is zero; the centre of mass will be at
1
=--rad/s rest before and after the collision.
20 The skaters and the pole-will rotate around·the of mass
Thus the table rotates clockwise (opposite to man) with (at the midpoint -of the pole ) . Free rotation of a system
angular velocity. Q.05 iad/s. always takes place about CM: · ·
. (b) ·Iftheman completes one·revolution relative to the Applying the conservation of angular momentum about
· table, then ·· · an axis through C and perpendicular to the plane of the
amt = 21t; . 21t = am -0, figure, · · ·
2lt = COmt-co,t . (where.tis the time taken) mvl + mvl = Ico where I = 2m(l) 2
21t · 2lt 2mvl v
t::~--- C0=--=-
Crom -co,) 0.5 + 0.05 2m12 l
Angular displacement of, table is co= 20/3 rad/s
.(' . 27t (b) When the skaters· reduce their separation , no
8,
'
=co,t' = -0.05 X - -
0.55 external torque acts on the system; hence we can apply'
21t" d"ian conservation of angular momentum about CM. Due to
=--ra redistribution of mass about axis of rotation, moment of
11
inertia changes.
The table rotates through 2 1t/ll radians clockwise.
Since the separation reduces to 2r = 1 m,
(c) lf .the man completes one revolution relative to the
Iro = I'CO'
earth,
(conservation of angular momentum)
then em ": 21t ,
. . 21t 21t , Iro 2m1 2co
Tune=-=- co = - = - - = 9co = 60 rad/s
com 0.5 I' 2m1' 2
=> angular velocity increases
During this time, angular displacement of the table,
· . 2lt .!. r co' 2
a,= co,(tune) = -0.05x- (c) KE1 = =2~_ = _(I_'co_'_)co_" = co" = 9
0.5 KE-' I_ 100 2 (Ico)co co
.

a, = _2:_ radian 2
5 .. The kinetic energy increases because the skaters do
, 0, = 36° in clockwise direction. positive work in pulling themselves towards the centre of
the pole.
_!Lex~>
l,:,.E._~ar.r;.
=e--=' "'"'~
I.e
E .. c I~
44- ~ ~
~~)kateri,each,of mass 'so kg; approach each other along!
:parallel pat~ ·separated by 3m. '[hey have equal and opposite !A thin roddfmas Mand le~gth dis attached to apiv;;~i thel
velocities ofl O m,1s.. The first skater carries a long lightpole,3 !top. A piece ofclcij, of mass m·and speed v hits the stick ~
m long, and i-lie second skat.er grabs the end of it as he passes
dis~ance_xfro.m thep·iv··o.- t ands·tl····°!'5_ ~o it (Fig. 5E.-!4); Find the
1
(assume frictionless ice). __·· . -- . [ratio of'tlieji.pal· en~!:.i/Y to the mtt1'1 ener~- . _ _,_
www.puucho.com
Anurag Mishra Mechanics 1 with www.puucho.com

r - ·----- - -·- -~-- ---


1 RIGIDBODYMOTION _ _ _ _ _ _ _ _ _ _ _ _ _ _ _ _ _ _ _ _ _ _ _ _ _ _ _ _ _4~6:.::..J9]
- ------·---
--------------
~ .~x(!mnfe,
-
~-·,-::,,.
-~ ~~"--J, ~=:> , ~
M
r --- - - - -- --- - - -- - - . ---- - - - - - - - - - --
;A disc slides towards a stationary bar pivoted at its end, on a:
X lfrictionless surface. The disc sticks to the bar so that after\
d :collision the two rotate about the nail. Detennine: ·
(a) the angular velocity of the system after collision,
,- '(b) the kinetic.energy before and_ after the collision, '
1
'-'--Li (c) the total linear momentum befogand after the collision~ ]
- Fig.5E.44 Solution:
Solution: Problem Solving Strategy: The collision_ Before
is inelastic, so mechanical energy of rod is not conserved.
During the collision, the pivot exerts a large force on the
stick, so linear momentum is also not conserved. However,
there are no external torques about the pivot point on the
clay-rod system, so angular momentum is conserved. The
kinetic energy after the inelastic collision can be written in
z
~

(pivot)
M

terms of the angular momentum LI and the moment of


inertia I' of the combined clay-stick system. Conservation of (a) (b) . '
angular momentum allows you to relate L1 to the mass m I
:.. ------·-- ___ Fig. 5E.45 ·····----- ------·-------·- ..... J
and velocity v of the clay. r-·- - - - · - - --· - - - - - ______ , -- ---- - - - - - ------ ·,

The kinetic energy of (rod + ball) after the collision in 1 Concept: Force generated at the pivot is unbalanced!
terms of the magnitude of angular momentum LI and the ;external force on a system of bar and disc. This force exert~ no!
moment of inertia I' of the combined stick-clay system, Itorque since its momentum is zero. Therefore angular'.
, LJ !momentum of the system is conserved. Kinetic energy is not(
E1 = 2I' :conserved, since the collision is inelastic. The force at the pivot'
!may have a component in the direction of the _disc's velocit;y;,
Applying conservation of angular momentum about _therefore mome.:1tu111:_ma1 no"t be_c:_o!"~':'.e~---- _ -------~--j
pivot,
Lt =L, = mvx
(a) From conservation of angular momentum,
-t -t -t -t
Moment of inertia of system after collision I' in terms ,of Ldisc + Lbar = Ldisc + Lbar
m,x,M and d,
I'= mx 2 +.!Md 2
3
mvL =( Mt 2
+mL )~

Substituting these expression for Lt and I' into your mvL


W=-----
equation for E 1 ,
(Mt +mL2)
L2
Ei =_L= (mvx)
2

2 2 1 2
2I' 2( mx +½Md ) (b) KE-=-mv
I 2

3 m2x 2v2
23mx 2 +Md 2
Dividing the energy after the collision by the initial 2
energy of the clay; 2
3 '
-m 2x 2 v 2/(3mx 2 +Md 2)
=·_!(ML +mL2Jx[ 2mvL ]
E1 2 - 2 3 _ ML +mL2
-=
3

3mx 2
=-----
3mx2 +·Md 2 .

www.puucho.com
Anurag Mishra Mechanics 1 with www.puucho.com

. -~ I
~;4_1_0_~:_:>rz~i2.:1. ~:~-· .:'. _f,:J:: __.{.· -~-...o...-t.,·""_,_'-,._,-"-,_-_.____ !.__:,_;.f~·-~ - ' - - ~ - - ~
,-----------··-----·------·--·---- - - --·-
. MECHANICS-I
·-· -----··1
: C~ncept: The energy of system is not consen•ed during I
lcollision as collision is inelastic: But after collision we can)
lapply energy- conservation equation. ,,·
--- ---~- --- ------- --------··------------ - -- -- --- _,
Note that KE 1 ·< KE, Calculating the minimum value of v 0 , we write an
. . 7t •
(c) . P, = mv equation of energy. When the rod rotates at an angle of - the
2
P1·=mv'+mvCM ~mLco+M(L;)
centre of mass of rod rises a distance of!'... In addition, the
. 2
=(m+~)Lco point like mass rises a distance L ·above the initial height.
Therefore, · ·
- . - --- - - . -- ---,-.-- --·-·- . --. -- --· - - - - - -·- - --- '
1 3 3
Concept:, Nqte,:thqt final momentum of system ~ morel 2 2. L·
-IAco =-mv 0 =mg-+mgL=-mgL ... (4)
.than initi~l linedr. nianientum.'"When the disc strikes the bru;i 2 8 , 2 2
·the bar pWjhes the pivot backward. ,Th~- reaction of pivQt is tol 1 2
where - I A co is the kh1.etic energy of the system
j,ush forw_ard cifi 'the pivot T]tiiforward push of tlie P,ivotj 2
imparts·momentuni fo the bar in the same direction in which'. immediately after the collision. Substituting expressions for
the bar Wils.:'.,...moving·
, \! 1-• • •
initially,' thereby,;
I
increasi",,g- the\
., ' • ' • ",
I A and co, we ·find
mome~tum oJ,~~~-~s~em: . . . · · · 1 ~; : ...(5)
. ~-
Writing Newtori's second law for rotat_ion, L't = I a,
mgL · .4 2
. - --- ----:-·--..-- - - .. ----- ----- ---- .- '
--- mgL + - = IAa = -mL a ... (6)
'A thin uniform :£
rod ofl~ngth an<J mass m is free rotqte! to 2 3
·about a· sniboih pivot, which passes through its one eiidA. k Therefore, we finally have
!point like m~f-11!-,,~,shot ~o.$ofitapy, at a velocity 'of vol 27 g
a=-~ >(7)
'towards·the lower ev,ifiof.the rbd.(ppint B). When i(~its,thei 32 L
-rod, it sticks to it [See Fig: SE.46.(a)J. . ,... -;·.
1. Whai ii the)niii1,num·value,oli>o requiredfor the'rod t~' l ' ,r~ i
• ' . • 7t . . . . • ..•. , 'A 8
reach an angle''of,- (horizontal stafe) ? I. I

' ' ,-, ' 2 .' ·, ~· ·!' ' . :



CM
2. What istlre'a'ngulizqzcceleration of tlie rod at this statel°''\ I
, mg mg
3. What are the -·magnitude anil direction of the forcei I • , (
•.... app)(eg_ by.tht.(ajs .!H! the fq,:U.h _the /!ori..ohtal:smi:e '(;
I '
. l·
-
Fig, 5E.46,(b)
- ·---··------ .,._. ~-----1
i
Solution: Using the principle of·
Fig. SE.46 (b) shows free 'body diagram of system. r
conservation. of. angular momentum for
the collision about the point A, we write denotes the distance of the centre of mass from A and is
mv 0L = IAco, where IA · denotes the calculated by using
. i . L
moment of inertia of the system (rod + "m,l! :Em;x, mL+m
point mass) about point A . .Notice that I r=-'--=·--~~ 2 ~L ... (8)
during the collision, the force of gravity is im. Vo L111·
i ni+m 4,
I l ,,

parallel to L and, therefore, its torque We now write Newton's second law for our system.
about A is zero. We first calculate the . Fig. SE:46 ~) I mg+ mi;-N = (2m)a ... (9)
moment of inertia of the rod. about A,
using the parallel axis theorem, Since we are interested in the linear acceleration at a
__J'.. =ICM +mr 2 point which is a distance~ L from the axis of rotation (this is
. 4
2
the distance from the cnetre of mass), we have
1'.. = _!_ mL2 + m(!:.) = .!3 mL2 .. (1)
.,.• . 12 2 a= ra = -La
3
... (10)
The total moment of inertia, inclui:ling the point like 4
mass m, is Therefore, by substituting for a and a calculated in the
'fA = [A +m£ = 1m£2 2
... (2) previous section, we obtain
81 47
Therefore, .
mv L
C O =0- - e a - 0-
3v
... (3)
N = 2mg(l -
. 128
)=
64
mg ... (11)
JA 4 L

www.puucho.com
Anurag Mishra Mechanics 1 with www.puucho.com

! RIGID BODY MOTION 471'

A bullet of mass m moving with velocity v strikes and becomes: ilwo uniform discs rotate separately on parallel axels. The
;embedded at the edge of a cylinder of mass M and radius R 0 ,, iupper disc is given an angularvelocityco 0 and the lower disc is
,as shown in Fig. SE.47. The cylinder, initially at rest, begins to! at rest [Fig. 5}:!.48 (a)]. Now the two disc., are moved together
•rotate about its symmetry axis, which remains fixed in! so that their rims touch. After a short time, the two discs
.position. Assuming no frictional torque, what is the angular ,rotate without slipping. Find the final angular velocity of the
velocity of the cylinder after this collision ? Kinetic energy :upper disc.
conserved?
m

M
0
+ a;;)
F

Fig. SE.47

Solution: We take as our system the bullet and


cylinder, on which there is no net external torque. Thus we
can use conservation of angular momentum and we
+ (a)
Fig. SE.48
(b)

calculate all angular momenta about the centre O of the Solution: The two discs exert equal and opposite
cylinder . Initially, because the cylinder is at rest, the total forces on each other when in contact [Fig. SE.48(b)]. The
angular momentum is solely that of the bullet. torque due to these forces changes the angular momentum
of each disc. From angular impulse-angular momentum
L =Ir x Pl= R 0 mv theorem, we have
Since R 0 is the perpendicular distance of p from 0. After
Fal!.t =11 (co 0 - co 1 ) ... (1)
the collision, the cylinder ( I cyl =½MR5) rotates with the and Fbl!.t =l 2co 2 ... (2)
From eqns. (1) and (2), we get
bullet (I =mR5) embedded in it at angular velocity co a I 1 (co 0 -co 1 )
L= Cicy1+ mR5)co = (½M + m)R5co. b
...(3)

Since angular momentum is conserved, Concept: When slipping ce(1$es between the dLscs, the
mv contact points of the two dLscs have the same linear velocity,
co=~--~-
;~e:!
(½M+m)R0
aco 1 =bco 2 ... (4)
Note that angular momentum is conserved in this On substituting co 2 in eqn. (3) we get
collision, but kinetic energy is not. ([,cool
1 1( 1
K1 -K, =-I 1CO 2 +- 2
mRo)CO 2
--mv 2
'2cy 2 2
_ _ vz
__mM
2M+4m '
Which is less than zero. Hence K f < K ;- This energy is iA particle. of mass m moves with speed v O in a circle of radius
transformed to thermal energy as a result of the inelastic ir0 on a frictipnless table-top. The particle Ls attached to a
collision. string that pas,;es through a hole in the table as shown in Fi,
,SE.49. The string is slowly pulled downward so that the
,partfcl~ m<:_ves in a smaller circle of radius r1.

www.puucho.com
Anurag Mishra Mechanics 1 with www.puucho.com

~:;:47:: ~::::::=;::;t:';::)':;z!,:::;'~;=::::::=~-~";: : ·""':~:_~;


__2_:=-.::::':·=-~::::::::__:;:~;_;.i;;:1:-_·:1=·~-: ~ : ; : ; :;;··~~~m:;;::;,1:;;:;'
; ; : : ' ;,:;;, ·,_,C'-',,..._
- - : : : _-
- _- ~·'=?_' ·:. . , -. ME(H~~J~
~~'·

L. _- - ~ ~t;,/>.G~~~
IA sm~·iswrap~ed se~eral t1m~],Td~lili~rofmasi*~~4l
, _i ; j Iradius R. Tlie.fcylinde'r is, pi~'oted. about its was· 'ofrblpcic1
. ·
1
s}mmet!'Y: .A ~lock oJ mass m, ti,(id .toithJ itrinl re§ts:ihit-fqj
'I , support pos,itfo~ed so thatthe_smf!g,has no ·slack.'Jliel'{4~'isl

".:';,-, , .!I carefully lifted :vertically a-dist~n,ce,h ar;d .the ~upp&rt; ·iil
removed'•cis shown in Fig. 5E.-50;: / ·; _ ·, - · • ;'., : ,:,, ', J

I,) F
.. 'd' th .fin'?; l I . F'.g.tSE.4~':-:;-:- d '' ., ' ' ' ·__ .,' ·•· .. R ' ·.,' ' - '. \.1'} '·_··-11,

r~
,la. ln , e , !1 Ve_OC!ty.,m. ermf,PJ ro,Vo,an .r,, , ,·,r,;' , .

~-~~!u_t:_·.-_:_l_~_~:_:=h:t!h_',e.__ :~_-_:_l_,_~_.h_._~. :~~~i~~_-·._m_c.


1.;~_-.~-~-_._·~--~~ ·,-.· 1
'(c) Calculate the.;wprk done 071tltf,particle by the.teruiofi.''E m
( · J.xpress yourjlnswjcirt teiJns~of,r~and L0 . ~ ~ ', ' , /
Solution: (a) The net force acting on the particle is _ ),,; ' ,,J
tension in _the' string which is always directed toward the ·I - ·.;,
axis of rotation. The torque of tlie tension is zero as it does '"'j Fig, SE.SO · :;•
1
_:),,;f': 2· ·
~: :;::~ ~~::s:~~- Hence the angular momentum of (a) Just.b~/{r{the:;;n;li~cp,rt~;taftt,ev~luate,t{{JFiz~n1
mv 0 r0 = mv 1 r1 Ii ,·,. velo·c.it;Y_.·•._ro_A.°{ t~: cylinder, t,h_,~.._fp¢~d___v.~,of_pie_ f~!_z;_'J%_:,,,_'~_'.-~dy-_:j
m and,~h.ekmetzc ener~ftJ;,?fth~:Y~_tel7!,,. :-,,:,::: , '.,
ro (b) Evaludte the correspondmg•guantztzesro,,v and.K1for1
VJ =-Vo
, rf I the instant-just after the stringbeiomes tautf _ ·;, ., ,'·i·
1
(b) From Newton's second law, . 'cc) ·Why isK{Jess thanKo? W!Jer'i,;doi!s'ihe ene,rgyg~?,:
v2 :f
d) If M =o_"!,,,Whatfractio7'! 6f;Sh~k.inetic energy is;JQ~tipne[i
T=m-
r
the
L., strmg becomes taut?, ';t:; · , . ,.,,;:_,,•,:·L.. •. ·

· Also,
or
mv::rs} =Lo
Solution: (a) Just before tile string becomes taut, the
block falls freely, so v O = ~ 2gh. There is no tehsion in the
string, so nothing causes the cylincler to spin; so ro 0 = 0. The
kinetic energy of the system is
Hence T =in (Lo/mr)2 K0 =.!.mv~ =lc2mgh),,,; mgh.
r· 2 2 .
L2 (b) When the string experiences a jerk, the large
~~ impulse developed is of very short duration so. that the
. mr
Work must be done to pull the string downward. Since
contribution of weight mg can be neglected during this time
inteivaL ·
r1 is less .tha,h r0 , the work is positive. ·From work-energy
theorem, the work done is change. in kinetic energy. Using The angular momentum of the system is consetved, as
2 the tension is internal force for the system. Thus we liave
.K =L , th~ change in kinetic energy is _, _,
21 L, =-Lr
- L2 , L2
K1 -K,=-_-o____o_ 1 2
·· 2mrf2 2m,02 mv 1R +-MR ro 1 = mv 0 R
2
Li (rf-2 -ro-2) = m~2ghR
=-
2m The string is inextensible, so v 1 = Rro 1 • On solving for
I which is same as obtained above. ro 1 , we get
I

www.puucho.com
Anurag Mishra Mechanics 1 with www.puucho.com

v, ·'-==r
RIGllll!ODY MOTION_ .' "' :.
'--'-c...c.---·~------~--'----~....:......:.._ _ _ _ _c~----....:..-·:.t,
~Zgh mv 0 r0 = mvr cos0
001 =-----~
· R[l + (M / 2m)] Also, energy is conserved. so that E1 = E 2 . Thus
v 1 = Rro 1 1 2 1 2
-mv 0 + mgh = -mv +0
2 2
= ~Zgh Vo
=-~~- V = ~~V-5~+_Zg_h_
[1 + (M/2m)] ( l + ;,)
· Eliminating v and substituting r 2 = rt - h 2 give
The final kinetic energy K1 is given by
vor0 = ~v5 + 2gh~rt-h 2 cos0
1 2 1 2
K1 = -mv 1 +-Jro 1
2 2
0=cos-'RIN
1 2 1(12MR 2)(vf)
=zmv,+2 R2

=½( m+~}; l..iE:ke;~ ..•~:t~5-


2 cl("''"
E:l,5--·----~~~~~~.....::_J~
r· - . . . . • ··-··--,
IA uniform rectangular block o'fdimehsions shown .in.Fig.I
=½[ l+:J2m)] SE.52 is sliding on the horizontal surface with a velocity vi
Ko wlien it strikes a small kerb in tli.e silrfdce. J)etermine the
=--~- minimum valile of v for which.the ):,lock will pivqt about the
l+(M/2m) kerb .and)u.st ,reach the verticcrf 'positi,on with tiO velo,ity.
(c) The situation in this case'is analogous to the energy sume neglig{Ble, rebound at .the step; - ·-- . . .
loss in completely inelastic two-body collisions. The lost I.
.
,.
·-;('·.. b
.

kinetic energy is converted to heat energy or elastic


potential energy of the string or in the two objects.
2Ko
(d) For M = m, K 1 = - - , so the fraction lost is
. ! !::~L:'.·~:+ill§@L:- I b

' 3

(Ko; K,) = ½
0
V

""";\·'. CM

~~£1,~~~~:J 51
r--~-,so~ - --"~.------··
t> ·- - - - - - - ~--~
R, _,..-RI
!A small mass, particle is projected with an in_itial velocity v 0
mg
!tangent to the horizontal ri/n of a smob_th hemi spherical _bowl
)ata radius roftom. the verticatcentre line, as shown at pq{nt Fig. SE.52
1A As the particle slides past point 13, a distance h belowA and
'.a.distance r from the ·vertical centfe'line, it;i; velodty v makes Solution: For small time of impact the angular impulse
\an angl~ 0 withtlie horizontal tangent to the bowlthroughB, about 0 due to weight is negligibly small. The force of
,Determine 0: . reaction at kerb does not have ·any angular impulse about 0
I -· ....... - ···- ··- -
I :O' because their line of action passes through 0. Therefore we
Vo: may assume that angular momentum about 0 is conserved.
A :
Moment of inertia of block about 0,
=ICM+MR 2
2 2
= {l~M(b + c ) +M[ (~r +(%r]}
0
= M (b2 +c2)
Fig. SE.51 3 .
L, =Lt
Solution: Problem Solving Strategy: The forces
on the particle are its weight and the normal reaction Mv!:. = M (b 2 + c 2 )ro
2 3
exerted by the smooth surface of the bowl. Neither force 3vb
exerts a moment about the axis 0-0', so that angular Ol=----
2(b2 + c 2 )
momentum is conserved about that axis. Thus,

www.puucho.com
Anurag Mishra Mechanics 1 with www.puucho.com

I474
If kinetic energy ·of rotation· equals the increases in From conservation of energy,
potential energy in the vertical position the block will attain KE; +GPE, =KE 1 .+GPE1
this position.
From law of conserva?on of ettergy, I. M
2 2
/3i) ·+I.Mgi
(L123 ) VT
2
(
4
2

= o+I.MgLcose
4
KE; +GPE; = KE! +GPEJ
2 w~ich on solving for cos 8 yields
I_M(b2+c2)[ 3vb ] +Mf 1
2 3 . 2(b 2 + c 2 ) g2 case= -,hence 8 = 60°.
2 .

= O+Mg[ (%r +{%r J When the rod breaks, the velocity of the upper end of its
lower half is ~ to the left and the velocity of the lower end
f(1+E..:.)c.Jb 2+c 2-b).
2
is roL, also to the left. The moticin of this part is combination
3 b of translation (linear velocity 3roL to .the left) and rotation
4
(angularvelocityro, clockwise about its centre of mass). The
centre of ma:ss of-the free fragment will follow .a parabolic.
- - - - - - - '.---··------- ·~ ' - - - - --.- '-.. ~1
\A uniform. rod of·length L is fre~lY, pivoted at one end. W isj path downwards· and to the left, while the fragment rotates

ritiall~ hdd.h~rizonta~l
and t:ef!r.rgsedfrom rest.. :j !
clockwise at a constant angular velocity.

ig_6;~gm~~~~
I U2 1:,;:·;;;;,.~~;;,,,-;od-;,~/len~h 'i,"'is ~h!itiall; ~;-;.;;;~ ;,,:At; ',a~I
...e. .rod ,is tapped ,at.one" en1'
inertial [rame:.,of.;refer.ence. .Th
[ -l I Iperpendicu1a~ t() ,ts length- How far does the cenr:.e ,:,fmass
l. Fig. SE.53 (a~ I· ··, !translate wbile, the rod com,pletes one revolution (lbqµt its
)centre of mass?: Neglect_gravitational effect. . ; ..~~--
!What is the angular velocity at the. instant when the-rod. is
!vertical? When. th.e rod is vertical"ici'breaks at its midpoint. Solution: The. impulse
[What is the. largest angle from the vertical reached by the
iupper part of,t.he rod ln its, subse.quen. t motion? Describe the .
delivered perpendicular to the rod
at one end gives some linear '
1.1· _, ·_ ,_."'•:-. ,·.,.,_
. ~':
' ' ' '

Imotion of the: lower part of the rod: ,4ssume that no impulsive 1 momentum to the CM (centre of
mass) of the rod and also some
:
.' ·,
_, _:vcM
,,,~' ' <
!forces are geniirated when the rod!!_reaks. - -- --· ·--- '.J
angular momentum about the CM. . '·''\

Solution: From conservation of energy we may obtain The rod will rotate about CM. Free Fig.'SE.54 ,'
angular velocity of the rod in the vertical position. rotation of any rigid body always
·--··--'~
KE; +GPE; = KE 1 +GPE 1 takes place about CM.
2 Translation of CM: Applying _imp~lse-momentum
,. . O- lML 2 (-MgL)
,, 0 + - -2- -3( ! ) + -2-
... (1)
equation,
where· reference level is assigried at pivot. :EF =dP-
dt
From eqn. (1), ro = ~ - · fFdt =M&vCM
Since no impulsive force -is generated,
JFdt = M (v CM - OJ [initially CM is at rest]
the angular velocity 9f the uppf{.rart _i- ... J2 1 Rotation about the CM:

Moment of inertia of upper part


3
immediately after the break is still g.
. . L ,I t' II
I

I
'.

. •.·
'

'
:E~ = dL
dt
or !:.JFdt =ICM Aro
2
Let .M be the time required for one complete revolution
of 21t rad. Multiplying both sides by At gives
! 1.(2
=½(~)(L~2r I
L
-vCMM = -w&t
L2
[_:: VcMM = x]
If the rod now rotates through an angle
I! . ! J 2 12
. LS
8, its centre of mass will rise through a [Fi~(b) x=-
6
distance . L1t
L X=-
-(1- cos8) 3
4
www.puucho.com
Anurag Mishra Mechanics 1 with www.puucho.com

RIGID BODY MOTION 4751.l


For the translatory equilibrium of the wedge z-axis,
N k +(mg+ F sin30°)(-k) =0,
A wedge of mass m and triangular cross-section,
(AB= BC= CA= 2R) is moving with a constant velocity-v,:
i.e., N =[mg+ y;v
( 3)/J.t
]k
towards a sphere of radius R fixed on a smooth horizontali
(b) As the wedge comes back without rotation
table as shown in Fig. SE.55 (a). The wedge makes an elastic I
about its centre of mass, we have
collision with the fixed sphere and returns along the samel
-+ -+ -+ -),
path without any rotation. Neglect all friction and suppose I ~R = ~,+~N+~mg = 0
that the wedge remains in contact with the sphere for a ve,y:
short time /J.t, during which the sphere exerts a constantforcei Since weight, mg, passes through centre of mass,
F on the wedge. ~mg =0.
4mvh
(a) Find the force F and also the normal force N exerted by:
the table on the wedge during the time At.
So, ~N-~,=0 or I~N=Fxh=--
I -,J3/J.t
(b) Let h denote the perpendicular distance between the. KINEMATICS OF RIGID BODY ROTATION
centre of mass of the wedge and the line of action of R
Relative Angular Velocity
Find the magnitude of the torque due to the normal force
N about the centre of the wedge, during the interval At. Consider two particles having velocities v 1 and v 2 at
angles 01 and 0 2 respectively with line AB joining them.
C Velocity components v 1 cos0 1 and v 2 cos0 2 determine the
z rate at which the particles approach or separate from each
y
other
-dS =v 2 cos0 2 -v 1 cos0 1
X A B dt
Fig. 5E.55 (a)

Solution: (a) The sphere collides elastically with the


wedge; so the change in momentum of the wedge,
Ap = mvi - (-mvi) = 2mvi
Let F be the force of contact; it will act normal to the
contact surface if the colliding surfaces are smooth. The
impulse along the horizontal direction, Fig. 5.78 (a)
(J) H = (F cos 30°) X /J.t(i)
The orientation of the line changes due to velocity
From impulse-momentum equation impulse causes
components perpendicular to the line AB.
change in the momentum of the wedge, i.e., (J) H = Ap; so
co A!B = angular velocity of A with respect to B,
considering B as centre of rotation
VAIB perpendicular to line joining them

Separation between A and B


.,
' v 1 sin0 1 + v 2 sin0 2
=~-~~~~-~ (clockwise)
s
Component of V81 A perpendicular
mg mg
Fig. 5E.55 (b) (l)B/A = - - -to-line-~ joining B and A
-~-----
Separation between B and A
F
(2.J3) x /J.t(i). =2mvi,. = v 1 sin0 1 + v 2 sin0 2
s
(clockwise)

F= 4mv We can see that, co A/B =co 81 A (in magnitude as well as


i.e., direction)
(-.fi)/J.t

www.puucho.com
Anurag Mishra Mechanics 1 with www.puucho.com

!476
·-.,-,-,
' Concepti Rigid body constritii!t Implies th.at alongtlie circle. Our aim is to determine the angular velocity of A w.r.t.
line joining any .. two particles- o~'- a, rigid bod.>, velodty B when their radii vectors w.r.t. common .centre are

component ~ii>!: ~e .the sa7!1e,
, . . .. "AJ. i.e/Y;~
'::11 ~I
I
·'.,} perpendicular to each other.

I; t·

,.

·. Fig. 5.78 (b)


Ve

ruustration: Consider two particles A and B moving


along a circ'ufaf path with angular velocities OJ1 and OJ 2
respectively. Our aitn is to determine. the angular velocity of ~---F~lg:j,JII!_..~-~
A w.r. t. B in· two· cases: · · Case (i) Same Sense of rotation:
(i) Same Sense of rotation: , OJ 1r1 cos0+0J 2r2 sine
~----· OJNB = S

=
OJ1r1(J) +OJ2r2( t)
s
2 . 2
m1r1 +ro2r2
=~~-~~
s2
2 ·2
= m1r1 +co2r2 (clockwise)
r2
I
+ r22
·Case '(ii) Opposite Sense of rotation: If sense of
rotation" of.A and B is opposite (OJ 1 anticlockwise and OJ 2
component of VA/B perpendicular
clockwise),
· · to line joining B and A OJ1r1 cos8+0J2r2 sine
(!) A/IJ
Separation between A and B s
. I

_.OJ 1 r sin(!}+ OJ 2r sin(!)

- '. . 2r~i~(!)

= (OJ 1 : OJ 2 ) (anticlockwise sense)


·Pure Rolling _
. (ii). Opposite Sense of rotation: . The term pure rolling means that there is. _no slipping
at the point of contact with the ground-no skidding takes
. , OJ1rsin(!)~OJ 2rsi~(!)
place. The wheels of a car that is moving forward while its
CO A/B
2rsin
(
l
) tyres are spinning at high speed,Jeaving behind black stripes
on the road, is an example of rolling motion with slipping.,

or =( OJ1 ~OJ2)
'Rolling Constraints
The disc in Fig. 5.81 rolls to the right and point O on the
axis moves to O' as A moves to A' and B moves to B'. The arc
if OJ1 > OJ 2,
then (!1 NB "is in antidockwise sense. length from B to A equals x just as length BA' = 00' = x.
If OJ 1 < OJ 2, then OJ NB is in clockwise sense. · Thus, the linear distance travelled by the centre of mass is
If OJ 1 = OJ 2, then OJ NB = 0. XCM =RB
llitistration: Two particles A and:B are moving on two
concentric and coplanar circles of radius r1 and r2, r1 > r2,
with angular velocities OJ 1 and OJ 2 respectivelyw.r.t. centre·of
\• www.puucho.com
Anurag Mishra Mechanics 1 with www.puucho.com

; __RIG1D_ B_~DY MOTION


·----·--------------------- ----------- ----- ---- ---- --- 477
Concept: If a symmetric rigid body rolls without
,slipping, there exists a relationship between:
1. the distance travelled by the centre of mass and
'corresponding angle through which the rigid body rotates
about the axis through the centre of mass
2. the speed of the centre of mass and the angular speed of

1
rotation, and
3. the magnitude of the acceleration of the of mass and
vao B A'
:the magnitude of the angular aq:eleration of the system.
Fig. 5.81
Relations (1), (2) and (3) are known as rolling
If the disc rotates through an angle 8 (in radians), as in '.C!)nstraints.
Fig. 5.82, then the centre moves through a distance Pure Rolling: Superposition of Translation and
XcM =RS ... (1) Rotation
Pure rolling is superposition of translation and rotation.
When you throw a ball, the translation motion is described
by the motion of CM, superposed with translation of CM is
spin of ball about an axis passing through CM.
y

Fig. 5.82

One complete revolution of a rolling disc translates the


centre of mass by a distance equal to the- circumference of
the circular cross-section (see Fig. 5.83).
Fig. 5.84

Consider a panicle on the disc whose position relative to


a fixed coordinate system is shown in Fig. 5.84.
1; = position vector of ith panicle on the rolling disc.
,___ _ S = 2sR - - > - I
Initial Position after ~ = position vector of CM
one rotation
Fig. 5.83
r;,CM = position vector of ith particle relative to the CM.
Differentiating eqn. (1) w.r.t. time, we obtain _, _, _,
Here, ri = rCM + r~CM
Taking time derivative of this equation, we get
_, _, _,
vi= v cM+v~CM
Thus, the rolling condition is equivalent to the following
relation between the translational speed of the centre and l Concepts: The velocity of ith particle is superposition of
the angular speed. ·two velocity vectors
VCM =Rm ... (2)
~ = Velocity of CM
Differentiation of eqn. (2) relates the magnitude of the
linear acceleration of the centre of the wheel, icentre, to the i r;,cM = Velocity of particle in a reference frame fixed with'
_, 1
CM
magnitude of the angular acceleration a about the axis '
through the centre of the wheel, , How will you describe the motion of a particle if you are·
,to sit as an observer at the CM? At any instant any particle on
Similarly, acM =Ra ... (3)
:tJze rol/ir,g object will appear _in rotation.
Rolling without slipping
Significance of rolling without slipping is that the
relative velocity of contact point of rolling object and the
surface on which it rolls is zero.
www.puucho.com
Anurag Mishra Mechanics 1 with www.puucho.com

--MECHANicsX '
: 478 -------- ---------
y' Reference frame fixed The velocity of the contact point with respect to the
at CM, frame 2. In this frame' ground
objects appears in rotation I --> .... ....
,,Y about axis through CM
VP, ground = VP, CM + V CM, ground
--+ : :
=VcM 1-roRi

~ c'M~t--+--• x' Therefore, criterion for rolling without slipping is


0
X
....
VP, ground =0
Fig. 5.85 vcM = roR
For pure translational motion shown in Fig. 5.87,
Reference frame fixed at CM, frame 2: In this imagine that the disc does not rotate, so that each point on it
frame the objects appear in rotation about an axis through
CM. P' P'
VcM v = Rro
Ground reference frame, frame 1: In this frame
the object translates as well as rotates.
Translational velocity of each point on object. v= Rro
CM ..._.....vcM CM•v=O
·y v = Rro
Translation velocity of
each point on object
"cM v = Rro p
p
Pure translation Pure rotation

(a) (b)
Fig. 5.87

moves to the right with speed v CM. For pure rotational


VpcM = - Rro
Velocity due to rotation about CM motion imagine that a rotation axis through the centre of
Frame 1. mass is stationary so that each point on the disc has the same
rotational speed ro. The superposition of these two motions
y'
is pure rolling motion as shown in Fig. 5.88. Denoting
velocity due to translation as vCM and that due to rotation
vR, total velocity of any point is
C I~+ v°bMI = 2yZM

vP,CM = Rro

Frame 2
Fig. 5.86 B"----
1~ + VbMI = 2ro
Velocity due to rotation about CM: The velocity of
CM with respect to ground is
....V C.'vl, ground = V CM 'l
The velocity of P with respect to CM is
--> A
Fig. 5.88
VP, CM = ---OlRi .... --> ....
Concepts: In the CM reference frame any point on the V total = V translation + V rotation
object is in uniform circular motion. The velocity of P is' --> ....
=vcM+vR
'backwards and is given by the formula for uniform circular!
motion. f
'

www.puucho.com
Anurag Mishra Mechanics 1 with www.puucho.com

RIGID BOD! ~OT!~ _________,_______,rill


Reference frame fixed to ground and another Ol=.':.!.= V2 = V3 =~
reference frame fixed to CM of disc r1 r2 r3 r4

".~~:~~~,::_,·:I Concept: The angular velocity Ol of the body about the·


;instantaneous axis is the same as its angular velocity ro' about'
CM 0
JvcMJ=Rco Jts centre of mass. ·
A Qualitative Proof: Let the body of radius R be rolling
JvCMJ=Rco
with speed v CM· This is the translational velocity for a
B
fvcMI =Rco stationary observer at O andvCM = Rro [see Fig. 5.90 (b) (i)J .
.c J1CMJ=aRCO
D
JVcMJ=Rco Centre
Ol
Centre
of mass VcM = roR of mass

\
A A
Instantaneous Centre of Zero Velocity
The special point of interest is A, here velocities due to
R R
rotation and translation are exactly opposite. Hence the
resultant velocity is zero.
0 Instantaneous VcM 0
Consider a disc in rotation about a fixed point A as axis
shown in Fig. 5.89. ·
i-~- - -·- - --- _,_ ---·-- ,,- - - --
(i) (ii)
. c 2.Rro, Fig. 5.90 (b) .
- -- ''
Forthe observer at A the centre of mass is at rest and the
./2 Rro : ID
surface is moving with speed v CM [see Fig. 5. 90 (ii)l.
' ~
ii B _. ...........,:•.••••.
Rro D !
··•··• ./2 Rro ,
Because the part of the body in contact with the surface does
I . ·- . .· . not slip, the observer at.A must see that part of the body is·
I ·----"2R I"2R _/ having the same speed as the surface; hence Rro' = v CM.
··.. : ..··
I •, • .• •
\ Comparing the two vajues of v CM, we have Rro = Rro', so
··.:.·· ro = ro'. Although this equality has been derived for the two
A points O and A, but it is general. Because the body is rigid,
_·--- --··- _F!~- 5!9 ___ .. -·- J
the angular speed ro of A about O must be the same as the
All the points on the disc will move in a circle centred at angular speed of any other part of the body about 0.
P. Tangential velocity at a distance R from axis will be Rro; at Therefore ro' is the angular speed of any part of the body
2R, 2Rro; at .,/2.R, .,/2.Rro and so on. Here, we realise that aboutA.
resultant velocities of points A, B, C, D and CM are same. in Concepts: 1. 1klocity of any point B on a rigid
case of pure rolling motion and pure rotation about A. This : --+ ~ --'t--+ '
prompts us to state that a body in pure rolling motion appears /body can be expressed as vB = vA+roxrBIIC· Ifwe choose,
to be in pure rotation about contact point at the instant under ''.reference point as, IC, _,v A = Ci then
consideration. The body appears to rotate about the ·- --~ - '

instantaneous axis.
When a body rolls along a
surface, the point of contact is
instantaneously at rest. We can vo.= roro11c
i .,
think of the whole body as rotating
about this fixed point, shown as
the instantaneous centre of
v)
1
Vs= rora11c j
i
i
rotation (IC), 0 in Fig. 5. 90. This
point is referred to as IC because at
some later instant of time a Fig. 5.91
different point on the periphery of _, _, _,
the body is in contact with the _____ Fig. 5.90 (a) .....
VB=(I) X 1"8/lC
surface. ·
In Fig. 5.90, (is the instantaneous centre of velocity __ f'1Bf= rorB//C
·such that angular velocity with respect to I is given as
www.puucho.com

.
Anurag Mishra Mechanics 1 with www.puucho.com


where, oo is the angular velocity of the body and r8;,1c 1 V.A
rNIC =(J)-
is radial . distance of B w.nt. JC. Due to circular!
VB
rB/tC =-· .,.(2)
motion, the direction of 'it B
must always be1 0)
. I
. ·/
perpendzcu ar to ->
rB/C· :I rA/lG + rB!IC = d ... (3)
2. Location of the IC: Use, thefiI.ct that the wlocity, In Fig. 5. 94 (b), eqn. (3) becomes
of a point on the body is always perpendicular to the!
relative tpsition vector from IC to the point. I rB/IC -rA/IC = d
IC

IC
v 1c= 0 j
Ol

Fig. 5.92

(a) Ifthevelocityv A ofapointAand angularvelocity(J)


. .-+
of the body is given. IC lies on perpendicular to 'VA
. ~ig. 5.9_! (I>)_ .
and iS' at a distance rA/JC = VA from A. I
0)

(b) Lines of action of two non-parallel velocities v Aand


-> . 3. If the object is translating A and v v
8 and IC isj
I
-:, . ! located at infinity, in that case I
v B are known._____ . r A/IC = rB/IC -> op
-> ...
Also, ro= VA = v. --) 0
rA/IC rB/IC ;
4. Concept of instantaneous centre can only be, used!
for an instant of time, because orientation ofbody is;
changing. ·
' 5. Note that IC does not have zero acceleration,~
l · therefore it should not be used for finding the
··-· -~!)· 5.9~ .I I
-+ -4 ; accelerations.Pf PQint§..iV a /Jody. ______
Construct line segments perpendicular to v A and v B·'
The point of intersection of these line segments Velocity of a Point of the Rolling Body
locates the IC at the instant considered. The velocity of any particle P on
(c) Magnitudes and direction of two parallel velocities a rigid body may be expressed as
-+ . -+ " . l -> -> ->
v A and v 8 are known [Fig. 5.94 (a) and (b)J IC is Vp = vo+Vp/o
located by similar triangles,)n Fig. 5.94 (a), where v O is the velocity of
translatory motion i.e., the net
velocity of the centre O of the body

i
d
and ~P/o is the velocity of P relative
to 0.
Fig. 5.95 (a)

Fig. 5.95 (a) shows a disc that translates as well as

l
rotates.
re11c

Ii ~/
[_______ !__ Fil!:_5.9_4!:>._________ _ i
www.puucho.com
Anurag Mishra Mechanics 1 with www.puucho.com

481 ·

:
/l __ky p
Case n. If v > wr then frictional force (FK) acts
in negative direction.

cM,:9--+v 0

(b)
....

1,
/ __ .,--·-n··:---
'
p·.

''--+
:'
:•
:i '. .
•• · · · : : · · - · - - . ~ ~ -
0
·.-c--.:
••
.' -~~VPground
... ""7
:,Vp.CM
""7

'
8
Fig. 5.97 (a)_
~ CM } VP,CM If v < wr then frictional force (FK) acts in positive
I ... , ....

' ··.. ..· direction.


If v = wr then there is no relative motion between
Uniform circular motion
of P relative to CM contact surface but tendency of relative motion is there due
Fig, 5.95 (c) to mg sin 9 so F5 acts in negative direction.
Fig. 5.95 (b) shows a point P at distance r from CM. Case m. If v > wr then frictional force (FK) acts
Relative to CM it is in uniform circular motion. Fig. 5.95 (c) in positive direction.
shows resultant velocity of point P that is superposition of
two velocities ';J O and v P, CM.
.... .... .... /
+ve
VP, ground =VP, CM + V CM, ground
In special case of pure rolling,
iv'P CM!= rw
8
l.v'cM, ground I= Rw [rolling constraint] Fig. 5.97 (b)

I';JP, ground!= ~(Rw) 2 + (rw) 2 + 2Rrw 2 case If v < wr then frictional force (FK) acts in negative
direction.
=w)R 2 +r 2 +2Rrcos9
If v = wr then Fs acts in negative direction.
Similarly for acceleration,
.... .... .... Illustrations Based on Constraints
ap = a 0 + ap;o Illustration 1: A sudden impulse acts on a rod kept on
Direction of Friction in Various Cases a smooth surface; as a result, the rod translates and rotates
Case I. If v > wr then frictional force (FK) acts about its centre of mass.
in negative direction. J

s f---+ VcM +
(+ve direction)

Fig. 5.96

If v < wr then frictional force (FK) acts in positive


Resultant motion == Motion of CM + Motion w.r.t. CM
direction.
If v = wr then FK and F5 do nut act.

www.puucho.com
Anurag Mishra Mechanics 1 with www.puucho.com

.
482 - . -. MECHANlCS-Q
Velocity of any point P Concept: Immediately after, the CM of the rod,
on the rod is the resultant of '.accelerates vertically downwards as there is no external forcej
velocity of P w.r.t. CM and in the horizontal direction, and rotates about the centre of1
velocity due to motion of CM
as shown in Fig. 5.99. If
• ,mass. In CM reference frame the .rod appears to be in purei
,rotation. Fig. 5.102 (b) shows that the acceleration of any
point P lies below CM, p 1
point on the rod is the resultant of aCM and tangential,
[Vpi=vcM-rro VP.CM= roo 'acceleration is !'.a. Note that immediately after release, the!
2
If P lies above CM Fig. 5.99
.rod has no angular velocity, therefore no centripetal/
[vp I= VcM + rro :acceleration of CM.
Illustration 2: Fig. Constraint equation of A: AB the left string is fixed
5.100 shows a spherical ball along its length, net acceleration is zero.
in pure rolling motion on a· L
aAy = acos0-acM = 0
moving plank. Velocity of CM 2
of sphere w.r.t. ground is v 2 L . 0
and angular velocity about aA
X
= -ctSIIl
2
CM is ro. Fig. 5.100
Immediately after release, end A accelerates
horizontally.
- I
Concept: In case of pure rolling motion, velocity of Concept: Resultant acceleration of twoi
contact point of sph_ere ~-r.t. contact surface _is_zero. i
:rep~esenta~ve points has be~n shown in Fig. 5.102 (c)

Qr
y

B
Point on sphere'
Point on plank
~a.case
L:
I
B
Fig. 5.101 .ba ~
2
...,
VA.B =0
e k2
--- ----·>
[vAl=vz -Rro acM
~ctcos0
[vBl=v,
Constraint equation is:
[v,l=vz -Rro Fig. 5.102 (c)
Note that if ro is in anticlockwise sense, then
Case 2: If the strings are at an angle:
[vAi=v 2 +Rro
illustration 3:
Case 1: If the strings are vertical: A rod hangs
from two vertical strings. Suddenly the right string snaps.

y
Lx
Snaps Translation.

8
~l acM
e a,

-- + Fig. 5.103 (a)

Rotation ~ - ~ • + ·L;,
. 2a
(a) (b)
Fig. 5.102

www.puucho.com
Anurag Mishra Mechanics 1 with www.puucho.com

I·---RIGID-BODY MOTION
----------. /,
- ---- ---------------'------~-- 4831
Tsin 8 for velocity and acceleration of the CM of the bar as a
function of 0.
. T
X and Y:coordinates of centre of mass are respectively.

x =.
L
cos0; . e
y= L sm
2 2
. a,
_./a ··... dx L . de v =dy =_!:_case de
v =-=--sme-
Mg x dt 2 dt '"dt 2 dt
ay (b) (c) d2 x L . d2e d2y L - d2e
"x = dt2 =-2sme dt2 a =-=--cos0-
Fig. 5.103 Y dt2 2 dt2
- - -- ----
At end A, along the length of the string, a~celeration is _!:_ case (de) (de) _!:_ ,me(de) (de)
zero, i.e., 2 dt dt 2 _ dt dt

ax cose+!:asinB-ay sine= O
2
Illustration 4: Falling rod
. - - - -- --- --- --·1 ;-···-- - ------·. -- ---- .. - .--~-.----· - --- - ~-----
lA spool con;;ists of a cylindrical core an_d t;li!o identical solii
I ;heads. The core rolls without., slipping_ 'piong a rough·,
ihorizontal block with a con;;Ja»ivelocity·,v.~The radius of the:
:core is rand that of the heads isfl:(Fig: SE,16 (a)]. Determine:
1the in;;tantaneous velociry of points A.pmi'/3,Qn the rim. What'
I - , '.,.
A !points on the heads have an instantaneous:velc,city equal in
jrragmtJ!de _to (h~ ':'.~lodty of_t_h~.spool!'C!~?_'_.
1., 11 A VA

2 ·- . jl ___.;..;......,,------0,
''
I:
+ d
,: A
I
1.,1 :;
2 ! I.
Fig. 5,104
I/
t I'
:'
V
'I
(a) (b)
·: i Fig. 5E.56 Ii
I_, __ -- ---·

Solution: The instantaneous centre of rotation is at the


point of cbntact C. The velocity of pointA relative to block is
vA =v(R;r) [asro=~]
Fig. 5.105

Fig. 5.104 shows a rod falling in the vertical plane on a Vdocity of point Bis v =v(R ~ r)-
8
horizontal surface. End A of the rod is constrained to move
along the horizontal surface. Therefore, its resultant velocity Points on a circle with radius r whose cenue is at point C
normal to the surface must be zero, i.e., have an instantaneous velocity equal to that of the spool
vCM -
L
-ro cos0 = 0 'core.- . -. . ··---··· - --- .. - .. - ~-------- - . -
2 Concept: Fig. SE.56 (c) shows a i;;bbin that can!
L -
vCM = -rocos8 lroll without slipping on a horizontal surface. The point o/
2 ;contact with surface C is the in;;tantaneous cent,·,? of:
Illustration 5: A uniform Acceleration; ro~a_ti_o~: '
rod of length L stands vertically 0 I
touching a wall; its lower end i
begins to slide along the floor 'I
(x-axis). Obtain an expression - _j

www.puucho.com
Anurag Mishra Mechanics 1 with www.puucho.com

-~-·-·· ____ ~ -~-~ ...:.~.. _M!c~~NIC~:!J


·,I K =.!LmV 2
=.!Lm-rv.-v .)
V 2 (
I
l l l I
2
= .! Lm;(v CM+ VI)· (VcM+vn
2
I' 1 -2 * 1 *2
I =-Lm·VcM+vcM·(Lm-v)+-Lm·V·.
2 I I I
2 l l

I Fig. SE.56 (c) Now L mi = M, the total mass of the body.


Note that, from definition of the centre of mass
Angular velocity about C = _v_ in Fig. SE.56 (a) rcM = ..!_ L mi ri = 0 if the CM is at the origin, therefore its
R-r M
= _L in Fig. SE.56 (b) derivative L miv, = 0.
R+r 1 2 1 •2
K= MvcM+ Lm,v, ... (1)
Velocity of rods (point 0) = vR in Fig. SE.56 (a) 2 2
R-r Eq. (1) obtained above is general. We can apply this
= vR in Fig. SE.56 (b) equation to a pure rolling disc. In this case the axis of
. .. .... .• R. +.r . . . -· .,_ - rotation is fixed in direction (perpendicular to the plane in
Total Kii:,Eitic Energy of a Body which the body moves), although it moves along with the
Derivatio~-".~f !< = KcM + K,01 centre of mass and so is not fixed in position. For each
particle,v; = roR;, where R; is the perpendicular distance of
The total.kinetic energy of a moving body is equal to the
translational kinetic energy of its CM plus the kinetic energy the ith particle from a line passing through the CM and
·associated witn motion relative to the centre of mass. Let perpendicular to the plane of motion. Then
rcM = XcMi+ Y,c~J + zCMkrepresent the position of the CM 1 •21 •221 2
-LmV- t I =-(LmR- I I )ro =-IcMOl
at any moment in some inertial reference frame. Let 2 2 2
J
r1 =xii+ y + z,k be the position vector of the ith particle of and
1 2
K=-MvcM+-JCMro
1 . 2

mass m, in this inertial reference frame, and let 2 2


rt x; y; J z;
= i+ + k be the position vector of this particle [rigid body, axis fixed in direction]
with reference to the CM (may or may not be inertial In this equation, I CM is the moment of inertia of the
reference frame), Then (see Fig. 5.107): body about an axis through its centre of mass and
1-··- .--- ·~--··-·- - . -·~ y,* perpendicular to the plane of motion.
y
1·· Co~ce~~:--Th~ .. tot~! . kinetic energy qf a bo,jy!
, Yt ••••••• ..•••••••• 0
· · - · · · · · · ••
imoving in a plane with both translational and rotational;
motion, such that the rotation axis does not change direction, I
i 1is the sum of the translational kinetic energy of the CM plus·
!the rotational kinetic energy about the CM,

.!:.2 rCMro 2 •
L .. .. ........ ... ·-.
'.:Y ....... r-.:=,

~ --~·~~ -~<"·~=-·-g. .I...__J~


,EE_xg,m;gl
r·------
57 l,07:>-
-• - --- - • I

:A ball ofradiu.s Rand mass m is rolling without slipping on al


Fig: 5;107 . .. l/wrizonta!-.surface with velocity of its centre of mass uGM. It:
-) -+ -+.,, -) -) -i, --¼ -) -+
ithen rolls without sliJ)ping up a hill to a height h before;
XcM+xi, Yi= YCM:+Yi' and Z1 =ZcM+zt.
Xi=
. . to. rest.
I'momentarily coming --·- Find h. . . .:
The velocity of the ith particle in the inertial reference
frame is
i
I
1~I
.., I Fig. SE.57
where· v CM is the velocity of the CM in this reference L.
Solution: Problem Solving Strategy: Mechanical
frame and v, is the velocity of the ith particle relative to the energy is conserved because static friction does not perform
· CM. We can use the vector dot product and write v 2 = V· V, any work. The initial kinetic energy, which · is the
so that the total kinetic energy K is translational kinetic energy of the centre of mass, .!.. mv ~M,
www.puucho.com 2
Anurag Mishra Mechanics 1 with www.puucho.com

.RIGID BO.DY MOTi.ON ~':-45~


l.~~-~--....:....-..;.....;...~--~.___--;::==::::::::====::::::;:~:::::.:::::::::::::·
~.
= ·', '',
-- --··-- - - ·- ~--------1
plus the kinetic energy of rotation about the centre of mass,
--1:rCMco 2 , is convened ~o potential energy mgh. Since the
2
ra;~~X,~~{~c
i
through which thecyliMer rotatdbefoie its(
leaves con.tact with the edge, .' · _ • · • . . -i
sphere rolls without slipping,- rolling constraint equation is (b) the speed of .the centre of mil§s of the cylinder l1efore/
applicable, v cM = Rco. I leaving contact with the edge, and -- I
Apply conservation of mechanical energy with U; = 0 (c) the ratio of the translation to rotationalkinetic energies!
andK1 = 0.
Ei=E; Ui=K;or
The total initial kinetic energy K; is v CM and angular
L O! the cylinder _when its centre of.mass is in ho,rizontal./ine/
_with .the edge. ________ - .
Solution: (a) As the cylinder rolls without slipping
- •

~~dD • about an axis passing through CM, hence the-mechanical


1 2 I 2 energy of the cylinder will be conserved, i.e.,
K; = mvcM +-z1CMco
2

I • ~'~-u)
-v - ·2
Substitute co = CM and I CM = -mR 2 and solve for K;
R 5
in terms of the mass and v CM.
2
1 2 •)
K-=-mvCM+--mR
' 2 2 5
2)(VCM
-
R
(2 ) After break

·
,, of contact
Sidevlew
7 2 1
= 10 mvcM
l
.(1)- (2) · (3)
. ___ • ~ Flg_.s_E_•.s_s-"./b_,>~._ _ _ _ ___,
Set this initial kinetic energy equal to the final potential
energy mgh. (U + KE1l = (U + KE2J -
1' I'
7
-mvcM 2
=mgh mgR+ 0 = mgRcose+-Ico 2 +-mv 2
10 2 2,
But V - d
co=- an I=- 1 mR2
h = 7v~M R 2
10g Therefore, _
Analytical Skill Problem 2 2
Mark correct .option: mgR = mgRcose+½(½mR )(;: )+½mv
(a) The height to which a ball rises is independent of v2 4
radius of sphere. or - =-g(l-cos8) ... (1)
R 3
(b) Heavier and larger square rises to a greater When the cylinder leaves the contact, normal reaction
height. N=0ande=e,.Hence ·
2
( c) Friction force exerted by surface is static friction mv
mg cose = - -
and no energy is dissipated. ' R
(d) A spherical shell will rise to a greater height. v2
or -
R = g case C ... (2)
~}lllJ!?:!J~> From eqns., (1) and (2), N
~ r;cta,,;,iar ri:gid hed block has, a, Jong horizo~tal edge::AI .ig(l- case;)= g case,
3
'
'soUd homogeneous cylinder ofradi_u.s E,,is.place.d honz.
· ovt·a·'.1/y-.l 4
lat rest with its length parallel to the edge such that the axis bf,) or cose =-
' 7
.the cylinder and the. edge of the block. are·in· the same. ve.rt,.'cal
. . .•. •
Iplane as shown in Fig. SE.SB (a). There is sufficientfriction or. . e, = cos·
1
(-~) mg
!present at the edge so that avery S1J1.all displacement caus~s
I the cylinder to roll off_ the edge without slipping. · At the time it leaves the
Fig. SE.SB (c)_~

. ' contact,
. cos0 :::::! cos0c = 1,
7
/"'
(b) On substituting it in e n. (1), we get

i Fig. SE.SB (a)


4
v. = . -gR(l -·- = ~R-
3
-g .4)7.. 7
L
www.puucho.com
Anurag Mishra Mechanics 1 with www.puucho.com

There.fore,J·peed of CM.of cylinder just before it leaves Solution: In accordance with conservation of
4 ., . mechanical energy, the kinetic energy of the sphere:at the
the contact is - gRi ·· · · ·
. - 7 . . top position in the loop is equal to the decrease mgh "in
(c) At: the moment,· ~hen the cylinder leaves the potential energy as it falls· from the initial position to this
contact, position. The kinetic energy of the sphere is composed of
,,, ''
. 14:: two parts; the translational kinetic· energy of the sphe,re and
Y=fjgR . the rotationalkinetic energy of the sphere about its .centre of
mass. Let m,.K, U,ro be respectively the mass, kinetic energy,
Therefore,, rotational kinetic ·energy,
1 2 velocity of the centre of mas~ and ,mgular velocity _about the
. KR·=-Iro
2 . centre of mass of the 'sphere. ,Then

or
;>
KR,"'.!
· , 2 t2
mR 2 (2 )(!C_)
R2
= '.!mv
·4 .
2
_
K =.!mv 2·+l!ro 2
2 2

.
~ mr~.
0

., =-m
1 (4-gR) ·with 'I '=
.5 .
As the sphere tolls without slipping
., , , 4,, 7. ,v =ror and
, mgR 2
or ,, KR=-- ... (3) ·1(•2:2·2•V 7
. . 7 K=- mv +-mr - 2 ) =-mv. 2
'.' 2 . 5 r 10 · ·
When tlie. cylinder loses its contact, i.e.; the frictional.
force vanishe~, also-torque due to gravitational force is zero. In the critical case when- .the sphere Just negotites the
Hence its angular velocity is constant and rotational kinetic ' loop, the force exerted by the ·1oop. on the sphere is zero
energy becomes constants, while its translational kinetic when the latter reaches the top of the loop. The centripetal
energy increases. . .. force needed for the circular motion of the_ sphere is s11pplied
Applying c~n'servation. of energy at position (1) and . entirely by gravity · ·
position (3), · · mv 2
-·-=mg , where v 2 .;,, Rg
Decrease in gravitational PE = Gain in rotational KE + R . .
translational KE and K = 2. mRg =mgh.
10 ·
or Kr.=CmgR)-m;R=imgR
From (3) and (4);, we have
.... (4)
Hence h = i~ is the minimum initial height required.

6 . Dynamics of Rigid Body in. Plane Motion


~mgR
Kr =-7_.- or When various forces act on a rigid body; the resultant of
KR mgR all the forces may result in acceleration of centre of mass
.7 and angular acceleration of rigid body about an axis passing
through centre of mass.
.··- ··-·.-;··1
. .. .

. '
'

I
I
I
l
- ry,
! ,c·t' , , ' · en

R~sultant of lorce~has.alrans)a.~qr\'ofcM and.relation al>oOt,CM


, ,:Fig. 5.1Q~
~-• ___ _,,_,.,,.,.~-·-=··---· r -

_The equations of motion are


I,Fx = max
LFy '= may
L~ = Ia

www.puucho.com
Anurag Mishra Mechanics 1 with www.puucho.com

RIGID BODY MOTION ------ 4571

where The moment of inertia of the disc


LFx and LFy = Algebraic r a'y to use here is the moment of inertia Rn
summation of the components I' I CM of the system about the point
of the external forces along x- where we calculated the torque.
and y-axes respectively.
'I Along the length of the string,
Lt = Algebraic
summation of the torques of
external forces about the
I a'
'
l
'' ~
acceleration must be. zero, i.e., the
resultant of aCM and Rex is zero, i:e.,
aCM = Ra. acM
centre of mass,
I = Moment of inertia of
'
IL---· -- Fig. 5.109
I The equations of motion of disc are _. -~I_!!· 5E 060 (c)

the body about the mass --- Lt=1R=ICMcx=ICM(;) ... (1)


centre.
LF=mg-T=nia ... (2)
Note that we may take torque about a point O whose
acceleration either is zero or is directed through the centre Solving for a, we obtain
mR2
of mass of the body. a=----g
Lt=I 0cx mR2 +IcM
Component of force in the direction of acceleration is Choose the origin at the point where the string is in
considered positive. contact with the disc. With this choice the torque of the force
If the sense of torque is same as that of the angular T of the string is zero since its line of action passes through
acceleration, it is considered positive. the chosen origin. Thus the total torque on the object is the
torque of the weight.
k·Exam~ :Et=·mgR
In this case moment of inertia is obtained from the
~ solid disc of mass M has a thi-~~;;:i;~rap;ed several timesi parallel axis theorem.
;around its circumference. The string is fixed at one• end and .· I=ICM+mR
2
'the disc is released. Detennine the magnitude of the 2
downward acceleration of the mass as it falls. Lt= mgR = UcM + mR )cx ... (3)
From eqns. (2) and (3), we obtain the same result for a.

T
System a=(mR~:CM)g
Wustration: A uniform sphere rolls down an inclined
plane rough enough to support pure rolling.
Acceleration of the rolling body can be determined by
considering its pure rolling motion as superposition of
mg rotatory (about the centre of mass) and translatory motion.
(a)
Rct
For translatory motion,
mg sin 8 - f, = ma ... (1)
For rotatory motion.about the centre of mass,
,- --
+ •

I
( acM acM acM Ra '
I. Translation and rotation{b)
L..._ __,__ _ _ _ _ _ _F;.c:,i!!~~~~------
II mg cos 8
.. e·········-··············
, Fig. 5.110 i
Solution: We choose the centre of mass of the disc as I__-------------·----
origin. The weight has zero torque since its line of action 'tc.=lca
passes through ~e chosen origin. Torque of tension, fs ·r=l,cx
t=1R a ... (2)
f, =I,2
r
www.puucho.com
Anurag Mishra Mechanics 1 with www.puucho.com

2
Pure rollirig constant, a =-a . For a sphere, k=-
·r 5
From equations (1) and (2), we get 5 ·0dJ 2 ··0
acM =-gsm an, =-gsm
· mg siri0 g sin0 7 · 7 ,.
a=_ ( I ) = K2 For an object rolling down an incline without slipping, f
. m+--'- l+-
r2 r2 - is less than the maximum force of friction.
I C -~mI(2 . JSµ,N(=µ,Mgcos0) ·

If moment-pf inertia is expressed as, I,= cmr 2 _k_Mg sin0 s µ,Mg cos0
l+k
where 'c' is ·a numerical which is different for different
shapes of bodies, . µ
s
~ (-k-)'tan0
l+k
' .
gsin0 2
a=-- For a sphere µ,-tan0 ~
l+c 7
From the above equation, 7
a Nmgsin0 tan0 S µ,
f, =I, 2 , f, 2
But
. '
. r

f, S µ,mg cos0
Nmg sin0 ·
--"'--- < µ,mg cos0
_
l+c
f, S µ ,N· where N =mg cos 0
_-,-~~i;j:!:~~e~t oJ~~e d~Ie
, :· · ~l~;, the· ~phere
pji~:lin_e

1¥illf'.f .a: it
~-~~v:~:~d:;~;~~i~S:~hll
.l+c
µ, ~
c+l
(-c)tane .... (A) 2
' ' · · . Yf!J!in~e :;u~~to/t~iti~~;;,lli~Jatrii!,~
This is· the condition for p).lre rolling of the body. I., Jii:effantct,ieously: i1 ?ontac,trtli ,th_e inclin&, ¼CXp1f:~-m
For ex~ple: For a· solid sphere .c =(~) , therefore ' ~
· We i qa71not substitute• f,r:;,µ kN m _the absence,, ofi .
'.'"• ". _0 \", _ ~ :_·_, •, 0 ,;__·." , ___ L
I''., ,; ": , _ .} ·:"--,/S\l'.,"P1:\~,
. . . . 5
. • slip11inlf. Wecannottisef-;;rlf,N either,._becaus~f'(uite
µ, ~ (~)ta~0, _hence coefficient of friction should be . is n~'.'zogic•to sugges{tltd(tfidforte "offrictlotJ:~l~b~
7 . . ·:_maxift1m1L ,Therefor~. ,;/,</re~_ of {ricqon,'~["~n
.· indep~rident unknown;;it'is,it(lti_c friction saqsfyingi
_greater than (;}'."1afor pure rolling of a solid sphere.
, -ols[Ffr,c\·sµ,N.·. , , r
·,:--_,_.. ·
~ ~ 0 ,
. · ·'',\;
'c
1
7,"h' , {
.,.~ • '
j
Case I. If-µ, = 0, then the body will perform pure 3. ' When h round object;is J,~leased, the poi'niA':On tlie
sliding and. its acceleration will be maximum. -
•, sp·h·.·ere_ _,_·n_contg·c..t (i'!,5. ta_n. 11.tl. heous. ly) "ll e_ 1Ja:_to_ ~l_i_?e_l
___ t.

· . down the plane, which is.prevented by frictionfgrce;


WI.

-· · e;gsin0 ·
a_ max-

. Case II.' If O < µ, < _c_ tan 0, then the motion will
.thus, /orcitof friction! pfi.in4 up' the, incline,'rekji'fng' 0

. . c+l . .tlii ilo'(Vr,ward motion·oJA.and thereby ienerdtl.ng-a


be rolling with the sliding and then ' ' torque about centre of majs, which turns the sp1;tete:
· · .gsin0 . It is force of friction that causes the rotatio.n df the
·--<a<gsm. 0 -~ J·,"' _i,bJiCtt-: _ :!<'.::' ::, ' , ... ~;t
c+l -
. 4. .As,_the;. object accelerclt?s '"'1own · the in~li!)e; )he
Case III. If µ, ~ _c_ tan 0, then the motion will be 'cingulcir velocity or rotation' must . iricrei!tse to
c+l
. pure rolling and the acceleration will be minimum.
inatriiatn pu~e rolling cdiiien~ of slippitig)O'{-.. j
· g sin0 . 5.' We,d~ not take frictioiiJ;to account in, tli;e~nergy~
amin = - - . eqµlltipn, because 'it. 1sit<1/i4 friction 'ana:,d!?~,:n~·
f . -.
.
c+l worlCForr:e of friction. dci:fpC1Tallel to·tlie pldii~J:irid
One must be careful about sign convention for torque. .. \. . ' the J,oi,.;t .of contar:fdg~ n~t sUde; .rathen;,,(t\oves

L
Imagine that the torque under consideration is the only pe'rpendii:ul'!r to,the p/d)te'((nitiallf do1'i(i Cllj~t,lienj
torque acting on the body. Judge the sense of rotation due to
this torque, and the direction in which the ·object will try to
:¼eJ;cis,th_ e sph,-er~ _rolls: tf!.f1r,~e[~ri~ disP,liff:ef~~6ar~,
_p?,:PJmdtcUlar,_noworlq,;]lone. , ,,.,.,,,, j
translate due to torque. If the translation thus observed is in
positive translation direction, assign it positive sign. If the Consider two cylindrical cans full of tomato· soup rolling .
torque under consideration (acting alone) tends to roll the · down an incline. One of the cans has thick soup, Each starts
object in the negative direction, assign it negative sign. with the same gravitational potential energy which is
converted· into KE.
www.puucho.com
Anurag Mishra Mechanics 1 with www.puucho.com

-~-·--,
r·RIGID eoDv riionori ------ -·--·------"
4891
--------------·~~---- -----~
KEroll. = KErrans. + KE rot. where 8 is the angle of inclination, 'm' is the mass, 'r' is
the radius and I c is the moment of inertia of the body about
the centre of mass.
R Case 1. If the sphere of mass m1 is filled with a liquid of
A
• mass m2 ,
(m1 + m 2 )g sin8
a=
Fig. 5.111 2 2
-m 1r
When the can rolls, part of its KE is rotational and part (m1 + mz) + ~3_ _
r2
translational. The thin soup does not rotate whereas the
thick soup does (it can stick to the can). The can with thick Case 2. If the liquid inside the sphere freezes,
soup has more rotational KE than the can with thin soup, m,
therefore it rolls slowly.
Comparison of Pure Rolling and Pure Sliding
Pure Rolli~g Pure Sliding;,:)(
1. Acceleration g sine a=gsin8 Fig. 5.113
a=-- - - - - - ·--
n+l
_ g sin8
-1+ k2
. r2
2. Velocity 2gh V = .j2gh
v=
--;;, Concept: - --- . - . -------- -!
\ 1+-
• r2
( ' l. Two uniform solid spheres made of different materia~:
3. Time taken to I unequal mass and unequal radii are released from the;
cover the. distance same height on a rough incl_ine, The spheres roll without 1
s, Where k is the
radius oft =
~(1+~J slipping on the incline. The velocity of CM at the· base
given by
is:
gyration. g sin8
VCM = ~1+-
2g~-2
The comparison reveals that r2
arolling < asliding where k is the radius of gyration
V rolling < V sliding k2 2
For a given uniform sphere, - irrespective of
trolling < t sliding -;:-, -?
Illustration: Acceleration of an object pure rolling on mass and radii.
an incline is given by ·The two spheres will reach With the same velocity and
take the same time to reach the base.
2. (a) If a solid sphere and a hollow sphere of- the same
mass anii same radii are released, the hollow sphere,
will take a longer time and acquire less velocity. .
(b) If a body with circular cross-section having moment',
s h, of inertia I CM = nmr 2 is released from the top of an;
incline and rolls without slipping, from conservation!
of energy, we get
2
I
Fig, 5.112 mgh=Imv 2 +I(nmr)(:i!.) '
2
-- - - . -
2
- - - r-- i
mg sin8 ~

a=~--
m+ le
r2

www.puucho.com
Anurag Mishra Mechanics 1 with www.puucho.com

i490
,__ . ,
- -MECtff~~s:!_]
-----·----------
1 2 mgh The equations of motion are
or KE translation = 2 mv = 1 + n rEx = P - f = Ma ... (1)
1 nmgh 2 :rF =N-Mg=-0 ... (2)
or KE rotation =- nmv :::: - - y 1
2 l+n :Et =Px h+ f xR = -MR 2a ... (3)
(c) If a solid sphere and a . hollow sphere (same mass: 2
and radii) are compared, the solid sphere' y
acquires more translational kinetic energy;;
whereas the hollow sphere acquires more·
rotational kinetic energy. '
3. (a) A sphere cannot roll on a smooth, fixed inclined I
plane even if placed with velocity v and angular:
velocity ro that are related as v = rro; the I
gravitational force will increase v whereas .OJ.
remains constant in the absence of frictionaf
torque. Fig. SE,61

Here we have assumed clockwise torque to be positive.


If the cylinder rolls without slipping, a = Ra.
1
Now, eqn. (3) becomesP x h + f xR = MRa ... (4)
2
From eqns. (4) and (1), we eliminate a to obtain
Fig. 5.114

(b) It can roll without slipping if the incline;


3f =P (1-:)
accelerates with a= g tan 0. In this case, in the; Note that force of frictjonfis zero when (1- 2h/R) = 0
reference frame of the incline, the resultant force: i.e., h = R/2
along the incline becomes zero. i Thus rolling motion of cylinder may take place even
4. (a) A solid sphere, a hollow sphere and a disc 9fthei without friction.
same. mass and radius are released from. the top: If P is applied at h = R,f = -P/3 which shows that
of a smooth incline. All the three bodies Willi
friction force has reversed its direction; it acts to the right.
reach the bottom at the same time. They slide!
like a block
For f = -P/3, eqn. (3) yields

V = ~2gh; t = ff PxR-]:_PxR =]:_MR 2a


3
a=--
2
4 p
(b) If the three objects are released on a rough' 3MR
incline, such that friction is not sufficient for, a is positive, which indicates that cylinder rolls to the
pure rolling, still the o/,jects reach the bottom at, right.
the same time because acceleration of centre of,
If force P is applied at the centre of mass, h = 0.
mass is the same in each case. Each object has'
3f = P[l - (2 X 0)/R] =P
same v CM but different ro . . _J
f=P/3
Friction force is positive, that means, assumed direction
of friction force is correct. As P is increased, the force of
friction also increases; for large P it may exceed maximum
·A horizontalforc'e.P is applied at a height h above the centre!
possible value of friction µ ,N, which is not possible.
·of a solid cylinder of mass M, radius R. Determine force of
Therefore we will have to take a new assumption that the
friction in terms of hand show that friction force is zero for h_
cyliner slips. The equations of motion are now
= R/2. Assume. that the cy}inder ro)ls without s/jpping. ·
:r.Fx = P-µkN A= Ma
Solution: As you have learned during rolling motion, :r.Fy = NA - Mg = 0
friction force is static; its magnitude as well as direction are 1 2
unlrnown. Therefore we assume friction to act towards left. :Et=Pxh+(µkNA)R=-MR a
2
If its value is found to be positive, our assumption is correct,
otherwise we rewrite the equations.

www.puucho.com
Anurag Mishra Mechanics 1 with www.puucho.com

4911

' --.-.--.-----,.-,
lA cotton reel has a hub of radius .r and .end caps of radius. R,
:Mass of the reel is M and iniiment of inertia about
\longitudinal rotational axis.is L A force Pis applied at the free
•end of thread wrapped on the reelas shown in Figs. SE.62 (a)
'and (b),_ ______ · _ _ _ _ _ ----·- - - - ~ ·
N
' - - Fig.
-- ___
SE.63 (a) ,
(a) What is .the·linear acceleration of the wheel question?
'
YLI x·
j(b)fa/ctilate the frictianalfarce thgtacts an the wheel. __ _
Solution The equations of motion of the wheel are:
F- f = ma ... (1)
f p Fxr+fxR=la ... (2)
p
In case of pure rolling, a= Ra ... (3)
---,
(a)
Fig. SE.62
Mg
(b) r-- ·N '

'ca) Determine the frictional force exerted by the table oii the
, reel and the direction in which it acts.
(b) Jn what directiJLn does, the reel begiR to. move? _ .
Solution: We assume that the reel moves to the right.
Force P exerts anticlockwise torque; therefore friction force
must exert a larger clockwise torque to produce clockwise
rolling. The equations of motion are '
:r.Fx =P-f=Ma ·-. .•. (1)
F x r+ (F-ma)R = !.a
:r.Fy=N-Mg-=0 · ... (2) R
L1 = f X R - P X r = Ia ... (3) F(r+R)R
or a=-'---'--,
As the reel rolls without slipping, (I+mR2)
a=Ra and the. frictional force is
From eqns. (1) and (3),
P-f =R(fxR-P;<r)
M I
f = F- ma= F[l- (r +R)R 1= F[(I/m)-Rr]
2
(~ +R2) [(l/m)+R ]
f = P(I +MRr)
l+MR 2 For I/ m = Rr, frictional force is zero.
Force of friction comes out to be positive, therefore our For I/m > Rr, frictional force is positive, Le., it acts in
assumption about the. direction of friction force was correct.
backward direction.
On substituting the expression for f in eqn. (1), we
For I/m < Rr, frictional force is negative, Le., it acts in
obtain a= PR(R ~ r)
I+MR 2 forward direction.
As R > r,, a positive and ·towards right. Similarly from
eqn. (3), we obtain a =·P(R - r)
· l+MR 2 'A spherical biill of mass M and radius Ris projected along a
which is positive, i.e., net torque on the reel is clockwise. rough horizontal surface so that'inttialiy (t = O)'it slides with
a linear speM. v O but does not rotate. As it slides, it begins ta!
L;¥~~,~ciiil~@J> spin and eventually rolls without slipping, How long does itl
l@ke t9 bggin]dllirig.!l!itlwut slipgi_r:ig ? _____J
iA wheel of radius R, mass m and moment of inertia I = mR 2 Solution: Equations of motion for translation of
is pulled along a .horizontal swface b.Y appUcation of force F
'i'

ta a rope unwinding from the axel of radius r as showr, in sphere are


'Figs. SE.63 (a) and (b). Friction is sufficientforpure rolling :r.Fx = -f = Ma ... (1)
:r.Fy =N-Mg = 0
j, ' ' '
of the wheel: . , , · ... (2)

www.puucho.com
Anurag Mishra Mechanics 1 with www.puucho.com

I --- -
- - -MECHANICS-I 7
1_"!__9~_ - - -- ------ --------------------- -------- ___ ___, ,

From eqns. (1) and (2),


a= -µkg ... (3)
N y

Lx
CM VcM
' Fig. 5.115

[~~~I?~>
r- --- -- ,- --- - - - -- , - - ---- -- -- ,
Mg , ;A uniform disc of mass m and radius R is projected'.
lhorizontally with velocity v 0 on a rough horizontal floor so-
- -Fig. 5E.64
- --·
The velocity of the centre of mass at time t is
lthat it starts off with a purely sliding motion at t = 0. After t O '
!second, it acquires a purely rolling motion as shown in Fig.!
vCM ;:;; v 0 + axt !_SE.65. (a)_
=Vo-µkgt ... (4)
Equation of motion for rotation is
2 2 ----+-Vo ----+-V
fa= µkMgR = -MR a ... (5)
5

or ... (6)
t=O t = lo
Then the angular velocity of the ball at time t is
Fig. 5E.65 (a)
ro = ro 0 +at= O+ Sµkgt ... (7)
2R '.W Calculate the velocity of the centre of mass of the disc att 0 .'
When the ball comes into contact with the horizontal :(ii) Assuming the coefficient of friction to beµ, calculate t 0 • •
surface, the friction force has two effects. Friction force acts ' Also calculate the work done by the friction force as a
to decrease the linear velocity of the CM (slow down the functional of time and the total work done by it over a
translation motion). Secondly, friction tends to rotate the ___ _p,n_e t mush longer_than t 0 •
ball clockwise. The sphere starts rotating immediately after Solution: (i) During the time interval t =O tot= t 0
it touches the ground, but it rolls with slipping. It eventually'
stops slipping at the instant the contact point has zero there is forward sliding; so kinetic frictional force f acts
towards left. For time t = t 0 , frictional force f will be zero,
velocity w.r.t. ground and centre of mass has velocity
because there is no relative motion between the edge of disc
v CM = Rro . The condition for pure rolling is that
and surface.
VCM = roR
From eqns. (4) and (7),
Vo -µkgto = SµkgtO R
2R o-/--+-v;
where t O is the time at which pure rolling starts. "' I'
2v '- l
So t 0 = - 0-
?µkg f = fmax = µmg f =0
t = lo
the't'
t= 0
Concept: Note that the force f acts to decelerate Fig. SE.65 (b)
sphere from the perspective of transfotional motion, butj So, for time t < t 0
,torque of friction increases the angular velocity. In fact all the I Linear retardation, (f = µmg)
,bodies are deformable; therefore the sphere flattens slightly.
;and the ground surface also acquires a slight depression at the 1
'contact point. The normal reaction, instead of acting at al (X
.point, acts over a definite area. ' 84-t---+-.
I The torque exerted by normal reaction slows down the
:rotation of the sphere. The normal reaction N acts at a
1distance l_infrol'!t of the CM as shown in_Fig. 9)J5_. ____ _J f= µmg
~ig: SE:.65 (c)

www.puucho.com
Anurag Mishra Mechanics 1 with www.puucho.com

RIGID BODYMOTION - 493 I


' - ------·-r:7-...
and angular acceleration,
~
a=-=---=-
JR 2µg ~~q,,_~~.Jli~:J 66
~--- --"----------~---·
v -·-.-,-="1
I .!.mR2 R . Two thin drtular discs of:.mass 2 kg and radius 10 cm each!
2 are joined by a rigid massless rod of length 20 cm. The axis ofl
Let v be the linear velocity and ro the angular velocity of the. rod is along the perpendicular to the planes of the_ disc.
the disc at time t =t 0 ; then through their centres. An object is 'kept ,on .a truck in such a
... (1) way that the.axis of the object is honzontal and perpendicular
to the directio"n of motion of the truck. Its friction with .the
and ... (2) oor of the triick is large enough so that the object can roll on
he truck. without slipping. Take X-axis as the direction of
From pure rolling constraint, we have
motion of the truck and Z-ax/s as the vertically upwards
v =Rro m/
itection. If the truck has· a_n accelerqtion 9 s2 , calculate:
i.e., v 0 -µgt 0 = 2µgt 0 [from (1) and·(2)] ~ (i) the force of friction on eqch ,disc.
Vo (ii) the magnitude arid direction of the frictional torque:
to= 3µg
acting on each disc about the centre of mass O of the objectl
Substituting the value oft O =~ in equation (1), express the. torque in the vector form in tenns of unit vectors!
3µg i,]andkinX, Y-andZdirections:
~-·-·-- ---~---"'· -- _, ·----~-~--
_,
----·---
!
····------·" •. J

we have v=v 0 -µg(~) Solution: (i) FBD of · any one disc is shown in Fig.
SE.66 (a).
2
V = -Vo
3
(ii) Work done by friction: Fort ,,; t 0 , linear velocity
of disc at any time t is v =v 0 -;- µgt and angular velocity is
ro = at = Zµgt . From work-energy theorem, work done by
R . _____ :.}~·-51:!~ (a)___ _ _ ~ j
friction upto time t is equal to kinetic energy of the disc at Frictional force on the disc should be in forward
time t minus kinetic energy of the disc at time t =0. direction.
1 2 1 2 1
W=-mv +-Iro --mv 0 -
2 Let a0 be the linear acceleration of CM of disc and a the
2 2 2 angular acceleration about its CM.
2
Then
=-m[v
1 0 -µgt] +- -mR
2 1(1 2)(2µgt)
- 1
--mv 20
f f '
2 2 2 R 2 a0 = - = - ... (1)
m 2
= .!.[mv~ + mµ 2 g 2t 2 - 2mv µgt + 2mµ 2g 2t 2 - mv~]
2
0
a=-=---
~ JR
W = mµgt [3µgt - 2v 0 ] 1 .!.mR2
or
2 2
For t > t 0 , frictional force is zero, i.e., work done by = l:[_ = _lj_ = lOf ... (2)
mR 2x0.1
friction is zero. Hence the energy will be conserved.
Since there is no slipping between disc and truck,
Therefore, total work done by friction over a time t
therefore
much longer than t O is total work done upto time t 0
(because beyond that work done by friction is zero), which a 0 +Ra=a
is equal to-

Substituting,
W = mµgto [3µgt 0
2
-

t 0 = v 0 /mµg, we get
2vol
or (

or
f) + (O.l)(lOJ)

f = 2a = 2 x 9.0 N

f=6N
3 3.
=a

,
~:1
i/ w- a, -,
W = mvo [v 0 -2v 0 ]
6 Since this force is acting in 1' .
I __ Fig. SE.66 (~) -- _
j
positive X-direction,
=-mv~ --+ -
6 f ='(6i)N

www.puucho.com
Anurag Mishra Mechanics 1 with www.puucho.com

i494
'--------------------------------
..., ..., ..., t
____
-·---- . - .. - .. -MECHA~ICS-IJ
fR
~------------~----- --·· ~· -----·
2f
,

(ii) t = rx f U=-=--=- ... (2)


...,
rp
...,
= r 1 =-0.lj -
- 0.lK,. [ .!c·mR 2 mR
2
..., ...,
rQ = r 2 = O.lj-0.lk
- Rolling constraint a = Ra ... (3)
..., ..., ..., Solving equations (1), (2) and (3), we get
t1 = r 1x f a= ~a 0 = ~(1) = ~m/s 2
! z 3 3 3
y Displacement of CM of cylinder in reference frame of
truck is s = 4m in time t given by
. ....,
X
f s = .!cat 2 =
2 2 3
t2 .!:(~)
Fig. SE.66.(c)
or t =.Ju~ 2--J3s
= (-0.lj- 0.lk) x (6i)N-m The linear velocity of the cylinder in reference frame of
truck is
= 0.6k-0.6j
--+ ~--+" " v =at= (~)2-13
3
= ....±.m/s
-/3
(towards left)
t 2 = r 2x f = (O.lj- 0.lk) x (6i)N - m r

= 0.6(-j - k)N-m Velocity of truck after time t,


..., ..., vr = a 0t = (1)2-/3 = 2-/3 m/s (rightwards)
[t2[ = [t2[ Hence at the moment the cylinder leaves the truck, its
r~~~ami;p1·~~
.
!6~> - . -
absolute linear velocity [ground reference frame],

OJ
A cylindrical pipe of diameter 1 m ls kept on a truck as shown:
in Fig. SE.67 (a). If the truck now starts moving with a: ~--+-•v = 21,/Jm/s
·constant acceleration of 1 m/s 2, the pipe rolls backward'!
•without slipping on the floor of the truck and finally fall, on i
the road. If the. pipe moves a total length of 4 cm on the floor! Fig. SE.67 (c)
.of the. truck, what ls the velocity of the pipe relative .to the 1
:truck and relative to ground at the instant it leaves confactJ v=v -vr=....±_-2-J3=-2m/s
'with the truck ? What ls the angular velocity of the pipe at the, r --J3 --J3
isame instant ? (g = JO m/s2 ). :
or v =-
2
mis (rightwards)
. - 1 I
-/3
_.a;
and its angular velocity is
ro = ~ = -/3 = ~ rad/s (anticlockwise)
4
R 3x0.5 -v3
Conceptual Illustration: Fig. 5.116 (a) shows a rod
Fig. SE.67 (a) kept on a smooth horizontal surface. A particle of mass m
m Vo

I~~2mIU2
Solution: Let a be the
OJ
acceleration of CM in reference : - :2 • c2
ma0
frame of truck and ex be its
angular acceleration about an (Pseudo
force)
c,.
axis through its CM as shown in
Fig. SE.67 (b).
Fig. SE.67 (b)
Equations of motion in
reference frame of truck are C1 : CM of rod After collision the
ma 0 -f=ma C2 : CM of roct system has VcM
and particle and ro about CM
a=a 0 -1._ ... (1)
m (a) (b)
Fig. 5.116

www.puucho.com
Anurag Mishra Mechanics 1 with www.puucho.com

[ RIGID BODYM-"-Ol_lO~N-----"-----------'-------~-------··:_-_ __
and moving with velocity v 0 strikes at the end and sticks to
A
the rod. We wish to set equations to determine the angular
velocity and velocity of centre of mass of the rod.
Method 1. From conservation of angular momentum
about centre of mass of the system comprising of rod and
particle, we get

Vpa,tict~C2 =Vo-(:::.)=:::.
Mv Before After
collision collision
vc,,c2 =-M+m
_, _, _, j Fig; 5.118
,,
(Lsystern,C2 )Initial = LP,C2 + LC,,C2 2

ILsystern,c2 l1nitiai = [ m(;::,) ~~ m +2(!m+ ~) 2 ]


2
0 = MvCM ----ro
L ML
2 12
... (1)

From conservation of momentµm, we get


_,
(Lsystem,c2 )Final = I SystemCO /.
rrl\. mv 0 =MvCM +m(vcM +½ro) ... (2)
2 2 2
I = m( ML/2 ) +[ML +M( ML/2 ) ] illustration: Fig. 5.119

-
M +m M +m A

r:
Sy5tem 12
...,
shows a uniform and thin rod kept
on a frictionless horizontal surface . Vo Ix
From Linitial
...,
= LFinai, we get ro 6mv
= ---- A particle collides inelastically with m
(M + 4m)L
the rod; coefficient of restitution is c,.
CM I ,
Method 2. From conservation of momentum, we get
mv 0 =MvCM +m(vcM +%ro) ... (1)
3/4. Consider v 2 = velocity of
centre of rod (rightwards), v 1 =
velocity of centre of. ball (assumed
c,

B
1;
' ', ''

:~- -- ---
,, (vc~ ~,J2 oo) l, ro
leftward) and = angular velocity Mass of rod ?- _5 m
Mass of particle = m
of the rod after collision
! (a) Write equations to Fig. 5.119
' determine the above parameters.
''
Applying conservation of mom~ntum, we get
I mv 0 = 5mv 2 - mv 1 ... (1)
I Applying conservation of angular momentum about a
' i
point on ground just below the centre of mass of the rod, we
!After colliS!on, v·cM repr'esents !
0

relocity.ofcentre of mass-of rOd i get


i_ __ _ Fig. 5.117 ·----·· , mv 0 (½- -mv (½- )+
x }· 1 x
5 2
~~ ro ... (2)
From conservation of angular momentum on a point on
ground just below CM of the rod, we get From definition of coefficient of restitution, we get
2 V2n -vln

mvo 2L = m ( VcM +2ro


L )L
2 + Uro
ML
'--,,---'
... (2)
e=-~~~~

U2n -Vin

Angular momentum [{v 2 +ro(l/2- x)}-(-v 1 )]


Angular momentum
of particle of rod = 0-v 0
... (3)
_, _, _,
Note that LTotal = Lspin +. Lorbi~I. (b) If velocity of end B just after collision is zero,
_, _, determine x, the position of impact point .
Lspin =ICM ro Motion of end B is combined rotation and translation.
_, _, _,
Larbital = rx M v CM
_, ...,
Lorbital for rod is zero because. r is zero.
Method 3. From conservation of angular momentum
about a point on ground just bel,ow end A (at which the Resultant velocity of end ... (4)
particle strikes), we get

www.puucho.com
Anurag Mishra Mechanics 1 with www.puucho.com

!496 ~ - - - -~ECHANIC~
From conservation of angular momentum about a point
on the ground just below centre of mass of the rod, we get
2
mv 0 l=m(v,+lwJl + mZ m ... (3)
VcM = Vz
4 4 4 __g___,
Angular momentum Angular momentum
of particle of rod

On solving eqns. (1) ' - - ' -- -- -- -------- ·1


----> ----> 12v 0 4u 0/11 ,
V,01-~~-v1rans
and (2), we get OJ= - - r. 7vo '
B 111 -v 1 +-ro-
4 11 j
Fig. 5.120 4v 0
V1 = - -
. 11
.,-.LLl.--'"-j-->,--f---, V2
From eqns. (3) and (4), we get Vo
ml 001 2 V2=2 Fig. 5.123
Vz = 2 = 12(1/2- x)
-+ v0 -: 4v 0 ~
l Vrod =21+UJ
Or x=-
3 -) v 0 -:
0 7v -:
2lv 0 2lv 0 2lv 0 VPanicle = -1+--J
For this position, m---·v 2 11
2 =--·v 1 = - -
- 381 ' 76 ' 76 I
Concept: Note that in all the three pre.yiousl
Illustration: A uniform bar of mass m and length 1 is :illustrations, we have applied conservation of angular!
translating on a smooth horizontal plane with velocity v 0
:momentum about a point on the ground below CM of
when a particle of same mass m travelling with same
irod. We can always conserve angular momentum about CM;
velocity v O as shown strikes normally and becomes
iofsystem because in a collision, the velocit;y of the CMofthel
embedded in the bar; then determine the angular velocity of
lsystem is unchanged. Differentiate between CM of Rod and 1
the system after collision, velocity of centre of mass of the
jCM of system. If we write angular momentum about a moving.j
rod and velocity of the particle.
:point, we must take v relative, but only the-moving point about.
iwhich the angular momentum is conserved is CM of system. I
1-- - -- ---- ····t

i
i
I 3U4 U4 ''
I
I
c_Jj
y

_L;_V2i
i
[;~~~ ::~e have cons~dered afixed p~int o~ -t~e gr:~~d i~\
, 'I

ITJg~:~~e~•'irJ 6a (>
I• U4.t• !/4 0 I
¼cue stick hits a cue ball horb:ont~lly ~ distan~; x ;;;~~e th~!
!I icentre of the ball. Find the value of x for which the cue ball,
Fig. 5.121 __ ; l .. Fig. 5.122
-' lwill instantaneously roll without slipping. Calculate the:
C1 =centre of mass of rod, !answer in terms of the radius R of the ball.
! . - " . .. .... '
I
-

======--oF :
~ i
v1 =y-component of velocity of CM of the rod after
collision,
v 2 =x-component of velocity of CM of the rod + i
particle, ! ~.r. . ; ...
v1 + l OJ =y-component of velocity of the particle after
4 Fig. ,SE,68
collision.
Solution: Problem Solving Strategy: What would
From conservation of momentum, we get
happen if the stick hits at the level of the ball's centre ? Will
Fory-component mv 0 =mv 1 +m(v +¾OJ J ... (1)
1 the ball rotate ? The answer is that the ball initially
translates with no rotation. If the stick hits below the centre,
Note that initially only the ball has momentum in the ball initially has reverse spin. At a certain value of h, the
y-direction. ball has just the right forward spin and forward acceleration
For x-component, mv 0 = mv 1 + 2 ... (2) mv to satisfy the rolling constraint, non-slip condition. The
Note that initially only the rod has x-component of torque (and hence its angular acceleration) exerted on the
momentum. ball depend on x. The linear acceleration a is F/m

www.puucho.com
Anurag Mishra Mechanics 1 with www.puucho.com

[ ~IGID BODY MOT!ON '


independent of x. For the ball to roll without slipping from P = mv-(-mu) = m(v +u)
the start, set a =Ra (non-slip condition) to find x. The · The line of action of impulse ·p passes at a distance·
weight and normal force act through the centre of mass and (h - r) from the centre of the ball.
thus exert no torque about it. The frictional force is much From angular impulse-angular momentum equation,
smaller than the collision force of the stick and can be
neglected. J~ M=JI dro·
The torque about the centre of the ball,
~=Fx
~(h-.r) = 1(~)-(-I~)
Applying Newton's second law for rotation and P(h- r) = I(u +v)
translation, we have r
..,
LF=ma
.., ..,
and · L ~.=-Ia
.., m(u+v~(h- r) = r(u ;v)
F=ma and ~=Fx=Ia ... (1) I 2r 2
Hence r(h-r)=-=-
a=Ra ..• (2) . · m 5
Expressing a· and a iri. terms of the force F from step 2, 2 2r2
hr-r = -
.F =·RFx 5
m I h =7r
.. I 5
Solving for X, . X ·= -·-
mR l,!Ex.,,.mr..ale-}
·== ~-,~-,r:::.~·-·~
7·0 !">
~
· For a sphere, '[ ~ 3,mR2
5 )An· oxygen ato~ moving with -;;;;d--~~-~;Uid;_;·-wi;h a;
X= (4"S)mR2 =3_~ stationary diatomic molefule.as shown in Fig. SE. 70. Describe:
mR 5 the finaf mi}tion of the system and find the rotational'kinetic,
Analytical skill problem energy of the .molecule, assuming that the total kinetic energy'.
is cO~~rved/ 0 1
~ '":___ , _ _ _•_ _ _ _ _ _ _. • • • • \
Mark the correct choices:
(a) If the ball is struck at a height greater than 2·R/5 'it
shows top spin.
(b) If the ball is struck"at a height less than 2R/5.it shows
back spin. · · · ·
(c) Frictional force is much smallerthan the impulse·of the
bal~ therefore it is neglected. · atom
X
( d) A ring will show pure rolling only if impulse acts ~ l l ' J ._ _
tangentially at the topmost point. · · · · Vo

~~~~~· Before
(a)
after
(b)
..•''
I
Fig.' 5E.70 . I
IAb~ll of.radius r hits a cushion with a pure rollirifrizoti6~ - - - - - ~ ~ - ~ ~ -~--~~.,._J
jand rebounds., "th a. pure rolling m:.otio.ri. Find tile ra. ti.~.. o·~·.
WI.

the height h of the cushion to the radius r of the; balL_ Assume


_Solution: Problem Solving Strategy: We model·
the molecule as an ideal dumb-bell: two particles (the
jthat the force exerted on the ball by t~e cushion is horizontal oxygen atoms) joined by a massless rod. A convenient choice
ldiLT'iMJhe impact and tJLqt the ball hits the cushion norm_a/ly, of origin is the centre of the molecule. The. impulsive force
Solution: Let the mass of the ball be m .and itis initially . exerted by the incident oxygen atom exerts a torque about
rolling to the right with speed u arid rebounds to the left at the centre of the molecule, causing the molecule to rotate.
speed v. If the impulse P exerted on the ball by the cushion is . The f'inal state of the system is .shown in Fig. SE.7.0 (b}..
P, from ·impulse-momentum equation, We apply' problem solving strategy using the coordinate
.. system sh()WJl in the figure. After the collisicin, the molecule .'
has· intrinsic energy due to .its ro_tation as well as kinetic
. _energy associated with its translation motion. Each of its ·
'. 'fh ·atoms has speed v = ror = rol/2 with respect to the centre of
the molecule.
_.. , 1 I · -· Here ~e summ,irise the 'kinetic energy, lineai-
---- Fig. SE.69.~_.d inonientum, angular momentum before and after collision.
in a tabular form: ·
www.puucho.com
Anurag Mishra Mechanics 1 with www.puucho.com

I498 --------~---'--MECHA~KS•!]
' '1
Find the mol.ecul_e's centre of mass velocity and the. velocity iJft
Atom Molecule : ,each atom irUhe CM frame: What is the molecule's intrinsic!
1energy (kinetic energy measurea in l_ab reference frame)?i
Kinetic energy -1 mv20 1 2 {
- mu1 + 1 ,What is the molecule's intrinsic angular momentum (its:

~ 2Inu~ + 2[! m(!<ll)


2
2 Iangular momentum about its CM, measured in the CM'
] :
2 0 2 .2 I 'frame)? --------··- ------~------:
I Solution: We
, x-Component of lin· model the molecule as
ear momentum a dumb-bell-two 2vof3
z-Component of angu- identical particles m
lar momentum connected by a
massless, rigid rod of
From conservation of energy we have length 1. · By · lab CM
vof3 i
mv~ = muf + 2mu~ + m1 ro
2 2
/2 .•.. (1) reference frame we '.,)
mean a coordinate m
From conservation of linear momentum, we have . '
system attached with
mv 0 = mu 1 + 2mu 2 .·.. (2) '
'the observer. It may be
From conservation of angular momentum, we have fixed or moving.
J. mlv_ 0 = mlu 2 + mro1 2 · ... (3) Since the lab
Fig. 5E.71 (b)

On solving eqns. (1), (2) and (3); we_get velocities of the two particles have only x-components at the
rol instant, the CM velocity is in the x-direction. Velocity of CM
U2 =- . ,.. (4) at this instant is .
2

u, = 0 and 0) -~ Vo
l
-,;CM=~~ m(-v
--+·

M 2m
"
0 /3)i+mv 0 i=(v 0
.
;;

3
)i
·. Vo
. U2=-. The particle velocities in the CM frame are found by
2 ...,
subtracting v CM from their lab velocities.
Finally, we use eqn. (4) 'to fine! that the rotational kinetic
energy of the ·molecule is · . -> -> -> v 0 ) : 2v 0 •
vlCM =v,-VcM =( Vo-- 1=--i
½cm{~r] m:f.
· 3 3
Kro, = 2 [ = -> -> ..., 0( ·v v 0 ): 2v 0 •
V2CM a=V2-VCM =_ -3--3 1=-3i

The intrinsic kinetic energy is ·


2
1 2
Kim =- L.mivi CM
j -- - - ,/" - • .- - • .s- -·---: -·- - - '
----~·-7 2 i= 1
<A diatomic molecule, two identical atoms of mass m bound''
:together a distance .l apart,. is confined in a lab reference:
iframe. At a .certain tinie, one of the two atoms is observed toi
-;have speed v 0 and the other, moving irrthe.opposite direction;'
The intrinsic angular momentum is the sum of the two
ls;eedv 0 /3[Fig. 5Ji.;?JCfz)}--------- _ !-
particles' angular momenta in the CM frame about the CM
l .
as origin.
Vo! We_ obtain the direction using the right-hand rule:
m !
,.' I
--+ --+ --+
= rlCM X (mV1cM) + r2CMX (m V 2CM)
--+ --+

'i,.,
!- "-.
rr CM
, I

!
LcM
..., ...,
= lr1CMl!mv 1CMl(sin90°)(-k) ·

il
,
•I
'vof3
m
X
..., -;
+ Ir2cMII mv 2CMI (sin 90° )(-k)
="2(½)m (2~o )c1)(-k)= -2~volk.
I . Fig. 5E.71 (a)
.:-.-=-~ _,. ~ ... --- -..,'°....,,,,__,,,,_ I _ .-·--··· · · - - · · ·

www.puucho.com
'"
Anurag Mishra Mechanics 1 with www.puucho.com
. r,

L.,l:Rl_'G~ID~BO~_D=it',M=J)J''"'fl,O""-,N'-;_··:---'-_,;_...""':'""''""-i.i!:""'lf;~•·.----'-~:"'".~'t...
c."""
'¥' _ _ _ .....;c.·=·- - - ~ ~ - - - - - - - ' :~!)!]'
i,fr-¥e~-~1?27~
- _ - ~
_ odnlybexbellrts a force·bdirec~y aft~r ,the collision,fwhen tAnhe
um · e rotates a out its staUoiiary. centre o : mass, _
. !11v
1
0 i_de'lti¢fi_-.I !luinb_~bells move \hJ!,v_}i_rils ia_di_,-_ .othe,; Q..~·;a· interesting point in the probleni is that the dumb-bells
, , - · · · ·- collide again after half a tum of each i.e., after a time
5
· [h.qriz.qntal a/r·--.'_!IS_._ ·~-ioned ta-ble,_ ~·~-~o···w·_,.n.-in Fig.. ~·72__ 1_.·..
Each can be considered as two point masses m Jomed /zy/ a;
c~_ t "'it/oo. tising the previous re;tilts, the resulting·motion·
· · ,· · can be predicted without writing equations. The rotation of
1111eightless. rod _of; length .21. Initial/JNhese are nqt mtii?~' the dumb-b_ells stops and they move again with the same
·Describe "the motion. of th e dumli-'1/,ells· after their el~/!i~ speeds as before the first collision. Their path is the same
· collision. 'Plo(th11 speeds of the . i:ilnttgs of mass 'of_ t/ie
dum:b-bellsas a ju_-nction oi time.· ·· straight line but they are now traveling 'upside down': In
7, other words, the dumb-bells spend the time between the two
m
collisions turning round. The speed o(the dumb-bells as a
functic:m of time is shown in Fig, SE.72 (c). ··

.~~1_'.'~ m 2
6;~-~~~1 731;;>
6 m_
·Ii uniform rod of length 1 and -~-as-·s-2-m-·-~-es_ts_o_n_a-smcio~,;J
. horizontal_ tcible. Appint mass m moving horizontally at rightj
Fig. SE.72 (a) angles to the ,rod with an initial velocity v collides with one,
Solution: Conservation of momentum implies that the
· end ofthe,rod. a11cl sticks to it.
Deter:mine: · · . ,
_ · ·
-
·' I
:
centres of mass of the two dumb-bells always move at ;<aJ the anmf1!"" vefocity of t~e syste(!l after coll~ion, . · '., j
identical speeds and in: opposite directio11s. (b) the position of the_ pomt ·on the rod whzch, rem:m1!5
When the · · - I statit),:uiry ~rnm~dia'.ely after col/isi01:z; . .. . _ ;
1

dumb:bells collide, '


v¥,9. .·.' ·. '···' ' 6-v, .w( 9_'.
1 · (c) the· chqnge:;m kmetzc energy of.the system as ,a result-of,
I the col&,ion.
::;is::~:: --:.6. .ei.ti~:~1-._·:-~:--;·:g'
,_}_
both their energy , ·· '

'.I Solution: After collision the system will rotate about


its centre of mass. From the definition of centre of mass,
collision is perfectly
elastic and no'
: -d ;)
. '. 5;' XCM = mx0+2nix(l/2) =!_
, 3m · 3
external· torque acts ~---'-"f'.e=-'_-s_E..;.7_2.o(b...:l_ _ _w Thus axis of rotation is at a distance of (!/2' - 1/3) = 1/6
on them. The states froni the centre' of, the rod.
before and after. the collision a,e shown ih Fig, SE.72 (b), I
Before the collision the dumb-bells· have only
translatim,uil kinetic energy, while a rotati9nal term appears 2m
· after· the colljsion. When writing down ' the ,angular
momentum equations for the dumb-bells; we calculate the
latter with respect.to their point of contact, P:
2(.!2
2m~ = 2 J 1
(.!
2mV 2 + -_ 2ml 2oo 2·) ... (1)
2 ) · 2, ,, 2
· 4lmv = 4lmV + 4mi oo 2
..• (2) . , {II) '.
Xo (b)

----'-"Flg:,_5§,!!._- - - - - - ~ ·
!
The non-trivial .solution From parallel axis theorem, moment of inertia of the rod
(V ;< v,ro ... O) of the above
_about centre of niass 'of composite system, ·
set of equation is found to be
V ~ O,w = v/l; · i.e., the ' ;rod~(2~~z2 +2m(ir=-2m_91_,2,,
centres .of mass of .the r·!····}roc.o
• : : ""J' ' . . ···2 '
dumb-bells stop moving after
the collision ' and that the ·~'.~;~~--···~; ,} L...,,.:- I point mass '7 m (½)
. colliding point · masses , Fig. SE.72 (c) ' , . 2m! 2 ', m! 2,, ml 2
change velocities while the I,ota1 =-9-+~.=~
non-colliding ones keep their original velocities. This can be
· interpreted in the following way: point masses joined by a (a) No external forces act during collision; so the linear
rigid but weightless rod.· are not aware of each other's · and angular momenwm, of system is conserved, '
presence in the course· of a momentary collision. The· 'rod ' '
mV= 3mV' ·
www.puucho.com
Anurag Mishra Mechanics 1 with www.puucho.com

,mVl ml 2ro 2Mv'= 3Mv 0 · ••• (i)·


- - = - - so ro =V 3 '
,3 ,3 1 Therefore v' =
(b) ,Consider a point at. a. distance y to th~ left of the
2v 0 • Velocity
-+l
of centre _of .m';l~s,....,

centre of mass. Its velocity is sum of velocities of translation


_, .
Im,v;:
. 2
~(2M)v
0 3 •. ,,. ,,
and rotation, 'i.e., .. u - roy. So the position ofthe point on the VCM = I • =~--- 2"oi ... (2)
rod, that remains stationary has zero' velocity. ' Im,
. i'
2M
. u V/3 l .
u-roy = 0 or y = ro = V/l = 3 ,: (b) The position of the centre of mass [See Fig. SE.74
(b)] is :
This p~int is ata distance 21/3from the ~nd to which the y
mass .sticks. ·
·1 2
(c) The inidal kinetic energy,. KE· =-mV
' 2
and the final kinetic energy;
. _ 1 , : 2 1· 2
KE 1 = (3m)(V'), + Iro
2 2
J 2 ·1 2 1 2
=-mV +.-mV =-mV M
6 6 3 I R a,: .. -
'. 1 y2 --m
1 y2 ·=-m
1 y2 ll. . . , FJ9;'§!3,74 (I>) , ,,
Losso fki netlcenergy=-m <?-~- ---~.--t:---~t!!l,Ll<:;..;_;__"..., _ _ ,
. 2. .3 6
02"'"' •:L'•;,£'.:.':~ . _, , 0'.:-1 +M(½+R)(-J)
~~~~~ RCM = --~---'-.- -Rj _.,.(3)
'2M,
where ,we use the fact that.R = !:_.
' 2
The moment of inertia about an axis perpendiculaI' to
the rod· and the disk; which passes through the· centre of
mass, I cM , is given liy . ·
Icu=[~MR
_,,.. _ 2
2
+MR 2 ]+~ML
3,
2
= 17
6
MR· 2 ... (4)

~Con~~pt::The angular mcilheri~ aboutpo}nt,•O~~'.th~i


laborator:yJtame[See Fig. 2E,74Xl!}1 ii conservedan.lis,zerd
-+ ·,. ''\ . . . . • ,,, ,-· :·.,- \- ;_:~ < ,' . ,( :::~·;;_"j<:{;~
(J = O) . a'h§r:efore, - r. .. · , · · , . · · :,:·;.. _"""'
• ·~ , . •; -:<; < ,- -1-+ ;_I··,·, -4 "-+, :~/;:,>;•<:' ' °1
-Jo·= 0 = ICM ro+ 21\_'L!!_l;Mx v CM , : . J::;,CSJ •j
The first . term is an. expression for the angular
momentum in the centre of ·mass frame, and the· second
term is the momentum of the cehtre of mass about 0.
Therefore, ·
-+ 2M~- -+
ro = ---. RCMx.vCM
IcM · · ·
'2MR · 3 ·. S·'O''
=- , . Vo(-JXl)
17
-MR.2. 2,
6 ' .
'= _·18 Vo k. ..,(6) .
· 17 R
We can write the kinetic, ~nergy before the re.lease as
' ' 3 2, ' ' ·'
K1 =. Mvo , .. (7)
2
and the 'kinetic energy after the release, ..
2
2M (3 ... )· ·,i 2 ' '
K 2 = - -v 0 +_-ICMro
2 · 2 ·. 2 ,,

www.puucho.com
Anurag Mishra Mechanics 1 with www.puucho.com

, . K -+
. .
SubstJtutmg ro and I CM, we outam 2 = 207 MVo2 Solution: (i), (ii). We know thatv =v 0 j. According to
A ' '

68
the definition of centre of mass, we have
Hence,
3
K=K 2 -K1 = ~v~ (::-1) _,
Lmivi
.
1 •
_,

35 3 2 35 vCM =-'-'--=~voJ ... (1)


=--Mv 0 =-K1 ... (8) I,m; 2
342 34
__ ~deed, tile_ kine_tic energy increa~es.
Concept: This process can be seen as the opposite of,' =-~'=- _m_;-;_; = m(½L)i+m(-½1)J
;perfectly inelastic collision, where two masses stick together'
!Therefore, the kinetic energy is not conserved. In usual plastic;
and
I,m; 2m
!collisions, the kinetic energy decreases but the kinetic energy! =2_L(i-]) ... (2)
1in this case must increase. This conclusion is quite apparent· 4
;when one considers the intemalforce ca~edby an "explosion. ; where we choose the origin to be the point of collision
r- xa .. ,-. .
ia-F-" ... P!M~2,'=~- J 75
,e. · 1~ - ""'-
j___.-
[see Fig. SE. 75 (a)].
14----L---
l - . : "'- .. . . ' U4
1
'
A thin rod of length L and mass m moves on a smo.oth •
_, U4 f1'\
i
jhorizontal surface at a velocity v O in the direction .of its,
length. The end of the rod hits the end of a second rod of the'
-·,·7 )"'
L
'.same length and mass, which is initiallyat rest, perpendicular .'-"} .1AA,

Ito the first rod (see Fig. SE. 75). At the moment they touch, the / RcM (t =0) = 4 (i-j)
1two poles stick together, while the angle between the_m
I
l . 7t
(emams .
2 Fig. 5E.75 (a)
! (i). Velocity of CM of system is ;
(iii). In the centre of mass frame, the angular
I (a) ':'.9..j (b) voj momentum about the centre of mass before the collision is:
. 2 -+ -+-+ -+-+
i (ii). CM is located at ; JcM 8 =(r1 -RcMl x m1 (v 1 -v cMl
I 1 A A
1 • - -+ -+ -+ -+
(a) -L(i- j) (b) -L(i + j) +(r1 - RcM) x m2 (v 2-v cMl
3 2
(c) lL(i ~ j) CdJ l1cl + Ji L
=--mv 0 k

.... (3)
4 4 4
i (iii).,Angular velocity of system after collision aoout an axis: -, 1· ->1,
Where we substitute r 1 = - -Lj and r 2 = -Li. We
through CM is .: ' 2 2
(a) -~':'.9..fc (b) -~ Vo k calculate I c using the parallel axis theorem:
2 L 4 L

1
(c) -~-Vo k (d) ...:~Vo k Ic=2(½mL )-2m[(%r +(%r] 2

5 L 3 L
l(iv). The maximum velocity attained by the pole's end/ ... (4)
! denoted l2. A, after the c.ollision is : _,
I! (a) iv 0
2
2+2]
-/s
. (b) lv 0
2
[3+2]
-/s Hence,
.
COcM = ~ =
J
le
lI. 1 [ -3 ·] - L"J
(c) 2v 0 1+ .J'fo
h 1 '[ 3]
2v 0 1+ .Js. 1 .,.(SJ ,
.: (v). Maximum velocity is attained for the first time after'
time: (iv). The maximum velocity is reached when the
5 L -1 S
(a) ~l:.tan-
2v 0
1
2 (b) --tan
2v 0 •
. _, .
velocity of the po\nt A in the centre of mass frame is in the
direction ofvcM,i.e,,J [see Fig. SE.75 (b)].
(c) -~l:.tan-1 2 (d) ~l:.ra11·1 3
______ ,3v 0 __ .... _ ·- ... 3_v 0
www.puucho.com
Anurag Mishra Mechanics 1 with www.puucho.com

'' ... (1)

¼ rigid, is,
zerp\nass ;od of length:$1 f,la~d on ,a,srnoo£ii -
ro 0t = -1t and' t = -1t- = -1tL .,.(2)
~orizontalpla.tie, and is free to mirv~'.ofi it. At t < O, dte'r.o<i is 2ro'o 2v
at rest on thery0 axis (-L toL).''lwo,small bullets ofidenticdl ',
· 2
.
. (iii) .The new position of the centre of mass, x (the point
mas'sm hit th~rod at t = 0. Thefir,st,one.hits th~fower~ml':A
marked by @' on the x-axj~ in Fig'. SE. 76) . .
at a ielocity of•vx, and the second bullet hits the roil, q~1ts
.upper end '. B,~,vith a velocity of-vi;.as. depicted .in Fii,5E,16. · · i='t;m;X; m(2L'-L) =.!.L ... (3)
I - - - ---:-_- ' - .· •. ,._ . ._ - ·-:'' ,,
. L;m 1 3m 3 .
!IThe iwo_ bif,ll!~s~tick to the rodsimllltan~o'!:ly, at t:=- g;fdni! ". '' ·- - . J '
imove ,vtth ./lit, · · :-:.'=- . , .. · - .. , . ·. · · The angular momentum about x is conserved. ·.Before
' -y ' .. , tlie _collision the_angular momentum is'
J B J 1 ;=. mrooL,(.(. + x) + mro 0L(L-' x)
4--------e V, m . . = ·2mrooL2 - ... (4)'
L while after the collision,
.J2 = [;2ro•(23L'.) 2
+ro ('4 L:), m= ~mroL
.
3
2
24
] 2' ' ... (5)

'- m,v~ · Using ·the. ·principle of. conservation of angular


1' T' '! momentum (applicable becAuse there are no external ·.
torques), we obtain
\?5 1'. ,
' L;:,___ _,__,__ _ _ _ _Fig.
_ SE_..76
_....;_ _ _ _,_.;._:,;.;.;_.cf.
3. 3v
ro=-ro 0 =-- ...(6)
. '4 - 4L
" ; \'

-, ' www.puucho.com
Anurag Mishra Mechanics 1 with www.puucho.com

RIGIDBOQ)'.1,\()Tl,-ON~,~--~----'--'~_,___------'"-----"-----.,.-.-·-:'..,;'=g0_,_,3]
(iv) The'centre of mass is .at rest all the time, because of , df F dt
the conservation of linear momentum; therefore, the linear From eqn. (2), Ol=---
velocity of the system is zero. · ' lcM
Illustration: Centre of percussion: Consider a dfF dt
. • J ~ It k is radius of gyration of pody,_ ·OJ=~~-·
stationary cricket bat being hit by an impulse J as shown in · mk 2
Fig. 5.124. We may locate the 1------- -·-:-o~-·,-:;:i
Frpm impulse-momentum, equation, instantaneous centre of rotation
_,
J=LlP
.... (Jc)· of this body. Instantaneous I . ' .....,...
centre is, that point about which : "' ' "b

Magnitude of angular impulse = YI JI.


... (1) '•
....
a body in plain motion
(continued , · rotation and
translation), may be ~ssumed to
·+
I · CM~
. d
From angular impulse-angular be in pure rotation. Consider a
· .. i~.
·h i
momentum equation, point distant ,b from centre of l
.... _, I . Fig. 5.126
YJ = lcM Ol .•.(2) ' CM I mass on a line. perpendicular to. •-----..-. ,.-----'
1ii line of action of F.
Suppose . the batsman holds the .
bat at a height h from the centre of
"' Y,
-t -t -t'
Vo ,;VqcM+VcM
mass. This point will have two _, '
velocities: lvqCMI= -bro
(1) v CM, due to translation . . ---, . ---, · 'fFdt bdfFdt ·
Fig, 5,1,24,
motion, '--_--C.--'--'----'
lvol=lvCMJ-bro= - . -
(2) tangential velocity, v,, due to rotation of bat. m ·mk2
I • • •

Two :velocities acquired are in opposite directions. If , If velocity otpoint O is to be zero,


v CM = v,, for the point h, it acquires no net velocity due to · =_fF_d_t _ ~bd~f_f d_t• or ~ = k2 ..
impulse, i.e., no sting on hands frolll eqns. (1) and· (2), 0 7
. m mk 2 · , _'d
.!_ = /z yJ O; y ,;, l CM . When force F Is applied ' ....,.. ·-- - · ·-· ,---~·-
M ICM Mh. .- . k2,
at a distance Ii ·from, centre· p ....
of mass, the point O is, at ,d
rest, defined as centre of ( , h
i
R CM f
.
percuss101!:· ' '

1
Suppose a solid cylinder J .....__ _..:::,,__,.....,:::__......_
of .mass M and' radius R \
(radius. of gyration I = R/./2 1 Fig. 5.127
is struck by a force F. If we • - - ... -- -·--'----'
w:ish to have no sliding at point of contact 0, then the force F
must be applied at centre. of percussion, whose location is
·given by ·
\ ' k2
Fig. s,125 ;: h=-+RR .
-~-----~----~
This pohlt whose position in given byy is called Centre Illustration: Consider a
-of percussion. · _ solid sphere of mass· M and
, radius R, rolling with a velocity
For a body at rest on a smooth horizontal plane, if it is ,- ·
struck by a force Fata distance, from the centre of mass:
d_ Vo along a horizontal, plane . .It
suddenly encounters ·ah·
From impulse-momentum equation, obstacle. . rn' · such sudden
.f
Fdt = m(v~ - 0) ... (1) inelastic· collisions, large
From angular impulse-angular momentum equation, impulsive forces are generated
. J Fd dt = I CM (Ol ·-: 0) , .. ,(2).
at the contact point. These
forces suddenly provide large
. Fdt . f accelerations, therefore
Fromeqn. (1), VcM = - . -
I m
www.puucho.com
Anurag Mishra Mechanics
'. 1 with www.puucho.com
r,·:
,-~. I

·orpinary force.like weight are"rieglected which allo~s us to v 0 as. _shown in Fig. 5.130. _ ~-J~stantaneogs ''A/ ,c "'"'"
\1se Jaw of conservation of angulaqnomentum. , .. Agnular momentum of the rod [~. ,:•~tre vi:i
' • . .
_Initial angular momentum,
. . l· ..
about point A before collision, !':::·_ 0)~_a:,::,..,,, •
· L; ~~=MR 2
(v 0 )+Mv~Rt9se _ILJ= Mv 0lcosa ... (l) ;l:llf;l'sina;

::c:~~:~:;~::~~j ·t~: ;';~:~; ;·• ~:


5 . R , • ·
' '
· .. Angular momentum iinmepiately afte_r collision, imp!~ ;;,,,/\7.:;
·._,. -;f =~MR (;J+MyR ... 2
· direction and end A moves ~ Fig. ·. . '. 5 130
c'~}:tt
.·-, >From conservation ~f angular-mom~ntuin,
..., ..., .
h~rizontally. As· a. result ·the
centre of mass acquires a vertical velocity v and an angufar
. velocity co.: · · ·
2
-MR 2
L(;:L)t . ·.
__Q_ +Mv 0 R_cos0=-MR
. . ·,2 ·2(v)· --·:
-·_ +MvR·
.· _Angular momentum of the rod about point A after -
collisi~il,
.5 - R · -. · 5 . R
.-1-,L 1- M~!cosa. M12 . (2)
. v=v 0 (1t.~os_0) _ · t :- 2 , + I2 ~ . .. ·
: . -'-Note that immediately aftercoJiislon the conta~t pointB F.to1;11 faW of conservatiori of angular momentum,
. becomes .instantaneous centre of .rotation, if friction is -;t -t,.

L; := Lt
~- sufficient to ptl:r~vent ~lipp~g at B. :, _-.,..-:--;,,,--...--""-..,·.,,,.,,.,.,,, .
· ·- ruustra on::.. : A plane·•: " Mv 0 lcosa _,. Mv.lcosa + M1 2ro ·
... (3)
·lanilila of any arbitrary shape has. ., 2 ·2 · · 12
-:niciment of inertia le about centre + _Note· that we have ignored ·gravitational force
of gravity c, velocity _conipo_nents .. (why?)'·:· .
'i · 'vx _anci v;iapgular velocity' co as· ·+ We can determine- the position of instantaneous · ·
• .shown in;Fig. 5.129: -S~dden,Jy a
. ' • .,.;,C-;. -rotat:icin by drawing normals to velocities at poiiits C
· point O· distant 't from centre of and A. The p_oint of intersection of these normals is
· ., gravity C is
fixed; ~e wish to instantaneous centre of rotation: '
· · calculate ·, 'the:·_ new_ angular ·
velocity.·. Now v = colc;sa '· ... {4)
'· ' · ,· ~ .. .' __ ' ''
.·. Initial _angular momentum, L; -~' (Mv yr +_Ico), where I_is . On solving eqns. (3) and (4), we obtain
· · . ·6v 0 cosa .
. moment of inertia. abo_ut centre of gravity. When point_.o is · CO•= -~---~
fixed,large impulsive forces are generated but torque of all- · · l 1+3cos 2 a
these forces about O is zero, as momentum for these forces is · •, cos,2 CX
·· _zero. T1',erefo~e 'angular momentiHn·is·cons_erved. . - v = 3v 0 ·· . ;.
. ~ ' . · _ .
.·' ;
. . l.+3cos.a .
' Lt =.(J'+,·M:r 2 )co'
·Torque on the' conical pendulum: Here we show'
' ' -t, -t, ' . ; .· .
. -t, -to··
L; =Lt " that the reladon -r 7 d L/ dt is satisfied in tase ·of a conical· ·
· pe_ndulum, . · · · ·
(Mvyr~ico) =·(I +Mt 2 )ro;
.: '(Mv··-Y r +_ · · · I
·_ro'=.·
:·· ·. I+Mr 2 - - ••
I~) ,-_. -
Fig. 5.131 shows. the forces
acting on ·the , bcib. For uniform
circular motion' there is no v,ertical
acceleration · and .consequ,ently · · · •
· Sµppose a .ilisc rotat~s ·on 1t tabl/~b~ut its centre of · vertical component of tension T ·
gravity, if a point.on'its circumfere'nte is suddeµly fixed. The balances the weight of the· b9b, Mg.
. ·,.~
.new angular velocity, . . , .
. ·Tcosa-Mg = O• r=,
~
Torque' about origin_ at A:
. co'~[~:~~ · 1- = The resultant force F_ on the bob is
· ·· · ·--co+MR . 2 radially inward (Fig. 5)32). The
•,•· • , 2 • . . I'• . ' , '
.
· .
torque on M _ab.out A is
.• •• .• i • ,· . •'. . • •. ' ,."
.~ •--+ --¾
,Here initial;velocity 9f c~nt,e is zero, · . . _, -rA=rAxF ··
.. :mu~trat'fo~:,A:thin ro4 6f htass M.~d i¢ngt11Jis
dropp"~d:pn·,ilJii!~<;iµi:ho;faonfaf plane: The rod"falls without
anf i4)gjilar,~el_oi;ity, ;wlie,r,easvelocifyof its centre of mas_s is
www.puucho.com
'-·,-\,i~;:;:.~l1l:\:it··::· , ,' " . --
Anurag Mishra Mechanics 1 with www.puucho.com

..., ..., ..., .


Since rA and F are both in the r radial directlon. swings through angle A9 = coM, and L, rotates through the
...,
dLA ·· . same angle. The .magnitude of the vector difference AL, is
Hence - - = 0 given approximately by
dt c,~(1_+_1,.~1>_ __
...,
Therefore LA
.
= constant
...,
Torque about origin at B: The torque :rs is
--+'" --+ --+
•••••••.••• I
: :/'-;,~:v . . --·-
•. ' ~4(1)

·~8- .·!· (.
,'.ts=rsxF:
..., .
_.:'-i",'·, •. ;y ;, ~
--~~·,14 .- ';-
Hence · l:r8 1= l~osa F ·
= l cosaTsina = Mgl sin a .
'
•. ::-;r-1· "
.~,
.-: v- L~t)

[Where Tcosa = Mg]. ·-~-~·-~--F~ig~._5.~1_34_,.~·- - - - - - - - - '


The dire:ction of '.t B is tangential
...,
z IAL,I= L,A9.
to the line of motion of M; in
accordance with right hand thumb In the limit At -) 0, we have
rule.
...,
. dL, ---, d9 ---,
:r 8 =Mglsinail - - = Lr - = L, co,
dt dt .
· Where il is· the unit tangential
Since L, = Mlrco cosa, we have
vector in the plane of motion. Our ...,
aim is to show that the relation dL,
..., - - = Mlrco 2 cos a
---, dLs dt
'.tB=-- ...,
dt dL,/·dt lies in the tangential direction, parallel to :r8 ,

is satisfied. Angular momentum of as expected.


..., ...,
conical pendulum aboutB, Ls Method 2: Write Ls in vector form and then
has constant magnitude Mlrrn. differentiate:
..., ...,
In Fig. 5.133, Ls has a Ls = (Mlrco sin a) k + (M!rco cosa} r
vertical compone!}t
dL dfo
Lz = Mlrcosina · an~ a -'--1L=Mlrcocos9-·
·horizontal radial component dt dt
2
L, = Mlrco cosa. We can = M!rco cdsail,
express total angular where we have used dr/ dt = roil.
.momentum about as
--+ --+ --+ --+ ,, Concep;;.',Note that angula_r ,:;c-t-or-,-ca-n-ro-t-at_e_i_n ~p~;el
LB = L.+ Lr, we s~e that L. ' -~thout changf~g magnitude. .. , . . . .·
is constant, because :r 8 has no 1
..., illustration: Angular
vertical component. L, is not momentum of a rotating·•
constant; it change direction · skew rod: Consider a rigid
as the bob swings around. However, the magnitude of-L, i~ body consisting of two
constant. . particles of mass m joined by
. Concept: '. U,.e only way l2 '~ector A of constant · a massless ·rod of length 21.
magnitlide can i:hiznge•in time ,is to',fotate, and thdt if(ts, The midpoint of the rod is
instantaneous ·. rate . ·of· rotatioii' is. d9/dt, 'tMi:i attached to. a vertical axis I.
·which rotates at angular
! speed co. The rod is skewed at
•. jdA/dtl= A d9fdt,~~~;,_,,:.~. can o.btiz·i·.n .·. ';.:~···. , angle a, as shown in the
• • __ r ~Ji;-CO' . ~-' figure. We wish to find the
' 1
"''. . dt. .. - -
., '•:~; r.
:-t-
, .;;;.,
.->
angular momentum of the
system. Fig. 5.135
Geometrical approach: The vector diagrams.show
L,, at instants t and au+ M. During the interval M, the bob·
www.puucho.com
Anurag Mishra Mechanics 1 with www.puucho.com

1 sos .MEC!fANICS-1 _ j
Method 1: Each mass moves in a circle of radius I cosa· ... ..,
L points along the direction of ro .L. Hence L swings
with angular speed ro. The momenttim of each mass is
_, -
...
around with the rod; the tip of L traces out a circle about the
IpJ = mrol cosa , tangential to the circular path. Taking the ~ ' ..
_, z-axis. Note that L is not parallel to ro . This is generally true
midpoint of the skew rod as origin, IrJ = I; vector r lies along
- _, for non-symmetric bodies.
the rod and_ is perpendicular to vector p. Hence Torque on the Rotating Skew Rod
_, . -

ILI = 2mrol 2 cosa. In accordance witli the right hand chull)b The angular momentum of. a uniformly rotating skew
_, ' rod is constant in magnitude but rotates in space with the
rule, Lis perpendicular to the skew rod and lies in the plane rod· and continuously changes direction. ·
. - _,
of the rod and the z-axis, as_ shown in Fig. 5.136. L turns
... ..,
The torques on the rod is given .by ,: = d L/. dt . First we
with the rod, and its tip traces out a circle about the z-axis. resolve angular momentum in components ..The component
7 L, parallel to the z-axis, L cosa; is constant. Hence, there is
no torque in the z-direction. The horizontal, component of
L, Lh = L sin a, swing~ with the ·rod. If we ch9ose ;zy-axes so
.a ..., ..., chat Lh coincides with the x-axis_ at t = 0, then_ at time t, we
..., ...,(r 'Pla
..., +
have
-(r •p)•
Pa
. (Out of
Paper)
...
P• (into Paper)
Lx = Lh cosrot ·
. ·= L sina cosrot
..., ->
(r •p)• Ly = Lh sin rot .
= L sin a sin rot:

(a)._, -r~

..(a),, Fig. 5.138


(b)

Fig. 5.13~,(b)
L_;_,;.-'-~-----------------·· Hence, · L = L sina(lc<is{Ot~ jsiµc,;tJ :t Leos.:&.·-,
Method 2: .This method for calculating L emphasizes ch~ dL
vector nature of ro. First we· resolve The torque is ' t = -
·~ . ·- . r------_;__~- dt
1 ' ' "it
w = rokinto componentsro.L andro 11 , = Lro sina(--<0 lsinoit + j cosrot}
'
perpendicular. and parallel to the ........... . Using L =-2ml 2ro cos a, we obtain
skew· rod. From Fig. 5.137, we see ( ......... ......::···~ 't = -2m1 2ro 2 sinacosa~inrot
chat ro .L = ro cos a and ro 11 =rosin a. 7
.Since · the masses are point
_.. -- ..... 2
'ty = 2ml2ro sinacosacosrot.

't ~ 1,: +-r = 2m1 ro ~inacosa


particles, -ro 11 produces. no angular Hence, 2 2 2 2
momentum. Hence, the angular '\J
X y - .

momentum is due entirely to ro .L ; .. ~ - - - - ·- ~. >


= roLsinix.
the momentum of inertia about the ······r···· Note chat r _= 0 for a = 0 or a = rc/2 · Tiie above
direction . of ro .Lis 2m1 2 · and the _analysis illustrates chat angular velocity and angular
magnitude
momentum is·
of the angular Fig. 5.137 ____, momentum are real vectors which can be resolved into
-componenti._ ·along any axes we choose.
L;. Iro .L = 2m1 2ro .L
= 2m1 2ro cosa.
www.puucho.com
Anurag Mishra Mechanics 1 with www.puucho.com

I RIGHI BOD'(MQJION . 507


Method 2: Torque on the
rotating skew rod (geometric
method): We resolve L into a
vertical component Lz = L cosa
and a horizontal ·component Lz
Lh = L sin a as shown in the sketch.
·since L. is _constant, there -is no Fig. 5E.77 (a)
torque about the z-axis. Lh has
constant magnitu.de but it· rotates Solution: We will check whether the cylinder slips
with-'the rod. Angular momentum down the incline·or not. · _ ,
.. is changing due to .change in . As mg sin 3Q =.µmg cos 30°, initially the cylinder slips
0

direction._ Fig. s:139


at 'its place till the angular velocity becomes zero.
" _ From the yector diagram,_ we have . mR 2' .
:E~ = (µmgcos0)R'= --a
IM.hl~iLhit.0 . 2 .
dLh =L d0_ 2µcos0
a=~--g
dt h dt' R
= Lho:i.
• ' 0=ro 0 -ut 1 ~/t
The torque is given by r-:---------~=--~-----i1 or

t1
OJ
= _Q__ )6(. _
dLh f •" : IA4.f""LhA8. a
~=-
dt. .,
Next the sphere will come mg sine mgcos8
· = Lhro down ·- with initial angular and
= roLsinu. linear velocity both zero. , ·,, Fig. 5E,7_7 {bl_
- The torque ~ is parallel · m.R 2 mRu
to Af. in the limit. For the· • JR=Ta., f=-2-
skew rod, ~ is· in the
L·· . , Fig. 5, 140
and mgsin9- f = ma
tangential _direction in the horizont_al plane arid rotates with
If pure rolling takes place,
the rod. ·.
a =Ra
. Conce~t: Torque on a rotdtjttgs,yste~ does not alwi;zys] f - mg sin9 = -2f
cause the sp'ied of rotation to change. Here the speed ofi
- rotation is constant and the torque, causes .the,direction .of L or, f =.!:.mgsin9=-mg ·
3 6
to change. Th.~iorque is produc,ed bythrforces. on the rgtating · · 1 ./3 mg
bearing of the ikew rod. .· , . · · -. • lemax =µmgcos0=-mgx-=-_
./3 2 2
I
and f max < f, so pure rolling is 'possible.
-. . 2f 2(-mg/6) g
a=-=----=.-.
m . m · 3'
Lett 2 b~ the time taken to reacll t),e foot of the incline.
F
3R. = _!:ft~ -
sin30° 23
I ·,. or, t2 -= 6.ffe.1i
Fig. 5.141 Hence total time, t =t 1 +t 2 = (rooR+ 6 /K)
_,. ' g fi
ki~~~i'~ 77 ~ -__ &~i•iii 78~.
IA spin~inir cyiinder 'of mass_ m_ ~nd-r-~a-_1I...
_R___is_l_o-_w_er_e_d~o-,-n-__-,a

~
ius,-.
1- uniform_ t~d ofmass m an~ ,length. 1 is suspended b.Y two
lrough inclinedplane of angle 31J"• with the honzontaL.and inextensible strings at A and B: If the string at B- suddenly
µ =1/./3. The: cylinder is releasec'f at:a height of 3R from
snaps, d~~erm_ ine the__tension 7;' i~ lbe string at 4 immediately!
/horizontal. find th_e to_tal time takiJ~;bJ,the cylinder to r¢ach aft~the:breakgg;urs,~---~··---'·~·~-________ _J
[the bottom al th!l. 'mclme. -_ - · _ - -J

www.puucho.com
_I
Anurag Mishra Mechanics 1 with www.puucho.com

,[508
:=:::;:::::::::::;
}·:, MECl!,!!~I~~-~ I
"'~]

I . I ..
I" "'I ~.
' "'
' ~·: , *

1 · . .\>9.° ., ':··"11
L
. I -~~:
>· ;. \

'l
_L___·A_F!_!!; 5E.~BJ!,_,s_.___,_____~, 450 . '. : ' ' 1·
. ., -\>'::·,->ft,:,\,;
'. ,c; ,·::,,, :l'
Solution: When the string at B snaps, the rod has a
'------'--"-"·-----..~----·--·~-
Fig.SE.79.(a)
horizontal acceleration and a vertical acceleration. In
Solution: If the block and cylinder move
addition, .the rod will have an angular acceleration a about
independently on the incline, their accelerations are
t hcentre
o eo.f m.·.ass. Thus the equations ot_ motion are-",...
ablock = g· sm
· e-µ. cos e = grn
..·,,
•-·
. T0J,j · AT sin'60°
,.. i
, 1
2-v2
. ,. -•~I g sine g sin 45° · ./2.g
. -~.' ' ,c-.J>~qo-: r·
Q,a 'l a cylinder = 1 + mR 2 I/ l+(mR /2)/mR
2 2
3
lax ' A ·· T cos 60\~·: ~: - - ·' ·· \
As ai,iock < acylinder• both the bodies will move in contact
q.'.
, .•. . 1. _
I
I.

L •••• ___ : _ : ' ' . F!9-!!g,Z~(~);


mg
· ,a,J with each other with common acceleration a.
,_,,.-;""~- µN1 .. •• ,, ,~~r~::;:;r·7
. :E1:=Tsin60°i = (m!
·2 12
2
)a .. (1)
: N2

LFY = mg -T sin 60° = may ... (2) , ~N


.µN
LF'x = T cos 60° = max ... (3)
,•, "'' ~
We have three equations in four unknowns. We will
seatch for a kinematic relation. The point A on the rod will
..··
move on a circle in the subsequent motion. It can have radial
Ii- . •"
. •rng
and tangential . acceleration. The radial · acceleration l •.,.@
(= v 2 / r) is iero at the instant the string snaps, so the total LI--~"--'-~~--Fi~g._S_E~.7~:c..• _~~<---~·
acceleration of point A along the string is zero initially. From the force diagram block we h'ave
al
Thus we.have, ay cos30°--cos30°-ax cos60°= 0 l:Fx = ~ +N-µN 1 = ma ... (1)
2 -v2 .
Solving eqns. (1), (2) and'(3), we get . . mg
LFy =N1 -µN-:- .Jz =0 ... (2)
2
g _I_Sin60°) COS30°- 3T sin60° COS30°-I_COS 60°= 'o
( m . m . m From eqns. (1) and (2), we eliminate N 1
g cos 30° = I_ [ ../3 ../3 + 3../3 ../3 +
m '2 2 2 2
I.]
4
. mg +N-µ 2N-µmg =ma
.Jz . .Jz
... (3)

~ :[¾+~+¼] = :[1:]
The equations of cylinder are
· 2mg · . ·
LP =---N-f=2ma ... (4)
2
X .Jz .
.· 13
T = mg(.±) c~s 30° = mg
' 13
../3
and :E,; = (f - µN)R = -1 (2m)R
·
2
2(a)-
R .
The point A has tangential acceleration; therefore the
relation 't = la is not applicable about point A. This relation or f-_µN = ma ... (5)
,;-~ la is valid either about a fixed point or about the centre On solving eqns. (4} and (5), we have
of mass of the object even though it is accelerated.
N = m(-J2g-3a) .._. (6)
l-_,,e,xam,:;;ll,e,·.~79 ,v,c),,'
=·---xtr::P-!f.2- ~d / ' : I ~
1+µ
On substituting Nin eqn. (3) and solving for a; we get
I ·block-of md.Tm. and a cylinder,ofmass 2m are.released;~ :,11 , · 3(1-µ)g.
a=-=-~=--- -
3g
... (7)
rough inclined, plane; inclined at an, angle e ='45° with the! .Jz(4-3µ) s.Jz
horiwn.tal. 11,!.e.coeffi.c.iertt.offriction fi._qr an the contactswfacesj
~
. is 0.5. Find_ tlie-accelerations of the}lor.k and cylinder. ·· . '
www.puucho.com
Anurag Mishra Mechanics 1 with www.puucho.com

RIGID BODY fv!~TION 5091


~-~~~--------'--"-----~---------~------'-·-~------"-----''J
From eqn. (7), we get Ifµ is the coefficient of friction at the contact surface,
N = -.J2.mg then
15 f,;; µN = 3 µMg cos0
From eqn. (5), on substituting for a and N, we get or Mg sine ,;; 3µMg cos 0
1
f = -./2.mg or µ:2-tane
3
3
So the minimum value ofµ is 1/3 tan 0.
From force diagram of the cylinder, we ·have·

N2 =
2
'lz
-µN = -.J2.mg(1- ; )
5
k~~~~~'~::_Jsil~
I - · · 7

2g-.J2.mg ;A man pushes a. cylinder ofmass. f!lJ with .the help ofa plank!'
=~-~ 1of mass m2 as shown. There is no.slipping at any contagt. The,
30
I horizontal component of the force applied by the main is El
As f < µN 2 we can say that pure rolling of cylinder takes
place. Both the block and cylinder move with common
!Find: , _____________ .. _____ ---, i
, ' F ------- '
acceleration -!Jz· I ~ ~ - . , , . . ~ , - - ~ m2 I
'

~~i.!;HimJ~I~,:-J ao C;>
fF~: 5E:SO (a) ,sl\-;,s ;liiht rin~ wJth,t;~e;~ods ;;ch-~fm;:1
0

Fig. 5E.81 (a) ,


lM welded_on1ici rim. The rods forrn. an equilateral t[i11.ngle. '1 -· -·----r--·-""'.:'"".-···· ·-·----.,--·! l
!The rigid .assembly is released· on a rough inclined plane. :Ca) ,the accelerations of ihe plilnk and the ·Centre of mass ofi
;Determine the minimum value of the . coefficient of static , the cylinder, and . .. . ,
1
friction th<J.t)\lill allow pure rolling of the assembly. (b) the· magnitudes and directions of.~frictional force at
! .. . .
r- ---'----- --~-- y ·1 · -~ontact,ppznts. ___~---··-.,;,n -------·•-roe,--.- 1

x> - Solution: We can choose any arbitrary directions of'


fractional forces at different contacts. In the final answer the

c:-- ~-~:-~<~)::
i f
negative values will show the opposite directions.

- '
3Mg '
(a)
Fig. SE.BO
(b) f Y %ff#J1:m;;;;yPL,,±u¾¼-1
L---"------ ., Fig. 5E.8.1 (b) . j
Solution: From Newton's second law, the equations Let f 1 = frictional force between plank and cylinder
are f 2 = frictional force between cylinder and gr_ound.
D'x = 3Mg sine- f = 3Ma ... (l) a1 = acceleration of·plank
D'y=N-Mgcos0=Q ... (2)
a2 =. acceleration of centre·of mass of cylinder and
};~ = f R = la ... (3) a = angular acceleration of cylinder about its CM. _
For pure rolling, ' a= Ra ... (4) Directions of f 1 and f 2 are as shown here.
The moment of inertia of the assembly about the centre -Since there is no slipping anywhere,
of mass from parallel axis theorem is
a1 = 2a 2 ... (1)
I= [M(R cos30'x2) 2
+ M(~)
2
x3 (Acceleration of plank = r--·----~----
1,_~i
,•
]
12 _ 2 - acceleration of top point of a,=2a~
2
cylinder) , _. a
=~MR
2 F-ft
· a1 = - - ... (2)
-
'
.'

' '
.
_-
2•
.,
On solving equations · (1), (2), (3) and· (4) m2 '
Fig. 5E.81 (c) .
simultaneously, we obtain ·. ft +f2 ~--=- -. ~-,---· ~
a2 = ~~ ... (3)
· f=Mgsine m1

www.puucho.com
Anurag Mishra Mechanics 1 with www.puucho.com


,a= Cf1 - f2)R C is the centre of mqSS of the hobp and the clamp
I 'r
where AC=-
(I = moment of inertia of cylinder about CM) 4
a= Cf1 -Ji)R and CB= 3r
1 2 4
-m1R
2 a =··'t• = mgr
(a)
2Cf1 - Ji)
a=~c..o_--'-''- IP · 6mr 2 + 2mr 2
... (4)
m1R
For no slipping
or, a_;,, .!,.(J p = I hoop +·rblockl
Br . .
2(f1 -f2l
a 2 =Ra ...(5) 'Hence. the angular acceleration of hoop is .f. .
m1 . . Br
· Solving equations (1), (2), (3) ancl (5), we get
8F . . 4F . ~) _a8 =· (PB)a = ( 5 ) (!r) = ~
(a) a 1 = · and a 2 = - - - -
3m1 + Bm 2 · . 3m1 + Bm 2 '
Hotjzontal component of aB is
' ...
(b) f1 = 3m1F and f2 m1F .Jz' .· '
3m1 +Bm 2 . , 3m1 +Bm 2 2g cos 45° or f --;
B B .
Since all are positive, so they are correctly shown in
figure. · · and verti~al component of aB is
...
.Jz .
k-,~~i1,e~
r - , ~
2g_sin45°
B . B
or ft.

(c)
. . AC r/4 1
tane = - = - = -
I~3ms'mall' cla1'.!p_ •."_._if_·_dl~-· m is -~t.ta;hea··· ._Ji: B' to ahQ?P_ ,o_if_ m
and radius r. The system releasell.from rest withe = 90°1
is
_ ·_ f_ I_
'as'._·,._ '
AP r 4
_and rolls without sliding. Determine · -. " • · or 0= 14°
i ,, . . . . . . _·: _.,
(a) the angular,acceleration of the hiiop, ·.. ' ,
(b) the horizontafand vertical co,riponents oj the acceleration and PC=K=,l.03r
I,)· afB. l_ · ·
,c norma reactto_n
dfri : ·1fi ,;._ ,,.. h · 1···. . . Acceleration of centre-of mass C is .
. . an;
cttona.
' arce,Just
,. : . a,,ert' ere ease., ' :'
.
ac = (PC)a
= (1.O3r) (.L)
Br
= _g_
7.767
Horizontal component of ac is
...
'ax = ac case= g cosi4° = O.125g ~
(7.767) . •
Fig. 5E.82 (a)
'--~--~'"'--''---'--==--·-6--'-----~--' f =(4m)ax = 0.5mg ~
Solution: The hoop rolls without sliding. Therefore,· .
Similarly vertical component of ac is
...
instantaneous axi~ of rotation passes through the
bottommost point. a = a sine= _g_sin14°= 0.03g-l,
7.767 ·
,rx
y, .
Now
Hence
y C

4mg - N = (4m)ay
N = 4mg .:. 4may ·
N,:::: n.ormal N = 4m(g - ay),
reaction
N = 4m(g-O.O3g)
N=3.BBmg1' .

'
Fig. 5E.82 (~);

www.puucho.com
Anurag Mishra Mechanics 1 with www.puucho.com

I'RIGID BODY MOTION'

---------- -
--------.---------------,
..

1 · Only One Alternative is Correct


- ~ - - - - - - - - - - - - - ------- ---- -- -------------~~

1. A homogenous rod of length I = T\X and mass M is lying (a) Mr 2 (b) _!Mr 2 .
on a smooth horizontal floor. A bullet of mass m hits 2
. 1
the rod at a distance x from the middle of the rod at a (c) -Mr 2 (d) 7:_Mr 2
velocity v O perpendicular to the rod and comes to rest 4 5
after collision. If the velocity of the farther end of the 6. A ring of mass M and radius R is released
rod just after the impact is in the opposite direction of on an inclined plane as showri. If the
v 0 , then: 1
coefficient of friction µ < -tan8, then
(a) T\.> 3 (b) T\ < 3 2
(cl T\ > 6 (d) T\ < 6 during a displacement 1:
2. A thin circular ring of mass M and radius r is rotating (a) Acceleration of the ring = g/2sin8
about its .axis with an angular speed 0). Two particles (b) Acceleration of the ring= g sin8 .µg case"
having mass m each are now attached at diametrically (c) work done by the force of friction
opposite points. The angular ·speed of the ring become: ·. = mgl(sin8-µcos8)
OJM' . ooM (d) work done by the force of friction is zero
(a) - - ' . (b) - -
M +m M+2m 7. The line of action of the resultant of two like parallel
(c) (M - 2m)
Ol (d) OJ (M + 2m) forces shifts by one ..fourth of the distance between the
M+2m . M forces when the two forces are interchanged. The ratio
3. A flywhee!rotates about an axis. Due to friction at the of the two forces is:
axis, it experiences an · angular retardation (a) 1 : 2 (b) 2 : 3
proportional to its angular velocity. If its angular (c) 3 : 4 (d) 3 : 5
velocity falls to half while it makes· n rotation, how 8. A mass m is moving with a constant velocity along a
many more rotation will it make before ~oming to line parallel to the x-axis, away from the origin. Its
rest? angular momentum with respect to the origin:
(a) 2n (b) n (a) is zero (b) remains constant
(c) !: (d) !: (ell goes on increasing
2 . 3 (d) goes on decreasing
4. The locus of all the points on the x-y 9. A person sitting firmly over a rotating stool, has his
plane about which the moment of arms stretched. If he folds his arms, his angular
inertia · of the rod is same as that momentum about the axis of rotation: I
about point b, is: (a) increases (b) decreases ·
(c) remains unchanged (d) doubles
(a) Circle (b) Parabola 10. Two uniform rods of equal length but ,--:'"-7
(c) Straight line (d) Ellipse different masses are rigidly joined to
5. The moment of inertia of a uniform semicircular wire
of mass M and radius r about a line perpendicular to
form an £-shaped body, which is then m 60°: Mi
pivoted about O as shown. If in
:~1
I
-the plane of the wire through the center is:
\
!
I
I ~o :
I
www.puucho.com
. \ .\ .
'
.
Anurag Mishra Mechanics .
1 with www.puucho.com
I
~/ I

,,,,-,

'a'
'
equilibrium· the body is in the shown. configuration, (cl cos 2 8 (d) independent of 8 ,
. Mwillb e:
rano- 17. One end of a uniform rod of mass m and length I is
m· .
clamped. The rod lies on a smooth horizontal surface
· (a) 2 (b) 3 and rotates on it about the clamped end ·at a uniform ·
(c} ./2 (dl 13 - angular velocity ro. The force exerted by the clamp on
11. A solid sphere of radius R has
moment of inertia I about its
geometrical axis. It is melted into a
disc of radius r and thickness t. If
it's moment of. inertia about the
p .

.
.

tangential axis (which is perpendicular to plane of the


.
. .
the rod has a horizontal component:
(a) mro 21
.
(c) mg
.
(b) zero
1 .
(dl -mro 21
2
·

18. A large platform is moving with constant acceleration


'a' perpendicular to its plane in gravity free space. A
disc), is also equal to I, then the value ofris equal to; particle of inass m is projected with speed 11 relative to
2 2 the platform at an angle 8 with its plane from a point O ·
. (al .ffsR (b) ../sR

· 3"
(c) · - R
- .ffs
fu3
(d) · -R.
15
on it. The angular momentum of the particle about 0:
(a) Always increases
(bl Always decreases
12. A body of mass m is moving with a constant velocity (cl Firs_t decreases and- then increases
(d) First increases- and then decreases
along a line parallel to the x-axis, away from the
origin. Its angular momentum 'with. respect to· origin:
(a} is zero · ·
19. Acubicalblockofsideamoving ~a-
· with velocity v on a horizontal
IE] · · '/
_ · ... j

(b) remains constant · smooth plane as · shown· in ·• M _ ~!' ·· .,


(c) goes on increasing : "figure_. It hits a ridge at poi~t 0. f",7""'"\'\~m\m ·oj
(dl goes on decreasing The angular speed of the block ' . . , · · .
after it hits O is:
13. -A disc of mass Mand radius R ~.-.· - ~ - . , ~
r::. (a) 3v/( 4a) ·. · (b) '3v/(2a}

~~:nlnJ~:: ~~1:. ·.:. ·,',;<l


is rolling with angular speed v,, ~: J (cl :..f3v/(..J2a) (d} zero
20,. A wheel of radius r 'rolls without slipping with a speed
v on a horizontal road. When it is at a point A on the
disc about the cirigin O is: road, a small blob of mud separates from'the wheel at-
2 -its highest point and lands at point B on the road:
(al MR ro ·(b) MR2ro
2 .. ca)All=~fi · ·. ·
(c) 3MR m
2'
(d) 2MR 2 00
. v'i
2 .·
14. A cylinder of mass m is rotated about
Cb)
. v'i
Aii = 2v Ti
its axis by an angular velocity ro and
lowered gently on an inclined plane as 1µ=)'£ ·.
~~--·
_.
(cl AB= 4v
v'i
f"i
shown in figure. Then: L 30
(d) if v > J4ii,
the blob of niud land on the wheel

107',J
(a) It will start going upward
(b) It ;will first going upward anci then downward · ·
(cl lt will go downward just after it is lowered
(d) It can never go upward
21.
· and not on the road
A string of negligible thickness
is wrapped several times around : , .
·
.i]·
15. A wheel _of radius R rolls_ without slipping on a
_horizontal ground. The distance travelled by a point
a cylinder kept on a rough
· IJopzontal surface. A_ man
standing at a distance 1from the
• ·
1/~]/li//»;11,1
·

on the rim .in, one complete_ rotation is: cylinder holds on end of the' string and pulls ,the
(a) 27tR - (bl BR · . ' .
cylinder towards him as shown in figure. There is no
(c) 2R · (dl 7tR slipping anywhere: The length of the string passed
Two identical rods_ are· joined to form an 'JC_: The through the )land of -the man. while the cylinder
smaller angle between the rods is 8. The moment· of .. n;aches· his hands is:
inertia of the system about an axis passing through (a) l _ - (b) 21 .
· point of intersection of the rods and perpendicular to (c) 31. (dl 4l
their plane_ is proportional to:
(al8 (blsin 2 8

www.puucho.com
Anurag Mishra Mechanics 1 with www.puucho.com

22. ABC is a triangular plate of uniform. thickness. The rests on the ground. Initially; P is fixed to the ground
sides are in the ratio shown in the figure. IAB,IncJCA by a pin N. If N is suddenly removed: ·
are the moments of inertia of the plate about AB, BC (a) S will begin to slip on P
and CA respectively. Which one of the following (b) P will begin to .move backwards
relation is correct: (c) The speed of S will decrease and its angular
' -A- - - - -- ·1 · velocity with increase

'i~i
L..s ..4 _______ ,
(a) I CA is maximum (b) I AB > I BC
(d) There will be no change in the motion of S and P
will still be in rest
28. Three identical solid spheres move down three inclin~
A, B and Care all of the saine dimensions. A is without.
friction, the friction between B and a sphere is
(c) Inc•> IAB (d) IAB +Inc= ICA sufficient to cause rolling without slipping, the friction
/
23. A solid homogenous sphere is moving. on a rough between C and a sphere causes rolling with slipping.
horizontal surface, partially rolling and partially The kinetic energies of A, B, C at the bottom of the
sliding. During this kind of motion of the sphere: inclines are EA,EB,Ec: i
(a) Total kinetic energy is conserved 1
(a)EA=En=Ec (b)EA=En>Ec
(b) Angular momentum of the sphere about the point (c)EA>En>Ec (d)EA>En=Ec
of contact is conserved 29. A sphere is rotating about a diameter:
(c) Only the rotational kinetic.energy about the center (a) The particles on the surface of the ·sphere do not
of mass is· conserved have any linear acceleration
(d) Angular momentum about the center of mass is (b) The particles cin the diameter mentioned above do
conserved not have any linear acceleration
24. Let I be the moment· of inertia ·of a uniform square (c) Different particles on the surface have different
plate about an axis AB that passes through its center angular speeds
anc\ is parallel to two of its sides. CD is a line in the (d) All the particles on the surface have same linear
plane of the plate and it passes through the center of speed ·
the plate, making an angle 0 with AB. The moment of 30. A thin spherical shell of radius R lying on a rough
inertia of the. plate about the axis CD is equal to: horizontal surface is hit sharply and horizontally by a
(a) I (b) I sin 2 0 cue. At what height from the ground should it be hit so
that the shell does not slip on the surface:
(c) I cos 2 0 (d) I cos 2 (-~)
(a) 3_R (b) ~R
25. 1\vo uniform solid spheres having unequal masses and 3 4
unequal radii are released from rest from the same · (c) ~R (d) ~R
height on the rough incline. If the spheres roll without 3 2
slipping: 31. 1\vo moving particles P and 6fTYSec;
(a) The heavier sphere reaches the bottom first Qare 10 m apart at a certain i " I
30
(b) The bigger sphere reaches the bottom first instant. The velocity of P is ,p•---~="'-- >

(c) The two spheres reach the bottom together 8 rn/s making an angle of I
(d) The information given is not sufficient to tell 30° with the line joining p L.. - Sm/sec_ --
which sphere will reach the bottom first and that of Q is 6 rn/s making an angle 30° with PQ as
26. A particle of mass m = 5 kg is moving with a uniform shown in t:l}e figure. Then angular velocity of P with
speed of V = 3-.12 rn/s in XY plane along the line respect to Q is:
y = ( x + 4). The magnitude of angular momentum (a) 0 rad/s (b) 0.1 rad/s
(in kg- m 2 /s) about origin is: (c) 0.4 rad/s (d) 0.7 rad/s
(a) zero· (b) 60 32. A uniform rod oflength 1and mass m makes a constant'
angle 0 with an axis of rotation, which passes through
Cc) 7.s Cd) 4oFz
one end of the rod. Its moment oi inertia about this
27. A sphere S rolls without slipping i ·-·- · · -- - I axis is:
moving with a co~st~nt speed on \ N ~ ~ v = wi m1 2
a plank P. The frtctmn between . I ' P · j
(a) m/2
3
(b) -sin0
3_
the upper surface of p and the. •J,; fmm2=' ----~
sphere is sufficient to prevent · m1 2 : 2
(C) --sm. 0
m1 2
(d) -cos 2 0
slipping, while the lower surface of P is smooth and 3 3
www.puucho.com
Anurag Mishra Mechanics 1 with www.puucho.com

,· : 'MECHANICS,! I
33. A sphere can _roll on a surface inclined ~t an· angle 0 if (d) Its linear velocity will decrease
the friction c~efficient is more than ~ g sin 0. Suppose 39. A uniform rod is kept vertically on horizontal smooth a
' . . . 7.
1 . . surface at a
point 0. If ,it is rotated slightly and
f. the fr!ction coefficient is -g sin 0. If a sphere is released, it falls down on horizontal surface. The
7 ·. ' lower . end will remain: ·
released from rest on the.incline: ·
(a) At 0
(a) It will ;tay at rest · ·
(b) It will make pure translational motion (b) At a distance less than ..!. from O
2
(c) it will translate and rotate about the center
•· · (d) The angular momentum of ·the sphere about its- (c) At a distance..!. from·O
2 .
center will remain constant .. .
''A . ' • 1
(d) At a distance larger than ..!. from 0
34. What is the' mo~ent of inertia of a
triangular plate ABC of mass Mand ~
side BC = 'a'
about an· axis passing • . . " :
I 40. · A uniform rod of length 1 and ,
.
~
2
r.----. -.- -
845°45_() mass M· is suspended on two I j... . ; .. TJ
through A and· perpendicular to the .. ....,_,8
plane of the plate? ·
2 ' .
, _J

·2
vertical. inextensible string as
shown in figure. Then tension T .
d . ;.. '
,
-'_ ' u·... . 1
(a) Ma (b), 3Ma . in the left string at the instant, . ,. ·
6 4 when right string snaps is? ~l · ·
. Ma
. (c) -·-.
2
(d) Ma
2· r • ~-

. · 24 ·. . 12 (b)T=Mg
(a) ,T=Mg
35. A rolling body is kept on a plank B .. 2
· There is·sufficient friction·,between '
! (c) T = Mg (d) T = zero
··A ·and B and no friction between B . . ' 4
and the incline,d plane. Then body:· . . :·.,~ 41. An equilateral triangie AB¢ . ----,--.,~:,_· ;_"·f_~_ ..-:
· (a) A rolls .....-·-··..t. formed- from a uniform wire. ~ •
(b) A ,does not experience any friction has two small identical ' ,: A '- •· · : '
(c} A a~d B has equal .acceleration and unequal beads initially-located at A. i; ::\ ~-· '1 ! !II\·''· ·1
. velocities . . · . ( The triangle is set rotating ·
(d) A rolls depending upon the angle of inclination d- · about .the vertical. axis AO.
0
. .. i,· , 9
. 36. Three identical rods, each of length !, are joined to· Then the beads are released ·:
· · · form a equilateral triangle. Its radius·of gyration about from rest simultaneously s· ·. :o C
-an axis passing through a comer and perpendicular to and allowed to slide down, ~-----'---'
.the plane of .the triangle is: · · · one along AB and the oth~r a)ong AC as shown .
Neglecting frictional effects, the quantities that are
· (a) i _(b) @z
.
2
l
.
.
•Vzl conserved .as the beads. slide down, are: .
(a) angular velocity ;µid total energy (kln;tic and
. (c)_ ../2 (d) .J3 · potential}: .
.Cb) total. angular momentum and total energy
· 3 7. · A sphere cannot roll on:: (cl· angular velocity and l!loment of inertia about the
·· . (a) A slnpoth,horizontal surfac'e axis of rotation .
(b) A smooth inclined plane . , · (d) total angular momentum ;ind moment of inertia
· (c) A rough horizpntal surface about the axis of rotation.
I (d) A rough inclined. ~urface . ·j

I · _-. . .
· 42. ·A translational velocity v O is imparted in a horizontal
direction to a- hoop of radius r placed on a rough_
.... ·plane-fixed
show,.~·"' kclmo!
in a car accelerating on a ~-~.I horizontal surface. What is ·angular "l{elocity of. hoop
, horizontal road. The angle cif incline . after it.stops slipping ? ·· J .
0 is .relatec! ·to the· acceleration 'a' of •. . )
(b) ~
as i
the ~M a ;,, tan e. If the sphere is ..... • _ ._ . ... ___.'.J
(a) Vo
r
" •
2r
set in pure rolling on· the incline: 2
· (cJ.. _v 0 ,l · (d) ze~o
(a) It will_ conti~ue pµre rolling r ... {,
· (b) It will slip down the plane
,· '

(c) Its linear velocity will increas~-


www.puucho.com
Anurag Mishra Mechanics 1 with www.puucho.com

L_R~~~ BO!)Y M_!)_TI_ON_ _ _ ___5_15J


43. A sphere of mass m is given i
some angular velocity about a
fi ' ' i 48. A round uniform body of radius R, mass· M and
moment of inertia' I', rolls down (without slipping) an
horizontal axis through the I om \ inclined plane making° an angle 0 with the horizontal.
center, and gently placed on a i ', Then the acceleration is:
i.m~....::.......-<---; _I
plank of mass m. The (a) gsinO (b) gsinO
coefficient of friction L - ---- -- ---- _:_ ~ 1.+~ l+MR2
between the two is µ. The MR I
plank rests on a smooth horizontal surface. The intial ·
~ g~O ~ g~O
acceleration of the sphere relative to the plank will be:
·1 _ _
I_ MR2
(a) zero (b) µg MR2 1--J-
(c) ~µg (d) 2µg

r;::y
49. A thin rod of mass m and length 1is hinged at the lower
44. A semicircular lamina of mass m ,· _ ··.-· __t___ · ··-- ·t end to a level floor and stands vertically. Then its
upper end will strike the floor with a velocity given by:
and radius 'r' and center C. Its
center of mass is at a distance 'x' l. ,, ' I (a) fii/. (b) ..pgi.
from C. Its moment of inertia L (c) -Jsif
50. A cubical block of side L
(di .Jmgl
·F___:_._~-T- '
about an axis through its center -- -- -- -·- -· ~
of mass and perpendicular to its plane is: rests on a rough horizontal , ~ .
surface with coefficient of I L i
(a) .!: mr 2
2 .
(b) .!: mr 2
4 friction µ. A horizontal l '.
force F is applied on the ' _ _Jc:s..;s_c:s....:,...;:,...,_~:
2
(c) .!:m~ +mx 2 (d) .:1:mr 2 -·mx 2 block a:s shown. If the • · - -·· -- · · · -- -~
2 2
coefficient of friction is sufficiently high so that- the
45. A disc of mass m0 rotates freely r - - - · - . - ---· ~
about a fixed horizontal axis I : .. I block does not slide before toppling, the minimum
through its center. A thin cotton pad !~~-'
force required to topple the block is:
is fixed to its rim, which can absorb : ' (a) infinitesimal (b) mg/4
water. The mass of water dripping [ \'a
(c) mg/2 (d) mg(l-µ)
onto the pad is µ per second. After L - - - ~ - 51. One quarter sector is cut from a ,-· --,- - --.---,
what time will the angular velocity
___ _
uniform circular disc of radius R. : Y ·
of the disc get reduced to half of its initial value: The sector has mass M. It is made to !
(a) 2m 0 (b) 3m 0 ,
rotate about a line perpendicular to ' ~
µ µ its plane and passing through the J : ~ I
centre of · the original disc. Its L_:__'____:_j
(c) -mo . (d) mo moment of inertia about the· axis of
µ 2µ rotation is:
46. A plank P is placed on a solid (a) ~MR 2 (b) ~MR 2
cylinder S, which rolls on a 2 4 ..
horizontal surface. The two are of
(c) _!:MR 2 (d) -.J2MR 2
equal mass. There is no slipping at 8
any of the surfaces in contact. The 52. A cylinder rolls up an inclined plane, reaches some
ratio of kinetic energy of P to the kinetic energy of Sis: height, and then rolls down (without slippfng
(a) 1 : 1 (b) 2 : 1 throughout these motions.) The directions of the
(c) 8 : 3 (d) 11 : 8 frictional force acting on the cylinder are:
47. A thin wire of length L and· (a) up the incline while ascending and down ,the
uniform linear mass density p x :x' incline while descending
is bent into a circular loop (b) up the incline while ascending as well as
with centre at O as shown. descending
The moment of inertia of the (c) down the incline while ascending and up
the
loop about the axis XX' is: incline while descending
(a) pL3 (b) pL3 (d) down the incline while ascending as well· as
81t2 16ii:2 descending
53. A circular platform is free to rotate in a horizontal
(c) SpL3 (d) 3RL3
plane about a vertical axis passing through its, centre.
l61t 2 81t 2 A tortoise is sitting at the edge 'of the platform. Now,
www.puucho.com ·.. \
Anurag Mishra Mechanics 1 with www.puucho.com

""'
:rT,:it""i~=~
(c) Particle moves in- a· spiral path with decreasing
radius . 1 ,, -

(d) The direction of angular momentum remains


constant
59. · From a circular disc of radius Rand mass 9M, a small
disc of radius R/3 touching he ·circumference bf larger
disc, · is removed. The moment of inertia of the
remaining disc about an axis perpendicular to plane of
.the disc and passing through center:
(a) 4MR 2 (b)
40
9
MR 2
37
(c) l0MR 2 (d) MR 2
9
-60. A 'T shaped body with dimensions
shown in the figure, is lying ~n a
...,
smooth floor. A force ' F is applied at

figure,' -· consisting of two _M f---.-'-L


· identical·balls, each of mass-M- ,; ., ,.
l
54. Consider a' body, shown· in (· ·. · ··:-·-x •• · · ,-. c- ,1
M: ·
J = MV
the end P parallel to AB, such that the
object has only translational motion
without rotation. Find the location ·of
connected by a light rigid rod. -' - · · - ' · ·· P with respect to C.
If an iIµpulse J = MV is imparted to·the body at one of 2 3 , 4
(a) -L (b) -L (c} ~L (d) L
.. its end~;what would be its angular .velocity? 3 2 3
(a) V/? •· (b) 2V/L 61. A thin circular ring of mass m and radius R is rotating
(c} V/3£- '· c- (d) V/4L about its axis with a constant angular velocity OJ. Two
55; A part(cle µndergoes• uniform circular motion. About objects each of mass M are attached gently to the
whi~li point on the plane of the•circle, will the angular · opposite ends of a diameter of the ring. The ring now
momentum of the particle remains conserved? rotates with an angular velocity ro' =
(a) centte of the circle . (a) . • rom (b) rom
(m+M) (m+2M)
(b) on !the 'circumference of the circle ·
(c) lo.side the circle (c) ro(m + 2M) (d) ro(m- 2M)
(<l) ,oiitside the circle m (m+2M)
•i\'', '-
62. A force of -Fk acts on 0, the origin of r··,-,,-- ... ·•:
56. A, disc ·is rolling on a horizontal
. $.urf;i~fC is its center and P and Q .
,• a,ri;,:i:w~ points eqtlidistance from : . : : ,; '''a. :
~l
the coordinate system. The torque r
about the point (1, -1) is: 1
1s-·
' ,. .'.': ' • ·

:;i:t:~·~::·· J
· ' i-
1 ,
'0 "
,
-
~, Y;!
e~~ti~rr~··oi~e;i~~iesv~fpbo~nt~;; \ ' •
.
I
I, ~ ~ "
I
. LX_•,·,,.,;,;..;,;..,,__,,~_ 0

Q a~d C respectively, then:· · " ...1


'' • l • ,- -
(a) F(i + j) (b) -F(i-j)
(a) Y~ ;, ".c >Cr' · .·, . (b) V6 <· Ve < Vp (c) F(i- J) (d) -F{i + j)
(c} VQ ~ YP;· 'vc = Vi, /2 (dj VQ < Ve <Vp
63. Four point masses, each of value m, are placed at the
57. A child'is standing with folded harids at the centre of a comers of a square AflCD of side l. The moment of
platform rotating about its central axis .. The kinetic inertia of this system about an axis passing through A
energy of the system is 'K. The child now stretches·his and· parallel to BD is:
hands sb· that the moment of inertia of the system (a}• 3ml 2 (b) m1 2 .(c) 2ml 2 ' (d) ..J3ml 2
doubles. The kinetic energy of the system now is:
(a) 2K I (b) K/2 . 64. For the given uniform square lamina ABCD, whose
centre is 0:
(c) Kj4 \ (d) 4K
i
58. · A particle moves on· circular path with. decreasing
(a) -J2JAc = JEF
(c) !Ac= JEF
(b) IAD = 3JEF
(d) !Ac= ./iIEF
speed. Choose the corr~ct statement.
(a) Angular momentum tei)lains constant 65. A uniform thin bar of mass 6 m and length 12 L is bent
..., to ·make a regular hexagon. Its · moment of inertia
· (b) Acceleration a is towards the centre
www.puucho.com
Anurag Mishra Mechanics 1 with www.puucho.com

!·RIGID BODYMOT.ION ·.
about an axis passing through the centre of mass and (a) -2%. · ' .. · .. (b)-, 1%.
' .
perpendicular to the plane of hexagon is: (c)-3% (d) 1%.·
. { - -
(a) 20mL2 (b) 6mL2 · 72. A solid sphere and a hollow sphere of equal inass and
12 2
(c) mL (d) 30mL2 radius are placed over a rough horizontal surface iifter
5 . rotating it about its mass centre with the same angular
66. Two rings of s~e raruus (r) and mass (m) are piaced velocity m0 • Once the pure rolling starts l~~ v 1 and v 2·
such that their centres_ are at a common point and their be the linear speeds qf their ~entres pf mass. Then: .
planes are perpendicular to each other. The moment of (a) V1 =V 2 ' ·, •
inertia of the system ab_out an axis passing through the (b) V1 > V2
centre and perpendicular to plane of one of the ring is:
1 ' (C) V1 <V2
(a) - mr 2 (b) mr 2 .(d) data is insufficient
2
73. In the above problem let t 1 -and t 2 be:the times when -
(c) ~ mr 2 ·cd) 2mr 2 pure rolling of solid sphere and of hollow sphere- is
2
·started. Then: · · · . · ·
67. In a rectangle ABCD, AB = 21 and BC = L Axes xx and
yy pass through centre of the rectangle. The moment . (a) t 1 =t 2 (b) t 1 <t 2
of inertia is least about: · · (c) t 1 > t 2 (d) none of these.
(a) DB 74. A circular platform is · mounted on -a vertical
(b) BC frictionless axle. Its radius is r = 2 m and its moment of
(c) xx inertia is I =200kg-m 2 • It is jnitially at rest. A 70 kg
(d) yy man stands on the edge of the platform and begins to
68. Moment of inertia I of a solid sphere about an axis walk along the edge at speed v O = LO m/ s relative to
parallel to a diameter and at a distance x fromit varies the ground. The angular velocity of, the_ platform is:

,.,~. ~,0
·as: (a) 1.2 rad/s ·· (b) 0.4 rad/s
(c)
. .
2.0 rad/s
. (d) 0.7 rad/s
. ·.
·75.. In the above problem when the man has walked once
around the platform so .that he is at his original
position. on , it, ·what is his angular displacement

,,[J
relative to ground ?
(~) ~1t (b) ~1t
. 5 6
4 ·
:.(c) -1t
5
(d) -1t
-5 4
69. A wire of length l and mass· m is bent in-the form of a 76. A ball of radius r . rolls inside a 1" 'm
. - ~.

hemispherical shell of radius R. It is r · YI


rectangle ABCD with AB = 2. The moment of inertia of ·released from res_t from point A as shown t_:_'._~a=-.J
. BC ... ·
this wire frame about the side BC is: . in figure. T])e angular velocity of centre of
the ball in positionB about the centre of the shell is:
. (a) __g_mz2 (b) ~ml 2
252 . 203 (a) 2 ~ (b) ~ -..
(c) ~ ml 2 (d) _?_ ml 2 Js(R-=-r) V7(R =-r)
136 . · 162
70. A hoop rolls on a horizontal ground , ~ - (c) er
-V5(R-'r) ,.·
(d) ~
.•. V2(R=-r)
without Slipping with Jinear Speed V, , V
Speed of a particle P on the _...:;-;:-' , , 77. In the ~hove· pro6lem th~ nonnal force b~t:ween the .
6 ball and the shell in position B is (m ·=mass of balli:
circumference of the hoop at angle 8 is: ._:__ :- · ·
12 . 7, .
(a) 2vsin(!) . (b)_vsin_B ._ . _. (a) -mg (b) -mg
7 . 9
(c) 17 mg (d) 10 mg
(c) 2v cos(!} ·: (d) v_cosB . 7. ·. 7_ ·
71. A. rigid spherical body is. spinning around an axis
without any external torque. Due to temperature its
volume increa~es by 3%. Then percentage change in
its angular speed is:
www.puucho.com
Anurag Mishra Mechanics 1 with www.puucho.com

l,518
78. A disc of radius 0.1 m rolls without 83. A billiard ball is hit by .a cue at a height h abpve the
sliding on a horizontal surface with a centr.e, It acquires ·a linear velocity v 0. Mass of the ball
velocity of 6 m/s. It then ~scends a is m and radius is r: The· angular velocity co O acquired
smooth continuous track as shown in· t~~'§;;,-l by the ball is: ·
figure. The height upto .which it will (a) 2v 0 h (b) 5v 0 h
· ascend is (g = 10m/s 2 )_: · · 2r2
'' 5r 2
(a) 2.4 ni (b) 0.9 m 2
. · (c) ,2v 0 r ~d) 5v~2
. (c) 2.7 m, (d) 1.8 m
· Sh
!s
79. A sphere 111?ving on a smooth r;::""" mof'. ·•
,;';::;il 84., A wheel ofradius R rolls on the ground with a uniform
surface with ~ear speed ~o and I ~.··- · v~ J_·.
t'.':_ -1 velocity v. The relative acceleration· of topmost point
angular-.velocity m0 • It find,s a .L~m,&:J of the wheel with respect to the bottommost point is:
rough inclined surface and it ·
· v2 2v 2
starts climbing up: (a)- . ,(b)-
·ca} ifv 0 > Rro 0 , friction force will act downwards R R
,', . 2 . 2
· (b) if v 0 • < Rm 0, friction force.will act upwards (c) .L (d) 4v
(c) if v 0 = Rro 0 , no friction force will act 2R R
(d) if all above· case fricti,on will act upwards·. 85. A plank with a uniform
80; An inclined plane makes an angle of 60° · with sphere placed on it
, horizontal. A disc rolling down this inclined plane resting on a smooth
without slipping has a linear acceleration equal to: horizontal plane. Plank is
pulled· to right by a constant force F. If sphere does not
(a) f (b} ~g
slip -o~er the plank. Which of the . following is
. '3
. . .· 4
·, g g incorrect? · .. · ' · . · ·' .,
. (c) - (d)- •
(a) Acceleration of the centre of sphere is less than
. ../3 2
that of the plank
.81. ·A t.qrce Fis applied ~t the twp of a ring·
of mass M and radms R placed on a 1. ·
l·M·CTJ'F .
, ;
(b) Work done by friction acting on the sphere is equal
to its total kinetic energy
rough horizontal surface as shown in [. , : (c) Total kinetic energy of the system is. equal to work
figure. Friction is ·sufficient to prevent '_.'_·_·_ _,_ ' . done by the force JI
slipping. The friction force acting on · (d) None of tlie these
the ring is:
F . F . 86. A rod of length Zis given two velocities v 1 & v 2 in
(a) - towards right (b) - towards left . oppo'site directions at its two ends at right angles to
2 3 '
2F' . the length. The· distance of the instantaneou_s axis of
(c) -towards right ' (d) zero rotation from v 1 is:
' 3 . . 5' .:
82. In both the figures ~:~ ~: , l · (a) zero (b)· · v,

same, except. that


.· in figure (i) AB !s
l '
all other factors are , 11
, "
B ' ·
1,
(i) . , "
h
,
'' 8 , '
,:
, (ii) •
•• i
i
(c) ·v2l

0
v1 +v 2
. (d)
V1 +V2

2
I
rough and- BC 1s ~-- - . · - "'--l 87. Two particle of equal mass m at A and B are connected
smooth while in figure (ii) AB is smooth and BC is by a rigid !ight,rod AB lying on a srn.ooth horizontal
rough. , Kinetic energy of the ball pn reaching the table. An impulse J is applied at A in the plane of the
. bottom: · · table and perpendicular at AB. The velo~ity of particle
(a) is same in both the cases at A is: ' ··
(b) is greater in case, (i) (a) _.!_ (b) !...
(c) is greater'in.case,Cii) 2m m
21
(d) information insufficient (c) (d) zero
,m I'.
' '
•'·I
·'

www.puucho.com
''
Anurag Mishra Mechanics 1 with www.puucho.com

.
; RIGID BODYM_O_T_IO_N___ -_-·-----~--==: ~---···_-·_·------'-------'--"'"----'---'---~-•c.'--~---~-,.2..:·,!&19 i
r 2-- -
-··- -----.
;,~;;than One Alte~n;~ive
-------· .
isi~~~ Corr;~~-:---
'
-----···---------------------- --- . .
-j
'
1. A ring of mass m and radius R · r - - -- ·-- ·-·-7
has ~hree particles. attached ·to j ~ ~..r
the nng as shown m the figure.
_ro_ o
:
, I ,
i
The centre of the ring has a speed · · !
v O• The kinetic energy of the l • - - - --- -- ... - -'~.
system is: (Slipping is absent). (a) If the ends A and A' are suadenly . fixed
(a) 6 mv6 (b) 12 mv6 - simultaneously, both rods will rotate with the
(c) 2 mv6 (d). 8 mv6 same angular velocity
2. A block of mass m moves on a horizontal rough surface (b) If the ends A . and A' are suddenly fixed
with initial velocity u. The height qf the centre of mass simultaneously, the rod Q will rotate with great~r
of the block is h from the surface. Consider a point A angular velocity ·
on the surface: (c) If the ends B and B' are suddenly fixed
(a) Angular momentum about A is mvh initially. simultaneously, both rods 'will. rotate with the
(b) The velocity of the block decreases as time passes
same angular velocity
(c) Torque of the forces acting on block is zero about (d) If the ends B and B' are suddenly fixed
A simultaneously, the rod P will rotate with greater
(d) Angular momentum is not conserved about A
. '
3. A uniform circular disc of radius r ._: . . ·- .·-.
·.--~~o_
angular velocity
6. A thin uniform rod of mass in and length L rotates with
the constant angular velocity OJ about .the vertical axis
,placed on a rough horizontal plane has
initial velocity v O and an angular
velocity OJ 0 as shown. The disc comes
to rest after moving some distance in
the direction of motion. Then
1
2 __
passing through the rod's suspensjon point · 0. It
describes a carried surface, then:
(a) centrifugal force acting on rod is m~sin0ro 2 and
. 2 .
will pass through centre of mass
(a) the friction force acts in the backwards direction
(b) the point of contact of disc with ground has zero · (b) centrifugal force acting on rod ism~ sin0ro 2 and
. . 2
velocity
will not pass_ through centre of mass
(c) v 0 must be equal to roio in magnitude
2 (c) e =. cos-1 (2m2L
~)
(d) v O must be equal to 2rro O in magnitude
(d) e = cos-1 (_.!_)
ro 2L
4. A non-uniform ball of radius R and radius of gyration
about geometric centre = ~, is kept on a frictionless 7. A thin uniform rigid rod of length 1 is_ hinged at one .
' 2 'end so that it can move in- a vertical plane by rotating
surface. The geometric centre coincides with the about a horizontal axis through upper end. The lower
centre of mass. The ball ·is struck horizontally with a end is given a sharp blow and made to acquire a linear·
sharp impulse = J. The pciint of application of the velocity v O•
impulse is at a height 'h' above· the surface. Then: Maximum height attained by lower f•·
(a) the ball will slip on surface for all cases end of the rod is: ,
· 2 I
Cb) the ball will-roll purely if h = SR (a) .':'_Q_ for v O < -J6i[
. 4 3g ..
3
(c) the ball will roll purely if h = R
. . 2 .(b) -~20 forv 0 < -J6i[
g L --·
•o. I
(d) there will be no rotation if h = R
5. Jwo equal uniform rods P and Q move with the same (c) 2lforv 0 ?::-/6i[
v_elocity v as shown in the fi~ure. The second rod has (d) 1 forv 0 = ,/3ii.

an angular velocity OJ ( < ~) (dockwise) about G_' in


addition to v.
www.puucho.com
Anurag Mishra Mechanics 1 with www.puucho.com

/
\
,/// r.:::~=~=======:;=e:==;=====~============
· Li52() , ." 'h , : MECHANICs.fJ

ii. A· cylinder is rotated clockwise and ,-,,...,..-7


I ' - EJ;W 12. Two rods OA and OB of equal
lowered slowly on a rough inclined length· and mass are lying on xy
plane \\tith =
(µ 0.8). Then: plane as shown in figure. LetI x, I y
(a) .cylinder will start going upwards '
·1 & I• be the moment of inertias of
both the rods about x, y and i axis
(b) cylinder will start going downwards ~-=,-.;,.,.,__,,;i ' respectively. Then:
(cj frictional force will act upwards
(d) frictional force will act downwards (a) Ix =ly >I, .(b)Ix =ly <I,
9; A wad of sticky clay of mass ~ .. : ---;,--,,.....,--,·7 (c) Ix->Iy·>I, (d)I, >Iy >Ix
mandvelocityv;isfiredat ·
.
a solid cylinder of mass M
. and : radius R figure. The ,
m~ .
d
cylinder is· initially at rest ",' ·
· '-.
· R
· ···
l'
, ' \
··
,'.
,

l 13. Two skaters approach each other as shown in figure
and lock hands. Given each has an initial speed of 2.50
m/s relative to ice. Each has a mass of 70.0 kg and
·their centres of mass are ·o.soo m from their locked

: ~- ~2-
,. and is mounted on a fixed ' . . '.L..... hands.' You may approximate their moments of inertia
)mrizonta! axle· that runs through the centre of mass. to be that of point masses at this radius: ·

r~~-~
The line of motion of the projectile is perpendicular to
the axle and at a -distance d; less tha;n · R, from the
centre. . , .
(a) Angular velocity just. after collision is
_ ID = :. .·.:. ·•_2m_v_,;d__·
(M+2m)R 2 •
(b) ynear momentum of' c,:ylinder. and·_ clay is.
r ;l. ,_~1--)-'---~--'
(a) Final angular velocity is 3.12 rnd/s .
conserved -
',' · (b) Skaters begin tci rotate about CM of system
(c) Angular momentum of cylinder a~d .' clay is
.conserved . (c)' There is no l_oss of energy .
(d) Mechanical energy is. conserved . · (d) If skaters pull each other · and reduce their
separation by. half their eriergy is incr:eased
10. A uniform disc of radius R lies in x-y plane with its
centre· at origin. Its moment qf. inertia about the axis 14 .. A wire of length I and mass is first bent'in a circle, m
x = 2R- and y ·= 0 is equal to· the moment of inertia then· in a square and then in an equilateral triangle.
about the axis y· = d and z = 0. Where· d is equal _to: . The .moment of inertia in these three cases about an
axis J)erpendicular to their pla_nes·and passing through
(a)iR . · (b) MR . ,- ·' · their centres of mass are J-1, I 2 & I 3 respectively. Then
3 . . 2 maximum· of them is: ·
(c} MR (<l) ,,ffs R : (a) 11 (b) 12
-' ... 2 (cl I 3 (d) d_ata-'insufficient
l, • .
· 15. A disc of radius ·r rolls on · a I" ·
11.· A unifqrm
radius. -R is·discpivoted
of massabout
Manda· ;: .··:.-.·€1.
horizontal· axis passing through
_its edge. It is released from. ·rest 1
with its ·centre of mass at the
5
·,---:_:_-_:--,. ::-~·.·. horizontal . ground with - !in.ear · , •
acceleration a and angular 1
acceleration a. as shown in figure.
The magnitude of acceleration of .
l
same height as the pivot. point P shown in figure at an ---
(a)· The angular velocity of disc wl_ien its centre of . instant whei;i its angular velocity is.-ro, will be:
I · -ar
. . mas~ i~ dire~y:below :the pivot is_~- .. ' . (a). ·'/(a+ ra.) 2 + (rro 2 ) 2 (b) _
- - R·
(b) The:fo~ce .exerted by ·the' pivot at this in~ta~t is· (c) ..Jrr~2a.~2~+-"---r~2ID-4~ (d) ra.
' . ?.mg· . - •. . 16. A blockwith a square base measuring a x_ a and height
. 3 .
h,: is,·placed on an inclined· plane. The coefficient of
(c) ·Angulat momentum of disc is conserved ·
friction is m. The angle of inclination (0) of the plane is
(d) Angular acceleration of disc at the given: instant is
zer9 ·.1 · ·
gradually increased. the block will:

www.puucho.com
''
.,,
Anurag Mishra Mechanics 1 with www.puucho.com

[~IGID BODY MOTION . 521]


-, I
(a) topple before sliding ifµ > .': (a) Angular momentum L of mass m .ibout pivot is
h
parallel to vector ii\
(b) topple before sliding if µ < .':
. h -, '
(b) Angular momentum L of mass m about pivot is
(c) slide before toppling ifµ > .': 1
never parallel to iii
I
h
(d) slide before toppling if µ < .': (c) Lo = -mr 2 ro sine cosei + mr 2ro sin 2 efc
h '
(d) Angular momentum of particle about Pis parallel
17. A sphere is projected npon a rough inclined plane. The -,
friction coefficient between the solid sphere and the to vector ro
incline isµ. The centre of the sphere is given an initial 20. A projectile is projected with a velocity v O at an angle 0.
upward' velocity at t .= 0 without imparting any initial with the horizontal as shown in figure. The angular
angular velocity. 1Jlen, which of the following momentum of particle about the origin:
f - - - -,
statement (s) is/are true?
(a) Pure rolling will defin!tely begin befo~e the sphere
reaches the highest point and the sphere will
i
l Vo v,
continue to roll purely after that, even while
.. -··
comh1g down. . .
··-------.....
(b) Pure rolling will definitely begin before the sphere -
II,-_
e
__ R'-----'
reaches the highest point but the ! V2 ,·
!_ _ _ _ _ _ - · - - - -
sphere will continue to roll purely after that (even
(a) is zero when particle is at the origil,J.
while coming down) only· if µ is greater- than a s . 2e cos e,. when
certain value. (b) • -mv sm "cl . th
. IS 2g ~ . part1e_1sat e
(c) The sphere will be rolling purely while coming
down.ffµ> (2tana) highest point of trajectory
. 7 • . -2mv 3 sin 2 0cos0.-_ . · .. ·
(c) 1s - - - - - - - ' - ' - K when particle is Just about
(d) Tii.e sphere cannot roll purely while moving up the g. '
incline .. to hit ground
18. A uniform rod of length 1 and mass 2m rests on a . (d) downward force of gravity exerts ~a torque in -z
·smooth horizontal table. A point of mass m moving direction
horizontally at righ~ angle to the rod with velocity v ·21. A woman of mass m stanc\s at the edge of ,a solid
collides witli one end of the rod and sticks to it, then: cylindrical platform of mass Mand radius R. At·t = 0
·,. (a) angular velocity of the system after collision is :'!. the · platform is rotating with negligible friction at
. . . . . l angular· velocity ro 0 about a vertical axis passing-·
(b) angular velocity of the system after collision is .!!.. through the centre. The woman begins to walk with
21 speed v, relative to the platform, towards the centre of-
(c) the loss in kinetic energy of the system as a whole_ the platform: · ,
· . . .- mv 2 (a) ~rlai;;elodty when woman reaches the centre
as a result of the collis10n 1s - -
. 6 IS V +-)roo ..
(d) the loss in kinetic energy of the system· as a whole M . ,
. . . 7mv 2 (b) Angular velocity as function of , time is
as a res uI t o f the co111s1on 1s - - M+m
24 (J)

19. A mass mis attached to 1-·-··- - -2- . : . . . . - - ~ M + 2m(l ~ vt/R) 2


a rigid rod of negligible j (c)· Energy of system i~ conserved
mass · as shown in . (d) Momentum of woman i!)creases in magnitude
. '
figure. The system is 1 22 . A solid cube of side 2a and mass M,. sliding on a
. pivoted at point O and · X
smooth surface with velocity v 0 , collides inelastically
rotates · about the
with the _raised edge of the table:
indicat_ed, z-axis wi~ ;,"-----.... -,-0~..-,._,
~ ro h:r
angular velocity ro, 'L 0
.,·
maintaining a fix ed
angle e with the axis. · ----
www.puucho.com
Anurag Mishra Mechanics 1 with www.puucho.com

\
_.,./:::.,
i 522 _ _,,,,,.....,..,..,...,;.,==--,;,.,,...,....,,,.,,,....,..,..,.,.,..=.;._====..,,,==~==,;,,====:,;,::;====~~~~":"i
- ·-·· . ... . - - - . -- ---- .. --- - . ----. . . ' -' ·" ... MECH(INJCS-1 I

direction. Speed of particle P at a .distance i from the

centre towar'ds A ofth e ro d a ft er rune


· t = -nml.1s:
· 12]
J ' J'
_(a) 2- (b)-
I m Jim
,;
.I !__ Jz!__

dj
C
(c) (d)
i m m
'-------
I
- ---- -- --- 27. A rod AB of length 1 m is placed at the , - · - _ _·B!
2 edge of a smooth table as shown in 1 .
(a) Moment of.mertla . -4Ma
. of cub e ab out e d ge 1s - _figure. It is hit horizontally at point B.)f
3
2
the displacement of centre of mass in 1 s A
(b) Moment of.mertla
. of cubeab out e dge IS
. -BMa
- is s-./2 m. The angular velocity of the rod :__ -·
3 is (g = !Om/s 2 ):
(c) Minimum value of v so that cube falls off the table · (a) 30 rad/s (b) 20 rad/s
is .Jl.19g · (c) 10 ritd/s (d) 5 rad/s
(d) Energy of system is conserved
28. 1\vo cylinders ha"!ng r~dii 2 R and (--·-·7,~--
23. A large spool of rope
stands on the ground_ with
.-,~~':..-!
' R and_ moment of.mert1a's 4 I and I• 101 8-;.
. 2~· ·
Olo

the end of the rope lying


on the_ top edge of the

I about their central axes are I
supported by axles perpendicular .
to their planes. The large cylinder
'l,_ ___ ~--
'
__ - ~ - '
\'
spool. A person grabs the . " - - I is initially rotating clockwise with angular velocity m0 •
end of the rope and walks L
a dista,;ce 1, holding onto
" - ---- - _____ :J The small cylinder is .mo_ved to the right until it
touches the large cylinder and is caused to ,rotate by
it figure. The spool rolls behind the p_erson without the frictional force between the two. Eventually
slipping. What is the length of rope that unwinds from slipping ceases and the two cylinders rotate at
the spool? How far does the spool's CM move? constant rates in opposite directions. D_uring this:
(a} Length of rope that unwinds from the spool is 1 (a) _angular moment of system is conserved
. l
(b) Length of rope that unwinds from the spool - (b) kinetic energy is conserved
. ' 2 (c) neither the angular momentum nor the kinetic
. . l
(c) Spool's CM moves through - energy is conserved
. 2 (d) both the angular momentum and kinetic energy
(d) Spools's CM moves through 1 are conserved
24. A constant power is supplied to a rotating disc. 29. In the above problem 28 the final angular "'.elocity of
Angular velocity ( m) of disc_ yaries with number of the small cylinder is:
rotations. (n) made by the disc as: · (a) mo (b) mo
(a) m ~ nl/3 (b) m ~ n3/2 4,

•~-7
(c) oi~n 213 (d)m~n 2 -(c) mo (d) mo
.-tl'I ·7
nc--_-_ _ ·.· .
25. A force Fis applied on the top.of a cube·as 2 8
shown in figure. The coefficient of friction V'-..; 30. The acceleration a of the plank P f]- a
between the cube and the ground isµ. If F re. quired to keep the centre C of a
is gradually increased, the cube will topple before cylinder in a fixed position during <? ·
sliding if: · · the motion is: (no slipping takes 8
' , .
' 1 place . between cylinder and' , . J
(a) µ > 1 (b) µ < _.
. ' 2 plank)
1 (a) fsin8 (b) 2g sin8
(c) µ > - (d) µ <-1 2
2
26. A uniform rod AB of mass m and length 1is at rest on a
(c) g sin8 (d) Jig sin8
. smooth horizontal surface. A impulse J is applied to
the ~nd B perpendicular to_ .the rod in horizontal

www.puucho.com
Anurag Mishra Mechanics 1 with www.puucho.com

RIGID BODY MOTION 52:f'


31. A rod of mass m and length 1is hinged at one
ofits end A as shown in figure. A force F is
applied at a distance x · from A. The '. F.
,

acceleration of centre of mass· (a) varies with :'


~I~,!
- - - - - - - ---------------~------------·--·---········- J

,_,,_,
XI
Ii
'
!
_ _ .,...j
36. Two men each of mass m stand on the rim of a
horizontal circular disc, diametrically opposite to each
other. The disc has a mass Mand is free to rotate about
a vertical axis passing through its centre of mass. Each
x as: man start simultaneously along the rim clockwise and
1a··-----,
()a 11········· i
I

!.. ••••uu•••>x'
I
(b): k '
1··-----1
'i .. ···········>X''
reaches their original starting points on the disc. The
angle turned through by the disc with respect to the
ground (in radian) is:
(a) Smit (b) 2mrr

(c),V .
j '" ------- _.J . •• -.--- - _,__ - ..I

ri-----: ,.~--,
' '
4m+M
(c).....".1::... (d)
4m+M
4mrr
Cd): I M+m 2M+m
'
,j••________ _•·•···•·•>XI
' I .
1-..:.~-~~-~-:.-~·t
'

37. A time varying force .F = 2t is


32. A uniform rod oflength 1is pivoted at point A. r·-· A'1
It is struck by a horizontal force which
delivers an impulse J at a distance x from , x. i
Icr· : applied on a spool as shown in
figure. The angular' momentum of
the spo_ol at time t about
point A as shown in figure, impulse delivered , 1
bottommost point is:
by pivot is zero if x is equal to: ,_J __..l
r2t2 2
l l (a)~ (b) (R.+ r) t2
(a) ~ (b) - R r
2 3
(c) (R + r)t 2 (d) data is insufficient

--j]
(c) 21 (d) 31
3 4 38. A spherical body of radius R rolls on a '@
· ·
·
33. In the figure shown masi of both, · ·- - -. horizontal surface with linear velocity v. : ,;
the spherical body and block ism. j ~ - , Let L1 and L 2 be the magnitudes of angular ( . • _ _!
Moment of inertia of the spherical I momenta of the body about centre of mass 1•. - ~ •. .1
body about centre of mass is 2mR 2 • I _____ . ___ _ and point of contact P. Then: ·
The spherical body rolls on the (a) L 2 = 2L1 if radius of gyration K ~ R
horizontal surface. There is no slipping at any surfaces (b) L2 = 2L1 for all cases
in contact. The r11tio cif kinetic energy of the spherical (c) L 2 > 2L1 if radi_us of gyration· K < R
body to that of block is: (d) L2 > 2L1 if radius of gyration K > R
3
(a) - (b) .!. 39. Atli.inuniformrodABofmass n=,·1-··· ·-·A---··· ·
4 . 3 1 kg move translationally with i , ;
(c) -
2
(d) .!. 2
acceleration a= 2m/s due to.! -t.F2 =sN'
3 2 two antiparallel force as , !
34. A particle ,is projected with velocity v at an angle of 8 shown. If l = 20 cm then: , .. ___ B .. . :
with horizontal. The average angular velocity of the
particle from the point of projection to impact equals: (a) F1 = 3N
. (a) g case (b) _g_ (b) F1 = SN
ev vsin8 (c) Length of rod is 1 m
(c) .!.. (d) ~ (d) Length. of rod is 80 cm
ve v~ne 40. In above question which of the following is/are true:
35., In the figure shown,.the plank :
1 (a) torque about any point on rod may be zero
is being pulled to the right
with a constant speed v. If the
:1' (b) torque about centre of mass will be zero
cylinder does not slip then: ·" I (c) acceleration of end A and B are same
(d) acceleration of end A and B will vary with time
(a) the speed of the centre of
mass of the cylinder is 2v ;._ · -·· ·= 41. The torque ~ on a body about a given point is found to
-)-) -) -)
(b) the speed of the centre of mass of the cylinder is be equal to A x L where A is a constant vector, and L
zero is the angular momentum of the body about that
(c) the angular velocity of the cylinder is~ point.: From this it ·follows that:
R
(d) the angular velocity of the cylinder is zero

www.puucho.com
Anurag Mishra Mechanics 1 with www.puucho.com

'.\1 -~
I '524
L-~·_ _ec....:..___ _ _ _.....;,
•. i"-"'1
>""
,~.0e'·--'---"--"-'
_:~, !" , ,
" - - - ~ - ' - ' •.. . .·· •...

(a) di/dt is perpendicular to Lat all instants ~f time: 43. · A uniform cylinder of mass M and radius R rolls
. _, _, without slipping down a slope of angle 8 with
(b) the component of Lin the direction of A does not horizontal. The cylinder is -connected to a spring 'of
change with time. · force constant k af the centre, the other side of which ·
_,
(c) the magnitude of L does not change with time. is connected to a fixed support at A. The cylinder is
_, released when the spring is unstretched. The· force of.
(d) L does not change with time. · fiction (JJ: . . · I
42. A ball moves over a fixed track as shown in the figure.
. ·From A to B the ba.J.J. rolls without slipping; If surface B
is frictionless and KA, K 8 and Kc are kinetic energies
of the ball at A; B and C respectively, then: · -~
IN~:.l
~ - _ ,..,.~

t.· 10he/
0
[>ct·
[ti;; - ~ ···:
A
· .\ . .~ [ : ' : (a) is always upwards '
(b) is always downwards_ ' .
(_ :u~,~~m~r . . (c} is initially upwards and then becomes downwards
(d) is initially upwards and then becomes zero
(a) h,, > h6KB Kc > (b) hA > hc;Kc >'KA
(c) hA = hc;;K8 _ = Kc (d) hA_ < h6Ka, > Kc

.., ' '

www.puucho.com
Anurag Mishra Mechanics 1 with www.puucho.com

. [ RIGIDBODYMOTION- -- --------- . 525

Comprehension Based Problems

, P:A SS~ G·E


1. PASS.A GE
A solid cylinder of mass m rolls on a rough surface A uniform rod mass m and length L is free to rotate
with velocity v 0 • It collides elastically with a cubical' about hinge 0. A slight disturbance causes the rod to
block of same mass at rest. The centre of mass of both' rotate freely about O and it strikes the ground.
the bodies are at same height. Coefficient of friction
between horizontal surface and cylinder as well as
horizontal surface and cube is µ. No frictional exists
between cylinder and cube. The cylinder collides the,
cube at t = 0, then.
·_1
~D Vo···· ..

1111111 I

1. Percentage of energy of cylinder lost due to collision is:


1. Velocity with which the non-hinged end of rod strikes
the surface is:
(a) )2gL
(c) )6gL
(bl )3gL
(dlfgi,
2. Horizontal force applied on rod by hinge just before
(a) zero (b) 50%
the rod hits the surface is:
(c) 66.67% (d) 33.33%
2. Regarding frictional force acting on cylinder we can (al mg (bl mg
4
say that:
(c) 3mg (dl 3mg
(a) before collision friction acts forward 2
4
(b) before collision friction does not act
3. Vertical force applied by hinge on rod at the moment it
(c) after collision friction acts backwards strikes the ground is:
(d) after collision friction does not act
(al mg (bl mg
3. Time after which the cylinder starts pure rolling again: 4
(a) 2vo (b) ~ (cl 3mg (dl 3mg
3µg µg 4 2
(c) 2vo (d) ~
µg 3µg
4, Velocity of cube when the cylinder starts pure rolling:
PASSAGE
2
A spool of mass'm has moment of inertia I= 2mR
(a)~ (b) ~ ,

2 3 about its axis of symmetry. The inner and outer radius


of spool R and 2R respectively. Thread is wounded on
(c) 2v 0 (d) ~
3 4 the inner cylinder and its one end is pulled a force F
5. Velocity of cylinder with which it will collide with cube acting at angle 0 with horizontal. The surface is
sufficiently rough and th.e spool never slides on the
again is:
surface .
(a)~ (b) Vo
2 3
(c) 2vo (d) Vo
3 4
6. Maximum separation between cylinder and cube
before it collide the cube again is:
2 2
(a)~ (bl~ 1. For 0 = 0°, the acceleration of spool is:
µg 2µg
2 (al !_ (bl .£..
(cl~ (dl zero m 2m
4µg (cl .£.. (dl .£..
6m 3m

www.puucho.com
Anurag Mishra Mechanics 1 with www.puucho.com

2. For 0 ~ 60°, the acceleration of spool is: 2. Acceleration of cylinder is:


ca)!_ Cb) L (a) g (b) !
m 6m 2
(c) L (d) zero (c) ! (d)· 3g
2m 4· 4
3. Regarding to the direction of frictional force, which of 3. Distance moved by cylinder during time taken by it to
the following statement is correct? complete one rotation is:
(a) For 0 $ 0 $ 90°, it always acts leftwards · (a) 27tR (b) 7tR
(b) For 0 $ 9 $ 60°, it acts leftwards and for ·
(c) 37tR (d} 41tR :
60°,;; 0 ,;; 90°, it acts rightwards 3 '
(c) it becomes zero at certain angle 0between 0 to90°
4. Distance moved by hanging mass during the above
(d) none time· interval is: · ·
4. Regarding to the direction of rotation of spool which (a) 27tR (b} 7tR
of the following statement· is true?
(c) 3~ (d) 4ltR .
(a) spool rotates clockwise 0 < 0 < 90° 3
(b) spool rotates anticlockwise 0< 9 < 90° 5. Velocity of point of contact 'of cylinder at this moment
(c) spool rotates clockwise for 0 $ 9 < 60° and is:
anticlockwise for 60° < 9 $ 90°. '
(d} spool rotates anticlockwise for 0 $ 9 < 60° and (a) zero ~) ~!tRg leftwards
2 .
clockwise for 60° < 9 $ 90°
5. Maximum frictional force acting on the spool is: (c) ~~·rightwards (d) .jitRg
(a) -
F
(b)
2F
-
2 3 6: Velocity of hanging block at this moment is:
(c) SF (d) .!:_ (a) ..j27tRg (b) ~
6 . 6
(C) ~!tRg. (d) ..j4.57tRg
6. Different possible ·angular positions. of the force are 2 .
shown in the figure then: ' · . ;" ,c,:.,
-t -~3-:"''7'!
• A .2.• '? r1 J-~1 ,ir; 1 .-r { 5 ·
' : :(';1
P,AS.SA,G;E
•·-1 ""'"" •• ,,,, %1Jf'
!( ·,_
_,. """' ,.,. '

a
A sph~re, a rin'g and disc i>fsame mass *ncl\,iJiµ~
~;l are. aifQwed . tci roll do\\lh three similar sufficie11tly
..c:..l rough i11cHned pl:mes ··as s~bwn in the figure ,ft:cim

L~~~I ~J
(a) for positio°' 1, spool will rotate clockwise
(b) for position 2, spool will hot rotate
(c) for position 3, spool will rota.te anticlockwise
(d) all of the above

i' ,-... - -,· •• ,-,: \,;,~·' •• ,, 1: Which of following order is true for final KE cif the
;; P"A SS'A' GE· l•;t
J ~~:, S:,..,i . 4r.,<J,.i
'\·
\,:,,
bodies? ·
.A string, wrapped around a •cylinder of mass m 'ah4 (a) sphere > disc> ring
(b) ring > disc > sphere
radius R. ifhe end of the st;J:ingJs connected to \)jgck
of same; mass hanging vertically. No frictiQn exists (c) disc > ring > sphere
between the J,grizontal.swfl!ce_;md cylinder.. (d) disc = ring = sphere
2. Which of the following order is true for final linear
.. 1 i2nmmnmnmn)4 , velocity of the bodies?
6 I .
(a) sphere > disc > ring
"--·Acceleration
1.
-·---'~======---..
of hanging mass is:
:..J (b) ring > disc > sphere
(c) disc > ring > sphere
(d) disc = ring = sphe,e
(a) g (b) !
2 3. Which of the following order is' true for tiine taken by
(c) ! (d) 3g the bodies to reach the bottom of incline?
4 4
www.puucho.com
Anurag Mishra Mechanics 1 with www.puucho.com

fyiGID BODY!vlOT!O_N~._ _ _ _ _ _ _ _ _ _ _ _ _ _
··· _···-----··---·---~--~~:__________ _5271

(a) sphere > disc > ring


(b) ring > disc > sphere MATCHING
- . TYPE..PROBLEMS
:C- - -- . - - ~

(c) disc > ring > sphere


(d) disc = ring = sphere -Match the statements, situations in Column I with that in
4, Which of the following order is true for frjctional force · Column II. One or more matching is possible.
acting on the bodies during their rolling? Ut/" Column I Column II
(a) sphere > disc > ring (A) (P) :Net work done by
(b) ring > disc > sphere friction force is
(c) disc > ring > sphere negative.
(d) disc = ring = sphere
r- ---. --------- . -
i ,, • -.. ·.. '" 5·-··:~~~~~~ I 1
A spinning ball lowered on a:
'~.ASSAGE '.: '.) ' '_;~~i),J - I rough surface ·
Two discs Aand Bare mounted coaxially on a vertical I (B) , (Q) Rotational work done
by friction is
axle. The discs have moments of inertia I and 21:1 ·negative.
respectively about the common axis. · Disc A is
imparted an initial angular velocity 2ro using the
entire potential energy of a spring.compressed by a
distance .x1 • Disc B is imparted an angular velocity co ,A ball is-projected with velocity'
. by a spring having the same spring constant and iv 0 on a rough surface
I compressed by a distance x 2 • Both the discs rotate in 1 (C)
' -
· (R) Work done by friction
L__ the tlockwise direction. __________________ _, ·is positive.
1. The ratio xi(x 2 is:
(a) 2 (b) .!
2
'A ball projected ;;th -velocity:
(c) -./2 (d) 2.. :v0 =Rco 0, projected on a
-./2 :smooth horizontal surface.
2. When disc B is brought in contact with disc A, they
acquire a common angular velocity in time t. the
(D) I' i (S) :Net work done by
l ! friction is zero.
average frictional torque on one disc by the other F
during this period is:
l.''
cos 8::: r/R·'
2Ico (b) 9Ico :
.. ( a ) - I I
3t 2t
,I
(c) 9Ico (d) 3Ico '
4t 2t ,A spool pulled by a force !{
,rolling without slipping on a,
3. The loss ofkinetic energy during the above process is: ,rou h surface.

(a) Ico
2
(b) [(03 ".(.:;;f:< !"">' .: ,;_;._-- --.:--·. c·o'·1u·:;mt:>n{·&:1;'¥1ii)r;,,_ 1
-- ·1!:_\,1t~L
2 3
(A) Solid sphere rolling with (P) ·Total kinetic energy is
(c) Ico3. (d) Ico3 slipping on a rough conserved.
1

4 6 hotizontal surface
1
4, A small object of uniform density rolls up a curved (B) ;solid sphere in pure: (Q) ;Angular momentum about
surface with an initial velocity v. It reaches upto a !tolling on a rough'. 1CM is· conserved.
3 2 ',horizontal surface :'
maximum height of v with respect to the initial
4g (C) ·Solid sphere in pure (R) Angular momentum about
'rolling on a smooth la contact point on contact
position. The object is:
:horizontal surface ' :surface is conserved.
(a) ring (b) solid sphere
(c) hollow sph~re (d) disc (D)_ ISolid sphere in rolling on l(S) ':Moment is conserved
'a rough incline surface f
I
'(T) ,Total Mechanical energy is
!
:Conserved

www.puucho.com
Anurag Mishra Mechanics 1 with www.puucho.com

l'---·
528-
_------ -- -- -- -- ----- ---~~-==---·---;:::-==··:..::·--::::·:.::-====:..::.:..:===--=:..::·-=-:.::·=-·=111=·t=CH=A=Nl:.::CS=-i:::.:j
~3_._ _ _ c.:0:_'.:/'-'~~j;~~,:~gJu~ri'~i-:~~~\~I~1t~~~,~~I~ Gtfniri~:iJI~~;:;
1
(C) (R) Energy is conserved.
(A) ..--s.:----- F (P) Body accelerates
forward.
: . i
!A small particle of mass m:
,strikes elastically, at end of a1
'horizontal rod keJ}t on a 1
Ring
,smooth surface.
(B) · (Q) Rotation about center (D) · (S) Momentum increases.
; :of mass is clockwise.
2U3
·.b· CM
m

A small particle of mass m


(C) : (R) iFriction force aets . I
,coUides with a vertical rod
_ _,,_ _..F : backward. :elastically.

5. Four different bodies of circular cross-section of same


mass and radius R are situated at rest at the top of a
Solid cylinder rough inclined plane of height h. At t = 0, they all
begin to roll without slipping wooden.
(D) ; (S) ,No friction acts.

• (A) .Wooden solid cylinder ;(P) I<Erot is maximum at


bottom.
(B) Wooden iron solid cylinder (Q) KEtrans is minimum at
Solid sphere
bottom.
(C) Thin iron cylindrical shell (R) Takes minimum time
to reach bottom.
'· ; ._._,··.,t:hi-\'4ifi':-,, , :.,_ -<J.?5:>://fj~,~"~F-. izv:, _7 -~-l
e--:.4=---~-2:j:.,"'f-,lL-'.hfi;'. C:Olumn _ ·. ;+~t;tJ~~t;C.O.l9ni"Jl,tSst:t
(A) (P) ;Momentum is (D) Thin wooden cylindrical shell (S) Takes maximum time
·f
conserved . to reach bottom.

. h=2/SR •

(A) (P) On a frictionless surface


An impulsive force acts at a,
,rolling is toward left.
height h =2 on a solid'
. SR
sphere, lying on a rough, .~
'surface. ' .

(B) , (Q) Angular momentum is (B) F (Q) On a rough surface friction


,conserved. acts toward right.

A small particle of mass m


(C) · (R) Friction force acts towards
moving horizontally collides:
with verdca1 ·hinged rod,· 'left.
inelasticall · e = 0. 1

www.puucho.com
Anurag Mishra Mechanics 1 with www.puucho.com

_RI_Gi_o__eo_o_f_M_or_io__~----~-----=--------=---------_--_--_-_-_--_-~_-_-_-------;===--::-----==c-:..:--=·===_::;::.__:;::..:=--==:.:..:..:==-=-5=--2:::9~]
~f
(D) , (S) Direction of friction force (C) (R) :Angular momentum
'. cannot be determined. increases .

. I

7, In column various bodies of same mass and radius R


'.
i
are being lowered on a rough horizontal surface. (D) (S) •Llnear momentum
!increases.
/>t(C,~1~·riJi{I ?}~--~Ji~\.~-~-,
(A) (P) Rotational work done by

.Ql· Ring
'
,friction is negative till pure
rolling beings.

(A) 'Conseivation of
(B) '(Q) 1Translational work done is
[positive till. ;angular momentum
-'' ~. i''
. '

' '
''
Cylinder ' '

(C) (R) 'When pure rolling beings

.02.
velocity of centre of niass A small particle of mass m is'
given an initial velocity in,
;is minimum.
·horizontal plane and winds. its 1
·cord aro_und the fixed vertical 1
Solid sphere , ,shaft of radius a.
(B) -~ (Q) 'Conservation of
(D) (S) 'Takes maximum time for kinetic energy.
pure rolling to begin. ··, D
:A smooth rod rotates with 1
Hollow sphere
angular velocity ro,. A small;
•sleeve starts slidjng along the:
:rod. I
(C) :Two iceskaters approach· each: (R) ,Conservation of
other at equal speeds along total mechanical
(A) i (P) !Translation occurs. ,parallel path separated by some energy.
MjT distance. They link hands as they
:pass by and pull each other to,

~
M 1 reduce their se·paration. ·

(D) Work done by


(impulse)
Rod is massless, dumb-bell is
placed on a smooth horizontal_;
~·;"' internal forces ..

surface. F ,
J,
(B) Rotation occurs.
1
•A small body _tied to a non i
' •stretchable thread mass over a,
1smooth horizontal plane. Other:

.end of thread is be"ing drawn into,


a hole 0. '

www.puucho.com
Anurag Mishra Mechanics 1 with www.puucho.com

10. Each object shown_ in column I has mass 2m. The ring
has mass m, and radius R: -The other components rod,
lamina have total mass m. · The ·shaded part in_ any
figure represent a.lamina axis I and_Il are iri plane of
figure, moment of inertia about their axis is ·.
represented by I 1 , I 2 . _Moment of inei:tla about axis
through O and perpendicular to plane· of figure· is Fnctiqn ·
· given by I O• --_ • J!!.11!• · right. '

(A)

At:~el~futi~n
of mass is ,..
right:.

,, .

www.puucho.com
Anurag Mishra Mechanics 1 with www.puucho.com

I ., '
1. I (d) ' 2. I (b)· l· 3. j (bl i 4. II (af. j · 5. ! (a) I 6. I
'1·
(b) ! 7. ! . {d) i
) 8. I· ,(b)
9. I (c) . ! 10. 'I (d) I 11.
I
I. (a) I 12. ! .(6). 'l 13. .. (c)
I
I 14. (d) j
'
15. t (b) I 16.· i ' (<l),:
17. I '(a)
'
I
! 18. I (a) !
19.
i
I .
,"
(a) ! ·20. 'I (c) i 21. (b) i 22. I (b)
'' 23. I (b) I 24 .,I (a) .
/'
I

25. ' (c) I 26. I (ti) 27 ... I (d) i 28. I (bj I 29. i <1>>
1
1 30. '
I (c) 31. I (d) I' 32. I (c)
' '
33.
'
(c) I
I . 34. !' (a) l 35. f ,(b) l 36. ! .(cl ·I 37. I (b) 1 38. I
I (a) I 39. I - (bl I
I 40. i (cj
i 'I ! '
I (b) i . (d) I I I I (d) j
'f"'
41. I {b) 42. 43. (d) 44. 45. (d) I 46. f (c) 47, 48. (a),
49. I (b) ;
I ! 50.
I
I (c)
'I'
51. I (a) i 52.
I .··.
i. (bJ I 53. r ·(b> I
I 54. I (a) I 55 ..
,.
I (a)
j
56. (a)
57. I (b) ,.
'
i 58. t (d)· i 59. t (a) ! 60.
I
I ·(c)' I.,l 61. I (bj I
62. I' (d) i 63. II (a) I 64. !' ·M,
65. I (a) I 66. !' (c) 'i 67. I '{c) ·1 68. I (a)' i . 69. i' "(d) ' 70 .
''
I
,, (a) i 71. '
'
'' (a) ! 72. I
I ·(cl .
I .l I .1 I ' i · (c)
1
73. (b) -74. (d) 75. ~. (b) 76. I (li) l 77. I (c) ~1 78. I (d) I 79. (a) ! 80. ,
81. I (d) I 82. i '(b) ' l 83 · i (b) I 84. I (b) .· 'l 85. I (d) I 86. I (b) I ·87.
' l (b) 'I ;• I
. .
' '

,. I.. . . '
1. I (a) i 2. ti"
1(a,b, d) I 3.
l
I (a,,c) I 4. I (b, dl I 5. I (b,d) !I 6. I (b,c) i 7. , (!:, d) I 8. i (b,c) -f', '

9. 1 (a, c) 110.
,,
rr . <l:i> i 11. r (a, c) L12. 'I (b) I
I 13. I,(a, l:l, d) :1 14. I . <a> I' 15. I (a) i 16. I (a, ciy
17. I (b,,c) ] 18. , (a, c) I 19. l<b, c; B> I ·20. f(a,b,c, d) I 21.
,,!(a; b, d) 22. I ,(b; c) l 23. I (b, c> l 24. I (al-,
25. ;: (c) .1 26, f.. (d) •. 1 27. II (a) SI 28 .. I (c) II 29. I (b) 30. I (b)
I
I 31. I.
'
(b) . !
'
32. l .,(c) •.
I 1 (c}. ,·1
'

33. ;
I
(c) · I 34. I :.(dl I
I 35.
I
36. ! (a) j 37. 'I (c)' 38. I, (a, ctJ I
39. I. (a, c) 1 I
40. I . (a)', .''
cJ! 42. 1•.(a: b) 'I I
,,
' <61 I ! I
41. i(a, b, I 43. 1 [ I !~ I I ! ,''
=~:,·cir.;';l';;n_sirr..sc1~~j~r9blem~~
.._, ·-=--0t"'twW11#0r ~w-1tt1tlt!W'tr·~:'it::J::i:¥µ;f 1 z

Passage-1:
1. (d) 2. (a) 3.' (d) 4. (c) 5. (b) · 6. (c)
Passage- 2:
1. (b) 2. (d) 3. (b)

Passage - 3:
1. (d) . 2. (d) 3 .. (a) 4. (c) 5. (b) 6. (d)
Passage-4:
1. (d) 2. (c) 3. (b) 4. (c) 5. (b) 6. (d)
Passage- 5:
1. (d) 2. (a) 3. (d) 4. (b)
Passage - 6:,
1. (c) 2. (a) _3. Cb), 4. (d)

www.puucho.com
Anurag Mishra Mechanics 1 with www.puucho.com

... ,- -~
, ,. ~

=~!~.
l[~~~,~;~h!:~,-'. -:~. : : :·:''.)~illif~~FM;~:~~i~-~i!~Pc~~~~:~;-:.,~~~J:~{f~~~nJ-.c't._·_,_...:.....=c.c.,ttLi:.,.:.'-'"_:.___.c.'.-:c.;;\__:i.c.;''.t(i;c;,~,~-Njsi[] .·.

o· ~
1. (A)~ P, Q, R;(B)-> P, (C) -> S, (D)-> S.
'f ., : ' -
2. (A) _:.,. ., (~) --t: P, Q, R, S, T, (C) -> P, Q, R, S, T, (D) -> T.
. 3. (A).:+ P,, Q, S, (Bj_; P, Q, R, (~) -t P,·Q, S, (D) -tP, Q, R. .
- . ;-:.,} '.. .,. ' . : '·. : . .
4, (A) -'+ S, {~) -> Q, S, (C) -> P, Q, R, (D) -> P, Q, R.
5, (A)-. R, (B) ~ R, (C) -> P, Q, R, (D) -> P, Q, R.
6. (Ai-.. P, Q, (B) .-> P, R, (C}-> P, S, (D)-> R )
7, (A)~# Q, S, (B)-> P, Q, (C) ~ P, Q, R: (D)-> P, Q. (·
8. (A) --+ r, Q; R, S, (B) -4 P, _s, (C) -> P, Q, R, S,(D) --+Q, R, S.
9. (A) .:.; Q; R, (II) -> P, Q, R, (C) -> P, S, (D) ~ P, S. ·
10. . (A) -'+ P,: Q, ·(B) -> P, Q, (C) -~ P, R, (D) -> P, R,
.
s; s:
11.• {A) --+ P, R, (B) --+ Q, R; (C) ~ R,_(D) -> Q, R.
12. (A) --+ P, Q; (B).-> Q, R; (C) -> P, Q, ·(D) -> P, Q.

., .,: .

. ,

_.,
I'

. ,. d;,.

www.puucho.com
~-··.
;-4..u_:s __ .1· .. In ..i. -{. ,'. L TI ;t,}!t;ZAU.i
j .
Anurag Mishra Mechanics 1 with www.puucho.com

=a:~~!.} £[~IV One Altern;.'iii,e is Correct~


4. (a)_ Io=--
Ml3
3
1. (d) '
Let after collision velocity of rod be V and angular
I p -=--+
Mz2 M';;2
y + cz -x'
· J2J
velocity be ro, then 12 . . _
Ml2 for I 0 =I,
mv 0 = mv + MV => mv 0 x = mvx +--ro 2
12
2 y 2 +(1-x) i = Ml-
Mrori x . 4-
mv 0 =mv+-~-
- 12 So the path is circle.
From above equations 5. (a)
2
Mrori x 2
MV => rori x = 12 V 2I = 2Mr 2 => I = Mr 2
12 6. (b)
velocity of farthe; end of the rod = V - lro tari8
For ring to roll down µmin = R1
2
l+-·
= i:x _ 7(~-l) ro: =
So for
. K2.
tan8
µ<-2--
If this velocity is opposite to v O then,
.:!l-1< 0=>1'] < 6 So ring will slide and a= g sine -111 co1i
6 7. (d)
2. (b) For 't=·o
By conservation of angular momentum F1 (L - x) = F2x
Mr 2ro = (Mr 2 + 2mr 2 )ro'
Mro
ro'=---
F2 (3t-x) =F1 ( x+~)
M+2m L-x X
=---'---
3. (b) x + (L/4) (3L/4)- x
-dro
-= - dro
Kro = -ro-- 3L 3x
-=x+-+-= .x _ 2x·
~ 3L-•.
• IX
dt de 4 4 4

Jroo"' dro = Jof "" Kd0


012 2
So putting the value of x _
SL 3L · .
roo roo F1 - =F2 - =>F1 :F2 = 3: S
-=K(21tn) =>K = - 8 8
2 · 41tn
8. (b)
NJ 0
ro 0 /2
- dro = Jore Kd0 Angular momentum = mvd
9. (c)
000
= K0 =>-0 = 000 = 2nn No external torque acts on the man iO his angular
2 2K
momentum will remains conserved. r
So it will complete n more rotations.

www.puucho.com
Anurag Mishra Mechanics 1 with www.puucho.com

.,
ti34 · '_" M~~lffl,~ICS-1 I
10.. (d) 20. (c)
·For rotational equilibrium .t = O Velocity of particle = 2v ·
mgLsin60°;,,Mg!'.sin30°~ M ="3 .· ~2x~
-. 2 . m Time taken to- reach 'the ground = - - -
. g
11. (a)
2 .· · 1
-MR 2 = -Mr 2 +Mr 2
5 2 . '
' . 2 21.
So·

(b)
. tr l
AB= -x2v=4v -
' g . g
zMR·. 2 _: 3M 2
or
- -- r · r=--R·
'5 2 '.M. Since velocity of top end is twice the center so it will
12. (b) L = mvx travel twice the distance traveled by the centre.
13. (c) 22. (b)
-+ -+ .. -+ -+ Farther the mass from axis, great_er will be the
We know·
.' · Lp =.LCM+·rx·PCM moment of inertia:
14. · (d) 23. (b).
' ,,
Since net force aiong the incline is zero, so· cylinder· Since all the forces passes through point of. contact, so
will remains in position till it stops rotating. After that angular momentum remain~ conserved.
, it will start moving downwards. 24. (;i) .,
16, (d) .. ' As shown in. the figure
. . ML2 ML2 Ico =Ic:o·
1=2X--=--
. 12 6 So, 2Ico =I =}!co =I/2.
17. (d) ·. Also 2IAB =I ~IAB =l/2
' • : 2
F
0
=f
1dmxro 2
·
1
=f·
~(xco 2 ) mlro
0·/ 2
= So
25. (c)
Ico =IAB

18 .. (a) · g sine
a=-.--
. The pseudo force ma downwards will .create a torque . K'
. due · to which angular . momentum will always 1+-'
. R2
increase. · . . ·K 2 2
19. (a) For any sphere. - = -
R2 5
The·. J:llock would start L

a~~~g~ :r'·. .: -~·:@:


so acceleration is independent of mass an_d radius.
. p~_:o~,.ans~t-nO: :~ 26. ,Cb)
.;111,nnn111rn1v~p1r.~ ' L = mvR = 5 X 3,/2 X 4sin45°= 60
Since 11~ external torque . ~,_.~ . -· · :~~ ~.,;.,.;, · 27. (d)
acts on'the block, its angular niomentum is conserved. When, sphere rolls over the plank no frictional force
"1ngular momentum of the bJock before hitting the acts between them.
ridge = mv(a/2) . 28. (b)
1'ngular momentum of the blocl!: after hitting the . For A and B energy will remains conserved but energy
ridge= I 0 ro of C will 'dissipate due to action,_of frictional fore~.
The, moment of inertia, I 0, o(i:he block about the axis 29. (b) · . .
passing through the point O is : -+ -+ ' -+
a=axr=O
10 =le +Mr 2
for particles on diameter (axis)
Ma 2 (a' :· a 2 ). Ma 2 Ma 2 30. (c)
=--+M - + - = - - + - -
6 4'' 4 · 6. · ,)
2
JFdt h = loo .
=~Ma'
3 · JFdt=mv
Hence, the conservation of angular momentum gives . · From above ~elations h = ~R
· (·a)· = (2-ma ')ro·or,co.=
mv ~
\: · -3v ·
. . ,. 5
3
2 3 ' 4a . Height from grou~d = R, ·
3

... , www.puucho.com
Anurag Mishra Mechanics 1 with www.puucho.com

. [RfGID BODYMOjlOlt 'J, _,


·,
31. (d) 42. (b)
-,
VPQ =Vp-VQ = -, -, (8..J3 ~ 8 ·) - (&ho
--i--J_ 6 ·)
--1+-J
By conservation of angular momentum about point of
- 2 2 2· · 2 contact: .
V Vo . Vo
=..J3i-.ffJ mv 0 r = 2Mr 2 x- =}V = - =}ffi = -
r 2 · 2r
-+ -+ -+ A 'r;::; A A

L = rxP = (-10 i) x m(-v3 i:-7J) = 70mfi:. 43. (d)


m10 2ro = m x 70 =} ro = 0.7 rad/s _ Acceleration of sphere = µ g (right wards)
33. (c) The sphere will start sliding. Acceleration of plank = µ g (leftwards)
34. (a) So relative acceleration = 2µ g
We can assemble three similar 44. (d)'
triangular plates to form a We know le= mr 2 /2
square. The moment of inertia Also. le =ICl,I +mx 2
for complete square . ·
ICl,I =Ie-mx 2 = mr 2 /2-mx 2
41 = (4M)a2 ·=}I= Ma2 45. (d)
6 6 . • 'i ' •

35. (b)
By conservation of angular momentum
2 2
Both the sphere and the plank will slide down with ·m
--0R m0 R
( J ) = . [ --+µtR
2] - m-
ro = } t = 0

same acceleration g sin 0 . 2 .2 2 2µ


2 2
.36. (c) . M/ M12
I=--+--+ --+M - (M/2 ) ._ (..f3z) 46. (c)
3 3 12 2 KE of P = .!.m(2v) 2 = 2mv 2
2
= ~M/2
' 2 ' KE~fS = .!.mv~+.!.Iro 2 ;,~mv 2 =} KEofP 8
2 2 4 KEofS 3
3 2 =}K=-
So_· 3MK 2 =-Ml 1 47. (d)
2 .Jz The moment ofinertia a_bout an axis passing
38. (a) through the centre is
Since for a = g tan_0, no force acts along the incline, so
it will continue its pure rolling.
39 . . (b)
I'= .!_MR2
2
. The moment of inertia
- a·
o• ,,_•.
No horizontal force acts on the rod, so center of mass about the XX' axis is
___ ..

rrI~~~·
-_ ~l
,
- remain at same point; · I= .!_MR 2 +MR 2
40. (c) Mg-T=MA 2
L ML2 =~MR2.
Mg-=-cx
2 3 . 2 .
' 3
cxL = 3g ,T-~ =~(pL)R 2 =~(pL)(L/211:J'= ~pL2
2, I - mg . - 2 2 . .8 1t
L 48. (a)
Also .a =cx-
2 MgsinB-f=Ma
Solving we get JR = 1 !!:. =}a= · g sin0
T=Mg R ··(1 + I- -)
4 . MR2
41. (b) 49. (b)
No torque · acts on the system so the: angular
By conservation of energy
momentum will remains conserved: ·Aiso if we 2
consider the total energy (KE and PE)· no external mgl = .!_ m1 ro2 =} ro = /3g
force does any work, so total energy Will remains · 2
.
2 3
. '
, 1/T
conserved. Velocity of uppe~ end = ..j3ii.·
,

www.puucho.com
Anurag Mishra Mechanics 1 with www.puucho.com

( · ~r •
._y5~3_6_'_'.·_.·""i.e.',:_·_·"----.c-.•cr.Z~----·---~-~=·'::c,J .__ _ _ :....;_r~__ , _ _·._M~E~.l!ANICS~I i
50 1

2)(M! }ro 2

' ~~!queduetoFaboutOis,: 1 =FL F· . -.-.-.] (MV)(~)=c ro=f

mg· about 56 _ (a) In =.,,r p,re rollfug hooom ~


2
Torque o(fLw)eight .,.·-.·.··.i.·
'tz = mg
2 . O is ~ '
· .
I most point is the instantaneous axis .
P
..

l of p1.1re rotation about which the . o; ·r l


The minimum force required to __ ,:mg'_· .J whole body rotates · with same \:/. I
1
topple the block is obtained when angular velocity. So farther the point ~ ___.__0__ j
-c1 slightly exceeds 't 2 , i.e., in the limit from O larger will be the velocity.
F · L=
mm
mg(!:.)
2
or F .
mm
= mg
2
57
' (b)
By conservation of angular momentum (fro =
51. (a) cons.tant) angular velocity will become half.
The result is independent of angle 0. Since K = !rro 2 , the rotational kinetic energy will
2
52. (b)
become half.
mg sine component is always down the plane whether 58. (d)
it is rolling up or rolling down. Therefore, for no
slipping, sense cif angular acceleration should also be Since sense of rotation is not changing so direction of
same in the both the cases. Therefore, force of friction angular momentum will remains same.
f always act:upwards. 59- (a)
53 .. (b) Moment . of inertia of remaining disc
2 2
Since, there is no external torque, angular momentum 9~R ~( M(R:3) +M(z:r)
will remains conserved. The moment of inertia will
first decrease till the tortoise. moves from P to S and 2 2 2
then increase as it moves from S and Q. Therefore ro
wili initially increase and then decrease.
= 9MR
2
-(MR + 4MR
18 9.
J
let R be the radius of platform m the mass of tortoise = 9MR2 - MR2 = 4MR2
and M is th.e mass of platform. 2 2
. moment of inertia when the tortoise is at p 60. (c)
. I mR2 .MR2
,=+--
2
. The force must be applied on the centre of mass of the
centre of mass of the system.
and moment of inertia when the
tortoise is at S

·here
_ 2
I 2 -mr +--
MR 2
2

r 2 =a 2 +[~R 2 -a 2 -·vt]
·2
61. (b)

~t;~ e::r:::1
angular
~~:::,ed.
::qu:J:,
momentum of
remams

r9-·-·
II
.
,
0)

M
cp j
L_ ·#·---·~-· __ : _ ~ ___ _
M

from conservation of angular momentum ... I 1ro=l 2 ro


·· · ro 0 (2m:+-M)R 2 => mR 2 ro = (mR 2 + 2MR 2 )ro'
ro 0 I 1 = ro(t )I 2 => ro(t) = ~ ----
2mr2 +MR 2
Clearly variation of ro(t) is non-linear, therefore option
(a) and (d) are rejected. ·
·-4 -4-4
54- (a) ,: = rx F
Let ' ro' be the angular IM = -(i - j) X (-Pk) ·
velocity
Applying;.
of the
.
rod. I. = (i-j) X (Pk) ~---<>X

Angular impulse = Change


! = F[(ix k-j X k]
· in. angular momentum about centre of ·mass of the . = F[-j-iJ= -F(i+ j)
system
..-··
J L .I .
.-2 = ._,
cro

www.puucho.com
Anurag Mishra Mechanics 1 with www.puucho.com

RIGID B_DDY MOTION · 537)


63. (a)
= _3._ ml2 + __!_ ml2 = 189 m/2 = ...?__ ml2
(A0)2 = (1r (1r + 70._ (a)
81 54 4374 162

= 21- 2
/4 This can be assumed as a pure rotation about point of
AO= 1/../2 contact say O with angular velocity ro = ~, where R is
. R
·1 = due toD + due toB + duetoC the radius of hoop ..
= 2m 1_2 + m(_3l)2 mC
-----'c..,..~.... Speed of P will be: vp = (OP)ro = ( 2Rsin!}
2 ../2 '-''-'------- --
2 2
= 2ml + 4ml = 3m12 or vp = (2Rro) sin(!)
2 ·2
64. (c) or Vp = 2vsin(f)
I AC = I EF (from _iar axis theorem)
65. (a) r=.J3L 71. (a)

The desired moment of inertia Volume


about O is:
i.e., V oc R3
or iiV xlO0= 3M xlO0
V R
Percentage increase in volume is 3%
66. :. Percentage increase in radius will be 1%
Now moment of inertia I = ~ mR 2 or I ~ R 2

r+
I2 = moment of inertia about one of
the diameters 5
· =-mr
1 2
:. Percentage increase in moment of inertia will be
2%.
2
' Axis Now since angular momentum J = fro will remain
3 2
1=1 1 +1 2 =-mr conserved. Hence the angular velocity 'ro' will
2
decrease by 2% to keep the angular momentum
67. (c) The distribution of mass. is nearest about=· constant.
68. (a) lxx =l=l,+mx 2 72. (c)
(m = mass of sphere) From conservation of angular momentum about point
.l=I,atx=O !Glj:.:x
!
; c+ : '' '
of contact:
r-. --·· --)
Therefore, I versus x graph is a
parabola with minimum value of
I
'
I .
'
''
'
'X .'
'
,X '~
1 ~v:
I= I, at x = 0. m~1111mimm1i~ \
v~Rm j
Therefore, the correct graph is (a).
V
69. (d) !ro 0 =fro+ mRv or !ro 0 = I -+mRv
AB R
-=2 fro 0 ro
BC or v = or v =_ _,,o_
I I !..+mR I.+_mR_
AB =DC=- and BC= AD=- R. R I
3 6
Now I solid sphere < I hollo~
Sintilarly mAB = mvc = m
3 V solid < Vhollow
and m V1 < Vz
mBc =mAD =~
6 73. (b)
Now I= 2IAB +!AD +Inc Friction force µmg acts in forward direction till pure
rolling is started. Hence linear acceleration
=2{;(½r-½}+(;)(½r +o a=µmg =µg
m
V = at
www.puucho.com
Anurag Mishra Mechanics 1 with www.puucho.com

t=!:'.= roo 78. (d)


or
a µg(½+~) Let m be the mass of the disc. Then translational
energy of the disc is:
again 1 2
I solid < I hollow · KT =-mv
2
t solid < t hollow
or . When.it ascends on a smooth track its rotational
t1 < t2
kinetic energy will remain same while _translational
74. (d)
kinetic energy will go on decreasing. At highest point.
Net external torque is zero. Therefore angular KT·= mgh
momentum of system will remain conserved i.e., 1 2 .
or -mv = mgh
L, =L1 . 2
Initial angular momentum L, = 0 v2 (6)2
or h=-=--=l.8m
:. final angular momentum should also be zero. 2g . 2xl9
or angular momentum of man = angular momentum 79. (a)
of platform in opposite direction If v 0 _> Rro 0 , friction will act downwards
or mv 0 r = Iro if v 0 > Rro. 0 , friction will act upwards
mv 0r (70) (1.0) (2)
(I)=--= if v O = Rro, friction will act upwards
I 200
80. . . . ll'mgof a d'1sc-=-
(c) Incaseo f purero KR l
.ro = 0.7 rad/s . ' KT 2
75. (b) Angular velocity of man relative to platform is
Vo l Where KR = rotational kinetic energy and KT =
· ro, = ro + - .= 0.7 +- = 1.2 rad/s translational kinetic energy
r 2
Time taken to complete one round Leth be the height of the plane. Then at the bottom of
21t 21t the plane.
t = - or t=-s · KR+KT·=mgh
ro, 1.2
Angular rotated with respect ground in this time: or KT =3mgh (as~; =½J
·e = (vor ).t = (.!)(
2 1.2 6-
2
1t) = .§.it
If a is the linear acceleration
· down the plane. Then
76. (b) K.E. of. ball. in position B
. = mg(R -. r)
Here m = mass of ball: .!mv 2 =~mgh
2 3
Since it rolls without slipping the ratio of rotational to
translational kinetic ~nergy will be ~ · or v2 = ~gh
3
5
KR 2 4
or - = - or 2as = -gh
3
KT 5
5 1
or -mv 2 5
= -mg(R- or a(~)=~gh
Kr =-mg(R-r) r)
... 7 2 7 ~m9 3 .
· ~,.._lO_g_(_R___r_)
or v=
v~ or . (I= ~g Sin60°
. 3
(9 = 60°)
7 ro= R-r=°V~
77. (c) From conservation of.mechanical energy. a=c.L
2
J3
mg(R. - r) = 2-mv
fR:p'l
... (i)
10 81. (dJ
and N-mg=-·-
mv
. R-r
Solving equations (i) and (ii),
17
... (ii)
u_J Let f be the friction on the .ring
towards right. ' a' its linear
acceleration and ex· the angular
acceleration about center of mass.
.
.
a
..

f'
. we get N=-mg Point of. contact P is momentarily -· :Pa,,.,
7
at rest Le., ring will rotate about
Note: 2- mv.2 is the translational+ ro~ational kinetic P• .
10
energy of sphere. at bottom ..
www.puucho.com
Anurag Mishra Mechanics 1 with www.puucho.com

RIGID B.ODY MOTION


tp F(2R) F
a=-=-- =- or
lp 2MR 2 MR
Now F+f=Ma=MRa=F·
6. (b, c)
or
82. (b)
J =0
f dmxsin0co 2 = J: (7 dx)xsin0co 2
r---~-:
--- ;
In case (i) work done by friction is zero, while in case : ::a
(L ) = mLsin0co :' . '

-m
(ii) it is·non-zero. 2 2 2
= msin0co ' ,•,·····:···· :
I '
83. (b) L 2 2 i~··.:.--f---··__ :
Let J be the linear impulse -~~
imparted to the ball.

Applying;_
We have
Impulse = cbange in momentum
l=mv 0
E
1/iilii)//jj//l@/~jj//jj/u

... (i)
. 2 2
J.h =Ico 0 = -mr co 0 ... (ii)
5
Tc= J'dmsin0co 2xcos0= msin0cosco 2J'x 2dx
. .
From equanons (')
I an
d ('')
11 , we get co 0 =
5 Voh 7 o _ L o
2
,------1 m 2 sm
= -co . e cos e - (L 3
)
2 2
mL-co-sin0cos0
=- ---

ill:-"
84. (b) L 3 3
Vr =.Vi>Q = 2v
v 2 4v 2 • 2v 2
I So
mL2co 2 sin0cos0 L .
= mg-sm 0

85. (d)
a=-'=-=·-
r 2R 2R R
Q
L.~JL" 3 2

cose=(l)
2Lco2
From the shown figure Cl
8. (b, c)
d+Ra = A
So. A>a
By work ·energy theorem, work , :
done by friction on sphere is ................;....A
:
...G:· Cylinder will move downwards as
mg sine>µ mg cose although frictional force acts
upwards .
10. (b)
cbange in its KE. r . --------

86.
If we consider sphere and plank as a system, then
cbange in KE of plank and sphere is work done by F.-
(b)
V1
X
V2
l-x
Vt
--=--=>X=-~-
v 1 +v 2
An axis passing through x = 2R,
y = 0 is in ® direction as shown
in figure. Moment of inertia· I1_
about this axis will be:
.
Xi
x=2R I
I

(0;'
I· - - . - Ry=O _i
J JI J
V=-=>00=--=- 1 2 2 9 2
2m 2 2ml 2ml 11 =-mR. +m(2R) =-mR ... (i)
. J J J
2 2
vA =v+lco=-+-=- axis passing through y = d, i = · 0 is shown as dotted
. 2m 2(11 m
line in figure. Moment of inertia about this axis will
be:
Level-2: More than One Alternative is/are Correct~..
. ... (ii).

1. Ca) By equations (i) and (ii), we get


• I 2
1 . 2)+-(mR)
1 2 (Vo
- ) 9mR·2 or d- .ffiR
K.E.=-m(v
2 0
2 R
-lmR2.+m dz -
- - -=--
4 2 ·2
+.!.(3m)(.J2v 0 ) 2 +.!.(m)(2v 0 ) 2 12. (b) I =I ~ 2f m/2 sin2 450} = mlz fx:. ---1
2 2 · X y cl 3 : 3
=6mv~
I z --2(ml2)--~mz2 1, ·v:8·i
3. (a, c) , 3 3 ; . · - _J
Vo . roo
t=-=- l·
a a· .HenCe,
www.puucho.com
Anurag Mishra Mechanics 1 with www.puucho.com

[540 ,___M_E_C~~m~:o
14. (a) .
After t h e given- .
tune t llml
= --, t he ro d w1
"JI rotate an
For a given perimeter area of a circle is maximum i.e., 121
distribution of mass is at a maximum distance from
angle 0 =rot=(~)(;;))=%
the axis in case of a circle.

~ =v
--> --> -->
15. .(a) ap = ap0 + a 0 ½,ro=(i) (~)=
-->
Here a,0 = acceleration of P with respect to 0
l;PI= --J2v = -J2:!_
m
27. (a)
--+ -) -) --+
ap+ aPo,+ aPon+ ao Let v be the velocity of rod in horizontal direction then
displacement of centre of mass in horizontal direction
--> -->
Here a,0 , = tangential component of a,0 and in ls will be: x = vt = v displacement of rod in
--> ~ --> vertical direction in ls will be: ·
a,,n = normal component of a,0 1 2 1 2
y=2gt "'2(10)(1) =5
--+ --+ --+ •
2
Ia 0 + ap0 ,I= a+ ra => Ia,,nl= rro 2
Given that ~x +y 2 = s,J2 or~v 2 +25 = 5--/2
l;pl=~(a+ra)2 +(no2)2 V = 5,m/S

24. (a) Comparing this with linear motion, Now let J be the impulse applied at B. then applying
Power P = F.v Impulse = change in momentum
We have P = t,Ol Wehave l=mv ... (i)

or dro) ·Ol=Porro
I· ( ro.- Pd 0
. . 2d ro=- and J. i2 = Iro ... (ii)
d0 · a
Dividing (ii) by (i), we get
· on integration, we find that ro = 91/3
mvl mvl 6v 6x 5
or ro = (n)1/ 3 ro =- =----,---c-orro = - = - - = 3Orad/s
25. (c) 2I 2( ~~) . l l
Let m be the mass of cube and' a' the side of cube. The
cube will slide if: 28. (c) Angular momentum of system cannot remain
F>µmg ... (i) conserved as some external unbalanced torque is
present due to forces at axels. Kinetic energy is not
and it will topple if torque of F about Pis greater than
conserved, because slipping is there and work is done
torque of mg about P i.e.,
against friction.
F.a >(~)mg or F >½mg =mg
29. (b) Let ro 1 and ro 2 be the final angular velocities
when the slipping is ceased, then
1 I ,
V1 = V2 or Ol1R = Ol2(2R)
or F>-mg ... (ii) a: .
2 Ol1
1 fo~ _P. or ro 2 = - ... (i)
From equations (i) and (ii) we L.!':iction a/2. 2
see that cube will topple, before Now applying angular impulse =
I 'd'mg I'f µ > -.
SI l change in angular momentum
2 we have
26. (d) ~R=~1 ... 00
Let v and robe the linear and angular speeds of the rod and J.2R = 4I (ro 0 - ro 2) ... (iii)
after applying an impulse J at B. Then from
Here J = linear impulse due to friction which will be
Impulse = change in momentum tangential and equal for both the cylinders.
J
We have mv = J or v = - ... (i) Solving equations (i), (ii) and (iii), we get:
m Olo
l ro 1 = ro 2 and co 2 = -.
Iro = J.- . 2
2
30. (b)
ml 2 l 61
or --.ro=J-orro=- ... (ii) Linear acceleration of cylinder is zero i.e., mg sine=
12 2 ml
frictional force (f) upwards (m ,,; mass of cylinder)
www.puucho.com
Anurag Mishra Mechanics 1 with www.puucho.com

RIGID BODY MOTION s411


't AB 21!
.-. angular acceleration about C is a=- 34. (d) <OJ>=-.-=-
I At T
or a=
~mR2
f.R· = 2f
mR
(R = radius of cylinder) =---
21!
2v siri0
~-
vsin0
2 g
2mg sine 2g sine
or. a= . 35. (c) ro=~andv =0
mR R R '
For no slipping between cylinder and plank
a =Ra= 2g sine 36. (a) 2{mR 2 }{2it-9}=~MR 20
31. (b) 2
The rod will rotate ab\lUt point A with angular 4mit - 2m9 = M 0
acceleration: 2
F.x 3Fx 1 3 Fx
9=
't Smit
a = - = - = - -2 a--a=--
I ml · m/ 2 2ml 4m+M
3 37. (c) f-rdt = AL
or a oc x
i.e., a-x graph is a straight line passing through origin. or I
L.= ;21:cR + r)dt = (R + r)t 2
32. (c) 38. (a, d)
. 2
From impulse =change in momentum we have L1 = Iro = MK ro ... (i)
J=mv , ... (i)
(v = linear speed of centre of mass of rod) L2 =Iro+MRv
ml 2 = MK 2ro + MR(roR) (as v = Rro)
J. x =· Iro = --ro ... (ii) 2 2
3 =.Mro(K
., ,,
+R ) ... (ii)
1
V = -(1)
2 . ... (ill) From equatim;1 (i) and (ii), we can see that L 2 = 2L1
When K = R and L 2 > 2L 1
(ro = angular speed of rod about When K >R.
point A) ·
39. (a, c) 5-F, =1X2=>F, =3N
Solving these three equations, we gee: x = ?:.' l Taking torque about CM:
. 3
Sx = 3(1 + x) => 2x = 3 x 20 => x = 30 cm
33. (c)
Length of rod = 2 x (! + x) = 100 cm = 1 m
Let v be the linear velocity of centre of mass of the -,
spherical body and ro its angular velocity about centre -+ -+ -+. dL -+ -+
41. (a, b, ·c) 't = A xL z.e., - =Ax L
of mass. Then ro = ~ · -,
dt
2R
K.E. of spherical body This relation inlplies that d L ~ perpendicular Aand
dt
1 2 1 2 -,
K1 = -mv +-Iro L. Therefore, option (a) is correct.
2 2
-, -, 2
2
1 1 v -) 3 L.L=L
or K1 = -mv 2 +-(2mR 2) ( - = -mv 2 ... (i)
· Differentiating with respect to time, we gee·
. 2 2.. .. 4R 2 4 -, -, -,
Speed of the block will be L. dL + "dL .L = 2L dL => 2L. dL = 2L dL
dt dt dt dt !it
v'= (ro)(3R) = 3Rro a; (3R)(~) = ~v -, ...
.. . .. ' 2R 2
Since L.L dL. so, =0 L.~
K.E." of block K2. = ½mv' 2 . dt .at

from equations (i) and (ii)


r
=½m(1v = ~mv 2 ••• (ii)
· dL

time. So option c is correct.


I
Therefore - = 0 and then L do.es not change with
dt I

Since L is not changing with time, therefor it is the


-,
K,· =~ case when direction of L is changing but its
. -, -,
.Kt 3 magnitude is constant and -r is perpendicular to L at
all points.
www.puucho.com
·~ ...
Anurag Mishra Mechanics 1 with www.puucho.com

[542 ·--- MECHANICS"] '

This can be written as


..., . .
· If L = (acos0) i+(asin0)j (bere a is a constant)
..., . . using v = u +at=> .':'..Q. = ugt => t = 1
~ = (asin0) i-(acos0)j . . 3 3µg
-t -) -t -t
So tbat, L.~ = Oand L..l~ 0 0v 2v
..., 4. (c) V=Vo-µgt=Vo--=--
3 3
Now A is a constant vector and it will always
..., ..., 6 . (c) Separation will be maximum when relative
perpendicular to ~- Thus A can be written as velocity is zero .
...,
A=Ak Velocity of. cylinder at time t = 0 + µgt
..., ..., ..., Velocity of cube at time t = v 0 - µgt
We can see tbat LA= 0 Le., L..LA
. ..., ..., µgt=v 0 -µgt =>t=1
Thus we can say tbat
..., tbe component
..., . L along A is zero 2µg
or component of L along A is always constant. Maximum separation = S cube - S cylinder
-t 4 -t
Finally we conclude tbat ~, A and L are always. 2 2
= ( v 0t -½µgt )-( o+½µgt )
constant.
2 2
42. (a, b) - 2 Vo Vo
=v 0t-µgt =---µgx-~-
On smooth part BC, due to zero torque, angular 2µg 4µ 2g2
velocity and hence tbe rotational kinetic energy
remains constant. While moving from B to C = v~ _ v~ = v5
translational kinetic energy · converts into 2µg . 4µg · · 4µg
gravitational potential energy; Passage-2
43. (c) i. (b) By conservation of energy
Initially tbe spring force kx is. less
tban mg sine i.e., tbe cylinder is
I. kx ---1 L· 1 mL2 2
mg-=--ro =>Lro=~3gL
2 2 3 ·
accelerated downward or force i~f
. gsln8; 2. (d)
tFy ----- -·-·· ---:
of friction f is upwards. It will '
I !' rr·-
reverse its direction when
kx> mgsin0.
-- - . . - --·····-·j

I•Fx l i. • m½w2!,
-===--------~
Level,3: C~mprehension Based Prc>blems
J • -
l _________ mg ___ · ---·· _I
L 2 mL 3gL 3mg
F =m-ro = - X -2 = - . -
I

x 2 2 L 2
Passage-1

1.
. .. . 1 1 mR 2 Ii~ 3mv~
(d) KEm1trnl=-mv 02 + - - - - - = - -
2 2 R2
30
~ng torque about 0. :..J·Fy ------ . . .-~t.,:,1:
2
KE final = I_ mR 2'.Q_ = mv o
2 2 2
4
mg!:_ = mL2 a.=> a.= 3g
2 3 2L
: Ex !
···------.mg _________ _j
:
2 2 R 2 .. 4
3
mv~ mg - f = m(!:.a.)= mg
y 2 4
% loss = 4 = I. = 33.33% 3mg mg
3mv~ 3 f =mg--=-
Y 4 4
4 Passage-3
2. (a) . 1. · (d) Fcos0- f, = ma ... (i)
Before collision no friction acts due to pure rolling
and after collision it acts in forward direction.
,----·--- - ----- --,
2
f,2R-FR = 2mR a.
2f, -F = 2mRa.
!@(, Fl
I
'
'.
. '
.,
:- _,a__.,,. II
I
3. (d)
By conservation of angular
momentum about point of
contact.
10;:0;:
:~,m~
\ Vo Vl
Since
so;
a= 2Ra.,
2Fcos0-F=3ma ... (ii)
F(2cos0-1)
.
,
---- -·-
.
--
· i
.
a
3m

www.puucho.com
Anurag Mishra Mechanics 1 with www.puucho.com

LRJGID,,_BO_D_Y_MO_T_IO_N_.- ~ - - - - - - - - - ~ - - - ~ - - - - - - - - - - - - - ~ · 5 _ 4 _ , : f ,
F
for 0=0, a=- 3m 5. (b) v=u+at=o+!t
4
2. (dl For 0=60°=>a=O g gt
CO=O+cxt=-t=>RCO=-
3. (a) f, = Fcos0- ma . 2R 2
F- (2cos0-l)
= F co a s------ Net velocity = gt - gt = gt
3 2 4 4
F(cos0 + 1)
~ J
=---~ 2
3 = BltR x g = 1tRg leftwards
g 16 2 ·

4. (c) Since
f, is +ve for O,;; 0,;; 90°
F(2cos0-li
a=----'-
, 3g ~81tR. 3
6. (d) v=u+at=0+- --=--v21CRg
4 g 2
=
3m
a will be positive for0 < 600 and-vefor0 > 60° Passage-5

5 • (b) As
_ F(cos0 + 1) 'f, _ 2F 1. ( d) Since energy is conserved for all tbe three
J.,- 3
. =>nmaxl--
3 bodies, so, final kinetic energy of all tbe three bodies
6. (d) will be same.
K2 .
F-f,=ma 2. (a) - for nng = 1
FR+ f,(2R) = 2mR 2cx f~l R2

F +2f, = 2mRcx- ma -~1 2


K for disc
R2
= 1/2 == 0.5
=> f, = 0
a=F/m
~,_..i.l K2 .
- for sphere = 2/5 = 0.4
Passage-4 R2

1. (d) For cylinder T


. mR2
= ma, RT= --ex. -· We know a·= g sin a
2 . K2
a+-
For block mg - T = m(q + Ru) R2

mg-T =T+2T =>mg= 4T =>T = mg a(sphere) > a (disc) > a (ring)


. 4 2
since v = 0 + 2as = ':' v (sphere) > v (disc) > v (ring)
a=!,Rcx= zr =! 3. (d)
4 . m 2 Higher tbe acceleration smaller _will be tbe time taken.
3
Acceleration of hanging mass = ! + ! = g 4. (b) Since a (sphere)> a (disc) > a (ring)
. 4 2 4 So, f (sphere) < f (disc) < f (ring)
3. (b) For cylinder ex=..!_ Passage-6
2R
1 2 =-I(2co)
1. (c) -kx 1 2
=>-1 kx22 =-(2I)(co)
1 2
So using 0 = O+ .! ext 2 => 21t = .!
2
(..!..) t
2 2R
1 2
2 1 2 2 2
x, = -./2
X2
t=~B~
2. (a) Applying conservation of angular moment
_so for cylinder using · (0
'
= J(2co) + 2I(co)
--'---'----'-~ 4co ... (i)
S =ut +.!at
. 2
2
~ o+.!.!x(
2 4
8
1tR) = 1tR
g· ' 't
3J 3
co=co+-t ... (ii)
21
4. (c) 2Ico
1 3g 2 3g 81tR From (i) and (ii), 't =-
For block; S=0+-x-xt =-x-=31tR 3t
2 "4 8 g
Alternately: Distance traveled = distance traveled by 4. 1
(d) -mv
2
2
1
+-ICM
2
-
R
(v) 2
= mg (3v
-
-4g
2
)
centre of cylinder + 21tR = 31tR
1 2
Hence ICM=-mR
2

www.puucho.com
Anurag Mishra Mechanics 1 with www.puucho.com

~-------·'--~---~-~
' l
(a) -
2
Cb) i
3
l
(c) -
6
1. A conical pendulum
consists of a mass M \ ccii L
4
suspended from a string . 1
of length t The mass )-y
executes a circle of radius
X I Solution : Let imptilse due to
I
particle be J
R in a horizontal plane ·I
with speed v. At time t, l113l
i •
· u6ro •
!l
• I
the mass is at position Rl i -,-'--!ol---.,.:vcM
H

and has velocity vj. At 1 l/6 ' l


I ,.._.__•vcM • CM 1

time t, the angular

l
ro • :
momentum vector of the . I
mass M about the point from which the string
suspended is: -----:r-- ,---
Fig, 5,3
________ l _,!

(a) MvRk J =;.mvCM


(b) Mvlk VCM = J/m
(c)
2
Mvl[ ..;-12-~-R- i+Tk] Jx=--ro
12
mz2

12Jx
(d) -Mvz[ ~12 ~R2 i+Tk] Ol=--
m!2

Solution : From Fig. 5.1


_, _, .
For point A to be rest insta.ntaneously
V trans+ V rot :::::: Q
-; =Rl-~1 2
-R 2
k _, _,
_, . or V CM/G+V A/CM =Q
p=Mvj J l 12Jx
Thus -=-X--
~
L= rxp
~ -4
' m 6 m! 2

l
2. -A thin uniform rod of mass m and length I is kept on a X=-
smooth horizontal surface such that it can move freely. 2
Where should a particle of mass m strike on the rod 3. A yo-yo arranged as shown, rests on a frictionless
surface. When a force F is applied to the string, the
such that the point A at a distance .!. from the end of yo-yo:
3
the rod is instantaneously at rest just after the elastic
collision?

www.puucho.com
Anurag Mishra Mechanics 1 with www.puucho.com

I RIGID ~ODY MOIION -----~- ------- _______.,_' -·~---- __ · 545


- ~------.
:Q· , . _·: Angular velocity of larger cylinder is
., ~

;~j 2V V

4R 2R
-=-

IL_' ----- --.. --- .


Fig. 5.4 '
VcM (2R{;)
= = V

6. A systelll of uniform cylinders and plates is shown. All

:f
(a) moves to the left and rotates counterclockwise
the cylinders are identical and there is no slipping at
(b) mcives to the right and rotates counterclockwise
any contact. Velocity of ~ • 2u
(c) moves to the left and rotates counterclockwise
(d) moves to the right and rotates clockwise lower & upper plates is v ~ ~~
Solution: Check torque abut CM, only external
force is R
:~iwnl_2;h:':?thde~ti;:f;(oth~~ ;v • : : , . :.:
angu ar spee s o e ,
4. A wheel of radius R rolls on the round with a uniform 1
upper cylinders to lower , t __ .., __ _
I_ Fig. 5.8
velocity v. The relative acceleration of topmost point of
cylinders is:
the wheel with respect
f ' . . . .to. the
....bottommost point is: (a) .!. (b} 3

:. (IDA:
'
'
. : 2
. '
(c) 1
3
(d) none of these
Solution: In the absence of slipping, velocities of
.

I E._ ' contact points of upper cylindrers and lower


I • R ''
! cylinders are resp_':c?ve_ly.- . . .. .....

2v·aC".
j B
I,____ Fig.
. --5.5

0
-
. A ·
(a) V2 (b) 2v2
1
R R 1 t ' . _
v2 2 I
~
I
D
V =O
(d) 4v
2R
•V
(c) -
R. Fig. 5,9
Solution: As aCM = OTvcM = const.]; Tangential ---~--,., -
acceleration of each point is zero. 3v VAB
=-=-·
.... I
aAB = -
2v
2
Ci)
Up 2R AB'
I R
C.Otower =
VCD
CD = _2R
V

5. Velocity of the centre of smaller cylinder is v. There is


no slipping anywhere. The velocity of the centre of Cilup
--=3
larger cylinder is:
r··-- - ...... . 00 1ower
,c:::;;:~===;:~ 7. If I represents the M.I. of a solid sphere about an axis
\ ----.v2 parallel to the diameter of the solid sphere at a
!'..,.;;,,...;;....1, distance x ftom it, then which of the following graphs
represents the variation or I with x?

(a) 2v
l_. __ ·- _Fig. 5.6 _
(b) V
(aji') ·. I ! :\'f / <bJ
(c) 3v (d) none of these :.2~x ~x
2
Solution: In the figure C1 and C 2 are IC of two
cylinders. In the absence of slipping between ''I ~ · I ''i
plank and cylinders, points A1 and A2 have same CcJ : . ·. . ;
('~--·· -·:0------ ·x:

i
A
2
2v ·~2v'
velocil,ty. ..... ___ ,,__ ......... .

vi C ,
Hint: I= le +Mx
_...
2
~--· -0 . X

. ,', '

·Fig. 5.7
--- --- -----
www.puucho.com
Anurag Mishra Mechanics 1 with www.puucho.com

~I5_4_6~~-"~-~-~-'-_'·_._·_;_·_,·'--'""·,;~\~··~·--~.......__ _,__ _ _ _ _ _ _ _M_E_cH_A_tl1_cs_-•---1I


8. A uniform rod of length I is
placed symmetrically on two
= ML
12
+M( 2-JZ
2
3L ) 2

walls as shown 'in figure. The rod


is in equilibrium. If N 1 and N 2 29ML2
=---
are the"normal forces exerted by 24
the walls on the rod then: 10. The figure shows a uniform f··
(a) N 1 > N 2 rod lying along the x-axis.
(b)N1 <N 2
(c) N 1 =N 2
L The locus of all the points
lying on the :zy-plane, about !
(d) N 1 and N 2 would be in ·the vertic~ directions which the moment of inertia lo-~===--•
of the rod is same as that I • Fig. 5,14 .,
Solution: about 0, is: '
(a) an ellipse (b) a circle
(c) a parabola (d) a straight line

Solution:
. !1'" ' - - ... ---· -·--····-1
~= \f (x- ~)'+ y2 i
,, •'

. Torque of all the forces about CM is zero.


9. A square plate of mass M and edge L is shown in Fig. i----X--*I
5.12. The moment of inertia of the plate about the axis Fig. 5:15 .
in the plane of plate and .passing through one of its -----~ -----·------~
2

('l ' 1.
vertex making an angle 15° from horizontal is: Ip=ICM+Mr
2

-.._,:i.L . .
=~~ +M[(x-½r +y 2
]

. . ML2
Io=--
3
i--'-L~
' ' .- ' ,- Apply IP =lo
_Flg.:5:12 .,
2
11. Find minimum height of the obstacle so that the
ML2 (b) 29ML sphere can stay in equilibrium.
(a) -
12 .24 (a) R (b) R
. 2
(c) 7ML l+cos9 . l+sin9
(d) none of these
12 (c) R(l - sine) (d) R(l - cos9)
Solution: From l'arallel axis theorem
m· -,

.,j1;:~t~;
~--: .:----·:: ___ _
~~~--"-'=i'I I
I
i.
I
i

r.: .-·;;,g. 5.13"·:. Fig, 5.16


I
mg
Fig. 5.17'.
ML2 Solution: Sphere is on verge of toppling when
I,=--
12 line of action of weight passes through edge
I 2 =I 1 +Mx 2 cos9 = (R-h)
R
h =R-Rcose

www.puucho.com
Anurag Mishra Mechanics 1 with www.puucho.com

I RIGID BODY MOTION


12. The spool shown in figure is placed on a rough Solution: From conservation of angular
horizontal surface and has --::;;::::::;::::--;, F momentum about 0, we get
inner radius r and outer mvR = (mR 2 + mR 2 )ro·oi = ~
radius 'R, The angle 9 between ' 2R
the applied force and. the 14. A sphere is placed rotating with its centre initially at
horizontal can be varied. The rest in a corner as shown in figures (a) and (b).
critical angle (9) for which the Coefficient of friction between all surfaces and the
spool does not roll and Fig.MB sphere is .!. Find the ratio of the frictional forces
remains stationary is given 3
by:
(a) 9= cos- 1 ( ; )

(c) 0 = cos- 1
~
(b) 9 = cos-
1

(d) 0 = sin"1(:~)
(!)
r~-- --~-l
fa I fb by ground in situations (a) and (b).

~ - ~ iI
Solution: If spool is not to translate
r·------- -. -- .... --·---·- (•l (bl
r• Fsine
A
Fig. 5.21 I
. F . (a) 1 (b) ~
10
(c) 10 (d) none of these
9
,c..: •••••:,._- - Solution: In figure (a)
1
µNt +N2 = mg ... (1)
static friction
Fig. 5.19 I
• I
N1=µN2
µN,
------
... (2)

Fcos9 = f ... (1) N,


If spool is not to rotate
Fr=FR ... (2)
From eqns (1) and (2), we get
Static friction
Fcos9 = f mg_
Fxr=fxR Fig. 5,22

or fR. cose = f N2 = _!!!!L_


r 1+µ2
or 3
fa =-mg
10
or In figure (b)
N 1 =0;N 2 =mg
13. A small bead of mass m moving with
velocity · v gets threaded on a i, ~ - - - - : fb =µN2 = mg
stationary semicircular ring of mass i --~--o··::J· .3
m and radius_ R kept on a horizontal . fa 9
·v -=-
table. The ring can freely rotate . . "" fb 10
about its centre. The bead comes to Fig. ~:2~ . 15. A block of mass m is attached to a pulley disc of equal
rest relative to the ring. What will be mass m, radius r by means of a slack string as shown.
the final angular velocity of the system? The pulley is hinged about its centre on a horizontal
(a) ~ (b) 2v table and block is projected with an initial velocity of 5
R R m/s. Its velocity when the string becomes taut will be:
(c) ~ (d) 3v
2R R

www.puucho.com
Anurag Mishra Mechanics 1 with www.puucho.com

r.-..-.

(a) 3 mis (b) 2.5 mis


. 5 10
(c) -mis (d) mls
: 3 3
Hints: Apply conservation of angular momentum
about hinge. al -ra=·o ... (1)
16.. System is shown in the figure. Assume that cylinder
remains. in contact with the two wedges. The velocity aB =; 3ra - a1 ... (2)
of cylinder is: or aB = 2a1
~t~r~:
:'~!~"-,~ -,:·-.: ,-c-yu~~J~r;:r~-: aB +ac
= U2
2

Ii,},; > , ,._ or

From eqns. (3) and (4), we get


ac = 2a 2 - 2a1
ac = a3 + 2ra
:.. (3)
... (4)
II~''.i!;:','t.1~
BL. ,.,"~'..!- v.: •..........._..Flg.-5.2_4.;
___ .
...
¼, • ..,,,~-- --·~.,,,..-~~ • _

·
(a) ~19- 4J3 '=- mis
. 2
(b) mu
2
mis 18. Four rods of side·
2a2 - a3 = 4a1

length 1 have been


(c) ./3 u mis (d) .fi u mis hinged to form a
Solution: Let V X and Vy be components of velocity rhombus. Vertex A is
of centre of mass of fixed to a rigid
cylinder at contact 1;:. ; •., ._. .:' · support, vertex C is
po.ints A. and B I · · v, being moved along
respectively, the (.,;,' ;· c. :i,o• the x-axis with
velocity along normal ; · · , 1.1:'·· A __ :-;f'c+---f::.-.. constant velocity V as shown in figure. The rate at
,,. , .··30° 1
which. vertex B is nearing the x-axis at the moment the
-~~:~:irly be ve~:~ jj;io/a;; ·· . ~Y F rhombus is in the form of a square, is:
·com~one_n(s . alo~g _.,:'·,,Fig. ~. 2 !)};-:
the hne Jommg pomt ~<c, ••..L_,_____ - ---····,,.....
t· , (a)~
4
(b) ~
-.fi.
A and C, and line joining B and C.
(c) ~ (d} vz
u cos60°= vy cos30°-vx cos60° ... (1) 2 g
=2u Px ... (2) Solution: Let velocity
From eqris. (1) and (2), we get v x = 2u; v y = ./3 u components of point B are
v = ~v~ + v~ = .fi u vx and vy. Along the
length of rod, velocity of
17, In the figure shown, suppose the compound-pulley and each· point must be same
the disc have the saIT)e angular acceleration, in (rigid body constraint)
clockwise direction_. If a1 is. the upward acceleration of
the co.mp'ound pulley's centre (inner radius r, outer
radius 3r); _a 2 is the downward accel_eration of the
V Vy V
block while a 3 is the upward acceleration of the centre ....£+-=- (along rod 2)
of the disc .(radius 2r). From kinematic constraints of -.fi. -.fi. -.fi.
the thread, the relation ben,veen 'them is: V
...L--=0
Vy
(along rod ·1)
(a) a 2 - a 3 = 2a1
. 2
(b 1 a 2 + a3 = :2.
. . ' 3
-.fi. -.fi.
V
(c) a 2 - ci3 = 3a1 (d) a 2 + 2a 3 = a, V =V =-
~ y 2

www.puucho.com
.r, .
Anurag Mishra Mechanics 1 with www.puucho.com

__ ,.__ - -- -- --~· - ---

19. In the figure shown, the instantaneous speed of end A (a) 4v2
2
(b) 3v
of the rod is v to the left. The angular velocity of the SR SR
rod of length L, must be:
(c) 3v2 (d) 6v2
(a)__!}__ (b) ~ 4R . SR
2L L
Solution:
(c) v..fj (d) none of these "' v2
2L faAcl=-
1·------------- ,.- D
1 • I l IC SR
! •!I .
:·,·...
'
L
'• •.
.
~ -)

aBA = aB/G- aA/G


~
V, la'AG i
IA , I
iL I
I

!
"' v2

1~
I
I'
!!
:i
ti
A'
:
:, 60D
L
I
i
t
laBAl=-
... R
... ...
aB/G = aBA + a AG -.Fig. 5.34
-·:
'->
,·213~

I
I
Fig. 5.30 [I Fig. 5.31 I ~ v2 v2 4v 2 '
L --· ·--· ---· -- --- - - - ' I aa;GI= R - SR= SR
Solution: Draw normals at A and B to locate IC.
V
. . "' v2 v2 6v2
Ol=- S1mtlarly I av;GI= - + - = -
L R SR SR
20. A uniform solid sphere of radius r 22. Angular acceleration of the cylinder C shown in the
is rolling on a smooth horizontal figure is (alf strings and the pulley are. ideal):
' - - ---- -----,,.-·--;,
surface with velocity V and m A , f
angular velocity oi(V = oir). The
'
· sphere collides with a sharp edge j
on the wall as shown. The
coefficient of friction between the sphere and the edge
µ = L Just after the collision the angular velocity of Flg.'5,35
s - - ·---- ------'
the sphere becomes equal to zero. The linear velocity (a) 2g (b) 2g
of the sphere just after the collision is equal to: 3R SR
(a) V (b) ~ . (c) 2g (d) _L
s R 2R
(c) 3V (d) 2V
s s Solution:
Solution:
~--~-
--- ;,}Nttt;

ff=
' V' '

oo= ~~i
i _ -- ---~ig. 5.~~' ·--- _<:.J
-f Ndt = -mV' - (mV) ... (1)

µRf Ndt =imR 2


(i) .. (2) mg
Fig. s:36
From eqns. (1) and (2), we get
fNdt=2mV
and V'=V
...
la8 1= a-,-Ra
21. In the previous question, the acceleration of the point
mg-T1 =ni(a+Ra) ... (1) .
on the ring which is farthest from the centre of the
pipe at the given moment is: T1 + mg - T2 = ma ... {2)
T2 ,,; m(a - Ra) ... (3)

www.puucho.com
Anurag Mishra Mechanics 1 with www.puucho.com

. MECHANICS-I I
. . mR2 Centre of mass of the lower
T1R+T2R=--a
2
On solving these eqns., we get.
two third part moves in a
circle of radius 2L. We
I R I T

a= 2g ·a= 2g
3' SR
apply Newton's second law
on this part
3
I ~Mg :•CM

23. Uniform rod AB is hinged at end A


I T
3 I 2M
__ 9

~-./~.·:
in horizontal position as shown in
the figure. The other end is
connect~d to a block through a
~-~- '. L__ ..!'.1.9· 5.40
3

massless string m as shown. The or T=2Mg


pulley is smooth and massless. A 25. A uniform rod of mass m and length 1 is fixed from
Fig. 5.37
Masses of block and rod are same point A, whiclt is at a distance i4 from one end as
and equal to m. Then acceleration
of block just after release from this position is:
Ca) 6g Cb) II
shown in the figure. The rod is
free to rotate "in a vertical r~---
. A
--:-··1
13
(c) 3g
4
(d) none of these
plane. The rod is released from
horizontal position:
.
Pivot
'I
J+---/--1,j
8 What is the reaction at hinge
Solution: mg-T=ma ... (1) when kinetic energy of rod is Fig. 5.4.1, ·
2 maximuffi?
Tl- mgl = ml a ... (2)
2 3 (a) ;mg (b) ;mg
Also a= la ... (3) 13
(c) -mg
11
(d) -mg
Acceleration of end B of rod is la 7. 7

~;¥~
L
I . ____ .5:~e"'
• Fi g ___________.J '
•> Solution: KE of rod is
maximum when it is in vertical
position.· From conservation of
energy, we get
. mgl
4 2 12
·
2 2]ro2 I ;;;·1"
= .!_[m1 + m1
16
·1
Thus a= 3g·T= 5mg ro 21. • 6g
-=- Fig.5.42
i
;
8 ' 8 4 7
3g
a=-
81
and Ry-mg= m({)ro 2

24. A thin uniform rod of mass M


and length L is hinged at its
upper end and released from rest
l--~i:.--l---
1 :, :' U3 2l .•
Ry =mg+m(~)

in horizontal position. The I ro~


,
.... ,, -:--· '
· ... 3_.
R = 13mg
y 7
tension at a point located at a I --,:a:--.- . ,
' :: .. 26. A straight metal rod of
9istance !:. from the hinge point, I\ . '--·'Fig.5.39
' length 31 is bent through
. 3
the right angle as
when the rod becomes vertical, shown. The bent rod is
will be:
then placed on a rough
(a) 22Mg (b) llMg
horizontal table. A light Fig. 5.43
27 13 string is attaclted to the
(c) 6Mg (d) 2Mg vertex of the right angle. The string is then pulled
11 horizontally so that the rod slides at a constant
· Solution: When rod becomes vertical velocity. Find the angle a that side 21 would make with
MgL 1 ML 2 2 the string: ·
--=---(0
2 2 3 (a) 1t-tan-i(½) (b) 1t-tan-'(½)
ro=fj (c) 1t-tan-1 (¾) (d) lt-tan-1 (2)
www.puucho.com
Anurag Mishra Mechanics 1 with www.puucho.com
f

RIGID BODY.MOJION '


.,
....
•'
... ,5511
Solution: Net torque about surface. Neglecting air
9 is zero
µ(2mg )I sin 9 = µmg i cosB
1
2
r.2~m~~.-~.. ;· ·
t ~~~~o~_
resistance, what happens as
the ball swings around the
pole?
tanB = -
4 ii__ Fig. 5.44
(a) The mechanical energy
and angular momentum.
a= 1t-tan-
1
(i) are conserved
(b) The angular momentum of the ball is conserved
27. In the above problem, the initial acceleration of the and the mechanical energy of the ball increases
lower end of the rod will be: (c) The angular momentum of the ball is conserved
.ff ../3 and the mechanical energy of the ball decreases
(a) g4 (b) gs (d) The mechanical energy of the ball is conserved
../3 and angular momentum of ball decreases
(c) 3g- (d) none of thes~ Solution: Torque of tension about, O is inward
7
(clockwise sense) therefore angular momentum
Solution: From constraint condition decreases.
~asin30°-a =·0 29. A uniform pole of length L and mass
2 M is pivoted on the ground with a
frictionless hinge. The pole makes
an angle 9 with the horizontal.· The
moment of.inertia of the pole about · ~
one end is '(½}ML 2
• If it starts
~~lg.5.47
N falling from the position shown in
a (accel. of Cll,1)
the accompanying figure, the linear acceleration of the
free end of the pole immediately after release would
be:
a (a) (~) g cosa (b) (~) g
Fig. 5,45

a=-a
4
L
... (1)
(c) g (d) (¾) g cosa
mg-N = ma ... (2) MgL ML2
Solution: --cos8=--a
N(~ sin 30°) = mL
2
... (3)
2
3 g cos a
3
Ry!~
ttr~g
2 12 a=---
2 L
From eqns. (1), (2) and (3), we get
3
a=l2g a= La= -g cos8
2 Rx= 0
7L
a= 3g Fig. 5.48
7 · 30. A uniform rod of!engtb I is ·kept
N= 4mg as shown in the figure. H is a
7 horizontal smooth surface and
Acceleration oflower end of rod is W is a vertical smooth wall. The
..., L " rod is released from this
a,= -acos30°(-i) position. What is the angular
2
acceleration of the rod just after Fig.'5.49
L 12g ../3 :
=-X-X-(-1) the release?
2 7L 2 (a) 6gcos8 (b) ~gcos8
3
= ../3g c-h l 2 I
7 (c)3g cosB (d) 2g cosB
28. A ball is attached to a string that is attached to a pole. l
When the ball is hit, the string wraps around the pole
and the ball spirals
.
inwards sliding
. on the frictionless
www.puucho.com
Anurag Mishra Mechanics 1 with www.puucho.com

, , .- , .. ,, r· ' .:.• ;'!.,:.


-~;i.~~"'/'e-.-'-"->- ·; ..;t>'. 1\_~'·-'-',,_"_·_ __...--t.;.._

Solution: ~c~'N,-·--1 Solution: Net acceleration of ball is:


1- ·-- . ----:-· - ---- --- - :..+ , •.
Consider torque about jc l ··-::;·····••-,___ ''
l
2
ml 2
mg-cos9 = --a
3
3g cos9
a=-----
2 l
t ' : ••••
I

IN'' .:
'
!,
1
:,
ll2
-'·••

.
mg
• ,

1
;!1 ~ m~
L Y / · ._-'-~ _. . ..

l' , 8 e ,
·
i
i____fjg,_5.50, .__ __J
I ' . Fig. 5;52
31. Consider three solid spheres. Sphere (i) has radius r j__ , _____ 1_ ' .:...·:...:...--'--------~---*":.,'
·-····--· - - - · - · ___ .,

..., ..., ...,


and mass m, sphere (ii) has radius r and mass 3m, &ball/ground = &ball/CM+ aCM/ground
sphere (iii) has radius 3r and mass m, All can be placed ..., ..., ...,
at the same point on the same inclined plane where aball/CM = avcM + ac;CM
tangential centripetal
they will roll without slipping to the bottom. If allowed acceleration acceleration
to roll down the incline, then at the bottom of the laball/grnundl= ~a~ +.(Ra) 2 + (co 2R) 2
incline:
(a) sphere (i) will have the largest speed ·= ~10 2 + (lx 2) 2 +[(2x 3) 2 x1] 2
(b) sphere (ii) will have the largest speed = .Jl400 m/s 2
(c) sphere (iii) will have largest kinetic energy
34. In the figure shown, a ring
(d) all the spheres will have equal speeds
A is initially rolling without .
2 sliding with a velocity v on
Solution: mg~=½mv (1+~Jk= radius of
the horizontal surface of
.gyration, For all solid spheres, ~=3 the body B (of same mass
R 5 as A). All surfaces are
V
2
is independent of m and R. smooth. B has no initial velocity. What will be the
32. A rod of length l is travelling with maximum height reached by A on B?
2 2
3
velocity uCM and rotating with (a)~ (b) ~
4g 4g
angular velocity co such th~t ucM = col_
2 v2 v2
(c) - (d) -
The distance covered by, the point B• 2g 3g
when the rod completes one full Solution: When ring is at maximum 'height, the
rotation is: wedge and ring have same x component of velocity. In
(a) 1tl

Solutiop,:
(b) 81
.
u=
(c) 4l

. 2 -
------------
-
(d) 2it1
(co1) 2
. 2 col .
+uCM + 2uCM -cos(it-9)
2
the absence of friction between ring and wedge
surface, angular velocity of ring remains· constant.
From conservation of energy, we get
els col - - - - - -
- =--./2+2cos(it-0)
.!_ mv 2 + .!.I' co 2 = .!. mv' 2 +.!.Ico 2 + .!_ mv' 2 +mgh
dt 2 .
2 2 2 2 2
els l Where v' is final common velocity;
- = -.J2(1- cos0) v2
dt 2 •. v'=~- h=-
,. 2r 2' 4g
Jels = I -J2(1- COS0)]/ d8· 2
i - - . ...-- -·---· - - - - - - - - - - - - · - -· - - - ~ - " \
: · . . AN8WER8 ·-~
~-----

Q 0 ~ ·--- - ·- -·- -- -- -· . --
3, !I Cb)1.l 4. ICb)\I 5, i(b)·J 6,. LCbJ"
2 2r· I. I
s =~ Jsin0d0· .. 1. j (c) 1 2, i (a) i
. 2 ~
=~
~,r::.: cos01Jo2r
.
~ 41 ..
l
7. : (c) , 8. I Cc}:
'
~·:

'
9. !(b) i 10. \ (b) i 11. \ (d) 12. l(ii) i
-·..
33. On a train moving with accel~ation 10 m/s 2, a ·ball
13.: cci j 14. ! (b): 15,: cd)l 16. !(d)i 17, Lea) I18. tc~J-
starts rolling on· floor of train along the width of the
i
.• ..I, 24. (4)
19. :.(b}j 20. i Ca); 21.; C4l] 22. I Cb} 23,;l (c)
. I • I '; ' , ·
I
train with angular acceleration 2 rad/s 2 , radius of ball
is 1 m. Find the acceleration of the top point ·of ball at 25.; (c) 126. i (c) : 27. ! (cl! 28. I(d) 29. ICtl) j 30, j O>J
the time t = 3 s as seen from _8EOUnd.
(a) 10 m/s 2 (b) 10v'14 m/s 2
31. '. cdi ; 32.: cc) i33. !Cb) I34. \ Cb)-! I l
.I
r··' -
(c) 100 m/s 2 (d) .j1396 m/s 2
www.puucho.com
Anurag Mishra Mechanics 1 with www.puucho.com

I RIGID BODY MOTIIIN


2. A wheel of radius R rolls without slipping and its
centre O has an acceleration a 0 • A point A on the
,-- ----- -- --·- ------------, wheel, is at a distance r from 0. For given values of a0 ;
1. In the given figure F = 10 N, R = 1 m, mass of the I

body is 2 kg and moment of inertia of the body about R and r, determine ·the angle 8 and the velocity iJ O ofi
an axis passing through O and perpendicu\ar to plane the wheel 'for which A has no acceleration in· thisl
ofbodyis 4_kg,1112 • Ojs t)t~ ~entr~ 9f_!P.ass of the body. position: I
' F--,...,........ : '
• ,.-1.._---'<--F I
l

L_-~.L ,
'

--!~;_5·~--·----~..,-- ---------- __j


(A) If the ground is smooth, what is total kinetic
''

(A) Angle 8 is given by:


Fig. 5.55

energy of the body after 2 seconds? r -e


(a) cos8 = - (b) SIU =r
-
(a) 100 J (b) 75 J R R
(c) 50 J (d) 25 J (c) sine= r (d) cose=-,==r==
(B) If ground is sufficiently rough to· ensure rolling,
what is kinetic energy of the body now in the given
.JR 2 + r 2 2
- r
2
.JR
(B) Velocity v 0 is given by:
time interval?
(a) 18.75 J (b) 25.67 J (a) Vo= ~a;R(R' -r')l/4
(c) 16.67 J (d) None of these·
Solution : (A) When frictional force is absent, rigid (b) Vo = ~a;R CR'+ r')l/4
body-does not translate,
--+ • --+
I F external = 0, Therefore aCM = 0. (c) Vo= {a;;if 1
Taking torque about CM 1J---;:- (R 2 + r 2 ) 114 .
Fx2R-FxR =lcMCY. (d)v=~a 0R .1
10xl=4a. o r (R' - r2)1/4
a.= ~rad/ sec 2 Solution: As resultant acceleration is zero; resolving
2 all the three components of acceleration and applying
co= a.t = 5rad/ sec Ia,= 0, we get
1 2 1
KE, 0 ta1 =-lcMCO =-X4X5X5=50J CM FRAME
2 2
(B) Taking torque about IC j
........
F x 4R-Fx 3R = UcM +M(2R) ]a. 2 I ... ···---... t-axis ..... ·-....
\. . >'
... ·-. ao .•
10 X1 = [4+ 2 X (2X 1) ]a. 2
·I ( ~--\···'
~ = lO rad/ sec 2
• I
·'2..c:::: oo2r ~ = acM
.. ·------ :)w'r
12 ··------.--·
I'
-~•. '

10 5 rcx = a1 ··n -axis


co =a.t = -x 2= -rad/ sec
12 3
KE,0131
1
= -[lcM + M(2R) ]co
2 2 Fig. 5.56
2
ao sine= r:>..
= ½[4+ 2x (2x 1) 2 ] x (%)' a 0 sine = r ( ~)
=16.67 J
sine=..':..
R
Ia.= a 0 cose-co 2 r = 0 · .... (1)

www.puucho.com
Anurag Mishra Mechanics 1 with www.puucho.com

I ss4 MECHANICS-I ]
ro 2 r 4. A circular hoop of mass m and radius R rests flat on a
cos0=-
ao horizontal smooth surface. A bullet of same mass m.
and moving with veTocity v strikes . the hdop
h=(~)'x:o tangentially and gets embedded in it. Negle9t the
thickneslQf hggp inGcomparis::s:~:yius of h~op.
On solving, forv 0 we get, v 0 =~¥(R 2
-r ;v
2 4

'• R •I
3. A disc of radius 20 cm is rolling with slipping on a flacl
horizontal surface. At a certain instant the velocity ofr
its centre is 4 m/s and its angular velocity is 10 rad/s:
The lowest contact point is 0.
r--1() rad/sec
L_ _ __c-~ ---;,~~~-----1__
(A) Mark the correct option
I~ (a) Energy of system (hoop + bullet) is

li2:4m/s
I p · conserved
I
(b) Momentum of system is conserved

,_______L. -F~~~~-7-·-"-
(c) Angular momentum of system is conserved
about any point in the plane of hoop
'(d) Angular momentum is not consen<ed due to
(A) Velocity of point' 0 is: torque of gravitational force
(a) 2 m/s (b) 4 m/s (B) Velocity of bullet in ground frame, after collision
is:
(c) 1 m/s (d) 3 m/s
(a) 2vo (b) Vo
(B) Instantaneous centre of rotation of disc is located 3 3
at: (c) ~ (d) ~
(a) 0.2 m below O (b) 0.2 m above 0 2 4
(C) Velocity of CM of hoop (ground frame) is:
(c) 0.6 m above O (d) 0.4 m below 0
(a) ~ (b) 2vo
(C) Velocity of point Pis: 3 3
(a) ./16 m/s (b) 2-Js m/s (c) Vo (d) Vo
2 4
(c) -Js m/s (d) 5 m/s
(D) Angular velocity of system after collision:
Solution: As het velocity of contact point is not zero, (a) ~ (b) 3vo
disc rolls with slipping, as ro about IC is same, 2R R
(c) ~

I__, _o_
lvro,1 = 2m/s
__,
lv1ransl = 4m/s i1
v=4m/sl
I 3R

(a) -2;vo i
(d) 3iio
R
(E) What is the impulse on bullet?
(b) -~Vo i
IC (d) mvo i
Fig. 5.58 ....._J · 2"
Solution: Let velocity of CM of hoop be V1 and
Vo V angular velocity of hoop be ro, resultant velocity of
CO=-=__£_
IO IC bullet just after collision is V1 + Rro.
2 4 From conservation of momentum, we get
- = - - or x = 0.2 m
x x+R mv 0 =mv,_ + m(V1 + roR) ... (1)
Velocity of P is given by
VP =(IP)ro

=2-Js m/s Fig. 5.59

www.puucho.com
Anurag Mishra Mechanics 1 with www.puucho.com

~G_ID_BO_DY_M_OTj_ON_ _ _ _ _ _ _ _ _ _ _ _ _ _ _~-----'-----'------5_5-'-'5j
-----·-·----·-7
Lx@z::
; - - - - - ----·--y 2 2
MvR=Mv 0 R+-MR oi 0 ... (1)

O
5
I
I
j m V
--···
Before collisibn
.
After collision
•1

(v 1 + Roo) !
I
i
L_____
~ E)
Fig. 5.63_ ·-·- ·-
!
--
Fig. 5.61 Rolling constraint v 0 = Roi 0 ••• (2)

From conservation of angular momentum about On solving eqn. (1) and (2), we get
5
0, which is a point on ground just below the point v 0 =-v
of impact 7
L; = 0 Note that v O is speed in truck frame, in ground frame
2
. . 5 2 . . .
L1 =-mV1R+mR oi velocity 1s =v- -v = -v
7 7
L; =Li r-- - ···-· - •••-----. ·-- • --· -·- -- --- • - ..... -·•·----·,

. V1 = Roi ... (2) i 6. A cylinder and a ring of same mass Mand radius Rare;
from eqns (1) and (2), we get placed on the top of a rough .inclined plane of]
oi=-
Vo
3R
and

Absolute velocity of bullet = V1 + Roi = v 0


V1 = -
3
2
Vo
l
··-- ~~li~:~~:lg!~~ :are released simultaneously from

(A) Choose the correct statement (s) related to the


3 motion of each body:
Absolute velocity of hoop = V1 = Vo (a) The friction force acting on each body
3 opposes the motion of its centre of mass
Impulse on bullet
--+ 2mv 0 ~ -: mv 0 ~ (b) The friction provides the necessary torque to
= 4p=--1-mv 01= ---1 rotate the body about its centre of
3 3 mass
' 5. (c) Without friction none of the two bodies can
roll
(d) The friction force ensures that the point of
contact must remain stationary
(B) Identify the correct statement (s):
(a) The friction force acting on the cylinder may
be more than that acting on the ring
(b) The friction force acting on t)ie ring may be
more than that acting on the cylinder
(A) What is the final speed of sphere's centre of mass (c) If the friction is sufficient to roll the cylinder
in ground frame when eventually pure rolling sets then the ring will also roll
in? (d) If the friction is sufficient to roll the ring then
5 2 the cylinder will also roll
(a) v 0 (b) v0
7 7 (C) When these bodies roll down to the foot of the
(c)
7v (d)
7v inclined plane, then:
0
50
2 (a) the mechanical energy of each body is
(B) If a sphere with twice the radius and four times the conserved
mass had been used; what would have been its
(b) the velocity of centre of mass of the cylinder is
fi11al speed?
5
(a) -v 0
7
2
(b) -v 0
7 ~
7v 7 (c) the velocity of centre of mass of the ring is
(c) (d) v0 .[gii .
5 0
2
Solution: In reference frame of truck angular (d) the velocity of centre of mass of each body is
momentum is conserved about P -.J2gh

www.puucho.com
Anurag Mishra Mechanics 1 with www.puucho.com

! 556 MECHANICS-I ;j

. 7. A plank of length 20 m and mass 1 kg is kept on a\ V contact = Ca1 + Ra)t


= 15 t ... (1)
· horizontal smooth surface. A cylmder of mass 1 kg.isl For plank v contact = 20 - .5 t ... (2)
~ept pear one end of t:J:ie plank. The coefficien! ofi From eqns. (1) and (2), we get
friction ,between the two· surfaces is 0.5. Plank is t = 1 sec
suddenly,gLVJ!J,.i\ y~lQ.dt}'._.,!Oim/s towards:left. ··
·, 0
!
/
~ m I•
1I
r ,
Till the cylinder slips on plank
1 . 2
Srel =Ure1t+-are1t
' I• •I I 2
I 1=2om i
'I Fig. 5.64
------·-·· .,,_____________
I
. ----------
1
= 20xl--x lOx 1
2
2

(A) Mark correct option (s) = 15m


(a) Initial acceleration of cylinder is 5 m/s 2 Velocicy of plank when pure rolling begins
towards left v=20-St·=l5m/s
(b) Initial acceleration of cylinder is 5 m/s 2 velocicy of cylinder = a1t = 5 m/s
towards right When pure rolling begins, friction force vanishes,
velocicy of plank and cylinder is constant. After
(c) Initial acceleration of plank is 5 mls 2 towards
pure rolling begins
right
5 5 1
(d) Initial acceleration of plank is 5 mls 2 towards t=--=-=-sec
Vrel 10 2
left
(B) Mark correct option (s)
·
---------._-------. ----·· ........ ---- _- . --·_-.·~
(a) Pure rolling of cylinder takes place
f 8. A uniform rod of mass. m is supported on two rollers
I ·each of ·nJass m and radius r and rolls do~ 'the
immediately i 2 . . " "
(b) Initially cylinder slips and then pure rolling , ,inclined · rough plane as · shown in the ,figure!
begins I. · . Assu!11ing rol~PP.irig :at any contact and treating the
·· rollers as urufo~. ~Qli~!_cyhn,~ers:
(c) Pure rolling never begins
(d) Time of slipping is 1 sec
(C) Velocicy ~f plank when pure rolling begins is:
(a) 10 mis
(c) 20 m/s
(b) 15 mis
(d) 25 m/s
(D) Time in which plank and cylinder separate: L
. ,:
' '

'
;
i~R-od:
)
I
'
.:
~----~---- ---- -~~~-!~.-- i -
0'
i

(a) 1 sec (b) 1.5 sec


(c) 2.5 sec (d) 2 sec (A) If acceleration. of rod down the plane is aR and
Solution: acceleration of centre of mass of roller is a 0 then:
MR 2 (a) aR > a0
fR = · --a;f = 5 N; M =1kg (b) aR < ao
2
(c) aR = a 0,

t::::J
(d) information is insufficient to decide
(B) If the friction force on roller at the contact point
with incline is f1 and at the contact point with rod
is f 2 , then:
Ra= 10m/s 2 (a) f1 > f2
f · 2 (b) f1 < f2
a1 = - = Sm/s
M (c) f1 = f2
2
a2 = 5 mls (d) data is insufficient to decide
(C) If the incline is smooth and rod still does not slip
For cylinder
on roller, then acceleration of rollers is:
acontact :::; a1 + Ra
(a) 2g sine (b) l2g. sine
11
(c) g sine (d) none of these

www.puucho.com
.,,
Anurag Mishra Mechanics 1 with www.puucho.com

....... -- . - .....••. l
LRIGID_~_ODY 1'0T!ON . ··-· ---- - .....•.
I
- ··-- --- ·--- -·· - - _-- _557J
9. A uniform beam of length L and mass m is supported, Solution.
as shown. If the cable suddenly breaks,
(1/4)L

rijr,,i ,?'LH
Cable

... 31 I

Al...:'.!'.:
-----L---<>i
Fig. 5.67 Rx .-ed~R_'....,____-=-3-1/-4 w2
(A) 1 he acceleration of end B is: l
mg
131/4 a._
(a) Sg (b) 9g
7 7 Fig. 5.68 (b)
(c) 3g · (d) 4g From COE, we get
7 7
(B) The reaction at the hinge is: mgl = ]:.
4 2
m[f12 + f16]ro ' ro 2• 2 = 24g
71
(a) 4mg (b) 9mg
7 7 From Newton's law
(c) 5mg
. 7
(d) 3mg
7
mgl = (m1 + m1 )a
2 2
... (1)
4 12 16
10. Uniform rod AB hinged .about a fixed point P is'
initially vertical. Rod is released from vertical mg-Ry= m (¾a) ... (2)
position. When rod is in horizontal position: '
R = ml ro2 ... (3)
1B y X 4
ro 1,
2 I,
L Lx c, =
12g
71;
...,
acM = -41 - 4c, J
l:• U4
6g, 3g,
=--1--J
7 7
Fig. 5.68 (a) ~~ =-ro 2(:)i-(:)aj,
(.A) The acceleration of centre of mass of rod is: ..., 6mg,..., 4mg,
6g , 3g ,
(a) - - 1 - - J
(b) 12g, 6g ,
--1--J
Rx= - --i;Ry =- -J
7 7
7 7 7 7
3g, 9g, (d) 9g, 3g, l 11. The, axis of the uniform cylinder in figure is fixed. The I
(C) - - 1 - - J --1--J
7 7 7 7 cylinder is initially at rest. The block of mass Mis•
(B) The acceleration of end B of rod is: initially moving to the right without friction and with'
6g , 12g , (b) Sg , 9g , speed v 1 • It passes over the cylinder to the dashed:
(a) - - 1 - - J --1--J position. When it first makes contact with the'.
7 7 7 7
cylinder, it slips on the cylinder, but the friction is:
18g , Sg , (d) 18g , 9g ,
(C) - - 1 - - J --1--J large enough so that slipping ceases before M loses.
7 7 7 7 contact with the cylinder. The cylinder has a radius Ri
(C) Reaction on hinge is: and a rotational inertia I: !
8mg, 12mg, -> ->
(a) - - - 1 - - - J
7 7
3mg , 9mg ,
Cb) - - - 1 - - - J - ~
7 7
6mg , 4mg ,
(C) ---1+--J Fig. 5.69
7 7
d) 12mg , 6mg , (A) For the entire process the quantity (ies) which will
C --7-l--7-J remain conserved for the (cylinder+block) system
is/are (angular momentum is considered about
the cylinder axis):

www.puucho.com
Anurag Mishra Mechanics 1 with www.puucho.com
----
I 558 MECHANICS-I I
(a) mechanical energy, momentum and angular 13. A circular disc of mass 300 gm and radius 20 cm can
momentum rotate freely about a vertical axis passing through its
(b) mechanical energy and angular momentum centre 0. A small insect of mass 100 gm is initially at a
only point.l! on the close (which is initially stationary). The
(cl momentum and angular momentum only in~ect :stai:ts walldng from rest along the rim of disc
(d) angular momentum only with such a time varying.relative velocity that.the c\isc
(B) If co is the final angular velocity of the cylinder, · rotates in the opposite 'direction with a constantj .
then: angular acceleration = 2inad/s 2 • After some time T;
(a) v 1 = roR (b) v 2 = roR ..__.,,th~insect is back_a_J: th~_p_qiht A.· ____ J
(cl v 1 < roR (d) none of these (A) What is the time taken by insect to reach its
(C) Question given below consists of two statements original position?
each printed as Assertion (A) and Reason (R). 2
While answering the question you are required to (a) T = ..fs
choose any one of the following four responses on
the basis of the above situation: (c) T = -
.Js
(a) If both (A) and (R) are true and (R) is the 2
correct explanation of (Al (B) By what angle has the disc rotated till how as seen
· (b) If-both (A) and (R) are true but (R) is not
by a stationary earth observer:
1t 1t .
correct explanation of (A) (a) - rad (b) - rad
5 3
(c) If (A) is true but (R) is false
4it 4it
(d) If (A) is false but (R) is true (c) -rad (d) -rad
5 3
Assertion: Momentum of the block-cylinder Solution: Let the mass of insect is M and radius
system is conserved.
of discR.
Reason: Force of friction between block. and
cylinder is internal force of block-cylinder system. . =-
M.I. ofd 1sc 3MR2 I of"msect ..= ·MR2
- . , M..
2
12. A .uniform rod is fixed- to a rotating turntable. so that
From conservation of angular momentum,
. its lower end is Ori the axis Of the turntable and it
· makes an angle of 20° to the vertical. (The rod is thus we get
rotating with uniform angular vel9city about a ->
10 roo1G+1 1 COJ/G
->
=0
vertical axis passing through one end). If the
turntable is roJ:_;lting clockwise as seen from above: On differentiating w.r. t. time, we get
I --+ -+ .--+
In an1a = -1 1 fJ.I1a;a.11a
·1,-, 3->
= --ao/G = --ao/G
•.
10 2
--+ --+ --+ 5 --+ --+ 2
a110 = a11G-ao1G = --ao1G;la11ol= Sitrad/s
2
· 1 2
Fig. 5.70 B11p = -d11ot
2
(A) What is the· direction of the rod's angular 2it = .! (Sit)t 2 or t = ~ sec
momentum vector (calculated about its lower 2 v5
end)? 1 2
(a) Vertically downwards 00/G = -aO/Gt
. .2
(b) Down at 20° to the horizontal
or 0 016 = -4it rad
(c) Up at 20° to the horizontal
(d) Vertically upwards
(B) Is there a torque acting on it, and if so in what
I 5
1;[ A-~p_o_o_l_o_f_m_a_s_s_m_,-with. moment of hl~rti~ ~bou~~i
direction?
(a) Yes, vertically I -~ 2
~sis placed. on a rough horizontal surface. An!
(b) Yes, horizontally
(c) Yes at 20° to the horizontal
(d) No
L =~~in~:~~;::plied through_ a thread_~

www.puucho.com
,_
Anurag Mishra Mechanics 1 with www.puucho.com

RIGID BODYMOTION :559!,


Solution: M.I. of rod about pivot
I= (2m)L2 + 2m(!:._ )2 = ~mL2
12 · 4 24
From conservation of angular momentum, we get"
mu!:.= mv!:.+
4 4 24
(2-mL r 2 \., . • •• (1)

Fig. 5.71
And from expression of coefficient of restitution, we get
L
oo--v
(A) The acceleration of CM of spool is: e=l=-4~- ... (2)
3F 3F u-0
(a) a=--(R+3r) (b) a=-(R+2r)
2mR 4mR Eqns. (1) and (2) on solving yield
4F (R+2r) 114u
(c) a=~(R+3r) (d) a= v=--
3mR 3mR 17
(B) Force of friction acting on the spool is: 24u
OJ=--
17L
(a) %(1- ~) (b) i(1-;) Now we apply impulse momentum equation on particle
and again on rod for particle. · ·
(c) .~(1- !) (d) F(1-:) Impulse, J
_, _,
= ti p; change in momentum
li 1s. A thin uniform rod of mass 2m and length L. is _pivoted
.about H; as shown in the figure. Rod is free to rot.ate
-Jp =-m(~~)-(mu)

i in vertical plane. A particle• of mass m strikes Jp = 28mu


elastically at centre of mass' of rod. The particle's line 17
of motion is horizontal:
r··--·-· . ·--···-·:
ii t,Jhlago
.
= JH !
L
.
!I .. . Jparft:I~-= J~
.

'
. . 5.73
Fig.

J_ Fig.5.7·~==""'---------'
(A) What is _speed of particle just after collision?
For rod
·-····-·'" -
· Jp -JH = (2m)vcM = (2m)OJ(~)

-Su -llu or
(a) V = - (b) V = - -
17 17
-7u -9u 16. One end of an ideal spring of unstretched length)
(c) v = - (d) v = -
17 17 10 = lM, is fixed on a frictionless horizontal table. 'The
(B) What is angular velocity of the rod just after the other.end has a small disc ofinass 0.1 kg attached to
collision? it. 'fhe disc.is projected witha velocity Vo =.11 m/S
13u 15u L _ _ pemendicular to the. spr(fig~_· ___ - - · - ~
(a) OJ=- (b) OJ=-
17L 17L (A) Choose the correct statement:·
24u 12u (a) Linear momentum of disc is conserved as the
(c) OJ=-.- (d) OJ=-
17L 17L spring force is always perpendicular to
(C) What is the impulse imparted by the hinge during velocity of dik ·.
the collision? (b) A]1gular momentum of disc about fixed end of
spring is conserved.
(a) J= Bmu (b) J= 9mu
17 17 (c) Kinetic energy of disc is conserved
(c) J= 4mu (d) J= l6mu (d) Angular velocity of disc remains constant.
17 17 (B) In the subsequent motion of disc, maximum
elongation of spring is ~- The velocity of disc at
10
this instant is:
www.puucho.com
Anurag Mishra Mechanics 1 with www.puucho.com

(a) 11 m/S (b) 10 m/S 2 2


mR
--(J) = --+ mR 2)(J),
(mR
(c) 5 m/s (d) 7 m/s 2 .
2 . ·
(C) What is the force constant of spring? , (J)
(a) 210 N/m (b) 100 Nim (J) =-
3
'(c) 110 N/m (d) 200 m/s
. Solution: "Angular momentum oMisc about fixed
end is conserved, as the spring force passes
·0;··.,,
. :d·. .
.
R~---
. . •.
.
' ~
throughrO_.---::::;;;:::---~::;--
v, ·.· :)··· .· ·····2~~ • ' "

;/·. ' , ''"l


':·,_"_:1:;,.. -,.....), _. mR<ii
..1!.!2 ·._ -
·_ . _,:_- .~
t IP,1--r."""'3
10
~~-' -· .
J~;J;S•tniRro
l /W,"
lo

Fig, 5,7~. ..., mRro


IP1l=-
·
mv 0 l 0 = mv 1 (111
10
0 )
....
3

· · · 10v
'IP1 1= ~mRro
0
v 1 =lOm/s=-- Impulse on particle = .change in linear momentum
.. 11
From conservation of energy, we get
2
?'-x:t, ·~lL:d"'"'Jel
. . . r ·:·<'i , . _, -~-l

2
1 mv 2
0 = 1 mv 12 + 1 v 2·(· lo )
2
-
2 10
~- : , I:
•ViJ~.,;H ,!: ,·) .·..·.._.-._J~~-.J
,- .. p ,' ;·:;:' . "'
= 210N/ni
Vz
. . Fig. _5,76. ·c . . ·.
it." A. uhlfci~discofmass Ill ;dr_a_d-iu_s_R_ro_t_a~~'abou;-~
0

~- ·-~~"'"'"~·~--·----~
-+ -+-+-+
.. ,· · -fixed,.~~rllcal axis passing_· through its c"entre Wifu J=ti.p=pi-Pi
:, ·angular v~ldcity ro. A particle of. same 111ass )Tfl ~na
, . moying'liorizontally wit:q.~locity 2roR towards c~~tre
......:_gf.the c\.1$.£tolliaeswith.t:J-i'~i4ijc:;and sticks to it§jj_iµ: ~
(A) The angular velocity of disc after particle sticks to
iti.;I=
Impulse on hinge is negative of intpulse ·on disc.
~('ni; 00
r +C~ro) = p;mRro
2

disc is: ·
(a) -
(J) •
(b)
(J)
18. T_w _ e_E~l-.l
:_()F_i~cle~_ofm_~~s. ~-;g_-._~hare.rotalli!-gi>_.~_-_'a(_,
.• 5 3
, xz,plllfle m· circle ofrad1us;R as shown. G011§1der,;the
(J) .
(d)
. . . situati~n wheri die p:irtk\e~ ru:e at (R, R; 0) ·~a,;C~-ll-j
(c) - (J)
-R, 0) answer following gµest1ons i;, .:,
2 4 ' ... \', ,,, I I- : ; - - ·--- . .. ... . i
~ Jf~\J
--
(B) The impulse on the particle due to disc is:
-10 ;,,~ 'Cb)
l.
(a) .fjs mroR ,. R
3 3 RV , fa.

(c) m_mroR •,(d) ./29 mroR '


:, ,.J.x .
. . .
. --\~"".
3 3
z £'.....--11!--,- ~ :,, . '
'i -~µ!.:::>' "/<j
(C) The impulse on.the disc due to hinge is:
t'
F l •, 1 jj
(a) ffi mroR

(b) .fjs mroR
3. L____~ -·-····' -~~L-__i::~:~:-·-· _--~-- ~ ~.-~~j
(A) The angular velocity vector of the system of two
(~) ../37 mroR (d) ./29 mroR particles ,and the total angular momentum of
, ,3 . 3 system about origin make an angle of:
Solution: Apply conservation of angular
(a) 45° (b) 90°
momentum
(c) 0° (d) 180°

www.puucho.com

·- ---
Anurag Mishra Mechanics 1 with www.puucho.com

L_RIGID BODY MOTIOl:I :__.~,===::.:·:::··r···-- ::;:'J

-7
;~'!·~"~·~'~·;::·'·':::-;;:;·"~<:;,j~l~?j~Jl,~1!;~,•;,::;>;;:':::::::;::::;:·'li==!i~fj
-----
(B) What is y-component of the total angular ·
,l.f ..·1
i

L
momentumofthesystemaboutorigin? ,· · m . .
(a) zero ,(b) 2mvR j
1
h ; m ./ i
(c) mvR (d) mvR
~
1 1'
,'}(~
,·~,

(C) Choose the correct statement , , :.; .·... -,____...;;;...._-_...;.._...L_,:__-'"~·~~J.


(a) They-component of the total torque acting on _;_,,__,_ 1 Fig. 5.79 1
the system is zero · (AlVelocify of centre of mass of sphere w.r.t. ground
._ , , - - I • •

(b) Net force acting on system is non-zero ' is: ·


. (c) . Net torque on the system rotates parallel to , (a) ~
xz-plane 1/7.IS"
(d) Magnitude of the total torque acting on the (c) ~i gh (d) ~171 gh.
system · · 7
Fig_:_s_h-ow-~-;d-u-~bbcll ~-ha_t_c_o-.il.-sis_ts_of _a_m_a_s_sl-es-;-.r-6d~ · (B) Velo~ty of wedge:
19.
and two particle size.spheresiin.figure (a) impulse is - · (a) ~gh (b) ~2gh
imparted perpendicular to the rod and in figure (b) · 7 · ·.
11
impulse js imparted parallel ,to the rod. Answer . (c) /3:: (d) ~ 7 gh
I
I
followiQg_q1,1~stiorts. . ---·~-,--- V7 IS"
1 (C) Angular velocity of sphere:
t I
I M T M f ' (a) ro' ='J-12g_h
7R2
(b) ro =J-27_g_h
7R2

i I I 11 (c) ro = J20gh2 . (d) oi = J44gh
I • M . 7R 7R 2
•~M
! llr/
1 t,p-
i
-
AN9WER9 ., ~.•:J
7··
I I '

I
'
'
(a) (b)
Fig. 5.78 -1. A-(c),.]r-(c) 11. 'A~(d), B,-(b), c:.,(d)'
~ - - ~ --~=---== ,_.=-,.,.. "'=""-,,~-'------'--'
(A) Velocity of CM and angular velocity of system in

.. < • ' ', '

- . ,,,

2. !A:-'(li), .B'-(a) 12: A-'Cb}, B-Ccl '


figure (a) are respectively: .. '
..

A-, A '
3. A--(a),:13-'-(a); C-(b) 13. A-(a), B"7(C)'
(a) ...E-ro = _£_ .. '
M' 2Ml ', ',
_,
Ap Ap
(d) - · r o = -
4. A:..:(b,-0),B-(a), C-(a), 14. A-'-(b), B-(a)
., ,.• ' ' "
'• :
.
D-(c), E-:(b)
2M' 2Ml
'
' .,. '. . , .

s. A-(c); B,-(d} 15. A-Cb), il-(c); c-ca) ·


",·-. l
(B) Energies imparted to dumbbell in figure (a) and ·,· ..
figure (b) are respectively: "
6. A"'(a,b; c), ~(b,c), C · 16. A-(b), ~Cc), c'-ta).
(
a) (Ap)2. (Ap)2
2M'4M
(b): .(Ap)2. (Ap)2
4M'2M
..-ca:, b; c) .. '
'.':
..
(Ap) 2 (Ap) 2 (Ap)2 (Ap) 2 cl), 17. :A:,.(b), :B-(c), C.-'(c)
(c) 2M ; 2M (d) "w;--:W- 7. ' ... , "·B-{b,
A.'.:(a}
. ~ •' "' ..
C-(b~, D-!(b)
•' .. , . . ' ..•···
120-
. .-A-h_o_JJ_ow spher~ ~-rele~;ed froin
the top of a wedr;c::/ . ,i"t

friction is sufficient for pure rolling of sphere ,on the! 8. A-(a),1,1-(a), C:.,(c)
. ' ,, ' 18. A-CbJ
. ' .B,-Cc)
. ' ·c-·cc)
.. ,.
.
' .
''
' '

wedge. There is no friction:b'etween. the wedge, 3:[\d,. ",, ".;··· ! " " . '
the ground. At the instant it leaves the wedgel 9. A-(bJ; B-(a}' 19. A-(b), B-(b)
_J . ,,-·:·;; -.·
horizo1it,!J1y,.._____ _ _ _ _ '. , .. ' . ' ' '

10. A--(~),·B
-, .
.... (d), .C-(c)· . ~
. 20. A-(a), B-(c), c:., (b)
:

www.puucho.com
Anurag Mishra Mechanics 1 with www.puucho.com
,_

MECHAtj~

6.' A box sliding on a horizontal. surface is brought to rest


·by friction. - ·
(a) Statement-I: The pamage to the bottom surface is
·Each· of the questi~n contaµis statement-I
greater near·the"leading edge than toward the trailing
:CAssertionf and statement-2 (Reason). edge. ·
(AJ ·If lioth the statenieii.ts are-true and statement-2 is the
·correi{ explJ~a,tlon of statement~l..: : - . . _ Statement-2': Due to surface roughness box has_
'(B)_ If ocith the statements are true but statement:2 is not tendency'to topple about front edge.
I , ,
the-correct ~,@~tion df statement-1. . · (b) Statement-I: The box's angular momentum remains
- (C) If statement' Oslhue· and statement-2 is false.· zero about any reference point located on a line
· (D) _If ~tatemel!?i.iiil!:'f$e_ and statement-2 is true: paraI]el to the velocity through the rate's CM. .
· 1. Stat~me!1t-ii ,\,vhen a cyclist is_g~ini at a speed v on Stateinerit-2: The box's weight and normal reaction
a circular horizontal road' of radius . r .which is not exerts .no torque about CM.
1
0 7. Statement-I: If tJu:ee forces act on object in
baf~ilf~~b~nds a;an a_ngle t~- (;;) mth the equilibrium, their lines_ of action must be parallel or
must intersect at a point.
vertical ,. . ' ; ' '. .
Statement-2:, If lh;1es of action· forces. do not
· Stateient"i: Total eriemai force· an:d total ~xternal -·intersect, torque cannot be ·,ero.
' tori:J.ue""rii.ust be zero. --+ --+ . ~

. '-+ . l -+ .8. Two forces F1 ancl F2equal in magnitude and opposite


2, · Statement•l:"The equation, 't exiem,lJ = 'ta is valid in in dir~ction act _on an object as shown in figure.
a
nonAnertial reference frame -if- the axis ·of rotation ,--+ ·• ,--+ ' '
pas Se? .through .the' centre of m~s ~nd torque and . 1 2

moment .. o_f inertia· are evall!ated -aliout the axis , F =-F • . :.~x;j·--~·
t:Qrough centre of mass. -:-•••• ..
' 'I,,..,.\
·State'ment-2: Torque of pseudo force about.centre of ' • ••• .....x
mass·:is,·zerb. ·· . ,. -> -,-
<. F, • - •
,I
3. Statement-I: . A sphe"ie -, . in pure rolling on a
horizontal surface· slides. dOW(l and eventually Stops. . f'ig. 5.81

State~ent-2: D~e t~ kinetic friction force kinetic Statement-I: The torque exerted by ·a couple is
eriergy of sphere ~ reduced to zero ... dependent on the origin about wliich it is computed.
:4, · Statement-I: A pure rolling rigid body-(on a rough Statement-2:- If origin shifts, moment arm for.
-+ -+ .
horizob.tal ~urface), µnder action of an external force,
individual forces F1 and F2 chan)les.
' acting..: at CM ·enters smooth surface. Its angular
' velocity. will' remain constant but linear speed will
9. · Consider the two mutually interacting particles shown
-+
.mcyea~e.. ·_.. : ..· '._. . . . . . ! •
in figure. _Let F1,2 be-the force exerted by particle 1 on
. . '" --+ ' .
S_tat~ment~2:· Angular ;icceleration of 'the body particle 2,- and F2,1, be that exerted by particle 2 on
about its centre of mass becomes zero but linear particl!} 1, in accordance with Newton's third law.
. acceleration suddenly increases... ·-.. .

5. A particle is travelling· with .


. -+
_ a, i:cinstajit velocity. V along
·a line
• - L
that is
' ,, . • , -
·a distance
.
b
dA_
from the origin 0. Let --
-- "dt
·rep;esents a real velocjty, _Fig. 5.80" ... ; _
J,Vhere dt is area swept by ' '
.!!position vect9r from Oto the particle ·in t:inJe dt . . • ·
._ !_~--- --~-_-_ ·_9
~ 5.82- - -

(a) ' Statement-I : Area· swept in first second is less than (a) Stateineni:-I: Angular momentum of two particle·
' ·area· swept in next second. '' .. ' . . system abouLO a fixed point in space is conserved. ,
s'tatement~2: Angular momentum of particle ·is ' .·--+ ~

. ' . -- . . . ,\ Statement-2: Resultant of F1,2 and F2,1 _is zeta_.


• ~ I • mcrea.smg. ,·. , · ·.• -:.
(b) Statement-I: Sum of torques exerted by these forces
-(b) Statement~.!: Arial felocityof p,micle is constant:" about O cannot be zero. · ' · ·· ·
. stiitement-21
.
Atigular'
.
.
moqj.entum
... of particle
.. -. . is
· constant. :: .

www.puucho.com
-----
Anurag Mishra Mechanics 1 with www.puucho.com

! RIGID BODY MOTION 563 I


.., .., 12. Consider a rigid body of ci;~ular cross-section in pure
Statement-2: Forces of F1,2 and F 41 have different
rolling motion on a fixed surface ..
-moment arm about 0.
Statement-1: Angular velocity of the body about the
(c) Statenient-1: Sum of t~rques about O is zero.
I .., .., .., instantaneous axis is the same as its_ angular velocity
Statement-2: Net torque is (r1-r2)xF:,,1; with about its centre of mass.
.., .., .., Statement-2: An observer at centre· of mass sees the
(r1- r2) parallel to Fzi.
contact point, move with same velocity as an observer
(d) Statement-1: For a system of only mutually at instantaneous centre will see the velocity of centre
-• interacting particles angular momentum remains_ of mass.
constant. · 13. Statement-1: Angular velocity of any point on a
State~ent-2: For all internal forces, ·the'. internal rotating rigid body about any other point on it is same.
torques cancel in pairs. , _ Statement-2: A rigid body is a body with in whicli
10. (a) A merry-go-round is rotating about a frictionless the distance between any two points does not change,
pivot. A cliild on the edge begins to walk inward. no matter how body moves.
Statement-1: The moment of inertia of the cliild '4\4- Consider a rigid body is projected up an inclined plane
merry-go-round system decreases, hence the total - (assume pure rolling).
kinetic energy of the system must increase. · StatemeO:t-1: At the top most point velocity of
Statement-2: The increase in kinetic energy of centre of mass is zero but angular velocity is non-zero.
system is due to work done by friction between shoes State~ent-2: bue to negative work done of friction
of cliild and the merry-go-round. force translation kinetic energy decreases, but due to
to move outward.
.
(b) When the cliild is at centre she is at rest and. she begin positive rotational work, rotational kinetic energy
increases.
Statement-1: Angular momentum of 15. Statement-1: For any rigid rod that rotates about a
merry-go-round decreases.
0 symmetry axis, the total angular momentum is parallel
Statement-2: The accelerating force on child is to the angular velocity.
friction, whicli exerts a torque on' her, increasing her Statement-2: The Vector sum of angular moments of
angular momentum. the individual particles is along axis of rotation.
-11. A puck on a frictionless plane is
given an initial speed v 0 • The
puck is attaclied to a string that
wraps around a fixed vertical I
post.
;~~il
, ro ·
L _ Fig:~.83 ~
1
I
'16. sta:teme~t-1: If there is no external torque on a body_
about its centre 6f ma_ss, then the velocity of the centre
of mass remains constant.
because
Statement-2: The linear momentum of an isolated
Statement-1: Meclianical energy of puck remains system remains constant.
constant but angular momentum must decrease'.
Statement-2: ·Torque of tension is in downward
direction.

www.puucho.com

You might also like